Critical Care: Board and Certification Review

127 54

English Pages [2019] Year 2022

Report DMCA / Copyright

DOWNLOAD FILE

Polecaj historie

Critical Care: Board and Certification Review

Table of contents :
Learn More
Introduction
Editors in Chief
Contributing Editors
Authors
Grammar Editors
Section 1 ( Questions 1 - 100 )
Section 2 ( Questions 101 - 200 )
Section 3 ( Questions 201 - 300 )
Section 4 ( Questions 301 - 400 )
Section 5 ( Questions 401 - 500 )
Section 6 ( Questions 501 - 600 )
Section 7 ( Questions 601 - 700 )
Section 8 ( Questions 701 - 800 )
Section 9 ( Questions 801 - 900 )
Section 10 ( Questions 901 - 1000 )
About StatPearls

Citation preview

Critical Ca re Board and Certification Review

1000 Questions and Explanations

2022

edition Alan Heffner , MD Susar Murin, BA, MD, MSc , MBA Christian Sandrock, MD, MPH

*

Introduction NOTE: The intent of StatPearls review books is to identify knowledge deficits and assist you in the learning process. Review books are not intended to be a source of the knowledge base of medicine. The authors or editors do not warrant the information is complete or accurate. The reader is encouraged to verify each answer in several references. All drug indications and dosages should be verified before administration. Copyright 2022 StatPearls Publishing, LLC. All rights reserved, including the right of reproduction, in whole or in part, in any form.

http://www.StatPearls.com Congratulations! This StatPearls book will help improve your fund of knowledge. Our books are designed to help you identify and resolve knowledge deficits. To achieve this goal the text is written in a multiple-choice format with answers and explanations. Explanations contain additional information intended to reinforce your knowledge. Emphasis has been placed on covering facts that are easily overlooked, quickly forgotten, and often show up in specialty reviews. For further information, all explanations are electronically linked to http://www.StatPearls.com . We suggest you note all questions answered incorrectly to allow you to rapidly review the content prior to your exam. We appreciate your comments, suggestions, and criticisms. Some answers and explanations may be in variance with your own knowledge. This is usually attributable to variability among sources; however, please make us aware of any potential errors you find as we update our books often. We appreciate your input in regard to format, content, or presentation. Please contact us at [email protected] or better yet, click on the flag button after each question to give us specific input on any errors or omissions. We look forward to hearing from you. Good luck with learning your specialty! StatPearls eBooks StatPearls books were created exclusively for use on eBook readers. Because we don't publish our books in paper format, we have the luxury of optimizing them for eBook use. How are our books optimized for the viewing on an eBook Reader?

1.

Efficient User Interface: Each multiple choice question is on a page by itself so you can see the question and four multiple choice answers, but you can't see the correct answer until you flip the page. This would be a complete waste of paper in an ordinary book, but it makes for an easy to use eBook. You don't have to cover up the correct answer with a piece of paper. Most devices have a dedicated button for page turning so with the click of a button, you can see the correct answer.

2.

Research Links: After each explanation we include web links to search for more information on the key concepts for that question. If you miss a question, this is when you probably want to do more research on the question concepts. This would be onerous and require a lot of typing using a paper book, but is a single click on an eBook reader, PC, or Mac.

3.

Flag a Question: Medical standards and drug standards change. Also, human errors can occur resulting in questions that have issues. StatPearls strongly believes in Continuous Quality Improvement (CQI). Flagging a question gives you, the

medical professional, the ability to tell us if something is wrong with a question and we will correct it and get back to you with an updated book (providing you give us a phone number or email address.) If you don't provide us with contact information, you can check for a book update using the normal methods from the books store and you will soon find an update with the question corrected by one of our health professionals.

4.

Volume of Questions: When we make a book we publish every question we have on a topic (up to 2,000 per book). This would be ridiculously expensive to do with traditional book publishing, but in the eBook world it costs no more to publish 2,000 questions than 1,000 questions. You will find that in most of our specialties, we dwarf the competition with breadth of material. This makes our books "just what the doctor ordered" for those wishing to go the extra mile when studying.

STATPEARLS AUTHORS AND EDITORS Over 6,000 physician, nursing, and allied health professionals authored a database of over 50,000 classified multiple-choice questions and explanations. Each question has been peer reviewed by two health professionals and a pharmacist. All questions are reviewed annually and updated through a peer review process. WE APPRECIATE YOUR COMMENTS! We appreciate comments, suggestions, corrections and additional contributing authors and editors. Please e-mail us at: [email protected].

Critical Care Specialty Review and SelfAssessment StatPearls Book ID: MD_CRITICAL_CARESPECIALTY_REVIEW_AND_SELF_ASSESSMENT-2021-12-20 Version: 12/20/2021

Editors in Chief Alan Heffner, MD; Director of Critical Care; United States, Charlotte, NC, USA Susan Murin, BA, MD, MSc, MBA; Professor; UC Davis School of Medicine; United States, Sacramento, CA, USA Christian Sandrock, MD, MPH; Professor; UC Davis School of Medicine; United States, Sacramento, CA, USA

Grammar and Illustration Editor Gerson Cordero Rubio

Table of Contents Please Contribute Learn More Introduction Editors in Chief Contributing Editors Authors Section 1 ( Questions 1 - 100 ) Section 2 ( Questions 101 - 200 ) Section 3 ( Questions 201 - 300 ) Section 4 ( Questions 301 - 400 ) Section 5 ( Questions 401 - 500 ) Section 6 ( Questions 501 - 600 ) Section 7 ( Questions 601 - 700 ) Section 8 ( Questions 701 - 800 ) Section 9 ( Questions 801 - 900 ) Section 10 ( Questions 901 - 1000 ) About StatPearls

Section 1 Question 1: A patient presents with fever, chills, and extreme pain 48 hours after cutting his hand. His hand is purplish in color with bullae, putrid discharge, and decreased sensation. In addition to starting empiric antibiotics, what is the most appropriate management of this patient?

Choices: 1. CT scan of the hand 2. Serial examinations to assess for progression 3. Surgical exploration 4. Incision and drainage to send specimen for culture and Gram stain

Photo:Contributed by DermNetNZ

Answer: 3 - Surgical exploration Explanations: This is a case of necrotizing fasciitis. It is a rapidly spreading infection along deep fascial planes with subcutaneous tissue necrosis. It typically is caused by Streptococcus pyogenes or Staphylococcus aureus, but anaerobes and Clostridium species may also be present. Urgent surgical exploration is indicated with aggressive debridement and should not be delayed for culture or radiographic results. A CT scan will not reveal anything new that can change surgical management. The patient must be aggressively resuscitated and undergo aggressive debridement. A second-look procedure may be required in 24 to 48 hours. Go to the next page if you knew the correct answer, or click the link image(s) below to further research the concepts in this question (if desired).

Research Concepts: Necrotizing Fasciitis

We update eBooks quarterly and Apps daily based on user feedback. Please tap flag to report any questions that need improvement.

Question 2: A 69-year-old heavy smoker presents to the emergency department with severe knife-like chest pain and diaphoresis that woke him up from his sleep 2 hours ago. He denies being short of breath but claims that the pain is severe, continuous, and radiates to his upper back and shoulders. His past medical history reveals longstanding poorly controlled hypertension, diabetes, and peripheral vascular disease. He is not compliant with his medications, smoked 1 pack of cigarettes daily for the past 30 years, and drinks alcohol regularly. By the time the physical exam is done, the patient’s chest pain has subsided. His blood pressure is 180/90 mm/Hg in his right arm and 120/63 mm/Hg in his left arm, pulse is 110 bpm, respiratory rate is 18 breaths/min, and temperature is 98.8 F. He is alert. His ECG shows T-wave inversions in leads V3 an V4. He undergoes a CT angiogram of the chest depicted in the image. What is the next best step in the management of this patient?

Choices: 1. Aspirin 2. Esmolol 3. Sodium nitroprusside 4. Observation for 24 hours

Photo:Contributed by chestatlas.com (H. Shulman MD)

Answer: 2 - Esmolol Explanations: This patient likely has an aortic dissection. Aortic dissection is a relatively uncommon illness that can be life-threatening. Early diagnosis and treatment remain the cornerstone of the management of aortic dissection and are crucial for the survival of the patient. The walls of blood vessels in the body are made up of 3 layers, from inside to outside, intima, media, and adventitia. In aortic dissection, a tear in the aortic intima layer occurs. This allows blood to pass from the aortic lumen, through the intimal tear into the space between the intima and media layers. This creates a false lumen between the intima and media layers of the aortic wall. 65% of aortic intimal tears occur at the sinotubular junction of the ascending aorta and extends to involve the remaining thoracic aorta, creating an ascending aortic dissection. Around 30% of intimal tears originate close to the area where the left subclavian artery branches off of the aorta and extends to involve the remaining descending aorta, creating a descending aortic aneurysm. The reason that aortic dissection most commonly affects these 2 areas is thought to be due to shear forces being the highest in these areas. Aortic dissection usually presents with stabbing chest pain that radiates to the upper back and shoulders and a greater than 20 mm Hg difference in the systolic blood pressure between both arms in the setting of a history of hypertension or a connective tissue disorder (e.g., Marfan syndrome). Depending upon the area and behavior of the aortic dissection, other signs and symptoms may be present. Ascending aorta dissections can be complicated by aortic valve regurgitation (diastolic aortic murmur), myocardial infarction (due to obstruction of the coronary artery ostia), cardiac tamponade (rupture of the dissection into the pericardium), stroke (carotid artery occlusion), horners syndrome (compression of superior cervical ganglia), vocal cord paralysis (recurrent laryngeal nerve compression). Descending aorta dissections can be complicated by hemothorax (rupture of dissection into the thorax), renal injury (renal artery obstruction), mesenteric ischemia (obstruction if superior mesenteric artery), lower limb paraplegia (spinal cord ischemia due to obstruction of intercostal vessels). The goal of management in both ascending and descending aortic dissection is pain control, lowering of the systolic blood pressure and decreasing left ventricular contractility to decrease aortic wall shear stress. IV beta-blocker (e.g., esmolol, labetalol) decrease the heart rate, both decreasing the blood pressure and left ventricular contractility and is the preferred medication used in the initial management of acute aortic dissection. In addition to the above, emergent surgical repair is indicated in cases of ascending aortic dissection due to its risk of life-threatening complications. The mortality rate of ascending aortic dissection is 1%-2% per hour after the onset of symptoms. Go to the next page if you knew the correct answer, or click the link image(s) below to further research the concepts in this question (if desired).

Research Concepts:

Aortic Dissection

We update eBooks quarterly and Apps daily based on user feedback. Please tap flag to report any questions that need improvement.

Question 3: A 65-year old male with uncontrolled type 2 diabetes mellitus who is admitted to the cardiac care unit after undergoing three-vessel coronary artery bypass graft after a recent myocardial infarction. He is currently receiving intravenous insulin infusion for his hyperglycemia. On a postoperative day 3, you note that he is off vasopressors and his blood glucose is maintained between 130 to 160 mg/dL on a stable rate of insulin infusion. However, his caloric intake is minimal. You plan to transition him to subcutaneous insulin based on the average rate of infusion over the last 6 hours of 1.5 units/hour. What are the best process and insulin doses needed to manage this transition?

Choices: 1. Two hours prior to discontinuing the intravenous insulin infusion, administer subcutaneous insulin glargine 25 units 2. Simultaneously, discontinue the intravenous insulin drip and administer subcutaneous insulin glargine 25 units 3. Two hours prior to discontinuing intravenous insulin, administer subcutaneous insulin glargine 18 units 4. Discontinue intravenous insulin drip and start correctional insulin coverage every 4 hours

Answer: 1 - Two hours prior to discontinuing the intravenous insulin infusion, administer subcutaneous insulin glargine 25 units

Explanations: Due to the extremely short half-life of intravenous (IV) insulin and delayed onset of action of long-acting subcutaneous insulin, it is important to overlap the two types of insulin by 23 hours. Premature discontinuation of IV insulin creates a hiatus in basal insulin supply, which risks rebound hyperglycemia or metabolic decompensation. When using the rate of infusion to calculate the basal insulin dose, the average rate of infusion over the last 6 to 8 hours is extrapolated to 24 hours. Seventy to eighty percent of this extrapolated dose represents the total daily dose (TDD) of insulin requirement. In a patient with minimal or no caloric intake, 100% of TDD is the basal requirement. In patients with optimal caloric intake, half of TDD should be administered as basal insulin and the other half as nutritional insulin. In the above case scenario, the average rate of infusion is 1.5 units/hour over the last 6 hours. On extrapolating this to the 24-hour period, the average amount of insulin is 36 units. As a patient has minimal caloric intake, 70-80% of the extrapolated dose (25-28 units) is the basal requirement and should be administered at least 2 hours before discontinuation of IV insulin. If the patient in the above case had an optimal caloric intake while on the insulin drip, then option three would have been the right choice. In that case, the calculate TDD (i.e.70-80% of the extrapolated dose) would represent both basal and nutritional requirements as correctional insulin alone does not provide any basal coverage and risks wide fluctuations in glycemic control. Go to the next page if you knew the correct answer, or click the link image(s) below to further research the concepts in this question (if desired).

Research Concepts: Diabetic Perioperative Management

We update eBooks quarterly and Apps daily based on user feedback. Please tap flag to report any questions that need improvement.

Question 4: A 42-year-old male presents to the emergency department with cough, dyspnea, and hemoptysis with a history of idiopathic pulmonary hemosiderosis. While in the hospital, the patient's hemoptysis worsens, and he is now progressing towards acute respiratory failure, leading to intubation. The patient underwent bronchoscopy with serial washings and was diagnosed with diffuse alveolar hemorrhage. The patient is stabilized on the ventilator in the ICU. Which of the following is the most appropriate treatment for the patient's ongoing disease process at this time?

Choices: 1. IV methylprednisolone 500 to 2000mg/day for 5 days 2. IV Azathioprine 2mg/kg/day for 6 weeks 3. Inhaled fluticasone 1760 mcg/day indefinitely 4. Stat consult to surgery for emergent splenectomy

Answer: 1 - IV methylprednisolone 500 to 2000mg/day for 5 days Explanations: There are currently no well-done studies to indicate that the current management is “gold standard”. Most treatment and recommendations are based off observation and clinical experience of the treating physician. Of the many approaches to idiopathic pulmonary hemosiderosis (IPH) treatment, systemic steroids and immunosuppressant therapy have shown the most favorable outcome either used independently or in combination. Patients with acute IPH and respiratory failure secondary to alveolar hemorrhage may require invasive ventilation support. For patients with this presentation, it is recommended they be started on IV methylprednisolone in pulse dosing of 500 to 2000mg/day for 5 days. In children, pulse dosing is typically 20mg/kg per day. Once the patient is stabilized this can be transitioned to oral steroids with tapering down to a maintenance dose of 10 to 15mg/day. Immunosuppressive agents including cyclophosphamide, hydroxychloroquine, and azathioprine have been shown to help with severe disease. Dosing, duration, and optimal indications for immunosuppressive agents are poorly defined. Studies for treatment of IPH have shown splenectomy does not provide any morbidity or mortality benefit. Treatment of possible underlying causes such as Goodpasture syndrome (anti-GBM antibodies) or drug-induced injury (such as that caused by amiodarone) should also be addressed. Go to the next page if you knew the correct answer, or click the link image(s) below to further research the concepts in this question (if desired).

Research Concepts: Idiopathic Pulmonary Hemosiderosis

We update eBooks quarterly and Apps daily based on user feedback. Please tap flag to report any questions that need improvement.

Question 5: A 17-year-old teenager with known sickle cell disease presents to the emergency department with severe back and extremity pain for the past two days. The patient has been taking oxycodone and ibuprofen at home with no significant relief. He denies any fever, cough, or shortness of breath. Past medical history is significant for several hospitalizations for painful crisis. He has not taken any medications for the past six hours and had one episode of vomiting prior to his arrival. Physical exam reveals that he is in significant discomfort and is writhing in pain. His pain score is 9/10. Vital signs reveal a pulse of 79/min, respiratory rate 22/min, temperature 98.5 F, blood pressure 124/85 mmHg, and oxygen saturation 97% in room air. He is given 4 mg morphine subcutaneously and IV access obtained. His pain score 30 minutes later is recorded as 8/10. What is the next best step in the management of this patient?

Choices: 1. Escalate the opiate dose given intravenously by 25% every 15 minutes until the pain is controlled 2. Immediate start dose of one gram ceftriaxone given intravenously 3. Give IV fluids at 2 times the maintenance dose 4. Withhold opiates as the patient may be drug seeking

Answer: 1 - Escalate the opiate dose given intravenously by 25% every 15 minutes until the pain is controlled

Explanations: Patients presenting with sickle cell crisis should be triaged to emergency severity index (ESI) category 2 (high priority) for prompt and aggressive treatment of pain and should receive their first analgesic dose within 30 minutes of arrival. Reassessment of pain is recommended every 15-30 minutes, and for persistent pain, escalation of the analgesic dose by 25% is recommended until the pain is controlled. Addition of non-steroidal agents (ketorolac) may provide additional analgesia. Nonpharmacological measures such as local heat, distraction, acupuncture, and hypnosis have been variably shown to be beneficial. Patient-controlled analgesia has shown to be more effective than intermittent dosing to reduce the length of stay in the hospital and increased patient satisfaction. This can often be initiated in the emergency department and then continued during the in-patient course. Antibiotics are not generally indicated for painful crisis except if the patient is febrile or shows clinical features suggestive of acute chest syndrome. Intravenous fluids should be used judiciously to maintain hydration as excess fluid therapy has been implicated as a factor in the occurrence of acute chest syndrome and normal saline bolus infusions have been associated with poorer pain control. Go to the next page if you knew the correct answer, or click the link image(s) below to further research the concepts in this question (if desired).

Research Concepts: Sickle Cell Crisis

We update eBooks quarterly and Apps daily based on user feedback. Please tap flag to report any questions that need improvement.

Question 6: A patient is being treated for malaria arrives in the emergency department with a chief complaint of ringing in the ears, blurred vision, nausea and vomiting, and confusion. On physical exam, the patient has a temperature, is hypotensive, and a fingerstick glucose reads 50. Which of the following would not be an expected?

Choices: 1. Massive hemolysis 2. Hemoglobinuria 3. Renal failure 4. Sweet taste in the mouth

Answer: 4 - Sweet taste in the mouth Explanations: Drugs like quinidine are associated with cinchonism, which is characterized by tinnitus, blurred vision, gastrointestinal upset, and delirium. Cinchonism or quinism is caused by an overdose of quinine or quinidine, or cinchona bark. Quinine is used to treat malaria. In small amounts, quinine is an ingredient of tonic drinks acting as a bittering agent. Cinchonism can occur from therapeutic doses of quinine, either from one or several large doses. Quinidine can cause cinchonism symptoms to develop with as little as a single dose. Symptoms of mild cinchonism include flushed and sweaty skin, ringing in the ears, blurred vision, impaired hearing, confusion, reversible high-frequency hearing loss, headache, abdominal pain, rashes, lichenoid photosensitivity, vertigo, dizziness, dysphoria, nausea, vomiting, and diarrhea. Large doses may result in severe reversible symptoms of cinchonism which includes skin rashes, deafness, somnolence, diminished visual acuity or blindness, anaphylactic shock, and disturbances in abnormal heart rhythm or conduction, and potentially death from cardiotoxicity. Quinine may trigger a rare form of hypersensitivity reaction in malaria patients termed blackwater fever with massive hemolysis, hemoglobinemia, hemoglobinuria, and kidney failure. Patients treated with quinine may develop hypoglycemia and hypotension. Most symptoms of cinchonism are reversible and disappear once quinine is withdrawn. Quinine inactivates enzymes in the lysosomes of cells, and has an anti-inflammatory effect. As a result, it sometimes used to treat rheumatoid arthritis. Switching off these enzymes can cause abnormal accumulation of glycogen and phospholipids in lysosomes, resulting in a toxic myopathy. This may be the cause of cinchonism. Go to the next page if you knew the correct answer, or click the link image(s) below to further research the concepts in this question (if desired).

Research Concepts: Cinchonism

We update eBooks quarterly and Apps daily based on user feedback. Please tap flag to report any questions that need improvement.

Question 7: A 16-year-old boy who had been recovering from a flu-like illness presented to the emergency department with acutely worsening dyspnea at rest, dizziness, and lethargy for two days. He had a low-grade fever, and vitals at the time of presentation were as follows blood pressure 74/35 mmHg, heart rate 118/min, respiratory rate 36/min, and oxygen saturation of 89%. EKG showed sinus tachycardia, and labs were significant for elevated troponin and proBNP. He was started on inotropic support after a failed fluid challenge. An echocardiogram revealed a globally hypokinetic myocardium with severely reduced left ventricular ejection fraction (18%), normal ventricular dimensions, and slightly thickened left ventricular walls. An endomyocardial biopsy revealed multiple foci of CD3+ cell-rich infiltrates with widespread myocyte necrosis and interstitial edema. The patient underwent intubation and was placed on mechanical ventilation as he became more lethargic, but he continues to be hypotensive despite maximal inotropic support. What is the most appropriate next step?

Choices: 1. Implantable cardioverter-defibrillator (ICD) 2. Biventricular pacemaker/cardiac resynchronization therapy (CRT) 3. Left ventricle assist device (LVAD) 4. Cardiac transplantation

Answer: 3 - Left ventricle assist device (LVAD) Explanations: Fulminant myocarditis occurs in children and young adults and is characterized by acute severe decompensation of cardiac function following a viral respiratory or gastrointestinal illness. It is seen on echocardiogram as a hypokinetic myocardium with normal left ventricular dimensions but a slightly thickened ventricular wall because of edema. It can resolve spontaneously but may require aggressive interventions like LVAD (mechanical circulatory support) during its active phase, which can improve cardiac output. Extracorporeal membranous oxygenation (ECMO) is a quicker and less expensive alternative for LVAD, which can be used as a bridge to recovery. However, it may not increase the cardiac output as much as an LVAD. Go to the next page if you knew the correct answer, or click the link image(s) below to further research the concepts in this question (if desired).

Research Concepts: Nonviral Myocarditis

We update eBooks quarterly and Apps daily based on user feedback. Please tap flag to report any questions that need improvement.

Question 8: A 56-year-old man with a history of alcohol use disorder and recurrent bouts of pancreatitis presents with severe abdominal pain, hypotension, and tachycardia. Lipase is elevated over 1000 U/L. White blood cell count is 14,000/microL, and hemoglobin is 6.5 g/dL. He is cachectic appearing and pale. He has tenderness to palpation in the epigastrium. His last admission for pancreatitis was two weeks ago, and at that time he had a CT scan without contrast that was concerning for a small peripancreatic fluid collection which may represent a developing pseudocyst. He denies any bloody emesis or hematochezia. What should be done next?

Choices: 1. Obtain an ultrasound of the abdomen 2. Obtain a CT angiogram of the abdomen and pelvis 3. Admit the patient for serial abdominal exams and serial hematocrit checks only 4. Emergent exploratory laparotomy

Answer: 2 - Obtain a CT angiogram of the abdomen and pelvis Explanations: While ultrasound can show filling of a pseudoaneurysm, the exam can be limited by bowel gas or body habitus and a more definitive test which can show cross-sectional anatomy is better This patient may have a visceral artery pseudoaneurysm secondary to chronic pancreatitis. A non-contrasted scan cannot reliably differentiate between pancreatic fluid and blood, and a contrasted scan would help identify both aneurysmal and pseudoaneurysmal degeneration of an artery exposed to pancreatic fluid. After the diagnosis is confirmed, an endovascular or open repair can be attempted. This patient shows signs of hemorrhagic shock. Observation alone is not appropriate. An exploratory laparotomy without a definitive diagnosis is ill-advised as the patient could instead have chronic anemia from malnutrition and chronic disease and a systemic inflammatory response syndrome from active pancreatitis. Go to the next page if you knew the correct answer, or click the link image(s) below to further research the concepts in this question (if desired).

Research Concepts: Pseudoaneurysm

We update eBooks quarterly and Apps daily based on user feedback. Please tap flag to report any questions that need improvement.

Question 9: A 17-year-old male presents after ingesting 100 pills of 81 mg aspirin 55 minutes ago. His Glasgow Coma Scale score is 15, airway reflexes are intact, respiratory rate is 25, heart rate 105 beats/min, blood pressure 120/80 mmHg, and temperature 101.5 F. He weighs 55 kg. What is the appropriate dose of single-dose activated charcoal?

Choices: 1. None, he is outside of the 30-minute timeframe for administration of activated charcoal 2. 100 grams of activated charcoal 3. 55 grams of activated charcoal 4. 80 grams of activated charcoal

Answer: 4 - 80 grams of activated charcoal Explanations: Activated charcoal (AC) can absorb certain non-polar toxic materials that are in the liquid phase within the gastrointestinal (GI) lumen. The typical timeframe of AC administration is within 1 hour of ingestion. However, substances with a large pill burden that may form bezoars, substances that undergo enterohepatic or enteroenteric recirculation, or substances that decrease GI motility may benefit from activated charcoal administration up to 4 hours after ingestion. In cases of a known xenobiotic dose, AC should be given at a 10:1 ratio of AC to the xenobiotic. In this case, approximately 8 grams of aspirin was ingested, so 80 grams of activated charcoal is indicated. When the xenobiotic dose is unknown, doses of AC should be given at 1 to 2 gram/kg of body weight. If the amount of aspirin was unknown, it would be reasonable to administer 55 grams of AC to this patient. Go to the next page if you knew the correct answer, or click the link image(s) below to further research the concepts in this question (if desired).

Research Concepts: Activated Charcoal

We update eBooks quarterly and Apps daily based on user feedback. Please tap flag to report any questions that need improvement.

Question 10: A 24-year-old man presents to the office for regular blood transfusion for beta-thalassemia major. His recent laboratory results reveal the development of alloantibodies against red blood cells (RBCs) surface antigen. His current blood pressure is 116/78 mmHg, the pulse is 74/min, and the temperature is 99 F. He appears in no apparent distress, and his last checked hemoglobin was 7 g. Which type of blood may be appropriate for transfusion for this patient at this visit?

Choices: 1. Leukoreduced RBCs 2. Washed packed RBCs 3. Frozen packed RBCs 4. Irradiated RBCs

Answer: 3 - Frozen packed RBCs Explanations: Patients with thalassemia are usually given packed RBCs with the removal of leukocytes for regular transfusion. But patients who develop alloantibodies may require frozen packed RBCs. The product of choice is packed red blood cells depleted of leucocytes. It should be matched with the patient’s red antigen phenotype for at least D, C, c, E, e, and Kell. Frozen (or cryopreserved) red cells is the component derived from whole blood in which red cells are frozen, preferably within 7 days of collection. These are used to maintain a supply of rare donor units for patients who have unusual red cell antibodies or who are missing common red cell antigens. Washed RBCs are given when a patient has a history of allergic reactions to blood transfusion in the past. Irradiated RBCs are usually used to prevent graft-versus-host disease. It is largely unnecessary unless the patient is undergoing a bone marrow transplant or has an underlying immunodeficiency. Go to the next page if you knew the correct answer, or click the link image(s) below to further research the concepts in this question (if desired).

Research Concepts: Thalassemia

We update eBooks quarterly and Apps daily based on user feedback. Please tap flag to report any questions that need improvement.

Question 11: A 65-year-old female is postoperative day one status post right hip replacement. She is hypotensive, tachycardic, and hypoxic. Which of the following is the appropriate therapy for this patient?

Choices: 1. Pulmonary embolectomy 2. Heparin 3. Lytic therapy 4. Hirudin

Answer: 3 - Lytic therapy Explanations: Acute pulmonary embolectomy sometimes is required in patients who are hemodynamically unstable with a confirmed embolus. However, current guidelines indicate that the treatment of choice is lytic therapy in unstable patients. Lytic therapy works rapidly and the dose of tPA is 100 mg over 2 hours. Surgery is only undertaken if there are contraindications to lytic therapy. Surgery in these patients carries a very high mortality and is the last treatment of choice. Anticoagulant therapy is the mainstay of treatment for patients who are hemodynamically stable. Placement of an inferior vena cava filter should be considered if anticoagulants are contraindicated or a pulmonary embolism occurs in a patient who is being anticoagulated. Go to the next page if you knew the correct answer, or click the link image(s) below to further research the concepts in this question (if desired).

Research Concepts: Acute Pulmonary Embolism

We update eBooks quarterly and Apps daily based on user feedback. Please tap flag to report any questions that need improvement.

Question 12: A 60-year-old male with a past medical history of hypertension, myocardial infarction, and congestive heart failure with reduced ejection fraction comes to the emergency department complaining of worsening exertional dyspnea over the past 3 days. He states that his lower extremity edema and orthopnea has also been progressively increasing. His current medications include metoprolol, losartan, furosemide, and aspirin. His blood pressure is 124/85 mmHg, and pulse is 95 bpm, irregularly irregular. On chest auscultation, there are significant bilateral crackles over the lower lung fields, a holosystolic murmur at the apex, and an S3 gallop heard at the left sternal border. Laboratory evaluation shows hemoglobin 9.5 g/dL, leukocyte count of 5,500/microliter, and platelet count of 250,000/microliter. Serum chemistries show that sodium is 125 mEq/L, and brain natriuretic peptide (BNP) is 800 pg/mL. Electrocardiogram shows atrial fibrillation. In cases of acute decompensation of heart failure, what have studies shown to be associated with poor prognosis?

Choices: 1. Maintaining oxygen extraction of 27% or less 2. Central venous oxygen saturation (ScvO2) 70% 3. Central venous oxygen saturation (ScvO2) 60% 4. Central venous oxygen saturation (ScvO2) > 80 %

Answer: 3 - Central venous oxygen saturation (ScvO2) 60% Explanations: In certain studies, maintaining oxygen extraction of 27% or less was found to lead to decreased incidences of organ failure and decreased hospital length of stay in patients in the perioperative setting. Studies also demonstrated poor prognosis of heart failure with central venous oxygen saturation of less than 60%. Studies have shown that ScvO2 70% is associated with poor prognosis in sepsis. Meanwhile, studies have shown that ScvO2 60% is associated with poor prognosis in heart failure. Studies also demonstrate that heart failure causes decreased oxygen delivery, thus increasing tissue oxygen extraction. This is exhibited in cardiogenic shock. Studies have found that ScvO2 60% within the first 24 hours is also a predictor of major adverse cardiovascular events. Go to the next page if you knew the correct answer, or click the link image(s) below to further research the concepts in this question (if desired).

Research Concepts: Anesthesia Monitoring Of Mixed Venous Saturation

We update eBooks quarterly and Apps daily based on user feedback. Please tap flag to report any questions that need improvement.

Question 13: A 17-year-old male with no past medical history presents to the emergency department severely intoxicated after reportedly ingesting a bottle of isopropyl alcohol. His labs are remarkable for a creatinine of 1.8 mg/dL and a BUN of 10 mg/dL. Ketones are detectable in urine. He vomited twice and urinated in his hospital bed. What is the most appropriate management?

Choices: 1. Observe until clinically well and discharge with a responsible adult 2. Admit to the intensive care unit 3. Administer fomepizole and admit to the intensive care unit 4. Consult nephrology for dialysis

Answer: 1 - Observe until clinically well and discharge with a responsible adult Explanations: Isopropyl is known to cause severe intoxication, hemorrhagic gastritis, and falsely elevated creatinine due to acetone (Jaffe reaction). Most patients improve with supportive care, including hydration, and can be discharged if ingestion was not a self-harm attempt. Fomepizole is not required to treat isopropyl alcohol ingestion because its metabolite does not cause acidosis or any specific end-organ toxicity. Dialysis is not necessary as the creatinine is falsely elevated. This requires a repeat basic metabolic panel when clinically improved to reassess creatinine, but the normal blood urea nitrogen does not correlate with the elevated creatinine, raising the concern for a falsely elevated creatinine. Go to the next page if you knew the correct answer, or click the link image(s) below to further research the concepts in this question (if desired).

Research Concepts: Isopropanol Toxicity

We update eBooks quarterly and Apps daily based on user feedback. Please tap flag to report any questions that need improvement.

Question 14: A 32-year-old female with placenta percreta is posted for an elective caesarian section and obstetric hysterectomy. After delivery, there was massive blood loss, and blood transfusion protocol was initiated as per institutional guidelines. Thromboelastogram report showed increased K time and R time and normal maximum amplitude. What should be administered to the patient to aid in hemostasis?

Choices: 1. Packed red blood cells and platelets 2. Plasma and cryoprecipitate 3. Plasma and platelets 4. Platelets only

Answer: 2 - Plasma and cryoprecipitate Explanations: R time (reaction time) is the time of latency from the start of the test to initial fibrin formation. It is dependent on clotting factors. The normal value is 4-8 minutes. K time is dependent on fibrinogen and signifies time taken to achieve certain clot strength (amplitude of 20 mm). Cryoprecipitate and fibrinogen concentrate are good sources for fibrinogen. The normal value is 1-4 minutes. Alpha angle (degrees) measures the rate at which fibrin builds up, and cross-linking assesses, in turn, the rate of clot formation. The function also depends on fibrinogen levels. The normal value is 47-74 degrees. Maximum amplitude (mm) represents the clot strength which is a result of the function of platelets (80%) and fibrin (20%). It helps to identify whether the cause of bleeding is due to coagulopathy or mechanical disruption. The normal value is 55-73 mm. Go to the next page if you knew the correct answer, or click the link image(s) below to further research the concepts in this question (if desired).

Research Concepts: Fibrinogen

We update eBooks quarterly and Apps daily based on user feedback. Please tap flag to report any questions that need improvement.

Question 15: A 20-year-old presents to the emergency in an active seizure. Friends report that she had taken a large amount of a medication prescribed for stomatitis. On arrival, blood pressure is 80/50 mmHg, pulse 130 bpm, and respiratory rate 8/minute. What medication should be avoided in this patient?

Choices: 1. Benzodiazepines 2. Levetiracetam 3. 20% lipid emulsion 4. Phenytoin

Answer: 4 - Phenytoin Explanations: Benzodiazepines are the first line for treatment of seizures. Levetiracetam can be given if seizures are refractory to benzodiazepines. Average adult patients should be given a loading dose of 1g or 20 mg/kg of levetiracetam. This patient presents with lidocaine toxicity after overdosing on lidocaine prescribed for stomatitis. Lipid emulsion should be used in an attempt to reverse the toxicity of the lidocaine. Phenytoin should be avoided in lidocaine toxicity as it also has sodium channel blocking effects. Use of phenytoin could potentially worsen the condition. Go to the next page if you knew the correct answer, or click the link image(s) below to further research the concepts in this question (if desired).

Research Concepts: Sodium Channel Blocker Toxicity

We update eBooks quarterly and Apps daily based on user feedback. Please tap flag to report any questions that need improvement.

Question 16: A 65-year-old female with past medical history of atrial fibrillation, hypertension, chronic old left bundle branch block and diabetes mellitus is admitted to the hospital with community-acquired pneumonia. She is on dofetilide for atrial fibrillation and apixaban for anticoagulation. She is started on levofloxacin 750 mg IV for pneumonia. Twelve hours after the admission, the patient becomes unconscious, and the telemetry shows she has polymorphic ventricular tachycardia. Immediate defibrillatory shock was delivered, and patient reverted to sinus rhythm. The patient was transferred to the cardiac intensive care unit. It is noted that she is on levofloxacin which prolongs QTc and dofetilide also has QTc prolonging effect. Both medications are discontinued immediately. Which of the following should be recommended?

Choices: 1. Dual chamber pacemaker as she went into ventricular tachycardia 2. List dofetilide in her allergies medications and never prescribe it to her again 3. Stop levofloxacin and hold the dofetilide until the QTc normalizes 4. Hold apixaban

Answer: 3 - Stop levofloxacin and hold the dofetilide until the QTc normalizes Explanations: A pacemaker with implantable cardioverter defibrillator (ICD) is indicated for patients who go into ventricular tachycardia (VT) without a reversible cause. It is a very common indication for the secondary prevention of VT. She may have had good control with dofetilide for her atrial fibrillation hence listing in her allergies is not reasonable. Reversible polymorphic ventricular tachycardia is not an indication for the ICD placement. In this scenario, stop the offending QTc prolonging drugs like fluoroquinolones and dofetilide. Patients with atrial fibrillation who have a CHADSVasc score greater than or equal to 2 should be prescribed an anticoagulation agent. Go to the next page if you knew the correct answer, or click the link image(s) below to further research the concepts in this question (if desired).

Research Concepts: Ventricular Tachycardia

We update eBooks quarterly and Apps daily based on user feedback. Please tap flag to report any questions that need improvement.

Question 17: A 48-year-old male presents to the emergency department with two-and-a-half months of progressively worsening left-sided chest pains and shortness of breath with associated low-grade fevers, chills, occasional night sweats, productive cough of foul tasting sputum, and 15 lbs unintentional weight loss. He does not have a primary care provider, denies any medical or surgical history, does not believe in "Big Pharma" and takes no medications. He denies any sick contacts, refuses all vaccinations, does not drink any alcohol, use any tobacco products or illicit drugs, but does report an extensive travel history in South Asia over the last two years as a Christian missionary. He reports that he got home about six weeks ago from Vietnam, but his symptoms started off milder probably 6 to 8 weeks before that. He states they seemed to coincide a week or so after he had gotten over a really bad case of food allergies. He was not sure what he ate that set it off but reported a two-week history of very high fevers, abdominal pains, and cramping with nausea, vomiting, loss of appetite, and generalized itching that gradually subsided. He did seek medical treatment at one point, but only got intravenous fluids, an antihistamine, and antiemetic medication. They did not do any blood work or imaging. He reports feeling better for about a week before the new symptoms started. He was busy preparing for his trip back to America, so he did not pursue any medical care until now. He comes in tonight because he can't take the fevers and night sweats any more which woke him up once again from sleep. He does admit that he searched his symptoms on the internet last week and is worried that he might have cancer. Vitals signs are temperature 38.4 C, heart rate 108 bpm, respiratory rate 24/minute, blood pressure 112/67 mmHg, and oxygen saturation 86% on room air. He appears acutely ill, no acute distress and is normocephalic and atraumatic. Sclera are nonicteric and pupils equal round and reactive to light and accommodation. The neck nontender with no jugular venous distention. He is mildly tachycardic with regular rhythm and no murmur. Lungs are clear to auscultation bilaterally. The abdomen is soft, nontender with normal bowel sounds. There is no costovertebral angle tenderness or bladder distension. He has normal speech with no neurologic deficits. His chemistry panel shows normal electrolytes, normal renal function, and normal hepatic function. His complete blood count shows mild leukocytosis with WBC count of 11.8/microL and differential reveals 6.8% eosinophils. His chest x-ray shows a there is a large cavitary lung lesion in the left upper lobe. CT scan of the chest shows there is a 4 cm x 6 cm ring-enhancing cavitary lung abscess in the left upper lobe with air-fluid level. There are no malignant findings. The patient is diagnosed with sepsis secondary to a lung abscess. He is admitted to the medical floor and started on broad-spectrum antibiotics with intravenous vancomycin and intravenous ampicillin-sulbactam. Over the next three days, the patient continues to fever and have persistent symptoms. He undergoes bronchoscopy and bronchoalveolar lavage on hospital day four. Cytology of the fluid is negative for malignancy but does note a high preponderance of eosinophils and trematode eggs that are yet to be characterized. His blood and sputum cultures remain negative since admission. Based on the above clinical scenario, what is the most likely causative organism?

Choices: 1. Taenia solium 2. Enterobius vermicularis 3. Diphyllobothrium latum 4. Fasciola giganticum

Answer: 4 - Fasciola giganticum Explanations: Ectopic fascioliasis is very rare, but the parasite may present in nearly any area of the body and has been described in the literature. While Fasciola gigantica go almost anywhere, Enterobius vermicularis and Diphyllobothrium latum stay primarily in the gastrointestinal tract. Taenia solium has been known to cause primarily central nervous system lesions from larval migration. Peripheral eosinophilia in varying degrees is common amongst all filarial worm infections and is nonspecific, but should clue one into a parasite as the culprit. Fasciola giganticum infection is more commonly associated with travel to Asia, the Pacific Islands, and some parts of Northern Africa than any of the other species listed. The patient had spent the last two years in South Asia as a missionary and had just returned. The patient had complained of a severe gastrointestinal illness lasting a week or two that preceded his current illness. This likely signified the acute phase of the fasciola infection when metacercariae migrate out of the intestines and through the liver causing an overwhelming inflammatory and immune response. Go to the next page if you knew the correct answer, or click the link image(s) below to further research the concepts in this question (if desired).

Research Concepts: Fascioliasis

We update eBooks quarterly and Apps daily based on user feedback. Please tap flag to report any questions that need improvement.

Question 18: A 73-year-old man with a past medical history of congestive heart failure, atrial fibrillation, and hypertension presents with jugular venous distention, exertional dyspnea, and bilateral lower leg edema for the past 2 days. Patient's ejection fraction is shown to be 20% by echocardiogram. In the evening of admission, the patient's blood pressure drops to 65/30 mmHg and is placed on continuous intravenous dobutamine. Hours later, his blood pressure increases to 90/45 mmHg, and EKG shows atrial fibrillation with rapid ventricular response. Which of the following medications would have been the best choice to prevent the abnormal EKG findings?

Choices: 1. Metoprolol after dobutamine administration 2. Digoxin before dobutamine administration 3. Atropine along with dobutamine 4. Amiodarone at any point during the infusion

Answer: 2 - Digoxin before dobutamine administration Explanations: Digoxin regimen is given prior to dobutamine to reduce the risk of rapid ventricular response in patients with a history of atrial fibrillation. Dobutamine increases the risk of risk of rapid ventricular response in patients with preexisting atrial fibrillation. Dobutamine can be administered in heart failure patients with systolic dysfunction and hypotension for the short-term. Dobutamine cannot be combined with beta-blocker medications because of its sympathomimetic effects. Go to the next page if you knew the correct answer, or click the link image(s) below to further research the concepts in this question (if desired).

Research Concepts: Dobutamine

We update eBooks quarterly and Apps daily based on user feedback. Please tap flag to report any questions that need improvement.

Question 19: A 65-year-old white female is admitted to the intensive care unit after presenting with acute onset of severe respiratory distress, productive cough, fever, and progressive obtundation. Initial vitals showed a temperature of 94 F with a heart rate of 110/min, respiratory rate of 34/min, and a mean arterial pressure of 55 mmHg. Respiratory status declined rapidly to require endotracheal intubation. She also became progressively hypotensive requiring 3 liters of normal saline bolus within 2 hours and is now on norepinephrine infusion. She has a central line in the right subclavian and a radial arterial line. She is asynchronous on the vent and breathing at 30/min with a set rate of 16/min. Tidal volume is at 6 ml/kbw. What is the most reliable index to ascertain the need for additional fluid bolus?

Choices: 1. Central venous pressure less than 10 cmH2O 2. IVC variability greater than 13% 3. Pulse pressure variability greater than 13% 4. Pulse pressure variability greater than 10% with passive leg raise test

Answer: 4 - Pulse pressure variability greater than 10% with passive leg raise test Explanations: The decision to administer fluid in shock is increasingly being guided by dynamic measures of volume responsiveness than static measures like central venous pressure (CVP). Dynamic measures of volume responsiveness include IVC variability with respiration, pulse pressure variability, stroke volume variability with respiration or with passive leg raising. The technique for performing passive leg raising involves 1) sitting patient at 45 degrees head-up semi-recumbent position, 2) lower patient’s upper body to horizontal and passively raise legs at 45 degrees up, 3) maximal effect occurs at 30-90 seconds, 4) assess for a 10% increase in stroke volume (cardiac output monitor) or pulse pressure variability (using an arterial line). The passive leg raise (PLR) has the advantage of being reversible; its effect is reversed once the legs are returned to horizontal. The volume expansion from a PLR is 250-350 mL. Pulse pressure variability or stroke volume variability with passive leg raising is the most specific index of volume responsiveness at the bedside, with the exception of significant tachyarrhythmias. Go to the next page if you knew the correct answer, or click the link image(s) below to further research the concepts in this question (if desired).

Research Concepts: Systemic Inflammatory Response Syndrome

We update eBooks quarterly and Apps daily based on user feedback. Please tap flag to report any questions that need improvement.

Question 20: A 30-year-old woman was brought to the emergency department by her mother altered mentation. This patient his on medications for anxiety, depression with suicide attempt and chronic pain. She has attempted suicide in the past. According to her mother, she has been under tremendous amounts of stress with work and personal relationships, and she reports that at home the patient was found with scattered pills on the floor. She is tachycardic at 135 bpm, hypertensive at 165/100 mmHg, not tachypneic and she is febrile at 38.4 C. She has dry mucous membranes with flushed skin and mydriasis which is equal to both pupils. She is awake but noted to be picking and swiping at the air as if something is in front of her. She is neurologically intact otherwise. What is the most likely substance she ingested?

Choices: 1. Phenelzine 2. Pseudoephedrine 3. Diphenhydramine 4. Tramadol

Answer: 3 - Diphenhydramine Explanations: For the patient with drug intoxication an possible medication overdose, the clinician must rely on a good history and physical examination for signs and symptoms. Her clinical picture is consistent with diphenhydramine toxicity/overdose which is a histamine H1receptor antagonist with moderate to high anticholinergic and antiemetic properties. Given the history of chronic pain and depression she likely has access to antidepressants and pain medicines, but over the counter drugs must also be taken into consideration. Exam findings of tachycardia, hypertension, fever, dry mucous membranes, flushing, dilated pupils, and altered mental status. fit with anticholinergic toxidrome. Most patients with anticholinergic poisoning do well with supportive therapy alone. With severe agitation and delirium, patients may benefit from treatment with physostigmine. Consultation with a toxicologist is recommended before giving physostigmine. Go to the next page if you knew the correct answer, or click the link image(s) below to further research the concepts in this question (if desired).

Research Concepts: Diphenhydramine Toxicity

We update eBooks quarterly and Apps daily based on user feedback. Please tap flag to report any questions that need improvement.

Question 21: A 65-year-old male presents complaining that he cannot move his legs. He denies any trauma, loss of consciousness, or visual or speech problems. On examination, there is significant motor weakness, numbness from the buttocks to the soles of the feet, no sensation in the perineal area, absent rectal tone, and a full, distended bladder. What is the best test for this patient?

Choices: 1. Lumbar myelogram 2. MRI spine 3. Spinal tap 4. CT head

Answer: 2 - MRI spine Explanations: Classic cauda equina syndrome presents with leg weakness, bladder or bowel failure, and hypesthesias from the buttocks to the feet. MRI will give the best view of the cauda equina and lower spinal cord. It will quickly determine if there is a lesion causing compression that requires surgical intervention. Cauda equina syndrome can be caused by trauma, neoplasms, and spinal abscesses. Go to the next page if you knew the correct answer, or click the link image(s) below to further research the concepts in this question (if desired).

Research Concepts: Cauda Equina And Conus Medullaris Syndromes

We update eBooks quarterly and Apps daily based on user feedback. Please tap flag to report any questions that need improvement.

Question 22: A 60-year-old Indonesian man is brought by ambulance with generalized weakness, joint pain, and double vision. The family reported that the patient had fallen twice this week due to the inability to ambulate. The wife states that the patient had been dragging his feet. Further history revealed that they had gone for a 30th wedding anniversary cruise two weeks ago and ate "just about everything." The patient returned home and had been feeling ill with fever, productive cough, sinus congestion that resolved with rest and acetaminophen. The patient received his annual influenza vaccination. On neurological exam, the patient had inaccuracies on the visual field testing, blunted corneal reflexes, numbness in his mouth, and loss of taste for sweets. Deep tendon reflexes showed trace in patellar deep tendon reflexes and +1 in upper extremities. Relevant past medical history includes past polysubstance abuse, intravenous heroin, and marijuana, with the last use being 25 years ago. A chest X-ray showed hilar adenopathy. A lumbar puncture was performed, and cerebrospinal fluid studies showed elevated protein. Serological studies were positive for anti-ganglioside antibodies. Pulse oximetry showed a saturation of 84% on room air. An ABG was performed and showed a PaO2 of 50 mmHg and a PaCO2 of 52 mmHg. Which of the following findings will predict poor prognosis in this patient?

Choices: 1. Presence of autoantibodies against ganglioside proteins 2. Hilar adenopathy on chest X-ray 3. Multiple focal cranial nerve deficits on exam 4. Evidence of hypoxemia on room air

Answer: 4 - Evidence of hypoxemia on room air Explanations: This clinical presentation is suspicious for Miller Fisher syndrome (MFS), a rare variant of Guillain Barre syndrome (GBS). The classical triad of symptoms includes acute ophthalmoplegia, areflexia, and ataxia. Distal lower extremity weakness, pain, and paresthesias may be present. Other physical exam findings include diplopia, cranial nerve palsies, and dampened corneal reflex. However, clinical symptoms by themselves do not predict the prognosis of this condition. The presence of elevated cerebrospinal fluid (CSF) protein with normal findings in the rest of CSF studies should be suspicious for GBS. Anti-ganglioside antibody is a specific finding for Miller Fisher syndrome, brainstem encephalitis, or other forms of GBS, including a pharyngeal-cervical-brachial weakness. In the setting of hyporeflexia and lack of oropharyngeal dysphagia, Miller Fisher is the most likely diagnosis. However, serology is not used as a predictor of mortality and morbidity, rather than a specific finding to confirm the diagnosis of MFS. Multifocal deficits in MFS have been reported in the literature, not surprising given the pathophysiology of this condition, including the targeting of the myelin sheaths of the central and peripheral nervous system. Although this phenomenon is known in many cases of MFS, it is not a predictive factor of mortality or morbidity in patients. The most worrisome finding in a patient more than 50 years of age regarding prognosis is the presence of hypoxemia on room air, as it is a risk factor for impending respiratory failure in patients. Additionally, this finding is a major criterion for intensive care unit admission in adults. Mechanical ventilation and ICU admission are recommended in patients with at least one major criterion or 2 minor criteria. Major criteria include hypercapnia with PaCO2 above 48 mm Hg, hypoxemia with PaO2 below 56 mm Hg while the patient is breathing ambient air, vital capacity less than 15 mL/kg of body weight, and negative inspiratory force less than -30 cm H2O. Minor criteria include an inefficient cough, impaired swallowing, and atelectasis. Life-threatening complications are more likely to occur in patients who meet the criteria for intensive care unit admission. These complications include sepsis, pneumonia, pulmonary embolism, autonomic dysfunction, and gastrointestinal bleeding. The risk of mortality or morbidity is higher in these patients. Among severely affected patients, 20 to 33% may be unable to walk for more than six months after symptom onset, especially if infected with Campylobacter jejuni. Patients may also suffer from chronic psychiatric illness due to persistent pain and disability, among other complications. Go to the next page if you knew the correct answer, or click the link image(s) below to further research the concepts in this question (if desired).

Research Concepts: Miller Fisher Syndrome

We update eBooks quarterly and Apps daily based on user feedback. Please tap flag to report any questions that need improvement.

Question 23: A 50-year-old male with a history of chronic obstructive pulmonary disease (COPD) was admitted for acute hypercapnic and hypoxic respiratory failure secondary to COPD exacerbation. He developed severe acidosis from hypercapnia and required immediate intubation and admission to the intensive care unit. On the second day of hospitalization, he was improving, and plans were made to start a weaning trial the next morning. Overnight the ventilator starts sounding alarms. When the clinician arrives, he sees that the ventilator shows a peak pressure of 58 and a plateau pressure of 26. A physical exam reveals bilateral breath sounds and good air movement. There is no engorgement of the jugular veins, and the blood pressure is within normal limits. What are the most likely diagnosis and the next step in management?

Choices: 1. Acute mucous plugging or kink in the airway; suction the patient and assess airway 2. Worsening COPD; give a pulse dose of corticosteroids and start continuous nebulizer treatment 3. Tension pneumothorax; place a needle in the second intercostal space followed by a chest tube 4. Air trapping secondary to auto-PEEP; adjust the I:E (inspiratory:expiratory) ratio

Answer: 1 - Acute mucous plugging or kink in the airway; suction the patient and assess airway

Explanations: An increase in the peak pressure without change in plateau pressures is indicative of increased airway pressures, which is likely secondary to a mucous plug, the patient biting down on the tube, or the tube being kinked. Initial management is to evaluate the airway and suction the patient for possible mucous plug(s). Worsening chronic obstructive pulmonary disease would likely present as decreased breath sounds bilaterally and poor air movement. Also, a synchronous increase in plateau pressure would be expected. Tension pneumothorax would present with hypotension, hypoxia, and increased plateau pressure. One would expect for the patient to have decreased breath sounds unilaterally and to have jugular venous distention. Auto-PEEP with air trapping causes an increase in both peak and plateau pressures. Patients frequently seem to be dyssynchronous with the ventilator. Go to the next page if you knew the correct answer, or click the link image(s) below to further research the concepts in this question (if desired).

Research Concepts: Ventilation Assist Control

We update eBooks quarterly and Apps daily based on user feedback. Please tap flag to report any questions that need improvement.

Question 24: A 45-year-old female with a past medical history of factor V, hypertension, hysterectomy, and a current smoker who presents to the emergency department with complaints of diffuse abdominal pain that started five days ago. She states that she is supposed to be taking apixaban due to her history of blood clots in the past but stopped taking it ten days ago. She denies any associated fever, chills, vomiting or diarrhea. Physical examination reveals a nondistended, soft abdomen with diffuse abdominal tenderness without guarding or rebound. Abdominal ultrasound reveals a thrombus within the inferior vena cava. The patient is started on IV heparin. What management is recommended at this time?

Choices: 1. No further management, continue heparin and admit 2. Call vascular surgery for catheter-directed thrombolysis or thrombectomy 3. Call vascular surgery for percutaneous transluminal angioplasty with stenting 4. Discontinue heparin and start warfarin

Answer: 2 - Call vascular surgery for catheter-directed thrombolysis or thrombectomy Explanations: Caval thrombi are at increased risk for lethal pulmonary emboli and can extend to the renal veins. Therefore aggressive treatment is recommended for acute development. Catheterdirected therapy has been shown to have an 85% early success rate. Acute clots respond more favorably to catheter-directed thrombolysis and thrombectomy than chronic thrombus. The type of thrombolysis chosen varies, but typically most common is tPA or urokinase. Some cases where there is an extensive clot mechanical thrombectomy is used in conjunction with thrombolysis, which has been shown to have greater success with removal of the residual clot. There are multiple mechanical thrombectomy devices that can be used. One commonly used is pharmacomechanical peripheral thrombectomy device which consists of high velocity and pressure saline jet to dissolve and aspirate the clot. Catheter-directed thrombolysis delivers the thrombolytic directly within the vessel at the thrombus which may be a safe alternative compared to systemic thrombolysis. It is typically performed from a femoral or popliteal approach, depending on the extent of the clot. There is a catheter with multiple holes for infusion, it is carefully positioned within the thrombus, and the thrombolytic medication is infused when the catheter is in the proper position. Lytic therapies may require longer lysis times, meaning that normally following direct thrombolysis the clot is reevaluated in 12 to 24 hours with venography and if needed the direct thrombolysis can be performed again. Normally this can occur due to the size of the thrombus originally. An extensive thrombus may require multiple treatments. Go to the next page if you knew the correct answer, or click the link image(s) below to further research the concepts in this question (if desired).

Research Concepts: Inferior Vena Caval Thrombosis

We update eBooks quarterly and Apps daily based on user feedback. Please tap flag to report any questions that need improvement.

Question 25: A 56-year-old man with congestive heart failure presents to the office for his regular blood transfusion for beta-thalassemia intermedia. His vital signs are significant only for a pulse of 96/min. He has had well-controlled diabetes mellitus type 2 for the last 12 years. He weighs 180 lbs (82 kg) and appears in no apparent distress. Which of the following is the most appropriate rate of transfusion for this patient?

Choices: 1. 2 mL/kg per hour 2. 7 mL/kg per hour 3. 10 mL/kg per hour 4. 15 mL/kg per hour

Answer: 1 - 2 mL/kg per hour Explanations: The recommended rate of transfusion for thalassemia patients is 5 mL/kg per hour, and post-transfusion hemoglobin should not exceed 14 g/dL. But for patients with severe anemia (Hb less than 5 g/dL) or cardiac compromise, the transfusion rate should be reduced to 2 mL/kg per hour to avoid fluid overload. Patients with cardiac comprise are prone to developing fluid overload states. In such patients, care must be taken in transfusing blood or any fluids. Diuretics can be used to control a fluid overload state. Transfuisng at higher rates (more than 5 mL/kg per hour) in this patient is risky and not recommended. Transfusing at 10-15 mL/kg per hour will put this patient's heart under extreme stress due to sudden volume overload. Go to the next page if you knew the correct answer, or click the link image(s) below to further research the concepts in this question (if desired).

Research Concepts: Thalassemia

We update eBooks quarterly and Apps daily based on user feedback. Please tap flag to report any questions that need improvement.

Question 26: A female was positioned in the dorsal lithotomy position and underwent a hysterectomy which was marred by bleeding and adhesions. The duration of the surgery was 6 hours, and because of the patient's age and comorbidity, she was extubated and monitored in the intensive care unit. Within 6 hours of surgery, she complained of her right leg pain. Her blood pressure was 95/65 mmHg, pulse 110 bpm, respiratory rate 18, and temperature 98.7 F. The leg did not show any ecchymosis but was slightly swollen. Blood work revealed a potassium of 5.9 mmol/L, creatinine of 2.2 mg/dL, WBC 14 10^9/L, hemoglobin 8 g/dL, sodium 133 mmol/L, chloride 90 mmol/L, BUN 22 mg/dL, blood glucose 95 mg/dl, creatine kinase of 33,000 units/L and myoglobinuria. What is the most important step in her management?

Choices: 1. Administer intravenous fluids and blood stat 2. Order a renal consult 3. Duplex ultrasound of the leg 4. Orthopedic surgery consult

Answer: 4 - Orthopedic surgery consult Explanations: Acute compartment syndrome is a real complication when patients are placed for prolonged periods in the lithotomy position. It is stated that when procedures are more than 4 hours, the risk of compartment syndrome greatly increases. The diagnosis of acute compartment syndrome is chiefly clinical. No one can agree on compartment pressures and how to measure them. It is known that the leg does become tense, and the pressure can be increased, but the key is where to measure the pressure as it may not be elevated at all locations. This patient has myoglobinuria and elevated creatine kinase, which indicates muscle breakdown. This, together with the pain and leg position, is suspicious for compartment syndrome. While one can rule out a deep vein thrombus, more important is to call an orthopedic surgeon to assess the patient for a bedside fasciotomy. Any delay in management and irreversible injury can occur to the leg, leading to an amputation. The patient should also be treated with intravenous fluids, and a renal consult would be appropriate. Go to the next page if you knew the correct answer, or click the link image(s) below to further research the concepts in this question (if desired).

Research Concepts: Acute Compartment Syndrome

We update eBooks quarterly and Apps daily based on user feedback. Please tap flag to report any questions that need improvement.

Question 27: A 39-year-old female was admitted to the hospital after she sustained extensive third-degree burn to her lower limbs and abdomen 4 days ago. She developed acute kidney injury and worsening respiratory distress that required mechanical ventilation. Chest radiograph showed bilateral pulmonary infiltrates. Respiratory rate is set at 14 breaths per minute, but she is breathing at a rate of 28 breaths per minute. Initial arterial blood gas showed ph 7.55, PaCO2 28 and PaO2 55 on 70% FiO2, positive end-expiratory pressure (PEEP) of 5 cm water, and a tidal volume of 10 ml per kg of ideal body weight. What is the next step in the management of this patient?

Choices: 1. Decrease respiratory rate to 10 and increase FiO2 to 100% 2. Start deep sedation and neuromuscular blockade, increase the PEEP to 10 and set the tidal volume to 6 ml per kg 3. Increase FiO2 to 100% and repeat ABGs in one hour 4. Sedate the patient deeply and reassess after 30 minutes

Answer: 2 - Start deep sedation and neuromuscular blockade, increase the PEEP to 10 and set the tidal volume to 6 ml per kg

Explanations: The patient meets the criteria for acute respiratory distress syndrome (ARDS) as she has developed acute respiratory symptoms within one week of clinical insult (4 days after severe, extensive burn) with radiographic features of bilateral patchy infiltrates and hypoxemia (PaO2/FiO2 ratio 78.5) that are not fully explained by cardiogenic pulmonary edema. The patient should be started on low tidal volume ventilation with a target plateau pressure less than 30 mmHg, a ventilation strategy that is associated with decreased mortality and improved clinical outcomes. Deep sedation and neuromuscular blockade are used in the early stage of severe ARDS to ensure safe, low-pressure ventilation and to avoid further lung injury especially in patients who are hard to control and are not synchronizing with the mechanical ventilation. Increasing FiO2 alone without increasing the PEEP is not enough in this case as higher positive end-expiratory pressure (PEEP) is recommended to prevent alveolar collapse (atelectrauma) and increase the tidal volume participating in the gas exchange. Furthermore, decreasing respiratory volume or deeply sedating patients are not appropriate in this patient with severe ARDS. Go to the next page if you knew the correct answer, or click the link image(s) below to further research the concepts in this question (if desired).

Research Concepts: Respiratory Distress Syndrome

We update eBooks quarterly and Apps daily based on user feedback. Please tap flag to report any questions that need improvement.

Question 28: A 65-year-old male with a history of alcoholic cirrhosis presents to the hospital with abdominal pain. His physical exam is significant for ascites. He also reports a decrease in his urine output for the past 4-5 days. His vitals reveal a blood pressure of 94/62 mmHg and pulse 89/min. Labs are significant for serum creatinine 2.7 mg/dL (from normal baseline one week ago). Urine analysis is negative for casts or proteinuria. Renal ultrasound is negative for hydronephrosis. Paracentesis is suggestive of spontaneous bacterial peritonitis, and he is started on ceftriaxone. He receives 2 L of normal saline and over 200 g of albumin in the next 48 hours without improvement of his serum creatinine or urine output. What is the most likely etiology of his kidney injury?

Choices: 1. Dehydration 2. Type 1 hepatorenal syndrome 3. Type 2 hepatorenal syndrome 4. Ischemic acute tubular necrosis

Answer: 2 - Type 1 hepatorenal syndrome Explanations: This patient likely has type 1 hepatorenal syndrome which is characterized by a rapid and progressive decline in renal function defined by doubling of the serum creatinine to at least more than 2.5 mg/dL or a decrease in the creatinine clearance by half or more over two weeks. This is associated with a urine output of less than 500 mL/day. It usually has a bland urine sediment with no or minimal proteinuria. Most patients with cirrhosis who present with an acute kidney injury are dehydrated and require stopping the diuretics and hydration. If the kidney function does not improve after at least 48 hours of volume resuscitation with albumin (1g/kg per day) and stopping the diuretics; given other etiologies of kidney injury are ruled out, the diagnosis of hepatorenal syndrome is made. Type 2 typically involves less severe kidney injury, and patients ordinarily present with diuretic-resistant ascites. This patient received fluid resuscitation without improvement ruling out dehydration. His urine analysis was bland without any casts or proteinuria making acute tubular necrosis less likely. Go to the next page if you knew the correct answer, or click the link image(s) below to further research the concepts in this question (if desired).

Research Concepts: Hepatorenal Syndrome

We update eBooks quarterly and Apps daily based on user feedback. Please tap flag to report any questions that need improvement.

Question 29: A 78-year-old thin female is undergoing an operation for insertion of a tunneled central venous catheter with a subcutaneous port. Initial venous access is unsuccessful after two attempts at left subclavian vein access. On the third attempt, the anesthesiologist states that the patient's oxygen saturation has acutely dropped to 88%. Heart rate is 101 bpm, and blood pressure is 101/83 mmHg. Which of the following is the next best step in the management of this patient?

Choices: 1. Switch to left internal jugular vein access as the patient is not tolerating left subclavian intervention 2. Abort the procedure and obtain a chest x-ray and place a thoracostomy tube if pneumothorax is present 3. Position the patient in the left lateral decubitus position with Trendelenberg 4. Intubate, start chest compressions and obtain an automated external defibrillator

Answer: 2 - Abort the procedure and obtain a chest x-ray and place a thoracostomy tube if pneumothorax is present

Explanations: Pneumothorax is a complication of both subclavian and internal jugular venous access and should be discussed as a risk while obtaining surgical consent. If there is a concern for an air embolism, immediately position the patient in the left lateral decubitus position and Trendelenberg to prevent the air bubble from traveling to the pulmonary arteries. If venous access is present, apply negative pressure via syringe to try to retrieve and suck the air bubble out. The subclavian vein in this patient was not successfully cannulated, so air embolism is unlikely. Although ensuring an airway is an essential first step, chest compressions are not indicated in a patient with a pulse. An internal jugular vein is an option for tunneled central venous access in case of unsuccessful subclavian cannulation. However, if there is a concern for pneumothorax, the procedure should be aborted, and an immediate chest x-ray should be done. Subclavian vein cannulation has a higher risk of pneumothorax as compared to the internal jugular vein. Go to the next page if you knew the correct answer, or click the link image(s) below to further research the concepts in this question (if desired).

Research Concepts: Vascular Tunneled Central Catheter Access

We update eBooks quarterly and Apps daily based on user feedback. Please tap flag to report any questions that need improvement.

Question 30: A middle-aged obese male presents with sudden onset of excruciating chest pain that is now radiating into his back. He has woken up from his sleep 3 hours ago. In the emergency department, he is diaphoretic, anxious, and complaining of chest pain. He is administered nitroglycerin by the triage nurse but this does not provide any relief. Blood pressure on the right arm is 190/90 mmHg and on the left arm, it is 145/75 mmHg. His respiration rate is 28/minute and he is afebrile. His initial set of cardiac enzymes is negative and the ECG reveals non-specific ST changes. In this patient what will most likely hear be heard during auscultation?

Choices: 1. Crescendo-decrescendo systolic murmur 2. Loud P2 3. Systolic ejection click 4. High pitched early diastolic murmur

Answer: 4 - High pitched early diastolic murmur Explanations: Aortic dissection is a serious disorder that can be rapidly fatal if the diagnosis is missed. The patient may present with excruciating pain that is often unresponsive to nitroglycerin. The physical may reveal unequal pulses, bounding pulses, diastolic murmur, abdominal pain, anxiety, fever, or neurological deficit. If the aortic dissection progresses retrograde it can tear into the aortic annulus and cause aortic regurgitation. While the initial chest x-ray is often normal, one may see signs of acute heart failure or elevated jugular venous pressure in some patients. Go to the next page if you knew the correct answer, or click the link image(s) below to further research the concepts in this question (if desired).

Research Concepts: Aortic Dissection

We update eBooks quarterly and Apps daily based on user feedback. Please tap flag to report any questions that need improvement.

Question 31: A 45-year-old healthy female patient presents to the emergency department after a snake bite from a coral snake 4 hours before arrival. The patient is currently asymptomatic, vital signs are all within normal limits and her physical exam is normal. She is requesting to be discharged home. What is the best recommendation?

Choices: 1. Explain to the patient that she is safe for discharge home at this time, any significant effects should have been noticed by now 2. Suggest that she enjoy some cheese, crackers, and a soda while being observed in the emergency department for an additional 2 hours before being discharged home 3. Proceed immediately to endotracheal intubation and treatment with North American Coral Snake Antivenin (NACSA) 4. Recommend hospital admission as respiratory failure and neurotoxic effects may be delayed up to 13 hours in coral snake bites

Answer: 4 - Recommend hospital admission as respiratory failure and neurotoxic effects may be delayed up to 13 hours in coral snake bites

Explanations: Coral snake bites may cause delayed systemic neurotoxicity up to 13 hours after the initial bite. Patients bitten by a coral snake should be admitted and observed in an inpatient setting for any development of respiratory muscle weakness. Observation in the emergency department only is not recommended for coral snake bites. Pre-emptive intuition is not required for all patients with coral snake bites. Patients who develop symptoms of respiratory weakness and or failure may require additional support via intubation and mechanical ventilation, but not all patients will develop these symptoms. Neurotoxic effects of coral snake envenomation may develop gradually, starting with mild weakness or sensory deficit that eventually progresses to full paralysis. Go to the next page if you knew the correct answer, or click the link image(s) below to further research the concepts in this question (if desired).

Research Concepts: Coral Snake Toxicity

We update eBooks quarterly and Apps daily based on user feedback. Please tap flag to report any questions that need improvement.

Question 32: A 35-year-old previously healthy female has had headaches for a few days and her family reports behavior changes. She has been more talkative, outgoing, and flirtatious with strangers. After she was sexually inappropriate with a store clerk her family brought her to her healthcare provider. The only finding was a low-grade fever. MRI was ordered urgently and showed an inferomedial temporal lobe lesion. Lumbar puncture is done. What treatment is indicated?

Choices: 1. Corticosteroids 2. Ceftriaxone and vancomycin 3. Acyclovir 4. Antipsychotic medication

Answer: 3 - Acyclovir Explanations: The patient has viral encephalitis most likely secondary to herpes simplex virus type 1. The lumbar puncture polymerase chain reaction (PCR) will most likely be positive but treatment with acyclovir should be started before the results are available. The patient should be monitored for seizures and increased intracranial pressure. Even with early treatment, a significant number of patients have residual neurologic deficits that are significant. Go to the next page if you knew the correct answer, or click the link image(s) below to further research the concepts in this question (if desired).

Research Concepts: Viral Encephalitis

We update eBooks quarterly and Apps daily based on user feedback. Please tap flag to report any questions that need improvement.

Question 33: A 75-year-old male is brought to the emergency department with confusion. Two months ago he presented with acute right-sided weakness and new-onset atrial fibrillation. He was diagnosed with a cerebrovascular accident and started on warfarin. CT head obtained in the emergency room today shows intracranial hemorrhage. INR is 4.0. Which of the following is the next best step in management?

Choices: 1. Administer four-factor prothrombin complex concentrate and vitamin K 2. Administer fresh frozen plasma (FFP) and vitamin K 3. Administer vitamin K alone 4. Administer FFP alone

Answer: 2 - Administer fresh frozen plasma (FFP) and vitamin K Explanations: This patient with warfarin-associated intracranial hemorrhage needs the reversal of anticoagulation. PCC is the first choice in patients with warfarin-associated major bleed. However, there is risk of prothrombotic complications with PCC. Patients with history of myocardial infarction, stroke or transient ischemic attack, disseminated intravascular coagulation within the last three months were excluded from clinical trials of PCC. FFP with vitamin K is preferred since he had a stroke within the last three months. Vitamin K should be coadministered to sustain the reversal of anticoagulation. Go to the next page if you knew the correct answer, or click the link image(s) below to further research the concepts in this question (if desired).

Research Concepts: Warfarin Toxicity

We update eBooks quarterly and Apps daily based on user feedback. Please tap flag to report any questions that need improvement.

Question 34: A 65-year-old male with COPD is in the emergency department with dyspnea, productive cough, and fatigue. Vital signs show a heart rate of 124/min, blood pressure 158/94 mmHg, respiratory rate 32/min, and SpO2 90% on 6 L nasal cannula. Physical examination is notable for diminished breath sounds bilaterally with scant end-expiratory wheezes. He is given non-invasive ventilation via a full face for two hours. Bronchodilators, antibiotics, and steroids are prescribed. Current ventilator settings are PSV mode, IPAP 20 cmH2O, EPAP 8 cmH2O, FiO2 50%. Now vital signs are heart rate 130/min, blood pressure 160/96 mmHg, respiratory rate 30/min, and SpO2 94%. He is alert and answers questions but appears in moderate distress and complains of dyspnea. Tidal volumes returned to the ventilator measure between 250 and 350 mL. ABG shows pH 7.25, pCO2 68 mmHg, and pO2 65 mmHg. What is the next best step in the management of this patient?

Choices: 1. Increase EPAP to 10 cmH2O and FiO2 to 75%; keep IPAP at 20 cmH2O 2. Perform endotracheal intubation and set the ventilator to a volume control mode 3. Perform endotracheal intubation and set the ventilator to a PSV mode 4. Increase IPAP to 28 cmH20 and FiO2 to 75%; keep EPAP at 8 cmH2O

Answer: 2 - Perform endotracheal intubation and set the ventilator to a volume control mode Explanations: PSV is not used as an initial mode of ventilation for intubated patients due to respiratory depression following sedation given during intubation. High airway resistance in patients with obstructive lung disease limits peak flow and can result in small tidal volumes. Work of breathing and thus oxygen consumption is higher in PSV than in control modes of ventilation. Tidal volumes in non-invasive PSV ventilation can be increased by increasing drive pressure, but this may be limited by high airway resistance, mask leak, or air-trapping. Go to the next page if you knew the correct answer, or click the link image(s) below to further research the concepts in this question (if desired).

Research Concepts: Pressure Support Ventilation

We update eBooks quarterly and Apps daily based on user feedback. Please tap flag to report any questions that need improvement.

Question 35: A 59-year-old obese male presents to the emergency department with a 4-hour history of sudden onset of sharp radiating chest pain. The pain he says started in the chest and is now radiating to his arm and lower back. He feels nauseated and feels as if he is going to die. Quick exam reveals a distressed male with right arm blood pressure of 210/110 mmHg and a left arm blood pressure of 140/82 mmHg. He is diaphoretic and extremely anxious. The ECG is unremarkable and the initial set of cardiac enzymes is negative. His pain is not responding to nitroglycerin. In patients with this pathology, what one feature will be seen on the chest x-ray?

Choices: 1. Enlarged aortic knob 2. Displacement of trachea to the right 3. Calcified aortic knob 4. Double aortic contour

Answer: 4 - Double aortic contour Explanations: The chest x-ray in patients with aortic dissection can be normal or show a number of features of which the most common is a widened mediastinum. Other features include a double or irregular aortic contour. There can be an inward displacement of the atherosclerotic calcification. If the patient is stable, a CT scan of the chest is the next study of choice. Go to the next page if you knew the correct answer, or click the link image(s) below to further research the concepts in this question (if desired).

Research Concepts: Aortic Dissection

We update eBooks quarterly and Apps daily based on user feedback. Please tap flag to report any questions that need improvement.

Question 36: A 45-year-old male comes to the emergency department in August complaining of dizziness, shortness of breath and palpitations. He reports having developed a red rash with central clearing and successive flu-like symptoms one month ago after having gone hiking with a group of friends in Pennsylvania. He took over the counter acetaminophen and symptoms subsided after five days with no other therapy. Physical examination is unremarkable except for a heart rate of 35 beats per minute. An electrocardiogram showed dissociation between P waves and QRS complex. What is the next step in the management of this patient?

Choices: 1. Admission, initiate treatment with oral doxycycline and telemetry monitoring 2. Admission, initiate treatment with intravenous ceftriaxone and telemetry monitoring 3. Admission, initiate treatment with intravenous ceftriaxone and temporary pacemaker placement 4. Admission, initiate treatment with intravenous ceftriaxone and permanent pacemaker placement

Answer: 3 - Admission, initiate treatment with intravenous ceftriaxone and temporary pacemaker placement

Explanations: Pacemaker placement is indicated in patients with Lyme carditis and high second degree or third-degree atrioventricular (AV) block. Intravenous antibiotics are preferred for moderate to severe disease. Need for permanent pacemaker has rarely been reported in patients with Lyme carditis and atrioventricular block. Most of the time atrioventricular block resolves after some days of treatment with antibiotics. It is safe to remove pacemaker once second or third degree AV block has resolved. Go to the next page if you knew the correct answer, or click the link image(s) below to further research the concepts in this question (if desired).

Research Concepts: Lyme Carditis

We update eBooks quarterly and Apps daily based on user feedback. Please tap flag to report any questions that need improvement.

Question 37: A 75-year male presents to the emergency department with left facial, arm, and leg numbness and tingling. He has a past medical history significant for type 2 diabetes mellitus, essential hypertension, prostate cancer, cataracts, and two prior myocardial infarctions 5 and 8 years ago. He has smoked two packs of cigarettes daily for the past 40 years. He takes metformin, atorvastatin, lisinopril, tamsulosin and undergoes radiation therapy every two weeks. He has been compliant with his medications and check-ups. His vital signs are 38 C, blood pressure 140/85 mmHg, respiratory rate 12/minute, and 98% oxygen saturation on 2 L nasal cannula oxygen. The patient is alert and oriented to person, place, and time. He can draw a clock fully with the time. The neurologic exam is significant for decreased pinprick, fine touch, and vibration sensation in his left face, arm, and leg. Visual fields are full bilaterally. Strength is 5/5 throughout, and reflexes are 1+ bilaterally in his upper and lower extremities. His cardiac exam shows a mild 1/6 systolic murmur in the right upper sternal border, and his pulmonary exam shows an increased inspiratory and expiratory effort with bilateral expiratory wheezes in all lung fields. Head CT scan is negative. Chest x-ray shows a flattened diaphragm and expanded lung fields bilaterally. Where is the location of his lacunar syndrome and what is the artery infarcted?

Choices: 1. Right parietal lobe and middle cerebral artery 2. Left internal capsule and lenticulostriate artery 3. Right pons and perforating pontine arteries 4. Right thalamus and the thalamoperforant arteries

Answer: 4 - Right thalamus and the thalamoperforant arteries Explanations: The anatomic distribution of lacunar syndromes and infarctions is most commonly the basal ganglia, the pons, and the subcortical white matter structures. These anatomical sites correspond to lesions at the lenticulostriate arteries, the anterior choroidal artery, thalamoperforant arteries, paramedian branches of the basilar artery, and the recurrent artery of Heubner from the anterior cerebral artery. In a pure sensory stroke, the patient presents with unilateral numbness of the face, arm, and leg without cortical signs or motor deficits. All sensory modalities will be impaired. A lesion at the contralateral thalamus, supplied by the thalamoperforant arteries, is the most common cause of a pure sensory stroke. Go to the next page if you knew the correct answer, or click the link image(s) below to further research the concepts in this question (if desired).

Research Concepts: Lacunar Syndromes

We update eBooks quarterly and Apps daily based on user feedback. Please tap flag to report any questions that need improvement.

Question 38: A 65-year-old man is scheduled to undergo a radical prostatectomy for prostate adenocarcinoma. During surgery, there is significant bleeding that becomes difficult to control. Immediate blood work reveals hemoglobin of 9 g/dL, platelets of 200,000/mm3, prothrombin time of 11.2 seconds, activated partial thromboplastin time of 41 seconds, and elevated levels of fibrinogen degradation products. The patient was started on intravenous aminocaproic acid to control the bleeding. Which of the following best describes the mechanism of action of aminocaproic acid?

Choices: 1. Accelerates the conversion of prothrombin to thrombin 2. Binds to antithrombin and inactivates thrombin and factor Xa 3. Blocks the conversion of plasminogen to plasmin 4. Accelerates the conversion of fibrinogen to fibrin

Answer: 3 - Blocks the conversion of plasminogen to plasmin Explanations: The formation of a blood clot, also known as coagulation, starts immediately after injury to a blood vessel occurs. Platelets are activated and form a platelet plug at the site of the blood vessel injury. The platelet plug is further stabilized by a fibrin mesh, a process that is facilitated by proteins called coagulation factors. Stabilization of the platelet plug with a fibrin mesh seals the site of blood vessel injury and stops the bleeding. Plasminogen is an enzyme in the blood that gets converted into plasmin by tissue plasminogen activator. Plasmin then works to degrade the fibrin mesh stabilizing the platelet plug, releasing fibrin degradation products into the blood. Degradation of the fibrin mesh will disrupt the platelet plug and lead to bleeding. Aminocaproic acid stops bleeding by inhibiting the conversion of plasminogen into plasmin, thus preventing degradation of the fibrin mesh stabilizing the platelet plug. On the contrary, thrombolytics, such as alteplase, works by activating tissue plasminogen activator (tPA), which will convert plasminogen into plasmin, which will degrade the fibrin mesh and lead to bleeding. Go to the next page if you knew the correct answer, or click the link image(s) below to further research the concepts in this question (if desired).

Research Concepts: Aminocaproic Acid

We update eBooks quarterly and Apps daily based on user feedback. Please tap flag to report any questions that need improvement.

Question 39: A 70-year-old male, with left community-acquired pneumonia and left pleural effusion being treated with ceftriaxone and azithromycin, undergoes diagnostic thoracentesis under ultrasound guidance for his left-sided pleural effusion. The pleural fluid is turbid, but no frank pus is seen. His pleural fluid pH is less than 7.2, with pleural glucose concentration normal. Which of the following interventions is most appropriate for this patient?

Choices: 1. Continue current antibiotic regimen 2. Broaden antibiotic regimen 3. Perform tube thoracostomy and switch azithromycin to metronidazole 4. Perform tube thoracostomy and inject intrapleural fibrinolytics

Answer: 3 - Perform tube thoracostomy and switch azithromycin to metronidazole Explanations: Parapneumonic effusions require diagnostic thoracocentesis to further evaluate its etiology and to guide therapeutic strategies. Aspiration of frank pus from the pleural space during thoracocentesis invariably needs surgical drainage. However, if there is uncertainty about whether a turbid fluid is infected, a pH of less than 7.2 measured via a blood gas analyzer warrants an invasive procedure for drainage. Anaerobic coverage should be initiated in any patient with suspected empyema, and appropriate antibiotics should not be delayed pending culture data. Failure to treat empyema appropriately with urgent drainage and adequate antibiotic coverage can lead to poor patient outcomes. Go to the next page if you knew the correct answer, or click the link image(s) below to further research the concepts in this question (if desired).

Research Concepts: Thoracic Empyema

We update eBooks quarterly and Apps daily based on user feedback. Please tap flag to report any questions that need improvement.

Question 40: A 75-year-old male is diagnosed with multilobar pneumonia and is intubated in the emergency department. A ventilator is placed on assist control mode with respiratory rate of 22, tidal volume of 6 ml/kg, FIO2 of 24%, and PEEP of 10 cmH2O. An arterial blood gas shows pH=7.21, PCO2=34 mmHg, and PO2=56 mmHg. Which of the following is the most likely cause of the continued hypoxemia?

Choices: 1. Shunting 2. Hypoventilation 3. Heart failure 4. Ventilation-perfusion mismatch

Answer: 1 - Shunting Explanations: The patient has shunting due to alveoli that receive perfusion but cannot exchange gas. A ventilation-perfusion mismatch most commonly occurs with pulmonary embolism. The low PCO2 indicates that there is not hypoventilation. The patient is acidotic not from respiratory sources but rather metabolic. Go to the next page if you knew the correct answer, or click the link image(s) below to further research the concepts in this question (if desired).

Research Concepts: Bacterial Pneumonia

We update eBooks quarterly and Apps daily based on user feedback. Please tap flag to report any questions that need improvement.

Question 41: A 62-year-old patient has an acute anterior myocardial infarction and develops a new holosystolic murmur and dyspnea. Vital signs are blood pressure 90/52 mmHg, heart rate 120 beats/min, and respiratory rate 20. Exam shows jugular venous distention, rales at the lung bases, and cool extremities with edema. What is the best initial test?

Choices: 1. Chest x-ray 2. Echocardiography 3. Left and right cardiac catheterization 4. CT of the heart

Answer: 2 - Echocardiography Explanations: Echocardiography will most likely detect either papillary muscle or ventricular septal rupture. This can be done urgently at the bedside. A chest x-ray is not helpful in finding the exact cause of congestive heart failure in a postmyocardial infarction patient who has developed a mechanical complication. Cardiac catheterization with coronary angiography will aid in diagnosis and management but is not the first test for making the diagnosis. Echocardiography remains the initial test of choice. Go to the next page if you knew the correct answer, or click the link image(s) below to further research the concepts in this question (if desired).

Research Concepts: Acute Myocardial Infarction

We update eBooks quarterly and Apps daily based on user feedback. Please tap flag to report any questions that need improvement.

Question 42: A 33-year-old African-American female with a history of intravenous drug use and alcohol use presented to the emergency department with eye discharge, painful eye movement, eye swelling on the right side and double vision for two days. The patient reported having a fall two days ago, but she was not able to remember the details. Vitals showed blood pressure 160/95 mmHg, heart rate 105 bpm, and temperature 37.5 C. She is alert and oriented x3. There is chemosis, periorbital edema, proptosis, mild restriction of all extraocular movements of the right eye and mild drooping of the right eyelid. The visual acuity was 20/20 normal on both eyes. A bruit was noticed over the right orbit. Pupillary reflexes were normal. Fundus exam showed mildly dilated vessels but no papilledema. There was a small bruise on the right side of the face on the maxillary area. The rest of the exam was unremarkable. Basic blood workup came back unremarkable. CT head without contrast showed a small maxillary fracture, proptosis, and orbital edema on the right side. What are the most likely diagnosis and the best evaluation for the definitive diagnosis?

Choices: 1. Cavernous sinus tumor so perform a biopsy 2. Cavernous sinus thrombosis so there is no need for further tests. Start stat treatment with broad-spectrum IV antibiotics including antifungal and anticoagulation 3. Carotid cavernous aneurysm so check intraocular pressure from right eye STAT 4. Carotid cavernous fistula so order conventional digital subtraction angiography

Answer: 4 - Carotid cavernous fistula so order conventional digital subtraction angiography Explanations: The most common symptoms on carotid-cavernous fistula (CCF) are ocular bruit auscultated over the globe in high-flow CCFs, proptosis, chemosis, and conjunctival injection, ocular and/or orbital pain, headache, diplopia, and blurry vision. Vascular etiologies can be seen on CTA, MRI, MRA, and orbital or transcranial ultrasound but conventional digital subtraction angiography is the gold-standard test for diagnosis of CCF. CCFs may be an indirect (low-flow), or a direct (high-flow) CCFs based on a connection between the cavernous sinus and the intracavernous carotid artery versus the internal or external carotid artery, respectively. The management of CCFs depends on the classification of CCFs, the onset of symptoms and the risk of long-term neurological impairment. Twenty to sixty percent of indirect CCFs will close spontaneously. Direct CCFs should be closed by transarterial/transvenous embolization or other treatment options if they are symptomatic and at risk of progression with attendant morbidity. Go to the next page if you knew the correct answer, or click the link image(s) below to further research the concepts in this question (if desired).

Research Concepts: Cavernous Sinus Syndromes

We update eBooks quarterly and Apps daily based on user feedback. Please tap flag to report any questions that need improvement.

Question 43: A 52-year-old man with a history of migraines, cocaine abuse, schizophrenia, and hypertension is transferred to a tertiary care stroke center from a community hospital with a cerebellar stroke. He received tissue plasminogen activator at the community hospital with improvement in his symptoms while en route. On the third day of admission, he complains of a worsening headache associated with nausea and vomiting. He locates the aching, severe pain in the middle of his head. It is improved when can keep the lights in his hospital room off. He asks for you to give him diphenhydramine and metoclopramide, which he says always relieve his migraines at home. His blood pressure is 151/87 mmHg, and his heart rate is 87 bpm. An admission EKG had a QTc of 416 ms. He has horizontal gaze-evoked nystagmus. What should be done next?

Choices: 1. Order diphenhydramine and metoclopramide 2. Order ibuprofen and ondansetron 3. CT scan of the head 4. MRI of the head and neck

Answer: 3 - CT scan of the head Explanations: Given this patient’s recent diagnosis of a cerebellar stroke, his presentation is concerning for worsening of his infarct potentially due to hemorrhagic conversion. He is at higher risk for this having just received thrombolysis. This could be due to reactive edema or spreading infarct. Emergent CT imaging is indicated to rapidly assess any changes that may require neurosurgical intervention. MRI would be helpful to visualize the evolution of the infarct but is slow, and CT will give most of the immediately useful information it provides. Symptomatic care may be helpful, but a migraine in this patient would be a diagnosis of exclusion. Giving an NSAID like ibuprofen with some antiplatelet activity would be contraindicated. Go to the next page if you knew the correct answer, or click the link image(s) below to further research the concepts in this question (if desired).

Research Concepts: Cerebellar Infarct

We update eBooks quarterly and Apps daily based on user feedback. Please tap flag to report any questions that need improvement.

Question 44: A patient was brought in by her family for "being sleepy." During the interview, she is drowsy and reports diarrhea, vomiting, weakness and frequent urination. On exam, her hands shake mildly. Her medication includes HCTZ, verapamil, metformin, and lithium. What is the best treatment for this probable overdose?

Choices: 1. Hemodialysis 2. Activated charcoal 3. D-Penicillamine 4. Ipecac

Answer: 1 - Hemodialysis Explanations: Lithium toxicity is difficult to treat and may present with vomiting, diarrhea, drowsiness, muscular weakness, and lack of muscle coordination. When the lithium levels are high, it is important to discontinue the medication. The best way to deal with an overdose is hemodialysis. Use of diuretics or activated charcoal usually does not help. Ipecac should never be given as there is a risk of aspiration. Ipecac is also associated with a cardiomyopathy in children. Go to the next page if you knew the correct answer, or click the link image(s) below to further research the concepts in this question (if desired).

Research Concepts: Lithium Toxicity

We update eBooks quarterly and Apps daily based on user feedback. Please tap flag to report any questions that need improvement.

Question 45: A 17-year old female from Connecticut presents with a 7-day history of fever and chills. In the last 48 hours, she has also developed malaise, anorexia, and muscle pain. Her physical exam is unremarkable except for a few petechiae. She was hiking in the woods recently, where there were many ticks and insects. Testing for Lyme disease is negative, but the infectious disease expert feels that she may have been bitten by the deer tick "Ixodes dammini." Which other disease, besides Lyme, can be caused by this same tick?

Choices: 1. Babesiosis 2. Rocky mountain spotted fever 3. Typhoid fever 4. Q-fever

Answer: 1 - Babesiosis Explanations: Babesiosis is a zoonotic infection transmitted by the deer tick, Ixodes dammini. The Ixodes tick is much smaller than the tick that causes Rocky Mountain Spotted Fever. Babesiosis also occurs in the same geographical location as Lyme disease. It is often mistaken for malaria and somewhat difficult to diagnose. This is because the infection has no specific signs or symptoms. Go to the next page if you knew the correct answer, or click the link image(s) below to further research the concepts in this question (if desired).

Research Concepts: Babesiosis

We update eBooks quarterly and Apps daily based on user feedback. Please tap flag to report any questions that need improvement.

Question 46: A 17-year-old patient starts a new drug and now presents with fever, muscle rigidity, altered mental status, and signs of autonomic instability. Treatment is started with antipyretics, intravenous hydration, and dantrolene. What is the most likely drug involved in this adverse reaction?

Choices: 1. Chlorpromazine 2. Diazepam 3. Phenytoin 4. Levodopa

Answer: 1 - Chlorpromazine Explanations: Chlorpromazine, a neuroleptic drug, can cause neuroleptic malignant syndrome (NMS). NMS is a rare but potentially life-threatening reaction seen with the administration of certain drugs. NMS is most often associated with the administration of older neuroleptics, such as haloperidol and chlorpromazine. Less commonly, NMS can be precipitated by any of the antipsychotic agents and some nonneuroleptic drugs such as lithium, metoclopramide, and amoxapine. Although serious side effects can be seen with SSRIs, phenytoin, and levodopa, they are not associated with NMS, even at toxic levels. Go to the next page if you knew the correct answer, or click the link image(s) below to further research the concepts in this question (if desired).

Research Concepts: Neuroleptic Malignant Syndrome

We update eBooks quarterly and Apps daily based on user feedback. Please tap flag to report any questions that need improvement.

Question 47: A 34-year-old male presents to the emergency department following a burn injury sustained on chest and arms after accidental exposure to gas leaking vent at home. He does not have any facial injury but has 2nd and 3rd-degree burn injury involving the upper chest and both arms. Initial vitals show a temperature of 100 F, a heart rate of 100/min, respiratory rate of 22/min, and mean arterial pressure (MAP) of 70 mmHg. Initial chest x-ray shows basal atelectasis. Blood work shows acute leukocytosis with a WBC count of 16000/microL. He is admitted and started on the appropriate course of management. He continues to improve until day 3 when he spikes a fever again with increased shortness of breath and increased tan-colored sputum. Vitals now show a fever of 102 F with a heart rate of 120/min, respiratory rate 26/min, and a MAP of 65 mmHg. It is difficult to appreciate any new consolidation in chest X-ray with preexisting basal atelectasis. Which combination of cytokines is most likely responsible for an infectious complication resulting from immunosuppression in this scenario?

Choices: 1. IL-4 and IL-10 2. IL-6 and IL-8 3. IL-1 and TNF-alfa 4. Activated Protein C and Plasminogen Activator Inhibitor -1

Answer: 1 - IL-4 and IL-10 Explanations: The proinflammatory state of SIRS can give way to a compensatory anti-inflammatory response syndrome (CARS) when the body is in a state of immune dysregulation, relative immunosuppression and therefore susceptible to nosocomial infections. Compensatory anti-inflammatory response is mediated by interleukins IL-4 and IL-10. They inhibit the production of TNF-alpha, IL-1, IL-6, and IL-8. The individual thus becomes susceptible to nosocomial infection, which can thus reinitiate the septic cascade. Go to the next page if you knew the correct answer, or click the link image(s) below to further research the concepts in this question (if desired).

Research Concepts: Systemic Inflammatory Response Syndrome

We update eBooks quarterly and Apps daily based on user feedback. Please tap flag to report any questions that need improvement.

Question 48: A 64-year-old male presents to the emergency department with chief complaints of fever for 3 days, cough with sputum production, and chest pain for 2 days. His vitals include a temperature of 103 F, respiratory rate of 35/min, pulse 84/min, and blood pressure 104/70 mmHg. He is admitted for inpatient treatment and is started on empiric antibiotic coverage based on the local resistance patterns. On the second day, his vitals continue to deteriorate, and an arterial blood gas analysis shows a partial pressure of oxygen to be 54 mmHg. He is transferred to the intensive care unit and is ventilated with supplemental oxygen therapy of 50% oxygen at a tidal volume of 7 ml/kg, and positive end-expiratory pressure of 10 mmHg. His condition improves, and his oxygen saturation is maintained at 89%. On the third day, his fever recurs, and his oxygen saturation drops despite being on high FiO2. Chest radiography shows new onset infiltrates. What is the best next step in the management of this patient?

Choices: 1. CT scan of the chest 2. Continue empiric antibiotic therapy 3. Blood cultures 4. Mini bronchoalveolar lavage (Mini- BAL)

Answer: 4 - Mini bronchoalveolar lavage (Mini- BAL) Explanations: Ventilator-associated pneumonia (VAP) is any pneumonia observed 48 hours after being on a ventilator in a hospital setting. VAP should be suspected in ventilated patients with newonset dyspnea, falling oxygen saturation, or new onset of lung infiltrates. Management of VAP, unlike the management of community- or hospital-acquired pneumonia, involves both radiological evidence and invasive sampling before the administration of antibiotics. After the culture and sensitivity reports of bronchoalveolar lavage are available, the antibiotic regimen should be tailored appropriately. Mini-BAL is done by inserting a catheter through the endotracheal tube, instilling normal saline, and aspirating the sample. Mini-BAL is a less invasive, less expensive, and less-cumbersome technique than BAL. Go to the next page if you knew the correct answer, or click the link image(s) below to further research the concepts in this question (if desired).

Research Concepts: Ventilator-associated Pneumonia

We update eBooks quarterly and Apps daily based on user feedback. Please tap flag to report any questions that need improvement.

Question 49: A 17-year-old male with AIDS presents with worsening exertional dyspnea, fever, and a nonproductive cough. His temperature is 102 F, heart rate 120 beats/min, and respiratory rate 24. A physical exam reveals mild crackles and rhonchi bilaterally. A chest x-ray shows diffuse bilateral infiltrates. PaO2 is 69 mmHg on room air. He has no known drug allergies. What is the best initial treatment?

Choices: 1. Intravenous trimethoprim/sulfamethoxazole 2. Intravenous pentamidine 3. Intravenous corticosteroids followed by IV trimethoprim/sulfamethoxazole 4. Intravenous corticosteroids followed by IV levofloxacin

Answer: 3 - Intravenous corticosteroids followed by IV trimethoprim/sulfamethoxazole Explanations: Corticosteroids decrease mortality in patients with severe Pneumocystis jiroveci pneumonia (PCP). Trimethoprim/sulfamethoxazole is the drug of choice, but pentamidine is used for those with sulfa allergy. Levofloxacin does not cover PCP. However, if there are infiltrates, it can be added for possible bacterial pneumonia. Adjunctive corticosteroids can decrease the inflammatory response associated with PCP. Also, they can reduce the decline of oxygenation and reduce the incidence of respiratory failure. Go to the next page if you knew the correct answer, or click the link image(s) below to further research the concepts in this question (if desired).

Research Concepts: Pneumocystis Jirovecii Pneumonia

We update eBooks quarterly and Apps daily based on user feedback. Please tap flag to report any questions that need improvement.

Question 50: A 72-year-old white man with prostate cancer treated ten years ago with resection who presents with the complaint of double vision and not able to completely open the eyelid on the right side over the past two months. The patient denies any other focal weakness, sensory loss or difficulty in breathing, eating or coughing. Vitals are normal. The neurological exam showed restriction of right eye abduction, right side ptosis with miosis. The fundal exam is unremarkable. Miosis is not changed after cocaine eye drop to the right eye but left eye dilated. Basic serum blood workup and CT head are unremarkable. Which of the following structures are involved to have these symptoms?

Choices: 1. Trochlear nerve and ophthalmic nerve 2. Abducens nerve and sympathetic fibers 3. Oculomotor nerve and sympathetic fibers 4. Maxillary Nerve and abducens nerve

Answer: 2 - Abducens nerve and sympathetic fibers Explanations: If anisocoria increases and one eye shows minimal dilation with cocaine eye drop, it indicates Horner syndrome. On examination, he had a right abduction deficit consistent with an abducens nerve palsy. The combination of an abducens nerve palsy and a Horner syndrome indicated a cavernous sinus localization. The history of slow progression over months indicated a compressive or infiltrative lesion. A biopsy of the cavernous sinus lesion is rarely needed for diagnosis because of the proximity to critical neurologic structures makes difficult for any procedure like biopsy or excision. Go to the next page if you knew the correct answer, or click the link image(s) below to further research the concepts in this question (if desired).

Research Concepts: Cavernous Sinus Syndromes

We update eBooks quarterly and Apps daily based on user feedback. Please tap flag to report any questions that need improvement.

Question 51: An 85-year-old female is brought to the emergency department for confusion. Her family reports she has become depressed and is talking to her long-dead husband. She complains of abdominal, back, and hip pain with constipation. The neurological exam is nonfocal. The cardiopulmonary exam is unremarkable. There is a 3-centimeter, fixed breast mass. What is the most likely ECG finding?

Choices: 1. Flattened T waves 2. Prolonged QT interval 3. Shortened QT interval 4. Peaked T waves

Answer: 3 - Shortened QT interval Explanations: Given the fixed mass and symptoms, the patient is presenting with paraneoplastic hypercalcemia secondary to metastatic breast cancer. Hypercalcemia results in a shortened QT interval. Rarely, it can cause an arrhythmia. Hypercalcemia also can cause calcium deposits in the coronary arteries and valves resulting in increased cardiovascular morbidity. Hypercalcemia can affect the gastrointestinal system, central nervous system, kidneys, and musculoskeletal system. Symptoms may include abdominal pain, constipation, confusion, depression, nephrolithiasis, polyuria, weakness, and myopathy. The mnemonic for hypercalcemia is stones, bones, abdominal groans, and psychiatric overtones. Go to the next page if you knew the correct answer, or click the link image(s) below to further research the concepts in this question (if desired).

Research Concepts: Malignancy-Related Hypercalcemia

We update eBooks quarterly and Apps daily based on user feedback. Please tap flag to report any questions that need improvement.

Question 52: A 36-year-old female presents to the emergency department with complaints of productive cough, malaise, and fever. She has no known medical history and does not take any medications. Her blood pressure is 105/92 mmHg, temperature 101.3 F, respiratory rate 24/min, and heart rate 96/min. Her BMI is 40 kg/m2. This patient's pulse oximetry is 93% on room air. Examination shows jugular venous distention, clear lung fields. A portable chest x-ray shows low lung volumes but is unremarkable otherwise. Laboratory results show hemoglobin of 15 g/l, platelets 290000/microL, and leukocytes 5000/microL. His sodium is 142 mEq/L, potassium 4.3 mEq/L, chloride 99 mEq/L, bicarbonate 35 mEq/L, BUN 26 mg/dL, creatinine 1.0 mg/dL, and glucose 100 mg/dL. Which of the following is the best next step in managing this patient?

Choices: 1. CT chest 2. Arterial blood gas 3. Acetazolamide 4. Ventilation/ perfusion scan

Answer: 2 - Arterial blood gas Explanations: The patient came to the emergency room with upper respiratory symptoms. She likely has underlying obesity hypoventilation syndrome. Her serum bicarbonate is elevated. An ABG on room air should be done on any patient with suspected OHS to look up for daytime hypercapnia. With her respiratory symptoms, it's possible she may not be well compensated. An ABG will help determine if this patient needs positive pressure. Go to the next page if you knew the correct answer, or click the link image(s) below to further research the concepts in this question (if desired).

Research Concepts: Obesity-Hypoventilation Syndrome

We update eBooks quarterly and Apps daily based on user feedback. Please tap flag to report any questions that need improvement.

Question 53: A 48-year-old female with past medical history of B cell lymphoma colitis on pembrolizumab presents with bilateral perianal abscesses. Patients states she was here 1 week ago with the same and now increase in pain and erythema. What is the appropriate treatment for this patient?

Choices: 1. Incision and drainage with a surgeon at the bedside 2. Addition of ciprofloxacin for recurrence of infection. 3. Obtain an endoanal ultrasound 4. Obtain blood cultures, start intravenous antibiotics and admit to the hospital for imaging and possible surgical drainage

Answer: 4 - Obtain blood cultures, start intravenous antibiotics and admit to the hospital for imaging and possible surgical drainage

Explanations: Patients with recurrent or bilateral abscesses should be evaluated by surgery for intraoperative drainage, including cases where internal draining may be necessary. Ciprofloxacin should be combined with metronidazole for appropriate antibiotic coverage. CT should be obtained in patients with significant comorbidities, complex suppurative anorectal conditions, and when surgical consultation is not immediately available. Patients who are immunocompromised with bilateral and recurrent abscesses should be started on intravenous antibiotics after cultures are obtained and have a prompt surgical evaluation for intraoperative drainage. Go to the next page if you knew the correct answer, or click the link image(s) below to further research the concepts in this question (if desired).

Research Concepts: Perirectal Abscess

We update eBooks quarterly and Apps daily based on user feedback. Please tap flag to report any questions that need improvement.

Question 54: A 67-year-old thin female has recurrent abdominal pain every time she eats. She has lost 9 pounds (4 kg) in weight over the past 2 months. Examination reveals a soft epigastric systolic abdominal murmur and her aorta is palpable but not pulsatile or enlarged. What is her most likely diagnosis?

Choices: 1. Abdominal aortic aneurysm 2. Portal hypertension 3. Colon cancer 4. Mesenteric ischemia

Answer: 4 - Mesenteric ischemia Explanations: Patients with mesenteric ischemia have a very typical presentation. However, the diagnosis is often missed because of vague complaints and signs. Patients may present with a history of postprandial pain, typically starting 20 minutes after a meal, which may last up to 90 minutes. Because of the pain, they develop food fear and experience subsequent weight loss. Patients may be severely malnourished upon presentation. Chronic mesenteric ischemia usually results from long-standing atherosclerotic disease of 2 or more mesenteric vessels. Symptoms are caused by a progressive decrease in blood flow. In normal people, there is a 15% increase in blood flow during food consumption. Go to the next page if you knew the correct answer, or click the link image(s) below to further research the concepts in this question (if desired).

Research Concepts: Chronic Mesenteric Ischemia

We update eBooks quarterly and Apps daily based on user feedback. Please tap flag to report any questions that need improvement.

Question 55: A 75-year-old male with a past medical history of diabetes mellitus type 2, hypertension, peripheral vascular disease, hypothyroidism, and rheumatoid arthritis presents with significant substernal chest pain and diaphoresis after acute emotional stress. His electrocardiogram reveals anterior ST-segment elevations. His cardiac troponin levels are slightly elevated. A coronary angiogram reveals 40% stenosis of the proximal right coronary artery. The left anterior descending artery is wrapped around the apex of the heart. Left ventriculography reveals apical ballooning in the mid, distal, and apical segments, and a hypercontractile basal segment. What is the treatment for this disorder?

Choices: 1. Intravenous fluids 2. Inotropic agents 3. Beta-1 blockers, aspirin, angiotensin-converting-enzyme inhibitors, intravenous diuretics, and direct thrombin inhibitors 4. Percutaneous transluminal coronary angioplasty

Answer: 3 - Beta-1 blockers, aspirin, angiotensin-converting-enzyme inhibitors, intravenous diuretics, and direct thrombin inhibitors

Explanations: If Takotsubo cardiomyopathy is diagnosed and there is no comorbidity in the patient, medical treatment is supportive. This patient has cardiovascular and other system comorbidities. Therefore, he should be treated initially as someone presenting with acute coronary syndrome or myocardial infarction. If the patient is hypotensive, pressors may be required as well as an intra-aortic balloon pump and left ventricular assist device. Management should include continuous ECG monitoring. Echocardiography should be recommended at approximately 4 to 6 weeks postpresentation to document normalization of left ventricular function. The patient should be monitored closely if receiving inotropic agents because these agents can exacerbate symptoms and result in a dynamic left ventricular mid-cavity obstruction. If a dynamic intraventricular pressure gradient develops, discontinue the inotropic agent and administer intravenous beta-blockers to increase diastolic filling time and left ventricular end-diastolic volume. Go to the next page if you knew the correct answer, or click the link image(s) below to further research the concepts in this question (if desired).

Research Concepts: Takotsubo Cardiomyopathy

We update eBooks quarterly and Apps daily based on user feedback. Please tap flag to report any questions that need improvement.

Question 56: A 52-year-old male presents to the office with three months of right-sided chest pains, productive cough, and shortness of breath with associated low-grade fevers, chills, and 10 lbs unintentional weight loss. He has no medical problems that he is aware of, does not have a primary care provider, has never had any surgeries other than his wisdom teeth removed, denies any sick contacts, refuses all vaccinations, does not drink any alcohol or use any tobacco products or illicit drugs. He does report an extensive travel history in the Middle East and Northern Africa over the last three years as a military contractor. He reports that he got home about two weeks ago from Iraq, but his symptoms started while he was there and had been getting progressively worse. Vital signs are temperature 38.2 C, heart rate 98 bpm, respiratory rate 24/minute, blood pressure 122/78 mmHg, and oxygen saturation 92% on room air. He appears acutely ill, no acute distress, and is normocephalic and atraumatic. Sclera are nonicteric, and pupils equal round and reactive to light and accommodation. The neck nontender is with no jugular venous distention. He is mildly tachycardic with regular rhythm and no murmur. Lungs are clear to auscultation bilaterally. The abdomen is soft, nontender with normal bowel sounds. There is no costovertebral angle tenderness or bladder distension. He has normal speech with no neurologic deficits. His chemistry panel shows normal electrolytes, normal renal function, and normal hepatic. His complete blood count shows mild leukocytosis with a WBC count of 12.6/microL and his differential 2.2% eosinophils. Chest x-ray shows there is a small cavitary lung lesion in the right upper lobe. CT scan of the chest shows a 2 cm x 3 cm ring-enhancing cavitary lung abscess in the right upper lobe with an air-fluid level. There is no hilar/mediastinal lymphadenopathy, and there are no other lesions. The patient is diagnosed with sepsis secondary to a lung abscess and admitted to the medical floor, and started on broad-spectrum antibiotics with IV vancomycin and ampicillin-sulbactam. Over the next three days, the patient continues to have low-grade fevers, and his cough continues. He undergoes bronchoscopy and bronchoalveolar lavage on hospital day four. Cytology of the fluid is negative for malignancy but does note a high preponderance of eosinophils and trematode eggs. His blood and sputum cultures remain negative since admission. Serological antigen testing reveals that the eggs belong to Fasciola giganticum. What is the next best step?

Choices: 1. Left upper lobectomy 2. Triclabendazole 3. Ivermectin and praziquantel 4. Diethylcarbamazine

Answer: 2 - Triclabendazole Explanations: The treatment for fasciola infection is oral triclabendazole 10 mg/kg daily taken after a meal for two days. It is typically very well tolerated and effective. Fasciola reportedly does not respond well to praziquantel, mebendazole, or albendazole. Ivermectin is the drug of choice for strongyloidiasis and onchocerciasis but has some activity against filarial worms. Praziquantel has activity against intestinal tapeworms, schistosomes, neurocysticercosis, and other flukes besides fasciola. Diethylcarbamazine has activity against loiasis, visceral larva migrans, and lymphatic filariasis but does not have any activity against fasciola. Go to the next page if you knew the correct answer, or click the link image(s) below to further research the concepts in this question (if desired).

Research Concepts: Fascioliasis

We update eBooks quarterly and Apps daily based on user feedback. Please tap flag to report any questions that need improvement.

Question 57: A 30-year-old, previously good health, non-smoker, triathlete is helped out of the water due to extreme exhaustion and dyspnea. While being examined, he begins coughing up blood-tinged sputum. He is transported to the local emergency room on 100% oxygen due to oxygen saturation of 91%. All other vitals are stable. What is expected on chest x-ray when he arrives at the hospital?

Choices: 1. Cardiomegaly 2. Unilateral pleural effusion 3. Widened mediastinum 4. Kerley B lines

Answer: 4 - Kerley B lines Explanations: While there is some component of heart failure in immersion pulmonary edema (IPE) it may or may not be significant enough to show up on chest x-ray. Pleural effusions may be seen with IPE but it would be bilateral. While there is a redistribution of blood centrally during submersion which may lead to increased diameter of the inferior vena cava, it may not be radiographically significant, and there is a rapid redistribution after removal from the water. Kerley B lines are indicative of pulmonary edema. Go to the next page if you knew the correct answer, or click the link image(s) below to further research the concepts in this question (if desired).

Research Concepts: Immersion Pulmonary Edema

We update eBooks quarterly and Apps daily based on user feedback. Please tap flag to report any questions that need improvement.

Question 58: A 78-year-old female, history of coronary artery disease, hypertension, diabetes mellitus, and hypothyroidism, presents to the emergency department with two days of fever, headache, and neck stiffness. Her physical examination is significant for fever of 101.1 F and heart rate of 110 bpm. She has a stiff neck, photophobia, and nuchal rigidity. Laboratory values are significant for leukocytosis. Lumbar puncture is pending. What is the most appropriate empiric regimen of antibiotics?

Choices: 1. Vancomycin and amoxicillin 2. Ceftriaxone and amoxicillin-clavulanic acid 3. Vancomycin, ceftriaxone, and ampicillin 4. Ceftriaxone and Ampicillin

Answer: 3 - Vancomycin, ceftriaxone, and ampicillin Explanations: Prompt initiation of antibiotics is critical in the management of suspected bacterial meningitis, which has been shown to improve long-term outcomes. Ampicillin is an important consideration in the empiric regimen in patients over the age of 50 and in neonates, given the increased prevalence of Listeria monocytogenes. Ceftriaxone has excellent blood-brain barrier penetration and covers most organisms that typically cause meningitis, such as Streptococcus pneumoniae, Neisseria meningitidis, and Haemophilus influenzae. The reason for coverage with vancomycin is less for methicilline resistant staphylococcus, but rather for coverage of penicillin resistant S. pneumoniae Go to the next page if you knew the correct answer, or click the link image(s) below to further research the concepts in this question (if desired).

Research Concepts: Bacterial Meningitis

We update eBooks quarterly and Apps daily based on user feedback. Please tap flag to report any questions that need improvement.

Question 59: A 38-year-old male presents to the emergency department after a 3-day history of right-sided neck pain and headache. He has no significant past medical history. His headache started after a day at an amusement park where he rode several roller coasters. He had taken aspirin without relief but became concerned when he had a 10-minute episode of confusion associated with left-hand numbness and clumsiness. An exam shows mild, right ptosis, anisocoria with the right pupil being smaller, and left arm drift. What is the most likely diagnosis?

Choices: 1. Right vertebral artery dissection 2. Right internal carotid dissection 3. Hemiplegic migraine 4. Paroxysmal hemicrania

Answer: 2 - Right internal carotid dissection Explanations: The patient has Horner syndrome, neck pain, headache, and transient ischemic symptoms on the right. The most probable diagnosis is right internal carotid dissection. The dissection could be spontaneous but was more likely secondary to neck movements on the roller coasters. Treatment is with anticoagulation or antiplatelet medication, but angioplasty and stent may be needed if there are persistent ischemic symptoms. Go to the next page if you knew the correct answer, or click the link image(s) below to further research the concepts in this question (if desired).

Research Concepts: Carotid Artery Dissection

We update eBooks quarterly and Apps daily based on user feedback. Please tap flag to report any questions that need improvement.

Question 60: A 65-year-old female underwent transfemoral coronary angioplasty with stent placement. 3 hours after the procedure, she is having hypotension and low back pain. Patient denies any chest pain or dyspnea. On physical exam, heart sounds are within normal limits with no jugular venous distention, and lungs are clear to auscultation. The dressing at femoral entry site is clear. What is the underlying cause of patient’s hypotension?

Choices: 1. Acute myocardial infarction 2. Acute stent thrombosis 3. Retroperitoneal hemorrhage 4. Coronary artery perforation

Answer: 3 - Retroperitoneal hemorrhage Explanations: Retroperitoneal hemorrhage (RPH) is an infrequent but fatal complication of transfemoral percutaneous coronary intervention. RPH present with hypotension with or without back pain. Severe hypotension or shock may need an urgent blood transfusion. Female gender is associated with an increased risk for RPH. Females have smaller diameter femoral arteries than men which may make access more difficult. This difference can lead to a greater likelihood of multiple and posterior arterial wall punctures, thus leading to RPH. A higher femoral arterial puncture site, such as above the inguinal ligament or above the middle one-third of the femoral head on fluoroscopy is an important procedure-related risk factor for RPH. Go to the next page if you knew the correct answer, or click the link image(s) below to further research the concepts in this question (if desired).

Research Concepts: Angioplasty

We update eBooks quarterly and Apps daily based on user feedback. Please tap flag to report any questions that need improvement.

Question 61: A 37-year-old patient with alcohol use disorder presents with 2 weeks of stomach pain and fatigue. Physical findings are confusion, jaundice, normal bowel sounds, right upper quadrant tenderness without rebound or guarding, liver edge 7 cm below the costal margin at the midclavicular line, splenomegaly, and asterixis. Labs show a marked white blood cell count without left shift and elevated transaminases. The patient has a calculated Maddrey discriminant function of 48. An ultrasound of the liver shows hepatomegaly, increased echogenicity, gallstones, and no ascites. A chest x-ray and urinalysis are normal. Which of the following is false?

Choices: 1. This presentation could reflect concomitant hepatitis C infection 2. Lactulose therapy is indicated 3. This patient can be managed as an outpatient 4. Morbidity and mortality can be decreased by IV methylprednisolone

Answer: 3 - This patient can be managed as an outpatient Explanations: This patient's severe acute alcoholic hepatitis must be managed as an inpatient. A Maddrey discriminant function score above 32 suggests increased disease severity and mortality. Encephalopathy is an indication for lactulose therapy. Concomitant hepatitis C infection increases morbidity synergistically. This patient's 30-day mortality rate of 30% to 50% can be reduced with IV corticosteroids. Go to the next page if you knew the correct answer, or click the link image(s) below to further research the concepts in this question (if desired).

Research Concepts: Alcoholic Hepatitis

We update eBooks quarterly and Apps daily based on user feedback. Please tap flag to report any questions that need improvement.

Question 62: A 55-year-old male with a history of chronic obstructive pulmonary disease and bronchiectasis, a current everyday smoker with a 50 pack-year history, was brought into the emergency department due to unresponsiveness. His wife confirms that the patient was not feeling well for the past four days with an upper respiratory viral infection. His symptoms included generalized malaise, fevers, shortness of breath, and cough. His home medications include fluticasone and salmeterol twice a day and albuterol as needed for shortness of breath. He went to urgent care two days ago and was prescribed azithromycin, prednisone, albuterol, and ipratropium nebulization which only provided temporary relief. In the emergency department, vital signs show a blood pressure of 140/72 mmHg, heart rate of 98/min, respiratory rate of 10/min, temperature 38.3 C, and pulse oximetry of 88% on room air. On physical examination, the patient is noted to be lethargic, mildly responsive to painful stimuli, and has decreased air movement on auscultation of his chest. Arterial blood gas (ABG) was drawn, and the results are as follows, pH 7.02, pO2 55 mmHg, pCO2 100 mmHg. Chest X-ray showed bilateral multifocal infiltrates. The patient was endotracheally intubated and placed on mechanical ventilation. Initial settings were a respiratory rate of 20/min, tidal volume 500 m, a fraction of inspired oxygen 1.0, and positive end-expiratory pressure of 5 cm H20. He is started on inhaled bronchodilators, intravenous methylprednisolone, broad-spectrum antibiotics, and oseltamivir. He becomes asynchronous with the ventilator requiring sedation and later neuromuscular blockade. His repeat ABG shows a pH of 6.95, pO2 70 mmHg, pCO2 120 mmHg, and he is currently becoming hypotensive. Due to difficulty with ventilation, the patient is placed on a venovenous extracorporeal membrane oxygenator (VV-ECMO). The patient is cannulated for VV-ECMO. The inflow cannula is inserted in the right femoral vein and outflow cannula in the right internal jugular vein with flows of 3 liters per minute, sweep four, and fraction of inspired oxygen 1.0. ABG on these settings show pH 7.20, pO2 50 mmHg, pCO2 70 mmHg. What is the next best step in management?

Choices: 1. Start the patient on inhaled nitric oxide 2. Increase the dose and frequency of intravenous steroids 3. Order a stat chest ultrasound 4. Increase flow to 4 liters/minute

Answer: 3 - Order a stat chest ultrasound Explanations: It is possible to perform VV-ECMO cannulation blindly, but it is recommended to cannulate for ECMO under direct visualization, usually with fluoroscopy or ultrasound, to confirm appropriate placement of the inflow and outflow cannulas. In the double VV-ECMO cannulation, there are two ways to insert the cannulas. The inflow cannula is usually inserted in the femoral vein and advanced to the junction of the inferior vena cava (IVC), and the right atrium and the outflow cannula can be inserted into the right internal jugular vein and advanced through the superior vena cava to the right atrium. The second approach is to insert the inflow cannula into the femoral vein and advanced to the mid-IVC, and the outflow cannula can be inserted in the contralateral femoral vein with the tip ending in the right atrium. The correct placement of the cannulas is important to avoid the recirculation phenomenon. This is when the oxygenated blood that is being returned to the patient is drained by the inflow cannula before being circulated through the body. This is usually due to the two cannulas being in close proximity. The flow rate, the fraction of inspired oxygen, the amount of recirculation of blood will all affect the ability of the VV-ECMO circuit to oxygenate the blood. Increasing the flow when pO2 is low is appropriate, but the placement of the cannulas should always be confirmed after cannulation. Go to the next page if you knew the correct answer, or click the link image(s) below to further research the concepts in this question (if desired).

Research Concepts: Extracorporeal Membrane Oxygenation In Adults

We update eBooks quarterly and Apps daily based on user feedback. Please tap flag to report any questions that need improvement.

Question 63: A 47-year-old male is brought to the emergency department for 2 days of fever, headache, and altered mental status. His wife reports that he has been confused and forgetful since yesterday. The patient is an accountant with no significant medical history. He has had no sick contacts and has not traveled recently. He does not use tobacco or illicit drugs but drinks alcohol occasionally. Temperature is 38 C, blood pressure is 126/84 mmHg, and pulse is 72/minute. Mucous membranes are moist with no lesions. No lymphadenopathy is present. Lungs are clear to auscultation, and heart sounds are normal with no murmur. The abdomen is soft and nontender, with no hepatosplenomegaly. He has nuchal rigidity and is alert and oriented only to self. He has no motor or sensory deficits but can only follow simple commands. Cerebrospinal fluid analysis shows a white blood cell count of 150/mm3, predominantly lymphocytes, red blood cell count of 8/mm^3, protein concentration of 130 mg/dL, and glucose concentration of 70 mg/dL. Which of the following is the best next step in management?

Choices: 1. Administer intravenous acyclovir 2. Obtain an MRI 3. Consult neurointerventional radiology 4. Administer intravenous vancomycin and ceftriaxone

Answer: 1 - Administer intravenous acyclovir Explanations: The patient’s presenting history, exam, and cerebrospinal fluid findings are consistent with viral encephalitis. Red blood cells may be present in the cerebrospinal fluid of patients with herpes simplex virus (HSV) encephalitis. Acyclovir should be started empirically before further confirmatory testing. Acyclovir has been shown to significantly decrease the mortality and morbidity of HSV encephalitis if started early. Acyclovir limits the severity of long-term behavioral and cognitive impairment seen with HSV encephalitis. The intravenous route has been shown to have greater efficacy than the oral form. Go to the next page if you knew the correct answer, or click the link image(s) below to further research the concepts in this question (if desired).

Research Concepts: Viral Encephalitis

We update eBooks quarterly and Apps daily based on user feedback. Please tap flag to report any questions that need improvement.

Question 64: A novel waste anesthetic gas disposal system is designed, with active capture and closed features on the receiving element. The manufacturer’s intended use is to minimize pollution and optimize resource recycling by attaching it in line with the ventilatory circuit, between the Y-piece and the patient. A 61-year-old 59 kg male undergoes laparoscopic cholecystectomy for acute calculous cholecystitis refractory to conservative therapy. His past medical history is notable for uncontrolled hypertension, COPD, and active tobacco use. The patient is induced with intravenous agents, and after uneventful intubation, he is mechanically ventilated. Insufflation and appropriate positioning are achieved, requiring minimal ventilator adjustments to maintain oxygenation. As the gallbladder is being dissected off of the liver, the anesthesia alarms indicate high airway pressures. Vital signs show a blood pressure 82/40 mmHg, heart rate 118/min, and SpO2 87%. Which of the following features of the waste anesthetic gas disposal system could have most likely prevented this patient’s pathology from developing?

Choices: 1. Longer connection tubing 2. Presence of a pressure relief valve 3. Smaller-diameter tubing 4. Passive system mechanics

Answer: 2 - Presence of a pressure relief valve Explanations: Pressure relief valves are important safety features that minimize the potential for causing barotrauma. The novel system described here both bypasses the native disposal system and has an active scavenging method, which utilizes pressure to achieve effective collection. This combination in a patient with underlying pulmonary disease raises concern for tension pneumothorax in the setting of compromised pulmonary compliance during a laparoscopic procedure. Longer connection tubing would not effectively reduce the potential for causing barotrauma leading to tension pneumothorax. Resistance to flow increases as the radius of the conduit decreases. This could potentially worsen already-elevated airway pressures in an abdominally-insufflated patient. Although passive waste anesthetic gas disposal systems do not utilize artificially-generated pressure to collect volatile agents, the patient’s underlying comorbidities would likely be better served by a pressure-limiting valve. Go to the next page if you knew the correct answer, or click the link image(s) below to further research the concepts in this question (if desired).

Research Concepts: Waste Gas Scavenging System

We update eBooks quarterly and Apps daily based on user feedback. Please tap flag to report any questions that need improvement.

Question 65: A 25-year-old female has had a 4-month history of substernal chest pain and dyspnea on exertion. Electrocardiogram shows right axis deviation. An arterial blood gas shows pH 7.46, PO2 80 mmHg, and PCO2 32 mmHg. A chest x-ray shows enlarged pulmonary arteries but no infiltrates. A spiral CT shows subsegmental defects not consistent with pulmonary embolism. Echocardiogram shows no primary cardiac disease but there is right heart strain. Which of the following would be the most appropriate next step in evaluation?

Choices: 1. Ventilation perfusion scan 2. Bronchoscopy 3. Right heart catheterization 4. Antineutrophil cytoplasmic antibody

Answer: 3 - Right heart catheterization Explanations: The patient most likely has primary pulmonary hypertension. Features on a chest x-ray include large central pulmonary arteries, right ventricular hypertrophy, and clear lung fields. An ECG typically shows right ventricular hypertrophy with right atrial enlargement, right axis deviation, and increased amplitude of P waves due to right atrial enlargement (lead II). Echocardiography is the most sensitive test, which helps assess right ventricular size and pressure-volume overload. It also gives an estimate of pulmonary artery pressure. In primary pulmonary hypertension, there is right atrial and ventricular enlargement and tricuspid regurgitation. An arterial blood gas may reveal an increased A-a gradient and hypoxia. Pulmonary function tests will show an impaired diffusing capacity of the lungs for carbon monoxide (DLCO). Right heart catheterization is the criterion standard diagnostic test to detect elevated pressures and confirm pulmonary hypertension. Once diagnosed, additional studies should be performed to evaluate for the etiology of pulmonary hypertension. Causes of pulmonary hypertension include thromboembolism, chronic obstructive pulmonary disease, obstructive sleep apnea, and heart disease. In those from undeveloped countries, infections with filariasis and schistosomiasis should be considered. Go to the next page if you knew the correct answer, or click the link image(s) below to further research the concepts in this question (if desired).

Research Concepts: Idiopathic Pulmonary Artery Hypertension

We update eBooks quarterly and Apps daily based on user feedback. Please tap flag to report any questions that need improvement.

Question 66: A 65-year-old male with a history of hypertension on multiple medications with good compliance, progressively worsening chronic kidney disease (over the last 12 months, creatinine increasing from 1.2 to 2.0 mg/dL, eGFR declining from 65 to 35 mL/min/m2) presents to the emergency department with sudden onset dyspnea which started an hour prior to presentation when he was asleep. He also reports an increased frequency of headaches over the last 12 months. The examination reveals the patient to be in mild distress with tachypnea, blood pressure of 230/132 mmHg, heart rate of 100/min, and oxygen saturation of 92% on room air which improved to 98% on 2 L supplemental oxygen by the nasal cannula. Chest x-ray reveals extensive pulmonary edema. Laboratory data reveals hemoglobin of 11 mg/dL, and creatinine 2.1 mg/dL. N terminal pro-brain-natriuretic peptide (NT-proBNP) is 1054 ng/L. EKG shows normal sinus rhythm with left ventricular hypertrophic changes. A bedside echocardiogram reveals normal ejection fraction with moderate left ventricular hypertrophy with no hemodynamically significant valvular lesions. The patient was started on intravenous nitroglycerin infusion, given IV furosemide, and transferred to the intensive care monitoring (ICU) for close monitoring. Coronary angiography is performed, which shows nonobstructive coronary disease. What is the next step to prevent further occurrences of this patient's pulmonary edema?

Choices: 1. Change furosemide to a long-acting loop diuretic like torsemide 2. Renal sympathetic denervation 3. Renal angiography and endovascular stenting 4. Initiate therapy with mineralocorticoid receptor antagonist like spironolactone

Answer: 3 - Renal angiography and endovascular stenting Explanations: This patient has flash pulmonary edema secondary to renal artery stenosis. Renal angiography is the gold-standard test to diagnose renal artery atherosclerosis and further, stenosis. Percutaneous renal artery stenting should be strongly considered in patients with labile hypertension, progressively worsening renal function, and flash pulmonary edema. A careful selection of patients for renal artery stenting must be conducted as some patients can continue to have a rapid decline in renal function due to atheroembolic disease after angioplasty. Along with consideration of renal angioplasty, the patient's hypertension should be tightly controlled to prevent further recurrences of flash pulmonary edema. The initiation of a longacting loop diuretic, as well as mineralocorticoid receptor antagonists, can be considered to manage hypertension. Renal denervation therapy has not shown statistically significant improvement in systolic blood pressure and is currently not utilized for the treatment of resistant hypertension. Go to the next page if you knew the correct answer, or click the link image(s) below to further research the concepts in this question (if desired).

Research Concepts: Pulmonary Edema

We update eBooks quarterly and Apps daily based on user feedback. Please tap flag to report any questions that need improvement.

Question 67: A 56-year-old man is found with altered mental status by his wife. His history is notable for back pain for which he takes opioid pain medication as well as clonidine for which he takes for hypertension. No evidence of trauma. He is brought to the emergency department and found to have miotic pupils, decreased respiratory status, bradycardia, and hypotension. In addition to getting peripheral access, all pertinent labs including point of care glucose, and maintaining his airway, what is the next medication that should be administered?

Choices: 1. Norepinephrine 2. Naloxone 3. Atropine 4. Dopamine

Answer: 2 - Naloxone Explanations: Naloxone is a very easy drug to give, and emergency departments should have this drug readily available. Even if it turns out this patient overdosed on his clonidine, naloxone should be given first as an opioid and alpha agonist overdose present very similarly. Nonresponse to naloxone can be helpful in diagnosing an alpha agonist overdose. Norepinephrine is the vasopressor of choice when dealing with a clonidine or alpha agonist overdose as it will correct both the hypotension and the bradycardia. The mainstay of treatment when treating an alpha agonist overdose is replacing the catecholamines that are not being endogenously secreted because of central alpha-2 receptor activation. Go to the next page if you knew the correct answer, or click the link image(s) below to further research the concepts in this question (if desired).

Research Concepts: Alpha Receptor Agonist Toxicity

We update eBooks quarterly and Apps daily based on user feedback. Please tap flag to report any questions that need improvement.

Question 68: A 75-year-old man with hypertension, diabetes mellitus, and hyperlipidemia presents with severe epigastric abdominal pain for three days associated with vomiting. His lipase is found to be extremely elevated, and CT shows acute necrotizing pancreatitis. Clinicians find discoloration due to ecchymosis on his flanks. What is the name for this finding?

Choices: 1. Cullen sign 2. Grey Turner sign 3. Bryant sign 4. Fox sign

Answer: 2 - Grey Turner sign Explanations: Retroperitoneal or intraabdominal bleeding may track to the subcutaneous tissue and present as discoloration or ecchymosis of the flanks known as the Grey Turner sign. Although classically associated with severe acute pancreatitis almost any pathology that causes intraabdominal or retroperitoneal bleeding may cause the Grey Turner sign. Researchers have not established the sensitivity and specificity of the Grey Turner sign for intraabdominal or retroperitoneal bleeding definitively. However, less than 5% of patients with relatively severe pancreatitis have a Grey Turner sign showing that the sign itself is not sensitive in detecting pancreatitis. Other subcutaneous manifestations of intraabdominal or retroperitoneal bleeding include ecchymosis around the umbilicus (Cullen sign), ecchymosis of the upper thigh just below the inguinal ligament (Fox sign), and ecchymosis of the scrotum (Bryant sign). Go to the next page if you knew the correct answer, or click the link image(s) below to further research the concepts in this question (if desired).

Research Concepts: Grey-Turner Sign

We update eBooks quarterly and Apps daily based on user feedback. Please tap flag to report any questions that need improvement.

Question 69: A 71-year-old male reports chest pain and shortness of breath during a right subclavian central venous catheter placement. After halting the procedure, you discover absent right-sided lung sounds. His vitals are stable with mild tachypnea. What is the first step in the management of this patient?

Choices: 1. Perform a needle decompression in the fifth intercostal space of the midclavicular line 2. Emergently place a tube thoracostomy in the seventh intercostal space of the midaxillary line 3. Call for chest radiography to confirm the diagnosis 4. Perform a needle decompression in the fifth intercostal space of the midaxillary line

Answer: 4 - Perform a needle decompression in the fifth intercostal space of the midaxillary line

Explanations: In recent studies, needle decompressions have been shown to be more effective in lateral versus medial approaches in patients with pneumothorax. For this patient, a needle decompression in the fifth intercostal space of the midaxillary line should be performed. Needle decompressions are considered temporary measures with final management consisting of a tube thoracostomy. Correct tube thoracostomy placement is commonly in the fifth intercostal space of the midaxillary line. Iatrogenic pneumothorax is a patient safety indicator (PSI) condition. It is defined as a traumatic pneumothorax that is caused secondary to an invasive procedure or surgery. Go to the next page if you knew the correct answer, or click the link image(s) below to further research the concepts in this question (if desired).

Research Concepts: Iatrogenic Pneumothorax

We update eBooks quarterly and Apps daily based on user feedback. Please tap flag to report any questions that need improvement.

Question 70: A 65-year-old female was admitted to the ICU after suffering head trauma and had eventually a complicated course with acute respiratory distress syndrome that required using Veno-venous extracorporeal membrane oxygenation (ECMO). A decision was made to perform an apnea test to assess for brain death. PO2 was 40 mmHg on FiO2 of 100% before connecting the patient to the ECMO machine. Which of the following is the most appropriate strategy to maintain oxygenation during the test while allowing the rising of CO2?

Choices: 1. Decrease gas sweep flow rate to 0 L/min, while maintenance of oxygenation by a cannula connected to the ET tube at 12 L/min 2. Decrease gas sweep flow rate to 0.5 L/min, and disconnect the mechanical ventilator 3. Decrease gas sweep flow rate to 2.0 L/min, while maintenance of oxygenation by a cannula connected to the ET tube at 12 L/min 4. Decrease gas sweep flow to 1.0 L/min, while maintenance of oxygenation by a CPAP delivery system at 10 cm H2O

Answer: 4 - Decrease gas sweep flow to 1.0 L/min, while maintenance of oxygenation by a CPAP delivery system at 10 cm H2O

Explanations: The gas sweep flow rate could be decreased to the range of (0.5-1.0 L/min) to allow the rising of CO2 while preventing severe hypoxemia. CPAP delivery system was found to be more effective than oxygen insufflation by a cannula through the ET tube to maintain oxygenation during the apnea test for patients on ECMO. Using ECMO machines isn't a contraindication to perform the apnea test. The apnea test should be aborted if the patient develops severe hypoxia. While performing the apnea test, using a CPAP delivery system for oxygenation is found to be more effective than oxygen insufflation through a cannula especially for patients on ECMO. Decreasing the gas sweep flow rate to 0.5-1 L/min is found to be appropriate to avoid hypoxia while allowing the rise of CO2 for patients on ECMO undergoing the apnea test. Go to the next page if you knew the correct answer, or click the link image(s) below to further research the concepts in this question (if desired).

Research Concepts: Brain Death Criteria

We update eBooks quarterly and Apps daily based on user feedback. Please tap flag to report any questions that need improvement.

Question 71: A 50-year-old woman survives a near-drowning episode. She is brought to the emergency department. Initial 12-lead electrocardiogram demonstrates T-wave inversions in the precordial leads. Her blood pressure is 70/40 mmHg, and the heart rate is 112/minute. Emergency coronary angiography demonstrates no evidence of epicardial coronary artery disease. Left ventriculography shows basal hyperkinesis with apical akinesis. She is admitted to the coronary care unit, given her significant hypotension and tachycardia. Which of the following vasopressors is most appropriate for this patient?

Choices: 1. Phenylephrine 2. Norepinephrine 3. Epinephrine 4. Dobutamine

Answer: 1 - Phenylephrine Explanations: This patient has Takotsubo cardiomyopathy which developed after stressful triggers, in this case, the near-drowning episode. Furthermore, this patient has developed a complication of Takotsubo cardiomyopathy, left ventricular outflow tract obstruction (LVOTO), which is causing her hypotension. Phenylephrine, a pure alpha agonist is an optimal therapy here because it reduces cardiac contractility and heart rate. Intravenous beta blocker or calcium channel blocker and volume expansion can also help to increase left ventricular end diastolic volume and relieve left ventricular outflow tract obstruction here. LVOT obstruction is an important complication to recognize in typical Takotsubo cardiomyopathy which involves basal hyperkinesis with apical akinesis such as in this case. It may result in hypotension, tachycardia, and cardiogenic shock. Vasopressors or inotropes such as norepinephrine, epinephrine, and dobutamine will be counterproductive as it will further increase contractility and worsen the clinical situation. Go to the next page if you knew the correct answer, or click the link image(s) below to further research the concepts in this question (if desired).

Research Concepts: Takotsubo Cardiomyopathy

We update eBooks quarterly and Apps daily based on user feedback. Please tap flag to report any questions that need improvement.

Question 72: A 52-year-old woman with a history of resistant hypertension presents to the emergency department with complaints of chest tightness radiating to the left arm, dyspnea, diaphoresis, and headache for the past 45 minutes. Physical examination reveals faint heart sounds and a bruit in the right flank. Troponin levels are elevated. EKG shows normal sinus rhythm with non-specific ST-T wave changes. The patient is admitted for emergent angiography which reveals type 2 spontaneous coronary artery dissection (SCAD). Which of the following condition is the most likely predisposing factor for SCAD in this patient?

Choices: 1. Systemic sclerosis 2. Systemic lupus erythematosus 3. Takayasu arteritis 4. Fibromuscular dysplasia

Answer: 4 - Fibromuscular dysplasia Explanations: Fibromuscular dysplasia (FMD) is a non-inflammatory and non-atherosclerotic disorder that leads to arterial stenosis, occlusion, aneurysm, and dissection. It is a common predisposing factor of spontaneous coronary artery dissection (SCAD). Suspicion for FMD is high in this patient due to the history of resistant hypertension and the bruit in the right flank In patients with SCAD, up to 85% patients will have angiographic evidence of FMD. The two angiographic subtypes of FMD are multifocal and focal. Multifocal is more common and gives a "string of beads" appearance on angiography. Focal FMD is less common and has the angiographic appearance of a "circumferential or tubular stenosis." In patients presenting with SCAD with high suspicion for FMD, a full-body, cross-sectional imaging study with CT angiography or MR angiography is recommended. Go to the next page if you knew the correct answer, or click the link image(s) below to further research the concepts in this question (if desired).

Research Concepts: Coronary Artery Dissection

We update eBooks quarterly and Apps daily based on user feedback. Please tap flag to report any questions that need improvement.

Question 73: A 17-year-old male is shot in the chest during an altercation. Emergency medical services (EMS) have arrived on the scene and have found the patient to be in cardiac arrest and begin advanced cardiac life support, including cardiopulmonary resuscitation (CPR). The emergency medical services crew is 15 minutes away and have been on the scene doing CPR for the last 10 minutes. The patient now arrives in the trauma bay, and EMS reports the patient is in asystole with a downtime of approximately 25 minutes with no signs of life and no return of spontaneous circulation. Which of the following would is the most appropriate management at this time?

Choices: 1. Give an additional dose of intravenous epinephrine and perform immediate defibrillation 2. Perform an emergency department resuscitative thoracotomy with aortic cross-clamping and internal cardiac massage 3. Place an echo probe on the patient's chest and if no cardiac motion is identified cease resuscitative efforts 4. Initiate the massive-transfusion protocol and send emergently to the operating room

Answer: 3 - Place an echo probe on the patient's chest and if no cardiac motion is identified cease resuscitative efforts

Explanations: Patients who present in traumatic cardiac arrest are managed differently with respect to the injury mechanism. Penetrating trauma, which leads to exsanguination, pericardial tamponade, tension hemothorax, or tension pneumothorax, can be treated if done expeditiously. The advanced cardiac life support (ACLS) and advanced trauma life support (ATLS) courses promulgated by the American Heart Association and the American College of Surgeons teach the 5 H's and T's (hypoxia, hypovolemia, hyperkalemia, hydrogen ions, hypothermia, tension pneumothorax, thrombosis, toxins, thromboembolism, tamponade) for potentially reversible causes of asystole. If not identified and treated early and return of spontaneous circulation is restored, patients have a very dismal prognosis. A patient who presents in traumatic cardiac arrest from a penetrating wound who arrives after 20 minutes of known downtime should be declared dead on arrival after an echo probe is placed on the patient's chest, and no cardiac motion is identified. Cardiopulmonary resuscitation, when performed correctly, can restore about 25% of a patient's normal cardiac output. Cerebral hypoxia/anoxia continues until circulation is restored and data has shown that survival in normothermic traumatic arrest from a penetrating injury approaches zero after about 20 minutes and resuscitative efforts can be ceased. Additional doses of epinephrine may be given to a patient in asystole from a penetrating injury, but defibrillation is not warranted unless in ventricular fibrillation or ventricular tachycardia without a pulse. Emergency department resuscitative thoracotomy with aortic cross-clamping and internal cardiac massage is reserved for patients who have a penetrating mechanism of injury leading to traumatic cardiac arrest with known downtimes of less than 20 minutes. When this time-frame is exceeded, performing this high-risk procedure is more of a risk to the resuscitation team than benefit to the patient as the patient's survivability nears 0%. Go to the next page if you knew the correct answer, or click the link image(s) below to further research the concepts in this question (if desired).

Research Concepts: Asystole

We update eBooks quarterly and Apps daily based on user feedback. Please tap flag to report any questions that need improvement.

Question 74: A 71-year-old male is brought to the emergency department (ED) by emergency medical services (EMS) after being found unresponsive at home by family. Upon discovery, the patient is not displaying spontaneous respiration and is found to be pulseless. Basic life support protocols were performed by a friend until the arrival of EMS, at which time advanced cardiac life support protocols (ACLS) were enacted. EMS obtained a history from a friend who notes that the patient has a medical history of hypertension, diabetes mellitus, and smoking cigarettes. She notes that the patient was recently discharged after being hospitalized for chest pain and that during his stay, he underwent cardiac catheterization and had two drugeluting stents placed. Upon arrival to the ED, ACLS protocols are continued by ED staff. Unfortunately, after 40 minutes of ACLS protocols, EMS and ER staff are unable to obtain a return of spontaneous circulation, and the patient is declared deceased. The family is notified and is asking what the most likely cause of death was. Which of the following best describes the cause of death in this patient?

Choices: 1. Silent stent thrombosis 2. Possible stent thrombosis 3. Probable stent thrombosis 4. Definite stent thrombosis

Answer: 3 - Probable stent thrombosis Explanations: Any unexplained death within 30 days of stent placement should be considered as probable stent thrombosis. This diagnosis should be considered if there is active ischemia on electrocardiogram or stress in the distribution of prior stent and absence of significant coronary lesion on angiography. Probable stent thrombosis should be considered in a patient with acute coronary syndrome within 30 days of stent placement. Diagnosis definite stent thrombosis requires confirmation of thrombosis with coronary angiogram. Go to the next page if you knew the correct answer, or click the link image(s) below to further research the concepts in this question (if desired).

Research Concepts: Stent Thrombosis

We update eBooks quarterly and Apps daily based on user feedback. Please tap flag to report any questions that need improvement.

Question 75: A 62-year-old female is treated with a ventriculoperitoneal shunt for hydrocephalus. One month later, she presents with a headache, unsteadiness and fever. She attends the emergency department who discusses the case with the on-call neurosurgical team. Following a normal set of labs, including inflammatory markers and CBC, a shunt tap is performed, and a plain CT head and shunt radiographs are requested. Which of the following is the most accurate statement about the patient's condition?

Choices: 1. The CT head is likely to diagnose the condition 2. The shunt tap is likely to diagnose the underlying condition 3. This lady needs a lumbar puncture to diagnose the underlying condition 4. The CT head will show ependymal enhancement and hydrocephalus.

Answer: 2 - The shunt tap is likely to diagnose the underlying condition Explanations: CSF in ventriculitis may show a raised cell count and protein, with low glucose. CSF culture may be positive, although this is not always the case, especially is sample is taken following antibiotic administration. The presence of oligoclonal immunoglobulin G or M bands, CSF lactate, procalcitonin, and lysozymes help make an early diagnosis. Non-contrast CT head demonstrates non-specific findings, including dependent hyperdense ventricular debris, hydrocephalus, periventricular low density as well as features of the underlying abnormality. It does not offer a definitive diagnosis. A CSF sample should be obtained. CSF sample can be obtained via a shunt tap, it does not need to be obtained via lumbar puncture. The patient had a plain CT head. Enhancement may be seen on a contrastenhanced scan. A CSF sample will also be required. A wound infection needs to be considered - and examination of her abdomen and cranial wound is imperative. A shunt series radiograph may help rule out a blocked shunt. Comparison of the new and previous CT scans may help decide if the patient has hydrocephalus, which may suggest a blocked shunt. Go to the next page if you knew the correct answer, or click the link image(s) below to further research the concepts in this question (if desired).

Research Concepts: Ventriculitis

We update eBooks quarterly and Apps daily based on user feedback. Please tap flag to report any questions that need improvement.

Question 76: A 70-year-old male with a past medical history of diabetes mellitus, chronic kidney disease, and hypertension recently admitted due to ST-segment elevation myocardial infarction. After five days the patient developed low blood pressure and harsh systolic murmur. Echocardiogram showed ventricular septal rupture (VSR). Intraaortic balloon pump (IABP) has been placed for hemodynamically stability, and cardiac surgery has been called for intervention. How does patient get benefit from IABP device in ventricular septal rupture?

Choices: 1. By decreasing preload 2. By decreasing afterload 3. By increasing heart rate 4. By dilating aorta

Answer: 2 - By decreasing afterload Explanations: VSR is one of the rare but lethal complications of acute myocardial infarction. VSR can present with hemodynamic instability due to cardiogenic shock. Ultimate treatment of VSR is surgical repair with a patch. Due to hemodynamic instability and cardiogenic shock patient needs an IABP as a bridge until the surgery. IABP reduce afterload so decrease left-to-right shunt through VSR. Go to the next page if you knew the correct answer, or click the link image(s) below to further research the concepts in this question (if desired).

Research Concepts: Ventricular Septal Rupture

We update eBooks quarterly and Apps daily based on user feedback. Please tap flag to report any questions that need improvement.

Question 77: A 42-year-old male is stabbed in the left chest during a bar-fight. Emergency medical services (EMS) have arrived on the scene and have found the patient to be hypotensive in hemorrhagic shock. En route to the hospital, the patient goes into cardiac arrest. Paramedics intubate the patient and find him to be in asystole. EMS calls the local emergency department to establish medical command. The medical command clinician is told that EMS is 5 minutes from the hospital and that they are having problems oxygenating and ventilating the patient because there is a lot of resistance when squeezing the bag-valve-mask. Which of the following would be the most appropriate management at this time for the medical command clinician to give to the paramedics?

Choices: 1. Continue to transport to the hospital with no further treatment advised 2. Extubate and reintubate the patient as the endotracheal tube is most likely displaced into the esophagus 3. Perform immediate left-sided needle decompression as the patient may have a tension pneumothorax 4. Perform immediate needle pericardiocentesis as the patient may have pericardial tamponade

Answer: 3 - Perform immediate left-sided needle decompression as the patient may have a tension pneumothorax

Explanations: Patients who present in traumatic cardiac arrest are managed differently with respect to the injury mechanism. Penetrating trauma, which leads to exsanguination, pericardial tamponade, tension hemothorax, or tension pneumothorax, can be treated if done expeditiously. The advanced cardiac life support (ACLS) and advanced trauma life support (ATLS) courses promulgated by the American Heart Association and the American College of Surgeons teach the 5 H's and T's (hypoxia, hypovolemia, hyperkalemia, hydrogen ions, hypothermia, tension pneumothorax, thrombosis, toxins, thromboembolism, tamponade) for potentially reversible causes of asystole. If not identified and treated early and return of spontaneous circulation is restored, patients have a very dismal prognosis. Tension pneumothorax usually develops when the pleura of the chest is violated, and the underlying pulmonary parenchyma is damaged. This leads to air being released between the visceral and parietal pleura creating a true space in which further air leak leads to further lung collapse or pneumothorax. With intubation and bag-mask ventilation, air is exchanged via positive pressure ventilation. This positive pressure leads to rapid expansion of the underlying pneumothorax. When intrapleural pressure exceeds the pressure within the mediastinum, low-pressure structures such as the right atria and ventricle as well as the venous system begin to collapse. Cardiac pre-load is then decreased drastically, which diminishes left heart filling leading to a significant reduction in cardiac output and lower coronary filling pressures during diastole. This, in turn, leads to cardiac ischemia and arrhythmias such as asystole if not corrected. Needle thoracostomy is a rapid way to convert a life-threatening tension pneumothorax into an open pneumothorax correcting the underlying cardiac dysfunction. Needle thoracostomy is performed via a large-bore 18-gauge or large needle at least 2 inches long placed in the mid-clavicular line over the third rib in the second intercostal space. The second rib can be identified as it is adjacent to the sternomanubrial joint, which can often be palpated. The third rib is then identified from there, and the needle is placed over the top of the rib to avoid the neurovascular bundle that runs on the inferior aspect. The needle is inserted until a rush of air is identified, and the tension pneumothorax is corrected evidenced by a return of normal hemodynamics. Alternatively, in heavier set patients or patients with large muscle mass of the anterior chest, needle insertion can be performed by placement at the anterior axillary line using the same landmarks. Ultimately, a formal tube thoracostomy must be performed as this is only a temporizing procedure. Pericardial tamponade has similar underlying pathophysiology as that of a tension pneumothorax except that blood is released into the pericardial sac leading to increased pressures which compress the right heart structures and venous system. Needle pericardiocentesis is an option to alleviate this pressure in an emergent situation, but clinically, the patient would not present with issues in performing bag-mask ventilation and meeting increasing resistance. It is commonly taught that patients with pericardial tamponade present with Beck's triad: jugular venous distension (JVD), muffled heart sounds and narrowed pulse pressure. However, this is seen in only about one-third of patients and is

less likely to be seen in a hypovolemic trauma patient who is in hemorrhagic shock as they will not have high venous pressure to see JVD. Go to the next page if you knew the correct answer, or click the link image(s) below to further research the concepts in this question (if desired).

Research Concepts: Asystole

We update eBooks quarterly and Apps daily based on user feedback. Please tap flag to report any questions that need improvement.

Question 78: A 17-year-old male collapses after being hit in the chest with a lacrosse ball. He has no known past medical history. Which of the following is most likely to improve his long-term outcome?

Choices: 1. Intravenous epinephrine 2. Early defibrillation 3. Implanted cardiac defibrillator 4. Endotracheal intubation

Answer: 2 - Early defibrillation Explanations: Commotio cordis is ventricular fibrillation prompted by direct trauma to the chest wall overlying the heart without structural cardiac injury. Early defibrillation is the most effective means of promoting the return of spontaneous circulation and improving long-term outcomes. Efforts to increase awareness, earlier recognition and intervention, and increased access to automatic external defibrillators have been associated with improved outcomes. An implanted cardiac defibrillator is not indicated for the treatment of commotio cordis with a structurally normal heart. Successful reversal of the ventricular fibrillation from commotio cordis does not appear to increase the long-term risk for adverse cardiac events. However, there may be secondary ischemic injury from decreased cardiac output during the fibrillatory period, especially if resuscitation is delayed. Go to the next page if you knew the correct answer, or click the link image(s) below to further research the concepts in this question (if desired).

Research Concepts: Commotio Cordis

We update eBooks quarterly and Apps daily based on user feedback. Please tap flag to report any questions that need improvement.

Question 79: A patient on 4 liters of oxygen by nasal cannula has an arterial blood gas performed. The patient has normal work of breathing, with a respiratory rate of 23. The arterial blood gas indicates a pH of 7.40, PaO2 = 92 mmHg, and PaCO2 = 42 mmHg. Which of the following is most appropriate?

Choices: 1. Decrease the oxygen to 2 liters 2. Continue present management and record the data in the chart 3. Increase the oxygen to 6 liters 4. Encourage the patient to breathe faster

Answer: 2 - Continue present management and record the data in the chart Explanations: The arterial blood gas shows the patient is receiving adequate treatment, and no changes are necessary. A normal physiologic range of PaO2 is 75-100 mmHg. If someone is requiring supplemental oxygen you need to asses WHY, instead of ONLY increasing the oxygen. There are multiple reasons for hypoxemia: V/Q mismatch, right-toleft shunt, diffusion impairment, hypoventilation, and low Fraction of inspired O2. at sea level, low inspired O2 is not the cause for hypoxemia. You can evaluate quickly an intubated patient with hypoxemia using the pneumonic DOPE. Displacement (moved endotracheal tube), Obstruction (secretions or object to be suctioning or removed from the airway), Pneumothorax, Equipment failure (detached ventilator, wrong ventilator settings, malfunctioning ventilator, malfunctioning surveillance equipment). Go to the next page if you knew the correct answer, or click the link image(s) below to further research the concepts in this question (if desired).

Research Concepts: Arterial Blood Gas

We update eBooks quarterly and Apps daily based on user feedback. Please tap flag to report any questions that need improvement.

Question 80: A previously healthy 17-year-old boy presents to the emergency department with a high-grade fever, malaise, neck rigidity, and a petechial skin rash. He reveals that he had developed an upper respiratory tract infection three days ago. Today, in the emergency department, he has a blood pressure of 70/40 mmHg, a heart rate of 120 beats per minute. Further evaluation with labs show a white blood cell count of 22,000 per microliter, sodium 120 mEq/L, potassium 5.8 mEq/L, chloride 89 mEq/L, blood urea nitrogen 34 mg/dL, creatinine 2.0 mg/dL, and glucose 50 mg/dL. Which of the following is most likely to be associated with his condition?

Choices: 1. Disseminated intravascular coagulation 2. Isolated thrombocytopenia 3. Respiratory acidosis 4. Metabolic alkalosis

Answer: 1 - Disseminated intravascular coagulation Explanations: The patient has hypotension with hyperkalemia, hyponatremia, and hypochloremia. These laboratory abnormalities and petechial skin rash suggest that he has WaterhouseFriderichsen syndrome (WFS). The syndrome is caused by meningococcus. Disseminated intravascular coagulation is a complication associated with meningococcemia. Other conditions associated with meningococcemia petechial rash, neurologic manifestations, and pupura fulminans. Metabolic acidosis due to adrenal insufficiency is seen on arterial blood gas analysis. Go to the next page if you knew the correct answer, or click the link image(s) below to further research the concepts in this question (if desired).

Research Concepts: Waterhouse-Friderichsen Syndrome

We update eBooks quarterly and Apps daily based on user feedback. Please tap flag to report any questions that need improvement.

Question 81: A 23-year-old man marathon runner, who was participating in a triathlon, presented to the emergency department for nausea and vomiting. In the emergency department, his vital signs showed a heart rate of 140/min and blood pressure 95/76 mmHg. Clinical examination showed dry lips, mouth, and skin. His labs showed a white blood cell count of 7,000 cells/mm3, hemoglobin 15 g/dL, platelets 450,000/microL, sodium 141 mEq/L, potassium 5.1 mmol/L, creatinine 5 mg/dL, CO2 17 mEq/L. He was given one-liter fluid bolus and discharged from the emergency department. The following day he came again to the hospital because of worsening vomiting and bloody urine. His repeated labs show a white blood cell count of 12,000 cells/mm3, hemoglobin 16 g/dL, platelets 456,000/microL, sodium 142 mEq/L, potassium 5 mmol/L, creatinine 2.0 gm/dL, CO2 15 mEq/L. Urinalysis shows a red color, pH 4.8, 2-3 RBCs, blood positive, nitrate negative and no bacteria. He is admitted to the hospital and give IV fluids. A few days later, he became hypoxic and was eventually intubated. Clinical examination reveals elevated jugular venous pressure, bilateral crackles and bilateral pitting edema. Which of the following best explains his recent decline in cardiac function?

Choices: 1. Acute kidney injury 2. Vasculitis 3. Ischemia secondary to hypotension 4. Hematuria

Answer: 1 - Acute kidney injury Explanations: This patient has type 3 cardiorenal syndrome which is defined as a sharp decline in renal function that results in a sharp reduction in cardiac function. The pathophysiology of type 3 cardiorenal syndrome is still not completely understood. However, it can be due to a direct effect of acute kidney injury on the heart through releasing immunomodulators like interleukin-1 and tumor necrosis factor which will induce cardiac apoptosis. The other possibility that cardiac injury can be due to the direct toxic accumulation from kidney failure (uremic cardiomyopathy). The patient has hematuria secondary to rhabdomyolysis, and it is unlikely due to systemic illness like vasculitis. Moreover, ischemic cardiac disease secondary to hypotension is unlikely in a healthy adult patient with mean arterial pressure at 75. Go to the next page if you knew the correct answer, or click the link image(s) below to further research the concepts in this question (if desired).

Research Concepts: Cardiorenal Syndrome

We update eBooks quarterly and Apps daily based on user feedback. Please tap flag to report any questions that need improvement.

Question 82: A 67-year-old patient with a history of coronary artery disease and COPD, on chronic steroid therapy, is admitted for coronary artery bypass graft. On the same day, the patient self extubates in the ICU with the endotracheal tube cuff still inflated. Subsequently, he develops hoarseness and dysphagia. Which of the following is the most likely reason for the development of hoarseness and dysphagia?

Choices: 1. Chronic steroid therapy 2. Traumatic extubation with the endotracheal tube still inflated 3. COPD 4. Coronary artery bypass graft surgery

Answer: 2 - Traumatic extubation with the endotracheal tube still inflated Explanations: Arytenoid subluxation is a rare complication that typically occurs after traumatic injury to the cricoarytenoid joint during laryngoscopy and intubation. They are usually associated with prolonged and traumatic intubation in cases of unanticipated difficult airways. Chronic steroid use may weaken the cricoarytenoid joint capsule and predispose to this complication in the event of traumatic intubation. Direct trauma by the laryngoscope during intubation is the proposed mechanism of trauma for anteromedial arytenoid subluxation. Posterolateral displacement of the arytenoid cartilage by a partially or fully inflated cuff is the proposed mechanism of trauma for posterolateral arytenoid subluxation and is the most likely explanation for the hoarseness in this patient. In rare instances, a history of severe cough, or spontaneous events has been reported. This patient had a history of COPD but no COPD exacerbation was reported so is unlikely to be the cause of the hoarseness and dysphagia. Major cardiac surgery involving the use of a transesophageal echocardiography probe may increase the incidence of arytenoid subluxation. Coronary artery bypass graft surgery by itself is not a risk factor for arytenoid subluxation. Go to the next page if you knew the correct answer, or click the link image(s) below to further research the concepts in this question (if desired).

Research Concepts: Arytenoid Subluxation

We update eBooks quarterly and Apps daily based on user feedback. Please tap flag to report any questions that need improvement.

Question 83: A 65-year-old homeless male is admitted with the chief complaints of fever, shortness of breath, and weight loss. He has been having a productive cough with hemoptysis off and on for one month. He is a known alcoholic. On examination, he is febrile with a blood pressure of 110/80 mmHg, pulse 98/min regular, respiratory rate 24/min and oxygen saturation of 90% on room air. On auscultation, there are coarse crackles bilaterally but more prominent on the right side. You notice that there is a draining sinus on the chest wall. A chest radiograph shows pulmonary infiltrates and a few cavitary lesions. Dark brown granules are seen on a frozen section. Gram stain shows purple-staining fine filaments. CBC shows leukocytosis with elevated neutrophils. C- reactive protein and erythrocyte sedimentation rate are elevated. Which of the following is used to treat this condition?

Choices: 1. Incision and drainage 2. Radiation therapy 3. High-dose penicillin 4. Amphotericin

Answer: 3 - High-dose penicillin Explanations: Dark brown granules can be sulfur and represent actinomycosis. Histologically, sulfur granules appear black. Actinomyces can be cultured. The preliminary diagnosis usually is made by looking for sulfur granules with microscopy. Imaging is usually needed to determine the extent of infection. The gold standard test is a histological examination and bacterial culture of a biopsy which can be obtained with bronchoscopy (pulmonologist), a CT-guided biopsy (interventional radiologist), or video-assisted thoracoscopy surgery (VATS) performed by a thoracic surgeon. Anaerobic cultures of pleural fluid almost never grow the organism. Sputum culture will not be useful unless the patient has a cavitary disease. The imaging for actinomycosis is nonspecific. In the acute presentation, it can look like any other pneumonia. In chronic forms, it can present as a pulmonary mass or can cavitate. In the case of a mass malignancy, Actinomycosis is an important diagnosis to consider. Especially when it cavitates, tuberculosis is an important differential diagnosis to consider. The computed tomography scan findings can vary depending upon the duration of illness and can include consolidation, cavitation, pleural effusion, lymph node enlargement, atelectasis, and ground glass opacification. Treatment is with high-dose penicillin. Go to the next page if you knew the correct answer, or click the link image(s) below to further research the concepts in this question (if desired).

Research Concepts: Actinomycosis

We update eBooks quarterly and Apps daily based on user feedback. Please tap flag to report any questions that need improvement.

Question 84: A 65-year-old woman status post-three-vessel coronary artery bypass grafting (CABG), and transcatheter aortic valve replacement (TAVR) is under the care of a critical care team in the intensive care unit. She was put on a ventilator due to respiratory insufficiency. The ventilator settings include a fraction of inspired oxygen (FiO2) of 70%, positive end-expiratory pressure (PEEP) of 20 cm H2O, rate of 12 breaths/min. She loses pulse suddenly. The electrocardiogram (EKG) shows narrow QRS complexes. There is absent breath sounds on the left side with tracheal deviation on the right side. Which of the following is the next step in management after initiating cardiopulmonary resuscitation (CPR)?

Choices: 1. Transcutaneous pacemaker 2. Needle decompression 3. Pericardiocentesis 4. Surgical evaluation

Answer: 2 - Needle decompression Explanations: Patients on high PEEP are at risk of ventilator-associated barotrauma. This patient most likely had a tension pneumothorax induced pulseless electrical activity (PEA) due to high PEEP. The best step is to decompress the chest with a needle. It can then be converted to a chest tube if necessary. A pacemaker is used in case of bradyarrhythmias or to pace rhythms. It does not have a role in treating PEA. Pericardiocentesis would be a good option if the patient had PEA due to tamponade. This would show raised jugular venous pressure, hypotension, and muffled heart sounds. Surgical evaluation can be done after the patient is stabilized, and if the cause of PEA cannot be fixed with supportive medical treatment, for example, in case of a massive pulmonary embolus. Go to the next page if you knew the correct answer, or click the link image(s) below to further research the concepts in this question (if desired).

Research Concepts: Pulseless Electrical Activity

We update eBooks quarterly and Apps daily based on user feedback. Please tap flag to report any questions that need improvement.

Question 85: A 75-year-old-female underwent percutaneous coronary intervention in the left anterior descending artery after presenting with abdominal pain for two days. Two hours after her procedure she developed an accelerated idioventricular rhythm. Medications are aspirin, clopidogrel, metoprolol, atorvastatin, and lisinopril. The patient denies any active complaints and hemodynamically stable. What should be the possible intervention?

Choices: 1. Add calcium channel blocker 2. Increase beta blocker 3. Defibrillator 4. No intervention needed

Answer: 4 - No intervention needed Explanations: Accelerated idioventricular rhythm is a common complication within 24 hours of reperfusion. The patient is on optimum medical therapy so there is no need to add another medication. She is tolerating the rhythm well, so there is no need to increase the beta-blocker dose. Accelerated idioventricular rhythm is a benign rhythm after reperfusion so no defibrillation is needed. Accelerated idioventricular rhythm (AIVR) is due to abnormal automaticity in subendocardial Purkinje fibers and is observed in 15% of patients undergoing reperfusion. There is no further intervention needed. Go to the next page if you knew the correct answer, or click the link image(s) below to further research the concepts in this question (if desired).

Research Concepts: Silent Myocardial Ischemia

We update eBooks quarterly and Apps daily based on user feedback. Please tap flag to report any questions that need improvement.

Question 86: An adult patient presents with the worst headache of her life. A noncontrast CT of the head is negative. A lumbar puncture produces four bloody tubes, each with RBC counts greater than 100,000/mm3. What is the next step in management?

Choices: 1. Repeat a noncontrast head CT the next day 2. Perform a CT angiogram 3. Perform a head CT with contrast 4. Administer mannitol

Answer: 2 - Perform a CT angiogram Explanations: A noncontrast head CT in the context of a nontraumatic, bloody tap will be positive in the vast majority of patients with subarachnoid hemorrhage. However, a normal CT does not mean a bloody tap can be ignored. Unlike a traumatic tap, the blood cell count does not diminish from the first to the fourth tube. Xanthochromia is the yellow appearance of cerebrospinal fluid caused by the degradation of heme to bilirubin confirming a hemorrhage. CT or MR angiography is needed to detect vascular pathology that might account for the lumbar puncture findings of subarachnoid hemorrhage. Initial management consists of preventing vasospasm, blood pressure control, fluid management, anticonvulsant therapy, pain control, and many neurosurgeons recommend early surgery. Go to the next page if you knew the correct answer, or click the link image(s) below to further research the concepts in this question (if desired).

Research Concepts: Subarachnoid Hemorrhage

We update eBooks quarterly and Apps daily based on user feedback. Please tap flag to report any questions that need improvement.

Question 87: A 65-year-old female has a left hemicolectomy for cancer. On postoperative day seven, she develops a temperature of 39.2 C. Physical examination shows clear lungs, no evidence of wound or line infections, and her calves are soft and nontender. Chest radiograph shows mild bibasilar atelectasis. Two days later, blood cultures grow Bacteroides fragilis and Escherichia coli, while urine culture is negative. Which of the following is the most appropriate approach to this patient?

Choices: 1. Start triple antibiotics empirically 2. Computed tomography scan of the abdomen and pelvis 3. Repeat blood cultures 4. Replace all lines

Answer: 2 - Computed tomography scan of the abdomen and pelvis Explanations: The evidence of polymicrobial bacteremia indicates that the patient likely has an intraabdominal abscess. Postoperative intraabdominal abscess imaging is best after postoperative day seven because tissue edema is reduced and nonsuppurative fluids are reabsorbed. If present and accessible, percutaneous CT-guided drainage is indicated. Ultrasound can sometimes reveal intraabdominal abscesses, but exams can be limited by obesity, dressings, open wounds, stomas, and intervening viscera. Go to the next page if you knew the correct answer, or click the link image(s) below to further research the concepts in this question (if desired).

Research Concepts: Abdominal Abscess

We update eBooks quarterly and Apps daily based on user feedback. Please tap flag to report any questions that need improvement.

Question 88: A 32-year-old male presents for complaints of headache, dizziness, and generalized weakness, that started this afternoon but is progressively worsening. He works as a painter and was operating a gas-powered paint sprayer indoors for approximately 10 hours today. He is alert, oriented, and has an unremarkable physical exam. He reports no medical problems and smokes 1 pack of cigarettes per day as well as alcohol use socially. Carboxyhemoglobin level obtained via blood gases is 12%. Which of the following is the next best step in the management of this patient?

Choices: 1. Transfer to a hyperbaric treatment center 2. Treatment with high-flow oxygen until carboxyhemoglobin level is 3% 3. Treatment with high-flow oxygen for 6 hours 4. Cardiac monitoring, ECG, and troponin in addition to normobaric oxygen

Answer: 3 - Treatment with high-flow oxygen for 6 hours Explanations: Current recommendations for asymptomatic patients without high-risk features is treatment with normobaric oxygen for 6 hours. Oxygen competes with carbon monoxide for binding at the heme group of hemoglobin. The half-life of carboxyhemoglobin in room air is 4 hours or more. However, this is substantially decreased to just 40 to 80 minutes with high-flow oxygen. By increasing the rate of oxygen flow up to 60 liters per hour, the high concentration of oxygen molecules can displace carbon monoxide from the hemoglobin molecule more readily, enhancing elimination. Though hyperbaric oxygen hastens elimination twice as fast, high-flow oxygen is more accessible, easier to initiate, and safer with much less concern for barotrauma. Treatment with normobaric high-flow oxygen would be appropriate for this patient. Recommendations currently suggest considering hyperbaric treatment in those with neuropsychiatric complaints, acute coronary syndrome, those who are comatose or have an altered level of consciousness, pregnant women, anyone over the age of 65, and those with initial carboxyhemoglobin levels greater than 25%. Hyperbaric oxygen is not the standard of care in any particular scenario, and any decision to initiate this should be made in conjunction with a toxicologist and/or Poison Control, taking into consideration the patient's age, comorbidities, mental status, as well as feasibility and distance from a hyperbaric center. Despite hastening carboxyhemoglobin elimination by decreasing the half-life of carboxyhemoglobin to just 23 minutes, the use of hyperbaric oxygen has not been translated into improved outcomes or decreased mortality. Of note, hyperbaric oxygen may cause barotrauma, pulmonary edema, and seizures as well as potentially increase oxidative stress and free radical production. Although this patient should certainly receive high-flow oxygen, the appropriate management for asymptomatic patients and without high-risk features (chest pain, loss of consciousness, altered level of consciousness, focal neurologic deficits) is treatment for 6 hours or until levels normalize. A small amount of carbon monoxide is produced endogenously with the metabolism of heme, so a baseline carboxyhemoglobin level is thought to be around 1 to 3 percent in non-smokers and as high as 10 percent in smokers. For this reason, it would be not reasonable to treat the above patient with a goal of 3 percent since their baseline is likely higher. ECG and troponin are not routinely obtained in carbon monoxide toxicity but are warranted in the presence of angina, dyspnea, or palpitations. Management should always include a carboxyhemoglobin level obtained via blood gases, as well as a pregnancy test in females since this may change management. Imaging with a chest x-ray or head CT is frequently obtained to rule out other causes but is not routinely necessary. In cases of non-accidental poisonings, a full toxicology panel including salicylate/acetaminophen levels and urine drug screen is recommended. Go to the next page if you knew the correct answer, or click the link image(s) below to further research the concepts in this question (if desired).

Research Concepts: Carboxyhemoglobin Toxicity

We update eBooks quarterly and Apps daily based on user feedback. Please tap flag to report any questions that need improvement.

Question 89: A registered nurse is brought to the emergency department after a seizure. She was found unresponsive with an undetectable blood sugar. Two ampules of 50% glucose were administered with the elevation of her blood sugar to 42 mg/dL. She has a history of depression. Which of the following finding would be consistent with exogenous insulin overdosage?

Choices: 1. Plasma insulin greater than 18 pmol/L, plasma glucose less than 55 mg/dL, and C-peptide greater than 0.6 ng/ml 2. Plasma insulin greater than 18 pmol/L, plasma glucose less than 55mg/dL, and C-peptide greater than 0.6 ng/ml 3. Plasma insulin greater than 18 pmol/L, plasma glucose less than 55 mg/dL, and C-peptide undetectable 4. Plasma insulin less than 18 pmol/L, Plasma glucose less than 55 mg/dL, and C-peptide undetectable

Answer: 3 - Plasma insulin greater than 18 pmol/L, plasma glucose less than 55 mg/dL, and C-peptide undetectable

Explanations: Exogenous insulin administration should be considered in patients with profound hypoglycemia and no history of diabetes mellitus. The measurement of C-peptide and insulin level is useful in differentiating between exogenous and endogenous insulin administration. The C-peptide is a marker for endogenous insulin. The C-peptide levels will be low in patients with insulin overdose and high in patients with hypoglycemia resulting from an insulin secretagogue overdose. The three needed tests are plasma glucose, plasma insulin, and C-peptide level. C-peptide levels less than 0.6 ng/mL would suggest exogenous hyperinsulinemia if plasma insulin measurement is high and hypoglycemia diagnosed. Go to the next page if you knew the correct answer, or click the link image(s) below to further research the concepts in this question (if desired).

Research Concepts: Insulin

We update eBooks quarterly and Apps daily based on user feedback. Please tap flag to report any questions that need improvement.

Question 90: A 28-year-old female presents with shortness of breath for 3 days, which is progressively getting worse. In the emergency department, the blood workup shows a white cell count of 14,000, with 78% neutrophils, 78%, and 11% eosinophils. Basic metabolic profile shows sodium of 136 mmol/L, potassium 4.1 mmol/L, chloride 100 mmol/L, and bicarbonate 20 mmol/L. The patient has seasonal allergies, and family history is positive for asthma in her mother. The chest x-ray is normal, EKG shows sinus tachycardia. Arterial blood gas (ABG) shows pH of 7.2, PCO2 of 60, and bicarbonate of 30. Initially, the patient was given albuterol and Ipratropium nebulization and intravenous corticosteroids, was put on BiPAP, but she did not improve. Her shortness of breath increased, and she was using accessory muscles. She is intubated and placed on a ventilator with settings AC mode tidal volume of 400, rate of 24, PEEP of 5, and FiO2 40%. ABG was drawn after 1 hour, which shows pH 7.16, PCO2 66, and bicarbonate of 25. What is the acid-base disorder patient has on last ABG, and how should the ventilator settings be adjusted?

Choices: 1. The patient has metabolic acidosis and the respiratory rate should be increased to 26, flow should be decreased 2. The patient has respiratory acidosis with metabolic compensation and respiratory rate should be lowered to 20, the flow should be increased 3. The patient has respiratory and metabolic acidosis. The respiratory rate and flow rate should not be changed 4. The patient has metabolic alkalosis with respiratory acidosis and respiratory rate should be increased to 30 and flow should be increased

Answer: 2 - The patient has respiratory acidosis with metabolic compensation and respiratory rate should be lowered to 20, the flow should be increased

Explanations: The patient has respiratory acidosis with metabolic compensation. The respiratory rate should be lowered to 20, flow should be increased. Respiratory acidosis is a state in which there is usually a failure of ventilation and an accumulation of carbon dioxide. The primary disturbance of elevated arterial PCO2 decreases the ratio of bicarbonate to arterial PCO2, which leads to a lowering of the pH. In the presence of alveolar hypoventilation, two features commonly seen are respiratory acidosis and hypercapnia. The patient's acidosis worsened as she developed auto-peep. To correct auto-peep her I:E ratio has to be adjusted. Lower the respiratory rate which will give her more expiratory time. Increasing the flow will also help in getting more expiratory time and will help in acidosis. ABG should be repeated in 30 to 45 minutes. Carbon dioxide plays a remarkable role in the human body mainly through its role in pH regulation of the blood. It is the primary stimulus to initiate ventilation. In its normal state, the body maintains CO2 in a well-controlled range from 38 to 42 mmHg by balancing its production and elimination. Go to the next page if you knew the correct answer, or click the link image(s) below to further research the concepts in this question (if desired).

Research Concepts: Respiratory Acidosis

We update eBooks quarterly and Apps daily based on user feedback. Please tap flag to report any questions that need improvement.

Question 91: An unidentified middle-aged male is brought to the hospital after being found in a pool of his vomit at the side of the road along with a "will work for food" sign alongside him. He has good chest excursions and pulse. Vital signs are within limits, and fingerstick glucose is found to be 45 mg/dL. Intravenous access is obtained, and he is started on intravenous saline and dextrose along with multivitamins. Routine lab work including toxicologic studies is sent. He wakes up groggily and admits that he has been a long-term alcohol user and had a binge last night as well. He is cachectic on exam with temporal hollowing but without any evidence of chronic liver disease including jaundice or ascites. Suddenly, the patient starts to have generalized tonic-clonic contractions, which are terminated by intravenous benzodiazepines but the patient goes into a coma, becoming unresponsive but able to maintain his airway. Lab tests show calcium level of 8 mEq/dL, magnesium level of 1.8 mg/dL, potassium level of 2.7 mEq/dL, and phosphorus level of 1.5 mg/dL. Which electrolyte abnormality most likely can be attributed to the clinical presentation and complications?

Choices: 1. Calcium 2. Magnesium 3. Potassium 4. Phosphorus

Answer: 4 - Phosphorus Explanations: The most likely cause of the patient's seizure and coma is hypophosphatemia. He has risk factors including prolonged malnutrition and alcohol use disorder and goes into tonic-clonic contractions on administration of glucose which stimulates insulin release which causes further worsening of hypophosphatemia by cellular uptake as is seen in refeeding syndrome. Calcium abnormalities would more often be associated with tetany, cardiac arrhythmias instead of seizures and coma and less likely precipitated by intravenous dextrose. There are risk factors including chronic alcohol use disorder, refeeding in the patient but magnesium levels are within acceptable limits and hypomagnesemia would more likely present with tetany, cardiac arrhythmias from resultant hypocalcemia and also respiratory failure. Potassium abnormalities would more often be associated with cardiac arrhythmias and myopathy instead of seizures and coma. Go to the next page if you knew the correct answer, or click the link image(s) below to further research the concepts in this question (if desired).

Research Concepts: Ketoacidosis

We update eBooks quarterly and Apps daily based on user feedback. Please tap flag to report any questions that need improvement.

Question 92: A senior man living in an assisted living facility came to the emergency department with cough and fever. He is diagnosed with pneumonia. Blood and sputum specimens were sent for laboratory testing. Empiric therapy started at that time. Now the sputum culture is back and positive with P. aeruginosa. Now therapy should be changed from previous empiric therapy to which of the following?

Choices: 1. Ampicillin and clavulanic acid 2. Azithromycin 3. Vancomycin 4. Piperacillin and tazobactam

Answer: 4 - Piperacillin and tazobactam Explanations: The senior man with cough and fever is most likely has bacterial pneumonia. He is living in an assisted living facility so most likely has healthcare-associated pneumonia. One of the most common causes of healthcare-associated pneumonia is Pseudomonas aeruginosa. Antipseudomonal therapy, for example, piperacillin and tazobactam, is the recommended therapy for this infection. Go to the next page if you knew the correct answer, or click the link image(s) below to further research the concepts in this question (if desired).

Research Concepts: Bacterial Pneumonia

We update eBooks quarterly and Apps daily based on user feedback. Please tap flag to report any questions that need improvement.

Question 93: A 56-year old female is admitted to the intensive care unit (ICU) due to hypotension and bleeding after she was involved in a motor vehicle accident. An ultrasound is performed, which shows splenic rupture. A laparotomy is carried out successfully, and the patient is stabilized. On the 3rd post-operative day, the patient complains of difficulty breathing, and her abdomen is tense and distended. The patient is tachycardic, hypertensive, and has a bladder pressure of 24 cm. Which of the following is the most appropriate next step in the management of this patient?

Choices: 1. Nasogastric tube placement 2. Avoidance of muscle relaxants 3. Head elevation of more than 30 degrees 4. Inotropic support to achieve a perfusion pressure of 60 mmHg in the abdomen

Answer: 1 - Nasogastric tube placement Explanations: Non-surgical therapeutic options for the treatment of intra-abdominal hypertension involves an overall goal to improve abdominal wall compliance. The goals of treatment are to decrease muscle contractions, evacuate luminal contents by decompression, and evacuate abdominal fluid by drainage. Correction of positive fluid balance through goal-directed volume resuscitation, along with placement of the nasogastric tube can aid in decompressing the abdomen. Muscle relaxants are a helpful adjunct in the conservative management of abdominal hypertension. Go to the next page if you knew the correct answer, or click the link image(s) below to further research the concepts in this question (if desired).

Research Concepts: Abdominal Compartment Syndrome

We update eBooks quarterly and Apps daily based on user feedback. Please tap flag to report any questions that need improvement.

Question 94: A patient with a history of cirrhosis of the liver presents with shortness of breath in a standing position which gets better in a lying down position. A chest x-ray is negative for pleural effusion. Ultrasound abdomen reveals mild ascites. Which of the following mechanisms is most likely responsible for the patient's symptoms?

Choices: 1. Impaired metabolism of body hormone 2. Impairment in the urea cycle 3. Increased portal pressure 4. Low serum albumin concentration

Answer: 1 - Impaired metabolism of body hormone Explanations: Patients with cirrhosis of the liver are prone to a hyperestrogenic state due to impaired estrogen metabolism. It leads to a hyperestrogenic state with the formation of arteriovenous malformation in the lungs, which leads to platypnea and orthodeoxia syndrome. Cardiac catheterization is indicated to confirm the diagnosis. The impaired urea cycle leads to hyperammonemia which does not cause shortness of breath. Increased portal pressure causes ascites, but the US abdomen reveals mild ascites, which does not explain her symptoms. Low albumin contributes to ascites which is mild in this patient. It is less likely to cause shortness of breath. Go to the next page if you knew the correct answer, or click the link image(s) below to further research the concepts in this question (if desired).

Research Concepts: Pulmonary Arteriovenous Malformation

We update eBooks quarterly and Apps daily based on user feedback. Please tap flag to report any questions that need improvement.

Question 95: A 66-year-old male is admitted to the hospital for abdominal pain, diarrhea, and fever of 2 days duration. The patient underwent stem cell transplantation three weeks ago. He denies blood in the stool. On physical exam, his temperature was 38.5 C (101.3 F), blood pressure 109/70 mmHg, pulse rate 109/mon, and respiratory rate 16/min. Abdominal exam reveals tenderness in the left lower quadrant. His labs show hemoglobin (Hb) 7 mg/dL, white blood cells (WBCs) 400/micoL, platelets 22000/micoL, and creatinine 1.3 mg/dL. The patient was seronegative for CMV before transplant. Computed tomography (CT) scan shows thickening of the bowel wall of the descending colon. What is the next best step in the management of this patient?

Choices: 1. Granulocyte infusion 2. Stool culture 3. Chest CT scan 4. Testing for Clostridium difficile

Answer: 4 - Testing for Clostridium difficile Explanations: The patient presented with signs and symptoms concerning for colitis. He had a recent stem cell transplant, and he is immunosuppressed and neutropenic. He needs prompt workup and management to prevent complications. The CT findings confirm the diagnosis of colitis. The cause of colitis still needs to be determined. Given his neutropenia, the patient might have had neutropenic enterocolitis (NE). However, the location of inflammation on CT scan (descending colon) is not typical for NE, but it still can occur anywhere in the colon. However, before diagnosing NE, workup should be done to rule out other causes of colitis that include infectious causes. Stool panel, stool culture, and clostridium difficile testing should be done. Clostridium difficile gives results faster than stool culture, and early testing for Clostridium difficile allows earlier intervention with appropriate antibiotics. Some studies suggest granulocyte infusion for the treatment of neutropenic enterocolitis. However, to diagnose neutropenic enterocolitis, one must rule out other causes of colitis first such as clostridium difficile infection which can give a similar presentation. Stool culture is part of the workup done for a patient presenting with colitis symptoms, especially in the setting of immune suppression such as the patient in the question. However, it takes a long time for the results to come back as compared to Clostridium difficile testing. Early testing for Clostridium difficile allows earlier intervention with appropriate antibiotics. The main concern in this patient is colitis, which is likely the source of his symptoms and signs. There is no evidence for a lung infection. Go to the next page if you knew the correct answer, or click the link image(s) below to further research the concepts in this question (if desired).

Research Concepts: Neutropenic Enterocolitis (Typhlitis)

We update eBooks quarterly and Apps daily based on user feedback. Please tap flag to report any questions that need improvement.

Question 96: A patient who is debilitated presents to the emergency department with fevers and general malaise. A two-view chest x-ray shows a mass in the periphery of the right lung with an air-fluid level. The patient is started on intravenous antibiotics. Over the next 2 days, the patient looks more toxic and shows signs of sepsis. The lung mass appears to have increased and the patient is not improving. What is the most appropriate next step in management?

Choices: 1. Thoracotomy 2. Video-assisted thoracoscopic surgery 3. Percutaneous decompression 4. Infiltration of the mass with antibiotics

Answer: 3 - Percutaneous decompression Explanations: Percutaneous tube drainage has been used for drainage in patients with an expanding lung abscess, signs of unremitting sepsis, abscess under tension, failure to wean from mechanical ventilation, and if there has been contamination of the contralateral lung. In such a case, percutaneous decompression is the best procedure. Studies have shown that the incidence of empyema after percutaneous drainage is low and that patients improve significantly after decompression. When these patients need surgery, a double lumen tube should be used to prevent contamination of the unprotected lung. Go to the next page if you knew the correct answer, or click the link image(s) below to further research the concepts in this question (if desired).

Research Concepts: Lung Abscess

We update eBooks quarterly and Apps daily based on user feedback. Please tap flag to report any questions that need improvement.

Question 97: A 58-year-old patient presents to the emergency department via emergency medical services following an apartment building fire. On primary survey, he is unconscious, does not appear in respiratory distress but appears pale. The patient's vitals are blood pressure 110/65 mmHg, oxygen saturation 98% on room air, temperature 38.3 C (100.9 F), respiration rate 18/min, and pulse 110 bpm. There are no external burn injuries noted, and physical exam reveals no wheezing, no stridor, and no rhonchi but does show bright red retinal veins. Which of the following is the most important next step?

Choices: 1. CT head without contrast 2. Emergent intubation and ventilation with 100% oxygen 3. Hydroxocobalamin 4. Initiate 100% FiO2 via nonrebreather

Answer: 2 - Emergent intubation and ventilation with 100% oxygen Explanations: Carbon monoxide (CO) poisoning should be considered in all patients exposed to fires in confined spaces, especially those patients with complaints of delirium, altered consciousness, hallucinations, fatigue, chest pain, or abdominal pain. Physical examination findings suggesting CO poisoning are often nonspecific but include tachycardia, hyperthermia, and pallor. "Cherry red" skin is a rare finding and most often seen postmortem. Bright red retinal veins are an early finding. Protection of the airway is of paramount importance in possible smoke inhalation injuries. This patient is unconscious, thus having a Glasgow coma scale of less than 8. He should be intubated and treated with 100% oxygen. Signs of upper airway edema and injury include carbonaceous sputum, stridor, burns around the face, and singed facial hair. Rapid intervention and delivery of oxygen is the basis of treatment for CO poisoning since there is competition with oxygen to bind with hemoglobin. The binding of CO to hemoglobin is reversible with high oxygen concentrations. Hyperbaric oxygen therapy, if available, should be considered in patients suspected of having carbon monoxide poisoning, but treatment should not be delayed if hyperbaric therapy is not readily available. Go to the next page if you knew the correct answer, or click the link image(s) below to further research the concepts in this question (if desired).

Research Concepts: Inhalation Injury

We update eBooks quarterly and Apps daily based on user feedback. Please tap flag to report any questions that need improvement.

Question 98: A 50-year-old man is brought by emergency medical services (EMS) after sustaining a gunshot injury. He has a past medical history of type 2 diabetes mellitus, coronary artery disease, tobacco smoking, and chronic obstructive pulmonary disease. He undergoes an exploratory laparotomy, during which he received 5 liters of IV normal saline during the surgery. Soon after extubation, he develops acute respiratory distress. He is tachycardic. Lung auscultation reveals bibasilar rales. Arterial blood gas analysis on room air reveals a PaO2 of 59, pH 7.47, PaCO2 36, and bicarbonate 21. His arterial blood gas does not improve with the administration of 100% oxygen. What is the most likely cause of his respiratory distress?

Choices: 1. Excessive anesthesia 2. Pulmonary edema 3. Pneumonia 4. Bronchial asthma exacerbation

Answer: 2 - Pulmonary edema Explanations: This patient has hypoxemia, and since he has an elevated A-a gradient leaves only shunting and V/Q mismatch as a potential cause for his condition. His failure to improve with 100% oxygen makes V/Q mismatch less possible, as might occur in the case of pulmonary embolism or exacerbation of bronchial asthma. Shunting would explain his hypoxemia. Pulmonary edema, pneumonia, and vascular shunt are frequent causes of shunting. His many risk factors make pulmonary edema the most likely explanation for his arterial blood gases abnormalities and rales on the pulmonary exam. Go to the next page if you knew the correct answer, or click the link image(s) below to further research the concepts in this question (if desired).

Research Concepts: Hypoxia

We update eBooks quarterly and Apps daily based on user feedback. Please tap flag to report any questions that need improvement.

Question 99: A 67-year-old male is admitted with fever, general malaise, headaches, and myalgias. He returned from a hiking trip in Connecticut 1 week ago. The patient is placed on doxycycline therapy for possible anaplasmosis. Two days after admission he develops hypotension and hemolysis. What is the next step in management?

Choices: 1. Increase doxycycline dose 2. Add intravenous vancomycin and cefepime for possible bacteremia 3. Send a peripheral smear and start clindamycin and quinine therapy 4. Consult hematology for hemolysis evaluation

Answer: 3 - Send a peripheral smear and start clindamycin and quinine therapy Explanations: The Ixodes tick transmits both anaplasmosis and babesiosis. Both diseases share the same geographic distribution, and patients can be co-infected with both. Anaplasmosis is treated with doxycycline, whereas Babesia is treated with clindamycin and quinine in severe cases, such as this one. Patients older than 60 years or splenectomized patients can have a severe presentation of babesiosis. This patient is older than 60 years and was initially started on doxycycline for possible anaplasmosis. Given the decline in his clinical status, he should be evaluated for co-infection with Babesia and be initiated on proper therapy. A peripheral smear should be obtained to look for the parasite Babesia inside red blood cells. Anaplasma can be diagnosed by identifying the presence of morulae inside neutrophils. Go to the next page if you knew the correct answer, or click the link image(s) below to further research the concepts in this question (if desired).

Research Concepts: Anaplasma Phagocytophilum

We update eBooks quarterly and Apps daily based on user feedback. Please tap flag to report any questions that need improvement.

Question 100: A 31-year-old male with previously diagnosed beta-thalassemia with a history of multiple blood transfusions presents to the emergency department with ten days of urinary and bowel incontinence. Vitals signs are within normal limits. The neurological exam is normal except for hypoesthesia in the S1-S5 dermatomes bilaterally. Laboratory studies are only remarkable for hemoglobin of 7 g/dL and a platelet count of 90,000/microL. MRI of the lumbar spine was obtained which showed multiple enhancing lesions in the epidural space between L5S1 with compression of the cauda equina. Which of the following is the most likely cause of the patient's clinical picture?

Choices: 1. Herniating disc 2. Epidural abscess 3. Tumor 4. Extramedullary hematopoiesis

Answer: 4 - Extramedullary hematopoiesis Explanations: Extramedullary hematopoiesis occurs in patients with thalassemia and chronic hemolytic anemias as a compensatory mechanism. It typically occurs in sites such as the spleen, liver and adrenal glands. It has been reported to occur in the spinal canal, causing compression of the spinal cord and its associated structures. A high index of suspicion must be kept for cauda equina and conus medullaris syndromes (CES/CMS) in patients presenting with urinary or bowel retention or incontinence, as these can be the only presenting complaints. Laminectomy with evacuation of the hematopoietic tissue would be the appropriate treatment to decompress the cauda equina in this case. Fungal abscesses have also been reported as rare causes of CES/CMS. Go to the next page if you knew the correct answer, or click the link image(s) below to further research the concepts in this question (if desired).

Research Concepts: Cauda Equina And Conus Medullaris Syndromes

We update eBooks quarterly and Apps daily based on user feedback. Please tap flag to report any questions that need improvement.

Section 2 Question 101: A 25-year-old male presents with 2 days of fever, headache, altered mental status, and involuntary movements. His wife reports that he recently returned from a summer camp program, where he works as a camp counselor and was complaining of mosquito bites. He has no significant medical history. He does not use tobacco or illicit drugs but drinks alcohol occasionally. Temperature is 38C, blood pressure is 126/84 mmHg, and pulse is 72/minute. On exam, he has involuntary chorea and writhing movements of his extremities. Which of the following will most likely be revealed by serology testing?

Choices: 1. Herpesvirus 2. Enterovirus 3. Arbovirus 4. Rabies virus

Answer: 3 - Arbovirus Explanations: In the United States, arboviruses are one of the most common causes of viral encephalitis. History, clinical presentation, and serology can be used to diagnose arbovirus encephalitis. Arboviruses cause disease during the summer months when mosquitos are active. Some arboviruses predominately affect the basal ganglia, causing motor symptoms such as choreoathetosis and parkinsonian movements. Go to the next page if you knew the correct answer, or click the link image(s) below to further research the concepts in this question (if desired).

Research Concepts: Viral Encephalitis

We update eBooks quarterly and Apps daily based on user feedback. Please tap flag to report any questions that need improvement.

Question 102: A patient is being admitted to the hospital for further evaluation due to concern for Hantavirus pulmonary syndrome after presenting with dyspnea, vomiting, and fever. What laboratory abnormality is most significant on presentation demonstrating the need for admission?

Choices: 1. Thrombocytopenia 2. Hypernatremia 3. Hypokalemia 4. Leukocytosis

Answer: 1 - Thrombocytopenia Explanations: Thrombocytopenia is often present at the stage of disease requiring admission for further observation. This often precedes the descent into pulmonary failure by a couple of days. Hyponatremia is more common than hypernatremia in Hantavirus pulmonary syndrome. Potassium is not normally greatly affected in the early stages of Hantavirus pulmonary syndrome. Atypical lymphocytes and premature white blood cells may be present on admission, but leukocytosis alone is not significant enough to warrant admission. Go to the next page if you knew the correct answer, or click the link image(s) below to further research the concepts in this question (if desired).

Research Concepts: Hantavirus Syndrome

We update eBooks quarterly and Apps daily based on user feedback. Please tap flag to report any questions that need improvement.

Question 103: A 65-year-old man with multiple vascular risk factors including hypertension, diabetes, hyperlipidemia a 30-pack-year history of smoking is brought by ambulance to the emergency department because he was found in bed unable to talk and move his face or his extremities. He was last seen normal 5 hours before presentation. Examination shows a blood pressure of 190/120 mmHg and pulse 90/min, regular. He is conscious and able to respond to verbal commands by blinking. He has normal vertical eye movements but no horizontal eye movements. He has quadriplegia with no facial movement. His NIH stroke scale score is 28. His CT head is unremarkable. Which of the following is the next best step in the management of this patient?

Choices: 1. Lower his blood pressure to prepare for IV thrombolysis 2. MRI brain to confirm the diagnosis of brainstem infarct 3. CT angiogram to confirm basilar thrombosis 4. Prepare for hospice and counsel the family

Answer: 3 - CT angiogram to confirm basilar thrombosis Explanations: The patient presented with classical locked in syndrome with basilar thrombosis as the obvious cause. He presented beyond the 4.5 hours window. His NIHSS score is high, also indicating large vessel occlusion. He should be a good candidate for thrombectomy therapy. The logical next step should be to confirm basilar thrombosis with CT angiogram in preparation for thrombectomy. His blood pressure needs to be lowered to below 180/110 mmHg only if he is an IV TPA candidate. MRI brain is best in confirming brainstem infarction yet this is not as important as CT angiogram. The patient does have a guarded prognosis; it is too soon to give up and consider hospice. Go to the next page if you knew the correct answer, or click the link image(s) below to further research the concepts in this question (if desired).

Research Concepts: Basilar Artery Thrombosis

We update eBooks quarterly and Apps daily based on user feedback. Please tap flag to report any questions that need improvement.

Question 104: A 28-year-old female is accidentally kicked in the abdomen by her horse and is brought to the emergency department complaining of severe abdominal pain. The triage nurse notes her vitals are unstable and immediately consults with the emergency department physician to assess the patient. The patient is found to have an acute abdomen, and the general surgeon performs an immediate exploratory laparotomy. The patient has a lacerated liver, ruptured spleen, and perforated transverse colon. The patient receives 22 units of packed red blood cells, 12 units of fresh frozen plasma, 12 liters of crystalloid solution, and 10 units of platelets. After surgery, she is taken to the ICU in relatively stable condition. Ten hours later, the nurse calls the resident surgeon on call to inform them that the patient's abdomen appears tense. Which of the following parameters warrants an immediate explorative laparotomy?

Choices: 1. Elevated intracranial pressure 2. Decreased jugular venous pressure 3. Decreased systemic vascular resistance 4. Decreased peak airway pressure

Answer: 1 - Elevated intracranial pressure Explanations: Physiologic cardiac sequelae include decreased cardiac output and increased central venous pressure (CVP) due to inferior vena cava (IVC) and portal vein compression, as well as an increased systemic vascular resistance which leads to hypotension. Pulmonary involvement can include decreased thoracic volumes and elevated peak pressures from compression of the diaphragm, a decrease in P/F ratio, and hypercarbia. Renal compression can lead to decreased GFR and low urine output. Visceral blood flow is also decreased. Neurologic symptoms result from increased intracranial pressure from the elevated CVP due to IVC compression; thus, contributing to a decrease in cerebral perfusion pressure (CPP). Go to the next page if you knew the correct answer, or click the link image(s) below to further research the concepts in this question (if desired).

Research Concepts: Abdominal Compartment Syndrome

We update eBooks quarterly and Apps daily based on user feedback. Please tap flag to report any questions that need improvement.

Question 105: A 65-year-old male with a history of constipation (on laxatives) and hyperlipidemia is admitted to the hospital with complaints of abdominal pain, vomiting, and bright red blood per rectum. He was found to have a pseudopneumoperitoneum on imaging and diagnosed with Chilaiditi syndrome with part of the cecum interposed between the liver and diaphragm. He underwent nasogastric tube decompression and improved. He was discharged with follow up. He returns to the emergency department three months later with similar complaints and imaging confirmed cecal interposition between the liver and the diaphragm. He undergoes nasogastric tube decompression with improvement and was again discharged with follow up. Eight months later he was re-admitted with the same issue. Which of the following is the next best step in the management of this patient?

Choices: 1. Exploratory laparotomy 2. Ceceopexy 3. Cecal resection 4. Colonoscopic decompression

Answer: 2 - Ceceopexy Explanations: Surgical intervention is indicated when the patient does not respond to conservative management and either the obstruction fails to resolve or there is evidence of bowel ischemia. There is no clear consensus on the optimal surgical approach to correct bowel interposition. Ceceopexy may be done to eliminate the possibility of recurrence in an uncomplicated cecal volvulus unless surgical resection is indicated. Colonic resection is the best option for volvulus of the transverse colon and attempts at colonoscopic reduction are not recommended due to a high frequency of gangrene with these types of volvulus. Go to the next page if you knew the correct answer, or click the link image(s) below to further research the concepts in this question (if desired).

Research Concepts: Chilaiditi Syndrome

We update eBooks quarterly and Apps daily based on user feedback. Please tap flag to report any questions that need improvement.

Question 106: A 24-year-old patient with asthma comes to the emergency department with acute shortness of breath and wheezing. She was given nebulization, corticosteroids, and was placed on BI-PAP. She continued to be in respiratory distress and was eventually intubated. Her initial arterial blood gas (ABG) was pH 7, PCO2 100, and PO2 60. After two days, a sedation vacation was attempted, and ABG was drawn on assist mode of ventilation which showed her pH was 7.15, PCO2 80, and PO2 80 on 30% oxygen. All other labs and images are normal. The patient is hemodynamically stable and following commands. Her negative inspiratory force (NIF) was -40 H20. Which of the following would be considered contraindicated for pressure support ventilation?

Choices: 1. pH-7.15, abnormal acid-base balance 2. NIF -40 cm H2O 3. Mental status 4. Normal blood pressure

Answer: 1 - pH-7.15, abnormal acid-base balance Explanations: Pressure support breathing is a mode of ventilation that is made up of patient-triggered, pressure-limited, flow-cycled breaths. It is valuable to note that the patient has consistent control over breath frequency, breath duration and flow while in a pressure support ventilation environment. The volume of each breath is a direct result of set pressures, patient effort, and potentially other mechanical settings that may oppose ventilation. Patients must initiate his or her own breaths. Apneic patients are required to be in a controlled breath environment in which the ventilator may fully sustain the respiratory function. It is essential that a patient be hemodynamically stable and have sufficient respiratory effort to qualify for pressure support mode. Patients who have a major acid-base imbalance on controlled setting should not be considered for pressure support mode as they can become more unstable. Go to the next page if you knew the correct answer, or click the link image(s) below to further research the concepts in this question (if desired).

Research Concepts: Pressure Support

We update eBooks quarterly and Apps daily based on user feedback. Please tap flag to report any questions that need improvement.

Question 107: A seventy-eight-year-old male is noted to have right arm and right leg weakness by his caretaker. Emergency medical services are called, and the patient is transported to a nearby Acute Stroke Ready Hospital without neurovascular imaging capabilities. His last known normal was 5 hours ago. Non-contrast brain CT is negative for hemorrhage. The emergency physician suspects an acute ischemic stroke. There are no contraindications to intravenous (IV) tPA if indicated. What is the most appropriate course of action?

Choices: 1. Admit for observation and physical therapy as the patient is outside the IV tPA treatment window 2. Initiate IV tPA therapy and transfer to Primary Stroke Center for admission and ongoing care 3. Initiate IV tPA and transfer patient to Comprehensive Stroke Center for possible mechanical thrombectomy 4. Transfer patient to Comprehensive Stroke Center for possible mechanical thrombectomy without starting IV tPA

Answer: 4 - Transfer patient to Comprehensive Stroke Center for possible mechanical thrombectomy without starting IV tPA

Explanations: The American Heart Association and the American Stroke Association recommend IV tPA be administered to all eligible patients as quickly as possible within 3 hours of last known normal with an extended window of 4.5 hours in a more selective group of patients. The American Heart Association and the American Stroke Association recommend mechanical thrombectomy as quickly as possible to eligible patients with large vessel occlusion within 6-16 hours of last known normal. The American Heart Association and the American Stroke Association consider mechanical thrombectomy reasonable in select patients within 6 to 24 hours of last known normal. Large vessel occlusion strokes may be identified with neurovascular imaging including CT angiography (CTA) and magnetic resonance angiography (MRA). Patients should be transferred to a stroke center capable of identifying a large vessel occlusion amenable to mechanical thrombectomy if they remain eligible for this intervention. Go to the next page if you knew the correct answer, or click the link image(s) below to further research the concepts in this question (if desired).

Research Concepts: Stroke Center Certification

We update eBooks quarterly and Apps daily based on user feedback. Please tap flag to report any questions that need improvement.

Question 108: A 50-year-old male is brought to the emergency department (ED) after being involved in a road traffic accident. On arrival, he is intubated immediately for airway protection. His vitals on arrival include a temperature of 99.2 degrees Fahrenheit, a blood pressure of 100/70 mmHg, a pulse of 110 beats per minute, a respiratory rate of 20 breaths per minute, and oxygen saturation of 88% on a ventilator. CT abdomen and pelvis shows grade 4 splenic laceration with fluid in the left paracolic gutter. Laboratory investigations reveal a hemoglobin of 9 g/dl, a white cell count of 12000/mm3, a platelet count of 250000/mm3, sodium 148 meq/dl, potassium 4 meq/dl, chloride 107 meq/dl, bicarbonate 24 meq/dl, creatinine 1 mg/dl, and blood urea nitrogen of 20 mg/dl. In the ED, he receives 2 liters fluid and is taken to the operating room. Open laparotomy is started under general anesthesia. After 15 minutes into the laparotomy, patient oxygen saturation drops to 70%. The patient position is double-checked to confirm that endotracheal tube is not kinked. What is the next best step?

Choices: 1. Flexible bronchoscopy 2. Rigid bronchoscopy 3. Deflating endotracheal tube balloon and adjust the position of the tube 4. Repeating chest x-ray

Answer: 1 - Flexible bronchoscopy Explanations: Flexible bronchoscopy is done to rule out any accidental intubation of aberrant bronchus. Flexible bronchoscopy allows providers to see the relation of the endotracheal tube cuff to the abnormal bronchus. Based on how far the distance of the tracheal bronchus from the carina and the endotracheal tube cuff, a decision is made to use a double-lumen endotracheal tube or use a bronchial blocker. After the appropriate adjustments are made, flexible bronchoscopy is repeated to confirm the appropriate positions of the tube. Bilateral breath sounds should be audible on both sides to ensure appropriate ventilation. Go to the next page if you knew the correct answer, or click the link image(s) below to further research the concepts in this question (if desired).

Research Concepts: Tracheal Bronchus

We update eBooks quarterly and Apps daily based on user feedback. Please tap flag to report any questions that need improvement.

Question 109: A 78-year-old female was diagnosed with deep vein thrombosis and started on rivaroxaban seven weeks ago. Two weeks ago, she was admitted to the intensive care unit for a gastrointestinal (GI) bleed and received andexanet alfa for rivaroxaban reversal. She is now hemodynamically stable and is being discharged from the hospital. What is the most appropriate next step in the management of this patient?

Choices: 1. The patient should never receive anticoagulation because she experienced a GI bleed. 2. The patient does not require anticoagulation because she completed the treatment course. 3. The patient should resume anticoagulation with rivaroxaban one month from now. 4. The patient should resume anticoagulation with rivaroxaban now.

Answer: 4 - The patient should resume anticoagulation with rivaroxaban now. Explanations: This patient had a recent deep vein thrombosis (DVT), requiring anticoagulation for at least three months The patient had a GI bleed, received a reversal agent, has stabilized, and is ready for discharge from the hospital. After the reversal, anticoagulation should be resumed as soon as medically appropriate to avoid thromboembolic complications. Given the history of DVT, this patient needs anticoagulation, which should be resumed now that she is medically stable. Go to the next page if you knew the correct answer, or click the link image(s) below to further research the concepts in this question (if desired).

Research Concepts: Andexanet Alfa

We update eBooks quarterly and Apps daily based on user feedback. Please tap flag to report any questions that need improvement.

Question 110: A 35-year-old male with history of daily alcohol use for acute alcoholic pancreatitis. This is his second admission to the hospital for this issue. He continues to have abdominal pain, but his nausea and vomiting are well controlled at this time. Which of the following interventions or treatments holds a strong level of recommendation based on moderate quality evidence from the American Gastroenterological Association?

Choices: 1. Keep the patient on a clear liquids diet until abdominal pain completely resolves 2. Initiation of prophylactic antibiotics if his hospital stay exceeds 48 hours 3. Goal-directed fluid therapy with hydroxyethyl starch (HES) instead of normal saline or lactated Ringer’s solution 4. Brief alcohol intervention during admission

Answer: 4 - Brief alcohol intervention during admission Explanations: The American Gastroenterological Association Institute guideline on initial management of acute pancreatitis, published in 2018, recommends that patients admitted for alcoholic acute pancreatitis receive brief alcohol intervention counseling during admission. Studies have shown that repeated interventions at regularly scheduled periods have shown to decrease alcohol use and decrease hospital admissions for recurrent attacks. These can be performed at 6-month intervals during primary care or gastroenterology clinic visits by a nurse or other trained medical professional. These interventions should be employed along with smoking cessation counseling and dietary optimization strategies, as these factors play a strong role in recurrence of disease when associated with chronic alcohol use. The most recent guidelines also recommend early initiation of enteral nutrition with a lowfat, soft or solid diet, fluid replacement with normal saline or lactated Ringer’s and avoidance of prophylactic antibiotics. Go to the next page if you knew the correct answer, or click the link image(s) below to further research the concepts in this question (if desired).

Research Concepts: Alcoholic Pancreatitis

We update eBooks quarterly and Apps daily based on user feedback. Please tap flag to report any questions that need improvement.

Question 111: A 38-year-old male with a significant past medical history of diabetes, hypertension and end-stage renal disease (on dialysis) is brought to the emergency room with shortness of breath. He is diagnosed with aspiration pneumonia and intubated due to respiratory failure. He is on appropriate antibiotics and propofol for sedation. He was very dyssynchronous with the ventilator, so midazolam infusion was started to achieve more sedation. After 2 days, sedation vacation was attempted, but the patient did not wake up even after 8 hours after turning all the sedatives off. Which of the following is the most likely explanation for the prolonged sedation?

Choices: 1. Propofol remains for a longer time in renal failure patients 2. He developed encephalitis 3. He had a stroke while he was sedated 4. Midazolam remains for a longer time in renal failure patients

Answer: 4 - Midazolam remains for a longer time in renal failure patients Explanations: The medications used to initiate and maintain sedation within an intensive care unit setting include benzodiazepines such as diazepam, lorazepam, and midazolam; opioid analgesics such as fentanyl, hydromorphone, morphine, remifentanil, propofol, dexmedetomidine, and ketamine; and antipsychotics such as haloperidol, quetiapine, and ziprasidone. No sedative is found to be superior in efficacy or mortality. However, The Society of Critical Care Medicine guidelines advises avoiding benzodiazepines due to evidence of longer duration of intubation especially in patients with renal impairment. The choice of which sedative is best lies in the practitioner's clinical assessment of individual patient scenarios, weighing the risk/benefit profile of the medicine to each patient. Go to the next page if you knew the correct answer, or click the link image(s) below to further research the concepts in this question (if desired).

Research Concepts: Sedation Vacation in the ICU

We update eBooks quarterly and Apps daily based on user feedback. Please tap flag to report any questions that need improvement.

Question 112: A patient presents to the emergency department following an overdose of her antihypertensive medications. She is noted to have profound bradycardia and hypotension. Which of the following statements is true when differentiating calcium channel blocker toxicity from beta-blocker toxicity?

Choices: 1. The electrocardiogram findings in each of the poisonings are completely different 2. An elevated serum calcium is indicative of calcium channel blocker poisoned patients 3. A normal mental status suggests calcium channel blocker overdose 4. A reliable marker for beta blocker overdose is an elevated serum glucose

Answer: 3 - A normal mental status suggests calcium channel blocker overdose Explanations: The vital signs and the EKG findings are not particularly helpful in differentiating between the two classes of drugs in overdose. In calcium channel overdose, the patient typically maintains a normal mental status. This may be due to the neuroprotective features of calcium channel blockers. In calcium channel overdose, the serum glucose is typically elevated, with beta blocker overdose, the serum glucose may be high, low, or normal so this is not a reliable marker. One of the first line treatments for calcium channel blocker overdose includes intravenous calcium. Go to the next page if you knew the correct answer, or click the link image(s) below to further research the concepts in this question (if desired).

Research Concepts: Calcium Channel Blockers

We update eBooks quarterly and Apps daily based on user feedback. Please tap flag to report any questions that need improvement.

Question 113: A 73-year-old female is being evaluated in the clinic for four months history of intermitted lightheadedness, dizziness, and presyncope. She has even had a syncopal episode 3 days that lasted briefly and was witnessed by her husband. The patient states that the frequency of her symptoms is about once a week. She denies palpitations, chest pain, or trouble breathing. Her past medical history is significant for hypertension and diabetes mellitus. Her medications include amlodipine and hydrochlorothiazide. EKG shows sinus bradycardia with a rate of 51/min and a normal PR interval. Which of the following is the next best step in diagnosing the patient's underlying problem?

Choices: 1. The patient will need an event monitor for 4 weeks 2. The patient will need 48 hours Holter monitor 3. The patient will need an echocardiogram 4. No further evaluation, the patient has sinus bradycardia which is a benign finding

Answer: 1 - The patient will need an event monitor for 4 weeks Explanations: This patient most likely will have a diagnosis of sinus node dysfunction as she is having inappropriate symptomatic bradycardia. The key to establishing this diagnosis is to correlate the symptoms with abnormal EKG tracing, which often necessitates continuous ambulatory EKG monitoring. Most patients will require an event monitor; however, rarely select patients with frequent symptoms can be diagnosed with a 48 hours Holter monitor. Sinus bradycardia could be a normal finding in conditioned athletes. However, this patient is having symptoms that are most likely related to bradycardia. Go to the next page if you knew the correct answer, or click the link image(s) below to further research the concepts in this question (if desired).

Research Concepts: Sinus Node Dysfunction

We update eBooks quarterly and Apps daily based on user feedback. Please tap flag to report any questions that need improvement.

Question 114: A 67-year-old patient with chronic obstructive pulmonary disease is on mechanical ventilation with the following settings: assist-control (AC) 12, tidal volume (TV) 650, positive end-expiratory pressure (PEEP) 10, and FiO2 40%. The patient suddenly develops desaturation to 75% and becomes hypotensive, diaphoretic, and tachycardic. What is the definitive management for this patient?

Choices: 1. Needle decompression 2. Normal saline fluid bolus 3. Tube thoracostomy decompression 4. Reduce PEEP

Answer: 3 - Tube thoracostomy decompression Explanations: Given the high ventilator settings and clinical signs, the patient likely has an iatrogenic pneumothorax. Treatment should never be delayed for imaging studies. Although needle decompression is regarded as a quick procedure that relieves air from the pleural space and improves symptoms, it is never definitive management and should be followed with a tube thoracostomy. Tube thoracostomies often are placed in the fifth intercostal space above the rib to avoid neurovascular injury and in the midaxillary line directed posterior and superior. Go to the next page if you knew the correct answer, or click the link image(s) below to further research the concepts in this question (if desired).

Research Concepts: Iatrogenic Pneumothorax

We update eBooks quarterly and Apps daily based on user feedback. Please tap flag to report any questions that need improvement.

Question 115: A 16-year-old white female presents to the emergency department with an acute exacerbation of shortness of breath, chest tightness, and wheezing that has not responded to her inhalers at home for over 4 hours. She is using accessory muscles to breathe and can only speak in one or two words. She mentions that she is tired. The difference in systolic blood pressure between inspiration and expiration is 10%. She needs 2 liters of oxygen to maintain saturation at 94%. She is diffusely wheezy but does not have any secretions. Which of the following treatment strategies is least likely to benefit her current disease process?

Choices: 1. Heliox with 40:60 parts of helium and oxygen 2. Use of BIPAP support 3. 125 mg of methylprednisolone followed by 60 mg every 6 hourly 4. 2 separate doses of 2 g magnesium sulfate IV an hour apart

Answer: 1 - Heliox with 40:60 parts of helium and oxygen Explanations: The patient appears to be in status asthmaticus. She is somewhat fatigued but still awake. She does not have any significant secretions. Treatment with noninvasive ventilatory support is appropriate as is methylprednisolone and magnesium at the mentioned dose. Heliox is rarely used in status asthmaticus in patients with definitive evidence of pulsus paradoxus with upper airway wheeze or stridor. Its ratio is 70:30 or 60:40 for oxygen to helium. The patient has no pulsus paradoxus which is interpreted as greater than 12% variation in systolic blood pressure with respiration. Heliox is avoided due to its prohibitive cost, infrequent indication and need for recalibration of gas blenders and flow meters when used with mechanical ventilation. Go to the next page if you knew the correct answer, or click the link image(s) below to further research the concepts in this question (if desired).

Research Concepts: Status Asthmaticus

We update eBooks quarterly and Apps daily based on user feedback. Please tap flag to report any questions that need improvement.

Question 116: An 18-year-old male is brought in by emergency medical services for confusion. He has dilated pupils, dry mucous membranes, blurry vision, tachycardia, and urinary retention. He begins to seize. Which of the following is the most likely medication he ingested in overdose?

Choices: 1. Acetaminophen 2. Doxylamine 3. Hydrocodone 4. Verapamil

Answer: 2 - Doxylamine Explanations: Pharmacologically, doxylamine has antihistamine and anticholinergic effects. In overdose, doxylamine causes an anticholinergic toxidrome. Clinical signs and symptoms may progress to hallucinations, rhabdomyolysis, seizures, and death. The half-life of doxylamine is 10 to 12 hours, which may be increased in the elderly. The median lethal dose (LD50) is 50 to 500 mg/kg. The toxidrome for a doxylamine overdose is the same as a diphenhydramine overdose. An acetaminophen level must always be checked initially and at four hours, as these medications may be combined with acetaminophen in over-the-counter medications targeted at analgesia and insomnia. Go to the next page if you knew the correct answer, or click the link image(s) below to further research the concepts in this question (if desired).

Research Concepts: Doxylamine

We update eBooks quarterly and Apps daily based on user feedback. Please tap flag to report any questions that need improvement.

Question 117: A 17-year-old male with leukemia presents to the emergency room with four days of vomiting and voluminous diarrhea. He has had several episodes per day. He has also had reduced oral intake but denies fever or any additional symptomatology. Serum chemistry reveals a sodium level of 145 mmol/L, a potassium level of 1.4 mmol/L, a chloride level of 90 mmol/L, a bicarbonate level of 20 mmol/L, and a magnesium level of 1.9 mg/dL. The physical exam does not reveal any significant findings, and his ECG is normal. What is the next step in management?

Choices: 1. Administer potassium chloride of 40 mEq orally every 3 to 4 hours for 3 doses 2. Administer magnesium sulfate 1 gram IV over 1 to 2 hours 3. Prescribe spironolactone 25 mg orally every 8 hours 4. Administer potassium chloride of 40 mEq intravenously every 3 to 4 hours for 3 doses

Answer: 4 - Administer potassium chloride of 40 mEq intravenously every 3 to 4 hours for 3 doses

Explanations: Rapid replacement therapy is warranted with severe hypokalemia or when clinical symptoms are present. Potassium chloride of 40 mmol given every 3 to 4 hours for 3 doses is preferred. Rapid correction can be provided via oral and IV formulation. Oral supplementation is preferred because it is associated with a lower risk of rebound hyperkalemia. However, intravenous (IV) repletion is administered if oral therapy is not tolerated. IV administration is also preferred in the setting of cardiac dysrhythmias, digitalis toxicity, and recent or ongoing cardiac ischemia. A potassium-sparing diuretic should be considered when the etiology of hypokalemia involves renal potassium wasting as potassium replacement therapy alone may not suffice. Pain and phlebitis usually occur with peripheral IV infusions when infusion rates exceed 10 mEq/hour. There is also a risk of rebound hyperkalemia when rates exceed a dose of 20 mEq/hour. Careful monitoring of serum potassium levels is required as the development of hyperkalemia is common in hospitalized patients. Go to the next page if you knew the correct answer, or click the link image(s) below to further research the concepts in this question (if desired).

Research Concepts: Hypokalemia

We update eBooks quarterly and Apps daily based on user feedback. Please tap flag to report any questions that need improvement.

Question 118: A 56-year-old female with renal failure was hospitalized because she missed two of her dialysis days due to lack of transport. The patient received her first dialysis treatment the day before and is doing much better in the hospital, but she developed a rash on her flank that is tender to the touch. She states it started four days ago and was much smaller than it was now. Physical exam shows a 4 cm x 5 cm black-purple retiform purpura with central eschar on the left flank that is tender to touch. Which of the following is the next best step in the management of this patient?

Choices: 1. Abdominal x-ray 2. Consult dermatology for an incisional biopsy to include subcutaneous tissue 3. Consult general surgery for wide local excision 4. Initiate sodium thiosulfate

Answer: 2 - Consult dermatology for an incisional biopsy to include subcutaneous tissue Explanations: A consult to dermatology for an incisional biopsy to include subcutaneous fat is necessary to make the diagnosis. The treatment approach is interprofessional to include medicine, nephrology, and dermatology. Calciphylaxis has high morbidity and mortality, and early recognition is important. Other causes of retiform purpura are important to consider, including vasculitis. Go to the next page if you knew the correct answer, or click the link image(s) below to further research the concepts in this question (if desired).

Research Concepts: Calcinosis Cutis

We update eBooks quarterly and Apps daily based on user feedback. Please tap flag to report any questions that need improvement.

Question 119: A 60-year-old male has a sudden onset of extreme dizziness, nausea, and vomiting while at work. He is taken to the hospital for a stroke workup. On exam, he opens his eyes to voice, is oriented to conversation, follows commands with all extremities and has profound bilateral dysmetria of the upper and lower extremities. His blood pressure is 180/100 mmHg, heart rate of 102 bpm, and oxygen saturation of 98% on room air. Imaging reveals a cerebellar hemorrhage of the bilateral cerebellar hemispheres with 40 mL of estimated blood volume and effacement of the fourth ventricle with lateral ventricular enlargement. There does not appear to be any blood in the ventricular system. What is his 30-day mortality based on the intracerebral hemorrhage (ICH) score?

Choices: 1. 26% 2. 72% 3. 97% 4. 100%

Answer: 1 - 26% Explanations: The intracerebral hemorrhage (ICH) score predicts 30-day mortality based on patient with non-traumatic intracerebral hemorrhage. The score is based on Glasgow coma score (GCS), age, volume of hemorrhage, intraventricular hemorrhage, and infratentorial origin of hemorrhage. The point distribution is GCS 13 to 15 is 0 points, GCS 5 to 12 is 1 point, GCS 3 to 4 is 2 points; age 80 or greater is 1 point, ICH volume 30 mL or more is 1 point, intraventricular hemorrhage is 1 point, and infratentorial origin of hemorrhage is 1 point. 30-day mortality is 0% for 0 points, 13% for 1 point, 26% for 2 points, 72% for 3 points, 97% for 4 points, 100% for 5 points, and assumed to be 100% for 6 points. Go to the next page if you knew the correct answer, or click the link image(s) below to further research the concepts in this question (if desired).

Research Concepts: Intracranial Hemorrhage

We update eBooks quarterly and Apps daily based on user feedback. Please tap flag to report any questions that need improvement.

Question 120: A 65-year-old male patient is being treated in the emergency department. He is experiencing congestive heart failure. The patient has had obesity and type 2 diabetes for much of his adult life. His diabetes is not currently being medically addressed. He has an arrhythmia which is being managed with disopyramide. The patient has no known allergies. During the maintenance dose of inamrinone, the patient’s blood pressure drops significantly from 149/95 to 78/55 mmHg. What is the next most appropriate course of treatment in this situation?

Choices: 1. Stop administration of inamrinone 2. Maintain the rate and concentration of inamrinone administration 3. Reduce rate and maintain concentration of inamrinone administration 4. Stop administration of disopyramide

Answer: 1 - Stop administration of inamrinone Explanations: Inamrinone administration should be stopped when there are significant drops in blood pressure and the patient becomes hypotensive. Inamrinone may be interacting with disopyramide, indicating that inamrinone administration should be stopped. Even a smaller dose of inamrinone may have the capacity to compound the hypotensive symptoms. Disopyramide is an antiarrhythmic drug taken orally. Stopping its administration would not result in the acute improvement of blood pressure as required. Go to the next page if you knew the correct answer, or click the link image(s) below to further research the concepts in this question (if desired).

Research Concepts: Inamrinone

We update eBooks quarterly and Apps daily based on user feedback. Please tap flag to report any questions that need improvement.

Question 121: A patient presents with sudden loss of vision in his right eye that lasted 45 seconds. He felt weakness in his left arm at the same time, but the symptoms improved rapidly. He is on metoprolol, aspirin, and atorvastatin. A workup reveals a bruit in the neck and a duplex ultrasound reveals 90% stenosis at the right carotid artery bifurcation. Also noted is that the internal carotid artery is severely calcified starting at the bifurcation, and this calcification extends distally for 5 cm. If this patient undergoes angioplasty and stenting, what hemodynamic complication is most likely?

Choices: 1. Premature ventricular complexes 2. Hypotension 3. Supraventricular tachycardia 4. Wolff-Parkinson-White syndrome

Answer: 2 - Hypotension Explanations: During stenting and angioplasty of the carotid artery, the patient is at increased risk for hypotension during the procedure if there is extensive calcification. The hypotension may persist for 24 to 72 hours. Some patients may need IV fluids and pseudoephedrine to overcome the hypotension. Most surgeons do not discharge these patients home until their blood pressure has stabilized. In the intensive care unit, inotropes may need to be started. Patients also can experience bradycardia as an effect of the stenting. Go to the next page if you knew the correct answer, or click the link image(s) below to further research the concepts in this question (if desired).

Research Concepts: Carotid Artery Stenting

We update eBooks quarterly and Apps daily based on user feedback. Please tap flag to report any questions that need improvement.

Question 122: A 65-year-old female from Southeast Asia traveled by air to Toronto and was found to have acid-fast bacteria in the sputum. She was immediately started on rifampin. Four days later, she was seen in the emergency department with a swollen right leg and started on heparin for 3 days and then placed on oral warfarin. Despite immediate treatment for her pathology, she expired 7 days later. At autopsy, she was found to have pulmonary emboli. Which of the following most likely triggered the chain of events that led to her death?

Choices: 1. Miliary tuberculosis 2. Starting rifampin 3. Overdosing warfarin 4. Underdosing heparin

Answer: 2 - Starting rifampin Explanations: Rifampin is often a first-line drug used to treat tuberculosis. It is frequently used in combination with other tuberculosis agents like isoniazid and pyrazinamide. Rifampin is also one of the most potent inducers of the cytochrome P450 enzymes and, thus, is often associated with drug interactions. Rifampin can increase the breakdown of warfarin, and thus the levels become subtherapeutic and lead to deep vein thrombosis and pulmonary emboli. When a patient on warfarin is started on drugs like rifampin, more frequent monitoring of the INR is mandatory or the rifampin should be discontinued. Go to the next page if you knew the correct answer, or click the link image(s) below to further research the concepts in this question (if desired).

Research Concepts: Rifampin

We update eBooks quarterly and Apps daily based on user feedback. Please tap flag to report any questions that need improvement.

Question 123: A 62-year-old male with a positive past medical history for DM, HTN, CKD stage 3 and COPD comes in with left-sided chest pain and shortness of breath for the past 2 hours. The pain is pressure like and is radiating to the jaw. Physical examination is positive for tachycardia, otherwise negative — Vitals, BP 160/45, Respiratory rate 32/min, Pulse 110/min, and afebrile. Labs show mild leukocytosis and elevated cardiac enzymes. EKG shows STsegment elevation in lateral leads. STEMI alert is called, and the patient undergoes cardiac catheterization. He got 2 stents in LAD and was started on appropriate medical treatment. After 3 days he is discharged home and was told to follow up with the primary care physician. Two weeks later he undergoes sudden cardiac arrest and dies. What is the likely cause of patients sudden cardiac death?

Choices: 1. Ventricular fibrillation 2. Pulseless electrical activity 3. Atrial fibrillation 4. Supraventricular tachycardia

Answer: 2 - Pulseless electrical activity Explanations: After a myocardial infarction, the risk of sudden cardiac death is highest during the first months due to fatal tachyarrhythmias, re-infarction, or myocardial rupture. Ventricular fibrillation (VF) and ventricular tachycardia (VT) were initially thought to be the most common causes of out-of-hospital cardiac arrest. More recent studies show pulseless electrical activity (PEA), and asystole as the most frequent cause of sudden post-MI death. Approximately 50% of patients initially have asystole, and 19% to 23% have PEA as the first identifiable rhythm. Following the hospital discharge, the PAE leads to the sudden cessation of the blood flow to the brain leading to the death. Go to the next page if you knew the correct answer, or click the link image(s) below to further research the concepts in this question (if desired).

Research Concepts: Sudden Cardiac Death

We update eBooks quarterly and Apps daily based on user feedback. Please tap flag to report any questions that need improvement.

Question 124: A 55-year-old obese male undergoes a diagnostic cardiac catheterization procedure for complaints of chest pain. His cardiac catheterization revealed moderate nonobstructive disease, and he was recommended aggressive medical therapy. Common femoral access performed the procedure, and hemostasis was secured by manual compression. Baseline vital signs revealed a blood pressure of 130/76 mmHg and heart rate of 70 beats per minute. Two hours after the procedure, the patient complains of feeling nauseous, is diaphoretic and has complaints of back pain. His blood pressure is 70/50 mmHg, and heart rate is 120 bpm. Which of the following is the likely diagnosis?

Choices: 1. Acute stroke 2. Acute myocardial infarction 3. Retroperitoneal bleeding 4. Arteriovenous fistula

Answer: 3 - Retroperitoneal bleeding Explanations: Vascular access site complications are the most common complications following cardiac catheterization procedures. Bleeding that extends into the retroperitoneal space cause result in significant bleeding resulting in hemodynamic instability and increases morbidity and mortality. Retroperitoneal bleeding usually happens when the artery is accessed above the inguinal ligament. Computed tomographic imaging or angiography may be needed for quick diagnosis. Treatment included reversal of anticoagulation, volume resuscitation with fluids, identifying the bleeding vessel and with coiling or use of stent grafts. Sometimes patients may need vascular surgical consultation and surgical repair of the bleeding vessel. Go to the next page if you knew the correct answer, or click the link image(s) below to further research the concepts in this question (if desired).

Research Concepts: Cardiac Catheterization Risks and Complications

We update eBooks quarterly and Apps daily based on user feedback. Please tap flag to report any questions that need improvement.

Question 125: A 75-year-old chronically ill male is admitted to the intensive care unit for septic shock for presumed vancomycin-resistant enterococci (VRE) bacteremia and is started on daptomycin. Seven days into treatment the patient reports severe muscle aches. What is the appropriate lab test to order to determine the etiology of the patient’s pain?

Choices: 1. Procalcitonin 2. C-reactive protein (CRP) 3. Creatine kinase (CK) 4. Basic metabolic panel (BMP)

Answer: 3 - Creatine kinase (CK) Explanations: Patients should be evaluated for myopathy while on daptomycin by obtaining weekly creatine kinase levels. Daptomycin is not effective for pulmonary infections as it is inactivated by surfactant. VRE is a rare cause of pneumonia. Daptomycin is one of the few drugs that is bactericidal against VRE. Go to the next page if you knew the correct answer, or click the link image(s) below to further research the concepts in this question (if desired).

Research Concepts: Daptomycin

We update eBooks quarterly and Apps daily based on user feedback. Please tap flag to report any questions that need improvement.

Question 126: A 66-year-old female presents with fever, chills, hematuria, dysuria, abdominal pain, nausea, and vomiting. Vital signs show temperature 37.8 °C, heart rate 100, blood pressure 120/80 mmHg, and respirations 18. The cardiopulmonary exam is normal. Her abdomen is soft with normal bowel sounds. There is significant left flank pain with suprapubic tenderness. What is the most likely diagnosis?

Choices: 1. Diverticulitis 2. Acute pyelonephritis 3. Gastroenteritis 4. Cholecystitis

Answer: 2 - Acute pyelonephritis Explanations: Dysuria, fever, chills, and abdominal or flank pain are characteristic of acute pyelonephritis. The elderly are at higher risk for urosepsis and subsequent mortality. Pyonephrosis will need to be ruled out by imaging. The classic features of pyelonephritis may not be seen in children or the elderly. In the elderly, there may also be mental status changes. Gross hematuria may be seen in young women but in males, this is unusual. Go to the next page if you knew the correct answer, or click the link image(s) below to further research the concepts in this question (if desired).

Research Concepts: Acute Pyelonephritis

We update eBooks quarterly and Apps daily based on user feedback. Please tap flag to report any questions that need improvement.

Question 127: A 72-year-old male in the intensive care unit (ICU) for respiratory failure is receiving total parenteral nutrition (TPN) and has signs of sepsis. He has multiple risk factors for fungemia including age, ICU stay, central venous catheter, and TPN. Cultures are positive for Candida and treatment is started with caspofungin. The central line is removed and he responds well to the caspofungin. What other intervention should be considered?

Choices: 1. Abdominal CT to rule out deep-seated tissue infection 2. Abdominal MRI to rule out deep-seated tissue infection 3. Ophthalmology consult and evaluation 4. Early discontinuation of therapy as the patient has improved

Answer: 3 - Ophthalmology consult and evaluation Explanations: All patients with candidemia should undergo an ophthalmologic evaluation to rule out endophthalmitis. Recommended duration of therapy is for 2 weeks after documented clearing of the infection and resolution of symptoms. When candidemia has been confirmed by blood cultures, treatment with an echinocandin and central venous catheter removal should be the first steps to take. CT scans only are warranted if deep-seated tissue infection is strongly suspected. Go to the next page if you knew the correct answer, or click the link image(s) below to further research the concepts in this question (if desired).

Research Concepts: Fungemia Candidiasis

We update eBooks quarterly and Apps daily based on user feedback. Please tap flag to report any questions that need improvement.

Question 128: A 17-year-old previously healthy female in Hawaii developed fever, chills, pharyngitis, muscle pain, scleral injection, headache, cervical lymphadenopathy, and photophobia lasting 7 days and was improving. The patient then seemed to relapse with additional symptoms of nausea, headache, emesis, and moderate neck stiffness. She raises pigs. Lumbar puncture showed 180 WBCs per microliter that were all monocytes and elevated protein. Which of the following is true?

Choices: 1. More than half of cases are severe 2. Involvement of the central nervous system is common 3. Hepatic and renal involvement is common 4. The most common vector is the mosquito

Answer: 2 - Involvement of the central nervous system is common Explanations: The patient has leptospirosis, a disease acquired from rats, livestock, cats, dogs, or other pets. Most patients have anicteric courses, but 50% to 90% have meningeal involvement. Fewer than 10% of cases are icteric, and these patients may also have renal involvement. Both anicteric and icteric illnesses start with the septicemic phase, as described in the question. Go to the next page if you knew the correct answer, or click the link image(s) below to further research the concepts in this question (if desired).

Research Concepts: Leptospirosis

We update eBooks quarterly and Apps daily based on user feedback. Please tap flag to report any questions that need improvement.

Question 129: A 72-year-old male with a past medical history significant for ischemic cardiomyopathy with reduced ejection fraction of 25% to 30%, essential hypertension, mixed hyperlipidemia, coronary artery disease, type II diabetes, gout, and chronic kidney disease stage III presented with worsening dyspnea, at rest and with exertion, orthopnea, and increased lower extremity swelling. Per his cardiologist, the patient is compliant with a low sodium diet, cardiac rehabilitation, and his diuretic regimen. Home cardiac medications include 80 mg of oral furosemide twice a day. He was recently started on metolazone after hospitalization for acute decompensated heart failure last month. Despite this adjustment, the patient continues to complain of progressive worsening of his symptoms. His vital signs showed a blood pressure of 110/70 mmHg, heart rate of 65 bpm, respiratory rate of 19/minute, temperature of 98.1 F, and oxygen saturation was 89% on room air. Pertinent physical exam findings were elevated jugular venous pressure with positive hepatojugular reflex and +3 pitting edema in the bilateral lower extremities. Chest radiograph was significant for an increased cardiopulmonary ratio greater than 60% and bilateral pleural effusions with increased cephalization consistent with pulmonary edema. Laboratory studies show sodium 132 mEq/L, potassium 3.4 mEq/L, Chloride 112 mEq/L, carbon dioxide 19 mEq/L, BUN 31 mg/dL, creatinine 1.4 mg/dL pro-BNP of 30,000 pg/mL. The patient was admitted to the hospital and was started on intravenous furosemide 120 mg BID, and his home metolazone was continued. The patient experienced minimal improvement in symptoms. Input and output was net –200 ml in 24 hours. What would be the next most effective way to remove fluids in the clinical scenario mentioned above?

Choices: 1. Increase the dose of intravenous loop diuretics 2. Switch scheduled IV furosemide to a continuous IV furosemide drip 3. Add IV positive inotropic agents 4. Ultrafiltration

Answer: 4 - Ultrafiltration Explanations: The patient is already on a high dose of IV furosemide so increasing the dose of loop diuretics would unlikely have any further diuretic effect. The patient is already to a high dose of IV furosemide, and therefore switching to a furosemide drip would unlikely have any further diuretic effect. Positive inotropic agents would have less effect that ultrafiltration in removing fluids. Ultrafiltration is the most effective way in removing fluids among all answers in this patient. Go to the next page if you knew the correct answer, or click the link image(s) below to further research the concepts in this question (if desired).

Research Concepts: Ultrafiltration in Acute Decompensated Heart Failure

We update eBooks quarterly and Apps daily based on user feedback. Please tap flag to report any questions that need improvement.

Question 130: A 17-year-old female arrives at the emergency department via ambulance. On exam, she is agitated and confused. Her blood pressure is 160/110 mmHg, heart rate 140 beats/min, and temperature 39C. She is diaphoretic, tremulous, and hyper reflexive but otherwise has a non-focal neurological exam. She is unable to give a detailed history. She has with her a bottle of phenelzine. Her friends report that she "wanted to celebrate her eighteenth birthday" with "something new." What other drugs may she have taken to cause this presentation?

Choices: 1. MDMA (ecstasy) 2. Marijuana 3. Phencyclidine (PCP) 4. Lorazepam

Answer: 1 - MDMA (ecstasy) Explanations: She is exhibiting neuromuscular effects, autonomic effects, and mental status changes associated with serotonin syndrome. She was taking phenelzine, a monoamine oxidase inhibitor (MAOI). The addition of methylenedioxymethamphetamine (MDMA), also known as ecstasy, or cocaine may precipitate serotonin syndrome. Serotonin syndrome occurs following the use of serotonergic drugs. The degree of symptoms can range from mild to severe. Symptoms include hyperthermia, agitation, increased reflexes, tremors, sweating, dilated pupils, and diarrhea. Temperature can increase to greater than 41.1C. Serotonin syndrome typically is caused by a combination of an SSRI with other serotonergic substances such as serotonin-norepinephrine reuptake inhibitors, MAOIs, tricyclic antidepressants, amphetamines, buspirone, triptans, St. John's wort, ecstasy, or cocaine. It is a predictable consequence of excess serotonin. The onset of symptoms occurs within a day of serotonin toxicity. Treatment consists of stopping the serotonin. In those who are agitated, benzodiazepines and a serotonin antagonist such as cyproheptadine may be used. In those with hyperthermia, active cooling measures may be needed. With treatment, the risk of death is less than 1%. Go to the next page if you knew the correct answer, or click the link image(s) below to further research the concepts in this question (if desired).

Research Concepts: Serotonin Syndrome

We update eBooks quarterly and Apps daily based on user feedback. Please tap flag to report any questions that need improvement.

Question 131: A 17-year-old male presents to the emergency department in Arizona with vomiting, dyspnea, fever, and generalized myalgias. The vital signs demonstrate fever, tachypnea, tachycardia, and mild hypoxia. The patient is found to have thrombocytopenia, metamyelocytes, leukocytosis, and bilateral pulmonary infiltrates on chest x-ray. Which organism is the most likely etiology?

Choices: 1. Influenza A 2. Hantavirus 3. Human immunodeficiency virus (HIV) 4. Pneumocystis jiroveci

Answer: 2 - Hantavirus Explanations: Hantavirus presenting as cardiopulmonary syndrome, such as in this patient, is common in the Southwestern United States. Thrombocytopenia is the most common significant laboratory abnormality on presentation when patients are reaching admission criteria. Metamyelocytes may also be seen in the setting of significant leukocytosis. Bilateral infiltrates seen on the chest x-ray may have an acute respiratory distress syndrome appearance due to the fluffy nature due to increased capillary permeability. Hantavirus often presents with fever, chills, diarrhea, malaise, and dyspnea. Hantavirus is predominantly acquired through aerosolization of rodent urine and droppings. Go to the next page if you knew the correct answer, or click the link image(s) below to further research the concepts in this question (if desired).

Research Concepts: Hantavirus Syndrome

We update eBooks quarterly and Apps daily based on user feedback. Please tap flag to report any questions that need improvement.

Question 132: A 17-year-old female is brought to the emergency department by emergency medical services. She is too delirious to give a history but alert enough to complain of stomach pain. She is noted to be hypotensive and tachycardic. She is afebrile. Her exam is unremarkable except for dry mucous membranes. Intravenous fluids are started. Serum chemistry shows that the potassium is elevated and sodium is decreased. Blood glucose is borderline decreased. The electrolytes are otherwise within normal limits. A CBC is unremarkable. After 3 liters of fluids, the patient is still hypotensive. What is the definitive treatment for this patient's condition?

Choices: 1. Vasopressors and broad-spectrum antibiotics 2. Insulin drip 3. Fluids and corticosteroids 4. CT abdomen and pelvis

Answer: 3 - Fluids and corticosteroids Explanations: This presentation, lab abnormalities, and hypotension unresponsive to fluid resuscitation are most consistent with an acute adrenal crisis. The definitive treatment for adrenal crisis is fluids and steroids. An insulin drip would be indicated in a patient with diabetic ketoacidosis (DKA). A patient in DKA would be expected to have elevated potassium and decreased sodium, but the glucose would most likely be elevated. Additionally, the sodium bicarbonate would be low in DKA. Vasopressors and broad-spectrum antibiotics would be indicated in a septic patient. While this patient is hypotensive and tachycardic, she was afebrile and did not have leukocytosis. The patient had altered mental status and complaining of abdominal pain. While it would be reasonable to consider getting a CT of the abdomen and pelvis, it would not be the definitive treatment of the underlying condition. Go to the next page if you knew the correct answer, or click the link image(s) below to further research the concepts in this question (if desired).

Research Concepts: Adrenal Crisis

We update eBooks quarterly and Apps daily based on user feedback. Please tap flag to report any questions that need improvement.

Question 133: A 65-year-old female, with a 30-pack-year smoking history, presents to the emergency department with acute onset of crampy lower abdominal pain associated with nausea and vomiting. She also has a history of rheumatoid arthritis for 15 years and has been off treatment for the last 4. She denies any fever or weight loss or rectal bleeding. On examination, she appears nontoxic but shows distress from the pain. The abdomen is diffusely tender, with hypoactive bowel sounds. She has deformities in her bilateral hand joints, with some erythematous swelling and tenderness in the metacarpophalangeal and interphalangeal joints. Laboratory testing reveals a total leukocyte count of 11,000/microL, hemoglobin of 10.5 g/dL, serum creatinine of 1.2 mg/dL and slightly elevated serum lactate of 2.5 mmol/L. Inflammatory markers, including C-reactive protein and erythrocyte sedimentation rate, are elevated. CT abdomen with contrast reveals multiple segmental stenoses and narrowing in the mesenteric arteries. There is no evidence of any bowel necrosis or perforation. Supportive management with IV fluids and antiemetics is started. What is the next best step in the management of this patient?

Choices: 1. Laparotomy and focal excision of the part of intestines supplied by the affected arteries 2. Laparoscopy with small intestinal and mesenteric biopsy 3. Supportive treatment only, no further treatment required 4. High-dose glucocorticoids and cyclophosphamide

Answer: 4 - High-dose glucocorticoids and cyclophosphamide Explanations: This patient has mesenteric vasculitis related to her rheumatoid arthritis. In patients with rheumatoid arthritis, vasculitis is a rare but serious complication that is usually seen in advanced stages of the disease. It is a significant cause of morbidity and mortality in these patients. Mesenteric vasculitis can be seen as a manifestation of rheumatoid vasculitis. These patients usually exhibit evidence of active rheumatoid disease and systemic vasculitis. In any patient with advanced rheumatoid arthritis presenting with abdominal pain, mesenteric vasculitis needs to be considered in the differential diagnosis. CT of the abdomen reveals the involvement of medium-sized arteries and arterioles, with areas of segmental narrowing. If blood flow to the intestines is critically affected, patient's can present with intestinal ischemia. Significant abdominal tenderness with signs of peritonitis and lower gastrointestinal bleeding should raise clinical suspicion of the same. In these patients, surgery is indicated if there is evidence of acute mesenteric artery occlusion or intestinal gangrene or perforation. In the absence of the above complications, immunosuppression is the mainstay of therapy. Supportive treatment alone, without immunosuppressive therapy, will result in continued inflammation and worsening of symptoms. Go to the next page if you knew the correct answer, or click the link image(s) below to further research the concepts in this question (if desired).

Research Concepts: Mesenteric Vasculitis

We update eBooks quarterly and Apps daily based on user feedback. Please tap flag to report any questions that need improvement.

Question 134: A 48-year-old female presents with epigastric pain radiating through to the back and on physical exam has epigastric tenderness. She denies any use of alcohol or tobacco products. Her heart rate is 115 beats/min, respirations 16 breaths/min, blood pressure 130/85 mmHg, and oxygen saturation 98% on room air. Her lipase is elevated to 13,800 units/L. It has been decided to admit her to the hospital. Which of the following is true concerning her prognosis at this time?

Choices: 1. Her severely elevated lipase level indicates a poor prognosis 2. CT of the abdomen is needed for accurate prognostication 3. Ranson criteria can be calculated to estimate mortality 4. The Bedside Index of Severity for Acute Pancreatitis can be used to calculate mortality

Answer: 4 - The Bedside Index of Severity for Acute Pancreatitis can be used to calculate mortality

Explanations: Estimating mortality of acute pancreatitis can be important at admission to guide disposition decisions for the patient. Lipase levels have not been shown to be an accurate indicator of the severity of acute pancreatitis. The Ranson criteria at admission include age, WBC, blood glucose, serum aspartate aminotransferase, and serum lactate dehydrogenase. The criteria at 48 hours include calcium, hematocrit fall, oxygen, BUN, base deficit, and sequestration of fluid. These can be used to predict the severity of acute pancreatitis but cannot be used at admission to estimate mortality. The Bedside Index of Severity for Acute Pancreatitis (BISAP) score uses five aspects to calculate the mortality of a patient and guide disposition. It is more commonly used now than Ranson or APACHE II criteria because it is less cumbersome and can be used at the time of admission. Increasing scores on the BISAP indicate higher mortality with a full score of 5 points indicating a mortality of 22%. The five aspects of the BISAP score are as follows: (1) BUN greater than 25 mg/dL, (2) Impaired mental status, (3 ) Fulfills two or more of the systemic inflammatory response syndrome (SIRS) criteria, (4) Age over 60 years, and (5) Presence of pleural effusions. Go to the next page if you knew the correct answer, or click the link image(s) below to further research the concepts in this question (if desired).

Research Concepts: Acute Pancreatitis

We update eBooks quarterly and Apps daily based on user feedback. Please tap flag to report any questions that need improvement.

Question 135: A 78-year-old man with a past medical history of stroke, diabetes mellitus, hypertension, and chronic obstructive pulmonary disease comes in from a nursing home for evaluation of increased effort necessary for him to breathe. On initial evaluation, he appears distressed and is placed on noninvasive positive pressure ventilation for an oxygen saturation of 80% on room air. He is found to be tachycardic and hypotensive, with a blood pressure of 80/40 mmHg. A chest X-ray shows a left lower lobe consolidation. What is the best initial antibiotic coverage?

Choices: 1. Azithromycin 2. Levofloxacin 3. Ceftriaxone and azithromycin 4. Cefepime and vancomycin

Answer: 4 - Cefepime and vancomycin Explanations: Azithromycin would cover atypical pathogens but is not broad enough coverage given that the patient is a nursing home resident. Levofloxacin would be an appropriate treatment if he appeared well and would be treated as an outpatient. Ceftriaxone and azithromycin is the correct treatment for community-acquired pneumonia. This patient is in extremis and requires broader antibiotic coverage given the risk of multidrug-resistant organisms. Cefepime and vancomycin are the appropriate broad-spectrum coverage. He would be broadly covered for gram-negative and gram-positive organisms, including Pseudomonas and methicillin-resistant Staphylococcus aureus (MRSA). In patients requiring intensive care unit admission with risk factors for multidrug-resistant organisms, broad-spectrum antibiotics are appropriate first-line treatment. Go to the next page if you knew the correct answer, or click the link image(s) below to further research the concepts in this question (if desired).

Research Concepts: Nursing Home Acquired Pneumonia

We update eBooks quarterly and Apps daily based on user feedback. Please tap flag to report any questions that need improvement.

Question 136: A 43-year-old male patient with past medical history notable for chronic obstructive pulmonary disease and moderate hereditary spherocytosis presents complaining of increased cough with sputum production for the previous week. On physical evaluation, the patient has a blood pressure of 84/57 mmHg, heart rate 102 bpm, and respirations 24 per minute. Auscultation reveals rales in the right lower base. Chest x-ray shows new consolidation on bilateral lung bases. Vital signs normalize after aggressive intravenous fluid resuscitation, and the patient is admitted for treatment of COPD. His bloodwork shows that troponin I levels increased to 1.2 ng/mL and then normalized within the same day. Hemoglobin is 9.4 g/dL (his baseline). His EKG did not show any acute abnormalities. Which of the following management decisions is most adequate considering his increase in troponin?

Choices: 1. The patient should be started on treatment for acute coronary syndrome, including aspirin, high-intensity statins, and anticoagulation 2. His increase in troponin is most likely due to demand ischemia from his underlying sepsis, and he does currently warrant treatment for acute coronary syndrome 3. Patient warrants immediate catheterization due to his TIMI score 4. Patient requires a blood transfusion

Answer: 2 - His increase in troponin is most likely due to demand ischemia from his underlying sepsis, and he does currently warrant treatment for acute coronary syndrome

Explanations: This patient has a low hemoglobin baseline due to a history hereditary spherocytosis. In this setting, an acute decrease in blood pressure may decrease the supply of oxygen to the myocardium, which is the most likely cause of his transient increase in troponin. Patients hereditary spherocytosis may remain asymptomatic even with low hemoglobin levels. They may, however, have a higher predisposition to myocardial ischemia in acute conditions where coronary blood supply decreases, such as in sepsis. His hemodynamic status improved with administration of intravenous fluids. Administration of blood transfusion is warranted in hemodynamically stable patients with hemoglobin greater than 8 g/dL and without a history of underlying coronary disease. For patients with acute coronary syndrome in the setting of anemia, an optimal transfusion threshold is unclear. Some authors advocate for transfusing when hemoglobin is below 8 g/dL. Go to the next page if you knew the correct answer, or click the link image(s) below to further research the concepts in this question (if desired).

Research Concepts: Hereditary Spherocytosis

We update eBooks quarterly and Apps daily based on user feedback. Please tap flag to report any questions that need improvement.

Question 137: A 64-year-old male presents to the emergency department with complaints of shortness of breath and chest palpitations. He has a significant history of hypertension, chronic obstructive pulmonary disease (COPD), and is an active tobacco smoker. He states that he recently had upper respiratory symptoms about one week ago, and since has progressively had worsening shortness of breath. Medications include an albuterol inhaler, maintenance combination inhaler, and hydrochlorothiazide. On exam, vitals are blood pressure 150/85 mmHg, heart rate 160 bpm, respiratory rate 24/minute, and oxygen saturation 94% on 2 liters by nasal cannula. Cardiac exam reveals tachycardia and regular rhythm. Lung exam reveals bilateral endexpiratory wheezes. ECG reveals narrow complex tachycardia at 160 bpm with electrical alternans. Chest x-ray reveals hyperinflated lung volumes. Which of the following is the next step to control the patient's heart rate?

Choices: 1. Adenosine given at an initial decreased 3 mg bolus secondary to COPD 2. Vagal maneuvers 3. Diltiazem 4. Digoxin

Answer: 2 - Vagal maneuvers Explanations: Active wheezing is a contraindication for adenosine administration as it can cause bronchospasm, worsening respiratory failure in these patients. ACC/AHA/HRS guideline therapy for first-line management of supraventricular tachycardia (SVT) in a hemodynamic stable adult is vagal maneuvers. Other pharmacologic therapies for SVT include nondihydropyridine calcium channel blockers and beta blockers are considered as second-line therapy in patients who fail firstline therapy or have a contraindication for use. Adenosine, as well as dipyridamole, are also contraindicated for pharmacologic stress testing in patients with active wheezing secondary to the risk of bronchospasm subsequent respiratory failure. Go to the next page if you knew the correct answer, or click the link image(s) below to further research the concepts in this question (if desired).

Research Concepts: Supraventricular Tachycardia

We update eBooks quarterly and Apps daily based on user feedback. Please tap flag to report any questions that need improvement.

Question 138: A male patient with a mechanical aortic valve is on warfarin. He now presents to the emergency department with life-threatening bleeding. His blood pressure is in the 90s and his heart rate is 130 bpm. His INR is 6 and he has already lost about 700 ml following a lower gastrointestinal bleed. His hematocrit is 18 and the hemoglobin is 5.4. Besides resuscitation, what is the best approach to reverse his bleeding?

Choices: 1. Immediate 10 mg intravenous vitamin K 2. 12 units of intravenous fresh frozen plasma 3. Prothrombin complex concentrate 4. Desmopressin followed by cryoprecipitate

Answer: 3 - Prothrombin complex concentrate Explanations: Prothrombin complex concentrate is the ideal way to reverse serious bleeding in patients on warfarin. The concentrate is not always available and should only be used in life-threatening cases of bleeding. It is used in hemophiliacs and in patients who need emergency surgery while on warfarin. The prothrombin complex is not used in patients with known heparin-induced thrombocytopenia as it does contain heparin. The complex contains clotting factors 2, 9 and 10 and some versions also contains factor 7. Minor bleeding should be managed with vitamin K. Go to the next page if you knew the correct answer, or click the link image(s) below to further research the concepts in this question (if desired).

Research Concepts: Warfarin Toxicity

We update eBooks quarterly and Apps daily based on user feedback. Please tap flag to report any questions that need improvement.

Question 139: A 45-year-old female lawyer presents one hour after experiencing constant projectile vomiting lasting 10 to 20 minutes just before a court hearing. She had trouble walking out of the bathroom at the courthouse and called 911 using her cell phone for help. On arrival to the emergency department, she is sitting up in the stretcher with her eyes closed but awake, alert, oriented, and appropriate. Her vital signs are heart rate of 82 bpm, blood pressure 122/85 mmHg, respiratory rate of 15/min, temperature of 98.2F, and a point-of-care glucose is 105. She says she still feels nauseous but has not vomited in the last 30 minutes. She also complains that she feels as though she’s spinning constantly, and her voice is slightly slurred but still readily understandable. On exam, she has some dysmetria, and her gait demonstrates mild ataxia. She has intact extraocular movements, no facial weakness, and vertical nystagmus. She complains of discomfort when you palpate her abdomen, worst in the epigastrium, but no guarding or rebound. What is the most appropriate imaging modality to identify the cause of her symptoms?

Choices: 1. Unenhanced magnetic resonance imaging (MRI) of the head 2. MRI of the head with diffusion-weighted (DWI) and angiographic (MRA) sequences 3. Unenhanced computed tomography (CT) of the head 4. Computed tomographic arteriogram (CTA) of the head and neck

Answer: 2 - MRI of the head with diffusion-weighted (DWI) and angiographic (MRA) sequences

Explanations: Of these options, magnetic resonance imaging including diffusion-weighted and angiographic sequences would be most helpful to guide the evaluation of a possible cerebral infarct because it can localize an area of perfusion deficit, a vessel occlusion, and possibly also the presence of collateral feeding vessels. Without these sequences, MRI alone will likely miss an early stroke. While CTA of the head and neck can be helpful, especially if the patient had other ipsilateral signs suggesting a larger vessel in the neck, a separate CT perfusion sequence would also be required to provide comparable information. CT of the head without contrast is often an important part of emergent evaluation look for a hemorrhagic infarct, but has comparatively poor sensitivity in the posterior fossa due to surrounding osseous structures, especially in the early phases of an infarct. Though the patient has some epigastric tenderness, this may be due to her extensive vomiting, and as she has normal vital signs at this time, the potential for a neurologic emergency should be ruled out prior to an abdominal one so imaging of the abdomen would not be indicated first. Go to the next page if you knew the correct answer, or click the link image(s) below to further research the concepts in this question (if desired).

Research Concepts: Cerebellar Infarct

We update eBooks quarterly and Apps daily based on user feedback. Please tap flag to report any questions that need improvement.

Question 140: A 45-year-old male presented with an intentional overdose of his home regimen of antihypertensive medication. He is otherwise healthy apart from a history of hypertension. The initial presentation was with profound refractory shock and bradycardia requiring initiation of norepinephrine ( running at eight mcg/minute) and vasopressin. His mean arterial pressure is 59 mm Hg. A minimally invasive solution (arterial line) that provides dynamic and flow-based hemodynamic monitor was used to assess hemodynamics at the bedside and his cardiac output is 5.1 liters/min with a stroke volume variability of 11%. 6 hours after the presentation he starts becoming rapidly short of breath and was therefore electively intubated. On low tidal volume ventilator settings with a PEEP of 5 and FIO2 of 100%, his PO2 is 82 mm Hg. Chest x-ray shows diffuse bilateral infiltrates worse than prior. Which intervention will be most effective at his condition to improve his outcome?

Choices: 1. Additional IV fluids as his stroke volume variation is elevated 2. Addition of milrinone as an inotrope 3. Increasing dose of insulin dextrose infusion 4. Referral for extracorporeal membrane oxygenation

Answer: 4 - Referral for extracorporeal membrane oxygenation Explanations: With a normal cardiac output, and significantly low P/F ratio and bilateral alveolar infiltrates, this individual is likely developing acute respiratory distress syndrome with noncardiogenic pulmonary edema, a manifestation seen in rare cases of calcium channel blocker overdose Beyond low tidal volume mechanical ventilation, strategies that would improve clinical outcome if instituted early will be prone ventilation or referral to extracorporeal membrane oxygenation (ECMO). If cardiac function is well preserved V- V ECMO should be considered without further delay His stroke volume variation is within 13%. cardiac output is normal. Neither additional fluids nor inotropes will be helpful in the scenario. Fluids may worsen his pulmonary edema and respiratory failure. Go to the next page if you knew the correct answer, or click the link image(s) below to further research the concepts in this question (if desired).

Research Concepts: Calcium Channel Blocker Toxicity

We update eBooks quarterly and Apps daily based on user feedback. Please tap flag to report any questions that need improvement.

Question 141: A 68-year-old male presents with sudden onset of back pain. He says he has never had back pain before and denies any trauma. He says he just does not feel good and feels like he is going to die. On examination, he looks ill with a blood pressure of 70/54 mmHg, a pulse of 110 beats/min, and respirations 18. His abdomen is tender to palpation. There is a large pulsatile mass. What is the next step in his management?

Choices: 1. Fast track ultrasound 2. CT scan of the abdomen 3. Angiogram of abdomen 4. Surgery

Answer: 4 - Surgery Explanations: In any patient who is hypotensive with signs of abdominal distension, the diagnosis of a ruptured aneurysm must be at the top of the list. These patients need urgent surgery. The individual should never be sent to a CT scan suite. An ultrasound can be performed in the operating room while the patient is receiving general anesthesia. A ruptured aortic aneurysm has a mortality of 90%. Go to the next page if you knew the correct answer, or click the link image(s) below to further research the concepts in this question (if desired).

Research Concepts: Abdominal Aortic Aneurysm

We update eBooks quarterly and Apps daily based on user feedback. Please tap flag to report any questions that need improvement.

Question 142: A 56-year-old female is in the hospital 3 days after an aneurysmal subarachnoid hemorrhage. She has hyponatremia with serum sodium of 130 meq/L with decreased skin turgor and elevated hematocrit. Her urine sodium is elevated. Her vital signs are unremarkable and chloride is within the reference range. The patient is conscious, alert, and well oriented to time, place, and person. What is the most rational initial step in the management of the patient presenting with such clinical characteristics?

Choices: 1. Fluid supplementation with isotonic saline 2. Fluid restriction 3. Nimodipine 4. Hypertonic saline

Answer: 1 - Fluid supplementation with isotonic saline Explanations: Cerebral salt wasting syndrome is characterized by hyponatremia with elevated urine sodium in a hypovolemic state, as evidenced by decreased skin turgor, hypotension, decreased central venous pressure, and elevated hematocrit. Syndrome of inappropriate secretion of antidiuretic hormone (SIADH) is characterized by hyponatremia with elevated urine sodium in a hypervolemic to euvolemic state. Cerebral salt wasting syndrome can be differentiated from SIADH by fluid status. In cerebral salt wasting, the patient is hypovolemic. In SIADH, the patient is hypervolemic to euvolemic. Cerebral salt wasting syndrome is treated with fluid resuscitation. SIADH is treated with fluid restriction. Go to the next page if you knew the correct answer, or click the link image(s) below to further research the concepts in this question (if desired).

Research Concepts: Cerebral Salt Wasting Syndrome

We update eBooks quarterly and Apps daily based on user feedback. Please tap flag to report any questions that need improvement.

Question 143: A 50-year-old male presented with hemoptysis and melena. He had following vitals. Blood pressure 90/60 mmHg and pulse 105 bpm. CBC shows a hemoglobin 9.8 g/dL, platelet count 120,000/microL. He was managed for a variceal bleed. Endoscopy and band ligation were done. Which of the following drug could be used as prophylaxis in this patient to stop further episodes?

Choices: 1. Octreotide 2. Terlipressin 3. Omeprazole 4. Propranolol

Answer: 4 - Propranolol Explanations: Propranolol is used as prophylaxis for variceal bleed. It has least cardiac side effects. Octreotide and terliprresin are used for acute management of variceal bleed. Other vasopressors can cause coronary vasospasm and cannot be used for long term. Go to the next page if you knew the correct answer, or click the link image(s) below to further research the concepts in this question (if desired).

Research Concepts: Chronic Liver Disease

We update eBooks quarterly and Apps daily based on user feedback. Please tap flag to report any questions that need improvement.

Question 144: A 73-year-old African American female with past medical history of congestive heart failure, atrial fibrillation, hypertension, type 2 diabetes mellitus, and hyperlipidemia presents to the emergency department with dizziness and palpitations. The patient gives a history of being on digoxin. On physical examination, she has a regular heart rate of 50 bpm; blood pressure is 133/72 mmHg, respiratory rate of 18/minute, and temperature of 98.4 F. Her electrocardiogram shows junctional tachycardia with no specific ST-T wave changes. Her laboratory testing shows toxic levels of digoxin. What is the next step in the management of this patient?

Choices: 1. Give 1 liter of intravenous normal saline bolus and monitor the patient 2. Administer digoxin-specific antibody 3. Administer intravenous atropine and digoxin-specific antibody 4. Intravenous phenytoin

Answer: 3 - Administer intravenous atropine and digoxin-specific antibody Explanations: This patient is currently suffering from digoxin toxicity. Junctional tachycardia is a manifestation of digoxin toxicity, and the rhythm should be terminated with intravenous medications. Intravenous atropine and digoxin-specific antibody should be administered to a patient presenting with digoxin toxicity and bradycardia. Intravenous phenytoin is used to treat patients who are refractory to other pharmacological treatments, and it should be given in only monitored setting due to its side-effect of hypotension. Go to the next page if you knew the correct answer, or click the link image(s) below to further research the concepts in this question (if desired).

Research Concepts: Digoxin Toxicity

We update eBooks quarterly and Apps daily based on user feedback. Please tap flag to report any questions that need improvement.

Question 145: A 65-year-old male presented to the emergency department with shortness of breath. Initial arterial blood gas (ABG) showed a pH of 7.31 with a partial pressure of carbon dioxide (PCO2) at 44 mmHg, and partial pressure of oxygen (PO2) at 45 mmHg. A chest x-ray showed bilateral pulmonary infiltrates. Non-invasive ventilation with a bilevel positive airway pressure mask (BiPAP) was tried without success. His blood pressure dropped to 80/40 mmHg. He was intubated immediately. He was treated with antibiotics, vasopressors, and corticosteroids for septic shock due to bilateral pneumonia. Over the next few days, the patient ‘s clinical status improved, and he was no longer in need of vasopressors. The patient was placed on a spontaneous breathing trial. After 40 minutes, his ABG shows a pH 7.39, PCO2 of 38 mmHg, and PO2 of 63 mmHg on 60% oxygen at 5 cm of water of positive end-expiratory pressure (PEEP). He is following commands and has mild to moderate airway secretions. Which of the following is the next best step in the management of this patient?

Choices: 1. Avoid extubation due to high oxygen requirements and attempt weaning trial the next morning 2. Extubate patient to non-rebreather mask and consider non-invasive ventilation at night and during daytime naps 3. Extubate patient to venturi face mask 50% and consider high flow nasal cannula if clinical status or gas exchange deteriorates 4. Extubate patient to high flow nasal cannula

Answer: 4 - Extubate patient to high flow nasal cannula Explanations: High flow nasal cannula offers alternative therapy for patients with acute hypoxic respiratory failure in the immediate post-extubation phase as it has been linked to positive clinical and physiologic outcomes. High flow nasal cannula allows for high oxygen supplementation without concerns for airway secretions dryness, which could impair mucosal cilia clearance and pulmonary toilette. Low flow Oxygen delivery systems such as face mask do not deliver humidified oxygen, leading to drying of the secretions and potentially compromising clearance of this patient's secretions during the immediate post-extubation period. High flow nasal cannula, different from non-invasive ventilation, will not interfere and facilitate airway secretion clearance. High flow oxygen systems offer the advantage of providing PEEP to improve oxygenation and alveolar recruitment. This mechanism of oxygenation improvement is not found with low flow oxygen systems. Go to the next page if you knew the correct answer, or click the link image(s) below to further research the concepts in this question (if desired).

Research Concepts: High Flow Nasal Cannula

We update eBooks quarterly and Apps daily based on user feedback. Please tap flag to report any questions that need improvement.

Question 146: Clindamycin is often added to antibiotic coverage for necrotizing fasciitis which already includes carbapenems or combinations like vancomycin with piperacillin/tazobactam. Why is clindamycin used to treat necrotizing fasciitis?

Choices: 1. To cover Clostridium butyricum 2. To decrease the effects of tissue necrosis factor 3. To act as a bacterial protein synthesis/toxin inhibitor 4. To improve gram-negative antibiotic coverage

Answer: 3 - To act as a bacterial protein synthesis/toxin inhibitor Explanations: Protein synthesis inhibitors, such as clindamycin, may help by inhibiting toxin production, which can be crucial for controlling the inflammatory response in patients with necrotizing fasciitis. Clindamycin’s main role is with necrotizing fasciitis caused by Group A Streptococcus. Because the causative organism is not known at the time of antibiotic initiation, clindamycin should be used initially in all cases of necrotizing fasciitis. Clindamycin has been found to be beneficial even for infections caused by clindamycinresistant strains. It is likely due to the virulence factor-inhibiting properties of clindamycin rather than a directly lethal effect on group A Streptococcus. Clindamycin has no Gramnegative activity and is effective only against Gram-positives. Clostridium species are often found in necrotizing fasciitis. Most commonly Clostridium perfringens and Clostridium sordellii, but occasionally others. Clostridium butyricum rarely causes disease in humans. Go to the next page if you knew the correct answer, or click the link image(s) below to further research the concepts in this question (if desired).

Research Concepts: Necrotizing Fasciitis

We update eBooks quarterly and Apps daily based on user feedback. Please tap flag to report any questions that need improvement.

Question 147: A 68-year-old male with a history of hypertension presents to the emergency department with a sudden onset headache, nausea, vomiting, and dizziness. A CT scan of the head is unremarkable with no intracranial hemorrhage and no acute ischemic lesions seen. He has a non-focal neurologic exam while lying down. As soon as he stands up, he has severe unsteadiness and falls to the floor. He is unable to walk. What is the most appropriate next step in the management of this patient?

Choices: 1. Discharge the patient with an anti-emetic and follow up with his primary care provider the next day 2. Order an MRI brain and MR angiogram of the intracranial and neck vessels 3. Admit him under "observation" with a diagnosis of vertigo, with neurological evaluation every 8 hours 4. Perform an urgent lumbar puncture looking for a subarachnoid hemorrhage

Answer: 2 - Order an MRI brain and MR angiogram of the intracranial and neck vessels Explanations: The patient has symptoms suggestive of either an acute vestibular disorder or an acute cerebellar ischemic event, for which a CT is commonly unrevealing. An MRI brain with diffusion-weighted imaging and an MR angiogram are the best way of imaging the posterior fossa and posterior circulation when there is a negative CT. Lumbar puncture is contraindicated in cases where the posterior fossa pressure gradient is likely to be increased, as in ischemic vertebrobasilar stroke. The risk of herniation of the edematous cerebellum in this context is high. This patient should be admitted, with frequent neurological evaluations to observe for mental status changes from obstructive hydrocephalus. However, the best initial step is ordering an MRI brain and MR angiogram. Go to the next page if you knew the correct answer, or click the link image(s) below to further research the concepts in this question (if desired).

Research Concepts: Ischemic Stroke

We update eBooks quarterly and Apps daily based on user feedback. Please tap flag to report any questions that need improvement.

Question 148: A 48-year-old male comes in with altered mental status. The patient is very agitated, and dexmedetomidine is started. After 1 hour he spikes a fever of 102 degrees F and is still very agitated and trying to climb out of bed. He was intubated, and lumbar puncture is done. Lumbar puncture shows no WBCs and meningitis is ruled out. His labs show acute kidney injury with elevated CK levels. As the patient takes SSRI and baclofen, a working diagnosis of serotonin syndrome was established. After 3 days of sedation, vacation is attempted, and the patient becomes extremely agitated. The patient was started on antipsychotics and sedation vacation was again attempted for the next 3 days with no difference in the patient’s mental status and agitation. He was started on dexmedetomidine and sedation vacation was attempted again with no help. What’s the next best step?

Choices: 1. Restart baclofen 2. Extubate on ketamine 3. Extubate on fentanyl 4. Extubate on propofol

Answer: 1 - Restart baclofen Explanations: The patient is likely withdrawing from baclofen, and it should be restarted. The reason for sedation vacation or breathing trial failing should be identified and addressed accordingly. If the patient is getting agitated and is delirious, antipsychotics can be tried; if anxious, anxiolytics can be tried; or if in pain, low dose fentanyl drip/morphine drip or patch can be tried. Patient delirium and agitation can be very well controlled on dexmedetomidine, and as the medication does not suppress the respiratory center, the patient can be extubated while on continuous infusion of dexmedetomidine. Attention should be paid to home medications, and if clinically permitted, a patient who is on any anxiolytics, antipsychotics, or any chronic pain medications should continue them while sedated or restart them in a timely fashion to prevent withdrawal. Go to the next page if you knew the correct answer, or click the link image(s) below to further research the concepts in this question (if desired).

Research Concepts: Sedation Vacation in the ICU

We update eBooks quarterly and Apps daily based on user feedback. Please tap flag to report any questions that need improvement.

Question 149: A 61-year-old woman with a history of focal seizures and right parietal cerebrovascular accident with no notable deficits presents to the emergency department after being found down in her apartment for an unknown length of time. She was found by a friendly neighbor who "heard a thud." She has no evidence of head trauma but appears confused. The patient is seen and evaluated the patient. She can follow simple commands, and her neurologic exam reveals 5/5 strength excluding her left upper extremity (LUE) which is 1/5. It is otherwise unremarkable. There is no evidence of swelling, pain or trauma to the extremity. CT brain showed no evidence of intracranial hemorrhage. On rounds the next day her confusion seems to have resolved, but her LUE weakness remains and is unchanged. The MRI brain is complete but is unavailable for review due to what IT calls "technical difficulties." The following day on rounds, approximately 34 hours after being found, the patient has regained 5/5 strength in her LUE. IT has gone on vacation starting yesterday and has not fixed the "technical issue" yet. Which of the following is the most likely cause of the patient's findings?

Choices: 1. Transient ischemic attack 2. Autoimmune myositis flare 3. Conversion disorder 4. Todd paresis

Answer: 4 - Todd paresis Explanations: Todd paresis is the most likely cause of her transient LUE weakness. The patient had a history of focal seizures which are strongly associated with the condition. She had a history of right parietal CVA which can cause scarring and may act as a focus for focal seizure activity. Her symptoms resolved completely in 34 hours which rules out TIA as this is a less than a 24-hour condition. Symptoms of Todd paresis resolve, on average, by about 15 hours but can range from 30 minutes to 36 hours before resolution. Conversion disorder is incorrect. While possible, the patient had no history given relating psychiatric disorders of any kind or increased stressors, making this not the best answer. Autoimmune myositis is not associated with focal, transient extremity weakness. A transient ischemic attack is incorrect as, by definition, it is limited by 24 hours duration. Go to the next page if you knew the correct answer, or click the link image(s) below to further research the concepts in this question (if desired).

Research Concepts: Todd Paresis

We update eBooks quarterly and Apps daily based on user feedback. Please tap flag to report any questions that need improvement.

Question 150: A patient post coronary artery bypass graft (CABG) surgery presents with shortness of breath, fatigue, peripheral edema, and ascites. ECG reveals non-specific ST changes. The chest x-ray shows cardiomegaly with flecks of calcium around the pericardium. What is the best treatment for this patient?

Choices: 1. Non-steroidal anti-inflammatory agents 2. Corticosteroids 3. Pericardiectomy 4. Pericardial drainage

Answer: 3 - Pericardiectomy Explanations: Constrictive pericarditis can occur after coronary artery bypass graft (CABG) surgery. It causes dense adhesions of the pericardium to the epicardium. This results in progressive dyspnea, fever, peripheral edema, and fatigue. The jugular venous pressure (JVP) is elevated but pulsus paradoxus is rare. Echocardiogram will show pericardial thickening but a right heart catheterization will show equilibration of diastolic pressures and demonstrate a diastolic dip (square root sign) in right ventricular pressure. Pericardiectomy is the best treatment for this patient. Go to the next page if you knew the correct answer, or click the link image(s) below to further research the concepts in this question (if desired).

Research Concepts: Constrictive Pericarditis

We update eBooks quarterly and Apps daily based on user feedback. Please tap flag to report any questions that need improvement.

Question 151: A 93-year-old male is being evaluated in the emergency department after an episode of syncope. The last thing he remembers is shopping at the grocery store and pushing the cart. He did not sustain any injuries. Medical history is significant for hypertension. Otherwise, he is healthy and independent. His physical exam is significant for a crescendo-decrescendo murmur at the right sternal border with radiation to carotids bilaterally. EKG shows a right bundle branch block (RBBB). 2D-echocardiogram confirms the diagnosis of severe aortic stenosis. After detailed consideration, a transcatheter aortic valve replacement (TAVR) is planned. Which of the following statements is most accurate about this treatment for this patient?

Choices: 1. He is at a higher risk for further conductions disturbance and the need for a permanent pacemaker 2. Worsening hypertension after TAVR would worsen the overall survival 3. The mortality rate would have been the same if this patient undergoes aortic balloon valvuloplasty 4. His risk for further conductions disturbance and the need for a permanent pacemaker is similar to other patients without RBBB

Answer: 1 - He is at a higher risk for further conductions disturbance and the need for a permanent pacemaker

Explanations: Studies have shown that RBBB at baseline is associated with a high incidence of permanent pacemaker implantation post-TAVR. Symptomatic aortic stenosis has a high mortality if left untreated. TAVR is a great alternative treatment in patients with symptomatic aortic stenosis. Development or worsening of hypertension after TAVR is associated with improved survival despite an increase in postoperative heart failure exacerbations. Patients who undergo valvuloplasty have a higher mortality rate and it can be used as a bridge to surgical aortic valve replacement or TAVR. Go to the next page if you knew the correct answer, or click the link image(s) below to further research the concepts in this question (if desired).

Research Concepts: Transcatheter Aortic Valve Replacement

We update eBooks quarterly and Apps daily based on user feedback. Please tap flag to report any questions that need improvement.

Question 152: A 67-year-old female is admitted to the hospital with a 1-week history of fatigue, fever, nausea and abdominal pain. She has history of hypertension, hyperlipidemia, diabetes mellitus and osteoarthritis. She takes metoprolol, simvastatin, glyburide, and ibuprofen at home. She was started on amoxicillin-clavulanate for sinusitis and received a total of 10 days of treatment one month ago. There is no history of travel in the last year. Examination reveals jaundice. The liver is palpable 3 cm below the right subcostal margin. Lab investigations reveals WBC count 13,400/L, hemoglobin 11.2 g/dL, platelet count 156x10^6, sodium 135 mEq/L, potassium 4.4 mEq/L, albumin 3.5 mg/dl, aspartate aminotransferase (AST) 150U/L, alkaline phosphatase 345 IU/L, bilirubin 1.8 mg/dl, and INR 1.0. Hepatitis panel is negative. Epstein Barr virus and Cytomegalovirus are negative. Ultrasound reveals normal a liver and common bile duct. Which of the following is the most likely cause?

Choices: 1. Simvastatin 2. Glyburide 3. Amoxicillin-clavulanate 4. Metformin

Answer: 3 - Amoxicillin-clavulanate Explanations: Simvastatin can cause elevation of alanine aminotransferase (ALT) but usually resolves even after the continuation of the treatment. Very rarely it causes liver disease 3 years after the start of treatment. Glyburide can cause drug-induced hepatitis. It causes hepatotoxicity in less than 1% of patients. This usually appears within 3 to 12 weeks of starting therapy. It can cause hepatotoxicity months to years after changing the dose. Metoprolol can cause elevation of aminotransferase which is transient and improves even after ongoing treatment. Amoxicillin-clavulanate is the most common cause of drug-induced liver toxicity and usually presents 3 to 8 weeks after the start of treatment. Go to the next page if you knew the correct answer, or click the link image(s) below to further research the concepts in this question (if desired).

Research Concepts: Liver Toxicity

We update eBooks quarterly and Apps daily based on user feedback. Please tap flag to report any questions that need improvement.

Question 153: A 17-year-old patient was involved in a motor vehicle accident. He was found to be hypotensive at the scene of the accident and had to be resuscitated. He suffered multiple injuries to his lower extremities and required numerous surgeries and prolonged mechanical ventilation. He was started on a high concentration of enteral glucose feeds on a 24hour protocol. Four weeks later, he is still dependent on the mechanical ventilator, but his chest x-ray remains clear. He has profound respiratory muscle weakness and the MRI shows significant thinning of the diaphragm. Which of the following is the most likely cause of his muscle weakness?

Choices: 1. Hypocalcemia 2. Hypophosphatemia 3. Zinc deficiency 4. Hypernatremia

Answer: 2 - Hypophosphatemia Explanations: Low phosphate levels can cause significant weakness of skeletal and smooth muscle, which can affect the eyes as well as the diaphragm. Respiratory insufficiency in patients on ventilators is common after hypophosphatemia. Impaired cardiac contractility can also occur and usually reverses when phosphate is reinstituted in the diet. Administering high carbohydrate diets can lead to a lowering of phosphate by stimulating insulin release. Insulin moves phosphate into the cells. Diabetic ketoacidosis is another important cause of hypophosphatemia. However, routine replacement of phosphate in these patients has not been shown to reduce morbidity or mortality. Go to the next page if you knew the correct answer, or click the link image(s) below to further research the concepts in this question (if desired).

Research Concepts: Hypophosphatemia

We update eBooks quarterly and Apps daily based on user feedback. Please tap flag to report any questions that need improvement.

Question 154: A patient is rushed to the medical facility in Surinam from a local smallscale gold mine. He was working on separating the gold from amalgam when he suddenly started coughing and became short of breath. Upon arrival, he is in respiratory distress. His vitals are heart rate 144 bpm, respiratory rate 26/minute, blood pressure 146/92 mmHg, and pulse oximetry 82%. He is intubated. His chest x-ray is concerning for acute respiratory distress syndrome. Resuscitation is started but what other therapy would be indicated?

Choices: 1. Sodium bicarbonate 2. British anti-Lewisite (BAL) 3. Methylprednisolone 4. Activated charcoal

Answer: 2 - British anti-Lewisite (BAL) Explanations: Mercury is used in the mining of gold by binding gold creating an amalgam. The mercury is then separated from the gold by heating and creating mercury vapor. Small-scale gold mining using mercury affects 10 to 15 million miners worldwide. Eleven percent of human-generated mercury and 5 percent of all mercury released is generated from gold production. Following an inhalation, patients develop acute shortness of breath and can go into acute respiratory distress (ARDS) rapidly. Patients should be treated aggressively and may need to be suctioned if it is an acute aspiration. Go to the next page if you knew the correct answer, or click the link image(s) below to further research the concepts in this question (if desired).

Research Concepts: Mercury Toxicity

We update eBooks quarterly and Apps daily based on user feedback. Please tap flag to report any questions that need improvement.

Question 155: A 17-year-old, previously healthy, female presents with an acute flu-like illness, fever, myalgias, shortness of breath, and chest pain. Her ECG shows sinus tachycardia. Her labs show elevated white blood cells and an abnormally elevated troponin. Her chest x-ray shows mild pulmonary edema with findings consistent with congestive heart failure. What is her likely diagnosis?

Choices: 1. Hypertrophic obstructive cardiomyopathy 2. Brugada syndrome 3. Acute myocarditis 4. Prolonged QT

Answer: 3 - Acute myocarditis Explanations: Hypertrophic obstructive cardiomyopathy is an inherited, usually autosomal dominant, cardiac disorder that may lead to sudden death, typically in young people. The chief abnormality found is left ventricular hypertrophy with asymmetrical thickening of the anterior interventricular septum. Classically, patients will present with exertional syncope caused by acute left ventricular outlet obstruction and ventricular dysrhythmias with the potential for sudden death. Brugada syndrome is a well-known cause of sudden death secondary to a genetic mutation in cardiac sodium channels. Also known as a sodium channelopathy, over 60 mutations have been described. Classic ECG findings include the "Brugada sign" in type 1 Brugada syndrome with a coved ST-segment elevation greater than 2 mm in more than one of V1-V3 followed by a negative T wave. Other types have been described as well. This abnormal sodium channel with corresponding ECG findings can lead to sudden death. It presents as ventricular dysrhythmias such as ventricular fibrillation and ventricular tachycardias. Myocarditis, by definition, is an inflammatory process of the myocardium that is caused mostly by viruses. Patients typically will present with a flu-like illness, with symptoms including fever, malaise, myalgias, vomiting, and even diarrhea. Adults typically will present with dyspnea, chest pain, and arrhythmias. Vital signs typically are abnormal, including fever, tachycardia, tachypnea, and sometimes hypotension. No single sign or symptom is specific to make the diagnosis, but a presentation with chest pain or congestive heart failure often indicates a poor prognosis. The QT interval is measured from the beginning of the Q wave to the end of the T wave. The corrected QT or QTc interval is considered long if greater than 440 milliseconds in men and 460 milliseconds in women. Any QTc greater than 500 milliseconds poses a significantly increased risk for torsades de pointes. Causes of prolonged QT include hypokalemia, hypomagnesemia, hypocalcemia, certain drugs, and congenital prolonged QT syndrome. Patients often are asymptomatic but also may present with syncope or ventricular dysrhythmias such as torsades de pointes. Go to the next page if you knew the correct answer, or click the link image(s) below to further research the concepts in this question (if desired).

Research Concepts: Viral Myocarditis

We update eBooks quarterly and Apps daily based on user feedback. Please tap flag to report any questions that need improvement.

Question 156: A 51-year-old man comes to the emergency department with worsening shortness of breath for the past two weeks. He reports a history of human immunodeficiency virus (HIV) infection, but he is not compliant with his antiviral or prophylactic therapy. On physical examination, he is in tachycardic, tachypneic, and using accessory muscles with prolonged expiratory phase and bilateral wheezing. A chest x-ray shows bilateral pulmonary infiltrates. A computed tomogram of the chest reveals bilateral ground-glass opacities scattered throughout both lung fields. His arterial blood gas shows pH 7.45, the partial pressure of carbon dioxide (PCO2) of 35 mmHg, the partial pressure of oxygen (PO2) at 45 mmHg, and bicarbonate level of 24 mmol/L. As the patient is extremely dyspneic but alert and awake, it is decided to attempt non-invasive mechanical ventilation. A few minutes after the institution of therapy, the patient becomes agitated, removes the non-invasive mask, and states, “ I don’t like it, I feel claustrophobic, and it makes my breathing worse.” Which of the following is the next best step in the management of this patient?

Choices: 1. Provide sedation and place the patient back on non-invasive ventilation while trying different mask sizes 2. Place the patient on high flow nasal cannula and monitor his gas exchange and mental status closely 3. Place the patient on high flow nasal cannula for preoxygenation while setting for endotracheal intubation 4. Place the patient on Venturi face mask at 50% and continue to coach the patient

Answer: 2 - Place the patient on high flow nasal cannula and monitor his gas exchange and mental status closely

Explanations: High flow nasal cannula is an effective alternative for the treatment of non-hypercarbic respiratory failure. This intervention could potentially avoid endotracheal intubation in a highly selected patient population. High flow nasal cannula enhances comfort as well as compliance with therapy. High flow nasal cannula offers an alternative intervention for patients who cannot tolerate noninvasive mechanical ventilation. High flow nasal cannula improves oxygenation prior to intubation when compared to other conventional oxygen supplementation systems. High flow nasal cannula provides decrease work of breathing by decreasing airway resistance, providing positive end-expiratory pressure (PEEP), and increase carbon dioxide washout. Go to the next page if you knew the correct answer, or click the link image(s) below to further research the concepts in this question (if desired).

Research Concepts: High Flow Nasal Cannula

We update eBooks quarterly and Apps daily based on user feedback. Please tap flag to report any questions that need improvement.

Question 157: A 52-year-old man presents to the emergency department via ambulance with altered mental status. His vital signs reveal blood pressure 80/55 mmHg, heart rate 127/min, temperature 103.5 F, respiratory rate 16/min, and SpO2 95% on room air. He is unable to provide any reliable history. Nursing staff report previous homelessness and alcohol-related emergency department visits from his electronic medical record. Physical examination is remarkable for a confused, disheveled male with fine diffuse tremors, severe tenderness, erythema, crepitus, and purulent drainage from a 3 cm eschar on the ventral surface of the scrotum. After initial resuscitation with intravenous fluids and antibiotics, what is the next best step in the management of this patient?

Choices: 1. CT scan of the abdomen/pelvis with IV contrast 2. Surgical debridement 3. Intravenous immunoglobin (IVIG) 4. Treatment for alcohol withdrawal

Answer: 2 - Surgical debridement Explanations: Broad-spectrum antibiotics and intravenous fluids should be started initially with emergent surgical consultation that coincides with resuscitation efforts. Radiographic imaging can be helpful, demonstrating air in the soft tissues as well as the depth of involvement but they should not be obtained if they delay the definitive surgical treatment. Fournier gangrene is usually due to polymicrobial infection, which includes facultative organisms and anaerobes. Surgical intervention, along with antibiotics, takes precedence in this case. A CT scan of the abdomen/pelvis is not currently indicated at this time, would delay care, and increase mortality. The administration of IVIG has been shown to confer some benefit in cases of necrotizing soft tissue infections related to streptococcal toxic shock syndrome, a monomicrobial infection. Clinical suspicion of alcohol dependence, which could manifest as alcohol withdrawal syndrome, in this case, is understandable. However, management should be focused on definitive care for the Fournier gangrene. Go to the next page if you knew the correct answer, or click the link image(s) below to further research the concepts in this question (if desired).

Research Concepts: Fournier Gangrene

We update eBooks quarterly and Apps daily based on user feedback. Please tap flag to report any questions that need improvement.

Question 158: A patient has been receiving mechanical ventilation with a balloon-cuffed tracheostomy tube for the past 3 weeks after suffering a severe blunt laryngeal injury. On two occasions within 24 hours, about 30 ml of blood have been suctioned from the tracheostomy tube. What is the most likely source of the bleeding?

Choices: 1. Lung pathology 2. Granulation tissue at the tracheostomy site 3. Erosion into the internal jugular vein 4. Erosion into the innominate artery

Answer: 4 - Erosion into the innominate artery Explanations: Any patient with tracheal bleeding that amounts to more than blood streaking of the tracheal aspirate, 48 hours or more after tracheostomy, must be assumed to have tracheal erosion into the innominate artery until proven otherwise. Episodes of such minimal blood loss tend to precede or herald the development of a massive and frequently fatal hemorrhage. Recognition of the significance of such seemingly minor bleeding episodes, with appropriate subsequent management, offers the best chance for survival. Patients with severe bleeding should be immediately re-intubated and corrective surgery should be undertaken if the diagnosis is confirmed. Massive bleeding from the tracheostomy site invariably comes from an erosion of the innominate artery and is usually fatal unless rapidly controlled. In most cases, bleeding stops temporarily with further inflation of the balloon cuff until the patient is taken to the operating room. Sometimes the innominate artery can be compressed against the back of the sternum with a finger introduced through the tracheostomy incision, between the trachea and the artery. Go to the next page if you knew the correct answer, or click the link image(s) below to further research the concepts in this question (if desired).

Research Concepts: Tracheo Innominate Artery Fistula

We update eBooks quarterly and Apps daily based on user feedback. Please tap flag to report any questions that need improvement.

Question 159: A 17-year-old male is brought into the emergency department by paramedics after jumping off a two-story building. When found, he was unresponsive on a cement parking lot. On arrival to the emergency department, the Glasgow coma scale is 6 and the patient is immediately intubated for airway protection. On initial evaluation, there is a large area of ecchymosis behind his right ear. Which of the following sets of vital signs indicates impending brain herniation?

Choices: 1. A heart rate of 130 beats per minute, blood pressure of 90/50 mmHg, and respiratory rate of 30 breaths per minute 2. A heart rate of 115 beats per minute, blood pressure of 90/50 mmHg, and respiratory rate of 22 breaths per minute 3. A heart rate of 70 beats per minute, blood pressure of 205/120 mmHg, and respiratory rate of 23 breaths per minute 4. A heart rate of 48 beats per minute, blood pressure of 165/70 mmHg, and respiratory rate of 10 breaths per minute

Answer: 4 - A heart rate of 48 beats per minute, blood pressure of 165/70 mmHg, and respiratory rate of 10 breaths per minute

Explanations: Cushing reflex is a physiological response to increased intracranial pressure, which is seen in severe traumatic brain injury. Cushing triad is a late sign of impending herniation. Cushing triad includes bradycardia, hypertension, and respiratory depression. In children, it can cause either bradycardia or tachycardia. The presence of Cushing triad is an ominous condition and requires urgent evaluation and subsequent intervention. Go to the next page if you knew the correct answer, or click the link image(s) below to further research the concepts in this question (if desired).

Research Concepts: Cushing Reflex

We update eBooks quarterly and Apps daily based on user feedback. Please tap flag to report any questions that need improvement.

Question 160: A 41-year-old male presents to the emergency department for acute onset chest palpitations and discomfort. He states he has had similar episodes in the past; however, they usually resolve without intervention, and he has never seen a provider for it. He has no other known medical history and takes no medications. On examination, his blood pressure is 150/90 mmHg, heart rate 190 beats/min with regular rhythm, respiratory rate 24, and oxygen saturation 99% on room air. He appears to be in mild distress. His EKG shows significant tachycardia, a narrow complex at a ventricular rate of 180, and evidence of electrical alternans. Adenosine is planned to be administered. Which of the following should be discussed with the patient before administration?

Choices: 1. Adenosine can cause bowel irritation 2. Adenosine has been known to cause permanent central nervous injury 3. Most patients experience facial swelling and chest palpitations over the next 24 hours 4. He can expect to have facial flushing and chest discomfort, but symptoms are transient and rapidly resolve

Answer: 4 - He can expect to have facial flushing and chest discomfort, but symptoms are transient and rapidly resolve

Explanations: Facial flushing and chest pain are common with adenosine treatment. However, given the very short half-life of adenosine, these symptoms subside rapidly. Tremors and sympathetic overactivity occur with adrenergic agonist use. Because the symptoms experienced with the administration of adenosine can be frightening, patients should be informed of the side effects prior to administration. Other side effects of adenosine include dizziness, difficulty breathing, diaphoresis, and jaw discomfort. Go to the next page if you knew the correct answer, or click the link image(s) below to further research the concepts in this question (if desired).

Research Concepts: Adenosine

We update eBooks quarterly and Apps daily based on user feedback. Please tap flag to report any questions that need improvement.

Question 161: Three days after admission for an acute myocardial infarction, a 76-year-old man complains of severe, unremitting abdominal pain. Physical examination demonstrates pain that is out of proportion to the findings. Laboratory testing is significant for serum lactate level of 30 mg/dL (normal: 4.5-19.8 mg/dL) and white blood cell count 23,000 cells/uL. Serum creatinine level is 1.1 mg/dl. Which of the following is the most appropriate next step in management?

Choices: 1. Duplex ultrasonography 2. Mesenteric CT-angiography 3. Video laparoscopy 4. Abdominal CT with and without IV and oral contrast

Answer: 2 - Mesenteric CT-angiography Explanations: Intestinal ischemia is characterized by severe abdominal pain without significant findings on physical examination. The ischemia may be secondary to mesenteric vasospasm, occlusion of the mesenteric vessels by thrombotic or embolic events, or a low cardiac index. Mesenteric angiography is the gold standard for diagnosis. Direct infusion of vasodilators or thrombolytic therapy can be used if there is suspicion of ischemic bowel. Embolectomy or vascular bypass may be needed. If there is no occlusion, there should be attempts to increase cardiac output and dilate the mesenteric vessels. Abdominal CT should not be performed with oral contrast. Oral contrast will obscure the mesenteric vessels and bowel wall, causing a delayed diagnosis. Go to the next page if you knew the correct answer, or click the link image(s) below to further research the concepts in this question (if desired).

Research Concepts: Mesenteric Artery Ischemia

We update eBooks quarterly and Apps daily based on user feedback. Please tap flag to report any questions that need improvement.

Question 162: A patient in the ER was noted to have a significant renal contusion on imaging after he fell from a ladder. His vitals are stable and his labs are normal. What is the next step in management?

Choices: 1. Observation in the ICU with serial clinical exams, vital signs, and hematocrits every 6 to 8 hours 2. Discharge home with outpatient follow up with his primary care doctor 3. Admit patient to the floor for observation with follow-up hematocrit every 12 hours 4. Admit the patient to the floor and obtain a CT scan 12 hours after admission

Answer: 1 - Observation in the ICU with serial clinical exams, vital signs, and hematocrits every 6 to 8 hours

Explanations: Non-operative management in the hemodynamically stable renal trauma patient has a better overall outcome. Non-operative management initially entails supportive care in an intensive care unit (ICU) setting, serial clinical exams, serial hematocrits every 6 to 8 hours, transfusion of blood products and angioembolization or drain placement for urine leakage as indicated. Initial non-operative approach renal function can be preserved and unnecessary nephrectomies avoided. Patients with grade IV or V injuries, if managed non-operatively, should have a repeat contrast-enhanced CT 48 to 72 hours after the initial scan or earlier if clinically warranted to evaluate for the common complications of bleeding or urinoma in these high-grade injuries. Go to the next page if you knew the correct answer, or click the link image(s) below to further research the concepts in this question (if desired).

Research Concepts: Kidney Trauma

We update eBooks quarterly and Apps daily based on user feedback. Please tap flag to report any questions that need improvement.

Question 163: A 17-year-old male immigrant presents with a 6-month history of dyspnea on exertion. He complains of swollen legs. He had tuberculosis as a child that was treated with four drugs. ECG is normal. CT of the chest shows pericardial calcifications. Which of the following would not be expected on an exam?

Choices: 1. Pulsus paradoxus 2. Lack of decline in jugular pressure with inspiration 3. Ascites 4. Cannon A waves in the jugular veins

Answer: 4 - Cannon A waves in the jugular veins Explanations: The patient has congestive heart failure secondary to constrictive pericarditis. Physical examination findings in constrictive pericarditis include pulsus paradoxus and lack of decline in jugular pressure on inspiration. Ascites can be present with right-sided heart failure. Cannon A waves are seen with atrioventricular dissociation when the atria contracts against a closed valve. Go to the next page if you knew the correct answer, or click the link image(s) below to further research the concepts in this question (if desired).

Research Concepts: Constrictive Pericarditis

We update eBooks quarterly and Apps daily based on user feedback. Please tap flag to report any questions that need improvement.

Question 164: A 65-year-old man with a history of bitemporal hemianopsia is undergoing a surgical evaluation. MRI of the head and neck reveals a craniopharyngioma. Transsphenoidal hypophysectomy is performed without apparent complications. Two hours after surgery, he develops polyuria (375 mL/h) and hypernatremia (155 mEq/L). The patient describes severe thirst and seems lethargic and disoriented. Which of the following is the next best step in the management of this patient?

Choices: 1. Indomethacin 2. Amiloride 3. Intravenous desmopressin 4. Hydrochlorothiazide

Answer: 3 - Intravenous desmopressin Explanations: Diabetes insipidus is a common complication following pituitary surgery. Diabetes insipidus (DI) is a disease process that results in either decreased release of or response to antidiuretic hormone (ADH, also known as vasopressin or AVP), which can cause electrolyte imbalances. Most cases of diabetes insipidus following transsphenoidal hypophysectomy are transient. About 20% of patients who undergo neurosurgery will present with diabetes insipidus. Treatment is with fluid replacement and desmopressin. The most common findings in patients with diabetes insipidus are polydipsia, polyuria, and nocturia. Risk factors for developing diabetes insipidus due to transsphenoidal hypophysectomy include craniopharyngioma or extensive intraoperative handling of the pituitary gland. Go to the next page if you knew the correct answer, or click the link image(s) below to further research the concepts in this question (if desired).

Research Concepts: Diabetes Insipidus

We update eBooks quarterly and Apps daily based on user feedback. Please tap flag to report any questions that need improvement.

Question 165: A 17-year-old female is admitted with status asthmaticus. She is treated with continuous albuterol nebulization, intravenous corticosteroids, and bilevel positive airway pressure mechanical respiration. The patient's respiratory status improves, but she develops generalized weakness, fatigue, and myalgias. The cardiac monitor shows prolonged QT interval, flattened T waves, and ST depression. What is the most likely etiology of these changes?

Choices: 1. Hypocalcemia 2. Hypokalemia 3. Corticosteroid-induced myopathy 4. Cardiac ischemia

Answer: 2 - Hypokalemia Explanations: Beta-agonists, such as albuterol, can cause hypokalemia. This is secondary to insulin secretion and its action on the NaK ATPase. This increases the cellular uptake of potassium. Initially, this can cause fatigue, muscle weakness, and myalgias but can progress to complete paralysis and hypoventilation. Intravenous replacement of potassium is limited by pain and sclerosing of veins. Go to the next page if you knew the correct answer, or click the link image(s) below to further research the concepts in this question (if desired).

Research Concepts: Status Asthmaticus

We update eBooks quarterly and Apps daily based on user feedback. Please tap flag to report any questions that need improvement.

Question 166: A 54-year-old man working in the sanitation department presents with acute-onset shortness of breath. He says that over the past two days, he has suddenly become very short of breath. He denies smoking or any recent illness. His past medical history is unremarkable. He is admitted and requires immediate mechanical ventilation. Chest x-rays show diffuse, bilateral air-space opacities. Two days later, he expires. The pathology shows diffuse alveolar damage and fibrosis. What is the most likely diagnosis?

Choices: 1. Vaping associated pulmonary injury 2. Acute interstitial pneumonia 3. Cryptogenic organizing pneumonia 4. Bird flu

Answer: 2 - Acute interstitial pneumonia Explanations: Acute interstitial pneumonia (also known as Hamman-Rich syndrome) is an acute, rapidly progressive idiopathic pulmonary disease that often leads to fulminant respiratory failure and acute respiratory distress syndrome (ARDS). It can be distinguished clinically from other types of interstitial pneumonia by the rapid onset of respiratory failure in a patient without preexisting lung disease. Chest x-ray usually shows a pattern that is similar to ARDS, which is diffuse, bilateral air-space opacities. Acute interstitial pneumonia (AIP) has the histopathological pattern of diffuse alveolar damage (DAD) that is indistinguishable from the histologic pattern found in ARDS. The histologic discerption depends on the timing of the biopsy. The early phase (within a week of the initial tissue injury) is exudative, characterized by edema in the interstitium and alveolus. After that, a late phase, which is also called the organizing phase, shows fibroblastic proliferation and type 2 cell hyperplasia. Go to the next page if you knew the correct answer, or click the link image(s) below to further research the concepts in this question (if desired).

Research Concepts: Acute Interstitial Pneumonia

We update eBooks quarterly and Apps daily based on user feedback. Please tap flag to report any questions that need improvement.

Question 167: A 49-year-old female presents with 8 hours of progressive epigastric pain that worsens when recumbent and with radiation to her back with associated nausea. The patient is well known at your institution and has a well-documented history of alcohol abuse. On your evaluation, the patient’s vitals show temperature of 100.6 F, heart rate of 123 beats per minute, blood pressure of 95/60 mmHg, respiratory rate of 16 breaths per minute and saturating at 90% on ambient air. The physical exam reveals an uncomfortable and ill-appearing patient from whom it is difficult to elicit a history as she drifts off between sentences. Breath sounds are slightly diminished over the left posterior lung fields, and she displays epigastric tenderness with guarding but without rigidity or rebound tenderness. Bowel sounds are decreased but present. The patient’s laboratory results show hemoglobin of 17.0 g/dL, hematocrit of 58.0%, white blood cell count of 17 x 10^9/microL and platelets of 135,000/microL. The metabolic shows alanine aminotransferase (ALT) 62 U/L, aspartate aminotransferase (AST) 128 U/L, and alkaline phosphatase of 135 U/L with blood urea nitrogen of 34 mg/dL, and serum creatinine of 1.20 mg/dL. Lipase is 2524 U/L. The chest x-ray is significant for mild to moderate left pleural effusion without visualization of free air under the diaphragm and abdominal ultrasound without dilatation of the common bile duct or presence of any stones in the visualized biliary ducts but is unable to visualize the pancreas due to overlying bowel gas. What is the next best step in the management of this patient?

Choices: 1. Order CT of the abdomen with contrast to establish diagnosis of acute pancreatitis 2. Administer fluid bolus and admit to keep nil per os and for pain control 3. Start patient on piperacillin/tazobactam for empiric coverage of likely pancreatic necrosis 4. Admit patient to the intensive care unit for hydration with monitoring of fluid resuscitation with complete blood count and metabolic panel every 6 hours

Answer: 4 - Admit patient to the intensive care unit for hydration with monitoring of fluid resuscitation with complete blood count and metabolic panel every 6 hours

Explanations: The Bedside Index of Severity for Acute Pancreatitis (BISAP) score can be used to predict mortality in acute pancreatitis and has been shown to be similar in prognosticating mortality no other previous more complicated scoring systems such as Ranson's criteria or the APACHE II score. High scoring patients require intensive care unit admission. This patient scores high on the Bedside Index of Severity for Acute Pancreatitis (BISAP) with 4/5 points indicating a high risk of mortality and the patient will require close monitoring. Patients with 0 points have a less than a 1% risk of mortality while mortality increases significantly with a score of 3 or more. A score of five points is associated with a 22% mortality. In this question, the patient scores 4 out of 5 points and has a high risk for mortality In severe pancreatitis, BUN and hematocrit should be followed every 6 hours to ensure adequate fluid resuscitation. The first 12 to 24 hours of resuscitation are most crucial in improving outcomes. Go to the next page if you knew the correct answer, or click the link image(s) below to further research the concepts in this question (if desired).

Research Concepts: Acute Pancreatitis

We update eBooks quarterly and Apps daily based on user feedback. Please tap flag to report any questions that need improvement.

Question 168: A 65-year-old female presents with orthopnea, anorexia, and bilateral lower extremity edema. Physical exam reveals cachexia, jugular venous distention, a holosystolic, high-pitched, blowing murmur, bilateral rales, and hepatomegaly. Blood work shows an elevated total bilirubin, aspartate aminotransferase, and alanine aminotransferase. What is the best treatment for the suspected condition?

Choices: 1. Metoprolol 2. Digoxin 3. Furosemide 4. Nifedipine

Answer: 3 - Furosemide Explanations: Diuretics are the first-line option for cardiac cirrhosis. Aggressive diuresis is often required to reduce the edema. However, one must treat the primary cause of cardiac failure. There is no role of the use of beta blockers acutely in the setting of acute decompensated cardiac cirrhosis, especially if acute heart failure or cardiogenic shock could be the potential cause of cardiac cirrhosis. Go to the next page if you knew the correct answer, or click the link image(s) below to further research the concepts in this question (if desired).

Research Concepts: Cardiac Cirrhosis

We update eBooks quarterly and Apps daily based on user feedback. Please tap flag to report any questions that need improvement.

Question 169: A 50-year-old male is admitted to the intensive care unit with acute respiratory distress syndrome. The patient is intubated and on mechanical ventilation. Chest xray shows bilateral infiltrates with subcutaneous emphysema, pneumomediastinum, and pneumopericardium. Vital signs are within normal limits with no desaturation. What is the clinical intervention necessary at present?

Choices: 1. Pericardiocentesis 2. Needle thoracostomy 3. Observe and monitor with serial chest x-rays 4. Extubate patient

Answer: 3 - Observe and monitor with serial chest x-rays Explanations: The patient has subcutaneous emphysema, pneumomediastinum, and pneumopericardium. The air leak syndrome is not causing any hemodynamic instability. The best therapeutic option at present is clinical observation and monitoring with chest x-rays. Pneumomediastinum radiologically can be seen as a column of air in the mediastinum. Pneumopericardium is seen as a collection of air around the pericardial sac on routine chest X-ray Needle thoracostomy is the treatment for tension pneumothorax. Pericardiocentesis is the evacuation of fluid from the pericardial space. This is not indicated for pneumopericardium, which just requires monitoring. Go to the next page if you knew the correct answer, or click the link image(s) below to further research the concepts in this question (if desired).

Research Concepts: Air Leak

We update eBooks quarterly and Apps daily based on user feedback. Please tap flag to report any questions that need improvement.

Question 170: A 50-year-old male with a history of hypertension, diabetes mellitus, and chronic tobacco use presents to the emergency department with shortness of breath. The patient's symptoms have been progressively worsening over the past week. Associated symptoms include fevers, chills, cough, and fatigue. His wife was having similar symptoms, but hers have resolved. Vital signs show a blood pressure of 113/68 mmHg, respiratory rate of 25/min, heart rate 110/min, temperature 39 C, and pulse oximetry 88% on a non-rebreather mask. On physical exam, the patient is tachypneic with increased work of breathing. Lung auscultation reveals diffuse crackles on bilateral lung fields. The rest of the physical exam was unremarkable. Chest X-ray shows extensive bilateral infiltrates. The patient is intubated and placed on mechanical ventilation, with tidal volume 500 mL, respiratory rate 20/min, FiO2 100%, and PEEP 5. Arterial blood gas on these settings shows pH 7.20, pO2 100 mmHg, pCO2 60 mmHg. The next morning patient is noted to have worsening hypoxemia, and arterial blood gas shows pO2 50 on FiO2 100% and PEEP of 10. The patient is placed on a venovenous extracorporeal membrane oxygenator (VV-ECMO) at a flow of 3.5 L/min, a sweep of 3, and a repeat arterial blood gas is drawn, which shows a pH 7.35, pO2 120 mmHg, pCO2 42 mmHg. What is the next best step in the management of this patient?

Choices: 1. Continue current management, no changes necessary 2. Decrease the tidal volume 3. Increase the PEEP to 15 4. Make arrangements for prone ventilation

Answer: 2 - Decrease the tidal volume Explanations: VV-ECMO is not therapeutic on its own, but it is temporary support to help with oxygenation as the patient has severe ARDS and is unable to oxygenate or ventilate properly. While the patient is on this support, the ventilator should be placed on minimum settings to decrease the amount of insult to the lungs due to the ventilator. The patient should be on low tidal volume ventilation, between 4 and 6 ml/kg of ideal body weight. This will help with the healing process of the lungs by avoiding further insults from the ventilator. The partial pressure of carbon dioxide (pCO2) can be managed directly with the VV-ECMO by changing the sweep gas as needed to increase the removal of CO2. This means that we do not need to increase the tidal volumes on the ventilator, which could lead to volutrauma and worsening of ARDS. While on VV-ECMO, we still follow guidelines from the ARDSnet trial regarding oxygenation. The aim is to target a pO2 of 55mmHg or pulse oximetry of 88%. As long as the patient is above these parameters, there is no need to increase the fraction of inspired oxygen, which leads to the formation of harmful free radicals, and can avoid increasing PEEP, which could lead to barotrauma. VV-ECMO is the last resort for patients with worsening respiratory failure that is refractory to medical treatment. Before placement on VV-ECMO patient should have been having worsening respiratory parameters while on optimal management, including low lung volume ventilation strategy, neuromuscular blockade, and while prone. If this fails, then the patient is a candidate for VV-ECMO. There is no advantage to proning the patient while he is on VV-ECMO. Go to the next page if you knew the correct answer, or click the link image(s) below to further research the concepts in this question (if desired).

Research Concepts: Extracorporeal Membrane Oxygenation In Adults

We update eBooks quarterly and Apps daily based on user feedback. Please tap flag to report any questions that need improvement.

Question 171: A 16-year-old female sustains a circumferential burn of the left forearm in a campfire accident, with additional burned areas along the left side of her body. There was no significant impact. While assessing her total body surface area affected and evaluating her need for formal fluid resuscitation, she states that her left hand is developing “excruciating, deep, achy” pain. The initial inspection is normal. The radial or ulnar pulses on the left wrist are not palpable. Upon touching the patient’s hand, she screams in agony, and the posterior forearm is taut to palpation. What is the next best step in the management of this patient?

Choices: 1. Proceed with fluid management before addressing the extremity 2. Escharotomy is indicated before fluid resuscitation begins 3. Warm and cool compresses should be initiated after fluids are started 4. The forearm needs to be splinted after fluids are started

Answer: 2 - Escharotomy is indicated before fluid resuscitation begins Explanations: Compartment syndrome is a limb-threatening complication of circumferential burns. In situations of compartment syndrome, all restrictive dressings such as splints or casts should be removed immediately. Although occult fracture can cause compartment syndrome, the patient’s injury was not traumatic, and inflammation alone will not cause fracture. Escharotomy and fasciotomy should be performed before starting fluid resuscitation because the resulting edema will worsen the compartment syndrome. Go to the next page if you knew the correct answer, or click the link image(s) below to further research the concepts in this question (if desired).

Research Concepts: Acute Compartment Syndrome

We update eBooks quarterly and Apps daily based on user feedback. Please tap flag to report any questions that need improvement.

Question 172: A 55-year-old male was admitted 2 weeks ago for medical management of pancreatic ascites after presenting with new onset abdominal distention in the setting of a personal history of chronic pancreatitis and alcohol abuse. The patient has shown very little improvement and continues to require repeated therapeutic paracentesis. At this time, it is decided to proceed with endoscopic retrograde cholangiopancreatography (ERCP) and cholangiography to better characterize the presumed pancreatic duct leak. During cholangiography the pancreatic duct displays opacification throughout the head and neck of the pancreas upon injection of contrast, however, in the tail of the pancreas, there is significant extravasation of the contrast and no further visualization of the pancreatic duct beyond that point. What is the next best step in management at this time?

Choices: 1. Attempt stent placement to bridge the ruptured pancreatic duct 2. Continue with medical management as the pancreatic duct rupture is distal enough that the smaller caliber duct has high chances of successful healing without intervention 3. The patient needs surgical intervention. Consult surgery for further management 4. Endoscopically inject N-butyl-2-cyanoacrylate (endoscopic sealant) to obstruct the fistula communicating with the peritoneum

Answer: 3 - The patient needs surgical intervention. Consult surgery for further management Explanations: Management of pancreatic ascites usually begins with medical management for up to two weeks if the patient is tolerating it well. Longer than two weeks, it is thought that the patient begins to be at higher risk for mortality and morbidity, particularly from infectious causes. Once a patient has undergone up to 2 weeks of conservative management without improvement, ERCP is recommended to further characterize the pancreatic duct leak and in most places to place a transpapillary stent to bridge the duct disruption or to drain any communicating pseudocysts. Disconnected pancreatic duct syndrome is where there is complete pancreatic duct disruption with no further reconstitution of ductal structures distal to the point of contrast extravasation on cholangiography. In this case, surgical intervention is required for anastomosis of the distal pancreatic duct to the jejunum to allow for drainage of pancreatic juices. Endoscopically bridging pancreatic duct lesions which are completely disrupted via stenting is an area where there is little available data and therefore is not currently recommended. Use of endoscopic sealants is also an area of ongoing research and is not broadly recommended, though has been used with some success to avoid surgery by some European groups. Use of these sealants would not be recommended in a completely disrupted duct, however. Go to the next page if you knew the correct answer, or click the link image(s) below to further research the concepts in this question (if desired).

Research Concepts: Pancreatic Ascites

We update eBooks quarterly and Apps daily based on user feedback. Please tap flag to report any questions that need improvement.

Question 173: A 50-year-old man presents for evaluation of confusion for 1 day. He has a past medical history of hypertension and seizure disorder. Vital signs are heart rate 75 bpm, blood pressure 130/85 mmHg, respiratory rate 8/minute, and pulse oximetry 92% on room air. The patient is somnolent but arousable. He is disoriented. Neurologic exam shows nystagmus. Lab workup and CT head without IV contrast are within normal limits. He is observed for 24 hours with gradual improvement in mental status. He later admits to taking his entire monthly dose of levetiracetam in a suicide attempt. What is the appropriate treatment for acute levetiracetam toxicity?

Choices: 1. Benzodiazepines 2. Supportive care 3. Dialysis 4. L-carnitine

Answer: 2 - Supportive care Explanations: The therapeutic serum concentration of levetiracetam is 12 to 46 mg/L. The toxic concentration is not well defined. Levetiracetam toxicity commonly presents with central nervous system depression, nystagmus, respiratory depression. Supportive care including fluid resuscitation, supplemental oxygen, and intubation is the primary method of treatment. Levetiracetam is not metabolized by the CYP P450 system. It has little to no interaction with other anticonvulsants. Thorough laboratory and imaging workup are necessary in the setting of levetiracetam toxicity as the presentation can mimic other neurologic and metabolic conditions. Go to the next page if you knew the correct answer, or click the link image(s) below to further research the concepts in this question (if desired).

Research Concepts: Anticonvulsants Toxicity

We update eBooks quarterly and Apps daily based on user feedback. Please tap flag to report any questions that need improvement.

Question 174: A 16-year-old male presents to the emergency department 12-hours after having his right forearm run over by a car. His blood pressure is 135/90 mmHg, and heart rate is 110 beats/minute. Radiographs of the right forearm show no fractures. His forearm is very swollen but soft and compressible. A pressure monitor is used, which shows intra-compartmental pressure of 35 mmHg. The patient does not have pain with passive extension of his fingers or wrist. Which of the following is the next best step in the management of this patient?

Choices: 1. Immediate transfer to the operating room for the release of compartments 2. Ice and elevation with serial clinical compartment checks in the hospital 3. Soft compression bandage application, discharge home with instructions to ice and elevate once the patient is comfortable 4. Repeat pressure measurement in 1 hour

Answer: 2 - Ice and elevation with serial clinical compartment checks in the hospital Explanations: This patient's clinical exam is not concerning for compartment syndrome. His absolute pressure is greater than 30 mmHg, but delta pressure is not less than 30 mmHg. The delta pressure is the difference between diastolic pressure and compartment pressure. A delta pressure less than 30 mmHg is more indicative of compartment syndrome than an absolute pressure greater than 30 mmHg. The patient's exam is not consistent with compartment syndrome. However, it can evolve over several hours so the patient should be admitted and monitored with serial clinical exams. Ice and elevation of the forearm will provide symptomatic relief. An invasive pressure needle to measure compartment pressures should only be undertaken when clinical exams are not reliable. Go to the next page if you knew the correct answer, or click the link image(s) below to further research the concepts in this question (if desired).

Research Concepts: Acute Compartment Syndrome

We update eBooks quarterly and Apps daily based on user feedback. Please tap flag to report any questions that need improvement.

Question 175: A 65-year-old woman comes to the emergency department because of abdominal pain and mild shortness of breath. Her past medical history is significant for chronic liver disease and two episodes of variceal bleeding. On physical examination, her vital signs are normal, and ascites is noticed. A paracentesis is performed, and a total of 4.5 liters is removed. The ascitic fluid shows 299 polymorphonuclear neutrophils (PMN) cells/ml. Which of the following would be the most appropriate option in her management?

Choices: 1. No indication of intravenous albumin 2. Albumin 5% infusion 3. Albumin 25% infusion 4. Crystalloids

Answer: 3 - Albumin 25% infusion Explanations: Administration of albumin 1.5 g/kg within 6 hours and 1 g/kg on day three along with antibiotics have a better effect in preventing renal impairment and reducing mortality in patients with spontaneous bacterial peritonitis (SBP). The use of albumin 25% solution to treat spontaneous bacterial peritonitis is not an FDA approved medication, but multiple clinical trials have shown benefits when used in conjunction with antibiotics. SBP is a common cause of mortality in patients with cirrhosis. The use of intravenous albumin has reduced that mortality. The diagnosis of SBP is established by a positive ascitic fluid bacterial culture and an elevated ascitic fluid absolute PMN count of 250 cells/ml. Go to the next page if you knew the correct answer, or click the link image(s) below to further research the concepts in this question (if desired).

Research Concepts: Albumin Colloid

We update eBooks quarterly and Apps daily based on user feedback. Please tap flag to report any questions that need improvement.

Question 176: A 17-year-old female with diabetes mellitus presents with pain in her left calf after a fall from her bicycle. She is febrile, tachycardic, looks ill, and her calf is tender to touch. The skin overlying the left leg has many bullae with black necrosis. There is a serosanguinous discharge. The distal pulses are decreased and there is crepitus. What is the next step in management?

Choices: 1. Broad-spectrum antibiotics and emergent operative debridement 2. Get a stat angiogram 3. Obtain x-rays to look for subcutaneous gas 4. Send patient for duplex ultrasound to rule out deep venous thrombosis

Answer: 1 - Broad-spectrum antibiotics and emergent operative debridement Explanations: Gas gangrene is a disorder where there is necrotizing myositis. The hallmarks of the disorder are rapid onset muscle swelling, severe pain, sepsis, and gas in tissues. This is a life-threatening surgical emergency. Clostridium species are gram-positive, spore-forming, and are a common cause of gas gangrene. The findings will reveal bulla, crepitus, necrotic skin, and serosanguinous discharge. Pulses may be present or absent. There may be no pain because the nerves are destroyed. The combination of aggressive surgical debridement and effective antibiotic therapy is the determining factor for successful treatment. Gas may appear on x-rays, but the patient must be resuscitated and taken to the operating room. Fasciotomy plus radical debridement may be required. Go to the next page if you knew the correct answer, or click the link image(s) below to further research the concepts in this question (if desired).

Research Concepts: Gas Gangrene

We update eBooks quarterly and Apps daily based on user feedback. Please tap flag to report any questions that need improvement.

Question 177: A previously healthy 55-year-old male with non-insulin dependent diabetes mellitus presents with neck swelling, 10 out of 10 pain, and fever. He states that he has a small lesion on the side of his neck, which has progressively increased in size. On examination, he has a temperature of 102.2F and a white blood cell count of 22,000 cells/mm3. The pain is not easily controlled with multiple doses of pain medication. In addition to antibiotics, the management of this patient includes which of the following?

Choices: 1. Admission to the intensive care unit 2. Surgical exploration and debridement 3. Moist compresses 4. Observation on antibiotics

Answer: 2 - Surgical exploration and debridement Explanations: In patients with diabetes mellitus, infections can progress rapidly and become fulminant. Necrotizing fasciitis is more common in patients with diabetes mellitus, patients who are obese, and patients who are immunocompromised. Pain out of proportion to physical findings or pain that is difficult to control should warrant further investigation and raise the possibility of a necrotizing infection. The surgery may need to be done in the operating room under general anesthesia, because the infection may be more extensive than is initially apparent. Go to the next page if you knew the correct answer, or click the link image(s) below to further research the concepts in this question (if desired).

Research Concepts: Necrotizing Fasciitis

We update eBooks quarterly and Apps daily based on user feedback. Please tap flag to report any questions that need improvement.

Question 178: A 65-year-old female with a past medical history of hypertension and a large ischemic stroke 5 years ago with resultant left-sided hemiparesis presents to the emergency department with acute onset of pleuritic chest pain with shortness of breath. She is tachypneic, diaphoretic, and in visible distress. Her vital signs show a heart rate of 125 beats/min, respiratory rate of 25/min, blood pressure of 100/60, and pulse oximetry of 85% on room air. CT angiography of the chest shows a filling defect in the proximal pulmonary vasculature bilaterally. What is the best treatment option for this patient?

Choices: 1. Argatroban infusion 2. Catheter-directed thrombolysis 3. Inferior vena cava filter placement 4. Heparin infusion

Answer: 4 - Heparin infusion Explanations: This patient presents with a massive to submassive pulmonary embolism (PE). Catheterdirected thrombolysis (CDT) is indicated for massive PE or for submassive PE with hemodynamic compromise. However, due to the risk of complications, contraindications for the use of CDT must be reviewed prior to consideration of this therapy. Though thrombolysis is administered locally during CDT, the thrombolytic agent can disperse systemically. Contraindications to CDT include a prior ischemic stroke, cerebral bleed, cerebral mass, vascular deformation, a recent bleeding ulcer in the gastrointestinal tract, recent brain/spine surgery, major abdominal or pelvic surgery, or any source of active hemorrhage. The most common and most feared complication of CDT is a hemorrhagic stroke. This patient's history of prior stroke puts her at a higher risk of intracranial bleeding following thrombolytic agent use. This is a relative contraindication to CDT for her. She should be treated with intravenous anticoagulation with close monitoring of her neurologic status. Inferior vena cava placement is indicated in patients who have contraindications to anticoagulation. Given her remote stroke and current hemodynamically devastating pulmonary embolism, a trial of anticoagulation is warranted. Argatroban is indicated in cases of heparin-induced thrombocytopenia. Go to the next page if you knew the correct answer, or click the link image(s) below to further research the concepts in this question (if desired).

Research Concepts: Catheter Directed Thrombolysis Of Pulmonary Embolism

We update eBooks quarterly and Apps daily based on user feedback. Please tap flag to report any questions that need improvement.

Question 179: A 58-year-old man is seen in the emergency department after suffering a smoke inhalation injury following a house fire. He was intubated for dyspnea with stridor and currently is on a ventilator with 100% FiO2. He has copious secretions that are being treated with aggressive pulmonary hygiene. His carboxyhemoglobin level is 10% and arterial blood gas shows pH 7.3. What is the best next step in management?

Choices: 1. Nebulized albuterol 2. Intravenous methylprednisolone 3. Transfer to a facility with hyperbaric oxygen therapy 4. Empiric broad-spectrum antibiotics

Answer: 1 - Nebulized albuterol Explanations: Nebulized beta-2-adrenergic agonists and muscarinic receptor antagonists have been proven in animal models to improve respiratory function following inhalation injury. Corticosteroids have not been proven to be beneficial in inhalation injury patients. Hyperbaric oxygen therapy (HBOT) is indicated for carboxyhemoglobin levels greater than 25%, unconscious patients, those with neurologic findings, or in cases of severe metabolic acidosis, pH less than 7.1. Treatment often requires transfer to a facility with HBOT capabilities, but treatment should not be delayed. Antibiotics should be started in patients with inhalation injury who have complications of pneumonia. Pneumonia is a common short-term complication of severe inhalation injury and usually presents 4 to 5 days after the initial injury. Go to the next page if you knew the correct answer, or click the link image(s) below to further research the concepts in this question (if desired).

Research Concepts: Inhalation Injury

We update eBooks quarterly and Apps daily based on user feedback. Please tap flag to report any questions that need improvement.

Question 180: A 50-year-old male with no past medical history presents to the emergency department with signs and symptoms of ischemic stroke. Symptoms have been present for the past 3 hours, and he is visibly upset because he states that he works out almost daily and eats a healthy diet. He also states that he goes to his primary care physician every year, and has always had a "normal" physical. His family history is non-contributory. He takes no medication except for a daily multi-vitamin and turmeric powder. Vital signs and physical exam are within normal limits. CBC, comprehensive metabolic profile, urine toxicology screen, and blood alcohol content all came back within normal limits. EKG showed normal sinus rhythm. CT scan of the brain did not show any evidence of bleeding. What is the next best step in evaluating for cryptogenic stroke?

Choices: 1. Transthoracic echocardiogram (TTE) 2. TTE followed by transesophageal echocardiogram with agitated saline contrast 3. Transesophageal echocardiogram 4. Pelvic magnetic resonance venography

Answer: 2 - TTE followed by transesophageal echocardiogram with agitated saline contrast Explanations: Patients less than 55 years old without risk factors for cardiac embolism or thromboembolism and who have had a negative workup for the standard evaluation for stroke should undergo a more advanced workup. When a cryptogenic stroke is suspected, the workup should include a transesophageal echocardiogram with injected agitated saline, prolonged cardiac monitoring, coagulopathy, and vasculitis workup. Transesophageal echocardiogram with agitated saline contrast should be injected at rest, with Valsalva maneuver, and with cough. This is because the Valsalva maneuver can transiently increase the intracardiac pressure turning a left to right shunt into a right to left shunt. Up to 45% of strokes in young patients are said to be cryptogenic. Patent foramen ovale (PFO) can be found in 30% of the population. Although PFO is a diagnosis of exclusion, it should be strongly considered in the setting of a cryptogenic stroke. Go to the next page if you knew the correct answer, or click the link image(s) below to further research the concepts in this question (if desired).

Research Concepts: Paradoxical Embolism

We update eBooks quarterly and Apps daily based on user feedback. Please tap flag to report any questions that need improvement.

Question 181: A 33-year-old farmer was referred to the tertiary hospital due to fever, shortness of breath, and chest pain for two weeks. He had no history of chronic disease, and before the present illness, he had been perfectly healthy. He also did not have any history of medication or travel for the past 3 months. During the first hospitalization, he was diagnosed having pneumonia of the left upper lobe and treated with antibiotics. Ten days after the treatment, the pneumonia progressed. The differential white blood count revealed 46% of eosinophils, and the serum IgE was elevated. The sputum culture was negative for any bacteria, the direct smear of the sputum was negative for fungi or acid bacilli, and blood culture was negative for any culturable bacteria. Stool examination was negative for ova or any parasites. Surprisingly, during hospitalization, the chest X-ray revealed migratory infiltrates of the lungs. What could be the possible condition in this patient?

Choices: 1. Interstitial lung disease due to bacterial infection 2. Acute respiratory distress syndrome due to viral infection 3. Flu-like symptoms due to viral infection 4. Loeffler syndrome due to helminths infection

Answer: 4 - Loeffler syndrome due to helminths infection Explanations: The patient is a farmer who has a higher risk to contract with soil-transmitted helminths. Treatment with antibiotics does not improve the patient condition, thus organism other than bacteria should be suspected. Eosinophilia and elevated serum IgE along with pulmonary symptoms define the Loeffler syndrome. The patient had no history of consuming drugs prior to hospitalization. Based on history taking and laboratory examination, Loeffler syndrome due to helminths infection is the possible cause. Go to the next page if you knew the correct answer, or click the link image(s) below to further research the concepts in this question (if desired).

Research Concepts: Ancylostoma

We update eBooks quarterly and Apps daily based on user feedback. Please tap flag to report any questions that need improvement.

Question 182: A 17-year-old female presents confused and uncooperative. Her vital signs are normal except for a respiratory rate of 32/minute. Blood work reveals a sodium of 137 mEq/L, potassium 2.5 mEq/L, bicarbonate 13 mEq/L, chloride 119 mEq/L, BUN 16 mg/dL, creatinine 0.6 mg/dL, glucose 80 mg/dL, calcium 9.2 mg/dL, urine pH 7.7, urine sodium 38 mEq/L, urine potassium 45 mEq/L, and urine chloride 20 mEq/L. Her arterial blood gas shows a pH of 7.27, PCO2 28 mmHg, and PO2 94 mmHg on room air. What is the most likely diagnosis?

Choices: 1. Proximal renal tubular acidosis 2. Distal renal tubular acidosis 3. Lactic acidosis 4. Malabsorption with diarrhea

Answer: 2 - Distal renal tubular acidosis Explanations: The patient has hyperchloremic, non-anion gap metabolic acidosis with appropriate respiratory compensation. This can be due to gastrointestinal bicarbonate losses such as from diarrhea or renal causes, including proximal renal tubular acidosis (RTA) type II, distal RTA (type I), or hypoaldosteronism RTA (type IV). Urine anion gap in diarrhea may be positive. A positive urine anion gap (urine sodium plus urine potassium minus urine chloride) points towards a renal etiology. The fact that the patient has an elevated urine pH and hypokalemia in the setting of acidosis points to distal RTA because the normal response would be a lowering of urine pH in the setting of metabolic acidosis. Distal RTA presents with rickets, growth failure, and osteomalacia due to metabolic acidosis. Hypercalciuria, hypocitraturia, and alkaline urine lead to nephrocalcinosis, specifically calcium phosphate stones and recurrent urinary tract infections. End-stage renal failure due to nephrocalcinosis may result. Incomplete distal RTA should be considered in any patient with calcium stones and a urine pH persistently 5.5 or higher in the absence of infection. Go to the next page if you knew the correct answer, or click the link image(s) below to further research the concepts in this question (if desired).

Research Concepts: Renal Tubular Acidosis

We update eBooks quarterly and Apps daily based on user feedback. Please tap flag to report any questions that need improvement.

Question 183: A 24-year-old primigravida presents to the emergency department with fatigue, nausea, vomiting, and abdominal pain. She is ten weeks pregnant, and so far the pregnancy has been uneventful. The physical exam is significant for pallor, tachycardia, and mild tenderness in the epigastrium. Other vital parameters show a blood pressure of 98/56 mmHg and a temperature of 97.5 F. Labs show hemoglobin of 11.1 g/dL, leukocyte count of 12100/microL, platelet count of 74000/microL, creatinine of 3.2 mg/dL, and blood urea nitrogen (BUN) of 45 mg/dL. Review of the peripheral smear shows schistocytes, giant platelets, thrombocytopenia, Burr cells, and anisocytosis. No atypical white cells are seen on peripheral smear. Following this, more tests are ordered, and results are significant for lactate dehydrogenase of 1563 U/L, haptoglobin is undetectable, and absolute reticulocyte index is 3.2. What is the next best step in the management of this patient?

Choices: 1. Send direct Coombs test and start high dose steroids 2. Send ADAMTS-13 and wait for results 3. Send ADAMTS-13 and start plasma exchange without waiting for results 4. Send testing for genetic mutation for atypical HUS and start eculizumab without waiting for results

Answer: 3 - Send ADAMTS-13 and start plasma exchange without waiting for results Explanations: The patient presented with microangiopathic hemolytic anemia (hemolysis with schistocytes on peripheral smear along with thrombocytopenia and renal failure). Thrombotic microangiopathy is rare in pregnancy, but it is associated with high maternal and neonatal morbidity. Once the diagnosis of microangiopathic hemolytic anemia is established, plasma exchange should be initiated without any further delay. Confirmatory tests, especially ADAMTS-13, should be sent before start plasma exchange, but there is no need to wait for results. ADAMTS-13, mutations for atypical HUS, Shiga toxin should be sent in a patient with microangiopathic hemolytic anemia for evaluation of thrombotic thrombocytopenic purpura, atypical HUS, and typical HUS respectively. Eculizumab is started only if a patient with microangiopathic hemolytic anemia does not respond to plasma exchange. Direct Coombs test is used to determine the cause of antibodymediated hemolysis. This patient presentation is quite characteristic for microangiopathic hemolytic anemia. Go to the next page if you knew the correct answer, or click the link image(s) below to further research the concepts in this question (if desired).

Research Concepts: Thrombocytopenia in Pregnancy

We update eBooks quarterly and Apps daily based on user feedback. Please tap flag to report any questions that need improvement.

Question 184: A 75-year-old patient with a history of atrial fibrillation, on apixaban for anticoagulation, is brought to the ER following a syncopal fall. She has been compliant with apixaban, and her last dose was this morning. She reports bright red blood per rectum since yesterday. Heart rate is 114 bpm, and her blood pressure is 86/54 mmHg. Lab work shows a hemoglobin of 5.2 g/dL. Which of the following is the best next step in managing this patient?

Choices: 1. Administer normal saline 2. Administer fresh frozen plasma (FFP) 3. Transfuse packed red blood cells (PRBCs) and vitamin K 4. Transfuse PRBC and prothrombin complex concentrate (PCC)

Answer: 4 - Transfuse PRBC and prothrombin complex concentrate (PCC) Explanations: Patients taking newer anticoagulants and presenting with a life-threatening bleeding emergency can be treated with 4 factor PCC. Vitamin K is not indicated in this patient taking apixaban. It is indicated for the reversal of warfarin-induced anticoagulation. Normal saline alone will be inadequate in this patient with life-threatening gastrointestinal bleed with a hemoglobin of 5.2 g/dL. FFP can be used to reverse anticoagulation and also has the benefit of providing volume resuscitation. But FFP needs to be procured from the blood bank, thawed, and takes several hours to administer. PCC can be administered in minutes with no need to thaw, making it a better agent in this scenario. Go to the next page if you knew the correct answer, or click the link image(s) below to further research the concepts in this question (if desired).

Research Concepts: Prothrombin Complex Concentrate

We update eBooks quarterly and Apps daily based on user feedback. Please tap flag to report any questions that need improvement.

Question 185: A 73-year-old white man undergoes emergency surgery for a perforated duodenal ulcer. The surgery went smoothly without any event. The patient comes out of anesthesia and is awake and alert. He receives post-operative pain management. Approximately 8 hours later, the patient develops a complication and requires immediate intubation with concurrent administration of naloxone. Which of the following presentation is the most specific for the complication?

Choices: 1. Pupillary miosis 2. Hypotension 3. Conjunctival injection 4. Respiratory depression

Answer: 4 - Respiratory depression Explanations: Opioid toxicity typically presents with a depressed level of consciousness, respiratory depression, and pupillary miosis. It is important to be aware that opioid exposure does not always result in miosis and that respiratory depression is the most specific sign. Drowsiness, euphoria, and conjunctival injection are seen frequently. Other vital presenting signs are ventricular arrhythmias, acute mental status changes, and seizures. Naloxone is indicated for the treatment of opioid toxicity, specifically to reverse respiratory depression from opioid use. Naloxone is a competitive opioid antagonist with a high affinity for the mu-opioid receptor, allowing for the reversal of the effects of opioids. Reliance on pupillary miosis to diagnose opiate overdose can be misleading. If sufficiently severe, the central hypoxia may present with hypertension and pupillary dilation. Go to the next page if you knew the correct answer, or click the link image(s) below to further research the concepts in this question (if desired).

Research Concepts: Opioid Overdose

We update eBooks quarterly and Apps daily based on user feedback. Please tap flag to report any questions that need improvement.

Question 186: A 25-year-old schizophrenic and asthmatic patient is brought to the emergency room with severe dyspnea. On physical examination, he was found to be wheezing. He was nebulized with IV magnesium and IV steroids and was put on bi-level positive pressure ventilation. He did not improve after 1 hour on BiPAP and was intubated. His psychiatric medications were held, and the Propofol infusion is initiated. After 3 days, his wheezing is better, and sedation cessation was attempted. After 30 minutes he became extremely agitated and was sedated again. Which of the following is the best plan for the next attempt at extubation?

Choices: 1. Extubate the patient on propofol 2. Extubate the patient on fentanyl 3. Restart the psychiatric medications and add dexmedetomidine for sedation 4. Extubate the patient on ketamine

Answer: 3 - Restart the psychiatric medications and add dexmedetomidine for sedation Explanations: The reason for sedation vacation or breathing trial failing should be identified and addressed accordingly. If the patient is getting agitated and is delirious, antipsychotics can be tried; if anxious, anxiolytics can be tried; or if in pain, low dose fentanyl drip/morphine drip or patch can be tried. Patient delirium and agitation can be very well controlled on dexmedetomidine, and as the medication does not suppress the respiratory center, the patient can be extubated while on continuous infusion of dexmedetomidine. Attention should be paid to home medications, and if clinically permitted, a patient who is on any anxiolytics, antipsychotics, or any chronic pain medications should continue them while sedated or restart them in a timely fashion to prevent withdrawal. Go to the next page if you knew the correct answer, or click the link image(s) below to further research the concepts in this question (if desired).

Research Concepts: Sedation Vacation in the ICU

We update eBooks quarterly and Apps daily based on user feedback. Please tap flag to report any questions that need improvement.

Question 187: A 68-year-old female with a history of decompensated cirrhosis and ascites presents to the hospital with a decreased urine output for the past three days. She currently takes furosemide and spironolactone. Labs are significant for serum creatinine of 3.5 mg/dL and BUN 50 mg/dL. Her blood pressure was noted to be 96/65 mmHg. Her diuretics were held, and she was started on IV fluids. 2 days later, her serum creatinine did not improve, and serum potassium was noted to be 4.2 mEq/L. A renal ultrasound did not show any evidence of hydronephrosis. Which of the following is the next best step in the management of this patient?

Choices: 1. Furosemide 2. Hemodialysis 3. Observation 4. Midodrine

Answer: 4 - Midodrine Explanations: Patients with cirrhosis and ascites are prone to develop hepatorenal syndrome secondary to systemic hypotension and renal vasoconstriction, causing the underfilling phenomenon. Splanchnic vasodilation in cirrhosis leads to decreased effective blood flow to the kidneys, which activates the RAAS system, leading to retention of sodium and water and renal vascular constriction. However, this effect is not enough to overcome the systemic vasodilation caused by cirrhosis, leading to renal hypoperfusion and worsened by renal vasoconstriction with the end point of renal failure. Acute renal injury in patients with cirrhosis is usually secondary to dehydration and it is managed by stopping the diuretics and starting the patients on IV fluids. If the kidney function does not improve after adequate hydration then the diagnosis of hepatorenal syndrome is made. Patients with hepatorenal syndrome are usually managed with a systemic vasoconstrictor given with albumin. Midodrine has shown a good response and is first-line for the treatment of hepatorenal syndrome. Diuretics should be held in cirrhotic patients who develop acute kidney injury. Hemodialysis is indicated in patients who develop refractory hypervolemia or hyperkalemia. Observation is inappropriate as the patient’s kidney function did not improve with IV fluids. Go to the next page if you knew the correct answer, or click the link image(s) below to further research the concepts in this question (if desired).

Research Concepts: Hepatic Cirrhosis

We update eBooks quarterly and Apps daily based on user feedback. Please tap flag to report any questions that need improvement.

Question 188: A 65-year-old female presents to the emergency department with a chief complaint of abdominal pain. The patient appears uncomfortable in the room and complains of severe pain that started suddenly less than an hour before arrival. On physical exam, her abdomen is noted to be soft and non-tender. Her pulse is 150 bpm and irregular, with a blood pressure of 140/95 mmHg, and an increased respiratory rate. The patient has no prior history of atrial fibrillation and is not currently anticoagulated. What is the best next test to confirm the suspected diagnosis?

Choices: 1. Abdominal ultrasound 2. KUB plain film x-ray 3. CTA of the abdomen and pelvis 4. Angiography

Answer: 3 - CTA of the abdomen and pelvis Explanations: Bowel necrosis is a life-threatening emergency that has many presentations and equally many underlying etiologies. One such etiology of bowel necrosis, acute mesenteric ischemia, frequently presents with abdominal pain out of proportion to physical examination findings. Rapid diagnosis of this disease process is vital to both mortality and disease course. Patients presenting with severe abdominal pain and a new diagnosis of atrial fibrillation are a high-risk group for acute mesenteric ischemia and bowel necrosis secondary to thromboembolism. Approximately half of all cases of acute mesenteric ischemia are thought to be secondary to an underlying embolic disease process. There are multiple imaging modalities available to test for acute bowel necrosis and mesenteric ischemia. In this clinical scenario, the next best imaging test would be a CTA of the abdomen and pelvis. CTA has a reported sensitivity between 94% to 96% and specificity between 96% to 98%. Although angiography is still considered the gold standard for diagnosis, this procedure is invasive and time intensive in a disease that is time sensitive. Patients with a diagnosis of bowel necrosis are managed according to a variety of factors including the underlying etiology of the disease process and comorbid conditions. The final pathway for management for many cases involves surgical consultation and intervention. In a high probability of patient presentation, emergent consultation on an unstable patient with a suspected diagnosis of acute mesenteric ischemia and bowel necrosis should not be delayed by imaging or laboratory results. Go to the next page if you knew the correct answer, or click the link image(s) below to further research the concepts in this question (if desired).

Research Concepts: Bowel Necrosis

We update eBooks quarterly and Apps daily based on user feedback. Please tap flag to report any questions that need improvement.

Question 189: A 55-year-old male who recently flew from India is brought to the emergency department for hemoptysis. The patient was first seen at the primary care clinic where he complained by fevers and chills as well as profuse sweating at night. He then proceeded to cough up a large volume of blood and was sent to the emergency department. The patient was tried on a course of inhaled tranexamic acid. This did not control the bleeding. Finally, the decision was made to intubate the patient. After, the patient was sent to the CT which showed a left upper lobe cavitary lesion and also a bronchial artery source of the bleeding. The patient has a blood pressure of 101/70 mmHg, a heart rate of 98 bpm, oxygen saturation of 100%, and currently being ventilated at a respiratory rate of 15 breaths per minute. What is the next step in management?

Choices: 1. Perform bronchoscopy and use cold saline lavage in order to control the source of bleeding and isolate the etiology of the hemoptysis 2. Take the patient to the operating room for surgical lobectomy to bleeding control 3. Replace the single-lumen endotracheal tube with a double lumen endotracheal tube 4. Perform bronchial artery embolization to stop bleeding

Answer: 4 - Perform bronchial artery embolization to stop bleeding Explanations: CT scan is as accurate as bronchoscopy in localizing source of bleeding, and CT is superior for finding the cause of the bleeding. The patient has risk factors that point to tuberculosis as the source of bleeding, and bronchoscopy can further exacerbate hemoptysis by stimulating coughing. Surgical lobectomy has high mortality rates in emergent cases. Although the patient may need a lobectomy in the future, a stable patient would benefit from embolization before going through scheduled lobectomy. Double-lumen endotracheal tubes are technically difficult to place and do not allow optimal source control due to the smaller diameter of the lumens. Immediate success rates of bronchial artery embolization are reported to be 82% to 98%, and would most benefit a stable patient in the immediate term. While surgical lobectomy may be required down the line due to recurrence, the patient can be further worked up and medically optimized during the interim. Go to the next page if you knew the correct answer, or click the link image(s) below to further research the concepts in this question (if desired).

Research Concepts: Pulmonary Hemorrhage

We update eBooks quarterly and Apps daily based on user feedback. Please tap flag to report any questions that need improvement.

Question 190: A 65-year-old female with a history of hypertension on hydrochlorothiazide presents with severe chest pain that radiates to her back and dyspnea. She is found to have a blood pressure of 205/98 mmHg, pulmonary rales, an ECG with ST changes, and a chest x-ray with a widened mediastinum. What the best initial management?

Choices: 1. Immediate cardiac catheterization with likely stent placement 2. Intravenous nitroprusside and urgent CT angiography with three-dimensional reconstruction 3. Intravenous morphine and nitrates, oxygen, and aspirin 4. Serial cardiac enzymes, intravenous beta-blockers, and heparin

Answer: 2 - Intravenous nitroprusside and urgent CT angiography with three-dimensional reconstruction

Explanations: This scenario is suggestive of an aortic dissection that can be diagnosed with urgent CT angiography with three-dimensional reconstruction. CT is useful in patients who are hemodynamically stable. Beta-blockers are used to decrease cardiac contractility but will aid in bringing the blood pressure down as well. Calcium-channel blockers can be used if beta-blockers are contraindicated. Nitroprusside is used to reduce blood pressure. A proximal, type A, dissection is associated with aortic insufficiency and hypertension and requires urgent repair, while a distal dissection is associated with arterial obstruction leading to renal failure and bowel ischemia. Go to the next page if you knew the correct answer, or click the link image(s) below to further research the concepts in this question (if desired).

Research Concepts: Aortic Dissection

We update eBooks quarterly and Apps daily based on user feedback. Please tap flag to report any questions that need improvement.

Question 191: A 91-year-old female becomes acutely unresponsive in her assisted living care facility. She is taken to the hospital for a stroke workup and found to have a right frontal intraparenchymal hemorrhage with intraventricular extension. The estimated blood volume of the hemorrhage is 65 mL. She takes warfarin for non-valvular atrial fibrillation and has an INR of 2.1. She has hypertension, diabetes mellitus, asthma, gastroesophageal reflux disease, and stress incontinence. On initial exam, her eyes do not open to painful stimuli, she moans and withdrawals to noxious stimuli in the right arm and has extensor posturing in the left arm with an extension of bilateral legs. What is her 30-day mortality based on the intracerebral hemorrhage score?

Choices: 1. 26% 2. 72% 3. 97% 4. 100%

Answer: 3 - 97% Explanations: The intracerebral hemorrhage (ICH) score predicts 30-day mortality based on patients with non-traumatic intracerebral hemorrhage. The score is based on Glasgow coma score (GCS), age, the volume of hemorrhage, intraventricular hemorrhage, and infratentorial origin of hemorrhage. The point distribution is GCS 13 to 15 is 0 points, GCS 5 to 12 is 1 point, GCS 3 to 4 is 2 points; age 80 or greater is 1 point, ICH volume 30 mL or more is 1 point, intraventricular hemorrhage is 1 point, and infratentorial origin of hemorrhage is 1 point. 30-day mortality is 0% for 0 points, 13% for 1 point, 26% for 2 points, 72% for 3 points, 97% for 4 points, 100% for 5 points, and assumed to be 100% for 6 points. Go to the next page if you knew the correct answer, or click the link image(s) below to further research the concepts in this question (if desired).

Research Concepts: Intracranial Hemorrhage

We update eBooks quarterly and Apps daily based on user feedback. Please tap flag to report any questions that need improvement.

Question 192: A 50-year-old male with alcohol use disorder presents to the emergency department. His medical history includes only alcohol use disorder, and he is not currently taking any medication. On presentation, he complains of nausea, vomiting, and abdominal pain. He says that he has been drinking heavily for several days but over the last day he has been unable to “keep anything down.” His vital signs show respirations 28, heart rate 126 beats/min, blood pressure 106/62 mmHg, and temperature of 98.5 F. On physical exam, he has dry mucous membranes, mild diffuse abdominal tenderness, no peritoneal signs, and clear lungs. His extremities are without edema. He has a visible resting tremor which is worsened with finger-tonose maneuvers. His bedside blood glucose is 70 mg/dL. He has the following arterial blood gas results: pH 7.2, pCO2 25, pO2 98, and bicarbonate 15 mEq/L. His basic metabolic panel shows sodium 138 mEq/L, potassium 4 mEq/L, chloride 100 mEq/L, bicarbonate 15 mEq/L, BUN 29mg /dL, and creatinine 1.6 mg/dL. Anion gap is 23. What is the patient’s acid-base status?

Choices: 1. Metabolic acidosis with adequate respiratory compensation 2. Metabolic alkalosis without respiratory compensation 3. Metabolic acidosis without adequate respiratory compensation 4. Metabolic acidosis without respiratory compensation

Answer: 3 - Metabolic acidosis without adequate respiratory compensation Explanations: The patient has metabolic acidosis without adequate respiratory compensation. The presence of large amounts of ketoacids (beta-hydroxybutyrate) lowers the pH and causes an increased respiratory rate to decrease the pCO2. By using Winter's formula one can tell if the patient is adequately compensating for the metabolic acidosis. In this case, the lungs are not adequately compensating for the metabolic acidosis. The patient uses up existing bicarbonate ions by combining them with the hydrogen ions to make carbonic acid (H2CO3). This is an example of buffering an acid. At the level of the lungs, the respiratory rate increases to release carbon dioxide in an attempt to decrease the amount of acid in the system and bring the pH back up. Go to the next page if you knew the correct answer, or click the link image(s) below to further research the concepts in this question (if desired).

Research Concepts: Alcoholic Ketoacidosis

We update eBooks quarterly and Apps daily based on user feedback. Please tap flag to report any questions that need improvement.

Question 193: Several workers from a local government office present to the emergency department shortly after being exposed to a white powder from an envelope that was mailed to their address. After undergoing thorough decontamination, they are brought into the department, and they are noted to be coughing, wheezing, and complaining of feeling short of breath. Responders on the scene giving a preliminary report that the white powder has been identified as ricin. Which of the following best describes an appropriate plan of care and likely clinical course for these patients?

Choices: 1. Their wheezing and shortness of breath will probably resolve after treatment with nebulized albuterol and oral corticosteroids, and they can probably be discharged after a few hours of observation if their symptoms are improving 2. They will probably progress to angioedema and anaphylaxis unless treated with intramuscular epinephrine quickly, but if treatment is initiated promptly and there are no signs of airway compromise, they could probably be admitted to a floor bed for observation 3. They need to be given prophylactic antibiotics and admitted to an isolation room 4. Their symptoms may quickly progress to pulmonary edema, pneumonia, respiratory failure, seizures, and organ failure. They should be admitted to the intensive care unit

Answer: 4 - Their symptoms may quickly progress to pulmonary edema, pneumonia, respiratory failure, seizures, and organ failure. They should be admitted to the intensive care unit

Explanations: While it is not incorrect to treat their symptoms, under no circumstances should these patients be discharged home because they could quickly progress to pulmonary edema and respiratory failure regardless of their initial response to treatment. Inhalational exposure to ricin does cause wheezing and difficulty breathing, but these symptoms are not because of an anaphylaxis reaction, and because these patients may rapidly deteriorate into respiratory failure, admitting them to a floor bed when they are already symptomatic is unwise. Prophylactic antibiotics are not indicated in ricin exposure, and once the patients have been thoroughly decontaminated there is no risk that healthcare workers will experience symptoms by coming into contact with these patients. Thus an isolation room is unnecessary. Patients exposed to ricin by inhalation can rapidly deteriorate and may develop all of these symptoms; because they are already symptomatic and may quickly progress to being critically ill, admitting them to the intensive care unit is the best choice. Go to the next page if you knew the correct answer, or click the link image(s) below to further research the concepts in this question (if desired).

Research Concepts: Ricin Toxicity

We update eBooks quarterly and Apps daily based on user feedback. Please tap flag to report any questions that need improvement.

Question 194: A 48-year-old male presents to the emergency department with self-report of suicidal ideations. He states that he drank a bottle of windshield washer fluid. He is asymptomatic with normal vital signs. His basic metabolic panel is unremarkable, and his ethanol concentration is negative. Methanol concentration is drawn and your hospital's laboratory has to send it to another laboratory for gas chromatography. The turn around time is 24 hours. What is the most appropriate next step?

Choices: 1. Check a serum osmolality 2. Calculate the osmolality gap 3. Check serial basic metabolic panels every 3 hours for 12 hours monitoring for acidosis while the patient remains hydrated and nourished 4. Give fomepizole

Answer: 3 - Check serial basic metabolic panels every 3 hours for 12 hours monitoring for acidosis while the patient remains hydrated and nourished

Explanations: The patient is asymptomatic with normal acid-base status. Fomepizole should be given when there is evidence of a toxic concentration of a toxic alcohol. Methanol is a toxic alcohol that is metabolized by alcohol dehydrogenase to formaldehyde, and then by aldehyde dehydrogenase to formic acid. The parent compound, methanol causes the serum osmol gap, while formic acid causes the anion gap acidosis and retinal toxicity. As the methanol is metabolized, the osmolal gap decreases while the anion gap increases. A serum osmolality gap can be helpful in this situation. If it is elevated, it may support the decision to give fomepizole. However, it cannot be used to rule out the ingestion of a toxic alcohol. This patient will require fomepizole to cease the progression of acidosis if it were to develop. Fomepizole should be given if bicarbonate falls below 15 and then resuscitation should ensue. To help avoid any form of iatrogenic acidosis, the patient should remain hydrated and nourished during the observation period. Go to the next page if you knew the correct answer, or click the link image(s) below to further research the concepts in this question (if desired).

Research Concepts: Methanol Toxicity

We update eBooks quarterly and Apps daily based on user feedback. Please tap flag to report any questions that need improvement.

Question 195: A 17-year-old male presents with hypothermia, malaise, anorexia, a semicomatose state, and hypotension. A chest x-ray shows findings consistent with pneumonia. The blood culture shows gram-negative rods. Which is the presumptive diagnosis and which cytokines may be involved?

Choices: 1. B-lymphocyte cell growth: Interleukin (IL)-2 and interferon gamma are involved 2. Septic shock: IL-1 and tumor necrosis factor are involved 3. Systemic autoimmune disease: IL-4 and IL-5 are involved 4. AIDS: IL-3 and IL-7 are involved

Answer: 2 - Septic shock: IL-1 and tumor necrosis factor are involved Explanations: Septic shock generated by gram-negative bacteria is mediated by cytokines, mainly tumor necrosis factor (TNF) along with other cytokines including interleukin (IL)-1 and IL-6. The similarities between IL-1 actions and those of TNF appear surprising at face value because the cytokines and their receptors are structurally different. Toxins, including lipopolysaccharide that is present in the outer membrane of the gramnegative bacterium, are involved in the pathogenesis of the TNF-mediated septic shock. IL-1 does not induce the apoptosis. Even at high systemic concentrations, IL-1 by itself it does not cause the pathophysiologic changes of septic shock. Go to the next page if you knew the correct answer, or click the link image(s) below to further research the concepts in this question (if desired).

Research Concepts: Interleukin

We update eBooks quarterly and Apps daily based on user feedback. Please tap flag to report any questions that need improvement.

Question 196: A 32-year-old male with a history of acute myeloid leukemia (AML) is admitted after a recurrence of AML following initial treatment with chemotherapy. During his admission, he develops perianal pain and has a complaint of severe pain with bowel movements. Digital rectal examination is performed and yields only tenderness to palpation at the anal verge without identifying fullness or erythema. What is the best diagnostic test to further assess his pain?

Choices: 1. Computed tomography (CT) of the pelvis 2. Endorectal ultrasound 3. No further imaging is needed, continue treatment for AML 4. Magnetic resonance imaging (MRI) of the pelvis

Answer: 4 - Magnetic resonance imaging (MRI) of the pelvis Explanations: CT is not the best imaging technique available in an immunocompromised patient with the suspicion of perianal abscess. CT may miss small abscesses. Endorectal ultrasound is extremely painful and often unnecessary in patients with perianal abscesses. Immunocompromised patients may present only with anal pain. It should not be dismissed when there are no outward signs of inflammation. These patients will need evaluation and treatment of any perianal abscesses identified on MRI before beginning their immunosuppressive regimen. They may not be able to combat this infection in their immunocompromised state. MRI should be utilized as this imaging modality can detect very small amounts of fluid in the perianal space, which may be causing discomfort in this patient. Go to the next page if you knew the correct answer, or click the link image(s) below to further research the concepts in this question (if desired).

Research Concepts: Perianal Abscess

We update eBooks quarterly and Apps daily based on user feedback. Please tap flag to report any questions that need improvement.

Question 197: A 17-year-old female with asthma presents to the emergency department with right upper quadrant abdominal pain, nausea, and jaundice for two weeks. Laboratory analysis shows alanine aminotransferase of 1048, aspartate aminotransferase of 1120, alkaline phosphatase of 212, and bilirubin of 4.1. INR is 2.1. She appears slightly confused and lethargic. Hepatitis C RNA by polymerase chain reaction (PCR) is not detected, HCV antibody is negative, hepatitis A IgM is positive, hepatitis B core antibody IgM is negative, hepatitis B core antibody IgG is positive, and hepatitis B surface antigen is negative. What is the next step in management?

Choices: 1. Admit to the hospital and initiate liver transplantation evaluation 2. Discharge the patient home as she has an acute hepatitis A infection which is a self-limited illness 3. Treat the patient for chronic hepatitis B infection with oral lamivudine 4. Admit the patient to the hospital for intravenous fluids, IV N-acetylcysteine, and close monitoring

Answer: 1 - Admit to the hospital and initiate liver transplantation evaluation Explanations: This patient has acute hepatitis A as indicated by the positive hepatitis A IgM. Acute hepatitis A infection is typically a mild, uncomplicated and self-limited illness in this young and healthy patient. However, this patient meets criteria for acute liver failure with coagulopathy and encephalopathy without preexisting cirrhosis, and with an illness of, 2 weeks' duration. Therefore, she should be admitted, and evaluation for liver transplantation should be initiated if she continues to clinically decompensate. Acute hepatitis B is diagnosed when patients have positive hepatitis B core antibody IgM plus or minus positive hepatitis B surface antigen. Patients with chronic hepatitis B will have persistence of hepatitis B surface antigen positivity beyond 6 months' duration. Go to the next page if you knew the correct answer, or click the link image(s) below to further research the concepts in this question (if desired).

Research Concepts: Hepatitis A

We update eBooks quarterly and Apps daily based on user feedback. Please tap flag to report any questions that need improvement.

Question 198: A 65-year-old man presents with complaints of painless visual blurring, which has happened twice in the last day. He has a history of hypertension, hyperlipidemia, and type 2 diabetes mellitus. He has a 30 pack-year smoking history, drinks alcohol occasionally, and has never used illicit drugs. He mentions that one day earlier, he lost his speech for a few minutes. On examination, it is determined that he has again lost vision in his left eye. What is the most appropriate next step in the management of this patient?

Choices: 1. Heparin infusion 2. Fibrinolytic therapy 3. CT brain scan without contrast 4. Acetylsalicylic acid

Answer: 3 - CT brain scan without contrast Explanations: Before beginning lytic therapy for a developing stroke, it is most essential to obtain CT brain imaging to ensure that there is no hemorrhagic component. Only embolic and ischemic strokes can be treated with lytic therapy. It is very dangerous to treat hemorrhagic strokes with lytic therapy. In this clinical scenario, painless loss of vision may be amaurosis fugax and may be associated with carotid artery stenosis. Thus, another recommended study is a duplex ultrasound of the neck to look at the carotid bifurcation. Alternatively, one could follow the unenhanced brain CT with a head and neck CT angiogram. Go to the next page if you knew the correct answer, or click the link image(s) below to further research the concepts in this question (if desired).

Research Concepts: Stroke Imaging

We update eBooks quarterly and Apps daily based on user feedback. Please tap flag to report any questions that need improvement.

Question 199: A 72-year-old female with a history of diabetes mellitus, hypertension, smoking, and previous myocardial infarction and multivessel disease presents with a history of central chest pain for 1 hour, severe in intensity, associated with nausea, vomiting diaphoresis, shortness of breath, and radiating to left jaw. Vital signs are BP 165/90 mmHg, pulse 127/min, RR 26/min, and afebrile. The chest is clear, and no significant findings on the cardiovascular exam are noted. ECG shows anterolateral myocardial infarction. Thrombolytics were given, and the patient improved. One day later she develops shock with BP 80/60 mmHg, pulse 140/min, RR=32/min, and SpO2=86% on room air. Auscultation of the chest reveals a blowing, high pitched pan-systolic murmur best heard at the apex, radiating to left axilla and accentuating on expiration with soft S1. There are fine inspiratory crackles to the mid-zone of the lungs. What is the initial treatment in this case?

Choices: 1. Intra-aortic balloon pump 2. Emergency surgical intervention 3. Intravenous nitrates 4. Intravenous diuretics

Answer: 1 - Intra-aortic balloon pump Explanations: The patient is suffering from acute pulmonary edema after papillary muscle rupture and acute mitral regurgitation. As the patient is in cardiogenic shock, the initial treatment of choice is intra-aortic balloon pump which increases the cardiac output and decreases the oxygen consumption/demand heart. Emergency surgical intervention the treatment of choice if the patient is hemodynamically stable. Mitral valve repair is done if the papillary muscles are not necrotic and mitral valve replacement if the muscles become necrotic. Intravenous nitrates are used to decrease the workload on the heart by decreasing both the preload and afterload when the blood pressure is within normal range. They cannot be used initially when the patient is hemodynamically stable. Intravenous diuretics are used for acute pulmonary edema but should be used with caution as they also tend to lower the blood pressure. As in this case, the patient is in cardiogenic shock, the initial step should be to resuscitate the patient and make him hemodynamically stable. Go to the next page if you knew the correct answer, or click the link image(s) below to further research the concepts in this question (if desired).

Research Concepts: Cardiogenic Pulmonary Edema

We update eBooks quarterly and Apps daily based on user feedback. Please tap flag to report any questions that need improvement.

Question 200: A 65-year-old male is found at home febrile and unresponsive. He is brought to the emergency department. His blood pressure is 60/40 mmHg, pulse 45 beats/min, temperature 38.4C, and respiratory rate 22/min. Upon administering fluids, a physical exam is conducted. There is marked tenderness at the costovertebral angle. Urinalysis is positive for leukocyte esterase and WBCs. Which of the following is the best medication to resuscitate him?

Choices: 1. Dobutamine 2. Epinephrine 3. Dopamine 4. Lidocaine

Answer: 2 - Epinephrine Explanations: Epinephrine is FDA-approved in adults with hypotension associated with septic shock. Pyelonephritis leading to septic shock is a medical emergency and needs to be treated as soon as possible. Epinephrine stimulates the body's sympathetic nervous system and plays a role in acute stress response. Another treatment for septic shock is norepinephrine. Go to the next page if you knew the correct answer, or click the link image(s) below to further research the concepts in this question (if desired).

Research Concepts: Epinephrine

We update eBooks quarterly and Apps daily based on user feedback. Please tap flag to report any questions that need improvement.

Section 3 Question 201: A 28-year-old African American female is brought to the emergency department with acute onset shortness of breath that has progressively worsened over the past 12 hours. According to her boyfriend at the bedside, she has not slept all night and has been using her inhaler every hour without any benefit. Symptoms suddenly became much worse over the past 3 hours. On arrival she appears anxious, in visible respiratory distress, and unable to speak more than a couple of words at a time. She is using her accessory muscles to breathe. The calculated difference between systolic blood pressure at end inspiration and expiration is 14 mmHg. She is promptly started on nebulized albuterol and ipratropium for continuous hour-long treatment and administered a dose of 2 grams magnesium sulfate, 125 mg methylprednisolone, and 2 liters oxygen supplementation. Two hours later, she is finally lying in bed and sleeping without distress. Boyfriend feels reassured that she is finally able to catch some rest. Her wheeze is less than before. Difference between systolic blood pressure at inspiration and expiration is now 5 mmHg. Which of the following is the next best step in the management of this patient?

Choices: 1. Wake her up and check her peak expiratory flow rate and FEV1 2. With a reduced respiratory variation of blood pressure and lack of wheeze get a chest xray to evaluate for pneumothorax 3. Get an ABG immediately and prepare for endotracheal intubation 4. Continue hourly nebulization treatment for 4 more hours and transition to 60 mg solumedrol every 6 hourly.

Answer: 3 - Get an ABG immediately and prepare for endotracheal intubation Explanations: While the ability to lie supine on a status asthmaticus patient who was sitting upright may reflect a treatment response it has to be assessed in relation to the entire clinical picture. She appears sleepy but could be lethargic indicative of respiratory fatigue. Lack of wheezing could be a sign of worsened air entry as well. Similarly, a sudden reduction in pulsus paradoxus can happen in the progression of respiratory failure and hypoventilation from respiratory muscle fatigue. Preparation for mechanical ventilatory support seems appropriate in the circumstance. Pulsus paradoxus is caused by a reduction in left ventricular outflow with air hunger and deep inspiration causing an increase in LV and RV afterload and ventricular interdependence. Greater than 12% is considered significant. However, in worsening status asthmaticus patient may be too fatigued to take deep enough breaths to result in a large variation of pressure. An ABG performed in a clinical setting as above may show normal PCO2 which again should be interpreted in light of prior carbon dioxide partial pressure. Patient in status asthmaticus presents initially with respiratory alkalosis and low PCO2. Normalization may, therefore, be an ominous sign indicating rising level and worsening fatigue of respiratory apparatus. If a patient does not excessive secretions, is somnolent but arousable, and has no ongoing nausea, noninvasive ventilatory support with BIPAP can also be tried for a short duration to be reassessed in 3-4 hours. Go to the next page if you knew the correct answer, or click the link image(s) below to further research the concepts in this question (if desired).

Research Concepts: Status Asthmaticus

We update eBooks quarterly and Apps daily based on user feedback. Please tap flag to report any questions that need improvement.

Question 202: A 25-year-old male, who has been HIV positive for about 8 years, is admitted to the hospital with respiratory distress. The patient had focal neurological deficits earlier and now is in a comatose state. The pulmonary and central nervous system biopsies show dichotomously branching septate hyphae. What is the most likely underlying condition?

Choices: 1. Severe neutropenia 2. CD4+ cell count less than 600 3. Diabetic ketoacidosis 4. Multiple myeloma

Answer: 1 - Severe neutropenia Explanations: Invasive aspergillosis is found primarily in patients with neutrophil counts less than 500/mm3. It also is seen with chronic granulomatous disease and cystic fibrosis. The use of antifungals and antimicrobials is needed. Immunological support includes the administration of IV purified immunoglobulins and antiretroviral therapy. Frequent evaluation of CD4+ count should be done. Go to the next page if you knew the correct answer, or click the link image(s) below to further research the concepts in this question (if desired).

Research Concepts: Neutropenia

We update eBooks quarterly and Apps daily based on user feedback. Please tap flag to report any questions that need improvement.

Question 203: A 78-year-old lady presents to the emergency department with acute symptoms of lightheadedness and dizziness for past 4 hours. She called her son home for not feeling well. He found her blood pressure to be 80/40 mmHg and brought her to the hospital promptly. She had only taken 6 of her prednisone pills as prescribed after dinner. However, he found the pill bottle with the medication name as verapamil. Depending on the severity of manifestations, which of the following is not a recommended therapeutic intervention?

Choices: 1. Urgent hemodialysis 2. Intravenous calcium chloride 3. High dose insulin with dextrose 4. Intravenous lipid emulsion

Answer: 1 - Urgent hemodialysis Explanations: Calcium channel blockers are highly protein bound and have a very large volume of distribution. Hemodialysis or hemofiltration are therefore ineffective in their toxicities. Hemodialysis may, however, be necessary if a patient develops acute renal failure due to acute tubular necrosis from hypoperfusion. Intravenous calcium chloride is the recommended first step in treatment with careful monitoring of serum calcium level. Severe toxicity is managed by incremental doses of intravenous insulin infusion. Go to the next page if you knew the correct answer, or click the link image(s) below to further research the concepts in this question (if desired).

Research Concepts: Calcium Channel Blocker Toxicity

We update eBooks quarterly and Apps daily based on user feedback. Please tap flag to report any questions that need improvement.

Question 204: An 18-year-old female patient is admitted to the hospital with refractory nausea and vomiting. About three days ago she had an upper respiratory tract infection and has been progressively worse since. History is significant for amenorrhea, repeated candida infections of the oral cavity, and debilitating cramps of her hands. Heart rate is 110/min, respiratory rate 18/min, temperature 99.6 F, and blood pressure 90/58 mmHg. On physical exam, she is noted to have hyperpigmentation of the knuckles, the absence of axillary hair, and white patches in the oral cavity. What is the next best step in the management of this patient?

Choices: 1. Obtain plasma ACTH and cortisol at 8 AM 2. Obtain MRI of the pituitary gland 3. Obtain CT scan of the adrenal glands 4. Start stress doses of hydrocortisone

Answer: 4 - Start stress doses of hydrocortisone Explanations: Plasma and cortisol can be drawn at the time patient is being evaluated in the emergency department or the hospital prior given the first shot of high dose corticosteroids, but given patient is acutely ill, waiting until 8 am to give steroids is not appropriate. Very high levels of ACTH may be seen in patients. Based on the clinical presentation the patient most likely has primary adrenal insufficiency, there is no need to obtain an MRI of the pituitary at this time. Imaging of the adrenals at this time is not needed given she can be diagnosed clinically and biochemically first. Furthermore, this would delay treatment and will not likely establish the diagnosis. Adrenal insufficiency can present with fatigue, abdominal pain, nausea, vomiting, and dizziness. Adrenal crisis is treated by starting stress dose hydrocortisone which results in marked improvement of the patient’s status. A history of chronic candidiasis and hypocalcemia as evidenced by cramps in her hands indicate a diagnosis of APS-1. Go to the next page if you knew the correct answer, or click the link image(s) below to further research the concepts in this question (if desired).

Research Concepts: Polyglandular Autoimmune Syndrome Type I

We update eBooks quarterly and Apps daily based on user feedback. Please tap flag to report any questions that need improvement.

Question 205: A 44-year-old female presents after a motor vehicle collision with an isolated tibial fracture of her right leg. She has swelling and bruising at the site of trauma. She has increasing pain medication requirements in the emergency department, and her pain is worse when her leg is passively stretched. She now reports tingling and numbness. On re-examination, the dorsalis pedis pulse is not palpable, and she cannot feel light touch to her foot. Which history or exam finding in acute compartment syndrome is most likely to present first?

Choices: 1. Loss of sensation and motor function in her foot 2. Loss of pulses of the affected leg and foot 3. Inability to regulate temperature with abnormal sweating 4. Worsening pain of the leg made worse with passive stretching

Answer: 4 - Worsening pain of the leg made worse with passive stretching Explanations: Acute compartment syndrome classically presents with the 6 P’s: pain, paresthesia, paresis, pallor, poikilothermia, and pulselessness. However, this is not the typical clinical picture. Most patients are going to have worsening pain out of proportion to the exam which is also made worse with passive stretching. Loss of sensation and motor function to the foot can be seen, but typically will not present first. Pulselessness is typically a late finding. Poikilothermia or inability to regulate temperature is a late finding and probably the least likely to be obvious by history or exam. Go to the next page if you knew the correct answer, or click the link image(s) below to further research the concepts in this question (if desired).

Research Concepts: Tibial Anterior Compartment Syndrome

We update eBooks quarterly and Apps daily based on user feedback. Please tap flag to report any questions that need improvement.

Question 206: A 25-year-old male is admitted to the hospital for rectal bleeding, diarrhea, purulent rectal discharge, tenesmus, and anal pain. He has no past history of similar rectal bleeding. Examination shows erythema around the perianal region and the perineum and few ulcers and vesicles. A colonoscopy reveals erythema, friability of rectal mucosa, ulcerations, vesicles, and mucopurulent exudates. Rectal biopsies show negative PCR for Neisseria gonorrhoeae and Chlamydia trachomatis. Tests for Treponema pallidum and HIV are all negative. Which of the following is the next best step in the management of this patient?

Choices: 1. Viral cultures 2. Multidetector CT scan of pelvis and abdomen 3. PCR for Mycobacterium tuberculosis 4. Microscopic examination of swabs

Answer: 1 - Viral cultures Explanations: The patient’s presentation (rectal bleeding, diarrhea, purulent rectal discharge, tenesmus, and anal pain together with skin erythema around the perianal region and the perineum and the presence of few ulcers and vesicles) are consistent with herpes simplex 2 infection. Other sexually transmitted infections including Neisseria gonorrhoeae, Chlamydia trachomatis, Treponema pallidum, and HIV were excluded. Therefore, viral cultures or swabs from rectal vesicles or ulcers to detect virus DNA by PCR should be ordered to confirm the diagnosis Multidetector CT scan of pelvis and abdomen may help in differentiating between bacterial colitis and inflammatory bowel disease. However, these scans cannot confirm herpes simplex-2 infection. Viral cultures or swabs from rectal vesicles or ulcers to detect virus DNA by PCR should be ordered to confirm the diagnosis of herpes simplex 2. The patient’s clinical presentation is not consistent with tuberculous infection. He is not HIV positive and there are no indications that he is immunocompromised or at risk of developing tuberculosis. Viral cultures or swabs from rectal vesicles or ulcers to detect virus DNA by PCR should be ordered to confirm the diagnosis of herpes simplex 2. The patient’s presentation is not consistent with bacterial colitis and Microscopic examination of swabs will not help. Viral cultures or swabs from rectal vesicles or ulcers to detect virus DNA by PCR should be ordered to confirm the diagnosis of herpes simplex 2. Go to the next page if you knew the correct answer, or click the link image(s) below to further research the concepts in this question (if desired).

Research Concepts: Infectious Colitis

We update eBooks quarterly and Apps daily based on user feedback. Please tap flag to report any questions that need improvement.

Question 207: A 35-year-old female is brought to the emergency department with complaints of abdominal pain. Her blood pressure was initially measured as 70/40 mmHg by Emergency Medical Service with a heart rate of 135 and a temperature of 39 C at her home. She was given 2 L normal saline en route resulting in an improvement in blood pressure to 95/50 and HR to 110 upon arrival to the emergency department. Physical examination shows lower abdominal tenderness. Her blood work is significant for lactate of 4.2 mmol/L. A urinalysis comes reports a high number of pus cells and bacteria while urine culture is pending. Despite repeated efforts for about an hour, the phlebotomist is unable to draw blood cultures. The patient has been in the hospital for about three hours now. Which of the following is the best next step in the management of this patient?

Choices: 1. Keep attempting blood cultures for another hour and defer antibiotics 2. Administer antibiotics despite the possibility of cultures being negative thereafter 3. Give another saline bolus while deferring antibiotics and retry cultures afterward 4. Administer antibiotics and cancel blood cultures as urine cultures were already drawn

Answer: 2 - Administer antibiotics despite the possibility of cultures being negative thereafter

Explanations: Delay to antibiotic administration has been associated with increased in-hospital mortality. Waiting to draw blood cultures in this instance would be inappropriate. Delaying antibiotic administration compared between 1 hour and 3 hours of sepsis identification has not been shown to affect patient-centered outcomes or mortality. However the time to antibiotic administration has been shown to increase in-hospital mortality, so delaying antibiotic administration beyond the 3-hour point in this patient would be inappropriate. While intravenous hydration is indicated, it should not delay cultures further. Blood cultures should be drawn as this patient’s presentation is consistent with pyelonephritis and acquiring urine cultures alone would not be enough. This can be from bacteremia seeding the kidney rather than from an ascending urinary tract infection. Go to the next page if you knew the correct answer, or click the link image(s) below to further research the concepts in this question (if desired).

Research Concepts: Bacterial Sepsis

We update eBooks quarterly and Apps daily based on user feedback. Please tap flag to report any questions that need improvement.

Question 208: An elderly male with long-standing chronic obstructive pulmonary disease (COPD) is rushed to the emergency room with exacerbation of his COPD. He is severely short of breath and gasping. Quick blood work reveals an arterial blood gas of pH 7.15, PCO2 55, PO2 54 and HCO3 15. He is admitted to the intensive care unit and the pulmonologist intubates him. He remains on the mechanical ventilator for 13 days and is eventually extubated. According to the 2015 Global Initiative for Chronic Obstructive Lung Disease, what is the mortality of such patients at 12 months after discharge?

Choices: 1. Less than 5% 2. 10% to 15% 3. 40% to 60% 4. 90%

Answer: 3 - 40% to 60% Explanations: COPD carries a high morbidity and mortality. The GOLD report indicates that patients with COPD who require mechanical ventilation have a high mortality at 12 months. Any acute exacerbation should be promptly treated because delays often result in protracted hospital admissions. The aim is to reduce the burden of COPD. Factors associated with negative prognosis include hypercapnia, low body mass indices, advanced age, low serum albumin, long-term use of corticosteroids and poor functional status. Go to the next page if you knew the correct answer, or click the link image(s) below to further research the concepts in this question (if desired).

Research Concepts: Chronic Obstructive Pulmonary Disease

We update eBooks quarterly and Apps daily based on user feedback. Please tap flag to report any questions that need improvement.

Question 209: A 64-year-old male with a past medical history of diabetes mellitus, hypertension, obesity, and tobacco abuse presents to the emergency department in a rural area with crushing 10/10 substernal chest pain of 30 minutes duration. He receives 162 mg of aspirin and 0.4 mg of sublingual nitroglycerin en route to the hospital. Emergency medical services report ST elevations in V3-V6 which is corroborated by the EKG acquired upon arrival to the emergency department. Initial troponin is 14.6 ng/mL. What is the most appropriate next step in treatment?

Choices: 1. Transfer to percutaneous coronary intervention (PCI) capable facility 4 hours away 2. Thrombolytic therapy followed by transfer to a PCI capable facility 4 hours away 3. Thrombolytic therapy followed by a heparin drip 4. Start patient on 1 mg/kg of low molecular weight heparin

Answer: 2 - Thrombolytic therapy followed by transfer to a PCI capable facility 4 hours away

Explanations: In patients where percutaneous coronary intervention (PCI) cannot be performed in under 2 hours, the decision should be made to begin fibrinolytic therapy. Patients who receive thrombolytic therapy should still be transferred to a PCI capable facility to undergo PCI in the first 3 to 24 hours after the ST-elevation myocardial infarction. Thrombolytic therapy alone is only effective in 50% of patients. Thrombolysis or PCI treatment is the treatment for ST-elevation myocardial infarction, anticoagulation alone is not sufficient. Go to the next page if you knew the correct answer, or click the link image(s) below to further research the concepts in this question (if desired).

Research Concepts: Acute ST Elevation Myocardial Infarction

We update eBooks quarterly and Apps daily based on user feedback. Please tap flag to report any questions that need improvement.

Question 210: You are seeing a 64-year-old obese male who was admitted with some chest pain. He says that the chest pain lasted a few minutes and he was admitted to ensure that he had not suffered a myocardial infarction. The cardiac enzymes turned out to be normal and the ECG is normal sinus rhythm. Today, you are asked to do the easy thing first. Measure the blood pressure. You measure the blood pressure and notice that it is 90/80 mmHg in the right arm and 95/75 mmHg in the left arm. You measure the blood pressure three more times and get the same result. What is the most likely cause of this patient's pathology?

Choices: 1. Smoking 2. Diabetes 3. Degenerative calcification 4. Rheumatic fever

Answer: 3 - Degenerative calcification Explanations: When the pulse pressure is narrow, think of heart failure, significant blood loss, or severe aortic stenosis. A low pulse pressure is commonly seen in shock and cardiac tamponade The most common cause of aortic stenosis is degenerative, meaning that calcium starts to deposit on the valves. Other things that can cause low pulse pressure include reduced preload and dehydration. Go to the next page if you knew the correct answer, or click the link image(s) below to further research the concepts in this question (if desired).

Research Concepts: Aortic Stenosis

We update eBooks quarterly and Apps daily based on user feedback. Please tap flag to report any questions that need improvement.

Question 211: A 16-year-old patient with uncontrolled severe asthma presents to the emergency department with fever, breathlessness, and cough with brownish expectoration for the last seven days. His Laboratory findings show hemoglobin of 11.7 g/dL, white blood cells 5600/cmm, platelets 2,56,000/cmm, total IgE level 4000 kU/L and peripheral blood eosinophil count 2540 cells/microL. Sputum examination shows fungal hyphae, fibrin, Charcot-Leyden crystals, Curschmann spirals & dichotomous branching of fungal hyphae seen at 45-degree angles. HRCT shows bronchiectasis in the right upper lobe. Which of the following organism is most likely responsible for the patient's condition?

Choices: 1. Aspergillus niger 2. A. flavus 3. A. nidulans 4. A. fumigatus

Answer: 4 - A. fumigatus Explanations: As per patient's history, laboratory findings and radiological finding, most probable diagnosis is allergic bronchopulmonary aspergillosis (ABPA). Aspergillus fumigatus is the most common ubiquitous airborne fungus causative organism for ABPA. Aspergillus conidia, because of its small diameter (2-3 µm), easily reach to the pulmonary alveoli and deposits there. A. fumigatus species need dead organic matters to survive, so they thrive on fallen leaves and its compost so infection present most profusely in the winter throughout the world. ABPA usually found in asthmatic patients mostly in severe asthmatic and people with cystic fibrosis. Aspergillus flavus, Aspergillus niger, Aspergillus terreus, and Aspergillus nidulans have emerged as important pathogens in immunocompromised individuals. Aspergillus nidulans is the second most encountered mold in CGD patients, causing almost exclusively invasive infections. Go to the next page if you knew the correct answer, or click the link image(s) below to further research the concepts in this question (if desired).

Research Concepts: Allergic Bronchopulmonary Aspergillosis

We update eBooks quarterly and Apps daily based on user feedback. Please tap flag to report any questions that need improvement.

Question 212: A 48-year-old female patient in the ICU recovering from a brain surgery presents with multiple small lesions on her legs. She has multiple 1 mm non-blanching lesions throughout her legs, with the majority concentrated on her shins. The patient has a past medical history significant for hypertension, diabetes insipidus, and glioblastoma. She has not regained consciousness since her surgery two days ago. Her family states that the patient has had these lesions before, but they are not sure of what precipitated them in the past. The patient is currently receiving lisinopril, hydrochlorothiazide, and desmopressin. She is also receiving trimethoprimsulfamethoxazole for a urinary tract infection she developed in the hospital. Which of the following is the most likely cause of this patient's symptoms?

Choices: 1. Desmopressin 2. Hydrochlorothiazide 3. Trimethoprim-sulfamethoxazole 4. Lisinopril

Answer: 1 - Desmopressin Explanations: Desmopressin is used in patients suffering from diabetes insipidus. A rare side effect of desmopressin is the occurrence of thrombocytopenic purpura. Patients with a history of thrombocytopenic purpura in the past are more at risk for developing purpuras when on desmopressin. Desmopressin acts as an ADH analog in patients suffering from diabetes insipidus to help retain water in these patients. Go to the next page if you knew the correct answer, or click the link image(s) below to further research the concepts in this question (if desired).

Research Concepts: Desmopressin

We update eBooks quarterly and Apps daily based on user feedback. Please tap flag to report any questions that need improvement.

Question 213: An 81-year-old male presents to the emergency department with a chief complaint of a severe headache. He reports the onset of his headache occurred approximately 1 hour after dinner at a friend's house where he ate charcuterie, assorted cheeses, pasta with meatballs, and had one glass of red wine. The patient is alert and oriented to person, place, time, and situation. He denies loss of consciousness or shortness of breath. His past medical history is remarkable for myocardial infarction 3 years ago treated with emergent angioplasty and a history of depression on and off for the past 20 years. The patient appears well groomed, well nourished, and in distress. A Cincinnati stroke assessment is negative. His vitals are a heart rate of 52 beats/min, blood pressure 210/120 mmHg, respiratory rate 26, and SaO2 is 97% on room air. The patient takes low dose aspirin, clopidogrel, metoprolol, and phenelzine daily. The patient states that he was recently seen by a psychiatrist for the return of his depression prompting a discontinuation of sertraline and the initiation of phenelzine. While waiting for a CT of the brain, which of the following is the most beneficial medication to administer to the patient at this time?

Choices: 1. Verapamil 2. Propanolol 3. Norepinephrine 4. Fenoldopam

Answer: 4 - Fenoldopam Explanations: Fenoldopam has an onset of action of 10 minutes in adults and 5 minutes in children. Peripheral vasodilation and blood pressure lowering will cause a compensatory tachycardia. Routine vitals such as blood pressure and heart rate, in addition to serial ECGs, renal and hepatic function tests, and serum potassium, should be monitored during fenoldopam infusion. A detailed evaluation of the chief complaint and inciting events can prove to be beneficial in this patient. The patient's past medical history of depression being treated with phenelzine and the recent ingestion of tyramine-containing food and drinks caused a hypertensive crisis. Tyramine is found in many types of cured meat, cheeses, beers, and red wine. When ingested in excess in a patient taking monoamine oxidase inhibitors (MAOIs), a catecholamine surge occurs, causing an increase in blood pressure and heart rate. Because this patient is taking a cardioselective beta blocker (metoprolol), the catecholamine surge is not affecting his heart rate. A reflexive bradycardia can be seen as a normal compensatory mechanism in patients with high blood pressure, depending on the etiology. However, it is exaggerated in this patient due to the unopposed parasympathetic effect of vagal stimulation from the carotid sinus baroceptors. In this case, fenoldopam, a dopamine D1 receptor agonist, is the best medication to give, as it has a short onset of action and will not further worsen the patient's bradycardia. Go to the next page if you knew the correct answer, or click the link image(s) below to further research the concepts in this question (if desired).

Research Concepts: Fenoldopam

We update eBooks quarterly and Apps daily based on user feedback. Please tap flag to report any questions that need improvement.

Question 214: A 32-year-old female who is obese and has a history of migraine headaches flew from Los Angeles, CA to Aspen, CO and spent a day at a local spa. While in the sauna, she became dizzy and sluggish and then developed a dull, moderately severe bilateral frontal headache and nausea. She tried to stand but fell and was found oriented but lethargic. What is the appropriate treatment?

Choices: 1. Acetazolamide, oxygen, and corticosteroids 2. Oral rehydration and bedrest 3. Have the patient take her own sumatriptan 4. Have the patient fly home to Los Angeles

Answer: 1 - Acetazolamide, oxygen, and corticosteroids Explanations: The patient has a high altitude headache with some features of acute mountain sickness. The patient ascended from sea level to 7,900 feet of elevation, was likely dehydrated, and is obese. These are all risk factors. A high-altitude headache occurs within 24 hours after assent. Acute mountain sickness can be life-threatening with resulting cerebral edema. Treatment with acetazolamide, oxygen, and corticosteroids are indicated. Go to the next page if you knew the correct answer, or click the link image(s) below to further research the concepts in this question (if desired).

Research Concepts: High Altitude Cerebral Edema

We update eBooks quarterly and Apps daily based on user feedback. Please tap flag to report any questions that need improvement.

Question 215: An 85-year-old man was hospitalized because of acute exacerbation of chronic obstructive lung disease secondary to lung infection. He has a history of chronic bronchitis, heart failure with reduced ejection fraction, hypertension and coronary artery disease. On the following night, his blood pressure dropped to 80/65 mm/Hg. He was started on a oneliter normal saline fluid bolus. After 30 minutes, his repeated blood pressure was 82/64 mm/Hg. On evaluation, the patient is feeling dizzy and not-fully comprehensive but denies any ongoing chest pain. Pulse is 85 /min and oxygen saturation 84% on room air. Chest examination shows bilateral fine crackles and jugular venous distention. The rest of the examination is unremarkable. The patient is started on oxygen therapy and his oxygen saturation improved to 90%. He is transferred to the intensive care unit, and bedside ultrasound shows dilated inferior vena cava. Bloodwork reveals worsened kidney function and elevated troponin. Which of the following is the next best step in the management of this patient?

Choices: 1. Continue fluid resuscitation with another 1 L fluid bolus 2. Start him on loading dose loop diuretic followed by a diuretic infusion 3. Start him on aspirin and heparin drip 4. Proceed with venous ultrafiltration

Answer: 2 - Start him on loading dose loop diuretic followed by a diuretic infusion Explanations: Patients with a history of heart failure with reduced ejection fraction are more susceptible to develop acute cardiorenal syndrome. This patient was admitted because of acute exacerbation of his chronic bronchitis which resulted in acute cardiorenal syndrome, (type-1 cardiorenal syndrome). Physical examination revealed bilateral crackles and jugular venous distention after 1 L of fluid resuscitation with ultrasound finding of dilated inferior vena cava would be a sign of fluid overload status secondary to the cardiorenal syndrome. The primary treatment of cardiorenal syndrome to target fluid removal from the body either with diuretic or ultrafiltration. Using inotropes are kept for resistance cases. There are different types of diuretics: loops, thiazide, and potassium-sparing diuretics. Loop diuretics including furosemide, torsemide, and bumetanide are the most potent diuretic they can be used alone or conjugated with other types of diuretic. Proceeding with further fluid therapy would result in worsening respiratory function status as the patient has elevated venous pressure. He has mild elevation in troponin with no active chest pain at the moment. It would be more reasonable to repeat the troponin for now before starting him on a heparin drip. Finally, venous ultrafiltration can be used in resistance cases, but recent studies showed that diuretic therapy was better than ultrafiltration on symptoms control and creatinine level decline in the initial approach toward euvolemic. Go to the next page if you knew the correct answer, or click the link image(s) below to further research the concepts in this question (if desired).

Research Concepts: Cardiorenal Syndrome

We update eBooks quarterly and Apps daily based on user feedback. Please tap flag to report any questions that need improvement.

Question 216: A patient with end-stage renal disease who has missed her last scheduled dialysis appointments presents to the emergency department with weakness, dyspnea, rales, and jugular venous distension. The monitor shows a wide complex bradycardia. Which medication is not indicated for her suspected electrolyte imbalance?

Choices: 1. Magnesium IV 2. Calcium gluconate IV 3. Polystyrene sulfonate PO or rectal 4. Glucose and regular insulin IV

Answer: 1 - Magnesium IV Explanations: Magnesium is not indicated in the treatment of suspect hyperkalemia, which is causing the wide complex bradycardia. Calcium gluconate stabilizes the cell membrane. Polystyrene sulfonate helps to excrete potassium. Glucose and insulin drive potassium back into the cells. Go to the next page if you knew the correct answer, or click the link image(s) below to further research the concepts in this question (if desired).

Research Concepts: Hyperkalemia

We update eBooks quarterly and Apps daily based on user feedback. Please tap flag to report any questions that need improvement.

Question 217: A 16-year-old female presented with fever, a target-shaped rash on her thigh, and episodes of dizziness three days after a hike through the woods in New Hampshire. Her electrocardiogram revealed a complete heart block with a junctional rhythm, and she received one dose of ceftriaxone for Lyme carditis. Five hours after the ceftriaxone dose, she became unresponsive with non-palpable pulses, telemetry showed ventricular tachycardia, oxygen saturation dropped to 82%, and auscultation revealed bilateral crackles. She was resuscitated with defibrillation and a 300 mg bolus of amiodarone, following which she was started on inotropic support. An echocardiogram revealed a severely impaired systolic function of the left and the right ventricles and a left ventricular ejection fraction of 12%. Coronary angiography revealed patent vessels, and a biopsy revealed multiple foci of lymphocytic infiltration (CD 3+) with associated myonecrosis. Serology was only positive for anti-Borrelia burgdorferi IgM. The patient continued to be in a complete heart block with episodes of pauses and ventricular tachycardia that persisted even after the placement of a temporary wire. Serial CRP revealed a worsening trend over three days. What is the next best step?

Choices: 1. Increase the dose of ceftriaxone 2. Permanent pacemaker 3. Switch to doxycycline 4. Start corticosteroid therapy

Answer: 4 - Start corticosteroid therapy Explanations: Spirochetal diseases like Lyme disease can display a Jarisch-Herxheimer reaction after initiation of antimicrobial therapy. It is caused by the widespread release of antigens from microbial killing, causing cytokine release. This triggers a systemic inflammatory response which may cause worsening of the myocarditis. Steroid therapy can suppress inflammation and improve cardiac function. Go to the next page if you knew the correct answer, or click the link image(s) below to further research the concepts in this question (if desired).

Research Concepts: Nonviral Myocarditis

We update eBooks quarterly and Apps daily based on user feedback. Please tap flag to report any questions that need improvement.

Question 218: A 63-year-old female patient who was found confused with altered mental status at home was brought to the emergency department which initial chest x-ray showed suspected aspiration pneumonia with right lower lobe infiltrate and was admitted to intensive care unit. Due to low hemoglobin of 5.1 g/dL, 2 packed red blood cells (PRBCs) were ordered, and 1 PRBCs transfusion was immediately started. After 6 hours posttransfusion, oxygen saturation dropped and required a non-rebreather mask from previous 2 L oxygen by nasal cannula to keep oxygen saturation up to 91%. Blood pressure dropped from 132/87 mmHg to 87/62 mmHg. Chest auscultation revealed coarse breath sounds from the anterior chest with bilateral crackles. Pretransfusion CBC had WBC of 10.2 and platelet of 180. Repeat CBC showed WBC of 2.1 and platelet of 78. Intravenous furosemide 40 mg was given with no effect. What is the most likely the diagnosis?

Choices: 1. Flash pulmonary edema 2. Transfusion-related acute lung injury (TRALI) 3. Acute respiratory distress syndrome (ARDS) 4. Transfusion-associated circulatory overload (TACO)

Answer: 2 - Transfusion-related acute lung injury (TRALI) Explanations: This case is most likely TRALI as, within 6 hours post-transfusion, the patient developed hypotension. TRALI does not respond to diuretic as this is noncardiogenic pulmonary edema as opposed to flash pulmonary edema and TACO. Other answer choices are possibilities, but this happened after transfusion Leukopenia and thrombocytopenia happen with TRALI as TACO, flash pulmonary edema and ARDS do not change WBC and platelet values. Go to the next page if you knew the correct answer, or click the link image(s) below to further research the concepts in this question (if desired).

Research Concepts: Transfusion-related Acute Lung Injury

We update eBooks quarterly and Apps daily based on user feedback. Please tap flag to report any questions that need improvement.

Question 219: An 83-year-old male developed a cholecysto-cutaneous fistula after prolonged untreated cholecystitis. He has mild pain and asymptomatic otherwise, except the drainage site at the right upper quadrant. Which of the following is the next best step in the management of this patient?

Choices: 1. Pressure dressing to minimize the drainage amount 2. Routine wet to dry dressing 3. Stoma bag well fitted on the fistula opening 4. Suture closure of the fistula opening

Answer: 3 - Stoma bag well fitted on the fistula opening Explanations: The appropriate management is to apply a stoma bag to collect and quantitate the drainage conveniently. It should be well fitted to prevent skin irritation. The management of cholecyst-cutaneous fistula requires initial drainage of any associated abscess and administration of appropriate antibiotics. Before the definitive surgery is performed, an effort must be taken to ensure that there is no associated bile duct obstruction or stones. An indication of an unresectable disease is if a gallbladder fistula occurs in the setting of cancer with an associate dated mass. In this case, the overall outcome is dismal because gallbladder cancer has a predilection for seeding and spreading to the adjoining anatomical structures. A pressure dressing is useful to stop bleeding sites. Fistula opening should not be sealed. This will facilitate creating other alternative tracts of the fistula. A regular routine dressing can be used. But it becomes wet and soaked easily which needs to be changed very frequently. This makes impractical for nursing care. Closing the fistula opening by suturing is contra-indicated. It promotes uncontrolled drainage through alternative tracts. Go to the next page if you knew the correct answer, or click the link image(s) below to further research the concepts in this question (if desired).

Research Concepts: Cholecystocutaneous Fistula

We update eBooks quarterly and Apps daily based on user feedback. Please tap flag to report any questions that need improvement.

Question 220: A 29-year-old male patient was admitted to the hospital after presenting an erythematous rash with central clearing on his back and flu-like symptoms followed by palpitations and dyspnea on exertion. There was evidence of third-degree atrioventricular block on the electrocardiogram at the time of admission. Immediate treatment with intravenous ceftriaxone was started. A temporary pacemaker was also placed. On day four of admission patient is feeling better, palpitations and shortness of breath have subsided. The repeat electrocardiogram shows no further P waves and QRS dissociation, heart rate of 76/min, and the PR interval of 350 ms. The patient is complaining that he feels uncomfortable with the pacemaker attached to his body and thinks he no longer needs it as his symptoms have improved. What is the next best step in the management of this patient?

Choices: 1. Remove pacemaker and continue treatment with intravenous ceftriaxone and evaluation with daily electrocardiogram 2. Remove pacemaker and discharge the patient on oral doxycycline to complete a total course of 21 days 3. Explain to the patient that pacemaker needs to continue attached to his body and that he will continue daily evaluations with an electrocardiogram 4. Place permanent pacemaker

Answer: 1 - Remove pacemaker and continue treatment with intravenous ceftriaxone and evaluation with daily electrocardiogram

Explanations: A temporary pacemaker in patients with Lyme carditis and atrioventricular block can be removed once second-degree or third-degree heart block has resolved. Treatment with intravenous antibiotics should be continued until the PR interval shortens to less than 300 ms. After PR interval shortens to less than 300 ms, treatment can be switched to oral amoxicillin or doxycycline to complete 14 to 21 days. The recovery time in patients with third-degree heart block has a median of 6 days with adequate antibiotic treatment. They do not usually need a permanent pacemaker. Go to the next page if you knew the correct answer, or click the link image(s) below to further research the concepts in this question (if desired).

Research Concepts: Lyme Carditis

We update eBooks quarterly and Apps daily based on user feedback. Please tap flag to report any questions that need improvement.

Question 221: A patient in respiratory distress presents to the emergency department. Her pulse oximeter reads 88%, and her respiratory rate is 22/minute. A severe asthma exacerbation is suspected. The other vital signs are normal, and a normal perfusion status is presumed. The only other history comes from the patient's adult daughter who tells you that she also has type 2 diabetes mellitus and takes an oral medication which she took 3 hours earlier. Arterial blood gas shows a high anion gap metabolic acidosis. How is this metabolic acidosis best classified?

Choices: 1. Diabetic ketoacidosis 2. Ethanol poisoning 3. Type A lactic acidosis 4. Type B lactic acidosis

Answer: 4 - Type B lactic acidosis Explanations: While diabetic ketoacidosis typically presents as a high anion gap metabolic acidosis in diabetes mellitus type 1, this diagnosis is unlikely in a non-insulin dependent patient with type 2 diabetes. Nothing in this patient's history suggests that she is intoxicated. Type A lactic acidosis is seen in low perfusion states such as shock. This patient's history is not suggestive of a shock state. Metformin-induced lactic acidosis is rare and often associated with hypoxic states. This patient likely suffers from type B lactic acidosis which is that which is not associated with low perfusion states. Go to the next page if you knew the correct answer, or click the link image(s) below to further research the concepts in this question (if desired).

Research Concepts: Lactic Acidosis

We update eBooks quarterly and Apps daily based on user feedback. Please tap flag to report any questions that need improvement.

Question 222: A 50-year-old male with alcohol use disorder is admitted with acute pancreatitis. He admits to drinking a fifth of vodka daily and is having epigastric pain, nausea, and has vomited once but denies hematemesis or hematochezia. Vital signs are pulse 115 beats/min, respirations 16 breaths/min, and blood pressure 160/80 mmHg. The abdomen reveals decreased bowel sounds and is tympanic. There is tenderness to palpation at the mid epigastrium without rebound, masses, or splenomegaly. The liver is 16 cm at the midclavicular line. Lipase is 1200 units/L, amylase 550 units/L, alanine aminotransferase 140 units/L, aspartate aminotransferase 270 units/L, alkaline phosphatase 90 units/L, and albumin 2.5 grams/L. What is the best management of this patient?

Choices: 1. Intravenous fluids and nasogastric tube 2. Analgesia, intravenous fluids, nothing by mouth, and antiemetics 3. Analgesia, nasogastric tube, intravenous fluids, and total parenteral nutrition 4. Analgesia, intravenous fluids, and clear liquids

Answer: 2 - Analgesia, intravenous fluids, nothing by mouth, and antiemetics Explanations: The patient will most likely recover in 3 to 7 days if he receives fluids and takes nothing by mouth. A nasogastric tube may need to be placed if there is protracted emesis. In mild cases, a patient can be treated with antiemetics if there are no contraindications. Nasojejunal feeding tubes are preferable to total parental nutrition in cases of severe pancreatitis when oral feeds are not tolerated but nutritional support is needed. This reduces the risk of infectious complications of total parental nutrition. In mild cases of acute pancreatitis, the patient initially should take nothing by mouth. Oral feeding with clear fluids or a low-fat diet can begin as soon as nausea, vomiting, and abdominal pain has resolved. Go to the next page if you knew the correct answer, or click the link image(s) below to further research the concepts in this question (if desired).

Research Concepts: Acute Pancreatitis

We update eBooks quarterly and Apps daily based on user feedback. Please tap flag to report any questions that need improvement.

Question 223: A 17-year-old male is struck in the chest by a hard-driven ball while playing baseball and immediately collapses. He is pulseless, and CPR is initiated by his coach, and EMS is called. What is the most likely rhythm EMS will observe upon arrival?

Choices: 1. Ventricular fibrillation 2. Ventricular tachycardia 3. Asystole 4. Pulseless electrical activity (PEA)

Answer: 1 - Ventricular fibrillation Explanations: This presentation is most consistent with an episode of commotio cordis, which most commonly present with ventricular fibrillation after impact to the chest during ventricular repolarization (T-wave). Commotio cordis is ventricular fibrillation precipitated by blunt trauma to the heart, not attributable to structural damage to the heart or surrounding structures. A history consistent with commotio cordis involves a sudden impact with the anterior chest overlying the heart, followed by immediate cardiac arrest. This is most commonly a baseball; however, any impact may be present in the appropriate circumstances. Ventricular fibrillation may be observed if monitoring or an AED is available. Patients generally have no history of structural heart disease to explain the dysrhythmia, and the injury is not attributable to physical damage to the heart, cardiac contusion, or rupture. Penetrating injury is not the cause for arrest. Ventricular tachycardia is less likely than ventricular fibrillation in a structurally normal heart with commotio cordis. Ventricular fibrillation is most likely the initial rhythm, although it may degenerate into asystole over time if perfusion is not maintained. The patient received immediate bystander CPR, so hopefully, his rhythm will still be shockable ventricular fibrillation when EMS arrives. Ventricular fibrillation is most likely the initial rhythm, although it may degenerate into PEA over time if perfusion is not maintained Go to the next page if you knew the correct answer, or click the link image(s) below to further research the concepts in this question (if desired).

Research Concepts: Commotio Cordis

We update eBooks quarterly and Apps daily based on user feedback. Please tap flag to report any questions that need improvement.

Question 224: A 72-year-old female presents with fever, cough, and moderate dyspnea since last five days. She is tachypneic with a respiratory rate of 34/min. Chest radiographs reveal lobar pulmonary infiltrates. Hospital admission can be decided based on the CURB-65 score. What score indicates the need for intensive care unit admission?

Choices: 1. Greater than 2 2. Greater than 3 3. Greater than 4 4. Greater than 5

Answer: 3 - Greater than 4 Explanations: CURB-65 and pneumonia severity index help stratify patients and determine if a patient needs hospitalization for treatment of pneumonia. CURB-65 Score of greater than or equal to 2 need hospitalization. A CURB-65 score greater than 4 indicates the need for intensive care unit admission and more intense therapy. CURB-65 assigns one point for each factor including confusion, uremia (BUN greater than 20 mg/dL), respiratory rate greater than 30/min, hypotension (systolic less than 90 mmHg and diastolic less than 60 mmHg), and, age more than 65 years. Go to the next page if you knew the correct answer, or click the link image(s) below to further research the concepts in this question (if desired).

Research Concepts: Typical Bacterial Pneumonia

We update eBooks quarterly and Apps daily based on user feedback. Please tap flag to report any questions that need improvement.

Question 225: A 52-year-old male patient was brought in 2 hours after an intentional overdose with 20 diltiazem pills. He was intubated on arrival due to poor airway protection. He has already received 3 liters of intravenous crystalloids and intravenous calcium chloride. High dose insulin infusion has just been started. Blood pressure continues to remain low with a MAP of 45 mmHg. Heart rate is 55/minute with sinus bradycardia and slightly prolonged QRS. A monitor was attached to a radial arterial line and shows the following parameters: stroke volume variability = 8%, cardiac output = 3.0 L/min, cardiac index= 2.1 L/min, systemic vascular resistance = 90 dynes/sec/cm5. What will be the most appropriate intervention for his hypotension?

Choices: 1. Start norepinephrine infusion 2. Start dobutamine infusion 3. Increase the dose of insulin infusion 4. Administer 1 liter of crystalloid bolus

Answer: 2 - Start dobutamine infusion Explanations: From the available hemodynamic parameters, his stroke volume variability (desired less than 12%) and systemic vascular resistance (800 to1200 dynes/sec/cm5 ) are both within normal limits. Neither intravenous fluids nor vasopressors will be as useful. It is important to identify the etiology of shock in a setting of severe calcium channel blocker toxicity. If reduced inotropic action is the root cause (reduced cardiac output and index) initiation of an inotrope is the next best intervention. Norepinephrine does have some inotropic action but dobutamine will be more potent in this setting. In refractory shock despite vasopressors and inotropes, intra-aortic balloon pump may need to be considered. (IABP). Go to the next page if you knew the correct answer, or click the link image(s) below to further research the concepts in this question (if desired).

Research Concepts: Calcium Channel Blocker Toxicity

We update eBooks quarterly and Apps daily based on user feedback. Please tap flag to report any questions that need improvement.

Question 226: A 17-year old is hit in the neck area during a football game. He is immediately brought to the emergency department because of shortness of breath. A lateral chest x-ray of the neck reveals subcutaneous emphysema. The hyoid bone appears to be high in the neck. What is the diagnosis?

Choices: 1. Lacerated thyroid gland 2. Transection of the cervical trachea 3. Ruptured thyroglossal duct cyst 4. Tracheoesophageal fistula

Answer: 2 - Transection of the cervical trachea Explanations: X-ray findings are important in patients with a tracheobronchial injury. If there is subcutaneous emphysema and the hyoid bone is elevated, one must suspect transection of the cervical trachea. Other features on an x-ray of a tracheal injury include tracheal deviation or deformity. Up to 20% of people with tracheobronchial injury may have no signs on the x-ray. Go to the next page if you knew the correct answer, or click the link image(s) below to further research the concepts in this question (if desired).

Research Concepts: Tracheal Trauma

We update eBooks quarterly and Apps daily based on user feedback. Please tap flag to report any questions that need improvement.

Question 227: A 17-year-old presented to the emergency room with chest pain, dyspnea, and fever. He is an intravenous drug user. His vitals are temperature: 103 F, pulse: 109 beats/minute, blood pressure: 86/56 mm Hg, and respiratory rate of 22 breaths/minute. Auscultation reveals findings suggestive of tricuspid regurgitation. Echocardiography supported the clinical findings. Blood culture samples were sent, and the patient is started on broadspectrum antibiotics. On day 7, the sample grew a fungus. Which of the following feature would suggest the fungus could be Aspergillus?

Choices: 1. Cutaneous macronodules 2. Black hemorrhagic skin lesions 3. Gram stain shows gram-positive cocci, arranged in grape-like clusters 4. Germ tube visible in high power microscopy of a centrifuged blood sample

Answer: 2 - Black hemorrhagic skin lesions Explanations: There are certain clinical features unique to certain fungal infections. Cutaneous macronodules suggest Candida infection while black hemorrhagic skin lesions suggest Aspergillus infection. These findings are unique but rare. The right-sided heart valve endocarditis is common in the intravenous drug users. Draw blood culture samples before starting broad-spectrum antibiotics. The patient of infective endocarditis often needs fluid resuscitation so initial stabilization of the patient is a key to the management. Go to the next page if you knew the correct answer, or click the link image(s) below to further research the concepts in this question (if desired).

Research Concepts: Fungal Endocarditis

We update eBooks quarterly and Apps daily based on user feedback. Please tap flag to report any questions that need improvement.

Question 228: A 17-year-old female presents to the emergency department with two days of vomiting, abdominal pain, and bloody diarrhea. In questioning him, she notes that she recently returned from a trip to the Louisiana coast where she visited a friend and says they ate a large quantity of seafood, including some raw oysters. Which of the following is the most likely infectious agent causing her current issue?

Choices: 1. Entamoeba histolytica 2. Campylobacter jejuni 3. Vibrio parahaemolyticus 4. Schistosomiasis

Answer: 3 - Vibrio parahaemolyticus Explanations: Infectious causes of bloody diarrhea include Campylobacter, Enterohemorrhagic strains of Escherichia coli, Salmonella, Shigella, Yersinia, Clostridium difficile, Aeromonas, Vibrio, Entamoeba, and Schistosomiasis. Vibrio parahaemolyticus is associated with raw or undercooked seafood and most often oysters. Seafood infected is most commonly found in warm coastal waters. Potential symptoms include nausea, vomiting, watery or bloody diarrhea, abdominal pain or cramping, and fever. Onset is usually within 24 hours of ingestion. The illness can last up to 7 days and is undercooked self-limiting. Go to the next page if you knew the correct answer, or click the link image(s) below to further research the concepts in this question (if desired).

Research Concepts: Gastrointestinal Bleeding

We update eBooks quarterly and Apps daily based on user feedback. Please tap flag to report any questions that need improvement.

Question 229: An adult patient is febrile, has a pulse of 120 beats/min, and a blood pressure of 65/40 mmHg. The patient has been given 4 liters of crystalloids with minimal response. What is the best next treatment for this patient?

Choices: 1. Norepinephrine 2. Dopamine 3. Digoxin 4. Vasopressin

Answer: 1 - Norepinephrine Explanations: Norepinephrine is more effective than dopamine in treating hypotension associated with sepsis. Urine output is greater and lactic acid levels are lower. There also is better perfusion of the splanchnic tissues. Dopamine can cause tachycardia and arrhythmias. Dopamine is used only in highly specific situations. Care should be given in identifying the right sympathomimetic agent by assessing perfusion before initiation. In early septic shock, the patient may be in a warm, vasodilatory shock state wherein norepinephrine is appropriate; however, patients who present in later stages of shock may be in a cool, vasoconstrictive shock state wherein epinephrine is appropriate. Most of the time, in fluid-refractory shock states, such as severe sepsis or septic shock, a combination of norepinephrine and epinephrine is used. Individuals with sepsis, who do not respond to early resuscitative fluid therapy in the emergency department, show evidence of end-organ hypoperfusion and hemodynamic monitoring and should be admitted to an intensive care unit. Go to the next page if you knew the correct answer, or click the link image(s) below to further research the concepts in this question (if desired).

Research Concepts: Septic Shock

We update eBooks quarterly and Apps daily based on user feedback. Please tap flag to report any questions that need improvement.

Question 230: A 66-year old male with emphysema had a sigmoid resection for cancer ten days ago. He now returns with complaints of mild shortness of breath which started a few days ago. He states that he is still not able to tolerate a solid diet and is only drinking liquids. He did have bouts of nausea and vomiting for the first three days, but now these symptoms subsided after he started to pass flatus. Because of the incisional pain, he has limited himself to the home. Vitals reveals that he is afebrile and his blood pressure is 90/45 mmHg, heart rate 90 bpm and his room air pulse oximetry is 87%. His electrocardiogram shows non-specific ST changes. Which of the following tests should be ordered immediately?

Choices: 1. Chest x-ray 2. Arterial blood gas 3. Cardiac enzymes 4. CT of the chest

Answer: 4 - CT of the chest Explanations: Specific historical features that assist in the diagnosis of a deep venous thrombosis (DVT) are those related to DVT risk factors and include a history of cancer, exogenous estrogen therapy, recent surgery, smoking tobacco, previous history of DVT, immobility, age, history of a hypercoagulable state, and other comorbidities. Patients often will present with a chief complaint of unilateral leg swelling and discomfort. Be mindful to ask about symptoms related to a pulmonary embolism (PE) as well, such as chest pain, shortness of breath, and syncope. Rapid diagnosis or rule-out by the emergency provider can expedite necessary treatment, reduce the length of stay, and is particularly useful where access to 24-hour ultrasound is unavailable. CT of the chest should be done to determine if the patient has had a PE. Go to the next page if you knew the correct answer, or click the link image(s) below to further research the concepts in this question (if desired).

Research Concepts: Deep Vein Thrombosis

We update eBooks quarterly and Apps daily based on user feedback. Please tap flag to report any questions that need improvement.

Question 231: A 17-year-old male presents to the emergency department via emergency medical services for altered mental status after intravenous use of phencyclidine. The patient is extremely agitated requiring physical restraint. He has a heart rate of 145 bpm, blood pressure 170/90 mmHg, respiratory rate of 22/minute, and oxygen saturation of 90% on room air. Fingerstick blood glucose is 102 mg/dL. He is warm to the touch, diaphoretic, and has rotary nystagmus. The patient then begins to have a generalized seizure. The patient has no IV access. What is the next most appropriate step?

Choices: 1. Administer 1000 g of IM fosphenytoin 2. Administer 2 to 4 mg of IM lorazepam 3. Administer 10 mg of IM midazolam 4. Place the patient in a quiet dark room

Answer: 3 - Administer 10 mg of IM midazolam Explanations: Intravenous benzodiazepines are the drug of choice to treat phencyclidine-induced seizures. Lorazepam 2 to 4 mg IM or IV, or diazepam 5 to 10 mg IM or IV can be given. Both of these medications take 15 to 20 minutes for onset when given IM. Therefore, IM midazolam is preferred when there is no IV access. Seizures, rhabdomyolysis, coma, hypertensive crisis, myocardial infarction, and intraparenchymal hemorrhage are all potential complications of phencyclidine use. Phencyclidine is available as a powder, crystal liquid, or tablet. It can be ingested orally, injected intravenously, inhaled, or smoked. Intravenous phencyclidine use can produce symptoms in 2 to 5 minutes. Go to the next page if you knew the correct answer, or click the link image(s) below to further research the concepts in this question (if desired).

Research Concepts: Phencyclidine Toxicity

We update eBooks quarterly and Apps daily based on user feedback. Please tap flag to report any questions that need improvement.

Question 232: A 45-year-old man presents for a six-month history of progressive dyspnea and nonproductive cough. He has smoked one pack per day of cigarettes for 20 years, and he works as a plumber. Current medications include antacids and acetaminophen as needed. His oxygen saturation is 92% on room air. Bibasilar crackles without wheezing is noted on auscultation. The rest of the exam is within normal limits. Pulmonary function tests show the following: forced vital capacity (FVC) 62% of predicted, forced expiratory volume at 1 second (FEV1) 59% of predicted, total lung capacity 70% of predicted, vital capacity 50% of predicted, residual volume 70% of predicted, and diffusion lung capacity (DLCO) 49% of predicted. Highresolution CT (HRCT) shows diffuse ground-glass disease bilaterally with no enlarged mediastinal or hilar lymphadenopathy. What is the best next step in the management of this patient?

Choices: 1. HRCT findings should be sufficient to confirm the diagnosis 2. Bronchoscopy 3. Surgical lung biopsy 4. Initiate antifibrotic therapy

Answer: 3 - Surgical lung biopsy Explanations: While high-resolution CT findings are helpful in the initial diagnostic workup, it is often nonspecific when classic radiological findings of idiopathic pulmonary fibrosis (IPF) are lacking. Classic findings include honeycombing with or without peripheral traction or bronchiectasis and subpleural basal predominant with heterogeneous distribution. Bronchoscopy is used to rule out infectious etiology; however, it is not recommended to confirm the diagnosis due to the inability to obtain adequate lung samples. Transbronchial biopsy specimens are considered inadequate to diagnose desquamative interstitial pneumonia (DIP) or to distinguish it clearly from overlapping disease patterns like respiratory bronchiolitis-associated interstitial lung disease (RB-ILD) in early stages and nonspecific interstitial pneumonitis (NSIP) in late stages. Surgical lung biopsy is considered the criterion standard in diagnosing DIP and other types of interstitial lung disease (ILD). In DIP, open lung biopsy shows infiltration of alveoli with macrophages rich in eosinophilic cytoplasm and a brown pigment called "smoker's pigment." The initiation of antifibrotics is only indicated in eligible patients with a confirmed diagnosis of idiopathic pulmonary fibrosis (IPF). Go to the next page if you knew the correct answer, or click the link image(s) below to further research the concepts in this question (if desired).

Research Concepts: Desquamative Interstitial Pneumonia

We update eBooks quarterly and Apps daily based on user feedback. Please tap flag to report any questions that need improvement.

Question 233: A 55-year-old female presents with 3 days of fever, headache, and progressive cognitive decline. She has no significant medical history. She has no sick contacts and no recent travel history. She does not use tobacco, or illicit drugs, but drinks alcohol occasionally. Temperature is 38 C, blood pressure is 126/84, and pulse is 72/min. Mucous membranes are moist with no lesions. No lymphadenopathy is present. Lungs are clear to auscultation and heart sounds are normal with no murmur. The abdomen is soft and nontender with no hepatosplenomegaly. She has nuchal rigidity and is alert and oriented only to self. She has no motor or sensory deficits but can only follow simple commands. Cerebrospinal fluid (CSF) analysis shows a white blood cell count of 150/mm3, red blood cell count of 3/mm3, protein concentration of 130 mg/dL, and glucose concentration of 40 mg/dL. CSF polymerase chain reaction (PCR) testing is positive for herpes simplex virus 1. Magnetic resonance imaging is most likely to show involvement of which part of the brain?

Choices: 1. Parietal lobe 2. Temporal lobe 3. Occipital lobe 4. Corpus callosum

Answer: 2 - Temporal lobe Explanations: Herpes simplex virus (HSV) encephalitis usually involves the temporal and frontal lobes, hence causing predominately psychiatric features, memory disturbance, and aphasia. Magnetic resonance imaging is the most sensitive and specific imaging modality for HSV encephalitis. Temporal lobe abnormalities on brain imaging are strongly suggestive of HSV encephalitis. Temporal lobe lesions are usually unilateral, but extra-temporal abnormalities may also be seen. Go to the next page if you knew the correct answer, or click the link image(s) below to further research the concepts in this question (if desired).

Research Concepts: Viral Encephalitis

We update eBooks quarterly and Apps daily based on user feedback. Please tap flag to report any questions that need improvement.

Question 234: A middle-aged male with long-standing chronic obstructive pulmonary disease (COPD) presents with dyspnea, fatigue, and copious sputum production to the emergency department. Twenty-four hours ago, he noted a low-grade fever and a runny nose. Physical exam reveals respiratory distress with the use of accessory muscles. He has marked wheezing and no air entry bilaterally. He is immediately treated with oxygen and nebulizer therapy but fails to improve. The pulmonologist feels that the patient may need a bronchoscopy for diagnosis and therapeutic reasons. During bronchoscopy, what percentage of COPD patients with exacerbations have microorganisms in the lower airways?

Choices: 1. Less than 3% 2. About 10% 3. About 50% 4. More than 90%

Answer: 3 - About 50% Explanations: Bronchoscopy is sometimes performed to sample the airways for the bacteria in patients who are not responding to treatment. Studies suggest that at least 50% of patients have microorganisms in the lower airway during exacerbation of COPD. B. R. Celli, P. J. Barnes. Exacerbations of chronic obstructive pulmonary disease European Respiratory Journal Jun 2007, 29 (6) 1224-1238; DOI: 10.1183/09031936.00109906 https://erj.ersjournals.com/content/29/6/1224 At the same time, a significant number of patients also have colonization of the airways, making it difficult to interpret the data. However, the one feature is that the bacterial burden increases in the airways during an exacerbation. New organisms are most likely in patients with symptoms of wet cough and purulent sputum. Go to the next page if you knew the correct answer, or click the link image(s) below to further research the concepts in this question (if desired).

Research Concepts: Chronic Obstructive Pulmonary Disease

We update eBooks quarterly and Apps daily based on user feedback. Please tap flag to report any questions that need improvement.

Question 235: A patient with head and abdominal trauma was admitted to the surgical intensive care unit and eventually was started on total parenteral nutrition (TPN). After three weeks, the area around the central line became red and indurated. The patient had no fever, and the white blood cell count was normal. Cultures and the Gram stain from the site reveal Grampositive bacterium. Based on this information, the clinician has ordered the removal of the line, and TPN stopped. The tip was sent for culture. Four hours later, the patient is very lethargic and obtunded. Which of the following is the next best step in the management of this patient?

Choices: 1. Obtain blood cultures and start antibiotics 2. Give glucagon intravenously 3. Give glucose intravenously 4. Obtain a CT scan of the head

Answer: 3 - Give glucose intravenously Explanations: Complications of total parenteral nutrition (TPN) vary, but the most common complication is glucose abnormalities. To avoid hyperglycemia, one should monitor plasma glucose and adjust insulin levels in the TPN solution as needed. Alternately, the sudden discontinuation of the dextrose solution can precipitate sudden hypoglycemia. Abrupt removal of a continuous dextrous source causes the circulating insulin to induce hypoglycemia. Management of TPN therapy includes monitoring of access site and changing of solution and tubing per protocol every 24 hours. The nurse should record daily weights and accurate intake and output in the patient's chart. The dietitian collaborates with members of the interdisciplinary team, using a calculation based on the Harris-Benedict equation to start feedings based on individualized nutrient needs. One orders TPN therapy for patients whose gastrointestinal tract is non-functional because of either traumatic injury or medical conditions. Central venous access is best for TPN to allow for higher concentrations of glucose. Go to the next page if you knew the correct answer, or click the link image(s) below to further research the concepts in this question (if desired).

Research Concepts: Total Parenteral Nutrition

We update eBooks quarterly and Apps daily based on user feedback. Please tap flag to report any questions that need improvement.

Question 236: A patient presents to the emergency department with witnessed prehospital cardiac arrest. The initial rhythm was ventricular fibrillation. After multiple defibrillation attempts, high-quality compressions, and medication administration return of spontaneous circulation (ROSC) has occurred. Hemodynamic stability is present, the comatose state continues, and the core temperature is 37.0 degrees C. Definitive airway established and suitable placement confirmed. What is the following appropriate treatment modality to consider in this patient?

Choices: 1. Continue high quality compressions 2. Targeted Temperature Management (Therapeutic hypothermia) 3. Needle decompression 4. Aggressive rewarming techniques

Answer: 2 - Targeted Temperature Management (Therapeutic hypothermia) Explanations: Targeted temperature management is recommended by the American Heart Association as long as the patient is still unresponsive. The purpose of targeted temperature management is to reduce cerebral metabolism and improve oxygen stores, as well as reversing ischemic depolarization of CNS to stabilize cell membranes. Monitor closely for complications such as hypotension, potassium abnormalities, hyperglycemia, and ongoing ischemia. Temperature is monitored by more than one probe. Bladder probes are not accurate if there is no urine output. Esophageal, rectal probes or pulmonary artery temperature probes may be better options for primary and secondary monitoring. Shivering should be avoided because it raises the body temperature. Transfer these patients to a critical care bed quickly. Targeted temperature management, formerly known as therapeutic hypothermia, has been proven to be of benefit in adults with witnessed, documented, prehospital ventricular fibrillation arrest with the deliberate reduction in body temperature to 32 degrees C to 36 degrees C. This hypothermic state has been proven quite beneficial in maintaining and restoring normal neurological function and is huge in the advancement of medicine. Many clinical trials are underway for its use in other arrest situations and the pediatric population. It is showing promise and of benefit in neonatal hypoxic-ischemic encephalopathy or asphyxiated newborns. More data is needed before cooling becomes the standard of care in children. A practitioner may decide to initiate cooling in patients with return of spontaneous circulation after pulseless electrical activity, asystole, or even an in-house cardiac arrest but the cause of the arrest will be a consideration. Sepsis patients are not good candidates for cooling due to compromised immune systems already, and hypothermia will further complicate the condition. The American Heart Association recommends cooling in adults for at least 24 hours post prehospital ventricular fibrillation arrest with a return of spontaneous circulation (ROSC) that remain unconscious. If hypothermia exists already, > 33 degrees C and hemodynamically stable after ROSC, do not actively rewarm. Cool as soon as possible by starting in the emergency department and maintain neuroprotective treatment such as elevating the head of the bed 30 degrees and seizure precautions. Hypothermia is contraindicated in patients with sepsis, impaired clotting systems, recent major surgery in the past 14 days, ingestion drug coma, or who have active bleeding, as in trauma patients. A targeted temperature goal usually is reached by the 3-4 hour mark after cooling initiated. Rewarming will start around 24 hours later. Do not delay cooling after ROSC. Cooling early preserves brain cells. Keep MAP over 80. Osbourn waves may be present with cooling. Bradycardia with a heart rate of less than 40 is common and does not need treatment unless an unstable patient. Rewarm slowly at a rate of 0.3-0.5 degrees C every hour. This process could last around 8 hours or more. Paralytics originally started to control shivering during cooling, should be discontinued for the start of rewarming. Go to the next page if you knew the correct answer, or click the link image(s) below to further research the concepts in this question (if desired).

Research Concepts: Targeted Hypothermia Temperature Management

We update eBooks quarterly and Apps daily based on user feedback. Please tap flag to report any questions that need improvement.

Question 237: A 45-year-old trauma patient with no comorbidities is diagnosed with blunt cardiac injury (BCI) due to abnormalities on ECG and an elevated cardiac troponin I level. However, the patient is stable and can be admitted to the floor under telemetry for continuous cardiac monitoring. What further studies should be ordered?

Choices: 1. ECG with cardiac troponin I level every 6 hours and an echocardiogram 2. ECG with cardiac troponin I level every 6 hours 3. ECG with cardiac troponin I level every 6 hours, an echocardiogram, and a cardiology consult 4. Cardiac troponin I level every 6 hours and an echocardiogram

Answer: 1 - ECG with cardiac troponin I level every 6 hours and an echocardiogram Explanations: Abnormal ECG findings warrant admission for continuous cardiac monitoring for 24 to 48 hours. Following an abnormal ECG and cTnI level, echocardiography is usually obtained to further characterize the injury. If there are wall motion abnormalities, cardiology should be consulted. Patients with BCI can be admitted to the intensive care unit or under telemetry depending on concurrent injuries, type of ECG change, and the grade of hemodynamic imbalance. Patients with a normal ECG in conjunction with normal levels of cardiac troponin I can be discharged home safely if there are no other clinical concerns. Go to the next page if you knew the correct answer, or click the link image(s) below to further research the concepts in this question (if desired).

Research Concepts: Blunt Cardiac Injury

We update eBooks quarterly and Apps daily based on user feedback. Please tap flag to report any questions that need improvement.

Question 238: A 65-year-old male patient is hospitalized for acute alcohol withdrawal. During the hospitalization, he becomes increasingly confused, combative, and has hallucinations of bugs crawling on him. He is tachycardic, hypertensive, febrile, and has a whole body tremor. What is the next best step in the management of this patient?

Choices: 1. IV antipsychotic 2. Immediate brain CT scan 3. IV vasodilator 4. IV benzodiazepine

Answer: 4 - IV benzodiazepine Explanations: Delirium tremens (DTs) is a severe form of alcohol withdrawal that can be life-threatening. DTs occur between 10 and 72 hours after the last drink and are associated with fever, nausea, hallucinations, agitation, hypertension, tachycardia, diaphoresis, and tremor. Benzodiazepines are the drug of choice in managing all degrees of alcohol withdrawal and are used for prophylaxis. Any patient suspected of alcohol use disorder should be placed on thiamine to prevent Wernicke encephalopathy. Go to the next page if you knew the correct answer, or click the link image(s) below to further research the concepts in this question (if desired).

Research Concepts: Delirium Tremens

We update eBooks quarterly and Apps daily based on user feedback. Please tap flag to report any questions that need improvement.

Question 239: A patient was admitted to the ICU with an acute ischemic stroke involving the right middle cerebral artery. She did not receive embolectomy. After 12 hours in the ICU, she began to demonstrate erratic respirations, tachycardic arrhythmias, with decreased responsiveness, followed by what appeared to be tonic-clonic activity. Assuming no hemorrhage was present on CT of the head, what strategy can be used to rapidly stabilize, excluding surgical interventions, this severely symptomatic patient?

Choices: 1. Furosemide drip 2. Dexamethasone 4 mg IVP every 8 hours 3. Blood pressure reduction with nitroprusside drip 4. Short term hyperventilation via mechanical ventilation

Answer: 4 - Short term hyperventilation via mechanical ventilation Explanations: Hyperventilation of an intubated patient to a goal PCO2 of 26 to 30 mmHg can reduce intracranial blood flow, and thereby pressures, by 3% for every 1 mmHg drop in PCO2. Hyperventilation strategies should not be continued beyond the first 24 hours to prevent the risk of vasoconstriction and worsened brain ischemia. Hyperventilation strategies should be used only in patients with intracranial pressure (ICP) monitoring. Care must be taken with patients on mechanical ventilation to ensure adequate blood pressure is maintained in order to maintain adequate cerebral perfusion pressure. Vasopressors may be used if necessary, as these are not shown to increase ICP as they increase blood pressure if blood pressure drops as a result of sedation used for the ventilated patient. Go to the next page if you knew the correct answer, or click the link image(s) below to further research the concepts in this question (if desired).

Research Concepts: Cerebral Edema

We update eBooks quarterly and Apps daily based on user feedback. Please tap flag to report any questions that need improvement.

Question 240: A patient who suffered a multiorgan injury is intubated in the intensive care unit. His ventilator settings rate is 14, FiO2 60%, total volume 450 ml, and positive endexpiratory pressure of 12. At night, the nurse notices a significant amount of subcutaneous emphysema and elevated peak pressures. What should be the next step in the management of this patient?

Choices: 1. Blood gas 2. CT scan 3. Place chest tubes 4. Bronchoscopy

Answer: 3 - Place chest tubes Explanations: In any ventilated patient with a high positive end-expiratory pressure (PEEP), barotrauma can often present with a pneumothorax. Sometimes it may present as subcutaneous emphysema. Many patients on the ventilator with high PEEP have prophylactic chest tubes inserted. PEEP can also cause a decline in cardiac output. In such a scenario the placement of chest tubes can be life saving, as hemodynamic compromise can occur anytime. No time should be wasted obtaining chest x-rays. In rare cases, the endotracheal tube can cause a posterior tear in the trachea and cause subcutaneous emphysema. Go to the next page if you knew the correct answer, or click the link image(s) below to further research the concepts in this question (if desired).

Research Concepts: Iatrogenic Pneumothorax

We update eBooks quarterly and Apps daily based on user feedback. Please tap flag to report any questions that need improvement.

Question 241: An 81-year-old man presents to the emergency department with four days of epigastric abdominal pain and non-bloody vomiting. He is afebrile but has a blood pressure of 90/50 mmHg and a heart rate of 130 bpm. On examination, he has a tender upper abdomen and bilateral ecchymosis of the flanks. Which of the following is the most likely diagnosis in this setting?

Choices: 1. Acute inferior myocardial infarction 2. Acute ascending thoracic aortic dissection 3. Acute necrotizing pancreatitis 4. Upper gastrointestinal bleeding

Answer: 3 - Acute necrotizing pancreatitis Explanations: Although it is an uncommon finding the appearance of ecchymotic discoloration on the flanks points to intraabdominal or retroperitoneal bleeding, classically due to severe acute necrotizing pancreatitis. The exam finding is referred to as the Grey Turner sign named after a British surgeon who published the finding in 1920. In a few patients with severe necrotizing pancreatitis, the liberated enzymes destroy peripancreatic tissues causing bleeding that tracks through various tissue planes to reach the subcutaneous tissues causing the discoloration on the flanks. The vast majority of patients with pancreatitis do not show subcutaneous signs of retroperitoneal bleeding. Thus, clinicians cannot use the absence of a Grey Turner sign to rule out severe pancreatitis. Almost any process that causes intraabdominal or retroperitoneal bleeding may result in blood tracking to the subcutaneous tissues resulting in the Grey Turner sign. Therefore, it is not specific to pancreatitis. Go to the next page if you knew the correct answer, or click the link image(s) below to further research the concepts in this question (if desired).

Research Concepts: Grey-Turner Sign

We update eBooks quarterly and Apps daily based on user feedback. Please tap flag to report any questions that need improvement.

Question 242: A 70-year-old male with a past medical history of hypertension and hyperlipidemia is brought to the emergency department by for complaints of left-sided weakness and numbness. He states that he passed out after having some beers and cigarettes and woke up two hours later with left-sided weakness. A CT scan of the head without contrast shows no evidence of hemorrhage. The patient is given IV TPA. His initial lab work was positive for an initial troponin of 2.96 ng/ml, and his second troponin is 3.55 ng/ml. His ECG shows a 0.5mm Twave inversion in lead V2 and nonspecific ST-T wave changes in other precordial leads. What is the most likely causes of the elevated troponins and what medication(s) should be started next?

Choices: 1. The patient has a stroke; start aspirin and statin now 2. The patient has a stroke; start aspirin, clopidogrel and statin now 3. The patient has a stroke; start aspirin and clopidogrel at least 24 hours later and start statin now 4. The patient has a non-ST elevation myocardial infarction; start heparin infusion and statin now

Answer: 3 - The patient has a stroke; start aspirin and clopidogrel at least 24 hours later and start statin now

Explanations: The elevated troponins are likely due to the patient’s stroke. Starting aspirin alone right away would not be appropriate. Instead, dual antiplatelet therapy with aspirin and clopidogrel should be held for at least 24 hours after giving TPA and then started. Dual antiplatelet therapy with aspirin and clopidogrel for 21 days has been proven superior to aspirin alone and should be given with a statin for secondary stroke prevention, but antiplatelet agents should be held for at least 24 hours after giving TPA. As the troponin elevation is due to stroke and the patient has no noted history of bleeding, dual antiplatelet therapy with aspirin and clopidogrel should be started and continued for 21 days for secondary stroke prevention followed by continuation on aspirin and careful outpatient follow up care. No contraindication to starting a statin right away is present. The elevation in troponins, in this case, is not an indicator of non-ST elevation myocardial infarction but is instead more likely a consequence of the patient’s stroke, which is part of the differential for medical conditions associated with positive troponins. As a result, dual antiplatelet therapy would be the more appropriate choice. Go to the next page if you knew the correct answer, or click the link image(s) below to further research the concepts in this question (if desired).

Research Concepts: Cardiac Enzymes

We update eBooks quarterly and Apps daily based on user feedback. Please tap flag to report any questions that need improvement.

Question 243: A 40-year-old woman with a history of seizure disorder, bipolar disorder type 1, and newly diagnosed atrial fibrillation presents to the emergency department for evaluation of altered mental status. The patient has a list of medications that was updated this week after starting a new medication. She has taken carbamazepine for a seizure disorder for ten years with no recent dosage changes. Vital signs are heart rate 60/bpm, blood pressure 105/55 mmHg, respiratory rate 10/minute, and oxygen saturation 96% on room air. The patient is somnolent but arousable. She is oriented to self only. Her physical exam is otherwise unremarkable. CBC, basic metabolic profile, urinalysis, and CT of the head without contrast are within normal limits. Carbamazepine level is 45 mcg/ml. What medication has likely contributed to the patient's current condition?

Choices: 1. Diltiazem 2. Lithium 3. Metoprolol 4. Warfarin

Answer: 1 - Diltiazem Explanations: The toxic concentration of carbamazepine is 40 mcg/ml. The risk of seizure increases significantly at level greater than 40 mcg/ml. Carbamazepine (CBZ) is metabolized by the cytochrome P450 enzyme CYP3A4. Many medications, including allopurinol, diltiazem, fluoxetine, isoniazid, and valproic acid, inhibit the activity of the CYP3A4 enzyme, which can lead to elevated serum concentration of CBZ. A serum CBZ level and current medication list should be obtained when evaluating mental status changes in a patient treated with CBZ. Carbamazepine is also a potent CYP 3A4 inducer and can cause decreased serum levels of other anticonvulsants such as valproic acid and phenytoin) and can induce seizures secondary to decreased serum levels of these medications. Dialysis is the definitive treatment for CBZ toxicity in the setting of high serum concentrations and significant central nervous system depression. Go to the next page if you knew the correct answer, or click the link image(s) below to further research the concepts in this question (if desired).

Research Concepts: Anticonvulsants Toxicity

We update eBooks quarterly and Apps daily based on user feedback. Please tap flag to report any questions that need improvement.

Question 244: A 17-year-old female with no significant past medical history was hospitalized after a gunshot wound to the abdomen. She had a complicated postoperative course requiring multiple re-interventions for anastomotic leakage. She is receiving total parenteral nutrition via a central venous catheter and is on broad-spectrum antibiotics for the bowel perforation. After 8 days in the intensive care unit, the patient became febrile and hypotensive. Urinalysis and chest x-ray show no clear focus of infection. Blood cultures are pending. Beta-dglucan is positive for candida of unknown species. What is the best initial therapy?

Choices: 1. Fluconazole 2. Amphotericin B 3. Micafungin 4. No additional medication, wait for the blood cultures

Answer: 3 - Micafungin Explanations: An echinocandin is the first-line treatment for candidemia. Echinocandin resistance is rare. Fluconazole is an alternative first-line treatment in non-critically ill patients or if the cultures show azole-sensitive candida species. Amphotericin B also is a therapeutic option in patients who are neutropenic or unable to tolerate echinocandins or azoles. The central venous catheter should be removed as early as possible if it is the presumed source of candidemia. Go to the next page if you knew the correct answer, or click the link image(s) below to further research the concepts in this question (if desired).

Research Concepts: Fungemia Candidiasis

We update eBooks quarterly and Apps daily based on user feedback. Please tap flag to report any questions that need improvement.

Question 245: A 32-year-old male is admitted to the intensive care unit for progressive change in mental status, delirium, and rapidly progressive cognitive impairment. His medical history is significant for advanced HIV disease. MRI of the brain shows evidence of periventricular inflammation and meningeal enhancement. What is the recommended treatment?

Choices: 1. Acyclovir 10 mg/kg IV every 8 hours for 14 to 21 days 2. Acyclovir 10 to 15 mg/kg IV every 8 hours for 10 to 14 days 3. Ganciclovir 5 mg/kg IV every 12 hours 4. Ganciclovir 5 mg/kg IV every 12 hours and foscarnet 60 mg/kg IV every 8 hours for 21 days

Answer: 4 - Ganciclovir 5 mg/kg IV every 12 hours and foscarnet 60 mg/kg IV every 8 hours for 21 days

Explanations: Immunocompromised individuals, such as those with advanced HIV disease, are at increased risk of developing cytomegalovirus (CMV) encephalitis. Neuroimaging of patients with CMV encephalitis can show periventricular inflammation and meningeal enhancement. Combination therapy of ganciclovir and foscarnet is shown to be more effective than ganciclovir alone. Recommended therapy for CMV encephalitis is ganciclovir 5 mg/kg IV every 12 hours and foscarnet 60 mg/kg IV every 8 hours for 21 days or ganciclovir 5 mg/kg IV every 12 hours and foscarnet 90 mg/kg IV every 12 hours for 21 days. Go to the next page if you knew the correct answer, or click the link image(s) below to further research the concepts in this question (if desired).

Research Concepts: Viral Encephalitis

We update eBooks quarterly and Apps daily based on user feedback. Please tap flag to report any questions that need improvement.

Question 246: Transesophageal echocardiography reveals an unruptured sinus of valsalva aneurysm in a patient evaluated for widened mediastinum on chest x-ray. What is the next best step in management?

Choices: 1. Chest MRI for confirmation of the aneurysm, and, if confirmed, immediate surgical consultation 2. Follow up echocardiogram in 6 weeks 3. Holter monitor 4. Referral for outpatient cardiothoracic surgery consultation

Answer: 1 - Chest MRI for confirmation of the aneurysm, and, if confirmed, immediate surgical consultation

Explanations: Sinus of valsalva aneurysms can cause compression of adjacent coronary arteries resulting in myocardial ischemia and infarction. Aneurysms should be confirmed with cardiac MRI. Urgent surgical intervention is generally warranted if the aneurysm is confirmed. Patients will undergo coronary angiography before cardiac surgery to assess coronary anatomy. Patients with low risk of coronary disease may have an evaluation of coronary anatomy using cardiac computed tomography, and may not need to undergo cardiac catheterization. Patients with intermediate or high risk for coronary artery disease will usually undergo cardiac catheterization for assessment of possible bypass grafting at the time of cardiac surgery. Go to the next page if you knew the correct answer, or click the link image(s) below to further research the concepts in this question (if desired).

Research Concepts: Sinus Of Valsalva Aneurysm

We update eBooks quarterly and Apps daily based on user feedback. Please tap flag to report any questions that need improvement.

Question 247: A 78-year-old female is admitted to the intensive care unit after a hemicolectomy for a perforated diverticulum. She has a temperature 39.5C, blood pressure of 75/45 mmHg, and pulmonary arterial wedge pressure of 22 mmHg after 2 liters of normal saline. Which of the following would be the most appropriate treatment for this patient?

Choices: 1. Expand intravascular volume with pentastarch 2. Begin dobutamine drip 3. Begin IV norepinephrine drip 4. Start antibiotics and obtain CT abdomen and pelvis

Answer: 3 - Begin IV norepinephrine drip Explanations: Colloids are not the recommended fluids for a hypotensive patient. The patient does not appear to be intravascularly depleted. She appears to be in septic shock. Dobutamine is not the recommended vasopressor for septic shock. Norepinephrine is the recommended vasopressor in septic shock. The patient needs further evaluation with an EKG and a transesophageal echocardiogram. The patient is unstable and requires vasopressors. Although antibiotics are also needed, the patient should be stabilized first. Go to the next page if you knew the correct answer, or click the link image(s) below to further research the concepts in this question (if desired).

Research Concepts: Inotropes And Vasopressors

We update eBooks quarterly and Apps daily based on user feedback. Please tap flag to report any questions that need improvement.

Question 248: A 72-year-old female with a BMI of 22 is ready for extubation as she has passed a spontaneous breathing trial as per the intensive care unit ventilator liberation protocol. Patient has been intubated for 11 days due to acute respiratory distress syndrome related to community-acquired pneumonia. Review of the record showed that intubation was difficult and multiple attempts had to be made to successfully intubate her. The patient is placed patient back on volume control with a tidal volume of 500 and the cuff deflated. An average of the lowest 3 tidal volumes was taken and none of them were less than 390 ml. What is the next best step for the management of this patient?

Choices: 1. Go ahead and extubate as the patient had a positive cuff leak test 2. Place patient back on full support and call otolaryngology for evaluation 3. The patient will most likely need a tracheostomy placement 4. Give a stat dose of methylprednisolone IV 20 mg now and then every 4 hourly for 4 total doses before re-assessing cuff leak test

Answer: 4 - Give a stat dose of methylprednisolone IV 20 mg now and then every 4 hourly for 4 total doses before re-assessing cuff leak test

Explanations: The cuff leak test is used to predict airway laryngeal edema and post-extubation stridor. It is not recommended in every patient but should be used in selected patients who are at risk. This group includes patients with difficult intubation, airway trauma, and prolonged intubation. Edema is most common in female patients with low BMI who may have a narrow airway in the first place. Cuff leak can be assessed qualitatively by listening to the leak while the cuff is deflated. It can also be done quantitatively by placing the patient on volume control ventilation and calculating the difference between inspired and expired volume while the cuff is deflated. If the difference is less than 110 ml or less than 20% of delivered volume, it suggests that cuff leak is absent. The best strategy for absent cuff leak is to initiate steroid therapy. 2 common regimen includes one dose of 40 mg methylprednisolone 4 hours before extubation or 20 mg every 4 hourly for 4 doses. If despite a course of corticosteroids, cuff leak is still absent, the patient should be extubated over an airway exchange catheter which can facilitate re-intubation or extubation should be done in the operating room with otolaryngology consultation. Go to the next page if you knew the correct answer, or click the link image(s) below to further research the concepts in this question (if desired).

Research Concepts: Extubation

We update eBooks quarterly and Apps daily based on user feedback. Please tap flag to report any questions that need improvement.

Question 249: A 16-year-old female has been transferred following a coral snake bite. Upon arrival, a dose of North American coral snake antivenin was initiated. The patient is currently on the third out of five vials but is now complaining of a red, itchy rash and severe shortness of breath. What is the next best step in management of this patient?

Choices: 1. Continue antivenom infusion until completion, he will feel better afterward 2. Continue antivenom infusion, but order oral antihistamines 3. Stop the infusion, order IM epinephrine, IV corticosteroids, IV antihistamines, IV fluids, and nebulized albuterol. 4. Stop the infusion and observe the patient

Answer: 3 - Stop the infusion, order IM epinephrine, IV corticosteroids, IV antihistamines, IV fluids, and nebulized albuterol.

Explanations: In patients exhibiting hypersensitivity reactions, infusion of antivenin should be discontinued immediately, not paused or continued to completion. In patients exhibiting hypersensitivity reactions treatment with epinephrine, steroids, antihistamines, and albuterol should be initiated at the first sign of allergic reaction without delay. North American coral snake antivenin (NACSA) is an equine-derived IgG medication that commonly causes mild acute allergic reactions and delayed serum sickness. Severe acute allergic reactions and anaphylaxis is uncommon but possible. Any delay in treatment of hypersensitivity reactions is inappropriate as they are acute allergic reactions and patients may quickly deteriorate, requiring additional respiratory or cardiovascular supportive care. Go to the next page if you knew the correct answer, or click the link image(s) below to further research the concepts in this question (if desired).

Research Concepts: Coral Snake Toxicity

We update eBooks quarterly and Apps daily based on user feedback. Please tap flag to report any questions that need improvement.

Question 250: A 35-year-old male is being evaluated for shortness of breath and chest pain. He is found to have a pulmonary embolism and started on anticoagulation. In his nine months' follow up appointment, he is found to have signs of right heart failure and pulmonary hypertension on echocardiogram. Right heart catheterization confirms pulmonary hypertension. His clot burden is unchanged on pulmonary angiography. Which of the following is the next best step in the management of this patient?

Choices: 1. Initiate a different anticoagulant 2. Surgery-pulmonary thromboendarterectomy 3. Admission to the hospital for intravenous anticoagulation such as unfractionated heparin or subcutaneous low molecular weight heparin 4. Double-lung transplantation

Answer: 2 - Surgery-pulmonary thromboendarterectomy Explanations: Surgery is the only definitive therapy for chronic thromboembolic pulmonary hypertension. The evaluation for surgery should occur early in the disease course for chronic thromboembolic pulmonary hypertension, even if the symptoms or hemodynamic abnormalities are mild because early surgery may prevent the development of an irreversible lung and heart disease. Medical therapy is not curative, and its effects are relatively modest. There are no studies that compare anticoagulation. Double-lung transplantation is an alternative surgical option for patients who are not candidates for pulmonary thromboendarterectomy. Go to the next page if you knew the correct answer, or click the link image(s) below to further research the concepts in this question (if desired).

Research Concepts: Chronic Thrombo-Embolic Pulmonary Hypertension

We update eBooks quarterly and Apps daily based on user feedback. Please tap flag to report any questions that need improvement.

Question 251: A 60-year-old patient is admitted to the intensive care unit after thoracic surgery. They are intubated with a left-sided double lumen tube. There is a concern for tube migration during routine positioning of the patient. The vital signs are stable and patients oxygen saturation decreased from 98 to 94. What's the next step in management?

Choices: 1. Send patient for CT scan 2. Extubate the patient 3. Confirm tube position with a flexible bronchoscopy at the bedside 4. Change to right-sided double lumen tube

Answer: 3 - Confirm tube position with a flexible bronchoscopy at the bedside Explanations: Tube malposition can be life-threatening due to compromise in oxygenation and ventilation. Whenever malposition is suspected, tube position should be reconfirmed with auscultation, Flexible bronchoscopy is the most important means to confirm the position of a double lumen tube (DLT). Patient with suspected endotracheal tube malposition should not be transported before confirming tube position. Transporting a critically ill patient carries risks of significant hemodynamic and respiratory compromise. Left-sided double lumen tube has a better "margin of error " compared to right-sided tube due to right upper lobe origin and shorter length of the right main bronchus. Thus right side DLT are more prone to malposition. With an unstable patient, priority is to maintain vitals and ensure oxygenation and ventilation. Treating hypoxia hypercarbia during ventilation with DLT involves confirming tube position, optimizing ventilator settings of tidal volume, respiratory rate, inspired oxygen concentration FiO2 and positive end-expiratory pressure PEEP, and the ratio of inspiratory to expiratory time; I:E ratio. Go to the next page if you knew the correct answer, or click the link image(s) below to further research the concepts in this question (if desired).

Research Concepts: Double Lumen Endobronchial Tubes

We update eBooks quarterly and Apps daily based on user feedback. Please tap flag to report any questions that need improvement.

Question 252: A 28-year-old male with no prior history is admitted to the intensive care unit for acute hypoxic respiratory failure. He has no prior medical problems but is a current smoker. He had recently started smoking a month back due to anxiety issues. His wife at the bedside denied any use of medications recently or illicit substances. She reports that he was well until about 2 days ago when he started feeling short of breath on walking, and today he could barely move without getting short of breath and gasping for air. When he arrived in the emergency department, he was found to have pulse oximetry of 65% on room air with a respiratory rate of 35/minute, so he was intubated. He is currently intubated on assist control setting of tidal volume 400, rate of 22, FiO2 of 80% and PEEP of 14. He is sedated and synchronous with the ventilator. On examination, he has bilateral crackles, and his plateau pressure on the ventilator is 28. S1 and S2 are heard, no murmurs or rubs, the abdomen is soft, nontender with normal bowel sounds, extremities are warm with good peripheral perfusion, no rashes are seen. Chest x-ray that shows bilateral opacities occupying most of the lung fields bilaterally, no lobar consolidation or pleural effusions are seen. CBC shows an elevated WBC count of 14.3 with 83% neutrophils. Electrolytes, renal function, and liver function tests are all within normal limits. A urine drug screen was done which was negative. He is started on ceftriaxone, azithromycin, pantoprazole, and enoxaparin. A bronchoalveolar lavage is done which shows RBC 176, WBC 652 with 35% eosinophils, 43% neutrophils and 22% macrophages. The gram stain is negative for organisms. What would be the best next step in management?

Choices: 1. Intravenous methylprednisolone 2. Chest CT without contrast 3. Upgrade antibiotics to vancomycin and cefepime 4. Consult thoracic surgery for a VATS guided lung biopsy

Answer: 1 - Intravenous methylprednisolone Explanations: Idiopathic acute eosinophilic pneumonia is a rare disease, predominantly occurring in young male smokers There is a temporal association with cigarette smoking. Most patients with acute eosinophilic pneumonia have a severe hypoxemic respiratory failure, with at least 70% patients needing mechanical ventilation. Intravenous corticosteroids are the mainstay of therapy with a clinical response seen as early as 12 hours, and almost always within 48 hours. Go to the next page if you knew the correct answer, or click the link image(s) below to further research the concepts in this question (if desired).

Research Concepts: Pulmonary Eosinophilia

We update eBooks quarterly and Apps daily based on user feedback. Please tap flag to report any questions that need improvement.

Question 253: Two days after abdominal surgery, a 66-year-old female patient develops left side chest pain, dyspnea, and ECG changes in the anterolateral leads. Myocardial infarction is confirmed by elevations of cardiac biomarkers. The patient's blood pressure is 85/45 mm Hg, pulse 125 bpm, respiratory rate 20/min, and she is afebrile. Pulse oximetry at room air is 94%. Which of the following treatment is not recommended?

Choices: 1. Thrombolytic therapy 2. Aspirin 3. Intra-aortic balloon pump 4. Coronary artery bypass surgery

Answer: 1 - Thrombolytic therapy Explanations: Postoperative myocardial infarction carries a grave prognosis with mortality exceeding 50%. Postoperative myocardial infarction is a well-known phenomenon, and it usually occurs within 48 hours of surgery. Postoperative myocardial infarcts tend to be silent in most cases and may be identified by ECG and blood work. If available these patients should undergo urgent coronary reperfusion with either medical therapy or coronary angioplasty. However, timely reperfusion is critical. Cardiogenic shock is a life-threatening condition that can occur secondary to massive myocardial infarction. Using a mechanical counterpulsation strategy, a device called the Intra-aortic balloon pump (IABP) could be effective in augmenting pressure and flow. Thrombolytic therapy is usually contraindicated in patients who have had recent surgery and an intervention-based procedure may be the best choice. If a fresh surgical patient is given thrombolytic drugs, there is higher mortality associated with it. Go to the next page if you knew the correct answer, or click the link image(s) below to further research the concepts in this question (if desired).

Research Concepts: Myocardial Infarction

We update eBooks quarterly and Apps daily based on user feedback. Please tap flag to report any questions that need improvement.

Question 254: A 75-year-old female with a 50-pack-year history of tobacco use presents with a 2-week history of worsening dyspnea on exertion. She has had a dry cough but no fever, night sweats, or chills. Her neck shows no jugular venous distention. Heart tones are distant but regular without murmurs. A lung exam reveals right lower lung dullness to percussion and decreased breath sounds. A chest radiograph shows a pleural effusion but no infiltrates or lymphadenopathy. Laboratories are normal. What is the appropriate management?

Choices: 1. Broad-spectrum antibiotics 2. CT of the chest 3. Bronchoscopy 4. Thoracentesis

Answer: 4 - Thoracentesis Explanations: There is no evidence of infection, so antibiotics would not be appropriate. CT of the chest may have a small yield, but thoracentesis will provide the most rapid diagnosis. Sampling the pleural fluid by thoracentesis will determine if the fluid is exudate or transudate in nature and aids in diagnosis. Thoracentesis also will provide symptomatic relief. Go to the next page if you knew the correct answer, or click the link image(s) below to further research the concepts in this question (if desired).

Research Concepts: Pleural Effusion

We update eBooks quarterly and Apps daily based on user feedback. Please tap flag to report any questions that need improvement.

Question 255: A 36-year-old female with a past medical history of type 1 diabetes mellitus and end-stage renal disease on hemodialysis presented with complaints of fever, chills, nausea, and vomiting. On examination, her temperature was 101F and heart rate 120 beats/min. The patient appeared to be in mild distress, and there was tenderness to palpation of the right chest wall permacath. An empiric antibiotic regimen was started for a presumed line infection. Blood cultures were positive for Staphylococcus aureus. On day 6 of antibiotic therapy, the patient remains febrile. A contrast-enhanced CT shows a 5 cm right perinephric abscess. What is the next step in the treatment of this patient?

Choices: 1. Continue current antibiotic regimen 2. Percutaneous catheter drainage 3. Surgical drainage 4. Nephrectomy

Answer: 2 - Percutaneous catheter drainage Explanations: Perinephric abscesses greater than 3 mm should be drained. Abscesses less than 3 mm should initially be treated with antibiotics. Initially, percutaneous catheter drainage should be done for abscesses greater than 3 mm. Surgical intervention might be indicated when percutaneous drainage is unsuccessful. Go to the next page if you knew the correct answer, or click the link image(s) below to further research the concepts in this question (if desired).

Research Concepts: Perinephric Abscess

We update eBooks quarterly and Apps daily based on user feedback. Please tap flag to report any questions that need improvement.

Question 256: A 74-year-old diabetic male presents to the emergency department for leftsided chest pain. His past medical history is significant for type 2 diabetes, major depressive disorder, and congestive heart failure. A bedside EKG is performed and shows a left bundle branch block. The patient takes doxepin for his depression. Which of the following is the next best step in the management of this patient?

Choices: 1. Advise patient to continue current antidepressant medication 2. Advise patient to stop his antidepressant medication 3. Increase the dosage of doxepin 4. Switch to another class of antidepressant

Answer: 4 - Switch to another class of antidepressant Explanations: Tricyclics have serious side effects such as cardiotoxicity, conduction abnormalities, orthostatic hypotension, syncope, dysrhythmias, bradycardia, and tachycardia. Thorough history taking including ruling out coronary artery disease risk factors should be gathered from a patient before prescribing an antidepressant. This patient has risk factors such as diabetes and a history of heart failure. It is recommended that the patient stay away from tricyclics as they can worsen his symptoms. Tricyclic antidepressants have serious adverse effects especially in those patients who have cardiovascular risk factors. It has been documented that TCAs can cause conduction abnormalities, heart blocks, as well as PR, QRS, and QT interval changes. It is advised to stay away from these types of medications in patients who have risks factors or coronary artery disease. Other antidepressants such as SSRIs are less potent and toxic. Risks and benefits must be assessed as SSRI can still cause dysrhythmias like atrial fibrillation, bradycardia or syncope. Go to the next page if you knew the correct answer, or click the link image(s) below to further research the concepts in this question (if desired).

Research Concepts: Doxepin

We update eBooks quarterly and Apps daily based on user feedback. Please tap flag to report any questions that need improvement.

Question 257: A 17-year-old patient presented to the emergency department following a crush injury to their thorax and abdomen. They undergo a CT scan as part of their trauma workup. The radiologist is concerned that the patient has a diaphragmatic hernia. Which of the following symptoms is most likely?

Choices: 1. Constipation 2. Shortness of breath 3. Shoulder tip pain 4. Visual disturbance

Answer: 2 - Shortness of breath Explanations: A diaphragmatic hernia can result in increased thoracic pressure which can lead to respiratory symptoms, including shortness of breath. Shoulder tip pain could potentially occur as this result from diaphragmatic irritation but in the case of crushing thoracic and abdominal trauma, this would likely to not be noticed by the patient. It is important to note that shortness of breath is not a specific sign of a diaphragmatic hernia especially in a trauma case, and there are more common causes that need to be explored and considered first. In this case, there is CT evidence of diaphragmatic herniation. Additional signs to look for include tracheal deviation and reduced chest expansion on the side of the herniation. Go to the next page if you knew the correct answer, or click the link image(s) below to further research the concepts in this question (if desired).

Research Concepts: Diaphragmatic Hernia

We update eBooks quarterly and Apps daily based on user feedback. Please tap flag to report any questions that need improvement.

Question 258: A 57-year-old female was admitted to the medical intensive care unit (MICU) for septic shock and acute respiratory failure requiring endotracheal intubation secondary to multilobar pneumonia. She was started on appropriate therapy; however her MICU course was complicated with the development of intermittent rapid, narrow complex tachycardia episodes that usually resolve with vagal maneuvers; however, it is not refractory. On physical exam, vitals are blood pressure 110/75 mmHg, heart rate 180 bpm, respiratory rate 18/minute, and oxygen saturation 90% on FiO2 of 70%. Her only access is a right internal jugular triple lumen central line in which she is administered intravenous fluids and antibiotics. Which of the following is the next step to treat the patient's tachycardia?

Choices: 1. Start patient on esmolol drip starting at 50 mcg/kg/min and titrate accordingly 2. Administer adenosine at 6 mg IV bolus followed by 20 mL of a saline flush 3. Administer adenosine at 3 mg IV bolus followed by 20 mL of a saline flush 4. Load patient with digoxin at 10 mcg/kg IV over the next 24 hours

Answer: 3 - Administer adenosine at 3 mg IV bolus followed by 20 mL of a saline flush Explanations: ACC/AHA/HRS guidelines state that first-line therapy for supraventricular tachycardia (SVT) in an adult is vagal maneuvers or adenosine. Adenosine should be administered at a lower dose when given through central access, starting at a dose of 3 mg IV bolus. Other causes to decrease the initial dose to 3 mg are in cardiac transplant patients or patients on dipyridamole or carbamazepine drugs. Any patient receiving adenosine should be on a form of cardiac monitoring. Patients treated for SVT are often on a 12-lead electrocardiogram rhythm monitoring to assess the underlying rhythm while adenosine is actively affecting the atrioventricular (AV) node. Adenosine can be further classified as a miscellaneous antiarrhythmic drug outside the Vaughan-Williams classification scheme. It acts on receptors in the cardiac AV node, significantly reducing conduction time through hyperpolarization. Go to the next page if you knew the correct answer, or click the link image(s) below to further research the concepts in this question (if desired).

Research Concepts: Adenosine

We update eBooks quarterly and Apps daily based on user feedback. Please tap flag to report any questions that need improvement.

Question 259: A multiplace chamber facility is contacted by another medical facility 10 miles away via a major highway. The contacting practitioner states that they have a patient who has just finished an intraoperative debridement for a necrotizing fasciitis infection. They want to send the patient for adjuvant hyperbaric oxygen therapy. She originally presented with tachypnea, developed septic shock, and acute respiratory failure. The patient has a medical history of uncontrolled diabetes mellitus, hypertension, hyperlipidemia, and congestive heart failure with a most recent ejection fraction of 25%. She is currently intubated, and her vitals show temperature 38.3 C (101 F), heart rate 110 beats/min, respiratory rate 20/minute, blood pressure 100/60 mmHg, and oxygen saturation 98% on a ventilator. Is this patient appropriate for treatment?

Choices: 1. No, the patient is too unstable for treatment. 2. No, the patient has an ejection fraction under 35%, which is a contraindication for treatment. 3. Yes, treat the patient daily at 3 ATA for up to 30 treatments. 4. Yes, treat the patient initially twice a day until the spread of infection has ceased.

Answer: 4 - Yes, treat the patient initially twice a day until the spread of infection has ceased.

Explanations: In necrotizing fasciitis, it is recommended to treat twice a day. Most patients will only need seven to 10 treatments for the spread of infection to cease. The patient's ejection fraction is lower than 35%, but this is only a relative contraindication, not an absolute contraindication. This patient does have an indicated diagnosis, does not have any absolute contraindications, and a multiplace facility has the capabilities for providing critical care during treatment if necessary. Hyperbaric oxygen therapy slows the progression of necrosis in these patients. Also, it augments the efficacy of antimicrobials, demarcating viable tissue from nonviable tissue and enhancing limb salvage. Go to the next page if you knew the correct answer, or click the link image(s) below to further research the concepts in this question (if desired).

Research Concepts: Necrotizing Fasciitis

We update eBooks quarterly and Apps daily based on user feedback. Please tap flag to report any questions that need improvement.

Question 260: A 56-year-old female presents to the hospital with shortness of breath. She has a past medical history of hypertension, hyperlipidemia, type 2 diabetes mellitus, and COPD. She also notes nausea but denies vomiting, chest pain, or abdominal pain. Physical examination shows a distressed female with deep, prolonged respirations; however, lungs are clear to auscultation. The abdomen is non-tender and non-distended. Vital signs show a heart rate of 120/min, respiratory rate 28/min, blood pressure 120/70 mmHg, temperature 37.2 C, and Sp02 99% on room air. Lab work reveals pH 7.1, pCO2 13 mmHg, pO2 90 mmHg, HCO3 less than 6 mEq/L, sodium 130 mEq/L, potassium 4.0 mEq/L, chloride 100 mEq/L, creatinine 1.3 mg/dL, BUN 24 mg/dL, and glucose 200 mg/dL. Urinalysis shows 3+ ketones. Which of the following medications most likely caused this patient's symptoms?

Choices: 1. Metformin 2. Glyburide 3. Insulin 4. Canagliflozin

Answer: 4 - Canagliflozin Explanations: Canagliflozin is an SGLT-2 inhibitor that has been shown to be associated with DKA with euglycemia. SGLT-2 inhibitors affect the proximal convoluted tubule of the nephron to prevent reabsorption of glucose. Over time, this can result in severe dehydration and acidosis, while maintaining euglycemia. Metformin can cause severe lactic acidosis, however, this patient is in DKA, which is not precipitated by metformin. Glyburide can cause hypoglycemia but is unlikely to result in severe acidosis. Insulin can result in hypoglycemia but is unlikely to cause severe acidosis. Go to the next page if you knew the correct answer, or click the link image(s) below to further research the concepts in this question (if desired).

Research Concepts: Euglycemic Diabetic Ketoacidosis

We update eBooks quarterly and Apps daily based on user feedback. Please tap flag to report any questions that need improvement.

Question 261: A 56-year-old female is referred to the clinic from her gynecologist where she had gone to get cervical cancer screening. During the office visit there, she fell asleep midconversation. Given her body habitus, the gynecologist was concerned for obstructive sleep apnea and made the referral. Blood work done at the OBGYN visit is significant for elevated serum bicarbonate at 32 mEq/L. She denies smoking. There is some concern for obstructive sleep apnea (OSA) and obesity hypoventilation syndrome (OHS). Which of the following is most accurate about OHS?

Choices: 1. An alternative neuromuscular, mechanical or metabolic explanation for hypoventilation can be coexistent 2. Presence of awake alveolar hypoventilation characterized by daytime hypercapnia (arterial PCO2 greater than 45 mmHg) 3. A consequence of diminished ventilatory drive and capacity related to individual patient being overweight (BMI greater than 25) 4. Presence of awake hypoxia characterized by arterial PO2 less than 65 mmHg

Answer: 2 - Presence of awake alveolar hypoventilation characterized by daytime hypercapnia (arterial PCO2 greater than 45 mmHg)

Explanations: OHS is defined as the presence of awake alveolar hypoventilation characterized by daytime hypercapnia (arterial PCO2 greater than 45 mm Hg [5.9 kPa]) that is thought to be a consequence of diminished ventilatory drive and capacity related to obesity (BMI greater than 30) in the absence of an alternative neuromuscular, mechanical or metabolic explanation for hypoventilation. Daytime Hypercapnia is the hallmark of OHS, differentiating it from OSA alone. Although OHS/Pickwickian syndrome could have a component of hypoxia coexistent in the more severe cases, it is not required for defining OHS. Other causes of hypoventilation have to be ruled out before making the final diagnosis. Obesity is required at a range of BMI greater than 30. Go to the next page if you knew the correct answer, or click the link image(s) below to further research the concepts in this question (if desired).

Research Concepts: Pickwickian Syndrome

We update eBooks quarterly and Apps daily based on user feedback. Please tap flag to report any questions that need improvement.

Question 262: A 56-year-old female who had a total abdominal hysterectomy via laparoscopy for fibroids and heavy vaginal bleeding is being evaluated on the second post-op day. Her pain is well-controlled, and she is eating a normal diet. Her major complaint is bilateral ankle swelling, stating, "my socks are too tight." What is the most likely cause of this symptom?

Choices: 1. Excessive IV fluids in the operating room 2. Physiologic vasopressin surge 3. Deep vein thrombosis 4. Body dysmorphic syndrome

Answer: 2 - Physiologic vasopressin surge Explanations: Vasopressin (or antidiuretic hormone) is secreted by the body in response to stress or shock. Increase of vasopressin is a well-known phenomenon after surgical procedures. It can cause water retention in postoperative patients and can explain the mild lower extremity swelling in this patient. While it is possible this patient got more fluid than necessary in the operating room, this is not the best answer available. Signs and symptoms of a DVT include unilateral lower extremity swelling, redness, and pain, especially with dorsiflexion of the foot. Because this swelling is bilateral, a DVT is not the highest differential diagnosis. Go to the next page if you knew the correct answer, or click the link image(s) below to further research the concepts in this question (if desired).

Research Concepts: Insensible Fluid Loss

We update eBooks quarterly and Apps daily based on user feedback. Please tap flag to report any questions that need improvement.

Question 263: A 45-year-old male patient from Guatemala presents to the emergency department with a complaint with "coughing up blood." A chest x-ray was performed that shows what seems like left upper lobe cavitary lesions with associated hilar lymphadenopathy. While reevaluating the patient, he suddenly starts coughing up large quantities of bright red blood. The patient was placed in the left lateral decubitus position and Trendelenburg position, but the patient is still having large amounts of hemoptysis. Oxygen saturation for the patient continues to drop, and the patient becomes lethargic. The decision is made to intubate the patient. The patient is intubated and currently hemodynamically stable. The pulmonologist is at bedside ready for bronchoscopy and hospital transport is also at bedside ready to take the patient to CT scan. The surgical team is also on their way for evaluation of the patient. What is the best plan of action?

Choices: 1. Wait for surgery to evaluate the patient for possible emergent lobectomy 2. Take the patient to CT before performing bronchoscopy 3. Perform bronchoscopy for definitive care before performing CT 4. Wait for all consultants to be at the bedside for further discussion

Answer: 2 - Take the patient to CT before performing bronchoscopy Explanations: In a stable patient such as this who is already intubated, CT has demonstrated equal ability as bronchoscopy for localization of the source of bleeding. CT scan is able to determine the cause of the bleeding more often than bronchoscopy and allows for guidance of embolization. Surgical lobectomy is considered last line treatment given the high rates of mortality and is significantly safer when performed non-emergently. Bronchoscopy has the advantage of being able to be performed at the bedside without transportation of the patient and allows for pulmonary isolation techniques if hemostasis has not been achieved. Go to the next page if you knew the correct answer, or click the link image(s) below to further research the concepts in this question (if desired).

Research Concepts: Pulmonary Hemorrhage

We update eBooks quarterly and Apps daily based on user feedback. Please tap flag to report any questions that need improvement.

Question 264: A 76-year-old patient with past medical history significant for heart failure with reduced ejection fraction (HFrEF) with 30% to 35%, chronic obstructive pulmonary disease, and iron deficiency anemia was admitted for acute dyspnea on exertion with wheezing. CBC revealed hemoglobin of 6.2 g/dL and hematocrit of 20%. A packed red blood cell transfusion was initiated. 7 hours later, the patient develops worsening of dyspnea with oxygen saturation of 72% on room air. Body temperature is reported as 101.2 F. Lung sounds show rales bilaterally. Neck vein is distended, and the patient developed hypertension. CBC showed WBC of 10.1 and platelet count of 237/microliter. What is the most likely diagnosis?

Choices: 1. Transfusion associated circulatory overload (TACO) 2. Hemolytic transfusion reaction 3. Acute exacerbation of chronic obstructive pulmonary disease 4. Transfusion related acute lung injury (TRALI)

Answer: 1 - Transfusion associated circulatory overload (TACO) Explanations: TACO and TRALI both can have rales, but TACO has neck distended vein due to circulatory overload and develops hypertension. TRALI develops hypotension and fever. Patients with systolic dysfunction are more vulnerable to TACO than TRALI with cardiogenic pulmonary edema. TRALI likely to show leukopenia and thrombocytopenia. Go to the next page if you knew the correct answer, or click the link image(s) below to further research the concepts in this question (if desired).

Research Concepts: Transfusion-related Acute Lung Injury

We update eBooks quarterly and Apps daily based on user feedback. Please tap flag to report any questions that need improvement.

Question 265: A 46-year-old female with a past medical history of cirrhosis of liver presents with shortness of breath and bilateral pedal edema. On examination, JVD is raised, spider angioma and gynecomastia are present. A chest x-ray is normal, and echocardiography reveals pulmonary artery pressure of 40 mmHg and a dilated right ventricle. Which of the following is the next best step in the management of this patient?

Choices: 1. Computed tomogram of chest 2. Diuretics and discharge 3. Cardiac catheterization 4. Stress test

Answer: 3 - Cardiac catheterization Explanations: Patients with cirrhosis of the liver are prone to arteriovenous malformation. Since chest x-ray is normal, pleural effusion and hepatopulmonary syndrome are ruled out. Echo is suggestive of pulmonary artery hypertension and right ventricular volume overload. Cardiac catheterization is diagnostic to rule out coronary cameral fistula formation or interatrial shunt formation due to hyperestrogenic state due to liver cirrhosis. Go to the next page if you knew the correct answer, or click the link image(s) below to further research the concepts in this question (if desired).

Research Concepts: Coronary Cameral Fistula

We update eBooks quarterly and Apps daily based on user feedback. Please tap flag to report any questions that need improvement.

Question 266: A patient has suffered an out-of-hospital cardiac arrest. Upon arrival, the patient is hemodynamically stable. On evaluation, the patient has a regular rate and rhythm. Coarse breath sounds are heard throughout both lungs; the patient is intubated and mechanically ventilated. The patient currently has a Glasgow coma scale score (GCS) of 3T, is areflexic, does not arouse, and is currently on propofol and fentanyl for sedation. The sedatives are temporarily held, with no change in his neurologic exam. The abdomen is benign. The patient's pupils are pinpoint and non-reactive. What can be determined about the possible use of targeted temperature management or therapeutic hypothermia in this patient?

Choices: 1. Therapeutic hypothermia is indicated immediately due to the neurologic examination 2. Targeted temperature management is indicated immediately due to the neurologic exam 3. Neither temperature option is initially indicated with these examination findings 4. The patient has progressed to brain death and further care is futile

Answer: 2 - Targeted temperature management is indicated immediately due to the neurologic exam

Explanations: Targeted temperature management is the process of maintaining the temperature at or below 36 degrees Celsius, whereas therapeutic hypothermia involves maintaining the temperature within the range of 32 to 34 degrees Celsius. Historically, the suggested temperature range was 32 to 34 degrees Celsius; however, newer trials suggest similar benefits at 36 degrees compared to 33 degrees giving rise to targeted temperature management. Cessation of sedation and neurologic evaluation, if stable, should be performed to assess for anoxic brain injury. It is not possible to determine brain death at this time due to sedation as well as the timeframe of illness. After cessation of sedation, an accurate neurologic examination should be obtained, and the patient may require temperature guidelines. It is recommended that, in addition to daily assessments, reapproaching this possibility at 72 hours is more appropriate. Go to the next page if you knew the correct answer, or click the link image(s) below to further research the concepts in this question (if desired).

Research Concepts: Anoxic Encephalopathy

We update eBooks quarterly and Apps daily based on user feedback. Please tap flag to report any questions that need improvement.

Question 267: A 17-year-old male patient with a history of intravenous drug use complains of a few days history of pain in his left arm, double vision, fever, and difficulty swallowing. He is awake, alert, and oriented to person, place and time, and denies any headache or neck stiffness. Exam shows a blood pressure of 110/81 mmHg, a temperature of 37.4 C (99.3 F), heart rate of 110 beats/min, and respiratory rate of 18. Kernig and Brudzinski tests are negative. A large erythematous, warm, and indurated rash on the left arm is noted with several track marks and pustular lesions. Cranial nerves II through XII are grossly intact, and the bladder is distended. Which of the following is the next best step in the management of this patient?

Choices: 1. CT head, antibiotics, admit to intensive care unit (ICU) 2. CT head, lumbar puncture, antibiotics, admit to floor 3. Antitoxin, antibiotics, admit to ICU 4. Antitoxin, antibiotics, admit to floor

Answer: 3 - Antitoxin, antibiotics, admit to ICU Explanations: The description is classic for wound botulism, most likely from Clostridium botulinum cellulitis secondary to subcutaneous injection of spores from contaminated drugs. Respiratory failure may occur suddenly, thus the admission to intensive care. Botulism can be acquired through exposure to the pre-formed toxin via improperly-stored food, iatrogenic injection, or bioterrorism, or it can result from the systemic release of the toxin in vivo, as in the cases of infant and wound botulism. Antitoxin should be administered as soon as botulism is suspected. Antibiotics for wound botulism include penicillin or metronidazole if penicillin-allergic. Go to the next page if you knew the correct answer, or click the link image(s) below to further research the concepts in this question (if desired).

Research Concepts: Botulism

We update eBooks quarterly and Apps daily based on user feedback. Please tap flag to report any questions that need improvement.

Question 268: A 61-year-old male with a history of kidney transplantation on immunosuppressant medications presents to the clinic for a 6-month follow-up after he had an episode of acute hepatitis E infection. He denies having any symptoms. His vitals are normal. BMI 23. Labs are significant for ALT 140 IU/L, AST 127 IU/L with normal serum bilirubin and ALP. Serum creatinine and GFR are at baseline. He denies any changes to his medications. Hepatitis A, B, and C serology are negative. CMV serology and PCR are negative. Hepatitis E PCR is positive. RUQ ultrasound is within normal limits. Liver biopsy shows diffuse inflammation in the portal and periportal areas. His immunosuppressant medications doses were decreased, however, without success in clearing his viremia. What is the best next step in the management of this patient?

Choices: 1. Orlistat 2. Ursodiol 3. Observation 4. Ribavirin

Answer: 4 - Ribavirin Explanations: Patients with a history of solid organ transplantation on immunosuppressant medications who develop acute hepatitis E infection have a high risk of progression to chronic hepatitis E. This patient’s liver biopsy is consistent with chronic viral hepatitis, and his hepatitis E viremia is consistent with chronic hepatitis E virus. Decreasing the doses of immunosuppressant medications has been shown to clear the virus in about 30% of patients; however, it was unsuccessful in this case. Ribavirin and pegylated interferon have shown to be effective in treating chronic hepatitis E infection and should be considered in this patient. Orlistat is used for weight loss and is not indicated in this patient. Ursodiol is used in patients with primary biliary cholangitis; however, it is not indicated in this patient. This patient can benefit from ribavirin to clear his chronic infection and should be considered. Go to the next page if you knew the correct answer, or click the link image(s) below to further research the concepts in this question (if desired).

Research Concepts: Hepatitis E

We update eBooks quarterly and Apps daily based on user feedback. Please tap flag to report any questions that need improvement.

Question 269: A 16-year-old male presents to the emergency department with difficulty breathing. His symptoms and clinical presentation are consistent with status asthmaticus. WIthin 3 hours, he rapidly deteriorates with evidence fo fatigue, altered mental status and combined respiratory and metabolic acidosis. He was therefore intubated and placed on mechanical ventilation. Initial ventilator settings had to be adjusted due to asynchrony on ventilator and hypotension from intrinsic PEEP. He is still diffusely bronchospastic and is getting albuterol via a metered dose inhaler attached to the inspiratory limb of vent circuit. He is on assist-control volume-control mode ventilation. Which of the following is most accurate regarding his ongoing treatment strategy?

Choices: 1. Ventilation mode should be changed to pressure regulated volume control or VC+ 2. Ongoing method of albuterol administration is appropriate as MDI has a better effect on inspiratory flow-resistive pressure 3. Albuterol administration should always be via a nebulizer in a ventilated patient 4. There is no definitive appropriate strategy of albuterol administration but an assessment of airway peak to pause pressure gradient can decide the appropriate therapy in an individual patient

Answer: 4 - There is no definitive appropriate strategy of albuterol administration but an assessment of airway peak to pause pressure gradient can decide the appropriate therapy in an individual patient

Explanations: There is an ongoing debate about the use of MDI versus nebulizers in ventilated patients. While small volume nebulizers have been shown to have reduced aerosol percentage delivery to lungs, MDI has been demonstrated to have a poor effect on inspiratory flowresistive pressure. Assessment of airway peak to pause pressure gradient can be a rational indicator to use when either one of the delivery modes is used. A 15% or greater decline in the gradient is considered to be a favorable response to be aimed for. A higher dose of the medication is needed in both MDI or nebulized delivery in an intubated patient. Go to the next page if you knew the correct answer, or click the link image(s) below to further research the concepts in this question (if desired).

Research Concepts: Status Asthmaticus

We update eBooks quarterly and Apps daily based on user feedback. Please tap flag to report any questions that need improvement.

Question 270: A 65-year-old female presents to his healthcare provider for the evaluation of fatigue and general malaise that has been present for the past few days. On further questioning, she reports a recent head cold with some sinus congestion that has since improved. She, however, claims that she is now having some mild chest discomfort and some increased work in breathing. Her vital signs show a temperature of 99.2 F, pulse 89/min, respiratory rate 20/min, SpO2 95%, and blood pressure 118/76 mmHg. Her ECG shows a peculiar pattern of varying levels of the height of her QRS complex. If left untreated, which of the following combination of symptoms is most likely to occur?

Choices: 1. S1Q3T3 complex on ECG, tachycardia, inverted T waves in lateral leads 2. Hypotension, jugular venous distension, hypoxia 3. Hypertension, bradycardia, decreased respiratory rate 4. Hypotension, distended neck vein, muffled heart sounds

Answer: 4 - Hypotension, distended neck vein, muffled heart sounds Explanations: Hypotension, JVD and muffled heart sounds is Beck's triad which is associated with cardiac tamponade which can result in the electrical alternans appearance on ECG Electrical alternans is defined as alternating QRS amplitude that is seen in any or all leads on an electrocardiogram (ECG) with no additional changes to the conduction pathways of the heart. This rhythm is typically associated with pericardial effusion via the “swinging heart” from the fluid surrounding the heart. However, electrical alternans is also associated with other pathologies including but not limited to ventricular tachycardia, Wolff-Parkinson-White (WPW), accelerated idioventricular rhythm, and supraventricular tachycardia. S1Q3T3 and T wave inversion may be seen in pulmonary embolism. Hypotension, JVD, and hypoxia are more likely related to pulmonary congestion. Hypertension, bradycardia, and decreased respiratory rate is Cushing triad, which is associated with herniation. Go to the next page if you knew the correct answer, or click the link image(s) below to further research the concepts in this question (if desired).

Research Concepts: Electrical Alternans

We update eBooks quarterly and Apps daily based on user feedback. Please tap flag to report any questions that need improvement.

Question 271: A 17 year-old-male mountain climber and his colleagues are attempting to summit Mount Everest. After ascending to an altitude of about 5500 meters on the second day, he complained of headaches, anorexia, nausea, and malaise. On day three of the expedition, he developed ataxia, impaired cognition, irrational behavior and errors in reading his map. What is the most likely diagnosis?

Choices: 1. Acute mountain sickness 2. High-altitude cerebral edema (HACE) 3. High-altitude pulmonary edema (HAPE) 4. Meningitis

Answer: 2 - High-altitude cerebral edema (HACE) Explanations: Acute mountain sickness (AMS) is a syndrome characterized by headaches, anorexia, nausea, and malaise. AMS occurs when exposed to altitudes of 4000 meters, typically for 2 or more days. Headache must be present for a diagnosis of acute mountain sickness. Treatment includes NSAIDs and supplemental oxygen. Prophylaxis is with acetazolamide. High-altitude cerebral edema (HACE) is thought to be the end stage form of acute mountain sickness. The hallmark symptom of HACE is ataxia, but patients may also experience seizure, altered mental status, or other neurologic symptoms. This patient has symptoms of acute mountain sickness and ataxia. Hence, he has HACE, not AMS. Treatment is descent from altitude, corticosteroids, and oxygen. Acetazolamide may also be started. Symptoms of high-altitude pulmonary edema (HAPE) are breathlessness, chest pain, headache, fatigue, and dizziness. It occurs at an altitude of about 2000 to 2500 meters, with significant elevation of pulmonary artery pressure leading to capillary breakdown and subsequent edema. Go to the next page if you knew the correct answer, or click the link image(s) below to further research the concepts in this question (if desired).

Research Concepts: High Altitude Cerebral Edema

We update eBooks quarterly and Apps daily based on user feedback. Please tap flag to report any questions that need improvement.

Question 272: A 59-year-old man is admitted for pulmonary embolism and rivaroxaban is initiated. Three days later he has a hypotensive episode, and his hemoglobin and hematocrit drop significantly. He is transfused with four units of packed red blood cells and given 2 liters of normal saline and prothrombin complex concentrate (PCC) therapy. Two days later, hemoglobin and hematocrit are within the normal range, and the patient is hemodynamically stable. What is the most appropriate next step in management?

Choices: 1. Administer andexanet alfa to completely reverse rivaroxaban effects 2. Administer a second dose of PCC to completely reverse rivaroxaban effects 3. Initiate aspirin and monitor the patient for signs of bleeding 4. Resume rivaroxaban and monitor the patient for signs of bleeding

Answer: 4 - Resume rivaroxaban and monitor the patient for signs of bleeding Explanations: Andexanet alfa is a reversal agent that is an FDA-approved factor Xa inhibitor (i.e., rivaroxaban) in patients with life-threatening or uncontrolled bleeding. Prothrombin complex concentrate (PCC) is a blood coagulation factor replacement product that is FDA-approved for the reversal of vitamin K antagonists. PCC has been used offlabel to reverse the effects of direct oral anticoagulants, like rivaroxaban. It is important to resume anticoagulation therapy as soon as it is medically appropriate to reduce the risk of thrombotic events. This patient has received PCC and is hemodynamically stable; therefore, he no longer needs a reversal agent. Anticoagulation with rivaroxaban should be resumed, and the patient should be monitored for signs of bleeding. Go to the next page if you knew the correct answer, or click the link image(s) below to further research the concepts in this question (if desired).

Research Concepts: Andexanet Alfa

We update eBooks quarterly and Apps daily based on user feedback. Please tap flag to report any questions that need improvement.

Question 273: A 22-year-old man admitted to the hospital complaining of diarrhea and fever of 2 days duration. The patient underwent stem cell transplantation five weeks ago. He denies blood in the stool. On physical exam, his temperature is 37.5 C (99.5 F), blood pressure 112/75 mmHg, pulse rate 107/min, and respiratory rate is 15/min. Abdominal exam reveals tenderness more on the left lower quadrant. A CT scan shows thickened bowel wall of the descending colon, and he is diagnosed with neutropenic enterocolitis. The patient is started on intravenous fluid support with bowel rest. Testing for Clostridium difficile is still pending. What is the next best step in the management of this patient?

Choices: 1. Ampicillin-sulbactam 2. Intravenous vancomycin and ceftriaxone 3. Piperacillin/ tazobactam and oral vancomycin 4. Piperacillin/ tazobactam and intravenous vancomycin

Answer: 4 - Piperacillin/ tazobactam and intravenous vancomycin Explanations: The patient's clinical presentation and CT findings correlate with a diagnosis of colitis, which can be neutropenic enterocolitis or other infectious colitis. Although there is no consensus on the treatment neutropenic enterocolitis, the treatment should cover for the most common organisms that may seed from the gastrointestinal tract - i.e., gram-negative bacilli including pseudomonas and anaerobic bacteria. Piperacillin/ tazobactam plus intravenous vancomycin is the empirical treatment suggested for NE by multiple studies. Although there is no consensus on the best treatment protocol, any regimen should cover gram-negative bacilli, anaerobic bacteria, and pseudomonas because who has NE is a highrisk patient because of neutropenia. Studies also found that adding coverage for grampositive bacteria decreases mortality. Ampicillin-sulbactam covers for gram-negative bacilli and anaerobic bacteria, but it doesn't cover for pseudomonas. So without adding anti-pseudomonal agent, the treatment will be insufficient. For the same reason, intravenous vancomycin plus ceftriaxone regimen would bot cover for pseudomonas or anaerobic bacteria. Clostridium difficile still pending for the patient. Treatment for C. difficile infection should not be given empirically to every patient presenting with neutropenic enterocolitis unless highly suspected, i.e., if the patient has risk factors for C. difficile infection which is not mentioned in the question. Go to the next page if you knew the correct answer, or click the link image(s) below to further research the concepts in this question (if desired).

Research Concepts: Neutropenic Enterocolitis (Typhlitis)

We update eBooks quarterly and Apps daily based on user feedback. Please tap flag to report any questions that need improvement.

Question 274: A 58-year-old male patient with rheumatoid arthritis and coronary artery disease managed is admitted for coronary artery bypass graft. Induction and intubation are uneventful. A blind attempt at transesophageal echocardiography (TEE) probe placement was unsuccessful after 2 attempts and was successfully placed with laryngoscopic assistance on the third attempt. The left ventricular ejection fraction (LVEF) was 50%, and no wall motion abnormalities were noted. The patient is transferred to the intensive care unit after surgery and is successfully extubated 2 hours later. He complains of hoarseness on postoperative day 2 but has no chest pain, dysphagia or shortness of breath. Which of the following is the most likely etiology for the hoarseness?

Choices: 1. Intubation 2. Coronary artery disease 3. Placement of a transesophageal echocardiography probe 4. Extubation

Answer: 3 - Placement of a transesophageal echocardiography probe Explanations: Airway management in patients with rheumatoid arthritis can be challenging due to limited mandibular motion and mouth opening, a narrowed glottis and cervical instability. In this patient, intubation was uneventful and is not likely to be the cause of the hoarseness. Coronary artery disease is not a predisposing factor for arytenoid subluxation. Placement of transesophageal echocardiography (TEE) probe. Major cardiac surgery involving the use of TEE may be associated with an increased incidence of arytenoid subluxation. TEE probes are usually inserted blindly and are only placed under direct vision with the help of a laryngoscope or video laryngoscope when resistance to placement is encountered. Traumatic extubation with a fully or partially inflated endotracheal tube cuff may also lead to the posterolateral displacement of the arytenoid leading to postoperative hoarseness. In this patient extubation was successful and no complications were noted. Go to the next page if you knew the correct answer, or click the link image(s) below to further research the concepts in this question (if desired).

Research Concepts: Arytenoid Subluxation

We update eBooks quarterly and Apps daily based on user feedback. Please tap flag to report any questions that need improvement.

Question 275: An adult patient presents with complaints of paresthesias and occasional nausea after having been recently started on a new drug. He is found to have hyperchloremic metabolic acidosis. Which diuretic is the patient taking?

Choices: 1. Hydrochlorothiazide 2. Furosemide 3. Acetazolamide 4. Mannitol

Answer: 3 - Acetazolamide Explanations: Acetazolamide is a carbonic anhydrase inhibitor used to treat some types of glaucoma as well as epilepsy. It is also effective for prevention and treatment of acute altitude illness and treatment of congestive heart failure. It is a mild diuretic that causes hyperchloremic metabolic acidosis. Common adverse effects of acetazolamide include paraesthesia, depression, drowsiness, nausea, and vomiting. Less common adverse effects include Stevens-Johnson syndrome, anaphylaxis, and blood dyscrasias. Go to the next page if you knew the correct answer, or click the link image(s) below to further research the concepts in this question (if desired).

Research Concepts: Acetazolamide

We update eBooks quarterly and Apps daily based on user feedback. Please tap flag to report any questions that need improvement.

Question 276: A 42-year-old female from Mexico presents with a 1-week history of fever, chills, and right upper quadrant pain. She is tender in the right upper quadrant. Ultrasound shows cholelithiasis but no thickening of the wall of the gallbladder or pericholecystic fluid. There is a 2 cm x 2 cm mass in the liver at the right lobe that is hypoechoic. She is started on cefoxitin. A CT guided drain is placed and only yields 15 mL of fluid over the next 5 days. She has persistent fever and leukocytosis. Repeat CT shows the abscess has not changed. What is the best next step in management?

Choices: 1. Catheter irrigation, catheter upsizing, insertion of another drain, or a combination thereof 2. Laparoscopic surgery 3. Open surgery 4. Change of antibiotics

Answer: 1 - Catheter irrigation, catheter upsizing, insertion of another drain, or a combination thereof

Explanations: Persistence of the liver abscess indicates there is not adequate drainage. Options include catheter irrigation with saline or tPA, switching out the initial catheter for a larger catheter, insertion of another catheter, or a combination thereof. Depending on the site of the abscess, a persistent abscess can be treated with laparoscopic or open drainage. Antibiotics, even adjusted for sensitivities will not be adequate. Go to the next page if you knew the correct answer, or click the link image(s) below to further research the concepts in this question (if desired).

Research Concepts: Liver Abscess

We update eBooks quarterly and Apps daily based on user feedback. Please tap flag to report any questions that need improvement.

Question 277: A 53-year-old woman who has a past medical history of atrial fibrillation who is brought to the emergency department with dizziness, light-headedness, and blood in her stool. She has been taking rivaroxaban 20 mg once a day for approximately one year. She received her last dose of rivaroxaban approximately 14 hours ago. Her hemoglobin is 6.7 g/dL, and blood pressure is 78/49 mmHg. She is given fluid boluses with normal saline, is transfused with two units of packed red blood cells and started on a norepinephrine infusion. The provider decides to use andexanet alfa. Which of the following is an appropriate order for andexanet alfa for this patient?

Choices: 1. 400 mg IV; target infusion rate of 30 mg/min followed by an IV infusion of 4 mg/min IV for up to 2 hours 2. 400 mg oral dose followed by 240 mg given orally every hour for up to 2 hours 3. 800 mg administered orally 4. 400 mg IV bolus followed by two doses of 240 mg tablets administered orally every hour

Answer: 1 - 400 mg IV; target infusion rate of 30 mg/min followed by an IV infusion of 4 mg/min IV for up to 2 hours

Explanations: Andexanet alfa is available as a lyophilized powder in 100 mg vials. Unopened vials should be refrigerated at 2 to 8 C. Vials are reconstituted with sterile water for injection, and the calculated dose is administered intravenously as a bolus followed by an infusion. Reconstituted andexanet alfa in vials is stable at room temperature for up to 8 hours and at 2 to 8 C or up to 24 hours. Reconstituted andexanet alfa in IV bags is stable at room temperature for up to 8 hours and 2 to 8 C or up to 16 hours. Dosing of andexanet alfa is based on the factor Xa inhibitor to be reversed, the dose last taken, and the timing of the last dose. Go to the next page if you knew the correct answer, or click the link image(s) below to further research the concepts in this question (if desired).

Research Concepts: Andexanet Alfa

We update eBooks quarterly and Apps daily based on user feedback. Please tap flag to report any questions that need improvement.

Question 278: A 40-year-old man with no significant past medical history presents to the emergency department with fever, malaise, and right-sided pleuritic chest pain. Two weeks before admission, he complained of sore throat with low-grade fever but did not seek medical attention. His chest imaging reveals a right-sided empyema, and his neck ultrasound revealed a right internal jugular vein thrombus. Thoracentesis and cultures of the pleural fluid reveal growth in the anaerobic culture. What is the best next step in management?

Choices: 1. Starting anticoagulant therapy for the management of his thrombus 2. Intravenous ampicillin /sulbactam for 4-6 weeks 3. Intravenous imipenem for one week 4. Oral Metronidazole for 6 weeks

Answer: 2 - Intravenous ampicillin /sulbactam for 4-6 weeks Explanations: Lemierre syndrome presents with fever and a brief upper respiratory illness later develops septic embolization and distant infection, e.g., lung empyema and internal jugular vein septic thrombophlebitis. The treatment plan is usually intravenous antibiotics targeted toward Fusobacterium necrophorum the most common causative organism. Fusobacterium necrophorum is usually the causative organism for Lemierre syndrome, but due to the concern of mixed infection with other oral microbial flora, Monotherapy with Metronidazole is not recommended. Fusobacterium necrophorum is an uncommon cause of pharyngitis, untreated is usually complicated by suppurative thrombophlebitis of Lemierre syndrome. Due to possible betalactamase production, it is recommended to treat with a combination of a ß-lactam antibiotic and a ß-lactamase inhibitor or carbapenem. Fusobacterium necrophorum a common inhabitant of oral mucosa that can cause oropharyngeal infection and result in complications of suppurative thrombophlebitis of Lemierre syndrome and distant infections and require prolonged antibiotic treatment from 4-6 weeks. Go to the next page if you knew the correct answer, or click the link image(s) below to further research the concepts in this question (if desired).

Research Concepts: Septic Emboli

We update eBooks quarterly and Apps daily based on user feedback. Please tap flag to report any questions that need improvement.

Question 279: A 16-year-old patient underwent placement of an external ventricular drain for the management of acute hydrocephalus secondary to tubercular meningitis. Two days later he developed a high-grade fever associated with chills and rigor. He had neck stiffness on clinical examination. The CSF culture showed the growth of Pseudomonas aeruginosa sensitive to amikacin only. The treating clinician planned to start intraventricular amikacin therapy. What is the safe daily dosage of amikacin that can be administered intraventricularly in the patient?

Choices: 1. 10 mg 2. 30 mg 3. 60 mg 4. 100 mg

Answer: 2 - 30 mg Explanations: Intraventricular instillation of antibiotics is a valid option in treating patients with ventriculitis. The dose of amikacin through the intraventricular route is 1/20th of the intravenous dose. The average daily dose of amikacin through the intraventricular route is 30 mg. The maximum daily dose of amikacin should not exceed 50 mg. The dosage of amikacin in excess of 30 mg daily has shown to be associated with a high incidence of adverse effects, including choroid plexitis and the risk of an intraventricular bleed. Go to the next page if you knew the correct answer, or click the link image(s) below to further research the concepts in this question (if desired).

Research Concepts: Ventriculitis

We update eBooks quarterly and Apps daily based on user feedback. Please tap flag to report any questions that need improvement.

Question 280: A 28-year-old female is admitted with jaundice for 1 week with associated confusion. Her initial labs show hemoglobin 12.1 g/dL, WBC 5400/microL, platelets 543000/microL, Alanine aminotransferase (ALT) 3200 U/L, aspartate aminotransferase (AST) 1079 U/L , bilirubin 4.3 mg/dL, alkaline phosphatase 345 U/L, INR 4.5, and creatinine 1.02 mg/dL. Blood culture and urine culture are negative, and ultrasound of the abdomen showed normal liver echotexture with ascites. Which of the following is less likely associated with poor prognosis in this lady with acute liver failure?

Choices: 1. Coagulopathy 2. Duration of jaundice 3. Encephalopathy 4. Ascites

Answer: 4 - Ascites Explanations: Coagulopathy is the most important prognostic factor in acute liver failure (ALF). Derangement of clotting mechanism suggest poor prognosis and need for referral for liver transplant Duration of jaundice to hepatic encephalopathy has been related to prognosis. Subacute acute liver failure (ALF) of less than 1 month duration from jaundice with encephalopathy had poorer survival compared to hyperacute of duration less than 7 days and acute 1 week to 1 month duration between jaundice to encephalopathy ALF. Encephalopathy suggests cerebral edema and it suggests worst prognosis in ALF. Development of ascites is rare in ALF as ascites favor underlying chronic liver disease like cirrhosis. Ascites has no correlation to survival in ALF. Go to the next page if you knew the correct answer, or click the link image(s) below to further research the concepts in this question (if desired).

Research Concepts: Hepatic Failure

We update eBooks quarterly and Apps daily based on user feedback. Please tap flag to report any questions that need improvement.

Question 281: A critically ill 65-year-old male patient with a past medical history of diabetes, hypertension, congestive heart failure, obstructive sleep apnea, and chronic obstructive pulmonary disease is undergoing a pelvic fracture external reduction with internal fixation. The patient was placed on an insulin drip to control blood sugars. Last blood sugar prior to being taken back to the operating room was 180 mg/dL. When blood sugar was checked 2 hours into the case, it dropped to 65 mg/dL. What is the best next step to take in this case?

Choices: 1. Stop insulin drip 2. Start Dextrose-5 solution at a rate of 100cc/ hour 3. Give Dextrose-50 and stop insulin drip 4. Do not titrate the drip and follow up blood sugar in 15 minutes

Answer: 3 - Give Dextrose-50 and stop insulin drip Explanations: Hypoglycemia is defined as any blood sugar less than 70 mg/dL. Severe hypoglycemia is defined as a blood sugar less than 40 mg/dL. Hypoglycemia is an independent risk factor for morbidity and mortality in hospitalized patient's. If a patient is experiencing hypoglycemia in the operating room, give the patient Dextrose50, stop the insulin drip, and check blood sugar every 15 minutes until blood sugar trending upwards. It is best practice to flush out the carrier line that is connected to the insulin drip to ensure insulin that is stuck in the line is not inadvertently administered to the patient and further exaggerating hypoglycemia. Go to the next page if you knew the correct answer, or click the link image(s) below to further research the concepts in this question (if desired).

Research Concepts: Diabetes Intraoperative Management

We update eBooks quarterly and Apps daily based on user feedback. Please tap flag to report any questions that need improvement.

Question 282: A 46-year-old female is rushed to the emergency department following a motor vehicle accident in which she fell asleep at the wheel and ran her car into a light pole. While still trapped in the wreckage, she received third-degree electrical burns from an active wire, exposed because of the crash, over the dorsal surface of her right arm and shoulder. Besides the burns, the patient is suffering from several broken ribs. An ultrasound-guided FAST exam reveals one of the broken ribs punctured the right lung causing the patient to experience respiratory distress. Vital signs are a weight of 156 pounds (71 kg), blood pressure 100/53 mmHg, heart rate 110 beats/min, respiratory rate 35, temperature 99.1F, and oxygen saturation of 75% that continues to drop. On physical exam, the patient appears obtunded, her lips show blue discoloration, and hypertonicity of the neck muscles is appreciated when palpating the trachea. The emergency care provider decides to intubate to stabilize the patient’s breathing while waiting for the trauma team to arrive. The emergency care provider infuses 35 mg IV rocuronium before intubating but has difficulty securing the airway due to continued muscle tightness appreciated in the larynx. Why would the administration of rocuronium not be the best choice for facilitating intubation in this patient?

Choices: 1. Rocuronium only works well in patients with a lower body mass. 2. The patient is suffering from third-degree burns. 3. Steroidal non-depolarizing neuromuscular blockers should not be used for intubation, only benzylisoquinolines. 4. Rocuronium was the right choice; the patient is a fast metabolizer.

Answer: 2 - The patient is suffering from third-degree burns. Explanations: Rocuronium is infused IV at 0.45 to 0.90 mg/kg for intubation with 0.15 mg/kg boluses if needed for maintenance. Burn patients exhibit resistance to non-depolarizing neuromuscular blockers due to being in hyperkalemic states. Hyperkalemia causes resistance, while hypokalemia augments activity. Other electrolyte abnormalities to keep in mind are hypocalcemia and hypermagnesemia, both of which augment blockade. Both steroidal (rocuronium, vecuronium, pancuronium) and benzylisoquinoline (atracurium, cisatracurium, mivacurium) agents exhibit the same mechanism of action. What differentiates them is their chemical structure and reversal; the steroidal agents are reversed with sugammadex and the benzylisoquinoline agents with neostigmine/glycopyrrolate. Rocuronium is one of the few agents in this drug class that is not metabolized but instead cleared mainly by the liver and slightly by the kidneys in its original form. Its volume of distribution is not affected by the renal disease; however, its action is prolonged by severe hepatic failure and pregnancy. Go to the next page if you knew the correct answer, or click the link image(s) below to further research the concepts in this question (if desired).

Research Concepts: Non-depolarizing Neuromuscular Blockers

We update eBooks quarterly and Apps daily based on user feedback. Please tap flag to report any questions that need improvement.

Question 283: A 70-year-old female with chronic obstructive pulmonary disease is placed on mechanical ventilation for respiratory failure. Intubation required rocuronium for paralysis. The ventilator was placed on assist-control (AC) mode with a rate of 12, a fraction of inspired oxygen (FIO2) of 1.0, a tidal volume of 500, and positive end-expiratory pressure (PEEP) of 0. Arterial blood gas after intubation demonstrates pH 7.23, PaCO2 75 mmHg, and PO2 350 mmHg, so FIO2 is decreased to 0.70. Half an hour later the patient becomes hypotensive with a blood pressure of 75/40 mmHg, heart rate 135 beats per minute, and respiratory rate 26/minute. The trachea is midline, and there are breath sounds in both lung fields. Bilateral wheezing persists until the next inspiration. The high-pressure alarm has triggered. What is the preferred initial management?

Choices: 1. Disconnect the ventilator and manually ventilate the patient 2. Administer intravenous fluid bolus 3. Perform bilateral needle decompression of thorax 4. Start dopamine infusion

Answer: 1 - Disconnect the ventilator and manually ventilate the patient Explanations: Mechanical ventilation of patients with chronic obstructive pulmonary disease may develop intrinsic positive end-expiratory pressure also known as auto-positive end-expiratory pressure (PEEP). Obstructive lung disease requires a longer time for expiration to avoid the development of auto-PEEP and dynamic hyperinflation. This can be avoided by having a low inspiratory: expiratory ratio (I:E). As auto-PEEP and dynamic hyperinflation progress there is an increase in transthoracic pressure. This results in decreased preload into the right ventricle. As preload decreases you can get resulting hypotension. The increased alveolar pressure can also result in pneumothorax, but this has not yet happened in this patient. The ventilator settings will need to be adjusted. There will need to be a prolonged expiration time. As the patient is overbreathing the ventilator, she will need to be highly sedated to allow for ventilator synchrony. If needed the inspiratory flow rate can be increased to help lower the I:E ratio. Go to the next page if you knew the correct answer, or click the link image(s) below to further research the concepts in this question (if desired).

Research Concepts: Mechanical Ventilation

We update eBooks quarterly and Apps daily based on user feedback. Please tap flag to report any questions that need improvement.

Question 284: A 42-year-old man was admitted two days back after an intentional overdose of his medications at home. His chronic medical conditions include hypertension, mild pulmonary hypertension, and Raynaud syndrome. He does not smoke or drink alcohol. Initial presentation was with acute shock and hypoxic respiratory failure requiring emergent endotracheal intubation. Initial lab work did not show any renal or hepatic dysfunction. He has since been on vasopressors with maximal norepinephrine dose of 10 micrograms per minute 24 hours back and slowly being weaned. He has also received insulin and dextrose infusion and several grams of calcium chloride within the first 24 hours. On the third day, the nurse noticed new skin rashes over extremities. He does not have any fever. Bloodwork shows normal coagulation parameters and platelets. However, glomerular filtration rate shows an acute drop to 32. In addition, transaminases are four times elevated from baseline. To what could one attribute these changes in his clinical picture?

Choices: 1. Progression of shock and multiorgan failure 2. Vasopressor toxicity with poor end-organ perfusion 3. Calcium chloride effect 4. Acute leptospirosis

Answer: 3 - Calcium chloride effect Explanations: The patient likely has a calcium channel blocker overdose, as his medical conditions of hypertension and Raynaud syndrome require treatment with a calcium channel blocker agent. He has no fever or leukocytosis and had normal renal and liver function. This is therefore unlikely an infection like leptospirosis Calcium chloride or gluconate are often used in the initial treatment of calcium channel blocker overdose. Overaggressive use in the first 24 hours is often seen and can rarely result in calciphylaxis acutely presenting with skin necrosis, rash, acute kidney injury, and abnormal liver function. Progression of multiorgan failure or vasopressor adverse effects is unlikely to develop now when his needs are slowly improving. Although hemodialysis is not indicated in the treatment of calcium channel blocker toxicity, it may be indicated with worsening toxicity of calciphylaxis and kidney injury in the setting of calcium channel blocker overdose. Go to the next page if you knew the correct answer, or click the link image(s) below to further research the concepts in this question (if desired).

Research Concepts: Calcium Channel Blocker Toxicity

We update eBooks quarterly and Apps daily based on user feedback. Please tap flag to report any questions that need improvement.

Question 285: A 65-year-old male was admitted to the hospital with fever, productive cough, and shortness of breath. Initial evaluation revealed leukocytosis with a left shift, and chest radiograph showed a left lower lobe infiltrate. He was started on appropriate intravenous antibiotics with oxygen support via nasal cannula. After 2 days of hospitalization, the patient experienced worsening respiratory distress with an increased requirement of oxygen, and he became more confused. Repeat chest radiograph showed the development of new-onset rightsided infiltrate and worsening of the left side infiltrate. Arterial blood gas, while the patient is breathing on 6 L/min via a face mask, showed a pH of 7.32, PaCO2 of 38 mmHg, and a PaO2 of 53 mmHg. What is the most appropriate next step in management?

Choices: 1. Transfer to the intensive care unit and start non-invasive ventilation with a serial assessment of the patient’s clinical condition 2. Transfer to intensive care unit, intubate and start volume assist-control ventilation with tidal volume 6 ml/kg of ideal body weight target plateau pressure of 30 cm H2O 3. Transfer to intensive care unit, intubate and start pressure support ventilation 4. Transfer to intensive care unit, intubate and start volume assist-control ventilation with tidal volume 10 ml/kg of ideal body weight, and target plateau pressure of 45 cm H2O.

Answer: 2 - Transfer to intensive care unit, intubate and start volume assist-control ventilation with tidal volume 6 ml/kg of ideal body weight target plateau pressure of 30 cm H2O

Explanations: Pneumonia is a common cause of acute respiratory distress syndrome (ARDS) in hospitalized patients. Early recognition of worsening clinical conditions and the development of ARDS is essential in the management of these patients. Patients who developed ARDS should be transferred to ICU, intubated and mechanical ventilation should be started with volume assist-control with a tidal volume of 6 ml/kg of ideal body weight with a target inspiratory plateau pressure of 30 cm of H2O to avoid barotrauma and further lung injury. Low tidal volume and low-pressure ventilation are associated with a significant reduction in mortality and various clinical outcomes when compared with the higher tidal volume of 12 ml/kg and higher plateau pressures of 45 cm H2O. Management of ARDS should also involve treatment of the underlying cause, prevention of complications associated with a hospital stay, including nosocomial infections, venous thromboembolism, stress ulcers, and aspiration. Go to the next page if you knew the correct answer, or click the link image(s) below to further research the concepts in this question (if desired).

Research Concepts: Respiratory Distress Syndrome

We update eBooks quarterly and Apps daily based on user feedback. Please tap flag to report any questions that need improvement.

Question 286: A 65-year-old patient is brought to the hospital by his family after he was found in bed, not talking, and not moving his right side. He is admitted to the hospital for a stroke work up. His initial head CT is negative, and the plan was to proceed to MRI brain and MRA of the head and neck since the patient had an iodine contrast allergy (rash and throat itching). While waiting to get an MRI, the patients mental status decompensates, and he becomes comatose requiring intubation. His exam off sedation reveals quadriplegia and no response to noxious stimuli. What is the most important next step in the management of this patient?

Choices: 1. Admit the patient to a neurological ICU and obtain an EEG of the brain to rule out subclinical seizures as the cause 2. Wait for the MRI brain and MRA head of the neck for the diagnosis 3. Repeat CT head and send the patient to cerebral angiography to evaluate for large vessel occlusion and possible treatment 4. Obtain a CTA of the head and neck

Answer: 3 - Repeat CT head and send the patient to cerebral angiography to evaluate for large vessel occlusion and possible treatment

Explanations: The patient presented with focal neurological deficits that progressed to coma and quadriplegia. This is a classical presentation of locked-in syndrome caused by basilar artery occlusion. The patients can have waxing and waning symptoms prior to the full onset of the syndrome. Also, the patients can have alternating hemiparesis, which is seen with tumbling embolism as goes up the basilar artery before it causes full occlusion and infarction. In this situation, the deficits happened acutely and would, therefore, need emergent intervention with an attempted thrombectomy. The iodine allergy should not have delayed the CTA in someone with suspected large vessel occlusion as these patients can be pre-medicated with steroids and antihistamines. The patient will eventually need to be admitted to the neurological ICU, and subclinical seizures are on the differential. However, this is not the most likely scenario as the patient did not have a generalized tonic-clonic seizure and the biggest clinical concern here is a basilar occlusion causing locked-in syndrome. If vessel evaluation is completed and unremarkable other etiologies such as infection, inflammatory, herniation or seizures should be considered. Waiting for the MRI brain and MRA head and neck can delay the diagnosis. These two studies can take up to 30-40 minutes to complete. Stroke diagnosis is made clinically and treatment is initiated before confirmation by MRI. Obtaining a CTA of the head and neck is reasonable. However, in this case, the suspected diagnosis is basilar occlusion, and delays in treatment should be avoided. Cerebral angiography can be diagnostic and therapeutic. Sending the patient directly to the angiography suite would avoid giving the patient contrast twice. Go to the next page if you knew the correct answer, or click the link image(s) below to further research the concepts in this question (if desired).

Research Concepts: Foville Syndrome

We update eBooks quarterly and Apps daily based on user feedback. Please tap flag to report any questions that need improvement.

Question 287: Assessing a child's neurologic function during an emergency can be challenging. However, there are standardized evaluation techniques that can be done quickly by any member of the healthcare team. They involve checking the child’s pupillary response to light and use of either the Alert, Verbal, Pain, Unresponsive (AVPU) pediatric response scale or the Glasgow coma scale (GCS). What are the three components of the GCS?

Choices: 1. eye opening, motor response, and memory 2. eye opening, motor response, and pain response 3. eye opening, verbal response, and motor response 4. memory, verbal response, and eye opening

Answer: 3 - eye opening, verbal response, and motor response Explanations: A Glasgow coma scale (GCS) score should be assigned to every child with significant head trauma. This scale assesses eye opening as well as motor and verbal responses. The GCS helps categorize neurologic disability, and serial measurements can help identify improvement or deterioration over time. The score is broken up into three sections with a maximum of 4 points for eye-opening, 5 for verbal response and 6 for motor response for a total score of 15. The higher the score, the better the chances of improvement and normal brain function. A child with severe neurological impairment with a GCS score of 8 or less should be intubated. The GCS provides a practical method for assessing consciousness in response to defined stimuli. Go to the next page if you knew the correct answer, or click the link image(s) below to further research the concepts in this question (if desired).

Research Concepts: Glasgow Coma Scale

We update eBooks quarterly and Apps daily based on user feedback. Please tap flag to report any questions that need improvement.

Question 288: A patient who is 8 weeks pregnant ate some unpasteurized cheese 2 days ago and now presents with muscle aches, diarrhea, low-grade fever, and malaise. She says the pregnancy has been uneventful, but she has persistent nausea all the time. Blood work and urinalysis are all normal. Examination of stools reveals a gram-positive organism with "tumbling motility." What is the most likely diagnosis?

Choices: 1. Hyperemesis gravidarum 2. Hepatitis A 3. Listeria 4. Escherichia coli

Answer: 3 - Listeria Explanations: Listeria is a gram-positive, facultative anaerobic, non-spore-forming bacillus with a characteristic "tumbling" motility. Listeria primarily affects older individuals, pregnant women, newborns, and immunosuppressed individuals. Individuals with listeriosis usually have fever and myalgia, sometimes preceded by diarrhea and other gastrointestinal symptoms. Unlike most bacteria, Listeria can grow and multiply in some foods in the refrigerator. Go to the next page if you knew the correct answer, or click the link image(s) below to further research the concepts in this question (if desired).

Research Concepts: Listeria Monocytogenes

We update eBooks quarterly and Apps daily based on user feedback. Please tap flag to report any questions that need improvement.

Question 289: A young female is brought to the emergency department with an acute onset of crushing substernal pain radiating to the left arm and shortness of breath. Significant past medical history includes hospitalization for a heart problem at age 3 years that was treated with aspirin and gamma globulin. Her electrocardiogram shows 3 mm of ST elevation in the anterior leads. She has followed up for yearly echocardiograms since this time. Select the most likely cause of the patient's symptoms.

Choices: 1. Vasospasm after cocaine ingestion 2. Thrombosis of a coronary artery aneurysm 3. Vasculitis of the left anterior descending artery 4. Dissection of the aortic root compromising blood flow into the left coronary ostium

Answer: 2 - Thrombosis of a coronary artery aneurysm Explanations: Kawasaki disease is a vasculitis of medium and small vessels that can lead to coronary artery aneurysms. Intravenous immunoglobulin (IVIG) treats the vasculitis and decreases the risk of aneurysm from 25% to 3%. These patients are also treated initially with high dose aspirin. Patients with large coronary artery aneurysms are treated with antithrombotic agents (lowmolecular-weight heparin or warfarin) and antiplatelet agents (aspirin, clopidogrel or dipyridamole). Most aneurysms regress, especially if fusiform, small and distal, but those greater than 8 mm can persist and develop rupture, thrombosis, recanalization, and stenosis at the outflow area. Cardiac stress testing for reversible ischemia is recommended for KD children with known coronary aneurysms. Go to the next page if you knew the correct answer, or click the link image(s) below to further research the concepts in this question (if desired).

Research Concepts: Kawasaki Disease

We update eBooks quarterly and Apps daily based on user feedback. Please tap flag to report any questions that need improvement.

Question 290: Purpura fulminans is the skin manifestation of disseminated intravascular coagulation, necrotizing fasciitis, and overwhelming sepsis. Aside from antibiotics, resuscitation and source control, what specific therapy can be started which directly addresses purpura fulminans?

Choices: 1. The 4-factor prothrombin complex concentrate (4F-PCC) 2. Activated protein C concentrate 3. Methylprednisolone sodium succinate 4. Recombinant activated clotting factor VII (rFVIIa)

Answer: 2 - Activated protein C concentrate Explanations: Purpura fulminans is an acute purpuric rash characterized by coagulation of the microvasculature which leads to purpuric lesions and skin necrosis. It is a true dermatological emergency and requires immediate diagnosis and management. Treating the underlying cause of purpura fulminans is the most important aspects of treatment. Early antibiotics, fluid resuscitation, and in certain cases early surgery are firstline therapy. Purpura fulminans is associated with a relative deficiency of protein C, an important regulator of blood coagulation, leading to consumption coagulopathy, intravascular fibrin deposition, downregulated fibrinolysis, and clotting of the microcirculation. Activated protein C is believed to decrease tissue damage and speed recovery. Use of activated protein C has been shown to improve mortality over predicted historical mortality, but no randomized controlled trials have been performed. Go to the next page if you knew the correct answer, or click the link image(s) below to further research the concepts in this question (if desired).

Research Concepts: Purpura Fulminans

We update eBooks quarterly and Apps daily based on user feedback. Please tap flag to report any questions that need improvement.

Question 291: A 56-year-old is admitted to the hospital with bleeding rectal varices. She is on the liver transplant list with a MELD score of 30 with a 3-month predicted mortality of 52.6%. Her Child-Pugh score is 8, making her a Child B with a 1-year mortality of 20%. Which of the following, if present, best explains the difference in mortality predicted by the two scoring systems?

Choices: 1. Creatinine of 6.0 mg/dL, on dialysis 2. Bilirubin 2.5 mg/dL 3. INR 1.8 4. Lack of ascites

Answer: 1 - Creatinine of 6.0 mg/dL, on dialysis Explanations: The MELD score incorporates renal function into the scoring system, including both serum creatinine and whether the patient is currently on dialysis. One main criticism of the MELD system is the heavy emphasis on renal function without incorporating factors such as ascites, encephalopathy, and nutrition status. Currently, the transplant allocation system uses the MELD score for organ allocation due to the wider range of continuous variables and lack of subjective scoring items. The main criticism of the Child-Pugh score is the use of subjective variables, smaller range of continuous variables, and exclusion of renal function. Go to the next page if you knew the correct answer, or click the link image(s) below to further research the concepts in this question (if desired).

Research Concepts: Use Of The Child Pugh Score In Liver Disease

We update eBooks quarterly and Apps daily based on user feedback. Please tap flag to report any questions that need improvement.

Question 292: A 58-year-old man with chronic obstructive pulmonary disease is brought to the emergency department for disorientation. He has been taking theophylline for six months, and the bottle is empty. His theophylline level is 51 mcg/mL. Which action should be taken first?

Choices: 1. Load immediately with phenobarbital 2. Give prochlorperazine to prevent vomiting and decrease the risk of aspiration 3. Administer 1 amp of calcium gluconate 4. Contact the on-call nephrologist to discuss arranging urgent hemoperfusion.

Answer: 4 - Contact the on-call nephrologist to discuss arranging urgent hemoperfusion. Explanations: Determining if a patient has an acute or chronic theophylline overdose is vital. Those with acute overdose are at higher risk for gastrointestinal and cardiac complications. Theophylline toxicity often present with seizures that are most effectively treated with a benzodiazepine, not a long-acting barbiturate such as phenobarbital. Additional medications with anticholinergic or antihistaminic properties, such as prochlorperazine, should be avoided. Neurologic complications are more likely in chronic overdose situations. This patient likely has chronic ingestion, and levels greater than 40 mcg/mL should be considered for urgent hemoperfusion. Go to the next page if you knew the correct answer, or click the link image(s) below to further research the concepts in this question (if desired).

Research Concepts: Theophylline Toxicity

We update eBooks quarterly and Apps daily based on user feedback. Please tap flag to report any questions that need improvement.

Question 293: A 47-year-old male with no relevant past medical history presents to the hospital complaining of dyspnea on exertion and chest pain. He states symptoms started one week ago and have progressively worsened. Upon further questioning, he also reports an episode of flu-like symptoms 4 weeks ago. He lives in central Pennsylvania, owns a farm, and spends most of his time working on the field. An electrocardiogram taken in the emergency department shows S-T depression of the anterior leads, but two sets of troponins have been negative. Lyme myocarditis is suspected. What would a transthoracic echocardiogram most likely show?

Choices: 1. Severe mitral valve regurgitation 2. Diffuse ventricular hypokinesis 3. Focal wall motion abnormalities 4. Increased right ventricular pressure

Answer: 3 - Focal wall motion abnormalities Explanations: Lyme myocarditis may present with symptoms and electrocardiographic findings similar to acute coronary syndrome. T wave inversion or ST-segment depression are usually seen; however, high troponin level and/or ST-segment elevation is uncommon. In acute coronary syndrome, echocardiography reveals diffuse ventricular hypokinesis. Echocardiography will show focal wall motion abnormalities in myocarditis. Go to the next page if you knew the correct answer, or click the link image(s) below to further research the concepts in this question (if desired).

Research Concepts: Lyme Carditis

We update eBooks quarterly and Apps daily based on user feedback. Please tap flag to report any questions that need improvement.

Question 294: A 50-year-old patient with chronic alcohol used disorder presents with the report of drinking windshield washer fluid after running out of vodka. He smells of alcohol and is intoxicated but is otherwise asymptomatic with normal vital signs. An arterial blood gas reveals a pH of 7.3 and bicarbonate of 22 mmol/L. His ethanol concentration is 200 mg/dL. A methanol concentration will take 24 hours to result. What is the next most appropriate step?

Choices: 1. Administer fomepizole 2. Perform serial basic metabolic panels monitoring for acidosis starting now 3. Admit to the intensive care unit 4. Perform serial metabolic panels monitoring for acidosis starting in 8 to 10 hours.

Answer: 4 - Perform serial metabolic panels monitoring for acidosis starting in 8 to 10 hours. Explanations: The patient is intoxicated with ethanol. He is not at risk of developing an acidosis from metabolizing any methanol while ethanol is competing for alcohol dehydrogenase. Ethanol's elimination rate is about 15 to 25 mg/dL/hr, depending on the individual. At approximately 20 mg/dL/hr, it can be anticipated that the patient's ethanol concentration will be approaching 0 in 8 to 10 hours. At that time, metabolism of methanol will begin and monitoring for acidosis should occur. Monitoring should continue for 12 hours after ethanol clears. Fomepizole should not be given empirically. In this situation, it would prolong the patient's length of stay as it prolongs the half-life of ethanol. Once ethanol clears, fomepizole should be given if an acidosis occurs. Provided the patient's mental status and vital signs are normal, the patient does not require intensive care monitoring. Go to the next page if you knew the correct answer, or click the link image(s) below to further research the concepts in this question (if desired).

Research Concepts: Methanol Toxicity

We update eBooks quarterly and Apps daily based on user feedback. Please tap flag to report any questions that need improvement.

Question 295: A 60-year-old male with a history of stable coronary artery disease, hypertension, hyperlipidemia, and constipation presents with complaints of a large amount of painless bright red bleeding per rectum for the last two days, which appears to have now subsided. He is lightheaded upon standing and appears somewhat pale. You suspect he has a large diverticular bleed. Supportive treatment is initiated. Intravenous access is obtained, and resuscitative fluids started. CBC shows a hemoglobin of 5 g/dL. What is the recommended goal hemoglobin level in this patient?

Choices: 1. Greater than 6 g/dL 2. Greater than 7 g/dL 3. Greater than 8 g/dL 4. Greater than 9 g/dL

Answer: 2 - Greater than 7 g/dL Explanations: The American College of Physicians recommends a restrictive transfusion threshold with a goal hemoglobin greater than 7 g/dL in anemic patients. Multiple studies have shown that a liberal transfusion strategy with a goal hemoglobin greater than 10 g/dL confers no mortality benefit and may be harmful to patients. The American College of Gastroenterology also recommends transfusing packed red cells to maintain a hemoglobin level above 7 g/dL but says a threshold of 9 g/dL should be considered in patients with massive bleeds, active ischemia, or delay in receiving therapeutic interventions. Transfusion strategies and thresholds do not differentiate between upper and lower gastrointestinal bleeds. Go to the next page if you knew the correct answer, or click the link image(s) below to further research the concepts in this question (if desired).

Research Concepts: Gastrointestinal Bleeding

We update eBooks quarterly and Apps daily based on user feedback. Please tap flag to report any questions that need improvement.

Question 296: A female patient presents with bilateral hydroureteronephrosis and acute renal failure secondary to retroperitoneal fibrosis. Bilateral percutaneous nephrostomies are placed by interventional radiology. Her urine output has been greater than 200 mL/hour for the past four hours, and she is tolerating oral intake well at this time. What is the best next step in management?

Choices: 1. With the patient’s excellent urine output, nothing more needs to be done. 2. Order frequent basic metabolic panels (BMPs) to monitor the patient’s electrolytes and renal function. 3. Order frequent BMPs to monitor the patient’s electrolytes and renal function; instruct the nurse to have the patient replace the fluid loss with oral hydration if the patient can tolerate and keep up with losses. 4. Order frequent BMPs to monitor the patient’s electrolytes and renal function; replace the fluid loss with an equal volume of intravenous half normal saline.

Answer: 3 - Order frequent BMPs to monitor the patient’s electrolytes and renal function; instruct the nurse to have the patient replace the fluid loss with oral hydration if the patient can tolerate and keep up with losses.

Explanations: After renal decompression, the patient must be monitored closely for post-obstructive diuresis, renal function status, and appropriate replacement of fluids and electrolytes. Post-obstructive diuresis is a prominent polyuria that occurs immediately after relieving severe urinary obstruction. Patients are often fluid overloaded initially, and urine output often exceeds 200 mL/hr. Patients are at risk for severe dehydration, electrolyte imbalances, and hypovolemic shock. The condition usually lasts less than 48 hours. Treatment includes close monitoring of serum BUN, creatinine, and electrolytes, and replacement of fluid. Oral replacement is preferred, but if the patient cannot tolerate oral liquids or is unable to keep up with the loss, intravenous half normal saline can be used to replace half of the fluid loss (500 mL for every 1 liter lost). After initial management, it is necessary to identify the potential cause of retroperitoneal fibrosis. Any potential inciting drug should be discontinued immediately. Go to the next page if you knew the correct answer, or click the link image(s) below to further research the concepts in this question (if desired).

Research Concepts: Retroperitoneal Fibrosis

We update eBooks quarterly and Apps daily based on user feedback. Please tap flag to report any questions that need improvement.

Question 297: A 57-year-old male with alcoholic liver cirrhosis presents to the emergency department after 2 days of hematemesis. He currently is not taking any medications. A previous chart shows esophageal varices listed in the history. Today's CBC shows a hemoglobin of 5.8 g/dL and hematocrit of 17.8%. The prothrombin time and INR are 22.5 and 1.7, respectively. One unit of packed red blood cells and one unit of fresh frozen plasma are ordered to transfuse. Within 3 hours of transfusion, the patient becomes dyspneic, and the lungs sound very congested. His blood pressure is 95/62 mmHg and heart rate is 112 beats/min. An emergent chest x-ray and arterial blood gas are ordered. What is the best next step?

Choices: 1. Intubate the patient 2. Stop the transfusion and notify the blood bank 3. Continue the transfusion and give IV furosemide 40 mg emergently 4. Slow the rate of blood transfusion

Answer: 2 - Stop the transfusion and notify the blood bank Explanations: The best treatment for transfusion-related acute lung injury (TRALI) is to stop the transfusion. The blood transfusion will worsen the patient's condition. TRALI usually is associated with plasma components such as platelets and fresh frozen plasma. Supportive care is the mainstay of TRALI treatment. Oxygen supplementation is needed. Also, IV fluids and vasopressors are needed for blood pressure support. Intubating the patient and protecting the airway is next step if the patient's respiratory status worsens. Diurectics should be avoided in TRALI treatment. Go to the next page if you knew the correct answer, or click the link image(s) below to further research the concepts in this question (if desired).

Research Concepts: Transfusion-related Acute Lung Injury

We update eBooks quarterly and Apps daily based on user feedback. Please tap flag to report any questions that need improvement.

Question 298: A 66-year-old female is admitted for a five days history of fever, bloody diarrhea, and abdominal pain. The patient has acute myeloid leukemia, and the last chemotherapy she received was two weeks ago as an inpatient. While the patient was in the hospital two weeks ago, she had a UTI and was treated with IV antibiotics. On physical examination, her temperature is 38.5 C (101.3 F), blood pressure 95/50 mmHg, pulse rate 103/min, and respiratory rate is 15/min. The abdominal exam shows diffuse tenderness. Imaging showed thickened bowel wall of the cecum and ascending colon, fat stranding, and air under the diaphragm. Her labs show hemoglobin (Hb) 7 mg/dL, white blood cells (WBC) 300/micoL, and platelets 15000/microL. What is the next best step in the management of this patient?

Choices: 1. Stool analysis and culture 2. Bowel rest and Nasogastric suctioning 3. Evaluation for surgery 4. Oral vancomycin

Answer: 3 - Evaluation for surgery Explanations: The patient presented with symptoms of colitis (fever, abdominal pain, and diarrhea) and in such patient stool analysis and culture is part of the workup. However, the patient has concerning signs, including the hemodynamic instability and the air under diaphragm seen on CT imaging. So the most important next step in management is to be evaluated by surgery for possible bowel perforation. Bowel rest and NG suctioning is part of the management of any patient with colitis, but the most appropriate next step the patient needs is to be evaluated by surgery for early intervention. She might have had neutropenic enterocolitis or colitis due to other infectious etiology. Regardless of the cause, the patient has hemodynamic instability, and CT imaging shows a sign of bowel perforation (air under diaphragm) and surgical evaluation is needed to do intervention as soon as possible to decrease complications and mortality. The patient has colitis of which the cause is still unknown. Clostridium difficile testing should be done as part of the workup, and if positive, she will be given oral vancomycin. However, the patient has complicated colitis that needs to be evaluated by surgery urgently. Go to the next page if you knew the correct answer, or click the link image(s) below to further research the concepts in this question (if desired).

Research Concepts: Neutropenic Enterocolitis (Typhlitis)

We update eBooks quarterly and Apps daily based on user feedback. Please tap flag to report any questions that need improvement.

Question 299: A 17-year-old female presents to the emergency department with a fever and a headache. She has no significant past medical history. Her temperature is measured to be 39 C while blood pressure is 102/74 mmHg. Physical examination shows petechiae on her legs. Neck stiffness is present as well. Lumbar puncture is performed after which appropriate antibiotics and fluids are started. The next day, she develops severe back pain, perineal anesthesia and is unable to urinate. An MRI confirms spinal cord compression. Which of the following is the most likely cause of the compression in this patient?

Choices: 1. Spinal abscess 2. Epidural hematoma 3. Disc herniation 4. Tumor

Answer: 2 - Epidural hematoma Explanations: Cauda equina syndrome (CES) is a known complication of lumbar puncture procedures involving direct trauma or an epidural hematoma. Medicolegal litigation is high in these syndromes due to missed diagnoses and a high degree of residual functional impairment. Surgical decompression within 48 hours is paramount in CES and conus medullaris syndrome. Saddle anesthesia is one of the most common presenting symptoms. The most common cause of cauda equina syndrome is disk herniation. Go to the next page if you knew the correct answer, or click the link image(s) below to further research the concepts in this question (if desired).

Research Concepts: Cauda Equina And Conus Medullaris Syndromes

We update eBooks quarterly and Apps daily based on user feedback. Please tap flag to report any questions that need improvement.

Question 300: A patient arrives with severe shoulder pain for one day. There is redness and swelling and limited range of motion but no history of trauma. The patient has a fever and looks toxic. While waiting for his x-ray, the patient notices the skin has become numb to the touch and bulla has started to form on the affected area. Examination shows irregular hemorrhagic necrosis. What is the name of this characteristic rash?

Choices: 1. Purpura fulminans 2. Erysipelas 3. Bullous pemphigoid 4. Staphylococcal scalded skin syndrome

Answer: 1 - Purpura fulminans Explanations: Purpura fulminans is a disorder characterized by rapidly progressing skin lesions with a typical morphology. As this disease is associated with severe illness and rapid progression, it is important to keep a high degree of suspicion. The skin findings of purpura fulminans have a characteristic appearance and evolution. Purpura fulminans begins with erythema which develops irregular hemorrhagic necrosis and in some cases, vesicles and bulla form. The affected skin initially is painful and indurated but in the later stages, there may be total loss of sensation. As the later stages of purpura fulminans may include large areas of necrosis, close monitoring and multiple surgeries for debridement are often required. Go to the next page if you knew the correct answer, or click the link image(s) below to further research the concepts in this question (if desired).

Research Concepts: Purpura Fulminans

We update eBooks quarterly and Apps daily based on user feedback. Please tap flag to report any questions that need improvement.

Section 4 Question 301: A 17-year-old male who lives in a homeless shelter is brought to the emergency department by emergency medical services for significant respiratory distress. He has a history of alcohol use disorder and intravenous drug use. He has had hemoptysis for several weeks. He is admitted to the intensive care unit and intubated for respiratory failure. A chest xray shows a left-sided, moderate size pleural effusion with mediastinal shift and perihilar lymphadenopathy. What is the most probable cause of his pleural effusion?

Choices: 1. Haemophilus influenzae 2. Staphylococcus aureus 3. Mycobacterium tuberculosis 4. Pseudomonas aeruginosa

Answer: 3 - Mycobacterium tuberculosis Explanations: Pleural effusion is the accumulation of fluid in the pleural cavity. The mechanism by which fluid accumulates in the pleural cavity includes infection producing exudates, increased capillary permeability causing the production of exudative fluid in the pleural cavity, and increased hydrostatic pressure or decreased oncotic pressure leading to the formation of transudates. In this vignette, all of the organisms listed can cause pleural effusion. The combination of the patient being homeless and using intravenous drugs make him more likely to have tuberculosis. The most common cause of pleural effusion in the pediatric population is infection. The infectious agents that can cause pleural effusion include Streptococcus pneumoniae, Hemophilus influenza type B, Staphylococcus aureus, Mycobacterium tuberculosis, and Pneumocystis jiroveci in immunocompromised patients. Diagnosis can be confirmed by an acid-fast bacilli (AFB) smear and culture, interferongamma release assay, or pleural fluid adenosine deaminase. According to the Centers for Disease Control and Prevention, culture remains the criterion standard for laboratory confirmation of tuberculosis disease. Go to the next page if you knew the correct answer, or click the link image(s) below to further research the concepts in this question (if desired).

Research Concepts: Parapneumonic Pleural Effusions And Empyema Thoracis

We update eBooks quarterly and Apps daily based on user feedback. Please tap flag to report any questions that need improvement.

Question 302: A 40-year-old woman with fevers, cough, and productive sputum for five days, is admitted to the intensive care unit requiring intubation and mechanical ventilation. She presented to the emergency department with acute respiratory distress syndrome (ARDS) secondary to pneumonia. Oxygenation saturation had been satisfactory on an FIO2 of 50% and positive end-expiratory pressure (PEEP) of 6 cm H2O but dropped to the low 80% despite an increase in FIO2 to 100%. On physical examination, the patient is intubated and sedated. Lung exam reveals diffuse rales and rhonchi. The examination of the heart is unremarkable except for tachycardia (heart rate of 115 beats/min). She has 2+ peripheral edema. A chest x-ray shows diffuse bilateral infiltrates. She is being ventilated using an assist/control mode with a tidal volume of 6 mL/kg and a plateau pressure of 25 cm H2O. Which of the following is the most appropriate strategy for PEEP in this patient?

Choices: 1. Increase PEEP in 2- to 3-cm H2O increments and lower FIO2 to at most 60%, if possible. Arterial oxygen saturation of at least 88% and 95% or less must be maintained 2. Set the PEEP below the lower inflection point on a pressure-volume curve of the lung 3. Maintain PEEP to correspond to the expiratory pressure that minimizes compliance of the lung 4. Maintain PEEP of at least 14 cm H2O and increase up to 20 cm H2O for FIO2 of 0.5 to 0.8. Monitor cardiac output using a pulmonary artery catheter

Answer: 1 - Increase PEEP in 2- to 3-cm H2O increments and lower FIO2 to at most 60%, if possible. Arterial oxygen saturation of at least 88% and 95% or less must be maintained

Explanations: In this patient with ARDS secondary to pneumonia, increasing PEEP in 2- to 3-cm H2O increments and subsequently lowering FIO2 to at most 60% while maintaining an arterial oxygen saturation of at least 88% and 95% or less is appropriate. This strategy has been used by ARDSnet and associated with improved outcomes. It is a "lung protective strategy". Lower PEEP is associated with reduced barotrauma. There is no utility for a pulmonary artery catheter in the management of PEEP. Go to the next page if you knew the correct answer, or click the link image(s) below to further research the concepts in this question (if desired).

Research Concepts: Pulmonary Edema

We update eBooks quarterly and Apps daily based on user feedback. Please tap flag to report any questions that need improvement.

Question 303: A 55-year-old male is presenting for follow up after recently being diagnosed with non-small cell lung cancer. He has no new complaints on this visit. No prior medical or surgical history and does not take any medications. He continues to smoke one pack per day for past 30 years. Staging workup for his cancer reveals a normal MRI of the brain. Contrast-enhanced CT of abdomen and pelvis does not reveal any evidence of distant metastasis but a left renal vein thrombus extending to the inferior vena cava. Both kidneys appear normal in size. Routine labs including complete blood count, renal and liver function panel are unremarkable. INR is 1. Urinalysis and microscopy are unremarkable. Which is the best treatment option for the patient’s renal vein thrombus?

Choices: 1. Aspirin 81mg and clopidogrel 75 mg daily 2. Enoxaparin 1mg/kg Q12 hours 3. Rivaroxaban 4. No treatment is necessary

Answer: 2 - Enoxaparin 1mg/kg Q12 hours Explanations: Patients with asymptomatic renal vein thrombosis are best treated with anticoagulation. The reason is to prevent progression of the thrombus or development of a new thromboembolic event such as pulmonary embolism. Initial treatment consists of starting unfractionated or low-molecular-weight heparin and then bridged to warfarin, for a goal INR of 2 to 3. Anticoagulation is generally given for 6 to 12 months Novel oral anticoagulants (NOAC) are not approved for the treatment of renal vein thrombosis. Go to the next page if you knew the correct answer, or click the link image(s) below to further research the concepts in this question (if desired).

Research Concepts: Renal Vein Thrombosis

We update eBooks quarterly and Apps daily based on user feedback. Please tap flag to report any questions that need improvement.

Question 304: A 47-year-old female patient with a history of antiphospholipid antibody syndrome who has been noncompliant with warfarin presents with a deep vein thrombosis and dyspnea. On examination, her blood pressure is 90/60 mmHg, respiratory rate is 22/min, and her heart rate is 92/min. Computerized tomography of the chest shows a saddle embolus. She does not respond to heparin and fluids. An echocardiogram shows right ventricular hypokinesis. What is the most appropriate next step?

Choices: 1. Urgent referral for surgical embolectomy 2. Recombinant tissue plasminogen activator 3. Continue administration of fluids and heparin 4. Add lepirudin

Answer: 2 - Recombinant tissue plasminogen activator Explanations: Anticoagulation alone is an inadequate treatment for this patient. Current guidelines for the management of unstable patients recommend immediate treatment with anticoagulation. Fibrinolytic treatment with recombinant tissue plasminogen activator is indicated unless there are major contraindications. If the patient does not improve or fibrinolysis is not possible, the best choice is surgical embolectomy. Surgery is the last resort and never the first consideration. Placing such a patient on bypass is associated with mortality in excess of 50%. Furthermore, even those who survive are faced with several complications like acute respiratory distress syndrome (ARDS), bleeding, and shock. After surgery, the patient will need lifelong treatment with warfarin. Today, tPA is recommended as the treatment of choice. A dose of 100 mg is given over 2 hours, and there is a rapid resolution of the elevated pulmonary artery pressure. Go to the next page if you knew the correct answer, or click the link image(s) below to further research the concepts in this question (if desired).

Research Concepts: Antiphospholipid Syndrome

We update eBooks quarterly and Apps daily based on user feedback. Please tap flag to report any questions that need improvement.

Question 305: A 23-year-old male with no significant past medical history is brought to the emergency department after being involved in a motor vehicle collision. He is found to have multiple injuries, including bilateral rib fractures, left pneumohemothorax, splenic injury, and cervical vertebral fractures. He is emergently intubated, and a left-sided chest tube is placed. He is admitted to the intensive care unit for further care. Two days later, the patient develops worsening hypoxemia. A chest X-ray shows worsening bilateral infiltrates compared to the previous day, and there is no effusion. The chest tube has been draining about 100 mL of serosanguinous fluid for the past two days. He is placed on a fraction of inspired oxygen of 1.0, positive end-expiratory pressure 15 mmHg, low tidal volume ventilation 6 ml/kg, and respiratory rate 30/min. Peak pressure is 40 cmH20, and plateau pressure is 38 cmH20. Arterial blood gas analysis on these settings shows pH 7.20, pO2 60 mmHg, and pCO2 68 mmHg. What is the best next step in management?

Choices: 1. Increase the tidal volume to facilitate the clearance of carbon dioxide 2. Initiate prone ventilation 3. Initiate venovenous extracorporeal membrane oxygenation (VV-ECMO) 4. Computed tomography angiography of the chest

Answer: 3 - Initiate venovenous extracorporeal membrane oxygenation (VV-ECMO) Explanations: In patients with respiratory distress, the mainstay of therapy is low tidal volume ventilation of 4-6 ml/kg of ideal body weight while maintaining plateau pressures of less than 30 cmH20 and permissive hypercapnia with an acceptable pH. If the patient is not a candidate for or fails other therapies for acute respiratory distress syndrome, the next step is venovenous extracorporeal membrane oxygenation (VV-ECMO). In patients with a purely gas exchange problem, whether it is hypoxemia or inability to clear carbon dioxide, VV-ECMO is indicated as opposed to venoarterial ECMO unless the patient has evidence of cardiogenic shock or pulmonary hypertension with a failing right ventricle. Prone ventilation is contraindicated in patients with unstable vertebral fractures; therefore, VV-ECMO is the best next step in this patient. In the absence of active bleeding, patients will need to be anticoagulated while on ECMO to prevent thrombosis. Hemorrhage is one of the complications of ECMO, and therefore, the risks of bleeding and thrombosis should be weighed on an individual basis. Go to the next page if you knew the correct answer, or click the link image(s) below to further research the concepts in this question (if desired).

Research Concepts: Extracorporeal Membrane Oxygenation In Adults

We update eBooks quarterly and Apps daily based on user feedback. Please tap flag to report any questions that need improvement.

Question 306: An 18-year-old with intellectual disability and cerebral palsy, who is tracheostomy and ventilator dependent had two days of increased secretions with color change to green. The worried parents bring him to the emergency department (ED) with one day of increased ventilator settings and oxygen requirement. In the ED his temperature is 102.5 F, heart rate of 140 beats/minute, blood pressure of 75/50 mmHg, and the oxygen saturation is 82% on room air. A chest x-ray shows a right lower lobe infiltrate. The mother states that he recently finished a 14-day course of levofloxacin. Blood and tracheal cultures are obtained, and it is decided to start antibiotic treatment. His last tracheal culture which was obtained five months ago grew methicillin-resistant Staphylococcus aureus (MRSA). Based on this information which of the following antibiotic/antibiotic combinations will provide adequate coverage while awaiting the blood and tracheal cultures?

Choices: 1. Vancomycin 2. Clindamycin 3. Vancomycin plus piperacillin-tazobactam plus gentamicin 4. Cefepime

Answer: 3 - Vancomycin plus piperacillin-tazobactam plus gentamicin Explanations: Empiric treatment for ventilator-associated pneumonia should include coverage against methicillin-resistant Staphylococcus aureus, Pseudomonas, and other gram negatives. Earlier treatment reduces the risk of complications. However, treatment should be tailored based on the identification of the culprit organisms on tracheal and blood cultures because prolonged broad-spectrum antibiotics are associated with development of resistant infections as well as side effects. This patient is presenting in septic shock and was on antibiotics over the past three months, which are regarded as a risk factor for starting broad-spectrum antibiotics. Other risk factors include acute respiratory distress syndrome, five days or more days of hospitalization before development of ventilator-associated pneumonia, acute renal replacement therapy before development of ventilator-associated pneumonia. In addition, one should use the local antibiotic susceptibility pattern as well as the patient's past microbiology data. In a patient with more than one risk factors for a multidrug-resistant organism, the empiric treatment should cover methicillin-resistant Staphylococcus aureus with either vancomycin or linezolid plus two anti-Pseudomonas agents such as piperacillin-tazobactam, cefepime, meropenem, and aztreonam plus an aminoglycoside such as gentamicin, amikacin, or tobramycin. In the absence of risk factors, only one anti-Pseudomonal agent should be used. Monotherapy with aminoglycosides should be avoided. The combination of antibiotics should be given intravenously initially. It should be tailored once the antibiotic susceptibility results return. The decision to tailor antibiotics should also take into consideration the progression of clinical and laboratory markers of infection such as fevers, white blood cell, and procalcitonin. Go to the next page if you knew the correct answer, or click the link image(s) below to further research the concepts in this question (if desired).

Research Concepts: Ventilator-associated Pneumonia

We update eBooks quarterly and Apps daily based on user feedback. Please tap flag to report any questions that need improvement.

Question 307: A 31-year-old male presents to the emergency department with a high fever for the past two days, vomiting, back pain, and dyspnea. The patient was staying in a rural cabin noted to have rodents in North Korea and recently returned. Upon admitting the patient to the intensive care unit for respiratory distress, thrombocytopenia, hypotension, and marked leukocytosis with premature white blood cells, the patient asks if there is a point that will predict a good outcome. What phase would predict good long-term recovery?

Choices: 1. Oliguric 2. Polyuric 3. Convalescent 4. Febrile

Answer: 2 - Polyuric Explanations: Hantavirus hemorrhagic fever and renal syndrome is found to have 5 distinct phases: febrile, hypotensive, oliguric, polyuric, and convalescent. The febrile period lasts around 3 to 7 days which can be accompanied by conjunctival hemorrhages and fine palatal petechiae in the mouth. This phase is followed by the hypotensive phase which yields one-third of the mortality of hemorrhagic fever and renal syndrome. The oliguric phase may need treatment with dialysis if renal function deteriorates significantly. One-half of the fatalities in hemorrhagic fever and renal syndrome caused by Hantavirus occur in this phase. The onset of the polyuric phase is a positive prognostic indicator. The convalescent phase is a return towards the patient's clinical and laboratory baseline that may take up to 6 months. Go to the next page if you knew the correct answer, or click the link image(s) below to further research the concepts in this question (if desired).

Research Concepts: Hantavirus Syndrome

We update eBooks quarterly and Apps daily based on user feedback. Please tap flag to report any questions that need improvement.

Question 308: A 17-year-old male presents to the emergency department with a 2-day history of fevers, headache, photophobia, and neck pain. Of note, the patient is 14 days status post craniotomy for a gunshot wound to the head. On exam, the patient is lethargic but oriented to person, place, and time. Workup is significant for a white blood cell count of 26,000 cells/microL and a lactate of 5.2 mmol/L. What should be the initial antibiotic therapy for this patient?

Choices: 1. Vancomycin 20 mg/kg and cefepime 2 g intravenously 2. Vancomycin 20 mg/kg, ceftriaxone 2 g, and ampicillin 2 g intravenously 3. Vancomycin 20 mg/kg and ceftriaxone 2 g intravenously 4. Vancomycin 20 mg/kg and piperacillin-tazobactam 4.5g intravenously

Answer: 1 - Vancomycin 20 mg/kg and cefepime 2 g intravenously Explanations: Cefepime is a fourth-generation cephalosporin with good Pseudomonas coverage. It is recommended in the setting of meningitis with recent neurosurgery or penetrating head trauma. Its structure allows better and more rapid penetration through the cell wall of aerobic gramnegative bacilli. Ampicillin is primarily used for Listeria coverage. Listeria should be suspected in the neonatal and elderly populations Go to the next page if you knew the correct answer, or click the link image(s) below to further research the concepts in this question (if desired).

Research Concepts: Bacterial Meningitis

We update eBooks quarterly and Apps daily based on user feedback. Please tap flag to report any questions that need improvement.

Question 309: A 17-year-old male presents to the emergency department via emergency medical services (EMS) with report of combativeness and encephalopathy that required chemical sedation and intubation en route. On arrival, he was tachycardic with a heart rate of 124 bpm, hypertensive at 190/110 mmHg, with mydriatic pupils and diaphoresis. Before administration of further sedation, he became combative and had a seizure. His friend that called EMS reports that they were only smoking cannabis. Your evaluations include a negative urine drug screen. Abuse of which agent is most consistent with this presentation.

Choices: 1. Cocaine 2. Synthetic cannabinoids 3. Cannabis 4. Atropa belladonna

Answer: 2 - Synthetic cannabinoids Explanations: The patient's presentation is most consistent with abuse of synthetic cannabinoids. Synthetic cannabinoids are sold as plant or plant-like substances that are laced with chemicals that are designed to agonize cannabinoid receptors. These chemicals are vast and newer ones are constantly engineered. Synthetic cannabinoids are known to cause similar effects as cannabis but are often associated with increased toxicity, including encephalopathy, seizures, and death. Urine drug screens do not test for synthetic cannabinoids. Go to the next page if you knew the correct answer, or click the link image(s) below to further research the concepts in this question (if desired).

Research Concepts: Cannabinoid Toxicity

We update eBooks quarterly and Apps daily based on user feedback. Please tap flag to report any questions that need improvement.

Question 310: A 48-year-old male with no significant past medical history came to the emergency room with a chief complaint of palpitations. EKG showed atrial fibrillation with a rapid ventricular rate. Intravenous diltiazem was given with improvement in rate control. The urine drug screen was positive for amphetamines. Transesophageal echocardiogram showed no atrial thrombus and electrical cardioversion was subsequently performed. After successful cardioversion, sotalol was started. An EKG showed QT duration of 400 ms, potassium 4.2 mEq/L, and magnesium 2.3 mEq/L. How long should he stay in the hospital for monitoring?

Choices: 1. He can go home today after two hours after the first dose of sotalol 2. One day after staring sotalol if serial EKGs are normal 3. He cannot be discharged due to the positive drug screen 4. Three days monitoring is required in the hospital with serial EKGs

Answer: 4 - Three days monitoring is required in the hospital with serial EKGs Explanations: Sotalol is recommended younger patients post cardioversion for maintenance for sinus rhythm due to its superior side effect profile as compared with amiodarone. QT prolongation is one of the most important adverse effects of sotalol as it also has class III effect along with beta-adrenergic blockade properties. QT prolongation can lead to torsade de pointes which is a lethal arrhythmia if not treated. To prevent these adverse events it is recommended to observe the patient in the hospital for three days with serial EKGs to monitor QT. Go to the next page if you knew the correct answer, or click the link image(s) below to further research the concepts in this question (if desired).

Research Concepts: Sotalol

We update eBooks quarterly and Apps daily based on user feedback. Please tap flag to report any questions that need improvement.

Question 311: The patient is a 75-year-old male with a past medical history of hypertension, diabetes mellitus, coronary artery disease status post coronary artery bypass grafts, and history of inferior vena cava (IVC) filter placement who presents to the emergency department with a chief complaint of shortness of breath. Physical examination reveals lungs that are clear to auscultation bilaterally, cardiac exam shows regular rate and rhythm, normal S1, S2, no murmurs. EKG was obtained showing sinus tachycardia. CT of the chest with contrast was obtained showing bilateral pulmonary emboli. The patient was started on intravenous heparin. What is the next step in management?

Choices: 1. Doppler ultrasound bilateral lower extremities 2. CT abdomen and pelvis 3. Abdominal ultrasound evaluating the IVC 4. MRI of abdomen and pelvis

Answer: 3 - Abdominal ultrasound evaluating the IVC Explanations: IVC thrombosis can be a bland thrombosis which is an isolated thrombus most commonly arising from a deep venous thrombosis or due to a current IVC filter in place. IVC filter thrombus can range from small, clinically insignificant to an extensive clot extending to the lower extremities or causing complete occlusion of the IVC, ultimately leading to stasis. It is important to determine if the IVC filter is a permanent or retrievable device. If it is retrievable many times, intravascular thrombectomy will be performed, and the IVC filter will be removed. If the IVC filter thrombus is extensive, it can go on to cause the postthrombotic syndrome. The patient will present with lower extremity pain and edema, claudication and stasis ulcers. It can also progress to potential pulmonary emboli which could be fatal. There have been studies where the IVC filters were retrieved, and thrombosis ranges from 0.6 to 8%. There is thought that this percentage may be higher as this study was only on IVC filters that were retrieved after one year, it did not assess the older permanent filters that were placed. It is known that IVC filter placement is associated with increased risk for thrombus development within the filter, ultimately leading to pulmonary emboli. The ultrasound imaging will show echogenic material within its lumen, with an incomplete filling of color when Doppler is applied to the vessel. Pharmacologic anticoagulation is normally the mainstay treatment, but many patients with IVC filters are unable to have anticoagulation, and therefore endovascular therapies are considered. If there is a question on the extent of a clot within the filter, endovascular ultrasound can better evaluate the degree of stenosis and thrombus size to help target management. Endovascular management consists of the catheter directly thrombolysis or thrombectomy, balloon angioplasty, and stent placement if possible removal of the filter is considered. Go to the next page if you knew the correct answer, or click the link image(s) below to further research the concepts in this question (if desired).

Research Concepts: Inferior Vena Caval Thrombosis

We update eBooks quarterly and Apps daily based on user feedback. Please tap flag to report any questions that need improvement.

Question 312: A 42-year-old male is admitted to the hospital for increasing shortness of breath with palpitations. The patient requires supplemental oxygen at this time, and his chest xray on admission is suspicious for bilateral alveolar infiltrates. On admission, it was also noted that he had been admitted multiple times in the past for similar findings and symptoms. On his last admission, he had undergone bronchoscopy with bronchoalveolar lavage which showed hemosiderin filled alveolar macrophages. He was discharged home on a steroid taper. Now he has been found to have new-onset atrial fibrillation with rapid ventricular response. His blood pressure is 155/85 mmHg, heart rate is ranging 120s to 140s per minute, respiratory rate is 19/min, and his oxygen saturation is 98% on 2 L nasal cannula. Which of the following medications has the strongest relative contraindication in this patient?

Choices: 1. Diltiazem 2. Amiodarone 3. Furosemide 4. Metoprolol

Answer: 2 - Amiodarone Explanations: Pulmonary hemosiderosis is typically grouped into three categories based on disease characteristics. Group 1 pulmonary hemosiderosis is defined by pulmonary hemorrhage associated with circulating anti-GBM antibodies. Group 2 pulmonary hemosiderosis is defined by pulmonary hemorrhage with immune complex disease. Group 3 pulmonary hemosiderosis (idiopathic pulmonary hemosiderosis [IPH]) is defined as pulmonary hemorrhage without immunologic association. As previously noted, repeated episodes of diffuse alveolar hemorrhage result in accumulation of iron in the form of hemosiderin inside pulmonary macrophages. These recurrent episodes also lead to thickening of alveolar basement membranes and interstitial fibrosis. Therefore, one can consider IPH is a diagnosis of exclusion after having ruled out primary and secondary causes of pulmonary hemosiderosis. Idiopathic pulmonary hemosiderosis most commonly occurs in children less than ten years of age. Due to the rare nature of IPH, the incidence and prevalence of which is relatively unknown. Many patients previously reported to have IPH are likely misdiagnosed and have diffuse alveolar hemorrhage. Amiodarone should be avoided in these patients. Amiodarone is known to have various pulmonary side effects. One of the more serious side effects of amiodarone is interstitial lung disease, pulmonary fibrosis, and it has been associated with increased rates of diffuse alveolar hemorrhage. There has been no correlation with adverse effects of diltiazem, furosemide, or metoprolol and IPH. Go to the next page if you knew the correct answer, or click the link image(s) below to further research the concepts in this question (if desired).

Research Concepts: Idiopathic Pulmonary Hemosiderosis

We update eBooks quarterly and Apps daily based on user feedback. Please tap flag to report any questions that need improvement.

Question 313: An 87-year-old female is admitted to the intensive care unit for severe sepsis secondary to a urinary tract infection. Despite appropriate fluid resuscitation and antibiotic therapy, she continues to be lethargic and is intubated due to her inability to maintain her airway. The ventilator is set to a tidal volume of 360 mL (6 mL/kg), a rate of 12 breaths per min, an FiO2 of 50%, and a PEEP of 15. Soon after intubation, her blood pressure drops, and she requires the initiation of inotropic agents. Which of the following maneuvers is most likely to help improve her blood pressure?

Choices: 1. Place the patient in a prone position 2. Decrease PEEP to 5 3. Increase FiO2 to 100% 4. Increase the tidal volume to 8 mL/kg

Answer: 2 - Decrease PEEP to 5 Explanations: Patients undergoing mechanical ventilation undergo a change in their respiratory physiology. In normal respiration, negative intrathoracic pressure caused by contraction of the diaphragm generates air entry into the lungs. At the same time, the negative pressure increases venous return to the right atrium. This, in turn, increases the preload, stroke volume, and cardiac output. In mechanically ventilated patients, this negative pressure is turned into positive pressure through the ventilator. Positive intrathoracic pressure effectively "squeezes" the right atrium, decreasing venous return and diminishing cardiac output. In our case of a septic patient, diminishing cardiac output by any minute margin can have significant deleterious effects on blood pressure. The higher the PEEP, the higher the intrathoracic pressure and the higher the effect on cardiac output. Diminishing PEEP to a more physiologic level can help restore some of the cardiac output and improve blood pressure in this patient. Of the given options, only decreasing PEEP would have an effect on blood pressure. None of the other displayed answers would have any effect on blood pressure. Go to the next page if you knew the correct answer, or click the link image(s) below to further research the concepts in this question (if desired).

Research Concepts: Ventilator Management

We update eBooks quarterly and Apps daily based on user feedback. Please tap flag to report any questions that need improvement.

Question 314: A 34-year-old male with no significant past medical history is involved in a motor vehicle collision with a suspected cervical spine injury. The patient is intubated in the field for respiratory failure and altered mental status and receives 1 liter of normal saline en route to the hospital. The patient’s vitals upon arrival were a blood pressure of 106/68 mmHg, heart rate 74 beats/min, temperature 98.4F, SpO2 99% on room air, and respiratory rate 15. He was started on a low-dose propofol infusion for sedation upon arrival. He was induced with 200 mg of ketamine and paralyzed with 8 mg of vecuronium approximately 30 minutes ago. A primary survey reveals no major injuries aside from some superficial abrasions and lacerations. While in the CT scanner, the patient’s blood pressure begins to drop with a sustained mean arterial pressure less than 60 mmHg. There are no other changes to the patient’s vital signs. Which of the following describes the vasopressor of choice to increase this patient’s mean arterial pressure?

Choices: 1. It functions as both a hormone and neurotransmitter 2. It may be used as a decongestant 3. It functions mainly as a beta-1 agonist 4. It is a PDE3 inhibitor

Answer: 2 - It may be used as a decongestant Explanations: Patients with high cervical injuries can experience a host of hemodynamic changes, including impairment of diaphragmatic and intercostal muscles resulting in respiratory failure, as in this case. Additionally, sympathetic tone decreases and a state of vasodilatory shock can ensue. It is imperative to ensure adequate resuscitation and initiate vasopressor therapy to increase mean arterial pressure (MAP) and thus spinal perfusion pressure. Most sources recommend MAPs greater than 85 mmHg, which in the setting of deep sedation or general anesthesia, will often require vasopressor support. Unlike septic shock, there is no clear consensus on the first-line or ideal vasopressor therapy for neurogenic shock resulting in significant institutional and geographic variations. It is pertinent to evaluate every patient individually and assess the primary underlying problem. In this case, the patient is young, with presumably normal cardiac function, was given 1 liter of fluids before arrival, and had no other major injuries. He has a suspected C-spine injury with a normal heart rate. Most likely, his preload is adequate. His contractility should not be impaired, but his afterload and systemic vascular resistance (SVR) are significantly reduced. Choosing an agent like phenylephrine, an alpha-1 adrenergic agonist, would be ideal in increasing SVR without unneeded ionotropic or chronotropic effects. It is also used as a decongestant in various nasal sprays. With the loss of cardiac accelerator fibers (T1-T4) in spinal trauma, it is possible to have a bradycardic response. If the patient were bradycardic, phenylephrine would not be advised. Reflex bradycardia is less likely to be seen in infusion dosing compared to bolus dosing. It is difficult to discern the hypotensive contribution of the patient’s sedation. The question was worded "low dose propofol infusion" to focus on the primary pathology, but propofol is a profound vasodilator. The first step in any hypotensive crisis is to decrease or eliminate anesthetic or sedative medications until the patient stabilizes. Because propofol also causes hypotension by a reduction in SVR, phenylephrine remains the best choice listed. Go to the next page if you knew the correct answer, or click the link image(s) below to further research the concepts in this question (if desired).

Research Concepts: Phenylephrine

We update eBooks quarterly and Apps daily based on user feedback. Please tap flag to report any questions that need improvement.

Question 315: A 17-year-old male presents with fever and chills that were not relieved with acetaminophen. According to parents, the fever started three days ago when the patient returned from school. He was fine prior to the fever. One of the patient's friends had upper respiratory tract infection the day he developed a fever. Past medical history is hemoglobin SC disease. He is in no acute distress. Cardiac, lung, and abdominal exam are unremarkable. After two days of hospitalization, the patient developed altered mental status and respiratory failure. He was immediately started on mechanical ventilation and IV antibiotics. The patient died 48 hours later due to cardiorespiratory failure. What would the peripheral blood of this patient show?

Choices: 1. Bite cells 2. Howell-Jolley bodies and sickle cells 3. Hairy cells and sickle cells 4. Sickle cells and bite cells

Answer: 2 - Howell-Jolley bodies and sickle cells Explanations: The peripheral smear of asplenic patients show Howell-Jolley bodies, pitted erythrocytosis, monocytosis, and lymphocytosis. After the peripheral blood shows these cells, spleen scintigraphy can be done to confirm the diagnosis If spleen scintigraphy is not available to confirm asplenia, sonogram of the abdomen, CT abdomen, and MRI could be done. Howell-Jolly bodies are present because the spleen has the inability to remove them. Go to the next page if you knew the correct answer, or click the link image(s) below to further research the concepts in this question (if desired).

Research Concepts: Asplenia

We update eBooks quarterly and Apps daily based on user feedback. Please tap flag to report any questions that need improvement.

Question 316: A 43-year-old male is brought in by emergency medical services to the emergency department after being involved in the house fire. On initial exam, the patient has deep, full-thickness burns covering approximately 50% of his body, including a circumferential burn around his entire upper left arm. The patient also has singed nasal hairs, as well as inspiratory and expiratory stridor. Which of the following is the most appropriate next step in management?

Choices: 1. Start 2 liters of lactated Ringer's through peripheral IVs 2. Perform rapid sequence intubation and mechanical ventilation 3. Administer IV fentanyl for pain 4. Perform emergent escharotomy of the left upper extremity

Answer: 2 - Perform rapid sequence intubation and mechanical ventilation Explanations: This patient is exhibiting signs of inhalation injury and impending airway compromise. Inhalation injury is a significant cause of mortality in critical burn patients and can lead to severe laryngeal edema, pulmonary damage, and respiratory failure. Physical exam findings include significant facial burns, singed nasal or eyebrow hair, soot around mouth and nose, stridor, wheezing, cough, and hoarse or muffled voice. Early intubation for airway protecting is the first step in management. This patient has large, full-thickness burns of greater than 20% of his total body surface area. Thus he will require aggressive fluid resuscitation during the first 24 hours of his care. However, as airway compromise can quickly lead to respiratory failure, the patient’s airway must be addressed before calculating fluid requirements. After the patient is intubated and ventilated, he will require IV fluid resuscitation with lactated Ringer's calculated using the Parkland formula. If the setting of full circumferential burns, emergent escharotomy may be indicated if there is evidence of vascular compromise to the limb, however, the patient’s airway, breathing, and circulation must be managed first. Although severe burns are extremely painful, dramatic, and distracting, the first steps in the management of any critical burn patient must be to address the patient’s airway, breathing, and circulation. Once this patient is intubated, he will require sedation and pain management while on mechanical ventilation. Go to the next page if you knew the correct answer, or click the link image(s) below to further research the concepts in this question (if desired).

Research Concepts: Parkland Formula

We update eBooks quarterly and Apps daily based on user feedback. Please tap flag to report any questions that need improvement.

Question 317: A 27-year-old female with systemic lupus erythematosus on daily prednisone presents to the emergency department with a 10-day history of headaches and lowgrade fever. Her head CT is normal, and a lumbar puncture shows a normal leukocyte count (2/microliter) with mostly monocytes; high proteins (55 mg/dL); normal glucose (50 mg/dL); and no erythrocytes. What test of the cerebrospinal fluid is most likely to provide a rapid diagnosis?

Choices: 1. Cryptococcal antigen detected by latex agglutination 2. Cryptococcal culture 3. Gram staining 4. Immunofluorescence

Answer: 1 - Cryptococcal antigen detected by latex agglutination Explanations: The patient is immunosuppressed and the normal cerebrospinal fluid findings support the investigation of cryptococcal meningitis. In one study of patients with AIDS, 26% of patients with cryptococcal meningitis had normal CSF findings; 40% had high protein levels, low glucose levels, and pleocytosis; and 55% had fewer than 10 lymphocytes/mL. The CSF may have a clear or turbid appearance in cryptococcal meningitis. Variable mononuclear pleocytosis is observed, and the white blood cell (WBC) count may be over 20 x 109/L. A high CSF opening pressure is present in about two-thirds of patients and is a poor prognostic sign. Rapid diagnosis is made by India ink stain, latex agglutination, or PCR. Cryptococcal antigen is present several weeks before overt signs of meningitis develop; therefore, its detection provides an opportunity to catch the infection early, and, hence, screening persons with HIV for cryptococcal infection when they access health care can identify asymptomatic infected patients, allowing for prompt treatment and prevention of death The initial diagnostic sensitivity of cryptococcal CSF antigen is 94.1%, followed by the serum antigen of 93.6%; however, this tool is unreliable in assessing the point of discontinuation of antifungal therapy Gram staining is valid for bacteria but does not provide a rapid diagnosis. Immunofluorescence is a forgotten cryptococcal-diagnostic practice. Go to the next page if you knew the correct answer, or click the link image(s) below to further research the concepts in this question (if desired).

Research Concepts: Cryptococcal Meningitis

We update eBooks quarterly and Apps daily based on user feedback. Please tap flag to report any questions that need improvement.

Question 318: A 28-year old was admitted with sudden onset of breath while walking. He is a heavy smoker. Chest x-ray reveals a 90% pneumothorax on the right side. The emergency room resident placed a size 32F tube to evacuate the pneumothorax. An hour later the patient complained of severe shortness of breath and his pulse oximeter revealed oxygen saturation of 89%. A portable chest x-ray revealed vascular congestion in the right lung. He was taken to the intensive care unit and placed on the ventilator because of severe respiratory distress. Which of the following is true about this condition?

Choices: 1. It only occurs in elderly people 2. The incidence is very high 3. It affects patients with extensive pneumothorax 4. It occurs only with exudate but not transudate effusion evacuations

Answer: 3 - It affects patients with extensive pneumothorax Explanations: Pulmonary re-expansion edema occurs when there is rapid emptying of the pleural cavity. It has a low incidence but has a high mortality. It can occur in people with effusions or lung collapse that has been present for more than 7 days. When effusions of more than 3 liters are rapidly evacuated the condition can occur. Go to the next page if you knew the correct answer, or click the link image(s) below to further research the concepts in this question (if desired).

Research Concepts: Pneumothorax

We update eBooks quarterly and Apps daily based on user feedback. Please tap flag to report any questions that need improvement.

Question 319: A 64-year-old male with a long history of back problems was in the swimming pool when he suddenly noticed that his legs had become weak. He struggled out of the pool and called emergency medical services. In the emergency department, he reports that his legs suddenly had no strength, and he was not able to move them. He denies any trauma, loss of consciousness, or visual or speech loss. His only other complaint is an inability to pass urine. On examination, he has significant motor weakness in both legs with numbness all the way from the buttocks to the soles of the feet. He has no sensation in the perineal area and the rectal tone is absent. A Foley catheter is placed and it puts out 800 ml of urine. Which of the following is best for the management of this patient?

Choices: 1. CT scan of the head 2. Complete bed rest for 24 hours 3. Nerve conduction studies 4. Emergency surgery

Answer: 4 - Emergency surgery Explanations: This patient presents with acute symptoms of spinal cord compression. The features of absent rectal tone and urinary retention, together with motor and sensory loss in the extremities, suggest cauda equina syndrome. This is a medical emergency and a surgeon should be notified as soon as possible. The patient needs a quick MRI to decipher the site of compression or fracture. Surgery should follow. This is a common scenario in males with prostate cancer metastases to the vertebral column. For chronic back pain with leg pain, numbness, or paresthesias bed rest with pain control is recommended. When acute symptoms arise, such as loss of rectal tone, an acute compression injury is treated with surgery rather than bed rest. Nerve conduction studies are used in a chronic situation to decipher diseases affecting the neuromuscular junction and to differentiate them from primary muscle myopathy or from disorders due to denervation. Nerve conduction studies are not indicated in this patient, whose etiology lies somewhere below L1-L2. Go to the next page if you knew the correct answer, or click the link image(s) below to further research the concepts in this question (if desired).

Research Concepts: Cauda Equina And Conus Medullaris Syndromes

We update eBooks quarterly and Apps daily based on user feedback. Please tap flag to report any questions that need improvement.

Question 320: A 25-year-old male is admitted to the medical intensive care unit after presenting to the hospital with acute shortness of breath due to multifocal pneumonia. The patient has a medical history of asthma and anxiety disorder, for which he is chronically on inhaled corticosteroids and clonazepam therapy. He is in severe respiratory distress with oxygen saturation 84% despite several nebulized bronchodilator therapies and 100% oxygen supplementation. Etomidate and rocuronium are given for rapid sequence intubation. Postintubation, his oxygen saturation improves to 95%. A chest x-ray confirms adequate positioning of the endotracheal tube. Twenty minutes later, the patient is tachycardic and hypertensive, with high pressures alarming on the ventilator. Oxygen saturation remains 95%. What is the next step in the management of this patient?

Choices: 1. Start nicardipine 2. Start fentanyl 3. Start propofol 4. Start metoprolol

Answer: 3 - Start propofol Explanations: This patient is still most likely paralyzed from the rocuronium but without sedation as the etomidate has most likely worn off by now. Analgesia and sedation are paramount in the intubated patient. It ensures ventilator synchrony, reduces patient agitation and improves outcomes. Analgesia-first sedation should be provided. Fentanyl is a great first choice for the mechanically ventilated patient due to its hemodynamic profile. If analgesia sedation is insufficient then propofol or dexmedetomidine should be added. Benzodiazepines should be avoided in most cases. However in patients chronically on benzodiazepine therapy fentanyl would not prevent benzodiazepine withdrawal. Propofol would be a better choice for this patient. Go to the next page if you knew the correct answer, or click the link image(s) below to further research the concepts in this question (if desired).

Research Concepts: Mechanical Ventilation

We update eBooks quarterly and Apps daily based on user feedback. Please tap flag to report any questions that need improvement.

Question 321: A 42-year-old-male was intubated the day before due to status epilepticus. He is currently in a nursing home on synchronized intermittent mandatory ventilation. The nursing home is employing state-of-the-art nursing interventions for liberating a patient from a ventilator. Continuous assessment of what two items is commonly used, backed by evidencebased bundle for nursing interventions?

Choices: 1. Pain and delirium 2. Sedation and antibiotic usage 3. Endotracheal tube placement and cuff pressure 4. Partial pressure of oxygen on a blood gas and fraction of inspired oxygen

Answer: 1 - Pain and delirium Explanations: The ABCDEF bundle was originally proposed by the Society of Critical Care Medicine in a campaign to help wean and liberate patients from ventilators. The bundle stands for: Assess, prevent, and manage pain; to have daily spontaneous awakening and breathing trials; choice of analgesia and sedation; to assess, prevent, and manage delirium; early mobility and exercise; and family engagement. Closely monitoring pain and delirium will help the patient receive the maximum benefit of mechanical ventilation while ensuring they are not in excruciating pain or too delirious to participate in ventilator weaning protocols. Sedation is part of the ABCDEF bundle, but antibiotics are not. Endotracheal tube placement and cuff pressure can be part of a respiratory therapist's ventilator checklist but is not a nursing intervention. PaO2 and FiO2 are the two variables required to diagnose acute respiratory distress syndrome. Go to the next page if you knew the correct answer, or click the link image(s) below to further research the concepts in this question (if desired).

Research Concepts: Synchronized Intermittent Mandatory Ventilation

We update eBooks quarterly and Apps daily based on user feedback. Please tap flag to report any questions that need improvement.

Question 322: A 17-year-old male patient with a history of intravenous drug use complains of a few days history of pain to his left arm, double vision, fever, and difficulty swallowing. He is awake, alert, and oriented to person, place and time, and denies any headache or neck stiffness. Exam shows a blood pressure of 110/81 mmHg, temperature of 99.3 F, heart rate of 110 beats/min, and respiratory rate of 18. Kernig and Brudzinski testing are within normal limits. A large erythematous, warm, and indurated rash on the left arm is noted with several track marks and pustular lesions. Cranial nerves II through XII are grossly intact, and the bladder is distended. Which of the following is the next best step?

Choices: 1. CT head, lumbar puncture, antibiotics, admit to intensive care unit (ICU) 2. CT head, lumbar puncture, antibiotics, admit to floor 3. Antitoxin, antibiotics, admit to ICU 4. Antitoxin, antibiotics, admit to floor

Answer: 3 - Antitoxin, antibiotics, admit to ICU Explanations: The description is classic for wound botulism, most likely from Clostridium botulinum cellulitis secondary to subcutaneous injection of spores from contaminated drugs. Respiratory failure may occur suddenly, thus the admission to intensive care. Botulism is seen in wounds, food, and in the infantile form. Antitoxin should be administered as soon as botulism is suspected. Antibiotics for wound botulism include penicillin or metronidazole if penicillin allergic. Go to the next page if you knew the correct answer, or click the link image(s) below to further research the concepts in this question (if desired).

Research Concepts: Botulism

We update eBooks quarterly and Apps daily based on user feedback. Please tap flag to report any questions that need improvement.

Question 323: A patient with cirrhosis of liver presents with anginal chest pain. A stress test is positive. The patient undergoes cardiac catheterization, and a selective coronary angiogram is negative for coronary artery disease. But the dyes injected into the right and left coronary ostia filled the ventricles. Which of the following mechanisms is most likely responsible for the patient's symptoms?

Choices: 1. Steal phenomenon 2. Increased oxygen demand due to chamber hypertrophy 3. Increased blood flow to ventricles 4. Due to hypoalbuminemia

Answer: 1 - Steal phenomenon Explanations: Patients with cirrhosis of the liver are prone to coronary cameral fistula due to hyperestrogenic state, which leads to blood emptying into the veins or chambers without perfusing capillaries. Impairment in supply-demand mismatch leads to anginal chest pain. Chamber hypertrophy is not found in a cirrhotic patient unless they have concomitant valvular heart disease. Increased blood flow to ventricles leads to hypervolemia and congestive heart failure. It is not known to cause angina. Hypoalbuminemia leads to ascites but no angina. Go to the next page if you knew the correct answer, or click the link image(s) below to further research the concepts in this question (if desired).

Research Concepts: Coronary Cameral Fistula

We update eBooks quarterly and Apps daily based on user feedback. Please tap flag to report any questions that need improvement.

Question 324: A 65-year-old patient has been admitted to the intensive care unit (ICU) for the management of acute respiratory failure needing intubation related to chronic obstructive pulmonary disease (COPD) exacerbation. The following morning the patient has been on spontaneous weaning trial for 4 hours since. The patient is getting very restless and anxious and pointing to his endotracheal tube (ETT) and wants to take it out. The patient is sitting upright, awake, alert and pointing to his ETT. His vital signs are BP 150/95 mmHg, HR 110 bpm, RR 26/minute, and SpO2 99%. He is on 25% FiO2. His vent screen showed VT in the range of 500600, RR 20-26, PEEP 5 and pressure support of 5. He has no respiratory distress. The patient has a nasogastric tube with tube feeding has been running at 65 cc/hour. What is the best strategy for this patient to be removed from the mechanical ventilator?

Choices: 1. Place back on full support for now. Hold tube feeds at midnight and then consider extubation if the patient able to pass SBT. 2. Go ahead and extubate patient while tube feeding is going at the current rate. 3. Hold tube feeding now and extubate one hour later. 4. suction all residual gastric contents through NG tube and proceed with extubation.

Answer: 4 - suction all residual gastric contents through NG tube and proceed with extubation.

Explanations: The concern about extubation is tube feeding. There is no other contraindication so delaying extubation for another day will prolong ventilator days and ICU days. The patient is already very anxious and may need to be sedated again which can prolong ventilator days even more. Extubation should not be delayed if the patient has passed SBT. It is possible that patient can be extubated with tube feeds going, and he may do just fine, but there is some risk of failure of extubation trial and need for reintubation. In that situation, the stomach should be empty. Also, the patient is awake, and he may not need ongoing tube feeding as he may be able to take oral nutrition very soon. Ideally, tube feeding should be held when a patient appears to do well on a spontaneous breathing trial (SBT). For another patient, it may have been ok to delay extubation for one more hour while holding tube feeding. But this patient has been on an SBT for 4 hours now which has led to anxiety and frustration. Extubation should not be delayed any further. The best strategy would be to suction all tube feed contents from the stomach and then proceed with extubation. It is possible as the patient has an NG tube. It may not have been possible in the presence of nasoduodenal or nasojejunal tube, but in that case, the stomach may not have many contents either. Before extubation, always consider enteral access needs for ongoing care. If the patient is likely to pass swallow, remove the NG tube and start a diet. If the patient is at risk for failure, keep the NG tube and do a swallow test and the decide about an ongoing need for the tube. For high-risk patient for extubation trial failure, be very careful about advancing the diet. Go to the next page if you knew the correct answer, or click the link image(s) below to further research the concepts in this question (if desired).

Research Concepts: Extubation

We update eBooks quarterly and Apps daily based on user feedback. Please tap flag to report any questions that need improvement.

Question 325: A 52-year-old male involved in a motor vehicle collision presents in the trauma bay with no complaints. He is awake, alert, Glasgow coma scale 15, hemodynamically stable with no abnormalities noted on primary and secondary survey. ECG was noted to have premature ventricular contractions. What is the next best course of action?

Choices: 1. Discharge the patient home with cardiology follow up 2. Admit the patient to the hospital for 24 to 48 hours of cardiac monitoring 3. Obtain a cardiology consult STAT 4. Discharge the patient home

Answer: 2 - Admit the patient to the hospital for 24 to 48 hours of cardiac monitoring Explanations: Patients with abnormal ECG and/or troponin I (cTnI) levels should be admitted for cardiac monitoring for 24 to 48 hours because life-threatening arrhythmias or cardiac failure will present during this time. Patients can be admitted to the intensive care unit or under telemetry depending on concurrent injuries, type of ECG changes, and the grade of hemodynamic imbalance. The subset of patients with isolated ECG and cTnI elevations are the most prevalent of the blunt cardiac injury (BCI) and usually have a benign course that usually resolves in a few hours and with rare occurrence of long-term functional impairment. Management of dysrhythmias should be approached and treated as in the patient without BCI. Replete electrolytes accordingly, avoid hypoxia and acidosis, and utilization of antidysrhythmics and defibrillation if needed. Go to the next page if you knew the correct answer, or click the link image(s) below to further research the concepts in this question (if desired).

Research Concepts: Blunt Cardiac Injury

We update eBooks quarterly and Apps daily based on user feedback. Please tap flag to report any questions that need improvement.

Question 326: A 35-year-old woman presents to the hospital complaining of abdominal pain, bloody diarrhea, and fever of 2 days duration. The patient is known to have systemic lupus erythematosus with renal involvement, and she is on immunosuppressive therapy. She has known penicillin allergy. On physical exam, her temperature was 39.5 C (103.1 F), blood pressure 100/70 mmHg, and pulse rate 112/min. The patient looks dehydrated and sick. Abdominal exam shows diffuse tenderness that is more prominent on the left lower quadrant. Her labs show hemoglobin (Hb) 9 mg/dL, white blood cells (WBC) 500/microL, and platelets 90000/microL. CT scan shows bowel wall thickening with fat stranding. No air under the diaphragm is noted. What is the next best step in the management of this patient?

Choices: 1. Piperacillin/ tazobactam 2. Vanco + ceftriaxone + metronidazole 3. Piperacillin/ tazobactam + vancomycin 4. Ceftriaxone + vancomycin

Answer: 2 - Vanco + ceftriaxone + metronidazole Explanations: The patient presented with a clinical picture of neutropenic enterocolitis confirmed by imaging findings. Although there is no consensus on the best treatment of NE, it is usually treated by piperacillin/ tazobactam to cover for bacteria originating from the bowels (gram-negative and anaerobic bacteria) in addition to IV vancomycin in sick or hemodynamically unstable patient to include gram-positive bacteria coverage. However, the patient is allergic to penicillins and cannot be given piperacillin/ tazobactam. So of the choices listed in the question, the most appropriate choice with a spectrum that covers both gram-negative bacteria and anaerobes is ceftriaxone + metronidazole along with vancomycin. Go to the next page if you knew the correct answer, or click the link image(s) below to further research the concepts in this question (if desired).

Research Concepts: Neutropenic Enterocolitis (Typhlitis)

We update eBooks quarterly and Apps daily based on user feedback. Please tap flag to report any questions that need improvement.

Question 327: A 33-year-old alcoholic drank some bootlegged alcohol and within a few hours presented to the emergency department with epigastric pain, nausea, and vomiting. He also complains of blurred vision that just started. Blood work reveals a CBC 4.5, WBC 9, Hemoglobin 11, Cr 1.2, BUN 33, Na 138, K 4.8, glucose 150 mg/dl elevated levels of amylase. Arterial blood gas reveals a pH of 7.23, PO2 77, PCO2 37, HCO3 17. His calculated osmolar gap was increased. Which gastrointestinal problem is he likely to develop?

Choices: 1. Gastric perforation 2. Mesenteric ischemia 3. Hemorrhagic pancreatitis 4. Small bowel obstruction

Answer: 3 - Hemorrhagic pancreatitis Explanations: Hemorrhagic pancreatitis has been described in as many as two-thirds of the patients with methanol poisoning. Even though blindness is of major concern, most people forget about the potential abdominal problems like hemorrhagic pancreatitis that can be severe. Hemorrhagic pancreatitis often presents with bruising and superficial edema in the subcutaneous fatty tissue around the umbilicus. This is Cullen sign. The treatment is the same as for regular methanol toxicity- this includes the use of fomepizole and hydration. Pancreatic hemorrhage is more likely to occur in patients with chronic pancreatitis or alcohol abuse. Go to the next page if you knew the correct answer, or click the link image(s) below to further research the concepts in this question (if desired).

Research Concepts: Methanol Toxicity

We update eBooks quarterly and Apps daily based on user feedback. Please tap flag to report any questions that need improvement.

Question 328: A 55-year-old male in septic shock secondary to pneumonia is brought to the intensive care unit (ICU) for further stabilization. A right internal jugular central line was placed in the emergency department and started on pressor support. A portable chest x-ray shows the line terminating 3 cm above the junction of the superior vena cava and the right atrium. The patient is hemodynamically unstable at this time. What should be done about the current placement of the central line?

Choices: 1. Carefully clip the sutures and advance the central line 3 cm into the internal jugular (IJ) vein then resuture the central line into place 2. Leave the line in place as it is still in the internal jugular vein and is able to draw and flush 3. After donning appropriate attire, sterile gloves, and using sterile technique, clip the sutures and advance the central line 3 cm further into the IJ then resuture the line into place 4. Ask the emergency room doctor to fix the line that they placed

Answer: 2 - Leave the line in place as it is still in the internal jugular vein and is able to draw and flush

Explanations: Advancing a central line without the use of proper sterile technique places the patient at risk of developing a catheter-based infection such as cellulitis, abscess, or bacteremia. The correct placement of an internal jugular central line tip is in the superior vena cava or at the cavo-atrial junction (i.e., at the level of 1st anterior intercostal space above carina). This can best be visualized and confirmed with a portable chest x-ray. This line can be used without being adjusted. If needed the line can be changed over a guidewire, or a new line may be inserted at a different site. At this time it would be unsafe to transport the patient down to the emergency department to ask the performing physician to correct the placement. Also, the patient is now under the care of the ICU, and the correction can be simply made with appropriate sterile technique. After placing internal jugular or subclavian central lines, portable chest x rays should always be performed immediately after to ensure correct placement and absence of pneumothorax. Go to the next page if you knew the correct answer, or click the link image(s) below to further research the concepts in this question (if desired).

Research Concepts: Central Line Management

We update eBooks quarterly and Apps daily based on user feedback. Please tap flag to report any questions that need improvement.

Question 329: Following a motor vehicle accident, a 35-year-old male is agitated and has multiple cerebral contusions on CT of the head. Hemodynamic measurements show a heart rate of 85 beats/min and a mean arterial pressure (MAP) of 84 mmHg. An intracranial pressure monitor reveals a pressure of 28 mmHg. What is the most appropriate management regimen?

Choices: 1. Hyperventilation to maintain a cerebral PCO2 of 25 to 30 mmHg 2. Administration of phenylephrine to increase MAP 3. Administer hypertonic saline 4. Placement of patient in Trendelenburg position

Answer: 3 - Administer hypertonic saline Explanations: Cerebral perfusion pressure (CPP) is the difference between mean arterial pressure (MAP) and intracranial pressure (ICP). It should be greater than or equal to 70 mmHg. ICP should be 20 mmHg or less. Methods for reducing ICP include elevation of the head of the bed (reverse Trendelenburg), administration of hypertonic saline, mannitol, other diuretics, sedatives to control agitation, prevention of hypovolemia, maintenance of adequate CPP, and keeping pCO2 in the proper range. Hypertonic saline has been shown to be superior to mannitol in patients with hypotension severe brain trauma. The patient also will require aggressive fluids management. Hypercapnia causes vasodilation of the cerebral vessels, which increases intracranial volume and pressure. Hyperventilation may be used for impending herniation to acutely lower ICP. However, prolonged hyperventilation decreases perfusion secondary to the vasoconstriction that occurs in an injured, ischemic brain. Go to the next page if you knew the correct answer, or click the link image(s) below to further research the concepts in this question (if desired).

Research Concepts: Increased Intracranial Pressure

We update eBooks quarterly and Apps daily based on user feedback. Please tap flag to report any questions that need improvement.

Question 330: A 16-year-old white female presents to the emergency department with acute severe worsening of her asthma symptoms that would not respond to home inhalers over the past 6 hours. Medical history is significant for anxiety and bipolar disorder, irritable bowel syndrome, hypothyroidism, GERD and a recent diagnosis of bronchial asthma. This is her 4th episode of severe asthma exacerbation in the past 4 months. Each one of them required endotracheal intubation. She also gave birth to a healthy boy by a caesarian section last year. Symptoms escalate rapidly in requiring intubation again. Her bloodwork shows an absolute eosinophil count of 320/microL. Alarmed by these recurrent episodes of severe asthma, a CT angiogram of the chest is ordered as well. It does not show pulmonary embolism or chronic bronchiectasis. What further investigations may be most helpful in treating her condition?

Choices: 1. Send aspergillus specific antibody and IgE level 2. Send 5-hydroxy indole acetic acid level and chromogranin assay 3. Perform bronchoscopy when extubated 4. Speech therapy evaluation once extubated

Answer: 3 - Perform bronchoscopy when extubated Explanations: Recurrent status asthmaticus requiring and reversed by endotracheal intubation should always raise concern for tracheal pathologies like tracheomalacia or excessive dynamic airway collapse (EDAC) which can be diagnosed by evaluation of upper airway postextubation. Risk factors for EDAC include prior intubation, GERD, chronic respiratory inflammation, and thyroid disease. CT scan may appear normal particularly if done in the inspiratory phase. Treatment once diagnosed may involve placement of a tracheal stent. Evaluation by speech therapy can be effective for pure vocal cord dysfunction once tracheal collapse or tracheomalacia has been ruled out by airway evaluation. Go to the next page if you knew the correct answer, or click the link image(s) below to further research the concepts in this question (if desired).

Research Concepts: Status Asthmaticus

We update eBooks quarterly and Apps daily based on user feedback. Please tap flag to report any questions that need improvement.

Question 331: A 67-year-old female presents with abdominal pain that occurs minutes to a few hours after every meal. This has been going on for the past 2 months, and she has lost about 15 pounds. She has become afraid to eat. Also, she has a history of a stroke. The physical exam is unremarkable, and the blood work is normal. What is the most likely condition?

Choices: 1. Abdominal aortic aneurysm 2. Chronic mesenteric ischemia 3. Gallstones 4. Chronic pancreatitis

Answer: 2 - Chronic mesenteric ischemia Explanations: Mesenteric ischemia is a rare disorder but carries high mortality if the diagnosis is missed. Chronic mesenteric ischemia typically presents with postprandial abdominal pain and weight loss. Patients also may develop a fear of eating. Patients often have a history of vascular disease that involves other organs including cerebral vascular disease, coronary artery disease, or peripheral vascular disease. Arteriography is the gold standard for making the diagnosis. Open or endovascular revascularization is the treatment of choice. Go to the next page if you knew the correct answer, or click the link image(s) below to further research the concepts in this question (if desired).

Research Concepts: Chronic Mesenteric Ischemia

We update eBooks quarterly and Apps daily based on user feedback. Please tap flag to report any questions that need improvement.

Question 332: A 65-year-old male smoker with a past medical history of chronic obstructive pulmonary disease (COPD) presents with worsening shortness of breath and cough with expectoration. On examination, he is awake but in severe distress and using accessory muscles. Chest auscultation reveals prolonged expiration and bilateral wheezes. Arterial blood gas analysis reveals a pH of 7.20, pCO2 of 72 mmHg, and pO2 of 50 mmHg. Pulmonary function tests show FEV1/FVC 60% of predicted. Chest x-ray shows flattened diaphragm and widened intercostal spaces. Inhaled short-acting beta-agonists, steroids, oxygen, and noninvasive ventilation with Bilevel positive airway pressure are tried, but the condition does not improve. The patient is intubated, and on day three does well on a spontaneous breathing trial. What is the best next step in the management of this patient?

Choices: 1. Extubate to nasal cannula 2. Room air 3. Tracheostomy and percutaneous endoscopic gastrostomy 4. Extubate to bilevel positive airway pressure

Answer: 4 - Extubate to bilevel positive airway pressure Explanations: Prophylactic non-invasive ventilation like bilevel positive airway pressure (BiPAP) following extubation has been shown to decrease the reintubation rate in chronic respiratory disease especially in hypercarbic chronic obstructive pulmonary disease (COPD) patients who were mechanically intubated for respiratory failure. Hypercapnia in this patient points towards the chances of extubation failure. To prevent extubation failure, this patient should be extubated to BiPAP. The early use of non-invasive ventilation (NIV) post-extubation also has reduced hospital mortality in such high-risk intensive care unit patients. Noninvasive ventilation post-extubation in mechanically ventilated patients with COPD has also has shown to reduce the duration of mechanical ventilation, length of intensive care unit stay and, decreased incidence of ventilator-associated pneumonia. Since the mechanically ventilated patients with COPD has passed the spontaneous breathing trial and is hypercapnic, the best next step is extubating to non-invasive ventilation. NIV prevents acute respiratory failure and decreases mortality in such patients. With BiPAP, such patients are likely to show a higher PaO2, lower PaCO2, respiratory rate, especially when compared with patients using an oxygen mask. Go to the next page if you knew the correct answer, or click the link image(s) below to further research the concepts in this question (if desired).

Research Concepts: Emphysema

We update eBooks quarterly and Apps daily based on user feedback. Please tap flag to report any questions that need improvement.

Question 333: A 60-year-old male with a past medical history significant for hypertension and diabetes mellitus presents for a physical examination. He complains of some shortness of breath with walking at a moderate pace and while he is mowing his lawn. On physical examination, his blood pressure is 160/60 mm Hg with a heart rate of 84 bpm. The cardiac exam reveals two murmurs. The first is a decrescendo, blowing diastolic murmur at the 3rd intercostal space at the sternal border. The second is a diastolic murmur at the apex and is low pitched and rumbling. Carotid pulses are bounding. What ECG finding is consistent with the underlying diagnosis?

Choices: 1. ST depression in leads II, III, and AVF 2. Q waves in the anterior leads 3. LVH with strain 4. Biphasic P wave

Answer: 3 - LVH with strain Explanations: This patient with wide pulse pressure has a murmur consistent with chronic aortic regurgitation (AR). In AR, a high frequency, blowing, decrescendo, diastolic murmur is heard best third intercostal space along the left sternal border. The low pitched rumbling mid-diastolic murmur heard best at the apex is thought to be caused by premature closure of the mitral valve due to the jet of AR. AR can cause LVH over time, which can be seen on an ECG. The test of choice to assess aortic regurgitation is an echocardiogram. Go to the next page if you knew the correct answer, or click the link image(s) below to further research the concepts in this question (if desired).

Research Concepts: Aortic Regurgitation

We update eBooks quarterly and Apps daily based on user feedback. Please tap flag to report any questions that need improvement.

Question 334: A 37-year-old male crashes his motorcycle at a very high speed and strikes his chest into a guardrail. Emergency medical services (EMS) have arrived on the scene and have found the patient to be in cardiac arrest and begin advanced cardiac life support, including cardiopulmonary resuscitation (CPR). The emergency medical services crew also performed endotracheal intubation, bilateral chest decompression, and established humeral intraosseous access with intravenous crystalloid resuscitation. The patient now arrives in the trauma bay, and EMS reports the patient is in asystole for downtime of approximately 10 minutes with signs of life initially present in the field. Which of the following is the most appropriate management at this time?

Choices: 1. Give an additional dose of intravenous epinephrine and electrically defibrillate the patient 2. Place an ultrasound on the patient's chest and if no cardiac motion is identified cease resuscitative efforts 3. Perform an emergency department resuscitative thoracotomy with aortic cross-clamping and internal cardiac massage 4. Initiate the massive-transfusion protocol and send emergently to the operating room

Answer: 3 - Perform an emergency department resuscitative thoracotomy with aortic crossclamping and internal cardiac massage

Explanations: Patients who present in traumatic cardiac arrest are managed differently with respect to the injury mechanism. Blunt trauma often leads to uncontrollable exsanguination, severe hypoxia, and ventilatory derangements or catastrophic neurologic injuries which often lead to the death of patients in the field. The advanced cardiac life support (ACLS) and advanced trauma life support (ATLS) courses promulgated by the American Heart Association and the American College of Surgeons teach the 5 H's and T's (hypoxia, hypovolemia, hyperkalemia, hydrogen ions, hypothermia, tension pneumothorax, thrombosis, toxins, thromboembolism, tamponade) for potentially reversible causes of asystole. If not identified and treated early and return of spontaneous circulation is restored, patients have a very dismal prognosis. Unfortunately, blunt traumatic injuries that lead to traumatic cardiac arrest are often not amenable to these treatments. A patient who presents in traumatic cardiac arrest from a blunt mechanism of injury who arrives with less than 10 minutes of known downtime with confirmed signs of life in the field would be a candidate for an emergency department resuscitative thoracotomy. For blunt traumatic arrest, total downtime with CPR of 10 minutes or less, and signs of life noted in the field, an emergency department resuscitative thoracotomy (EDT) could be performed. The first step is to perform a left-sided thoracotomy from the lateral sternal border to the posterior thorax as far as possible in the 4-6th intercostal space. The chest is then opened with a retractor exposing the lung. Blood and air are evacuated from the pleural cavity. The lung is swept out of the way anteriorly by first taking down the inferior pulmonary ligament, ensuring not to damage the inferior pulmonary vein, which lies just superiorly. Afterexposing the heart, internal cardiac massage is performed while further resuscitation is performed to perfuse the heart and brain. If the patient has a return of spontaneous circulation, they are often brought to the operating room for definitive repair of their injuries. Additional doses of epinephrine may be given to a patient in asystole from a blunt mechanism of injury, but defibrillation is not warranted unless in ventricular fibrillation or ventricular tachycardia without a pulse. Placement of an ultrasound probe on the patient's chest would be prudent to cease resuscitative efforts if the patient does not meet the strict criteria to perform an EDT. Go to the next page if you knew the correct answer, or click the link image(s) below to further research the concepts in this question (if desired).

Research Concepts: Asystole

We update eBooks quarterly and Apps daily based on user feedback. Please tap flag to report any questions that need improvement.

Question 335: A 40-year-old male with a history of alcohol use disorder is admitted to the hospital for skin lacerations following a motor vehicle accident. Urgent care is given with intravenous fluids and sutures where required. On physical examination, the blood pressure is 130/82 mmHg. The pulse is 86/min, respiratory rate is 18/min, and temperature is 37 C (98.7 F). After 5 hours, the nurse goes to draw blood, and the patient refuses, citing restlessness and agitation. The patient yells and states that his heart is pounding and he would not let anyone come close to him. He has a previous history of hepatitis C that has been treated. What is the next step in management?

Choices: 1. Administer haloperidol 2. Administer oxazepam 3. Administer lactulose 4. Administer diazepam

Answer: 2 - Administer oxazepam Explanations: Benzodiazepines bind as an allosteric modulator and increase the number of chloride ions crossing the cell membrane. Once benzodiazepines bind to the receptor, the gammaaminobutyric acid (GABA) receptor changes conformation and thereby begins to have an increased affinity for GABA. Oxazepam is an FDA-approved benzodiazepine used for the treatment of alcohol withdrawal, management of anxiety disorders, and agitation. The convention on psychotropic substances classifies oxazepam as a schedule IV substance, meaning that oxazepam has a lower potential for abuse relative to substances in schedule III substances (ex. ketamine). For alcohol withdrawal in adults, administer oxazepam 15 mg PO TID. Diazepam should not be used in this patient because it is metabolized in the liver. Oxazepam, on the other hand, is excreted through the kidneys. Go to the next page if you knew the correct answer, or click the link image(s) below to further research the concepts in this question (if desired).

Research Concepts: Oxazepam

We update eBooks quarterly and Apps daily based on user feedback. Please tap flag to report any questions that need improvement.

Question 336: A 36-years-old female presents to the hospital for back pain and fever. Symptoms started four days ago and are getting progressively worse. She was seen in an urgent care center and was given pain medication and antibiotics with no improvement. She had a skin infection some three weeks ago that lasted a few days but disappeared without treatment. Otherwise, she is healthy and takes no medication. Physical exam shows enderness over the thoracic spine, numbness in both thighs and low-grade fever. Her blood count shows leukocytosis. Blood cultures are drawn. The emergency department does not have the capability for emergency MRI. Which of the following is the next best step in the management of this patient?

Choices: 1. MRI of the spine with contrast to be done outpatient for better sensitivity and wait for results of culture to be available 2. CT of the spine with contrast followed by neurosurgical evaluation; start antibiotics (vancomycin and cefotaxime) 3. CT scan of the spine without contrast followed neurosurgical evaluation; start antibiotics (vancomycin and ceftriaxone) 4. CT scan of the spine with contrast followed by neurosurgical evaluation; start antibiotics (vancomycin and piperacillin/tazobactam)

Answer: 2 - CT of the spine with contrast followed by neurosurgical evaluation; start antibiotics (vancomycin and cefotaxime)

Explanations: Spinal epidural abscess (SEA) is a true medical emergency. 1/3 of patients have no clear etiology for acquiring the infection. Of the other 2/3, the more common site of origin is skin infection followed by complications of spinal surgery. Staphylococcus is the organism in the majority of cases followed by gram-negative bacilli. CT and MRI with contrast are needed for SEA diagnosis. MRI is preferred, but if not immediately available, CT is a feasible alternative. Vancomycin plus a later generation cephalosporin (cefotaxime, ceftriaxone, cefepime) is the empiric antibiotic treatment of choice. If Pseudomonas is suspected (prior surgery or invasive procedures) then ceftazidime is preferred. Go to the next page if you knew the correct answer, or click the link image(s) below to further research the concepts in this question (if desired).

Research Concepts: Epidural Abscess

We update eBooks quarterly and Apps daily based on user feedback. Please tap flag to report any questions that need improvement.

Question 337: A 16-year-old patient returns home after traveling to Africa and is seen in the emergency department. His vitals reveal a blood pressure of 90/60 mmHg, respiratory rate of 22, pulse of 43 beats/min, and temperature 103 degrees F. He appears slightly jaundiced and has numerous petechiae. The patient may have which of the following conditions?

Choices: 1. Tuberculosis 2. Yellow fever 3. Leishmaniasis 4. Lymphoma

Answer: 2 - Yellow fever Explanations: With fever, most people develop tachycardia. In yellow fever, the patient develops bradycardia. A fever with bradycardia is known as Faget sign and is often seen in yellow fever. The diagnosis requires a thorough travel history and record of immunizations. Patients may present with headache, malaise, and myalgia. A physical exam may reveal Faget sign or pulse fever dissociation, facial flushing, and conjunctival injection. Blood work may reveal leukopenia with elevated transaminase levels. If the liver is involved, a coagulation profile may be abnormal. Yellow fever can be diagnosed using enzyme-linked immunosorbent assay (ELISA) and polymerase chain reaction assay. Other investigations depend on what organ is involved. If there is evidence of altered mentation, a lumbar puncture and a CT scan are performed. Go to the next page if you knew the correct answer, or click the link image(s) below to further research the concepts in this question (if desired).

Research Concepts: Yellow Fever

We update eBooks quarterly and Apps daily based on user feedback. Please tap flag to report any questions that need improvement.

Question 338: An unidentified patient is brought to the nearest rural hospital following a traumatic car accident. An emergent exploratory laparotomy for hypovolemic shock is performed via a predominantly halothane based anesthetic plan. On day 11 of hospitalization, he reports jaundice, abdominal swelling, nausea, vomiting, malaise, and confusion. The following labs were obtained: Complete blood count reveals hematocrit levels of 42 %, platelets are 160,000/mm3, and leukocytes are 9000/mm3. Serum chemistry shows blood urea nitrogen 32 mg/dL and creatinine 1.9 mg/dL. Liver function studies reveal total bilirubin 3.3 mg/dL, alkaline phosphatase 220 U/L, aspartate aminotransferase 3207 U/L, alanine aminotransferase 4180 U/L and INR 1.6 (normal 0.8-1.2). What is the next most appropriate step in the management of this patient?

Choices: 1. Reassurance 2. NPO and IV fluids 3. Emergent liver transplant 4. Antibiotics and surgical exploration

Answer: 3 - Emergent liver transplant Explanations: This is a case of type 2 hepatotoxicity based on timing and severity. Type 1 is the self remitting variation of halothane toxicity. In mild forms of liver injury, supportive therapy is the correct answer. However, this patient is in acute liver failure indicated by encephalopathy (confusion), LFT's >1000, and liver synthetic dysfunction (abnormal INR). A liver transplant is needed for acute liver failure to prevent death. Even with a liver transplant, the mortality rate is over 50% for type two hepatotoxicity with acute liver failure. Go to the next page if you knew the correct answer, or click the link image(s) below to further research the concepts in this question (if desired).

Research Concepts: Halothane Toxicity

We update eBooks quarterly and Apps daily based on user feedback. Please tap flag to report any questions that need improvement.

Question 339: A 66-year-old male patient is currently in a hospital recovering from a cardiac revascularization procedure. He reports that he suddenly feels unusually weak and fatigued, even when considering his recent operation. The patient has had obesity for approximately 25 years and was diagnosed with type 2 diabetes 12 years ago, which is managed with metformin. Recently, the patient was diagnosed with ischemic heart disease, which required a revascularization procedure. The patient has no known allergies or hypersensitivities to any medications. Physical examination reveals a substantial decrease in the patient’s cardiac output and that the patient is being impacted by, for the first time, congestive heart failure. The patient has not yet received any treatment for his symptoms of congestive heart failure. Treatment with inamrinone is being considered. Which of the following is the most appropriate strategy for treating the patient with this medication?

Choices: 1. The clinician should initially administer digitalis, diuretics and/or vasodilators before administering inamrinone 2. The clinician should, initially, administer undiluted inamrinone at 0.75 mg/kg over 2 to 3 minutes 3. The clinician should, initially, administer diluted (with 0.9% saline) inamrinone at a dose of 5 to 10 mcg/kg/min 4. The clinician should, initially, administer diluted (with a dextrose [glucose]-containing solution) inamrinone at a dose of 5 to 10 mcg/kg/min

Answer: 1 - The clinician should initially administer digitalis, diuretics and/or vasodilators before administering inamrinone

Explanations: The administration of inamrinone is only indicated if the patient's cardiovascular function has not initally responsed positively to the administration of digitalis, diuretics and/or vasodilators. A loading dose of inamrinone at 0.75 mg/kg over 2 to 3 minutes will next be indicated if digitalis, diuretics and/or vasodilators have failed to sufficiently address the symptoms of congestive heart failure. A maintenance dose of inamrinone, diluted with 0.9 or 0.45% saline, at 5 to 10 mcg/kg/min is only administered after the loading dose of inamrinone. Inamrinone should never be diluted with a dextrose [glucose]-containing substance due to the chemical reaction that can occur. Go to the next page if you knew the correct answer, or click the link image(s) below to further research the concepts in this question (if desired).

Research Concepts: Inamrinone

We update eBooks quarterly and Apps daily based on user feedback. Please tap flag to report any questions that need improvement.

Question 340: A 59-year-old man, with the renal transplant, comes to the emergency department complaining of abdominal cramps, diarrhea, and nausea from 3 days. Today he also developed moderate fever and decreased appetite. The diarrhea is non-bloody and non-fatty. He is on prophylactic antibiotics for prevention of opportunistic infections. The temperature is 101.4 F, blood pressure is 100/65 mmHg, and pulse is 100/min. On complete blood count, his white blood cell count is 18,000/microL, serum creatinine is 2 mg/dL, and serum albumin level is 4g/dL. Stool comes positive for Clostridium difficile toxin. Doctors started oral vancomycin and after which ileus developed. What is the next step in management?

Choices: 1. Add oral metronidazole 2. Switch to intravenous metronidazole 3. Add intravenous metronidazole 4. Switch to rectal metronidazole

Answer: 3 - Add intravenous metronidazole Explanations: This patient's presentation is consistent with Clostridium difficile colitis. Advance age and antibiotic use are the risk factors he has been for C. difficile colitis. For mild C. difficile colitis oral metronidazole is used. In severe cases, oral vancomycin is preferred. In severe Clostridium difficile colitis, after oral vancomycin, if ileus develops there are two options. Add intravenous metronidazole or switch to rectal vancomycin. Go to the next page if you knew the correct answer, or click the link image(s) below to further research the concepts in this question (if desired).

Research Concepts: Bacterial Gastroenteritis

We update eBooks quarterly and Apps daily based on user feedback. Please tap flag to report any questions that need improvement.

Question 341: A 70-year-old woman complains of difficulty breathing towards the end of receiving 1 unit of packed red blood cell transfusion following right knee replacement. She has a history of hypertension, chronic kidney disease stage 4, and heart failure with preserved ejection fraction. Medications currently in the hospital are lisinopril, chlorthalidone, and subcutaneous heparin for deep venous thrombosis (DVT) prophylaxis. Before the transfusion, she had also been receiving lactated ringer's solution at 75 ml/hr. On evaluation, the patient is afebrile, blood pressure is 172/85 mm Hg, heart rate is 122/min, respiratory rate is 24/min, and saturation is 88% on room air. Jugular venous distension noted to be 7 cm H2O. Tachycardia with no murmurs is heard on the cardiac exam with extensive rales auscultated in both lung fields. There is 1+ bilateral pedal edema present. EKG reveals sinus tachycardia with left ventricular hypertrophic changes. No ST-T changes noted. A chest x-ray reveals pulmonary edema with bilateral airspace opacities. Laboratory data show hemoglobin of 8.0 g/dL (7.1 g/dL prior to transfusion), creatinine 3.3 mg/dL (3.2 mg/dL prior to transfusion), total bilirubin 1.0 mg/dL, lactate dehydrogenase 75 U/L. Urinalysis is negative for protein and blood. A brain natriuretic peptide (BNP) level is pending. Intravenous furosemide was administered with symptomatic improvement noted within 30 minutes. Which of the following clinical information can be used to distinguish between transfusion-related complications?

Choices: 1. Absence of fever 2. Favorable response to diuretics 3. Absence of hypotension 4. Presence of pulmonary edema on chest x-ray

Answer: 2 - Favorable response to diuretics Explanations: This patient has transfusion-associated circulatory overload (TACO). It is diagnosed when at least 3 of the following clinical features are identified within 6 hours of receiving blood transfusion: symptoms of respiratory distress, raised central venous pressure, high B-type natriuretic peptide, x-ray findings of pulmonary edema, presence of left heart failure and presence of positive fluid balance. Patients diagnosed with TACO show significant improvement in symptoms with the use of diuretics. However, this is not true in patients with transfusion-associated lung injury (TRALI). Risk factors for TACO include age more than 60 years, presence of chronic kidney disease, heart failure, number of blood transfusions, and the rate of transfusion. Since the etiology for the development of TACO is a positive fluid balance secondary to administration of blood, sometimes in addition to the intravenous fluids, diuretics remain the mainstay of treatment. Diuretics do not help patients with TRALI as these patients do not present with features suggestive of volume overload such as elevated central venous pressure, pedal edema). Fever and hypotension can be seen in TRALI, although only present in about 30% of cases. Evidence of pulmonary edema on chest x-ray is present in both TACO and TRALI however; the mechanism is cardiogenic in TACO and non-cardiogenic in TRALI. Go to the next page if you knew the correct answer, or click the link image(s) below to further research the concepts in this question (if desired).

Research Concepts: Pulmonary Edema

We update eBooks quarterly and Apps daily based on user feedback. Please tap flag to report any questions that need improvement.

Question 342: A 72-year-old female with end-stage renal disease on dialysis presents to her primary care physician for pain in the left upper extremity that began over the last week. The patient has a hemodialysis catheter in place on the left upper extremity without any signs of surrounding erythema, drainage, or warmth surrounding the catheter. She reports feeling like her upper extremity is "heavy." Which of the following imaging modalities is the preferred initial imaging study for her likely condition?

Choices: 1. Magnetic resonance imaging 2. Computed tomographic venography 3. Plain film 4. Compression duplex ultrasonography

Answer: 4 - Compression duplex ultrasonography Explanations: Magnetic resonance imaging (MRI) has a high sensitivity for the diagnosis of deep venous thrombosis (DVT) at almost 100%. It is not the gold standard for initial imaging due to its high costs, length of time to complete the study, and/or availability. Computed tomographic venography also has a high sensitivity for the diagnosis of DVT, however, is not the gold standard for initial imaging due to higher costs, radiation exposure, time, and/or availability. If ultrasonography is negative, but the clinical suspicion remains high for a DVT, venography should be obtained. Plain films would not adequately show a venous thrombosis but could help evaluate any bony abnormalities that could lead to venous stenosis and resultant thrombosis. Ultrasonography is the preferred initial imaging study for diagnosis of DVT and has a sensitivity of 97%, which is lower than MRI but is non-invasive, less expensive, widely available, and fast. Go to the next page if you knew the correct answer, or click the link image(s) below to further research the concepts in this question (if desired).

Research Concepts: Upper Extremity Deep Venous Thrombosis

We update eBooks quarterly and Apps daily based on user feedback. Please tap flag to report any questions that need improvement.

Question 343: A 16-year-old is seen in the emergency department with a hoarse voice and dysphagia. His symptoms started 12 hours ago and have progressed. He has a fever and appears ill. He is unable to speak and thus the mother provides the history. The tonsils are 4+/4 enlarged with grey exudates. There is bilateral cervical adenopathy. While waiting for the patient to be seen by an ENT surgeon, what one study could be ordered for this child?

Choices: 1. Blood culture 2. Lateral neck x-ray 3. Throat swab 4. Arterial blood gas

Answer: 2 - Lateral neck x-ray Explanations: The treatment of routine tonsillitis which is mild is usually pain control and antibiotics (macrolides or amoxicillin). For recurrent tonsillitis, tonsillectomy is recommended. The one imaging study sometimes performed in patients with enlarged tonsils is the lateral neck x-ray. The lateral neck x-ray will reveal the degree of airway obstruction. When the airway obstruction is severe, the patient should be admitted and a tracheostomy set must be available at the bedside. Go to the next page if you knew the correct answer, or click the link image(s) below to further research the concepts in this question (if desired).

Research Concepts: Tonsillitis

We update eBooks quarterly and Apps daily based on user feedback. Please tap flag to report any questions that need improvement.

Question 344: A 45-year-old male is admitted to the hospital after presenting with epigastric pain with nausea and vomiting and initial laboratories showing an elevated lipase consistent with a diagnosis of acute pancreatitis. A further review of laboratory values shows aspartate aminotransferase of 75 units/L, alanine aminotransferase of 270 units/L, alkaline phosphatase of 202 international units/L, and total bilirubin of 2.1 mg/dL. Which of the following is the most likely cause of his pancreatitis?

Choices: 1. Gallstones 2. Medication-induced 3. Hypertriglyceridemia 4. Alcohol

Answer: 1 - Gallstones Explanations: Basic laboratory values can be used to guide the determine the etiology of acute pancreatitis. Suspected obstruction should be confirmed with abdominal imaging, initially with an abdominal ultrasound. An elevated aspartate aminotransferase (AST) is more consistent with alcohol-related pancreatitis. Laboratory values suggesting pancreatitis secondary to bile duct obstruction are an elevation of alanine aminotransferase (ALT) greater than AST as well as an elevation of alkaline phosphatase. Also, total bilirubin levels likely will be elevated with full biliary obstruction. Go to the next page if you knew the correct answer, or click the link image(s) below to further research the concepts in this question (if desired).

Research Concepts: Acute Pancreatitis

We update eBooks quarterly and Apps daily based on user feedback. Please tap flag to report any questions that need improvement.

Question 345: A 30-year-old male with a history of sickle cell disease and ischemic stroke 1 year ago, undergoes a left hip replacement for aseptic necrosis of the left femoral head. He presents to the emergency department 2 weeks after the surgery with difficulty breathing and chest pain. Prior to complete evaluation the patient becomes unresponsive and has a cardiac arrest. Despite adequate cardiopulmonary resuscitation, the patient does not survive. The autopsy results show multiple fat emboli in the lung with pathologic evidence of ischemic stroke. When filling out the death certificate, which of the following will be noted as the immediate cause of death?

Choices: 1. Cardiorespiratory arrest 2. Air embolism 3. Fat embolism 4. Ischemic stroke

Answer: 3 - Fat embolism Explanations: The sudden onset of chest pain and dyspnea is highly suggestive of an embolic process in the lung in this patient. This is the most attributable immediate cause of death. Sickle cell disease causes chronic hypoxia which can lead to fragments of necrotic bone marrow tissue being dislodged from the marrow. This embolus can subsequently occlude the pulmonary artery. Gurd and Wilson proposed criteria for the diagnosis of fat embolism. This condition can be seen in patients with sickle cell disease and has a high mortality rate. The cause of death section in the death certificate should be filled out as specifically as possible. In Part I, one reports the causal events that lead to death and consists of Line Ia to Line Id. The most “immediate” or “recent“ event that leads to the demise is listed on Line Ia. The other conditions are listed on Line Ib to Line Id in a sequential manner. The last and most remote condition leading to death is listed as the “underlying” cause of death. All conditions that fall between the immediate and underlying cause of death are known as intermediate or intermediary. Go to the next page if you knew the correct answer, or click the link image(s) below to further research the concepts in this question (if desired).

Research Concepts: Death Certification

We update eBooks quarterly and Apps daily based on user feedback. Please tap flag to report any questions that need improvement.

Question 346: A 55-year-old female presents to the clinic with progressive shortness of breath and chest pain for the past 3 weeks. She is a smoker and social drinker. She is on a bronchodilator and takes several herbal medications. On the physical examination, she has a decrescendo diastolic murmur that is best heard while leaning forward. Palpation of the groin reveals abrupt distension and quick collapse of the femoral artery pulse. On further physical examination, which of the following is unlikely to be observed?

Choices: 1. A narrowed pulse pressure 2. Bobbing motion of the patient's head with each heartbeat 3. Visible pulsations of the uvula 4. Inferior displacement of the apical pulse

Answer: 1 - A narrowed pulse pressure Explanations: Severe aortic regurgitation usually presents with wide pulse pressure. Corrigan pulse is also known as a water hammer pulse. Quincke sign is heard over the femoral artery. Traube sign is hearing ‘pistol shots' over the groin. The point of maximal impulse is diffuse and often displaced inferiorly and towards the axilla. The murmur of aortic regurgitation is high pitched and correlates with the severity of the disorder. Go to the next page if you knew the correct answer, or click the link image(s) below to further research the concepts in this question (if desired).

Research Concepts: Aortic Regurgitation

We update eBooks quarterly and Apps daily based on user feedback. Please tap flag to report any questions that need improvement.

Question 347: A 78-year-old male presents via emergency medical services for ingestion of 200 tabs of 300 mg propafenone. On arrival, the patient moans to painful stimuli but makes no effort to remove the stimuli. Blood pressure is 80/40 mmHg, pulse 130 bpm, respiratory rate 6/minute, and pulse oximetry 89% on nonrebreather mask. What is the most appropriate next intervention?

Choices: 1. Bilevel positive airway pressure (BiPAP) 2. Airway support 3. Normal saline bolus 4. Atropine

Answer: 2 - Airway support Explanations: The patient has a Glasgow coma scale less than 8, so he is unable to effectively protect his airway necessitating airway support. BiPAP should be avoided due to the patient's significantly depressed mental status, and inability to protect his airway. One of the contraindications to BiPAP is the inability to protect the airway or remove the device themselves in the event of emesis. It is important to initiate fluid boluses as the patient is hypotensive, but the management of the patient's airway should be the priority. When managing critically patient's address the ABC's first. Go to the next page if you knew the correct answer, or click the link image(s) below to further research the concepts in this question (if desired).

Research Concepts: Sodium Channel Blocker Toxicity

We update eBooks quarterly and Apps daily based on user feedback. Please tap flag to report any questions that need improvement.

Question 348: A 30-year-old white female who is diagnosed with acute promyelocytic leukemia was started on all-trans retinoic acid (ATRA). On the third day of the therapy, the patient developed fever, peripheral edema, and shortness of breath. Her vital signs include a temperature of 102.3 F, blood pressure 90/58 mmHg, pulse 112 beats/min, pulse oximetry 84% on 10 L of high flow oxygen, and respiratory rate 24 breaths/min. Pertinent laboratory studies include WBC 31,000 cells/mL, hemoglobin 7 grams/dL, platelet count 50,000/mL, serum creatinine 2.4 mg/dL (baseline 0.9 mg/dL). A chest x-ray suggests pulmonary edema. She was transferred to the intensive care unit for further management. Which of the following is correct regarding the management of this condition?

Choices: 1. Add arsenic trioxide to ATRA 2. Supportive therapy and continue ATRA 3. Continue ATRA and start hydroxyurea 4. Discontinue ATRA and start the patient on IV dexamethasone 10 mg twice daily

Answer: 4 - Discontinue ATRA and start the patient on IV dexamethasone 10 mg twice daily Explanations: Differentiation syndrome is a cytokine release syndrome seen after initiation of differentiating agents characterized by fever, peripheral edema, pulmonary edema, and multi-organ dysfunction. It can be fatal, if not promptly recognized and treated. High dose systemic corticosteroids are helpful in these cases. All-trans retinoic acid (ATRA) can be continued in mild cases in addition to IV corticosteroids. ATRA must be discontinued in severe cases with multi-organ dysfunction and can be restarted when the differentiation syndrome resolves. Go to the next page if you knew the correct answer, or click the link image(s) below to further research the concepts in this question (if desired).

Research Concepts: Acute Promyelocytic Leukemia

We update eBooks quarterly and Apps daily based on user feedback. Please tap flag to report any questions that need improvement.

Question 349: A 55-year-old male with a medical history of hypothyroidism presented to the emergency department with complaints of severe insomnia, confusion, sweating, fever, and palpitations. On exam, blood pressure and heart rate were elevated, temperature 101 F, and generalized continuous fine muscle twitches movements in extremities were noted. The patient was admitted for further evaluation. Basic blood work including thyroid function panel was within normal limits. Brain MRI was normal. Cerebrospinal fluid analysis was also within normal limits. Next day patient developed agitation and hallucinations. Serum VGKC antibodies were found to be elevated. What is the most likely diagnosis?

Choices: 1. Morvan syndrome 2. Limbic encephalitis 3. Myxedema coma 4. Neuromyotonia

Answer: 1 - Morvan syndrome Explanations: The symptoms described are highly suggestive of Morvan syndrome which is predominant in men. It is characterized by central, autonomic and peripheral nervous system hyperactivity. Morvan syndrome is a clinical diagnosis. Insomnia, disorientation, confusion, amnesia, hallucinations, agitation, myokymia, neuropathic pain, loss of deep tendon reflexes, sensory deprivation, excessive sweating, pruritus, drooling, severe constipation, excessive lacrimation, cardiac arrhythmias, weight loss, palmoplantar erythema, hyponatremia due to SIADH comprise most of the symptoms caused by Morvan syndrome. Elevated serum antibodies against voltage-gated potassium channel (VGKC) complex proteins such as CASPR2 and LGI1 are quite diagnostic of MoS. Limbic encephalitis (LE) is a close differential of Morvan syndrome. However, subtle differences do exist. Severe insomnia, hyperhidrosis, and myokymia are characteristic of Morvan syndrome. Whereas LE has amnesia, seizures and temporal lobe structural abnormalities on imaging. LE differs from MoS with the conspicuous absence of neuromyotonia, dysautonomia. There are other biochemical and radiological differences also. Myxedema coma is a state of severely decompensated hypothyroidism. The patients are in a hypothyroid state. Unlike the above-mentioned patient, patients in myxedema coma show altered mentation, severe hypothermia, hypoglycemia, hypoxia, hypercapnia, bradycardia, hypotension, hyponatremia, and also hypoventilation. Thyroid functions are also abnormal. Neuromyotonia is characterized by peripheral nerve hyperexcitability only without overt central or autonomic nervous system dysfunction. Go to the next page if you knew the correct answer, or click the link image(s) below to further research the concepts in this question (if desired).

Research Concepts: Morvan Syndrome

We update eBooks quarterly and Apps daily based on user feedback. Please tap flag to report any questions that need improvement.

Question 350: A 38-year-old male with a past medical history of diabetes mellitus, hypertension, and endstage renal disease on hemodialysis presents with shortness of breath. He was found to have aspiration pneumonia and was intubated for severe hypoxemia, not responding to high flow nasal cannula oxygen. He was started on appropriate antibiotics and propofol for sedation. He was very dyssynchronous with the ventilator, so a midazolam drip was started to achieve more sedation. After two days, a sedation vacation was attempted, but the patient was not waking up even after 8 hours after turning all the sedatives off. What is the most likely explanation?

Choices: 1. Propofol remains for a longer time in patients with renal failure 2. He is likely sleeping 3. He likely had a stroke while he was sedated 4. Midazolam's metabolite is renally excreted and so it should be avoided in patients with endstage renal disease

Answer: 4 - Midazolam's metabolite is renally excreted and so it should be avoided in patients with end-stage renal disease

Explanations: The medications used to initiate and maintain sedation within an intensive care unit setting include benzodiazepines such as diazepam, lorazepam, and midazolam; opioid analgesics such as fentanyl; hydromorphone, morphine, remifentanil, propofol, dexmedetomidine, and ketamine; and antipsychotics such as haloperidol, quetiapine, and ziprasidone. No sedative is found to be superior in efficacy or mortality. The Society of Critical Care Medicine guidelines recommends avoiding benzodiazepines due to evidence of the longer duration of intubation, especially in patients with renal impairment. While midazolam is metabolized by the liver, its main metabolite, alpha hydroxymidazolam, is renally excreted. The choice of which sedative is best lies in the practitioner's clinical assessment of individual patient scenarios, weighing the risk/benefit profile of the medicine for each patient. Go to the next page if you knew the correct answer, or click the link image(s) below to further research the concepts in this question (if desired).

Research Concepts: Sedation Vacation in the ICU

We update eBooks quarterly and Apps daily based on user feedback. Please tap flag to report any questions that need improvement.

Question 351: A 55-year-old male with a history of decompensated cirrhosis secondary to cryptogenic cirrhosis and recurrent ascites on furosemide and spironolactone presents with a worsening abdominal distention and diffuse abdominal pain. He also reports a decrease in urine output over the past week. His physical exam is significant for ascites and mild abdominal tenderness. His vitals reveal a blood pressure of 93/59 mmHg and pulse of 85/min. Labs are significant for serum creatinine 2 mg/dL (from normal baseline one month ago) and potassium 3.9 mEq/L. Urine analysis is negative for casts or proteinuria. Renal ultrasound is negative for hydronephrosis. Paracentesis is suggestive of spontaneous bacterial peritonitis, and he is started on ceftriaxone. For his kidney injury, he is resuscitated with IV albumin 1 g/kg for two days without improvement in his urine output or his kidney function. What is the next best step in the management of this patient?

Choices: 1. Continue albumin 2. Hemodialysis 3. Transjugular intrahepatic portosystemic shunt 4. Start octreotide and midodrine

Answer: 4 - Start octreotide and midodrine Explanations: This patient likely has hepatorenal syndrome (HRS). The majority of patients with cirrhosis and acute kidney injury are dehydrated, and their diuretics should be held, and they should be started on IV hydration. Lack of improvement in renal function after volume expansion with intravenous albumin (1 g/kg/day) for at least two days and withdrawal of diuretics with the exclusion of other etiologies of kidney injury makes the diagnosis of HRS. Patients with the diagnosis of HRS should be started on albumin for volume expansion and octreotide and midodrine to increase and maintain systemic vascular resistance, blood pressure, and eventually renal perfusion. Albumin alone will not be enough to maintain his systemic vascular resistance and increase his renal perfusion. There is no indication for hemodialysis at this point. Patients who do not respond to medical therapy and are considered well enough can be considered for transjugular intrahepatic portosystemic shunt as it is sometimes successful in improving kidney function. Go to the next page if you knew the correct answer, or click the link image(s) below to further research the concepts in this question (if desired).

Research Concepts: Hepatorenal Syndrome

We update eBooks quarterly and Apps daily based on user feedback. Please tap flag to report any questions that need improvement.

Question 352: An African American female is brought to the emergency department with acute onset shortness of breath that has progressively worsened over the past 4 hours at home. She has a peak flow meter at home, and the number prior to arrival was 220 L, which is 51% of predicted. She has known bronchial asthma but hasn't had an exacerbation requiring an ED visit or hospitalization before. Her FEV1 in spirometry six months back was at 89%. After 2 hours of management in the emergency department, she feels significantly better and is able to speak in short sentences. Last breathing treatment was 15 minutes back. Repeat PEFR is 310 L/min, which is 68% of predicted. Her wheezing has improved, and she wants to go home. Which of the following is the next best step in the management of this patient?

Choices: 1. Discharge her home as her initial PEFR was above 50% and 200 L/min 2. DIscharge her home as PEFR improved by 12% and above 300 L/min 3. Assess one more peak flow and symptoms in another 2 hours in the ED 4. Discharge her home as she has normal FEV1 and no prior history of severe exacerbation

Answer: 3 - Assess one more peak flow and symptoms in another 2 hours in the ED Explanations: Kelsen and colleagues in a study showed a 50% relapse rate in patients treated for 2 hours or less in a facility as opposed to 4% in those treated and observed for an additional 2 to 4 hours. Therefore even though she feels better, she should be requested to stay back in ED for at least 4 hours. In a study by Stein and Cole, an adequate response to treatment in ED was characterized as visual improvement in symptoms which sustains 30 minutes or beyond the last bronchodilator dose, and a PEFR greater than 70% of predicted. Her response immediately post-treatment was 67%. In the same study by Stein and Cole, initial PEFR on presentation did not predict the need for hospitalization. An adequate response to treatment was also characterized by an improvement in FEV1 by 10% or above 70% of predicted, although it may not be the most practical approach at the bedside. Go to the next page if you knew the correct answer, or click the link image(s) below to further research the concepts in this question (if desired).

Research Concepts: Status Asthmaticus

We update eBooks quarterly and Apps daily based on user feedback. Please tap flag to report any questions that need improvement.

Question 353: A 41-year-old female with a past medical history of hypertension and type 2 diabetes mellitus presents to the emergency department in status epilepticus. Seizures are eventually relieved per seizure protocol though the patient does not regain consciousness. She is intubated and mechanically ventilated due to a Glasgow coma scale of 6. Her mother who came with her in the ambulance explains that the patient has no history of seizures and that she had a cough for the past few weeks. The patient has also had a few recent episodes of headaches and confusion. The patient’s mother explains that the patient has one healthy sister and two brothers who died when they were babies. The patient’s ammonia level is found to be 525 micromol/L and a blood sugar of 140 mg/dL. What is the next step to care for this patient?

Choices: 1. Genetic consultation 2. Endocrinology consultation 3. Dietary consultation 4. Nephrology consultation

Answer: 4 - Nephrology consultation Explanations: For suspected ornithine transcarbamylase deficiency (OTCD), this patient needs urgent hemodialysis, making nephrology consult the best next step. With ammonia levels above 500 micromol/L, hemodialysis should be done emergently. Ammonia levels above 800 micromol/L are associated with severe neuralgic damage, limiting treatment options. Using arginine as well as a combination of sodium benzoate and sodium phenylbutyrate help reduce ammonia by using alternative pathways nitrogen elimination. Go to the next page if you knew the correct answer, or click the link image(s) below to further research the concepts in this question (if desired).

Research Concepts: Ornithine Transcarbamylase Deficiency

We update eBooks quarterly and Apps daily based on user feedback. Please tap flag to report any questions that need improvement.

Question 354: A patient is involved in a motor vehicle accident from which he was thrown through the windshield and landed flush on the pavement. He suffered multiple injuries and requires intubation at the scene. His vitals are labile, and he is rushed to the hospital. Because of poor IV access, an intraosseous line is inserted in the right lower leg. A surgeon is called to perform a cutdown on a neck vein at the same time. The patient is resuscitated and stabilized. A CT scan reveals a fracture of the pelvis and four ribs in the left chest. Radiographs of the lower extremity show a comminuted tibial shaft fracture. After application of an external fixator, the patient is taken to the intensive care unit. Late in the night, the nurse notes no pulses in the right leg which is now tense, ecchymotic, and he is unable to dorsiflex his toes and ankle. Which of the following muscles is not involved in his pathology?

Choices: 1. Extensor hallucis longus 2. Peroneus brevis 3. Extensor digitorum longus 4. Tibialis anterior

Answer: 2 - Peroneus brevis Explanations: Along with other muscles of the anterior compartment that include extensor hallucis longus, peroneus tertius, and extensor digitorum longus, the tibialis anterior is prone to acute compartment syndrome. Trauma to the leg is the most common cause of acute compartment syndrome. Other causes include surgery, IV fluids, intraosseous infusion, intravenous chemotherapeutic agents, military anti-shock trousers, and tight splints. If the compartment syndrome is unrecognized, it can lead to ischemia and necrosis of the muscle and nerves. Go to the next page if you knew the correct answer, or click the link image(s) below to further research the concepts in this question (if desired).

Research Concepts: Tibial Anterior Compartment Syndrome

We update eBooks quarterly and Apps daily based on user feedback. Please tap flag to report any questions that need improvement.

Question 355: A 17-year-old female who was in a motorcycle accident comes to the emergency department. She is groaning and speaking in a low voice. She has multiple abrasions, a blood pressure of 80/50 mmHg, and a respiratory rate of 22. She is cold to the touch, and there is extensive subcutaneous emphysema in the right upper chest and neck. A FAST exam is performed. What is the next step in management?

Choices: 1. Chest x-ray 2. Arterial blood gas 3. Tube thoracostomy 4. Fluid resuscitation

Answer: 3 - Tube thoracostomy Explanations: Airways should be the top priority. One should always be suspicious of a reversible cause of respiratory distress. Subcutaneous emphysema with respiratory distress is a pneumothorax until proven otherwise. If available FAST imaging should be done. If not, the tube is placed at the fifth intercostal and the midaxillary line. In the field, needle decompression can be done. In the emergency department, tube thoracostomy is preferred. An arrow catheter can be used and later converted. Go to the next page if you knew the correct answer, or click the link image(s) below to further research the concepts in this question (if desired).

Research Concepts: Tension Pneumothorax

We update eBooks quarterly and Apps daily based on user feedback. Please tap flag to report any questions that need improvement.

Question 356: A 65-year-old male presents to the emergency department with a chief complaint of weakness following mowing the yard. The patient appears alert and oriented, wellnourished, well dressed, and slightly agitated. Vitals include a blood pressure of 106/58 mmHg, heart rate of 180 bpm, respiratory rate of 24/minutes, and oxygen saturation of 94% on room air. An ECG performed shows 2:1 atrial flutter. Past medical history is remarkable for a history of recurrent atrial fibrillation. The patient is currently taking simvastatin, clopidogrel, and metoprolol. The provider administers ibutilide, 1 mg over 10 minutes, without a change in rhythm and decides to switch antiarrhythmics. Which antiarrhythmic is contraindicated in this patient?

Choices: 1. Amiodarone 2. Dofetilide 3. Digoxin 4. Propafenone

Answer: 2 - Dofetilide Explanations: Amiodarone is a class 3 antiarrhythmic that has been shown to have no significant effect on the proarrhythmic potential of ibutilide and may be used safely in combination. Dofetilide is a class 3 antiarrhythmic with a mechanism of action similar to ibutilide. The administration of dofetilide in the presence of ibutilide is contraindicated due to the potential for excessive prolongation of the QT interval. Digoxin can be used safely in the presence of ibutilide. Propafenone is a class 1C antiarrhythmic that can be used safely in the presence of ibutilide. Go to the next page if you knew the correct answer, or click the link image(s) below to further research the concepts in this question (if desired).

Research Concepts: Ibutilide

We update eBooks quarterly and Apps daily based on user feedback. Please tap flag to report any questions that need improvement.

Question 357: A 56-year-old presented with fever, chills, dyspnea, and fatigue. The temperature is documented as high as 103.5 degrees Fahrenheit. He has a history of rheumatic heart disease with valve replacement surgery done three years ago with the left-sided mechanical valve in situ. The blood samples are drawn, and he is started on empirical antibiotics coverage for infective endocarditis. Blood samples showed candidal growth on day 4. Echocardiography showed an ejection fraction of 35% with mitral regurgitation. What should be the next step in the management of the patient?

Choices: 1. Continue antibiotics coverage 2. Start antifungal therapy 3. Start antifungal therapy and plan for early surgery 4. Wait for 1 week to see if culture come back positive for bacteria

Answer: 3 - Start antifungal therapy and plan for early surgery Explanations: From an initial presentation, it is very difficult to differentiate fungal endocarditis from bacterial endocarditis. The sign and symptoms often coincide. Prosthetic valve endocarditis caused by fungi or other highly resistant organisms warrants early surgery. Explanted valve or vegetations obtained from surgery is sent for further histopathological examination and culture. The initial antifungal treatment for Candida spp. endocarditis should be lipid formulation of amphotericin B with or without flucytosine or a high dose echinocandin (caspofungin, micafungin, or anidulafungin). Plan for surgery in case of persistent bacteremia in patients with prosthetic valve despite the use of appropriate antibiotics for 5 to 7 days. Rule out other sources of infection. Go to the next page if you knew the correct answer, or click the link image(s) below to further research the concepts in this question (if desired).

Research Concepts: Fungal Endocarditis

We update eBooks quarterly and Apps daily based on user feedback. Please tap flag to report any questions that need improvement.

Question 358: A 16-year-old female presents to the hospital with high-grade fever and headache for the last 2 days. She has had an upper respiratory infection for the last 2 weeks as well. On examination, she has right-side weakness and numbness. MRI brain shows cortical and deep grey matter enhancing lesions. Lumbar puncture revealed lymphocytic pleocytosis without any oligoclonal bands. Which medication has been shown to shorten the duration of neurological symptoms and halt the progression of the disease?

Choices: 1. Acetaminophen 2. Corticosteroids 3. Ibuprofen 4. Antibiotics

Answer: 2 - Corticosteroids Explanations: Acute disseminated encephalomyelitis (ADEM), also referred to as post-infectious encephalomyelitis, is an acute, rapidly progressive, autoimmune process that occurs in the central nervous system. ADEM is characterized by demyelination in the brain and spinal cord (and occasionally the optic nerve), as a result of inflammation that occurs in response to a preceding infection or immunization. Corticosteroids shorten the duration of neurological symptoms and halt the progression of the disease in acute disseminated encephalomyelitis. ADEM is treated with immunosuppression with high-dose intravenous glucocorticoids. These can be started simultaneously with acyclovir and/or antibiotics on the initial patient presentation. If a patient is not improving or is responding poorly to glucocorticoid treatment, try intravenous immune globulin (IVIG), plasma exchange, or cyclophosphamide. Ibuprofen is used for body pains, but it does not shorten the duration of neurological symptoms. Acetaminophen is used for fever management, but it does not shorten the duration of neurological symptoms. Go to the next page if you knew the correct answer, or click the link image(s) below to further research the concepts in this question (if desired).

Research Concepts: Acute Disseminated Encephalomyelitis

We update eBooks quarterly and Apps daily based on user feedback. Please tap flag to report any questions that need improvement.

Question 359: A 65-year-old Vietnam veteran with no medical comorbidities was admitted to intensive care unit in a tertiary care center following the sudden onset of high-grade fever, intense headache, cough, and hemoptysis. He was profoundly hypoxic at admission needing intubation and mechanical ventilation. Inotropes were needed for septic shock. Bloodwork showed leukocytosis with 13% bands, hemoglobin 12.4 g/dL, mild thrombocytopenia, serum creatinine, electrolytes, and blood urea were normal. Liver enzymes included aspartate aminotransferase 70 U/L, alanine aminotransferase 85 U/L, alkaline phosphatase and serum total bilirubin were normal. CT chest with contrast revealed multilobar infiltrates, most pronounced in the right middle and lower lobes, associated with a moderate sized effusion. A week ago he returned from a week-long stay in Martha's Vineyard at his son's house, where he also mowed the lawn for one full day and pulverized a rabbit. Blood cultures were drawn and he was started on vancomycin, cefepime, and azithromycin. In 48 hours his FiO2 was increased from 40% to 70%, platelet counts were 42,000/microliter and infiltrates increased on chest x-ray. What is the most appropriate next step?

Choices: 1. Add IV doxycycline 2. Change cefepime to meropenem 3. Thoracentesis to rule out empyema 4. Add IV gentamicin

Answer: 4 - Add IV gentamicin Explanations: Tularemia pneumonia must be included in the differential diagnosis of severe communityacquired pneumonia because of recent travel to Martha's Vineyard and using a lawn mower, both of which have been classically associated with previous outbreaks of pneumonic tularemia. Necrotizing pneumonia occurs following inhalation of aerosolized Francisella tularensis, either from accidental pulverization of flea infested or infected rabbits or rodents by a lawn mower or from soil containing excreta of an infected rodent. Streptomycin or gentamicin are first-line agents for the treatment of tularemia. Empiric therapy is indicated soon after suspicion arises. Diagnosis is usually made by serology, due to the high risk of inhalational hazard to the microbiologist by culturing this organism. Go to the next page if you knew the correct answer, or click the link image(s) below to further research the concepts in this question (if desired).

Research Concepts: Tularemia

We update eBooks quarterly and Apps daily based on user feedback. Please tap flag to report any questions that need improvement.

Question 360: A 65-year-old female who is diagnosed with liver chronic liver disease presents with fever, generalized abdominal pain, and confusion. Abdominal examination reveals abdominal distension with marked abdominal tenderness. Ultrasound confirms the presence of fluid in the peritoneal cavity. Which of the following is the most likely diagnosis?

Choices: 1. Fulminant hepatic failure 2. Abdominal tuberculosis 3. Spontaneous bacterial peritonitis 4. Perforated peptic ulcer

Answer: 3 - Spontaneous bacterial peritonitis Explanations: Spontaneous bacterial peritonitis (SBP) is an infection of a peritoneal cavity in a patient with ascites. It presents with fever and abdominal pain. SBP can precipitate hepatic encephalopathy resulting in confusion. The most important diagnostic investigation is a diagnostic tap with subsequent biochemical and cytological analysis of the ascitic fluid. Go to the next page if you knew the correct answer, or click the link image(s) below to further research the concepts in this question (if desired).

Research Concepts: Chronic Liver Disease

We update eBooks quarterly and Apps daily based on user feedback. Please tap flag to report any questions that need improvement.

Question 361: A patient presents for an electrophysiology study following the incidental finding of nonsustained ventricular tachycardia in an ambulatory surgery center. The electrophysiologist decides to administer a nonselective beta agonist during the exam. Which of the following would be most appropriately categorized as a relative contraindication to the use of this medication?

Choices: 1. Hypotension 2. Acute myocardial infarction 3. Pulmonary hypertension 4. Digoxin intoxication

Answer: 1 - Hypotension Explanations: Isoproterenol is a nonselective beta agonist that will induce hypotension via its effects on beta 2 receptors. Hypotension is a relative contraindication to isoproterenol administration. Acute myocardial infarction is an absolute contraindication to the use of isoproterenol. Isoproterenol can cause elevated blood glucose levels due to the induction of glycogenolysis via beta-2 receptors in the liver. Digoxin intoxication is an absolute contraindication to the usage of isoproterenol. Go to the next page if you knew the correct answer, or click the link image(s) below to further research the concepts in this question (if desired).

Research Concepts: Isoproterenol

We update eBooks quarterly and Apps daily based on user feedback. Please tap flag to report any questions that need improvement.

Question 362: A 17-year-old female presents with a diagnosis of mononucleosis made three weeks ago by her provider. Her mother reports she has steadily worsened since that time despite corticosteroids and one additional visit for intravenous fluids. More recently, her daughter has developed a productive cough, and she has noticed some right lateral neck swelling. She has had near continuous fever, and also a persistent sore throat. Vital signs show a fever of 102 F, pulse 120 bpm, blood pressure of 115/70 mmHg, and 100% oxygen saturation on room air. Exam shows a sick but non-toxic appearing female with a "cord sign" present deep to the anterior border of the sternocleidomastoid on the right side. Also, she has palpable lymphadenopathy present bilaterally. What imaging study listed below is most sensitive for confirming the diagnosis?

Choices: 1. Chest x-ray 2. Non-contrast CT of the neck 3. CT of the neck with intravenous contrast 4. Doppler ultrasound of the neck

Answer: 3 - CT of the neck with intravenous contrast Explanations: The patient presents with Lemierre syndrome. This is a rare condition, with an estimated incidence of 1/1,000,000. This diagnosis can be very difficult to make early in the course of the illness when the symptoms are very similar to common self-limited oropharyngeal infections. As the disease progresses, tenderness in the lateral neck becomes more prominent. A "cord sign," which represents thrombosis of the internal jugular vein, and presents as tenderness and swelling deep to the anterior border of the sternocleidomastoid, may be present. The imaging modality most used to establish the diagnosis of Lemierre syndrome is CT with IV contrast. The chest should be included as septic emboli are likely. It has the advantage of being able to detect internal jugular vein thrombosis as well as pulmonary septic emboli. Magnetic resonance venography (MRV) is likely more sensitive in the diagnosis, but it is a more difficult study to obtain and requires a hemodynamically stable patient. Ultrasound is useful to follow clot progression, but may miss acute clot and is not as sensitive secondary to the anatomic constraints of the neck and the lack of fibrin in a fresh clot. In addition to internal jugular vein thrombosis shown on imaging studies, other findings indicative of Lemierre syndrome include (1) Isolation of the causative organism from blood cultures. Fusobacterium necrophorum is the most common. (2) Evidence for septic emboli. These are most commonly seen in the lung but may affect almost any organ. There may be laboratory evidence of end-organ dysfunction or abscess. (3) Precedent oropharyngeal infection. (4) Evidence of sepsis. A high degree of clinical suspicion must be used to make a diagnosis of Lemierre syndrome. A healthy patient who is clinically deteriorating after a precedent oropharyngeal infection, especially with evidence of septic emboli should prompt the clinician to order the appropriate study to evaluate for internal jugular vein thrombosis. Go to the next page if you knew the correct answer, or click the link image(s) below to further research the concepts in this question (if desired).

Research Concepts: Lemierre Syndrome

We update eBooks quarterly and Apps daily based on user feedback. Please tap flag to report any questions that need improvement.

Question 363: A 50-year-old female polytrauma patient is admitted to the ICU. The patient is intubated and sedated. You get called by the nurse for a swollen left forearm. You order Xrays which show a both-bone forearm fracture. The compartments are soft and not tense, but very swollen. The patient is not arousable. Her blood pressure measures 100/60 mm Hg, and her heart rate is 105 bpm. Which of the following represents the correct logical sequence of events for the management in this patient?

Choices: 1. Immediate splinting of the fracture, followed by ice and elevation and serial exams 2. Stryker measurement shows forearm compartment pressures of 35 mm Hg. You take the patient immediately to the operating for the release of compartments because of an absolute pressure greater than 30 mm Hg. 3. Stryker measurement shows forearm compartment pressures of 35 mm Hg. You take the patient immediately to the operating for the release of compartments because of a delta pressure (delta p) less than 30. 4. Ice and elevation, followed by closed reduction and splinting and rechecking the compartment pressures every hour

Answer: 1 - Immediate splinting of the fracture, followed by ice and elevation and serial exams

Explanations: The delta pressure or difference between compartment pressure and diastolic blood pressure is less than 30 mm Hg, which is more specific to compartment syndrome than an absolute pressure of greater than 30 mm Hg. This patient has both a delta pressure less than 30 mm Hg and absolute compartment pressure greater than 30 mm Hg; there is no role for continued exams. She needs to be sent down to the OR immediately for the release of compartments. In patients whose clinical exam is limited, Stryker monitoring of compartment pressures is vital. In an awake and alert patient, a clinical exam, especially pain with passive stretch of the digits in extension is sufficient for the diagnosis of compartment syndrome. Go to the next page if you knew the correct answer, or click the link image(s) below to further research the concepts in this question (if desired).

Research Concepts: Forearm Compartment Syndrome

We update eBooks quarterly and Apps daily based on user feedback. Please tap flag to report any questions that need improvement.

Question 364: A 65-year-old female with a history of diabetes mellitus type 2 and alcohol use disorder presents with new onset jaundice and abdominal distention. She has been drinking 5-6 alcohol beverages per day for the past 20 years. Her vitals are significant for a blood pressure of 98/67 mmHg with a normal heart rate. She is afebrile. Her exam is significant for hepatomegaly and jaundice. Her labs are significant for WBC count of 6000/microL, platelet count 130000/microL, ALT 140 IU/L, AST 220 IU/L, ALP 340 IU/L, and total bilirubin 8 mg/dL with a direct fraction of 6.9 mg/dL. Abdominal ultrasound shows large ascites with hepatomegaly. She undergoes paracentesis that is significant for a serum-ascites albumin gradient (SAAG) of 1.3, an ascitic protein of 1.5 mg/dL and neutrophils of 67/microL. What is the most likely cause of her elevated SAAG?

Choices: 1. Increased blood flow within the splanchnic circulation 2. Decreased resistance within the liver 3. Increased right-sided cardiac pressure 4. Increase in systemic vascular resistance

Answer: 1 - Increased blood flow within the splanchnic circulation Explanations: This patient likely has alcoholic hepatitis with evidence of portal hypertension given the development of ascites with elevated SAAG and thrombocytopenia. Portal hypertension usually develops secondary to the increased resistance within the liver due to structural or dynamic changes. Structural changes are due to the alteration of the hepatic microcirculation such as hepatic stellate cell activation and the resultant fibrosis, regenerative nodules, vascular occlusion, and angiogenesis. The increased production of endothelial vasoconstrictors and decreased release of vasodilators within the liver leads to sinusoidal constriction. Portal hypertension stemming from this is augmented and perpetuated by the increased blood flow within the splanchnic circulation. This increased blood flow is due to the increased release of splanchnic vasodilators because of increased shear stress and reduced effective arterial volume. Thus, portal hypertension is a result of both increased resistance to portal venous flow and increased portal blood flow due to splanchnic vasodilation. Increase resistance within the liver is associated with portal hypertension. There is no evidence of elevated right-sided cardiac pressures given the normal JVP, and the ascitic protein is less than 2.5, which indicates a hepatic etiology if the ascites than cardiac. There is a decrease in the systemic vascular resistance in portal hypertension due to the release of splanchnic vasodilators. Go to the next page if you knew the correct answer, or click the link image(s) below to further research the concepts in this question (if desired).

Research Concepts: Portal Hypertension

We update eBooks quarterly and Apps daily based on user feedback. Please tap flag to report any questions that need improvement.

Question 365: A 54-year-old female with acute on chronic systolic congestive heart failure and acute hypoxemic respiratory failure is now day 3 on mechanical ventilation. Diuretics and inotropes prescribed resulted in diuresis and improvement in hypoxemia. Vital signs show a heart rate of 94/min, blood pressure 112/68 mmHg, respiratory rate 16/min, and SpO2 98%. She is sedated with fentanyl and propofol and opens her eyes to voice but does not follow commands. Current ventilator settings are pressure control mode, respiratory rate 16/min, driving pressure 20 cmH2O, PEEP 8 cmH2O, and FiO2 40%. Average tidal volume is 300 mL on these settings. An arterial blood gas shows pH 7.47, pCO2 38 mmHg, and pO2 88 mmHg. Sedation is stopped and 20 minutes later, the patient is alert and following commands. PSV mode is started, with driving pressure 20 cmH2O, PEEP 8 cmH2O, and FiO2 40%. After 10 minutes on these settings, the respiratory rate is now 28/min with average tidal volume 420 mL. Current vital signs are a pulse rate of 120/min, blood pressure 108/58 mmHg, respiratory rate 28/min, and SpO2 94%. The patient appears anxious. Chest X-ray shows stable cardiomegaly, clear lungs, and endotracheal tube in the appropriate position. An arterial blood gas shows pH 7.52, pCO2 30 mmHg, and pO2 68 mmHg. What is the best explanation for the patient's tachypnea?

Choices: 1. Oxygen consumption has increased 2. Airway resistance has increased 3. Oxygen delivery has decreased 4. Lung compliance has decreased

Answer: 1 - Oxygen consumption has increased Explanations: Work of breathing and thus oxygen consumption is higher in PSV than in control modes of ventilation. Patients with shock or low cardiac output may need more respiratory support. The flow delivered by the driving pressure can provide a tidal volume and minute ventilation higher than the patient could achieve without ventilator support. This higher minute ventilation improves oxygen delivery and carbon dioxide offloading. PSV is relatively contraindicated in patients who have a depressed respiratory drive, very high oxygen consumption, or high airway resistance. After PSV is initiated, the patient should be directly observed for several minutes to ensure that the goals of ventilation, oxygenation, and patient comfort are met. Go to the next page if you knew the correct answer, or click the link image(s) below to further research the concepts in this question (if desired).

Research Concepts: Pressure Support Ventilation

We update eBooks quarterly and Apps daily based on user feedback. Please tap flag to report any questions that need improvement.

Question 366: A 55-year-old male with PMH of asthma/ COPD overlap syndrome was admitted to the ICU for acute hypercapnic respiratory failure. He failed to non-invasive positive pressure ventilation and had to be emergently intubated for increased work of breathing. He was treated with intravenous steroids, bronchodilators, and his exam revealed audible wheezing. On Day 3 of being on a mechanical ventilator, he started following commands during the spontaneous awakening trial. Still, he became tachypneic with a respiratory rate of 32 during the spontaneous breathing trial (SBT) on a CPAP of 5 cm H2O. His rapid shallow breathing index was 116 breaths/minute/liter. What is the best next step in the management of this patient?

Choices: 1. Extubate the patient to CPAP 2. Obtain ABG 3. Administer a dose of succinylcholine and extubate the patient 4. Place the patient back on the assist control mode of mechanical ventilation

Answer: 4 - Place the patient back on the assist control mode of mechanical ventilation Explanations: A variety of respiratory, cardiac, psychological, neurologic, neuromuscular, and equipment related causes leads to difficulty in weaning patients from ventilation. Patients with RSBI less than 105 are likely to be successfully extubated and RSBI more than 105 are at increased risk for extubation failure. SBT should be performed only once a day. Several SBTs a day offer no additional benefit compared with one. The above patient with audible wheezing and tachypneic on SBT is most likely to fail extubation and should not be extubated until his underlying condition improves. Go to the next page if you knew the correct answer, or click the link image(s) below to further research the concepts in this question (if desired).

Research Concepts: Ventilator Weaning

We update eBooks quarterly and Apps daily based on user feedback. Please tap flag to report any questions that need improvement.

Question 367: An 80-year-old female presents to the emergency department with worsening shortness of breath over the past 2 days. She denies associated fevers or cough. She has a past medical history of idiopathic pulmonary fibrosis, chronic hypoxemic respiratory failure, chronic kidney disease, and ischemic cardiomyopathy. Her home medications include atorvastatin, aspirin, metoprolol, nintedanib, and nitroglycerin as needed or chest pain. She uses 3 L of oxygen at home. On examination, she is in mild respiratory distress. Her jugular venous pressure is elevated. Crackles are heard bilaterally. She has 2+ pitting edema. Her vitals are the following: BP 134/82 mmHg, HR 100 bpm, T 99.1 F (37.3 C), and sat 88% on 3L oxygen. She has a 6 lb (2.7 kg) weight increase from her baseline weight. A chest x-ray showed diffused interstitial chronic fibrotic changes. A CT scan shows stable interstitial fibrotic changes with superimposed edema, and an echocardiogram shows reduced ejection fraction. What is the best next step in management?

Choices: 1. High dose corticosteroids 2. Referral for a lung transplant 3. Bronchodilators 4. Furosemide

Answer: 4 - Furosemide Explanations: Managing a patient with interstitial lung disease (ILD) with other comorbidities can be difficult. Differentiating an acute exacerbation of ILD from any other causes comprises a challenge. Entities such as pulmonary edema or pneumonia should be excluded especially in the appropriate clinical setting. The patient's clinical history, physical exam, and imaging support a diagnosis of fluid overload secondary to heart failure. The most appropriate treatment is diuretics. Go to the next page if you knew the correct answer, or click the link image(s) below to further research the concepts in this question (if desired).

Research Concepts: Interstitial Lung Disease

We update eBooks quarterly and Apps daily based on user feedback. Please tap flag to report any questions that need improvement.

Question 368: A 38-year-old man presents to the emergency department after a 3-day history of right-sided neck pain and headache. He has no significant past medical history. He took aspirin without relief but became concerned when he had a 10-minute episode of confusion associated with left-hand numbness and clumsiness. The exam shows mild right ptosis, anisocoria with the right pupil being smaller, and left arm drift. Doppler ultrasonography reveals a thrombus and possible right-sided internal carotid artery dissection. Which of the following is the next best step in the management of this patient?

Choices: 1. Intravenous heparin 2. Oral clopidogrel 3. Arterial stenting 4. Reassurance

Answer: 1 - Intravenous heparin Explanations: This presentation warrants emergent CT angiography of the internal carotid artery. When a thrombus is detected, anticoagulant therapy should be initiated. Once hemorrhage has been ruled out, intravenous heparin followed by warfarin is typically used to prevent thromboembolic complications. If systemic anticoagulation is contraindicated, antiplatelet therapy can be used. Angioplasty and stenting may be needed if there are persistent ischemic symptoms. Go to the next page if you knew the correct answer, or click the link image(s) below to further research the concepts in this question (if desired).

Research Concepts: Carotid Artery Dissection

We update eBooks quarterly and Apps daily based on user feedback. Please tap flag to report any questions that need improvement.

Question 369: A 32-year-old pregnant female presents to the emergency department with bleeding gums and skin rashes, which started 24 hours ago. She is gravida 3 with two previous live births. She is currently 24 weeks pregnant. She was diagnosed with immune thrombocytopenia during her first pregnancy when she presented with similar symptoms. Her second pregnancy was uneventful. Physical exam shows bleeding gums and petechiae in the oral mucosa, groin area, and in the feet bilaterally. Labs are significant for a platelet count of 2000/microL. Other labs are unremarkable. Review of peripheral smear shows marked thrombocytopenia and occasional giant platelets. She is again diagnosed with immune thrombocytopenia. She had responded to steroids and intravenous immunoglobulins in the previous pregnancy, however, this time she does not respond to steroids and intravenous immunoglobulins. What is the next best step in the management of this patient?

Choices: 1. Thrombopoietin receptor agonist 2. Rituximab 3. Mycophenolate 4. Platelet transfusion

Answer: 2 - Rituximab Explanations: Rituximab is an acceptable option as a second-line treatment for ITP in pregnancy. It is labeled category C by the FDA, but several reports have established its safety in pregnancy. It is associated with transient B-cell lymphopenia in neonate which resolves by the 6th month of life. Splenectomy is also safe in pregnancy, but medical management is preferred over invasive methods. Platelet transfusion is almost never useful in a patient with immune thrombocytopenia. Mycophenolate is contraindicated in pregnancy. Go to the next page if you knew the correct answer, or click the link image(s) below to further research the concepts in this question (if desired).

Research Concepts: Thrombocytopenia in Pregnancy

We update eBooks quarterly and Apps daily based on user feedback. Please tap flag to report any questions that need improvement.

Question 370: A 66-year-old female patient with a history of obstructive sleep apnea, hypertension, diabetes mellitus, class 3 obesity, and asthma presents with five days of worsening dyspnea and associated wheezing and cough. Her vital signs on presentation are blood pressure 168/70 mmHg, oxygen saturation 85% on room air, respiratory rate 32 breaths per minute, and pulse 110 beats per minute. On exam, she has diffuse expiratory wheezing. A chest x-ray does not show any significant infiltrates. Her labs are grossly normal, except her arterial blood gas (ABG) shows a pH of 7.2, pCO2 56 mmHg, pO2 55 mmHg, and bicarbonate of 21 mmol/L. Nebulization and corticosteroids are started. She is placed on BiPAP and transferred to a stepdown unit. Repeat ABG shows a pH of 7.1 and pCO2 of 70 mmHg. She is prepared to be intubated and moved to the ICU. Rapid sequence intubation is attempted but fails after four attempts. The patient develops bradycardia then cardiac arrest. Which of the following could have been the cause of intubation failure?

Choices: 1. Placement of two fingers in the superior laryngeal notch 2. Placement of three fingers on the floor of the mandible between the mental angle and the neck near the hyoid bone 3. Placement of two fingers between the incisors 4. Placement of three fingers between the incisors

Answer: 3 - Placement of two fingers between the incisors Explanations: Measurement of three fingers between the upper and lower teeth of the open mouth of a patient indicates the ease of access to the airway through the oral opening. A typical patient can open their mouth sufficiently to permit the placement of three of their fingers between the incisors. Adequate mouth opening facilitates both insertions of the laryngoscope and obtaining a direct view of the glottis. Measurement of three fingers from the anterior tip of the mandible to the anterior neck provides an estimate of the volume of the submandibular space. A typical patient can place three fingers on the floor of the mandible between the mental angle and the neck near the hyoid bone. Normally this distance should measure close to 7 cm. If this distance is less than three finger-widths, the laryngeal axis will be at a more acute angle with the pharyngeal axis, indicating that alignment of the oral opening to the pharyngeal opening will be difficult. It also indicates that there will be less space to displace the tongue within the throat. The rule has limitations as the distance can vary according to height and ethnicity. For this reason, an alternative in the form of a ratio of height to thyromental distance (RHTMD) has been suggested. Measurement of two fingers between the floor of the mandible to the thyroid notch on the anterior neck identifies the location of the larynx relative to the base of the tongue. A typical patient can place two fingers in the superior laryngeal notch. If the larynx is too high in the neck, measuring less than two fingers, direct laryngoscopy will be difficult and potentially impossible; this is because the angle between the base of the tongue to the larynx is too acute to be negotiated for direct visualization of the larynx easily. Go to the next page if you knew the correct answer, or click the link image(s) below to further research the concepts in this question (if desired).

Research Concepts: 3-3-2 Rule

We update eBooks quarterly and Apps daily based on user feedback. Please tap flag to report any questions that need improvement.

Question 371: A 60-year-old male with history of hypertension, hyperlipidemia, and insulin dependent diabetes mellitus presents to the emergency department for substernal chest pain with radiation to the left side of his jaw. He is found to have ST segment elevations on EKG with a new right bundle branch block. Which artery is most likely affected?

Choices: 1. Left marginal coronary artery 2. Left anterior descending coronary artery 3. Sinoatrial node artery 4. Right marginal coronary artery

Answer: 2 - Left anterior descending coronary artery Explanations: The right bundle receives most of its blood supply from the left anterior descending coronary artery In the setting of acute myocardial infarction, a right bundle branch block is associated with increased mortality. ST segment elevations are not affected by a right bundle branch block. The right bundle also receives collateral blood supply from the right coronary artery or left circumflex coronary artery, depending on the dominance of the heart. Go to the next page if you knew the correct answer, or click the link image(s) below to further research the concepts in this question (if desired).

Research Concepts: Right Bundle Branch Block

We update eBooks quarterly and Apps daily based on user feedback. Please tap flag to report any questions that need improvement.

Question 372: A 70-year-old female was found unresponsive on the floor in her apartment. During a phone conversation two days ago with her daughter, her voice was loud, and she had reported myalgias. According to her daughter, she is allergic to penicillin, has recurrent methicillin-resistant Staphylococcus aureus skin infections, and consumes alcohol regularly. Two weeks ago, she returned from a cruise trip to the Bahamas. Emergency services noted vomitus soiling the floor adjacent to her, and she was intubated for airway protection. In the emergency department, her temperature was 39 C, heart rate 105 beats/min, blood pressure 90/50 mmHg, oxygen saturation 96% on FiO2 70%, pressure support mode ventilation with PEEP 10, pressure support 14 with spontaneous rate 28/min and minute ventilation 10 L/min. Blood alcohol level was 155 mg/dL, white cell count 24,000/microliter, serum creatinine 2 mg/dL, and serum sodium 130 mg/dL. The liver functions were normal. A chest x-ray confirmed correct placement of endotracheal tube and a dense right lower lobe infiltrate. CT of the brain was normal, and cerebrospinal fluid analysis demonstrated no white cells and normal glucose and protein. Blood cultures were sent, intravenous vancomycin 1000 mg and levofloxacin 500 mg were given in the emergency department. The patient was transferred to the medical intensive care unit. Which of the following changes to her antibiotic regimen would be most appropriate?

Choices: 1. Change vancomycin and levofloxacin to linezolid and aztreonam 2. Add ampicillin-sulbactam, increase levofloxacin to 750 mg, continue vancomycin 3. Continue vancomycin, increase levofloxacin to 750 mg daily, and add ertapenem or meropenem 4. Stop vancomycin and levofloxacin and treat with intravenous clindamycin only

Answer: 3 - Continue vancomycin, increase levofloxacin to 750 mg daily, and add ertapenem or meropenem

Explanations: Her risk factors comprising of alcohol use disorder and circumstantial suspicion for aspiration, necessitate a regimen targeting gram-negative, anaerobic, and oral gram-positive pathogens. Although the patient has an allergy to penicillin, the risk of cross-allergenicity with carbapenem is only 5%. Aztreonam has no activity against gram-positive or anaerobic pathogens. Levofloxacin has minimal activity against anaerobes, but the recent cruise trip to the Bahamas placed her at risk for Legionella that is usually susceptible to fluoroquinolones. Severe sepsis and a history of recurrent methicillin-resistant Staphylococcus aureus skin infections, require the use of vancomycin. Go to the next page if you knew the correct answer, or click the link image(s) below to further research the concepts in this question (if desired).

Research Concepts: Aspiration Pneumonia

We update eBooks quarterly and Apps daily based on user feedback. Please tap flag to report any questions that need improvement.

Question 373: A 26-year-old man presented with a 1-week history of shortness of breath and cough. He used over-the-counter cold medication and felt better. But for 2 days, he has been getting a high fever, productive cough, and chills at night. In the emergency department, his vitals were blood pressure 110/56 mmHg, pulse 130 bpm, temperature 39.6 C (103.2 F). Chest xray showed left pleural effusion. He was started on intravenous fluids and antibiotics for community-acquired pneumonia given after drawing blood cultures. Pulmonology service was consulted, and a bedside pleural ultrasound showed a left moderate pleural effusion with septations. Diagnostic thoracentesis findings were pH 7.0, lactate 649 mg/dL, and glucose of 2 mg/dL. What is the next step in management?

Choices: 1. Continue with intravenous antibiotics 2. Indwelling intrapleural catheter placement 3. Consult cardiothoracic surgery for video-assisted thoracoscopic surgery 4. Small-bore chest tube placement with intrapleural tPA and DNAase

Answer: 4 - Small-bore chest tube placement with intrapleural tPA and DNAase Explanations: This patient has community-acquired pneumonia, probably post-influenza. Staphylococcus aureus is the likely causative organism in these patients. S. aureus is notorious for causing empyema and abscesses. Thoracentesis confirmed a parapneumonic effusion. High lactate, low glucose, and pH of less than 7.2 is an indication for drainage of the parapneumonic effusion. The current recommendation is to place a small-bore chest tube followed by 3 days of intrapleural tPA and DNAase to lyse all the septations and adhesions. An indwelling intrapleural catheter may be indicated for chronic infective pleural effusions in highly selected cases. They have no role in the acute management of empyemas or parapneumonic effusions. Early video-assisted thoracoscopic surgery (VATS) has been shown to improve outcomes in a few small studies and the moment recommended by CT surgery guidelines, but there is no consensus among the societies. However, VATS should be done within 7 to 8 days of chest tube placement. Doing VATS later than 8 days increases the risk of a trapped lung. Go to the next page if you knew the correct answer, or click the link image(s) below to further research the concepts in this question (if desired).

Research Concepts: Intrapleural Catheter

We update eBooks quarterly and Apps daily based on user feedback. Please tap flag to report any questions that need improvement.

Question 374: A 40-year-old female presents to an emergency department (ED) for evaluation of recurrent abdominal pain, nausea, vomiting, diarrhea, and progressive bilateral lower extremity weakness. She has been hospitalized with similar complaints at least ten times in the last one and half years without a specific diagnosis. During her last admission, she was treated for acute gastroenteritis with empiric antibiotics. Her evaluation was negative, including an abdominal CT and upper and lower endoscopy. She also was suspected to have a psychiatric illness that might be contributing to her symptoms. Today, she had a witnessed seizure in the ED that lasted for 2 minutes. Her blood chemistry is notable for severe hyponatremia with a sodium of 120 mEq/L and mildly elevated serum transaminases. She is tachycardic with a heart rate of 125 beats/min, and her blood pressure is 165/95 mmHg. The physical exam is notable for generalized, mild abdominal tenderness without any guarding, rigidity, or rebound tenderness. The neurologic exam is significant for disorientation and 2/5 strength and decreased sensation in her lower extremities. Her CT head is negative for acute injury. An EEG has been ordered, and neurology has been consulted. She is admitted to the intensive care unit for further management. Besides correcting electrolytes abnormalities and management of acute problems, what else should be considered?

Choices: 1. MRI brain to evaluate for the cause of her seizure 2. Urine test for porphobilinogen (PBG) 3. Consult psychiatry for evaluation of a psychiatric cause 4. Repeat EGD (esophagogastroduodenoscopy) and colonoscopy

Answer: 2 - Urine test for porphobilinogen (PBG) Explanations: This patient presented with a "classic triad" of abdominal pain, central nervous system abnormalities, and peripheral neuropathy that should suggest acute porphyria. Therefore, screening for acute porphyrias by measuring urine porphobilinogen should be considered. Treatment for acute porphyria requires confirmation that the patient indeed has acute porphyria, based on the finding of elevated urinary porphobilinogen (PBG), either at present or previously. Knowing the exact type of acute porphyria is not required for treatment. IV hemin should be administered immediately once the diagnosis of hepatic porphyria is established. This patient had a negative head CT, and EEG results and neurology evaluation are pending. MRI brain is unnecessary at this stage of evaluation. It can be considered later if necessary. This patient had a normal EGD (esophagogastroduodenoscopy) and colonoscopy during her last hospital admission. Repeat evaluation will not be helpful. In addition, she is not stable enough for a procedure due to her severe hyponatremia. Go to the next page if you knew the correct answer, or click the link image(s) below to further research the concepts in this question (if desired).

Research Concepts: Acute Hepatic Porphyria

We update eBooks quarterly and Apps daily based on user feedback. Please tap flag to report any questions that need improvement.

Question 375: A 54-year-old man presents to the emergency department with dark stools. He is on warfarin for atrial fibrillation. He feels weak, lethargic and is unable to complete sentences. His vitals show a heart rate of 120 bpm, temp of 37.3 C, respiratory rate of 22 per minute, blood pressure of 88/49 mm Hg. CBC shows a WBC count of 6,000 cells/microliter, hemoglobin of 8.5 g/dL with a hematocrit of 28%, and a platelet count of 400,000/microliter, and his INR is 5.4. Which of the following methods would the best way to control the coagulopathy?

Choices: 1. Platelet transfusion 2. Prothrombin complex concentrate 3. Plasma transfusion 4. Observation

Answer: 2 - Prothrombin complex concentrate Explanations: Due to life-threatening bleeding and a supratherapeutic INR. The patient should receive both intravenous vitamin K and prothrombin complex concentrate. If PCC is unavailable, plasma transfusion can be used. While plasma which contains factors II, VII, IX, X in addition to other coagulation factors is useful, PCC can correct bleeds quicker. Vitamin K administration is necessary to permanently reverse the INR as PCC is a transfusion of clotting factors that is used for short-term correction. This patient would also benefit from a blood transfusion due to the brisk bleed and unstable vital signs. Go to the next page if you knew the correct answer, or click the link image(s) below to further research the concepts in this question (if desired).

Research Concepts: Anticoagulation Safety

We update eBooks quarterly and Apps daily based on user feedback. Please tap flag to report any questions that need improvement.

Question 376: A 65-year-old man arrives at the emergency department with severe, watery diarrhea and altered mental status. His Glasgow coma scale score is 12, and vital signs are a heart rate of 110 beats/min, respiratory rate 24/min, and blood pressure 70/50 mmHg. His past medical history is significant for hypertensive heart disease and diabetes mellitus type 2. His stool tests are positive for Cyclospora cayetanensis. What is the most appropriate treatment in such a case and why?

Choices: 1. Crystalloids, as they are readily available, cheap and are no less in mortality benefit than albumin 2. Albumin 25% to avoid fluid overload because of his medical history of hypertensive heart disease 3. Albumin 25% as he may be septic, and based on his blood pressure, he is in shock 4. Albumin 25% to correct his dehydration status

Answer: 1 - Crystalloids, as they are readily available, cheap and are no less in mortality benefit than albumin

Explanations: Crystalloids are considered the first-line treatment because albumin colloids have not proved a better outcome in the 90-day survival rate in critically ill patients compared to crystalloids. Albumin 5% would be an option to avoid fluid overload because of his past medical history of hypertensive heart disease. The recommended dosage is 500 mL of albumin, 5% repeated every 30 minutes as needed. Although albumin colloids have shown a slight difference in the outcome of more free days of ventilation and vasopressor therapy than the crystalloid group, crystalloids are less expensive than albumin colloids. Albumin colloid solutions are more efficient than crystalloids as the amount of fluid that remains in the intravascular space is more because of the oncotic effect. Therefore, less fluid is required when using colloids versus crystalloids to achieve reanimation. However, in clinical practice, there is no strong evidence to choose one over the other, in terms of mortality outcomes. Go to the next page if you knew the correct answer, or click the link image(s) below to further research the concepts in this question (if desired).

Research Concepts: Albumin Colloid

We update eBooks quarterly and Apps daily based on user feedback. Please tap flag to report any questions that need improvement.

Question 377: A 50-year-old female presents to the emergency department after a motor vehicle collision in which she was the unrestrained driver. Her vital signs include a heart rate of 110 bpm, stable blood pressure, and a respiratory rate in the high 20s. She looks uncomfortable and has a deformity of the medial clavicle, prominent sternum and no palpable SC joint on the left. What management is indicated?

Choices: 1. Stat chest x-ray followed by a stat CT angiogram of the chest 2. Chest radiographs and if normal, no further workup 3. Thoracotomy to evaluate pathology directly 4. Treat her pain, place her in a sling and refer to outpatient orthopedic follow up

Answer: 1 - Stat chest x-ray followed by a stat CT angiogram of the chest Explanations: This patient’s history and physical exam are concerning for a posterior sternal dislocation. This is a potentially life-threatening injury associated with high morbidity and mortality. Potentially injuries include esophageal or tracheal compression, pneumothorax, hemothorax, tracheoesophageal fistula, ipsilateral acromioclavicular dislocation or clavicle fracture, compression of subclavian artery or vein, compression of the innominate artery, late subclavian vein obstruction, laceration of the innominate vein, or brachial plexus compression. Symptoms suggesting mediastinal injuries include dyspnea, stridor, dysphagia, or paresthesia. The clinician may observe tachypnea or hypoxia, respiratory distress, difficulty controlling secretions, discoloration, swelling or decreased pulses of the affected limb. Management includes protecting the patient’s airway if necessary and emergent surgical consultation with an orthopedic surgeon. Open reduction and internal fixation in the operating room with vascular or cardiothoracic surgery as backup is indicated. Go to the next page if you knew the correct answer, or click the link image(s) below to further research the concepts in this question (if desired).

Research Concepts: Sternoclavicular Joint Injury

We update eBooks quarterly and Apps daily based on user feedback. Please tap flag to report any questions that need improvement.

Question 378: A 38-year-old healthy male, a software engineer with no known medical history, admitted to the step-down unit with supraventricular tachycardia. The patient also has continuous muscle twitching, agitation, fever, excessive sweating, and hallucinations. He had not slept in 7 days. On exam sensory loss and areflexia in the lower extremities was noted. There was no history of travel or exposure to sick contacts. Brain imaging including CT/MRI was unremarkable. Cerebrospinal fluid analysis also within reasonable limits. EEG showed findings consistent with encephalopathy. Serum antibody titers against CASPR2 and LGI1 proteins were significantly elevated. What is the next best step in the management of this patient?

Choices: 1. Anticonvulsant therapy 2. Intravenous corticosteroids 3. Evaluate for thymoma 4. Immunosuppression with intravenous immunoglobulin (IVIG) or plasma exchange

Answer: 4 - Immunosuppression with intravenous immunoglobulin (IVIG) or plasma exchange

Explanations: Elevated antibody titers against CASPR2 and LGI1 proteins are diagnostic of Morvan syndrome. Seizures are not a feature of Morvan syndrome and anticonvulsant therapy such as carbamazepine, phenytoin, phenobarbital would not be beneficial. Corticosteroids both oral and intravenously have been tried with variable but not very significant clinical responses. Morvan syndrome has a strong association with thymoma and could help in establishing a diagnosis. However, CASPR2 and LGI1 antibodies are even more diagnostic. Investigations to find a thymoma would not change the course of management at this time. Morvan syndrome is an autoimmune disease caused by autoantibodies to voltage-gated potassium channel complex proteins such as CASPR2 and LGI1. Immunosuppression with IVIG or plasma exchange has been shown to decrease antibody titers and significantly improve clinical outcomes. Go to the next page if you knew the correct answer, or click the link image(s) below to further research the concepts in this question (if desired).

Research Concepts: Morvan Syndrome

We update eBooks quarterly and Apps daily based on user feedback. Please tap flag to report any questions that need improvement.

Question 379: A 50-year-old Asian female with history of alcohol use disorder presents to the emergency department with severe abdominal pain and vomiting that began 3 hours ago. While working the patient up for pancreatitis, a CT scan of the abdomen is ordered. The image showed a high-density pancreatic tail fluid collection consistent with a pseudoaneurysm of a high flow vessel. The most likely structure is which of the following?

Choices: 1. Branch of the celiac trunk 2. Splenic artery 3. Short gastric artery 4. Left gastric artery

Answer: 2 - Splenic artery Explanations: Splenic artery pseudoaneurysm is a potential complication of pancreatitis as the release of pancreatic enzymes is capable of damaging the splenic artery which travels just superior to the pancreas. The fact that the CT finding showed high-density fluid should raise concern for the presence of blood within a pseudoaneurysm as opposed to a pancreatic pseudocyst which would more commonly have homogenous low attenuation on CT scan. Treatment options for a pseudoaneurysm include embolization of splenic artery, ligating the splenic artery, and splenectomy with or without distal pancreatectomy. The definitive test to confirm the presence of splenic artery pseudoaneurysm is CT angiogram. Go to the next page if you knew the correct answer, or click the link image(s) below to further research the concepts in this question (if desired).

Research Concepts: Pancreatic Pseudoaneurysm

We update eBooks quarterly and Apps daily based on user feedback. Please tap flag to report any questions that need improvement.

Question 380: A 42-year-old male in the northeast United States presents to the emergency department with confusion and leg weakness. On exam, he is noted to have a fever of 39 C, diminished strength in bilateral lower extremities, and altered mental status. A head CT is unremarkable. Cerebrospinal fluid is notable for pleocytosis (250 white blood cells/mm3), with no red blood cells, mildly elevated protein, and normal glucose. There are no organisms seen on Gram stain, and bacterial culture has no growth. Which of the following is the most likely vector of his illness?

Choices: 1. Mosquito bite 2. Fecal contamination of food 3. Deer tick bite 4. Respiratory secretions

Answer: 1 - Mosquito bite Explanations: West Nile Virus is spread by mosquitoes. West Nile Virus can cause viral meningitis. West Nile Virus should be considered in adults presenting with cerebrospinal fluid pleocytosis and lower motor neuron involvement. A travel history should be obtained in patients with viral meningitis as many viruses have specific geographical distributions. Go to the next page if you knew the correct answer, or click the link image(s) below to further research the concepts in this question (if desired).

Research Concepts: Viral Meningitis

We update eBooks quarterly and Apps daily based on user feedback. Please tap flag to report any questions that need improvement.

Question 381: The Grey Turner sign classically refers to ecchymosis or discoloration of the flanks related to retroperitoneal bleeding from acute necrotizing pancreatitis though other conditions may cause flank ecchymosis. Which of the following best describes the test characteristics of the Grey Turner sign for detecting acute necrotizing pancreatitis?

Choices: 1. The Grey Turner sign has very low sensitivity and negative predictive value for the detection of acute pancreatitis 2. The Grey Turner sign has high sensitivity but low specificity in the detection of acute pancreatitis 3. The Grey Turner sign has both high sensitivity and specificity for detecting acute pancreatitis 4. The Grey Turner sign has very low sensitivity but high negative predictive value for the detection of acute pancreatitis

Answer: 1 - The Grey Turner sign has very low sensitivity and negative predictive value for the detection of acute pancreatitis

Explanations: The Grey Turner sign is an uncommon finding among patients with acute severe necrotizing pancreatitis found in less than 5% of patients with known severe pancreatitis. Thus, both the sensitivity and negative predictive value of the sign for detecting pancreatitis is low. Discoloration of the flanks due to ecchymosis may occur from direct trauma to the flanks and not from intraabdominal or retroperitoneal blood dissecting through subcutaneous tissue planes. Clinicians must determine by history and physical if direct trauma occurred and if that explains the flank discoloration more consistently than other processes. Thus, flank discoloration is not specific to acute necrotizing pancreatitis. Authors have detailed multiple intraabdominal processes resulting in Grey Turner sign including ruptured ectopic pregnancy, spontaneous splenic rupture, ruptured abdominal wall varices, perforated duodenal ulcer, hemorrhagic transformation of intraabdominal cancers, ruptured abdominal aortic aneurysms, amoebic liver abscesses, and several others. Thus, the specificity of a Grey Turner sign for necrotizing pancreatitis is low and may occur for many reasons. Although the test characteristics of the Grey Turner sign show low sensitivity and specificity for acute necrotizing pancreatitis the presence of flank ecchymosis not due to direct trauma of the flank should lead the clinician to consider several serious and potentially life-threatening diagnoses. Clinicians often require rapid bedside ultrasound or CT to exclude these dangerous causes. Go to the next page if you knew the correct answer, or click the link image(s) below to further research the concepts in this question (if desired).

Research Concepts: Grey-Turner Sign

We update eBooks quarterly and Apps daily based on user feedback. Please tap flag to report any questions that need improvement.

Question 382: A 35-year-old Asian American male presents to the emergency department with a complaint of multiple episodes of syncope over the past week. Initially, he feels his heart racing and then he loses consciousness. His ECG shows an incomplete right bundle branch block, and ST-segment elevations only lead V2, along with an inverted T wave. J wave amplitude is 2 mm, ST-T configuration is cove-type, and the terminal portion of the ST segment is gradually descending. Transthoracic echocardiography is normal. He is unsure of his family history as he was adopted. Which of the following is the best therapy for this patient?

Choices: 1. Administer amiodarone at a dose of 1000 mg IV over 24 hours 2. Ablation therapy 3. Placement of an automatic implantable cardioverter-defibrillator 4. Infuse flecainide 2 mg/kg with a maximum of150 mg over 10 minutes

Answer: 3 - Placement of an automatic implantable cardioverter-defibrillator Explanations: Brugada syndrome is a genetic disorder that is associated with ventricular tachycardia and can lead to syncope, cardiac arrest, or sudden cardiac death. This patient has a type 1 pattern on EKG. Challenge with sodium channel blockers in type 1 pattern Brugada syndrome is not necessary as it adds no additional information. Radiofrequency ablation is a new mode of therapy that has promising results for some Brugada patients. However, the best-proven current treatment is the insertion of an automatic implantable cardioverter-defibrillator (AICD). To date, no drugs have been shown to decrease mortality and reduce the occurrence of ventricular arrhythmias or sudden death. There are conflicting results about using quinidine instead of AICD as a treatment for Brugada syndrome, but it has proven to be useful in patients with an AICD who experience multiple shocks or those who have a contraindication for ICD placement. In asymptomatic patients with no family history of sudden cardiac death, a close and frequent follow-up with conservative management can be considered. However, a personalized, interprofessional approach is suggested for those patients depending on each patient's risk factors and co-morbidities, if the decision is not to undergo AICD placement. Go to the next page if you knew the correct answer, or click the link image(s) below to further research the concepts in this question (if desired).

Research Concepts: Brugada Syndrome

We update eBooks quarterly and Apps daily based on user feedback. Please tap flag to report any questions that need improvement.

Question 383: A 46-year-old man with a history of chronic lower back pain and hypertension presents to the office for evaluation of worsening in his chronic back pain for the last six days. He denies any trauma, fever, chills, urinary incontinence or worsening of his chronic left lower extremity numbness. He has had some more difficulty ambulating due to back pain and has started using his cane again in the last two days. His blood pressure is 164/95 mmHg, pulse rate 108 bpm, respiratory rate 16/minute, temperature 98.4 F. Examination reveals point tenderness to the L3-L4 spinal region. Osteomyelitis is suspected. What imaging study has the highest sensitivity and specificity for the detection of osteomyelitis within one week of symptom onset?

Choices: 1. X-ray 2. CT scan 3. MRI 4. PET scan

Answer: 3 - MRI Explanations: MRI has the highest combined sensitivity and specificity for early detecting osteomyelitis at 78% to 90% and 60% to 90% respectively. MRI can detect bone infection within 3 to 5 days of the disease onset. MRI has a high negative predictive value, so a negative result is sufficient to exclude infection in a patient if symptoms have been present for more than one week. MRI has limited utility for detecting osteomyelitis in the presence of orthopedic hardware. Go to the next page if you knew the correct answer, or click the link image(s) below to further research the concepts in this question (if desired).

Research Concepts: Osteomyelitis

We update eBooks quarterly and Apps daily based on user feedback. Please tap flag to report any questions that need improvement.

Question 384: An 80-year-old male with multiple comorbidities presents to the emergency department from a nursing home with a fever. He has a chronic urinary catheter that is draining cloudy, foul-smelling urine. His nursing aide reports that he had a recent admission for vancomycin-resistant Enterococcus faecium. Prior cultures show that the organism has been resistant to ampicillin. Which of the following therapies would provide initial empiric coverage for the patient’s urinary tract infection?

Choices: 1. Linezolid 2. Ceftriaxone 3. Trimethoprim/sulfamethoxazole 4. Aztreonam

Answer: 1 - Linezolid Explanations: Linezolid can be given through intravenous or oral routes, as it has 100% bioavailability. Linezolid is bacteriostatic and acts through the inhibition of the pre-translational initiation complex formation. This, in turn, stops production of proteins needed for bacterial reproduction. While a patient is taking linezolid, the patient should be monitored for thrombocytopenia, anemia, and peripheral neuropathy. Linezolid should be used with caution in patients taking other serotonergic drugs, as it has been associated with serotonin syndrome. Go to the next page if you knew the correct answer, or click the link image(s) below to further research the concepts in this question (if desired).

Research Concepts: Vancomycin-Resistant Enterococci

We update eBooks quarterly and Apps daily based on user feedback. Please tap flag to report any questions that need improvement.

Question 385: A 67-year-old female with a history of hypertension, migraines, and nephrolithiasis is admitted from the emergency department for 4 days of flank pain, dysuria, and fever. She has no known drug allergies and takes 240 mg of propranolol daily for which she endorsed compliance at admission. A CT scan showed a mildly dilated right renal pelvis and proximal ureter with a 7 mm obstructing kidney stone. Her laboratory studies are remarkable for a serum WBC count of 14,000/microliter, urine dipstick positive for leukocyte esterase and WBCs, a creatinine of 1.3 mg/dL, and glomerular filtration rate 60. Her vital signs at the time of admission were heart rate 44 beats/min, blood pressure 116/72 mmHg, temperature 101.2F, and respiratory rate 18. She was started on broad-spectrum antibiotics after cultures were obtained and scheduled for a ureteroscopy the following morning. She was given multiple fluid boluses for hypotension overnight, totaling 3 liters. The following morning she is more somnolent with a heart rate of 47 beats/min, blood pressure 88/54 mmHg, temperature 103.4F, and respiratory rate 22. Which of the following inotrope is most appropriate for this patient?

Choices: 1. Norepinephrine 2. Phenylephrine 3. Epinephrine 4. Dobutamine

Answer: 1 - Norepinephrine Explanations: The first step in treating septic shock is adequate fluid resuscitation (30ml/kg) within the first 3 hours, sending cultures to the lab for evaluation, and early broad-spectrum antibiotics. If fluid resuscitation is unable to keep the mean arterial pressure greater than 65mmHg, vasopressor or ionotropic support should be initiated. Per the Surviving Sepsis Campaign (SSC), the vasopressor of choice after adequate fluid resuscitation is norepinephrine. The patient is not mounting a tachycardic response because she is on a high dose of propranolol. Propranolol has a half-life of approximately 12 hours, and the patient is still bradycardic. Phenylephrine would not be an appropriate choice because it could potentially worsen her bradycardia. If the patient is severely tachycardic or experiences a tachyarrhythmia in response to norepinephrine, phenylephrine would be an appropriate choice after volume resuscitation. Go to the next page if you knew the correct answer, or click the link image(s) below to further research the concepts in this question (if desired).

Research Concepts: Phenylephrine

We update eBooks quarterly and Apps daily based on user feedback. Please tap flag to report any questions that need improvement.

Question 386: A 17-year-old male presents to the emergency department with generalized weakness, nausea, and epigastric pain. On exam, he appears dehydrated and has no focal abdominal tenderness. Vital signs are stable. Basic metabolic panel reveals bicarbonate of 10 mmol/L and a glucose of 68 mg/dL, with an anion gap of 24. Venous blood gas reveals a pH of 7.0. Urine is positive for ketones. Serum ethanol concentration is 80 mg/dL. The patient was given two ampules of dextrose and two liters of normal saline. A repeat basic metabolic panel then showed bicarbonate of 14 mmol/L and an anion gap of 15. What is the most likely diagnosis?

Choices: 1. Methanol toxicity 2. Ethylene glycol toxicity 3. Isopropyl toxicity 4. Alcoholic ketoacidosis

Answer: 4 - Alcoholic ketoacidosis Explanations: Methanol and ethylene glycol toxicity both cause an anion gap acidosis. They do not generally improve with resuscitation, especially if the parent compound is continuously metabolizing. Isopropyl alcohol toxicity causes ketosis without acidosis. Symptomatically, a patient should be intoxicated after ingesting isopropyl, with possible hemorrhagic gastritis. Alcoholic ketoacidosis is not treated with fomepizole. It can prolong the half-life of ethanol. Treatment of alcoholic ketoacidosis requires replacement of dextrose and resuscitation. Patients respond much better to resuscitation than when intoxicated with methanol or ethylene glycol. Oftentimes, when serum ethanol is elevated, it is unlikely that the patient would be acidotic from a toxic alcohol unless they ingested the toxic alcohol long before the ethanol. Go to the next page if you knew the correct answer, or click the link image(s) below to further research the concepts in this question (if desired).

Research Concepts: Methanol Toxicity

We update eBooks quarterly and Apps daily based on user feedback. Please tap flag to report any questions that need improvement.

Question 387: Following a left lung lobectomy, a 66-year-old patient develops hypotension, tachycardia, and hypoxia. Fleischner sign is noted on chest x-ray. A loud P2 is heard on auscultation. The EKG shows variable ST-segment changes in the anterior leads. Which of the following is the diagnostic test of choice for this patient's suspected condition?

Choices: 1. Cardiac enzymes 2. Time of flight MRI 3. Contrast-enhanced CT chest 4. Bronchoscopy

Answer: 3 - Contrast-enhanced CT chest Explanations: Pulmonary embolism should be the first consideration in a postoperative patient who suddenly decompensates. A contrast-enhanced CT chest is the diagnostic test of choice for evaluating pulmonary artery emboli. CT typically demonstrates filling defects in the pulmonary arteries, including the main pulmonary arterial trunks or the segmental and subsegmental arteries. Radiographs often demonstrate nonspecific findings. Fleischner sign on a chest radiograph can be seen, which corresponds to an enlarged pulmonary artery. A Hampton hump, which corresponds to a peripheral lung infarction, can also be seen as a peripheral wedge-shaped opacity. Westermark sign corresponds to regional decreased blood flow with difficult to visualize peripheral pulmonary vasculature (oligemia). Go to the next page if you knew the correct answer, or click the link image(s) below to further research the concepts in this question (if desired).

Research Concepts: Acute Pulmonary Embolism

We update eBooks quarterly and Apps daily based on user feedback. Please tap flag to report any questions that need improvement.

Question 388: A 47-year-old male presents to the emergency department complaining of severe epigastric pain radiating to the back and associated nausea and vomiting. He has a prolonged history of alcohol use disorder and states that he often drinks 500 mL of vodka per day with many failed attempts at quitting. The patient appears to be in distress and using accessory muscles to breathe. His vitals show a temperature of 38.5 C, heart rate 115 beats/min, blood pressure 90/45 mmHg, and respiratory rate 42. Laboratory studies are significant for lipase of 5534 units/L and mildly elevated transaminases with aspartate aminotransferase greater than alanine aminotransferase. Hematocrit is 42%. A chest x-ray is obtained and shows bilateral, patchy opacities throughout the lung fields. Which of the following indicates a poor prognosis?

Choices: 1. Lipase greater than 950 units/L 2. PaO2 less than 60 mmHg 3. Hematocrit greater than 40% 4. Age greater than 45 years

Answer: 2 - PaO2 less than 60 mmHg Explanations: Poor prognostic indicators for acute pancreatitis are shock, hypoxemia, hemoconcentration with hematocrit greater than 44%, gastrointestinal bleeding, and renal failure. Ranson criteria include age greater than 55 years, WBC greater than 16,000/microliter, glucose greater than 200 mg/dL, lactate dehydrogenase greater than 350 units/L, and aspartate aminotransferase greater than 250 U/L. Ranson criteria measured at admission and at 48 hours correlate with risk of death. Ranson criteria can be used only to calculate mortality once at 48 hours after admission and cannot be used to determine response to therapy. Go to the next page if you knew the correct answer, or click the link image(s) below to further research the concepts in this question (if desired).

Research Concepts: Acute Pancreatitis

We update eBooks quarterly and Apps daily based on user feedback. Please tap flag to report any questions that need improvement.

Question 389: A 65-year-old man with a history of chronic hepatitis C-induced liver disease is brought to the emergency department for fatigue, worsening jaundice, and progressive abdominal distention. The patient was found on the floor at home by his daughter difficult to arouse in the afternoon. He responds to noxious stimuli but not to verbal command. On exam, flank fullness with shifting dullness is elicited. CT head is negative. Diagnostic paracentesis is performed, and blood is drawn for labs. The accuracy of serum ascites albumin gradient (SAAG) is most diminished in which of the following?

Choices: 1. Fever 2. Arterial hypotension 3. Anemia 4. Jaundice

Answer: 2 - Arterial hypotension Explanations: SAAG is based on oncotic-hydrostatic balance. Calculating SAAG entails simply subtracting the ascitic fluid value from the serum value. SAAG greater than 1.1 g/dL (11 g/L), is suggestive of the presence of portal hypertension and value less than 1.1 g/dL or 11 g/L may exclude it. SAAG does not provide any information about the pathogenesis of the ascites. In the presence of arterial hypotension, portal pressure is decreasing, causing narrowing of the SAAG gradient. Fever, jaundice, and anemia have not been shown to affect the SAAG value. Go to the next page if you knew the correct answer, or click the link image(s) below to further research the concepts in this question (if desired).

Research Concepts: Ascites

We update eBooks quarterly and Apps daily based on user feedback. Please tap flag to report any questions that need improvement.

Question 390: A 65-year-old male with history of stage IIIb squamous cell cancer of the lung presents with increasing dyspnea on exertion and orthopnea ongoing for four weeks with gradual worsening. On presentation, he is noted to be 90% on ambient air and is tachypneic to a respiratory rate of 30/minute. Labs are at baseline. A chest x-ray demonstrates large pleural effusion with contralateral mediastinal shift. He undergoes large-volume thoracentesis of 3 liters with immediate relief of dyspnea. The next morning, he develops respiratory distress with new hypoxia requiring supplemental oxygen. What is the most likely cause of the patient's newly developed respiratory distress?

Choices: 1. Pneumothorax ex vacuo 2. Hemothorax 3. Reexpansion pulmonary edema 4. Pulmonary embolism

Answer: 3 - Reexpansion pulmonary edema Explanations: Reexpansion pulmonary edema occurs after the removal of fluid or air from the pleural space quickly over a short period of time. The mechanism of edema is believed to be increased capillary permeability. Risk factors for this condition include young age, a long duration of lung collapse, and rapid reexpansion. Treatment is largely supportive. Thoracentesis operators should not perform large-volume thoracentesis. No more than 1.5 liters of fluid should be removed in any instance. Go to the next page if you knew the correct answer, or click the link image(s) below to further research the concepts in this question (if desired).

Research Concepts: Malignant Effusion

We update eBooks quarterly and Apps daily based on user feedback. Please tap flag to report any questions that need improvement.

Question 391: A 27-year-old female presents to the emergency department complaining of rhinorrhea and excessive lacrimation for the last 30 minutes. One hour ago, she was cleaning her windows with an ammonia-containing cleaner when the bottle slipped and fell on the floor. She attempted to clean the fluid away despite the strong irritating pungent smell. Which of the following findings on evaluation would best predict the development of chronic lung disease in this patient?

Choices: 1. Reduced oxygen saturation on presentation to the emergency department 2. Presence of chemical burns on the nose 3. Reduced peak expiratory flow rate on presentation to the emergency department 4. Injury to the bronchial lining, as evident on bronchoscopy

Answer: 4 - Injury to the bronchial lining, as evident on bronchoscopy Explanations: Prolonged inhalation of ammonia fumes or anhydrous ammonia gas can cause injury to the bronchial epithelium. Since ammonia is a light and water-soluble gas, most of the injury is limited to the upper respiratory tract if the exposure is of short duration. Prolonged exposure results in damage to the basal lining of lower bronchial and pulmonary epithelium and this can lead to the development of chronic lung disease. Reduced peak expiratory flow on presentation to the emergency department could be due to bronchoconstriction from the irritant action of the fumes. This is usually short-lived, and the patient experiences symptomatic relief on supportive management. Go to the next page if you knew the correct answer, or click the link image(s) below to further research the concepts in this question (if desired).

Research Concepts: Ammonia Toxicity

We update eBooks quarterly and Apps daily based on user feedback. Please tap flag to report any questions that need improvement.

Question 392: A 66-year-old female was admitted to the ICU after a motor vehicle accident causing a severe head injury. Brain death diagnosis was uncertain by physical examination, and the apnea test could not be performed due to CO2 retention. A transcranial Doppler (TCD) ultrasound on the middle cerebral arteries bilaterally did not detect any flow tracing. Which of the following is the next best step in the management of this patient?

Choices: 1. Declare brain death 2. Order somatosensory evoked potentials 3. Repeat TCD after one day 4. Continue medical management

Answer: 2 - Order somatosensory evoked potentials Explanations: TCD can confirm brain death if it shows absent diastolic pulsations or small peaked systolic pulsations. The absence of flow tracing is inconclusive of brain death, as it may be due to an unsuitable window. Somatosensory evoked potentials may be ordered to determine brain death as TCD is inconclusive for this patient. The absence of flow tracing by TCD does not confirm brain death, as it may be due to an unsuitable window. Another ancillary test should be performed. Normal PCO2 is a prerequisite to performing the apnea test. Go to the next page if you knew the correct answer, or click the link image(s) below to further research the concepts in this question (if desired).

Research Concepts: Brain Death Criteria

We update eBooks quarterly and Apps daily based on user feedback. Please tap flag to report any questions that need improvement.

Question 393: A patient presents with weight loss, anemia, and fatigue. Blood work reveals a white blood cell count of 88,000 cells/mm3 and a hemoglobin of 9 grams/dL. He has a mildly enlarged spleen and liver. Cytogenetic testing reveals a chromosomal translocation resulting in a BCR-ABL1 fusion gene. What is the most likely diagnosis?

Choices: 1. Acute myeloid leukemia (AML) 2. Acute lymphoblastic leukemia (ALL) 3. Chronic myeloid leukemia (CML) 4. Chronic lymphocytic leukemia (CLL)

Answer: 3 - Chronic myeloid leukemia (CML) Explanations: The translocation is t(9;22)(q34;22q11) and results in the Philadelphia chromosome. This results in the BCR (breakpoint cluster region) gene from chromosome 22 fusing with the ABL1 gene of chromosome 9. Abl is from "Abelson," a leukemia virus that has a similar protein. The Philadelphia chromosome is seen in 95% of patients with chronic myeloid leukemia (CML), which is detected by traditional karyotype. The remainder are positive for cryptic BCR-ABL1 fusion gene, which is detected by fluorescence in situ hybridization or reverse transcriptase-polymerase chain reaction. The Philadelphia chromosome also is found in B-cell acute lymphoblastic leukemia (BALL), acute myeloid leukemia (AML), and mixed phenotype acute leukemia (MPAL). Go to the next page if you knew the correct answer, or click the link image(s) below to further research the concepts in this question (if desired).

Research Concepts: Chronic Myelogenous Leukemia

We update eBooks quarterly and Apps daily based on user feedback. Please tap flag to report any questions that need improvement.

Question 394: A 65-year-old Asian male with known history of rheumatic mitral valve disease presents with a 1-week history of new-onset gnawing pain and swelling of both arms and wrist joints. He also has a high-grade fever for past four days with increased shortness of breath. According to his son, he is mildly confused since this morning. What would be the most obvious acute condition to suspect?

Choices: 1. Acute bacterial meningitis 2. Miliary tuberculosis 3. Acute infective endocarditis 4. Acute deep vein thrombosis

Answer: 3 - Acute infective endocarditis Explanations: Clinical findings of hypertrophic osteoarthropathy should trigger suspicion for a new complication on a damaged valve, likely a vegetation. Milliary tuberculosis does not present with acute confusion but with a generalized decline over months, fever, weight loss and intestinal symptoms. Acute bacterial meningitis causes meningeal signs in the axial, not appendicular skeleton. Bilaterally symmetrical upper extremity deep venous thrombosis is highly unlikely. Go to the next page if you knew the correct answer, or click the link image(s) below to further research the concepts in this question (if desired).

Research Concepts: Secondary Hypertrophic Osteoarthropathy

We update eBooks quarterly and Apps daily based on user feedback. Please tap flag to report any questions that need improvement.

Question 395: A 48-year-old left-handed white collar worker is transported to the emergency room with left sensory loss, aphasia, agitation, mild left-sided weakness, and a headache. The patient has no past medical history. The patient was recently hospitalized for transient speech problems, headache, and left-sided sensory loss. MRI/MRA of the head and neck were normal. The patient had been discharged as symptoms had resolved. Which of the following is most likely on lumbar puncture?

Choices: 1. Elevated opening pressure 2. Moderately elevated protein 3. Pleocytosis with lymphocyte predominance 4. Any of the above

Answer: 4 - Any of the above Explanations: The patient could have viral infection or headache with neurologic deficits and CSF lymphocytosis. If the patient's condition resolved HaNDL is most likely. This condition is characterized by 1 to 20 episodes of transient neurologic deficits with headache over 3 months. The syndrome of neurological deficit with CSF lymphocytosis may be associated with a viral infection. In some patients, the symptoms may resolve spontaneously but in others, the symptoms may persist for an indefinite period. Go to the next page if you knew the correct answer, or click the link image(s) below to further research the concepts in this question (if desired).

Research Concepts: Headache With Neurological Deficits and CSF Lymphocytosis

We update eBooks quarterly and Apps daily based on user feedback. Please tap flag to report any questions that need improvement.

Question 396: A 21-year-old male with a past medical history of severe asthma, anxiety, and obesity presents to the emergency department with acute onset dyspnea. A review of the medical records shows that he has been hospitalized four times in the last year and required intubation once when he was 18 years of age. On the initial exam, he was found to be tachypneic with a respiratory rate of 34, diaphoretic, and using accessory muscles for respiration. An arterial blood gas (ABG) done in the emergency department showed a pH of 7.35, PCO2 45, and PaO2 92 on room air. He was intubated and paralyzed due to difficulty syncing with the event. He was placed on AC mode ventilation with a tidal volume of 8cc/kg, respiratory rate 23, PEEP of 5, and FiO2 40%. A repeat ABG shows a pH of 7.19, PCO2 60, and PaO2 109. Apart from adjusting tidal volume and respiratory rate, what else can be done to improve his ventilation?

Choices: 1. Increase inspiration time 2. Increase FiO2 3. Increase PEEP 4. Square the flow curve (increase inspiratory flow)

Answer: 4 - Square the flow curve (increase inspiratory flow) Explanations: This is a young patient with asthma who has been difficult to ventilate requiring him to be paralyzed. His arterial blood gas (ABG) shows a respiratory acidosis even though he has a good tidal volume and a rapid respiratory rate on the ventilator. Increasing his respiratory rate could lead to auto-PEEP. One way to prevent this is by increasing the flow rate of air into the respiratory system allowing for faster delivery of the tidal volume and more time for exhaling. This is called squaring of the flow curve. Decreasing, not increasing, the inspiratory time (decrease in the I:E ratio) will allow for more time for exhalation, thereby helping to prevent auto-PEEP and allowing for an increased respiratory rate. Increasing FiO2 will not lead to improvement in the PCO2. The patient's oxygenation is already good without needing further improvement. Increasing PEEP will improve oxygenation but will not affect his CO2. Go to the next page if you knew the correct answer, or click the link image(s) below to further research the concepts in this question (if desired).

Research Concepts: Ventilation Assist Control

We update eBooks quarterly and Apps daily based on user feedback. Please tap flag to report any questions that need improvement.

Question 397: A 17-year-old female presents to the emergency department after ingesting an unknown amount of a food warming fuel. She has nausea, epigastric discomfort, and blurred vision. A physical exam reveals epigastric tenderness, sluggish pupillary response to light, and decreased visual acuity. Her basic metabolic panel demonstrates a bicarbonate of 4 mmol/L. What is the most appropriate treatment plan?

Choices: 1. Dialysis 2. Fomepizole, resuscitation, admit for dialysis 3. Fomepizole and resuscitation 4. Bicarbonate administration while awaiting confirmatory test results

Answer: 2 - Fomepizole, resuscitation, admit for dialysis Explanations: This patient ingested a food warming fuel, and food warming fuels are known to contain methanol. Symptoms of end-organ toxicity and acidosis are indications for dialysis in methanol toxicity. Dialysis has been shown to markedly decrease the length of stay. Fomepizole should be given immediately in this scenario so that the patient can respond to resuscitation. Normal saline should be administered. Confirmatory test results may take hours to days in some facilities and should not delay treatment. Bicarbonate infusion may be of some benefit. Bicarbonate alone will not be an effective treatment. Go to the next page if you knew the correct answer, or click the link image(s) below to further research the concepts in this question (if desired).

Research Concepts: Methanol Toxicity

We update eBooks quarterly and Apps daily based on user feedback. Please tap flag to report any questions that need improvement.

Question 398: A 72-year-old female who lives independently is brought in by emergency medical services after being found down at home by a friend who became suspicious when the patient had not arrived for their weekly book club. On initial assessment, the patient is tachycardic and hypotensive and is on supplemental oxygen via nasal cannula at 5 L. She is moaning and uncomfortable, non-verbal, but opens her eyes to command and moves all extremities. An exam reveals abdominal tenderness and ecchymosis around the umbilicus. Which of the following conditions is associated with periumbilical ecchymosis?

Choices: 1. Pancreatitis 2. Cholecystitis 3. Renal stone 4. Appendicitis

Answer: 1 - Pancreatitis Explanations: Cullen sign is a classic sign seen in some cases of hemorrhagic pancreatitis. The sign is characteristic of severe, acute pancreatitis, ectopic pregnancy, and hemoperitoneum. Cullen sign appears as a bluish to purplish discoloration around the umbilicus and can occur in conjunction with Turner sign. Cullen sign is most often present in severe pancreatitis where inflammation has caused the erosion of blood vessels leading to hemoperitoneum. Go to the next page if you knew the correct answer, or click the link image(s) below to further research the concepts in this question (if desired).

Research Concepts: Acute Pancreatitis

We update eBooks quarterly and Apps daily based on user feedback. Please tap flag to report any questions that need improvement.

Question 399: A 66-year-old female is admitted to the hospital for severe iron deficiency secondary to chronic kidney disease. After receiving her first two infusions of parenteral iron therapy, she begins to complain of muscle, and joint pain and her blood pressure drops to 92/65 mmHg. In addition to intravenous fluids, a drug with which of the following mechanism of action is most appropriate for this patient?

Choices: 1. Chelation 2. Neutralization 3. Oxidation-reduction 4. Isomerization

Answer: 1 - Chelation Explanations: Deferoxamine is a chelating agent that works by binding to iron in the bloodstream, creating a stable complex that is excreted renally. While the body has innate processes to actively absorb iron, it does not have a way of actively excreting iron. Only a certain amount of iron can be actively absorbed at any given time; excess is taken up passively via the paracellular route. Chelation is also a useful therapy for treating other types of heavy metal toxicity including copper in patients with Wilson Disease. Go to the next page if you knew the correct answer, or click the link image(s) below to further research the concepts in this question (if desired).

Research Concepts: Iron

We update eBooks quarterly and Apps daily based on user feedback. Please tap flag to report any questions that need improvement.

Question 400: A 70-year-old male presents to the hospital with an altered mental status. Physical examination is remarkable for bradycardia, anasarca, and hypothermia. His family reveals that he had thyroid cancer and a thyroidectomy. He has not been seen by a healthcare provider in 5 years. Which of the following is the most accurate regarding this condition?

Choices: 1. Increased respiratory rate and decreased blood pressure 2. Decreased respiratory rate and increased blood pressure 3. Decreased serum potassium level and decreased serum sodium level 4. Decreased serum sodium level and decreased blood pressure

Answer: 4 - Decreased serum sodium level and decreased blood pressure Explanations: The diagnosis is myxedema crisis, which is a medical emergency. The patients usually present with hyponatremia, hypotension, bradycardia. Myxedema coma occurs in patients with long-standing hypothyroidism and usually is precipitated by drugs, infection, trauma, cerebrovascular disease, or heart failure. Hypotension can indicate systemic infection, hemorrhage, and further cardiovascular deterioration. Prompt initiation of fluid resuscitation with broad-spectrum antibiotics , hydrocortisone, and levothyroxine is of paramount importance Additional complications from myxedema crisis include respiratory failure, coma, and hyponatremia. Mechanical ventilation is required if there is evidence of impending respiratory failure. Go to the next page if you knew the correct answer, or click the link image(s) below to further research the concepts in this question (if desired).

Research Concepts: Myxedema

We update eBooks quarterly and Apps daily based on user feedback. Please tap flag to report any questions that need improvement.

Section 5 Question 401: A 32-year-old woman with a past medical history of acute myelocytic leukemia presents complaining of cough with a small amount of bright red blood. She had a oneweek history of cough productive of thick brown sputum, fever, and pleuritic chest pain. A chest x-ray done 5 days ago revealed right upper lobe infiltrate; she was immediately started on oral antibiotics by her primary care provider. However, the symptoms persist. The patient underwent allogeneic stem cell transplantation for his acute myelocytic leukemia 5 weeks ago, which was complicated by acute graft-versus-host disease and neutropenia. Temperature is 39.5 C (103. 1 F), blood pressure is 100/62 mmHg, the pulse is 110/min, and respiratory rate is 20/min. The physical exam is significant for right-sided crackles. Laboratory results show leukocytes 1500/microL, hematocrit 28%, and platelets 138,000/microL. Chest x-ray shows a right upper lobe infiltrate, increased in size when compared to the previous x-ray. Chest CT-scan reveals several nodular lesions with surrounding ground-glass opacities in the right upper lobe. Sputum gram stain shows no organisms. What is the best initial therapy for this patient?

Choices: 1. Voriconazole 2. Voriconazole + caspofungin 3. Caspofungin 4. Embolization

Answer: 2 - Voriconazole + caspofungin Explanations: Caspofungin is effective against invasive aspergillosis in patients who are refractory to or not tolerating voriconazole. The main population at risk of developing invasive aspergillosis is immunocompromised patients. These patients will typically present with a triad of fever, chest pain, and hemoptysis. Chest CT reveals pulmonary nodules with the "halo" sign. Treatment consists of voriconazole with or without caspofungin. Caspofungin is not recommended as first-line therapy for invasive aspergillosis. Patients on caspofungin should be monitored for hepatotoxicity, as it has been associated with increased liver enzymes. Go to the next page if you knew the correct answer, or click the link image(s) below to further research the concepts in this question (if desired).

Research Concepts: Caspofungin

We update eBooks quarterly and Apps daily based on user feedback. Please tap flag to report any questions that need improvement.

Question 402: A 30-year-old female presents to the hospital for severe nausea and vomiting along with a two-day history of fever and cough. She has type 1 diabetes mellitus and has been using insulin since diagnosis 14 years ago. Her regimen is glargine insulin 25 U SQ BID and also uses an insulin pump. On examination, the patient has dry mucous membranes and appears somewhat lethargic. She has a bout of severe abdominal pain and that is relieved by some morphine. Two intravenous lines with normal saline and insulin are started. Labs are drawn showing blood glucose of 500 mg/dL. The laboratories show sodium 131 mEq/dL, potassium 3.1 mEq/dL, chloride 90 mEq/dL, bicarbonate 13 mEq/dL, BUN 28 mg/dl, and creatinine 0.88 mg/dL. Arterial blood gas shows pH 6.98, PO2 92, and PCO2 28. What are the changes to the management needed?

Choices: 1. Continue normal saline and insulin, supplement potassium, and start broad-spectrum antibiotics 2. Replace normal saline with a bicarbonate drip, continue insulin, supplement potassium, and start broad-spectrum antibiotics 3. Continue normal saline, stop insulin, supplement potassium, and start broad-spectrum antibiotics 4. Replace normal saline with a bicarbonate drip, stop insulin, supplement potassium, and start broad-spectrum antibiotics

Answer: 3 - Continue normal saline, stop insulin, supplement potassium, and start broadspectrum antibiotics

Explanations: Once the potassium level falls below 3.3, it is recommended that the insulin is stopped and reinitiated once potassium is supplemented adequately. If the insulin is continued, the potassium levels fall further, and there is a risk of cardiac arrhythmias with severe hypokalemia. There is no indication of bicarbonate replacement above a pH of 6.9 and may be associated with increased harm including cerebral edema Broad-spectrum antibiotics are indicated in this patient with a possible infectious trigger for ketoacidosis Potassium supplementation should be done on low and normal potassium levels in anticipation of the transcellular shifts lowering the levels during treatment of diabetic ketoacidosis with insulin. Go to the next page if you knew the correct answer, or click the link image(s) below to further research the concepts in this question (if desired).

Research Concepts: Ketoacidosis

We update eBooks quarterly and Apps daily based on user feedback. Please tap flag to report any questions that need improvement.

Question 403: A 68-year-old male is in the intensive care unit after being in a severe motor vehicle accident. At 9 pm on the third day of his hospital stay, he pulls out his intravenous line and urine catheter and begins screaming that people are trying to hurt him. Later that evening, he is found to be disoriented and sedated in appearance. What is the most likely diagnosis?

Choices: 1. New-onset dementia 2. Post-traumatic stress disorder 3. Psychotic break 4. Delirium

Answer: 4 - Delirium Explanations: Delirium is very common in the hospital setting and includes a disturbance of consciousness and changes in cognition. A diagnosis of delirium requires knowledge of an individual's baseline cognitive function. By definition, delirium must be caused by an organic process. Treatment of delirium requires treating the underlying cause. Go to the next page if you knew the correct answer, or click the link image(s) below to further research the concepts in this question (if desired).

Research Concepts: Delirium

We update eBooks quarterly and Apps daily based on user feedback. Please tap flag to report any questions that need improvement.

Question 404: A 72-year-old male with a history of hypertension, hyperlipidemia, diabetes mellitus, and congestive heart failure is brought to the hospital by his son for weakness, perioral numbness, vomiting, and abdominal cramping for 2 days. He denies any travel history, but he ate fish at a restaurant a few days ago. On examination, his blood pressure is 120/90 mmHg, pulse 52 beats/min, respirations 19, temperature 98.9 F, and oxygen saturation 96% on room air. An EKG shows sinus bradycardia without any ST abnormalities. A chest x-ray does not show any active cardiopulmonary processes. A CT head does not show any acute intracranial normality. He is admitted to the hospital and started on conservative treatment with IV fluids, antiemetics, and bedrest. Blood and urine cultures have been negative so far. After 3 days of hospitalization, the patient continues to have the neurologic symptoms, but the bradycardia and gastrointestinal symptoms have resolved. What is the best next step in management?

Choices: 1. MRI of the brain 2. Levetiracetam twice daily 3. One dose of mannitol 4. Acetazolamide three times daily

Answer: 3 - One dose of mannitol Explanations: Neurologic symptoms of ciguatera fish poisoning can last for some time and can be debilitating. Neurologic symptoms of ciguatera fish poisoning include perioral numbness, paresthesias, painful urination, the sensation of loose of teeth, cold allodynia, and pruritus without any skin lesions. Conservative management with volume repletion, antiemetics, and bedrest is the mainstay of treatment. One dose of mannitol can be tried for patients with neurologic symptoms despite conservative management. The role of mannitol for long-term use is controversial, but one dose of mannitol can be tried without serious adverse effects. The patient has to be euvolemic before mannitol can be administered, as it leads to fluid loss. Go to the next page if you knew the correct answer, or click the link image(s) below to further research the concepts in this question (if desired).

Research Concepts: Ciguatera Toxicity

We update eBooks quarterly and Apps daily based on user feedback. Please tap flag to report any questions that need improvement.

Question 405: A 56-year-old male with a medical history of hypertension, acute lymphocytic leukemia, non-ischemic cardiomyopathy underwent an automatic implantable cardioverter-defibrillator (AICD) placement to prevent sudden cardiac death due to a low ejection fraction of 20%. Next day he developed significant chest pain, shortness of breath and diaphoresis. Blood pressure was low, heart rate was high, but the temperature was normal. WBC counts were normal, troponins were elevated, and EKG showed ST-segment elevations in anterior and lateral chest leads. Emergent bedside transthoracic echocardiogram showed left ventricular hypokinesis. Urgent CT angiogram of the chest showed some pulmonary vascular congestion, but otherwise, it was normal. The patient was taken for an emergent coronary angiogram which was nonobstructive. However, nonspecific dilated coronary sinus was noted on the angiogram. What is the most likely diagnosis?

Choices: 1. Coronary sinus thrombosis 2. Pulmonary embolism 3. Cardiac tamponade 4. Endocarditis

Answer: 1 - Coronary sinus thrombosis Explanations: Coronary sinus thrombosis is a rare but severe post-procedural complication following invasive right heart instrumentation. This patient underwent AICD placement which involves instrumentation of coronary sinus for ventricular lead placement which most likely caused endothelial injury and triggered thrombosis of the coronary sinus. This patient is at a higher risk for development of coronary sinus thrombosis also due to the preexisting history of leukemia. A high index of clinical suspicion and emergent treatment with thrombectomy followed by long-term anticoagulation is needed to prevent mortality. Pulmonary embolism can present with chest pain and shortness of breath. However, STsegment elevation on EKG is not common. Echocardiogram shows right ventricular strain. Ct angiogram of the chest is diagnostic of Pulmonary Embolism. This patient’s most likely diagnosis is coronary sinus thrombosis, mainly due to the finding of bulging coronary sinus and a negative CT angiogram of the chest. A thrombus in the coronary sinus, especially if chronic can get further complicated by rupturing and embolising to the pulmonary vasculature. However, CT angiogram of the chest would be positive at that time. Cardiac tamponade can result due to many cardiac complications like myocardial infarction including acute worsening coronary sinus thrombosis. However cardiac tamponade can be diagnosed on echocardiogram. There no pericardial effusion or tamponade noted in this patient on echocardiogram. Due to the absence of fever, normal WBC, normal echocardiogram, and finding of coronary sinus dilation on a cardiac angiogram, endocarditis is less likely in this patient. Go to the next page if you knew the correct answer, or click the link image(s) below to further research the concepts in this question (if desired).

Research Concepts: Coronary Sinus Thrombosis

We update eBooks quarterly and Apps daily based on user feedback. Please tap flag to report any questions that need improvement.

Question 406: A 62-year-old male with a history of hypertension, diabetes mellitus, chronic obstructive pulmonary disease (COPD) on 2L of home oxygen, congestive heart failure, and atrial fibrillation on warfarin presents to the emergency department with shortness of breath. His initial vitals are temperature 101.3 F, heart rate 121 bpm, blood pressure 150/90 mmHg, respiratory rate 24/minute, and pulse oximetry (SpO2) 80% on 10 L non-rebreather mask. His exam is significant for rales in the left lower lung field and decreased air movement diffusely as well as 1+ pitting edema to his lower extremities bilaterally. Chest x-ray shows hyperinflated lungs, and a focal infiltrate in the left lower lobe. EKG shows atrial fibrillation with a rapid ventricular response without signs of ischemia. He is intubated for hypoxemic respiratory failure secondary to pneumonia. However, his oxygenation does not improve with standard ventilator settings, and inverse ratio ventilation (IRV)is being considered. Which of the following patient factors is most likely to complicate this patient's management if he were to be placed on IRV?

Choices: 1. Respiratory failure secondary to pneumonia 2. History of COPD 3. History of heart failure 4. Hemodynamic status

Answer: 2 - History of COPD Explanations: This patient's history of severe COPD, indicated by his need for home oxygen, decreased air movement on exam, and hyper-inflated lungs, put him at increased risk for auto-positive end-expiratory pressure (PEEP) if placed on IRV. Typical management of mechanical ventilation for patients with obstructive lung disease often involves increasing the expiratory time to allow adequate exhalation time, not decreasing it as is seen in IRV. Decreased expiratory time will lead to incomplete exhalation in this patient with subsequently decreased ventilation and increasing intrathoracic pressures resulting in barotrauma. In severe cases, intrathoracic pressure may continue to build with subsequent breaths as each additional breath creates additional volume which is unable to be exhaled. This may result in hemodynamic compromise or pneumothorax. Go to the next page if you knew the correct answer, or click the link image(s) below to further research the concepts in this question (if desired).

Research Concepts: Inverse Ratio Ventilation

We update eBooks quarterly and Apps daily based on user feedback. Please tap flag to report any questions that need improvement.

Question 407: A 35-year-old male presents to the Emergency Department in respiratory distress. On exam, you note some venous distention on his neck and have difficulty with auscultating heart sounds. Vital signs at this time show hypotension at 62/30 mmHg, heart rate 53 bpm, respiratory rate 26 per minute, and SpO2 92%. While evaluating the patient, he suddenly becomes unresponsive, and no pulses are detected. An ECG performed in triage shows the differing amplitude of the QRS complex. Following CPR and airway management, what is the next appropriate step?

Choices: 1. Give the patient an IV fluid bolus 2. Perform emergent bedside pericardiocentesis 3. Obtain bedside trans-thoracic echocardiogram (TTE) 4. Consult cardiology for emergent cardiac catheterization

Answer: 2 - Perform emergent bedside pericardiocentesis Explanations: This patient is suffering from cardiac tamponade physiology as seen in exam findings including hypotension, venous distension, distant heart sounds and ECG findings consistent with electrical alternans. This patient needs emergent management in reversal of cardiac arrest from tamponade physiology. Bedside pericardiocentesis is the next step in managing this patient. In a stable patient, both intravenous fluids and TEE are reasonable prior to pericardiocentesis as increased volume promoting filling and increasing cardiac output temporarily. Bedside TTE will also be useful in confirming cardiac tamponade. Cardiac tamponade is one the T's in the H's and T's list when evaluating pulseless electrical activity. Go to the next page if you knew the correct answer, or click the link image(s) below to further research the concepts in this question (if desired).

Research Concepts: Electrical Alternans

We update eBooks quarterly and Apps daily based on user feedback. Please tap flag to report any questions that need improvement.

Question 408: A 62-year-old man with a medical history of hypertension and pulmonary embolism. He takes amlodipine 5 mg a day and apixaban 5 mg BID. He presents to the emergency department with a chief complaint of a severe headache. He last took his medications 5 hours ago at 8 am. CT scan reveals intracranial hemorrhage. Apixaban therapy is held, and a decision is made to treat with andexanet alfa. What is the recommended dose of andexanet alfa for this patient?

Choices: 1. 400 mg IV infusion at 30 mg/min 2. 400 mg IV bolus at a rate of 30 mg/min, followed by an IV infusion at 4 mg/min for up to 2 hours 3. 400 mg IV bolus followed by a drip at 2mg/min 4. 800 mg IV infusion at 4 mg/min

Answer: 2 - 400 mg IV bolus at a rate of 30 mg/min, followed by an IV infusion at 4 mg/min for up to 2 hours

Explanations: Dosing of andexanet alfa for reversal is dependent on the factor Xa inhibitor given, the dose, and the time of the last dose. There are two dosing regimens, low and high. Low dose is 400 mg IV bolus at a rate of 30 mg/min followed by an IV infusion at 4 mg/min for up to 2 hours. High dose is 800 mg IV bolus at a rate of 30 mg/min followed by an IV infusion at 8 mg/min for up to 2 hours. If the last dose of apixaban was 5 mg or less and was given less than 8 hours ago, give the low dose. Administering more than one dose has not been evaluated in clinical trials. Go to the next page if you knew the correct answer, or click the link image(s) below to further research the concepts in this question (if desired).

Research Concepts: Andexanet Alfa

We update eBooks quarterly and Apps daily based on user feedback. Please tap flag to report any questions that need improvement.

Question 409: A 65-year-old male with a history of hypertension and depression on enalapril and citalopram presents with abdominal pain for 4 days. Exam reveals a blood pressure of 156/85 mmHg, pulse 115 bpm, temperature of 101.5 F (38.6 C), respirations of 16, abdominal rebound tenderness in the left lower quadrant, and a white blood cell count of 34,000. What is the next best step?

Choices: 1. Contrast enhanced abdominal and pelvic CT 2. Stool cultures 3. Flexible sigmoidoscopy 4. Plain radiographs

Answer: 1 - Contrast enhanced abdominal and pelvic CT Explanations: The best study would be an abdominopelvic CT, preferably with contrast. CT findings include fat stranding, bowel wall thickening, and presence of diverticula. CT scans are fast and readily available Colonoscopy has no role in either diagnosis or management of acute diverticulitis, as it can put the patient at further risk for perforation or worsening of inflammation. It is recommended that a colonoscopy be performed approximately 6 to 8 weeks after symptoms have resolved to rule out malignancy if the patient has not been scoped in the recent past. Plain radiographs can show nonspecific findings such as ileus or small bowel obstruction. They can also show free air if a perforation has occurred. However, none of these findings are specific to diverticulitis. Abdominal ultrasonography, when properly performed have a comparative sensitivity to CT. However, it is very operator-dependent. The findings associated with diverticulitis include a hypoechoic peridiverticular inflammatory reaction, mural and peridiverticular abscess formation with or without gas bubbles, bowel wall thickening at the point of maximal tenderness, and presence of diverticula in the surrounding segments. Go to the next page if you knew the correct answer, or click the link image(s) below to further research the concepts in this question (if desired).

Research Concepts: Acute Diverticulitis

We update eBooks quarterly and Apps daily based on user feedback. Please tap flag to report any questions that need improvement.

Question 410: A previously healthy 16-year-old male presents with fever and rash after recently having nasal packing placed following surgery. He is tachycardic with an initial blood pressure of 80/50 mmHg and a temperature of 102 F. The rash is diffuse, red, and painful, and he has ulceration of the oral mucosa. Which of the following is the appropriate management and disposition?

Choices: 1. Aggressive fluid hydration, antibiotics, and admission to the intensive care unit 2. Aggressive fluid hydration, removal of the packing, IV antibiotics, and admission to the intensive care unit 3. Removal of the packing, intravenous fluids, oral antibiotics, and discharge home 4. Intravenous hydration, removal of the packing, and admission to the floor

Answer: 2 - Aggressive fluid hydration, removal of the packing, IV antibiotics, and admission to the intensive care unit

Explanations: This patient has toxic shock syndrome and requires intensive care unit admission. While intravenous fluids and antibiotics also are appropriate, any source of infection and toxin production needs to be removed. Toxic shock syndrome can be caused by group A Streptococcus or Staphylococcus aureus. Intravenous penicillin should be administered in addition to a beta-lactamase resistant antibiotic. Go to the next page if you knew the correct answer, or click the link image(s) below to further research the concepts in this question (if desired).

Research Concepts: Toxic Shock Syndrome

We update eBooks quarterly and Apps daily based on user feedback. Please tap flag to report any questions that need improvement.

Question 411: An 89-year-old female is undergoing a cecopexy to repair a cecal volvulus at a rural hospital. The anesthesiology provider wants to give her general anesthesia, inducing with both IV and inhaled anesthetics. The patient’s surgical history is significant for an appendectomy when she was 12 years old. The hospital pharmacy calls and informs the anesthesiology provider that there is a national shortage of sevoflurane and desflurane. The anesthesiology provider orders halothane to be given because he knows based on the patient’s past surgical history she tolerated halothane induction with no complications. While recovering on the medical-surgical floor, the patient develops jaundice, periodic episodes of blood oozing from her IV site, and later in her hospital course, increasingly severe altered mental status. What could be causing this symptomology?

Choices: 1. Ascending cholangitis 2. Viral-induced fulminant hepatitis 3. Drug-induced fulminant hepatitis 4. Disseminated intravascular coagulation (DIC)

Answer: 3 - Drug-induced fulminant hepatitis Explanations: Fulminant hepatic failure symptoms include jaundice, nausea, vomiting, dizziness, coagulopathy, and later development of altered mental status due to hepatic encephalopathy. Halothane is a volatile inhaled anesthetic, one of the original gases used as an amnestic/anesthetic agent in surgical procedures. Like all volatile gases, its believed mechanism of action is the augmentation of GABA signaling, which decreases excitatory signaling in the central nervous system to induce sedation, amnesia, anesthesia. Halothane-induced fulminant hepatitis is an uncommon occurrence seen in those who have had past exposure to halothane with either no liver dysfunction or well-controlled liver disease. Another adverse effect seen with halothane administration is malignant hyperthermia. This presents as muscle rigidity, hyperthermia, rapid onset tachycardia, hypercapnia, hyperkalemia, and metabolic acidosis. Go to the next page if you knew the correct answer, or click the link image(s) below to further research the concepts in this question (if desired).

Research Concepts: Anesthetic Gases

We update eBooks quarterly and Apps daily based on user feedback. Please tap flag to report any questions that need improvement.

Question 412: A 44-year-old previously healthy female presents to the emergency department in septic shock. Initial vitals are temperature 101.5 F, heart rate 115 bpm, blood pressure 110/50 mmHg, respiratory rate 24/minute, and pulse oximetry (SpO2) 78% on a nonrebreather mask. She is intubated for hypoxemic respiratory failure and acute respiratory distress syndrome secondary to pneumonia. She is placed on inverse ratio ventilation (IRV) to improve her oxygenation. Over the next 5 minutes, her SpO2 steadily increases to 95%. However, her repeat blood pressure has decreased to 70/40 mmHg. What is the most likely etiology of her hemodynamic compromise?

Choices: 1. Pneumothorax 2. Increased systemic inflammatory response syndrome (SIRS) response and vasodilation secondary to inflammatory mediators released in the lungs in response to barotrauma 3. Distributive shock secondary to sepsis 4. Decreased venous return

Answer: 4 - Decreased venous return Explanations: Hemodynamic compromise secondary to decreased venous return is a known complication of inverse ratio ventilation (IRV). The increased mean airway pressures used in IRV translate to increased intrathoracic pressure, which may cause an iatrogenic obstructive shock picture. Patients with preexisting hemodynamic instability are at increased risk for shock when placed on IRV. Patients with sepsis may have an element of distributive shock that goes unrecognized due to compensation. However, if an additional insult occurs, such as an obstruction to right ventricular preload, this shock may rapidly decompensate. Go to the next page if you knew the correct answer, or click the link image(s) below to further research the concepts in this question (if desired).

Research Concepts: Inverse Ratio Ventilation

We update eBooks quarterly and Apps daily based on user feedback. Please tap flag to report any questions that need improvement.

Question 413: A 22-year-old healthy male presents to the hospital with abdominal pain, nausea, vomiting, and yellowish discoloration of his skin. He recently was on vacation in Mexico. His vitals are within normal limits. Labs were significant for ALT 1100 IU/L and AST 1000 IU/L with total bilirubin 9 mg/dl. Hepatitis A, B, and C serology is negative. Hepatitis E IgM is negative. He is HIV negative and not sexually active. Ceruloplasmin levels within normal limits. He denies taking any prescribed or over-the-counter medications. What is the best next step in the management of this patient?

Choices: 1. Liver biopsy 2. Hepatitis E PCR 3. Anti-tissue transglutaminase antibody 4. Ferritin

Answer: 2 - Hepatitis E PCR Explanations: The patient’s symptoms and labs are consistent with hepatitis. Given his history of travel to Central America, he is at risk of acquiring hepatitis E virus (HEV) infection. With negative hepatitis A, B, C, HIV, normal ceruloplasmin levels, and no history of new medications, he likely has acute HEV infection. HEV serology is used for the evaluation of HEV, however, the sensitivity and specificity vary between different lab assays. Patients with high clinical suspicion of HEV infection and negative HEV IgM then HEV PCR should be checked as viremia arises early during the incubation period. Liver biopsy is not indicated as this patient likely has acute HEV infection. Anti-tissue transglutaminase antibody is used in the evaluation of celiac disease. The patient did not report any diarrhea and celiac disease does not cause high elevations in serum transaminases such as in this patient that are consistent with acute viral hepatitis. Ferritin is an acute phase reactant that is elevated during inflammation but is not specific. Go to the next page if you knew the correct answer, or click the link image(s) below to further research the concepts in this question (if desired).

Research Concepts: Hepatitis E

We update eBooks quarterly and Apps daily based on user feedback. Please tap flag to report any questions that need improvement.

Question 414: A 65-year-old female with rheumatoid arthritis is on methotrexate 12.5 mg weekly and prednisone 7.5 mg daily. She is admitted for urosepsis. Her vital signs are blood pressure 86/52 mmHg, heart rate 120 beats per minute, temperature 38.9 C, respiratory rate 22 breaths per minute, and oxygen saturation 95% on room air. A complete blood count shows white blood cells 22,000 with a left shift and a hematocrit of 24%. Her electrolytes are normal, but the BUN is 45 mg/dL and creatinine is 2.2 mg/dL. She is started on appropriate antibiotics, but after 2 liters of fluids, she is still hypotensive with a central venous pressure of 20 cmH2O. What is the most appropriate treatment?

Choices: 1. Transfuse two units of packed red blood cells 2. Continue crystalloids at 500 mL/hour 3. Start dopamine 4. Start intravenous hydrocortisone

Answer: 4 - Start intravenous hydrocortisone Explanations: The patient is having an Addisonian crisis. She is steroid dependent and cannot adequately respond to the stress of sepsis with endogenous glucocorticoids. Continuing fluids would not be the best option. Her central venous pressure is high and fluids could precipitate congestive heart failure. If the hydrocortisone is not effective, pressors such as norepinephrine or dopamine may be needed. Go to the next page if you knew the correct answer, or click the link image(s) below to further research the concepts in this question (if desired).

Research Concepts: Addisonian Crisis

We update eBooks quarterly and Apps daily based on user feedback. Please tap flag to report any questions that need improvement.

Question 415: A 75-year-old man is admitted for abdominal pain. He is febrile to 103 F. His blood pressure falls to 90/60 mmHg with a pulse of 130 beats/min. He seems restless and agitated. A urinalysis shows many bacteria and WBCs. He also has thick, brown discharge in his Foley catheter. CT abdomen confirms active diverticulitis with a colovesical fistula. What is the next step in management?

Choices: 1. Intravenous fluids and antibiotics 2. Surgery 3. Bowel rest 4. Abdominal MRI

Answer: 1 - Intravenous fluids and antibiotics Explanations: Fluid resuscitation and intravenous antibiotics are the cornerstones of managing sepsis, regardless of the source. Antibiotic regimens for diverticulitis must cover gram-negative rods and anaerobes, the dominant bacteria within the gastrointestinal tract. These include ciprofloxacin plus metronidazole, trimethoprim/sulfamethoxazole, ampicillin/sulbactam, or moxifloxacin only if intolerant to the previous three regimens. If improving, the patient may be switched from intravenous therapy to oral therapy after approximately 3 to 5 days. They should complete a 10 to 14-day course in total. All cases of complicated diverticulitis including abscess, fistula formation, perforation, and obstruction require inpatient admission. Patients with uncomplicated diverticulitis plus any of the following also require inpatient admission, sepsis, immunosuppression, high fevers, severe abdominal pain, advanced age, decreased oral intake, noncompliance with medications, or those who have failed an outpatient therapy. Surgery, commonly bowel resection, is indicated if a patient’s condition continues to deteriorate despite appropriate medical therapy. In addition, complicated diverticulitis often requires surgical intervention for abscess drainage or resection of the affected colon if obstructed, perforated, or fistulized. Go to the next page if you knew the correct answer, or click the link image(s) below to further research the concepts in this question (if desired).

Research Concepts: Acute Diverticulitis

We update eBooks quarterly and Apps daily based on user feedback. Please tap flag to report any questions that need improvement.

Question 416: A male presents with new onset of bilateral leg pain and weakness. Three weeks prior the patient was hospitalized for severe gastroenteritis and dehydration. He describes the leg pain as a pins and needles sensation in both legs. Recently he just got a new job in a metal foundry and noticed that his symptoms began soon after. Physical exam reveals decreased vibratory sense in his ankle joint bilaterally, tachycardia and garlic odor from his mouth. Which study would not be useful if the exposure occurred more than a few hours ago?

Choices: 1. Plasma levels 2. Electrocardiogram 3. Abdominal x-ray 4. 24-hour urinary levels

Answer: 1 - Plasma levels Explanations: Serum arsenic levels have not been shown to be an effective or reliable marker for arsenic toxicity due to rapid clearance of arsenic from the bloodstream. Arsenic serum levels are only detectable for a few hours after ingestion. However, urinary levels last for days to weeks after ingestion depending on dosage and oxidative state. Whole blood is excellent for detecting other metal toxins such as lead secondary to tight binding of intracellular proteins. Treatment for suspected arsenic toxicity should be started prior to any laboratory confirmation. Go to the next page if you knew the correct answer, or click the link image(s) below to further research the concepts in this question (if desired).

Research Concepts: Arsenic Toxicity

We update eBooks quarterly and Apps daily based on user feedback. Please tap flag to report any questions that need improvement.

Question 417: A 36-year-old female is admitted to the hospital for gastroenteritis. On the second day after admission, she develops petechiae on her legs. Blood work reveals hemoglobin of 8 mg/dL, down from 14 mg/dL, and platelet count of 56,000, down from 250,000/ microL. Her creatinine is also mildly elevated from 0.8 to 1.4. She appears drowsy and has also developed a fever of 39 C. WBC count is 8000/microL which is the same as admission. A pregnancy test is negative. She denies any diarrhea. Which of the following is the next best step in the management of this patient?

Choices: 1. Watchful waiting and supportive treatment 2. Start plasma exchange and methylprednisolone 3. Start methylprednisolone alone 4. Obtain kidney ultrasound and further workup before starting management

Answer: 2 - Start plasma exchange and methylprednisolone Explanations: The suspected diagnosis for this patient is thrombotic thrombocytopenic purpura (TTP). Given the high suspicion of TTP from this patient’s labs, presentation, and lack of other clear causes including pregnancy leading to DIC, and eclampsia; and the high mortality of TTP, it is essential to start plasma exchange therapy (PEX) along with a glucocorticoid immediately as this reduces mortality. Key factors for distinguishing between hemolytic syndromes are age, the time course of the disease, and the degree of kidney injury. It is also associated with diarrhea and gastrointestinal discomfort, which can lead to worse kidney function. PEX is often used with a glucocorticoid to reduce the production of ADAMTS13 inhibitor antibody as well as reduce cytokine production. Rituximab, an antibody against CD20 of B cells, is also commonly used with PEX as this produces better outcomes. Go to the next page if you knew the correct answer, or click the link image(s) below to further research the concepts in this question (if desired).

Research Concepts: Acute Anemia

We update eBooks quarterly and Apps daily based on user feedback. Please tap flag to report any questions that need improvement.

Question 418: A 51-year-old male presents with fever, productive cough, and worsening chronic right shoulder pain. He has a history of interstitial lung disease and dermatomyositis. He admits he is undergoing immunosuppressive therapy and was admitted one month ago for pneumonia. He denies trauma or recent corticosteroid injections to the shoulder. On exam, there is evidence of dermatomyositis, and his right shoulder is limited in all planes of motion with a fluctuant mass in the superior and posterior aspects of the joint. His lungs are clear to auscultation. Bedside ultrasound reveals a large subacromial bursal fluid collection, and a radiograph of the chest shows interstitial infiltrates. What is the most appropriate treatment for this patient?

Choices: 1. Ethambutol orally and rifampin, isoniazid, and pyridoxine intravenously 2. High dose corticosteroids and levofloxacin intravenously 3. Incision and drainage of the bursal fluid collection followed by doxycycline orally 4. Surgical debridement of the right shoulder and vancomycin intravenously

Answer: 1 - Ethambutol orally and rifampin, isoniazid, and pyridoxine intravenously Explanations: This patient is immunocompromised and at risk for fungal and atypical infections. His likely diagnosis is septic bursitis and pulmonary infection likely from Mycobacterium. The patient exhibits pulmonary, and extrapulmonary signs and symptoms, including pulmonary infiltrates, evidence of dermatomyositis, and deep bursal infection. Diagnosis of Mycobacterial infection involves sputum and bursal fluid culture in this patient and histopathology. Treatment for disseminated disease is the same for pulmonary and includes ethambutol, rifampin, isoniazid, and pyridoxine. High-dose corticosteroid is the treatment for his dermatomyositis and levofloxacin for pneumonia in an immunosuppressed individual. However, the patient also has septic bursitis, and levofloxacin would be inadequate. Given the patient is immunocompromised, this patient should be admitted for intravenous antibiotics. If the patient was immunocompetent, then doxycycline could cover both septic bursitis and pneumonia. Given the patient's history, he likely has a Mycobacterium infection, which requires a 3 to 4 drug regimen. Go to the next page if you knew the correct answer, or click the link image(s) below to further research the concepts in this question (if desired).

Research Concepts: Septic Bursitis

We update eBooks quarterly and Apps daily based on user feedback. Please tap flag to report any questions that need improvement.

Question 419: A 70-year-old man undergoes surgical resection for prostate cancer. During the early postoperative course, he develops hypotension and congestive heart failure requiring intubation. His echocardiogram demonstrates akinesis of the basal segment and hyperkinesis of the apical segment. Which of the following forms of cardiomyopathy does he most likely have?

Choices: 1. Reverse or inverted stress cardiomyopathy 2. Dilated cardiomyopathy 3. Idiopathic cardiomyopathy 4. Stress cardiomyopathy

Answer: 1 - Reverse or inverted stress cardiomyopathy Explanations: The clinical history and echocardiographic images are consistent with reverse (or inverted) Takotsubo (stress cardiomyopathy) in which there are basal akinesis and hyperkinetic apex. The stressor, in this case, was the recent surgery. Several variants of Takotsubo cardiomyopathy have been described in the literature including the classical apical ballooning (akinesis) variant with basal hyperkinesis, midcavity ballooning, and basal–mid ballooning with apical hyperkinesis. There are also variations with right ventricular apical involvement. This patient has left ventricular involvement on the echocardiogram with the reverse variant. Although males have much less predisposition for the development of Takotsubo cardiomyopathy, they often have a worse prognosis because of the increased comorbidities burden. Go to the next page if you knew the correct answer, or click the link image(s) below to further research the concepts in this question (if desired).

Research Concepts: Takotsubo Cardiomyopathy

We update eBooks quarterly and Apps daily based on user feedback. Please tap flag to report any questions that need improvement.

Question 420: A patient presents to the emergency department with symptoms of vomiting, diarrhea, abdominal cramping, bronchospasm, miosis, bradycardia, excessive salivation, and sweating. He also exhibits muscle fasciculations and weakness and has difficulty breathing. He is treated with multiple doses of atropine and pralidoxime. This patient was most likely poisoned with which of the following?

Choices: 1. Aldicarb 2. Aldrin 3. Parathion 4. Physostigmine

Answer: 3 - Parathion Explanations: Parathion is an organophosphate insecticide. It acts on the enzyme acetylcholine esterase. Symptoms of toxicity include blurred vision, diarrhea, excess salivation, tremor, dyspnea, and unconsciousness. Atropine is often given to block the excess acetylcholine activity. Pralidoxime is given to reactivate the cholinesterase. Go to the next page if you knew the correct answer, or click the link image(s) below to further research the concepts in this question (if desired).

Research Concepts: Organophosphate Toxicity

We update eBooks quarterly and Apps daily based on user feedback. Please tap flag to report any questions that need improvement.

Question 421: A 66-year-old female with a past medical history of liver cirrhosis complicated by ascites, hypertension, and type 2 diabetes presents to the emergency department with a 3-day history of abdominal pain. She describes the pain as 6/10, limited to her right upper quadrant, and associated with some nausea but no episodes of vomiting. A plain abdominal x-ray shows free air under the diaphragm. An abdominal CT shows right bowel distension but no free air. Which of the following is the next best step in the management of this patient?

Choices: 1. Exploratory laparotomy 2. Nasogastric tube decompression 3. Endoscopy 4. Colonoscopy

Answer: 2 - Nasogastric tube decompression Explanations: The absence of free air under the diaphragm on CT scan indicates that initial findings are consistent with pneumoperitoneum. This is classic of Chilaiditi syndrome, in which a segment of a large bowel loop or small intestine is interposed between the liver and a diaphragm. Treatment is conservative with nasogastric tube decompression, bed rest, IV fluids. Surgical intervention is indicated when the patient does not respond to conservative management, and either the obstruction fails to resolve, or there is evidence of bowel ischemia. Go to the next page if you knew the correct answer, or click the link image(s) below to further research the concepts in this question (if desired).

Research Concepts: Chilaiditi Syndrome

We update eBooks quarterly and Apps daily based on user feedback. Please tap flag to report any questions that need improvement.

Question 422: A 78-year-old female came in the emergency with confusion and high-grade fever. After her urine and blood specimens were sent to the laboratory, her blood pressure drops to 89/60 mmHg. Suddenly she becomes out of breath with a respiratory rate of 30/min. Providers are considering acute respiratory distress syndrome secondary to sepsis and pneumonia. They ordered a chest x-ray and oxygen therapy, but they also wanted to start an empiric therapy for pneumonia. Which of the following is the most likely recommended therapy for her?

Choices: 1. Macrolides 2. Doxycycline 3. Beta-lactam 4. Beta-lactam and a fluoroquinolone

Answer: 4 - Beta-lactam and a fluoroquinolone Explanations: In elderly patients sometimes confusion is the earliest manifestation of pneumonia. Patients with community-acquired pneumonia are often risk stratified using the pneumonia severity index or CURB-65 criteria to help empiric treatment and treatment location. This patient with a score of 4 on CURB-65 criteria with confusion, age greater than 65 years, blood pressure less than 90/60 mmHg, and respiratory rate of 30/min, likely needs admission in the intensive care unit and empiric therapy. The recommended therapy for patients in the ICU is a beta-lactam and a fluoroquinolone or a beta-lactam and a macrolide. Go to the next page if you knew the correct answer, or click the link image(s) below to further research the concepts in this question (if desired).

Research Concepts: Bacterial Pneumonia

We update eBooks quarterly and Apps daily based on user feedback. Please tap flag to report any questions that need improvement.

Question 423: A 23-year-old male patient is brought to the emergency department by his family after he “drank something” in a suicide attempt. On arrival to the ED, he is confused, sweaty, salivating, and has pinpoint pupils. His confusion, vomiting, excessive salivation, and copious bronchorrhea make it hard to manage him. His GCS is 7. He is prepared to be intubated. Which of the following medications is most appropriate to facilitate intubation?

Choices: 1. Etomidate and rocuronium 2. Etomidate and succinylcholine 3. Midazolam and succinylcholine 4. Pyridostigmine

Answer: 1 - Etomidate and rocuronium Explanations: Usual airway procedures are used with one exception. During intubation, succinylcholine should not be used because the lack of acetylcholinesterase caused by the poisoning will cause prolonged paralysis. Depolarizing paralytics, such as succinylcholine, should not be used in cholinergic toxicity. Non-polarizing paralytics, such as vecuronium and rocuronium, may be used safely. The choice of induction agent is not changed when treating cholinergic toxicity. Etomidate, midazolam, and propofol may all be used. Rapid sequence intubation would give the best chance for first pass intubation in this patient who is not completely comatose. Go to the next page if you knew the correct answer, or click the link image(s) below to further research the concepts in this question (if desired).

Research Concepts: Cholinergic Toxicity

We update eBooks quarterly and Apps daily based on user feedback. Please tap flag to report any questions that need improvement.

Question 424: Which of the following is the safest positioning to preserve neurologic function prior to reduction and stabilization of a T12-L1 dislocation fracture with an incomplete neurologic deficit?

Choices: 1. Supine 2. Lateral decubitus with lower extremities extended 3. Lateral decubitus with knees flexed 4. Prone with rolls under chest and thighs

Photo:Contributed by Scott Dulebohn, MD

Answer: 3 - Lateral decubitus with knees flexed Explanations: Patients in the supine position have an extension of the thoracolumbar spine at the level of a fixed kyphosis secondary to the dislocation fracture. This causes further narrowing of an already compromised spinal canal. Placing the patient in the lateral decubitus position with the knees flexed may provide enough decompression to allow improvement in an individual who is neurologically incomplete. The change in position required to turn a patient prone causes unnecessary manipulation of the spinal column. Go to the next page if you knew the correct answer, or click the link image(s) below to further research the concepts in this question (if desired).

Research Concepts: Thoracolumbar Spine Fracture

We update eBooks quarterly and Apps daily based on user feedback. Please tap flag to report any questions that need improvement.

Question 425: A 58-year-old male is brought to the emergency department with the complaint of palpitations and dizziness. Past medical history is significant for diabetes. Vital signs are pulse rate 132/min, blood pressure 102/55 mm Hg, respiratory rate 21/min, and temperature 98.6 F (37C). EKG shows polymorphic QRS complexes around an isoelectric line. IV magnesium is given, but there is no improvement. What is the next step in the management of this patient?

Choices: 1. Amiodarone 2. Beta-blocker 3. Isoproterenol 4. Adenosine

Photo:Contributed by Wikimedia Commons, Panthro (Public Domain-Self)

Answer: 3 - Isoproterenol Explanations: Torsades de Pointes is a type of polymorphic ventricular tachycardia characterized by a gradual change in amplitude and twisting of the QRS complexes around an isoelectric line on the electrocardiogram. Intravenous magnesium is the first-line pharmacologic therapy in Torsades de Pointes. For a patient that continues to have Torsades de Pointes, despite treatment with magnesium, increasing the heart rate may also help. This can be done pharmacologically with medications such as isoproterenol. Isoproterenol has been shown to help prevent Torsades de Pointes in patients with prolonged QT that is refractory to magnesium. It is a non-selective beta-agonist, which increases the heart rate and shortens the QT interval. This lowers the likelihood of an R-onT phenomenon that can lead to TdP. Isoproterenol can be given as an IV push of 10 mcg to 20 mcg or an infusion titrated to maintain a heart rate of 100 bpm. Isoproterenol is, however, contraindicated in patients with congenital prolonged QT, because it can paradoxically lengthen the QT interval. Go to the next page if you knew the correct answer, or click the link image(s) below to further research the concepts in this question (if desired).

Research Concepts: Torsade de Pointes

We update eBooks quarterly and Apps daily based on user feedback. Please tap flag to report any questions that need improvement.

Question 426: A 32-year-old woman presents after a fall on an icy sidewalk where she hit her head and lost consciousness for several minutes. Her vital signs show oxygen saturation 98% on room air, respiratory rate 20 per minute, heart rate 55 beats per minute, blood pressure 140/90 mmHg, and temperature 98 F. On examination, she is awake, alert, and complains of a severe headache. There are no cranial nerve, motor, or sensory deficits. Babinski sign is positive on the right side. The patient undergoes a CT scan (see below). In which cranial nerve would you expect the patient to develop a deficit?

Choices: 1. CN I 2. CN III 3. CN IV 4. CN VI

Photo:Contributed by Scott Dulebohn, MD

Answer: 2 - CN III Explanations: The CT scan shows a subdural hematoma with subarachnoid hemorrhage on the right. The complication would include an increase in the size of hemorrhage as well as edema of the right temporal lobe. This may predispose to herniation of the medial temporal lobe leading to uncal herniation. Uncal herniation compresses CN III. The clinical presentation consists of a dilated ipsilateral pupil which does not respond to light. On further compression, the affected eye will be looking down and out. Further herniation may compress the brainstem and cause a decrease in sensation. The uncus and the adjacent temporal lobe slide down the tentorial incisura and compress the posterior cerebral arteries and the brain stem. It carries a poor prognosis. Go to the next page if you knew the correct answer, or click the link image(s) below to further research the concepts in this question (if desired).

Research Concepts: Uncal Herniation

We update eBooks quarterly and Apps daily based on user feedback. Please tap flag to report any questions that need improvement.

Question 427: An 81-year-old man is brought to the emergency department due to several hours of confusion, lethargy, vomiting, and abdominal pain. He resides in a nursing home, and his oral intake has been poor over the last 2 days. He has a history of aplastic anemia and underwent a bone marrow transplant recently. He also has a history of hypertension, type 2 diabetes mellitus, chronic kidney disease, and coronary artery disease. He has been taking her medications without issue, but his long-acting insulin was held last night. Vital signs show temperature 35.7 C (96.3 F), blood pressure 90/40 mmHg, pulse 98/min, and respiratory rate 18/min. Pulse oximetry shows 94% on room air. The patient is unable to answer questions or follow commands. Cardiopulmonary examination is unremarkable. No edema is present in the extremities, and the skin is warm to touch. There is mild pain on deep palpation on the right lower quadrant. Neurologic examination is limited due to the patient's mental status, but no obvious focal abnormalities are noted. EKG reveals normal sinus rhythm with minor T-wave abnormalities. Laboratory results are shown below. The patient is given a 500mL bolus of normal saline. Repeat vital signs reveal blood pressure 85/40 mmHg and heart rate 95/min. Which of the following is the next best step in the management of this patient?

Choices: 1. CT scan of the abdomen 2. Intravenous albumin and broad-spectrum antibiotics 3. Emergency exploratory laparotomy 4. Normal saline boluses and broad-spectrum antibiotics

Photo:Contributed by Evelyn E Inga

Answer: 4 - Normal saline boluses and broad-spectrum antibiotics Explanations: This elderly patient with hypothermia, hypotension, and leukocytosis likely has an ongoing infection, despite being an undetermined source, the physical examination suggests a septic abdomen. The next step is to determine if the patient has sepsis, as early sepsis intervention is associated with significant reductions in mortality. This patient was under immunosuppression due to bone marrow transplantation after being diagnosed with aplastic anemia. These antecedents plus the physical y laboratory findings suggest neutropenic enterocolitis (NE). Timely conservative treatment frequently allows the resolution of neutropenic enterocolitis without operation since this patient doesn't present peritoneal signs. Although the conservative treatment follows a classic case of NE, the diagnosis of sepsis demands aggressive intravenous fluid resuscitation and the early initiation of antibiotics since these measures have been shown to reduce mortality. Sepsis screening in the emergency department is most often performed with the qSOFA score, a simple bedside calculation that assigns 1 point each for respiratory rate >22/min, altered mentation, and systolic blood pressure 100 mm Hg. Patients with qSOFA scores >2 (as in this patient with confusion and hypotension) are likely to be septic. Two major initial interventions reduce the risk of sepsis-related mortality: aggressive intravenous fluids and early antibiotic therapy. Intravenous albumin is expensive and has not been shown to be superior to crystalloid fluids for the treatment of early sepsis. As such, albumin is not recommended. Colonoscopy is contraindicated in neutropenic enterocolitis, but endoscopy may be life-saving in other patients with GI bleeding. Surgery is reserved in cases of persistent decline in the patient's evolution or any finding suggesting bowel perforation. This patient signs, symptoms, and past medical history suggest sepsis - neutropenic enterocolitis. Sepsis is associated with high rates of death. Aggressive intravenous fluid resuscitation and the early initiation of antibiotics have been shown to reduce mortality. Empiric broad-spectrum antibiotics should be administered within 1 hour of presentation (ideally after blood cultures have been drawn). Go to the next page if you knew the correct answer, or click the link image(s) below to further research the concepts in this question (if desired).

Research Concepts: Neutropenic Enterocolitis

We update eBooks quarterly and Apps daily based on user feedback. Please tap flag to report any questions that need improvement.

Question 428: A 26-year-old woman presents with a six-month history of shortness of breath. This symptom started one week after she was diagnosed with lung cancer. All her vital signs are within normal limits. Chest x-ray shows moderate to large left-sided pleural effusion. The patient states she has had fluid in her lung drained before. The appropriate procedure will be performed. Which of the following is the most appropriate position for this intervention?

Choices: 1. Sitting 2. Supine 3. Left lateral decubitus 4. Right lateral decubitus

Photo:Contributed by National Heart, Lung and Blood Institute ( PD-US NIH)

Answer: 1 - Sitting Explanations: Thoracentesis is performed to remove fluid from the pleural cavity. Often even with stable vital signs, patients may have orthopnea. The preferred body position is sitting, with the patient leaning slightly forward and the arms supported on a bedside table. In the sitting position, ultrasound can readily visualize large fluid pockets much easier as well as identify organs of interest such as the diaphragm. For most patients, the sitting position is well tolerated and provides ample access to the back. The lateral decubitus position can be used to insert a chest tube. The arm must be over the shoulder to open up the intercostal space. Go to the next page if you knew the correct answer, or click the link image(s) below to further research the concepts in this question (if desired).

Research Concepts: Thoracentesis

We update eBooks quarterly and Apps daily based on user feedback. Please tap flag to report any questions that need improvement.

Question 429: A 35-year-old man is brought in by emergency services after an industrial accident. There was a 60% total body surface area full-thickness burn, which was covering the face, both hands, and the trunk. The patient was intubated en route and was started on mechanical ventilation on arrival. A chest x-ray shows bilateral alveolar infiltrates, and a fiberoptic bronchoscopy is performed that shows severe blisters and ulcers of the mucosa of the bronchus. Which of the following can be an immediate complication in cases such as these?

Choices: 1. Subglottic stenosis 2. Bronchiectasis 3. Laryngeal edema 4. Acute respiratory distress syndrome (ARDS)

Photo:Contributed by Wikimedia Commons (CC BY 2.0) https://creativecommons.org/licenses/by/2.0/deed.en

Answer: 3 - Laryngeal edema Explanations: In inhalational injury, the severity of respiratory tract injury is directly proportional to the length and intensity of exposure. Mild to moderate injury is largely self-limited with patients having no complications. In these cases, symptoms will often resolve within 2 to 3 days. Laryngeal edema, secondary to inhalation injury, can obstruct the airway and may necessitate tracheostomy in severe cases. Short-term complications are seen in more severe injuries within 4 to 5 days, and the most common issue is pneumonia. Acute respiratory distress syndrome and pulmonary edema are also seen in the short term. Long-term complications from smoke inhalation injury are much less common. They include subglottic stenosis, bronchiectasis, and bronchiolitis obliterans. Go to the next page if you knew the correct answer, or click the link image(s) below to further research the concepts in this question (if desired).

Research Concepts: Inhalation Injury

We update eBooks quarterly and Apps daily based on user feedback. Please tap flag to report any questions that need improvement.

Question 430: A 44-year-old man is brought to the emergency department from a paint factory, where he was overcome by fumes. An oropharyngeal airway (OPA) is in place. Breath sounds are present bilaterally. Which of the following statements is true?

Choices: 1. If upper airway bleeding occurs, the OPA should be left in place to control bleeding 2. The OPA should not be used in apneic comatose patients 3. Ventilation is as efficient and controlled with an OPA as with an endotracheal tube 4. OPA is contraindicated when the patient has a gag reflex

Photo:Contributed by Tammy J. Toney-Butler, RN, AS, CEN, TCRN, CPEN

Answer: 4 - OPA is contraindicated when the patient has a gag reflex Explanations: Since the airway is not protected by an oropharyngeal airway (OPA), upper airway bleeding will allow blood into the airways. OPAs are not designed to control bleeding. The gag reflex generally causes bleeding and therefore is a contraindication to use of an OPA. An oropharyngeal airway (oral airway, OPA) is an airway adjunct used to maintain or open an airway by preventing the tongue from covering the epiglottis. When a patient is unconscious, the muscles in their jaw relax and allow the tongue to obstruct the airway. Oropharyngeal airways come in sizes for infants to adults, and are used commonly in prehospital emergency care and for short-term airway management post or when manual methods are inadequate to maintain an open airway. Oropharyngeal airways are indicated only in unconscious patients because of the likelihood they will stimulate a gag reflex and vomiting in a conscious person. Nasopharyngeal airways are used instead as they do not stimulate a gag reflex. Oropharyngeal airways must be sized and inserted correctly to maximize effectiveness and minimize complications. The correct OPA size is chosen by measuring from the first incisors to the angle of the jaw. The airway is inserted into the mouth upside down. Once contact is made with the back of the throat, the airway is rotated 180 degrees, allowing insertion, and assuring the tongue is secured. The device is removed when the patient regains swallow reflex and can protect their own airway. It is removed pulling on it without rotation. Use of an OPA does not remove the need for the recovery position and ongoing airway assessment, and it does not prevent obstruction by liquids or the closing of the glottis. Go to the next page if you knew the correct answer, or click the link image(s) below to further research the concepts in this question (if desired).

Research Concepts: Oropharyngeal Airway

We update eBooks quarterly and Apps daily based on user feedback. Please tap flag to report any questions that need improvement.

Question 431: A 65-year-old man presents to the emergency department with complaints of severe crushing central chest pain radiating to the left arm, sweating, and vomiting. He has a history of hypertension, for which he takes amlodipine 10 mg once a day. On examination, his blood pressure is 80/55 mmHg, and there are fine bibasal crackles on auscultation of the chest. An electrocardiogram is obtained which is shown below. Despite oxygen supplementation, he is still dyspneic. What is the next best step in his management?

Choices: 1. Dopamine 2. Transcutaneous pacing 3. Digoxin 4. Internal defibrillator

Photo:By MoodyGroove at English Wikipedia - Own work, Public Domain, https://commons.wikimedia.org/w/index.php?curid=18055698

Answer: 2 - Transcutaneous pacing Explanations: Dopamine infusion can be attempted but is often not successful in third-degree atrioventricular block. In patients with symptomatic bradycardia, transcutaneous pacing is indicated without delay. Although digoxin can cause bradycardia, there is no role for digoxin in treating AV node block with symptomatic bradycardia. The patient should be stabilized before a permanent pacemaker is placed. Go to the next page if you knew the correct answer, or click the link image(s) below to further research the concepts in this question (if desired).

Research Concepts: Third-Degree Atrioventricular Block

We update eBooks quarterly and Apps daily based on user feedback. Please tap flag to report any questions that need improvement.

Question 432: A 72-year-old man presents to the hospital with shortness of breath. Past medical history is significant for COPD on 2 L/min of oxygen by nasal cannula. The patient is hemodynamically stable, saturating 94% on 2 L/min of the nasal cannula. CT thorax shows a right-sided pleural effusion. The echocardiogram is unremarkable. The patient undergoes diagnostic and therapeutic thoracocentesis. Light's criteria is suggestive of transudate effusion. The patient says his symptoms are not improved. Repeat CT thorax post thoracocentesis is shown below. Which of the following is the most likely diagnosis?

Choices: 1. Lung cancer 2. Aspergillosis 3. Pneumothorax 4. Trapped lung

Photo:Contributed by Albores J, Wang T. Images in clinical medicine. Trapped lung. N Engl J Med. 2015 May 7;372(19):e25. doi: 10.1056/NEJMicm1404964. PMID: 25946304. (Open Access)

Answer: 4 - Trapped lung Explanations: The CT is showing chronic atelectasis with thickened visceral pleura. Following drainage of the effusion, the lung failed to expand, creating a vacuum and an air-fluid level. Malignancy is one of the causes of recurrent pleural effusions and trapped lung. Trapped lung can present as one of the complications of pneumothorax. Complicated pulmonary infections causing loculated effusions can cause trapped lung. Go to the next page if you knew the correct answer, or click the link image(s) below to further research the concepts in this question (if desired).

Research Concepts: Trapped Lung

We update eBooks quarterly and Apps daily based on user feedback. Please tap flag to report any questions that need improvement.

Question 433: In which electrolyte abnormality in the advanced stage would you expect to see a widening of the QRS and decreased P wave amplitude?

Choices: 1. Hypocalcemia 2. Hypercalcemia 3. Hyperkalemia 4. Hypomagnesemia

Photo:Contributed by Wikimedia Commons,"Medical gallery of Mikael Häggström 2014" (Public Domain)

Answer: 3 - Hyperkalemia Explanations: The presence of peaked T waves on an ECG should suggest hyperkalemia. Severe hyperkalemia may also be associated with wide QRS complexes and reduction in the size of P waves. Hyperkalemia causes an overall membrane depolarization that inactivates many sodium channels. Peaked T waves are the earliest changes on the ECG in patients with hyperkalemia. This is usually associated with a short QT interval and ST segment depression. As the hyperkalemia worsens, one will develop widening of the QRS and decreased P wave amplitude. Go to the next page if you knew the correct answer, or click the link image(s) below to further research the concepts in this question (if desired).

Research Concepts: Hyperkalemia

We update eBooks quarterly and Apps daily based on user feedback. Please tap flag to report any questions that need improvement.

Question 434: A 31-year-old male is seen in the emergency department and undergoes a quick neurological evaluation following head trauma. He is unable to open his eyes, has no motor movement, and makes no verbal sounds. What is his Glasgow coma scale score?

Choices: 1. Seven 2. Five 3. Three 4. Zero

Photo:Contributed by Tammy Toney-Butler, AS, RN, CEN, TCRN, CPEN

Answer: 3 - Three Explanations: The Glasgow coma scale is a neurological measure that gives a reliable estimate of the conscious state of an individual. The lowest score is three and the highest is 15. The scale assesses the best eye, best verbal, and best motor responses. A severe score is any number less than 8. A moderate score is between 9 and 12. A minor score is greater than 13. Go to the next page if you knew the correct answer, or click the link image(s) below to further research the concepts in this question (if desired).

Research Concepts: Glasgow Coma Scale

We update eBooks quarterly and Apps daily based on user feedback. Please tap flag to report any questions that need improvement.

Question 435: A 52-year-old man is brought in by emergency medical services (EMS) after being involved in a motor vehicle accident. When performing the Glasgow coma scale (GCS), there is slow flexion of the elbow, wrist, and fingers with adduction and internal rotation at the shoulder. The legs are extended and internally rotated at the hip, with the extension of the knee and plantar flexion of the feet. Which of the following is the most likely position of the toes in this posturing?

Choices: 1. Adducted and hyperextended 2. Abducted and hyperextended 3. Abducted and flexed 4. Adducted and flexed

Photo:Contributed by Wikimedia Commons (Public Domain)

Answer: 2 - Abducted and hyperextended Explanations: Decorticate posturing is described as abnormal flexion of the arms with the extension of the legs. It involves slow flexion of the elbow, wrist, and fingers with adduction and internal rotation at the shoulder. The lower limbs show extension and internal rotation at the hip, extending the knee and plantar flexion of the feet. Toes are typically abducted and hyperextended. Decorticate posturing is a clinical diagnosis and should be considered a pathological sign of a neurological injury. Go to the next page if you knew the correct answer, or click the link image(s) below to further research the concepts in this question (if desired).

Research Concepts: Decerebrate And Decorticate Posturing

We update eBooks quarterly and Apps daily based on user feedback. Please tap flag to report any questions that need improvement.

Question 436: A 60-year-old male complains of left-sided weakness that started ten hours ago. The patient cannot provide a clear, detailed medical history because his speech is slurred. According to the patient's medical file that he brought with him, he has a past medical history of hypertension and diabetes mellitus type 2. His medications include hydrochlorothiazide and metformin. Further information cannot be elicited. The patient was admitted to the hospital for further evaluation. On examination, temperature is 36.3 C, blood pressure is 184/100 mmHg, pulse is 84 beats/min, and respiratory rate is 17 breaths/min. He cannot move the left side of his body. Sensation is lost on the left side of his body. Cardiovascular examination is unremarkable, and there are no carotid bruits. CT scan of the brain is shown below. An electrocardiogram shows left ventricular hypertrophy with non-specific T wave changes. Blood tests show WBC of 6,000/mm3, hemoglobin of 13.8 g/dL, platelets of 270,000/UL, calcium of 9 mg/dL, blood urea nitrogen of 21 mg/dL, creatinine of 90 U/L, and an INR of 1.1. A bedside swallowing screening test revealed no abnormalities. The patient was given oral aspirin. What is the best next step in the management of this patient?

Choices: 1. Low molecular weight heparin for deep venous thrombosis prevention 2. Calcium channel blocker to treat hypertension 3. Observation 4. Intravenous mannitol to reduce cerebral edema

Photo:Contributed by Scott Dulebohn, MD

Answer: 1 - Low molecular weight heparin for deep venous thrombosis prevention Explanations: This patient complaining of left-sided hemiparesis and sensory loss on the left side of his body with a CT scan showing hypodensity in the right middle cerebral artery territory likely had a right middle cerebral artery ischemic stroke. Patients who present with a stroke less than 4.5 hours after symptom onset should be given fibrinolytic therapy. Patients who present after 4.5 hours of symptom onset should not be given fibrinolytic therapy and should be managed with aspirin and permissive hypertension. A blood pressure up to 220/120 mmHg is permitted in patients who did not receive fibrinolytic therapy. A swallow screening test should be done in all patients with a stroke before administering oral medications or food to avoid aspiration. All patients with ischemic stroke should receive subcutaneous heparin or low molecular weight heparin for deep vein thrombosis prophylaxis. Patients with hemorrhagic stroke should be given mechanical deep vein thrombosis prophylaxis, instead of heparin. Pulmonary embolism from deep vein thrombosis is the most common cause of early death in patients with acute stroke. Go to the next page if you knew the correct answer, or click the link image(s) below to further research the concepts in this question (if desired).

Research Concepts: Middle Cerebral Artery Stroke

We update eBooks quarterly and Apps daily based on user feedback. Please tap flag to report any questions that need improvement.

Question 437: What is the total Glasgow coma scale score for an adult patient who opens their eyes when asked, uses inappropriate words, and withdraws in response to pain?

Choices: 1. 8 2. 9 3. 10 4. 11

Photo:Contributed by Tammy Toney-Butler, AS, RN, CEN, TCRN, CPEN

Answer: 3 - 10 Explanations: The total Glasgow coma scale score is 10 out of 15. He receives three points for opening his eyes when asked. He receives three points for speaking with inappropriate words. He receives four points for withdrawing to pain. If the patient opened his eyes to a verbal request, that is a movement following a command and this is 6 points. Go to the next page if you knew the correct answer, or click the link image(s) below to further research the concepts in this question (if desired).

Research Concepts: Glasgow Coma Scale

We update eBooks quarterly and Apps daily based on user feedback. Please tap flag to report any questions that need improvement.

Question 438: A 65-year-old male presents to the emergency department with multiple injuries following a road traffic accident. His cervical spine is immobilized, and high flow oxygen is started. His breathing is noisy and labored, and oxygen saturation is 90% on 15 L O2. His heart rate is 112 bpm, and blood pressure is 102/60 mmHg. On examination, the patient's eyes are closed, and he is not making any verbal communications. With supraorbital pressure, he flexes at the elbow with arms brought close to his body and flexed wrists. The same stimulus causes him to open his eyes and moan. Pupils are equal and reactive to light. What is the next best step in management?

Choices: 1. Perform a focused assessment with sonography in trauma (FAST) 2. Transfuse crossmatched packed red blood cells 3. Rapid sequence induction and intubation 4. Start levetiracetam as seizure prophylaxis

Photo:Image courtesy S Bhimji MD

Answer: 3 - Rapid sequence induction and intubation Explanations: The patient has a Glasgow coma score of 7 with eyes 2, verbal 2, and motor 3. With this level of consciousness, his airway is compromised, demonstrated by his labored breathing. His oxygen saturations are 90% despite 15 L of high flow oxygen. Therefore, it is best practice to intubate and ventilate to stabilize his airway and breathing. Once stable and safe, the patient can be transferred for CT evaluation. The adduction of the arms with flexion at the elbows and wrists is abnormal posturing. Specifically, decorticate posturing. This gives a motor score of 3 on the Glasgow coma scale (GCS). Decorticate posturing implies injury to the corticospinal tracts resulting in disinhibition of red nucleus reflexes causing flexion posturing of the upper limbs and extension posturing of the lower limbs. With a patient in a coma and a GCS of 8 or less, it is necessary to intubate and ventilate, thus protecting their airway and breathing. Obtaining a FAST scan to assess for abdominal trauma, initiating blood transfusion for presumed hemorrhagic shock, and initiating seizure prophylaxis are all important steps in this patient's management. However, they are all secondary to securing the airway. Go to the next page if you knew the correct answer, or click the link image(s) below to further research the concepts in this question (if desired).

Research Concepts: Decerebrate And Decorticate Posturing

We update eBooks quarterly and Apps daily based on user feedback. Please tap flag to report any questions that need improvement.

Question 439: A 16-year-old boy is admitted to the intensive care unit due to suspected sepsis. A radial artery catheter is inserted for continuous blood pressure monitoring. The nurse reports that the arterial waveform seems underdamped. Which of the following best describes the expected changes in blood pressure readings due to this issue?

Choices: 1. Systolic blood pressure would be unchanged, diastolic blood pressure lower 2. Systolic blood pressure higher and diastolic blood pressure would be lower 3. Systolic blood pressure lower and diastolic blood pressure would be higher 4. Systolic blood pressure would be higher and diastolic would be unchanged

Photo:Contributed by Divij Pasrija

Answer: 2 - Systolic blood pressure higher and diastolic blood pressure would be lower Explanations: Dampening of the arterial blood pressure waveform implies loss or gain of energy in any part of the circuit used for transduction. This starts from the arterial catheter to the snap tab device and tubing and the transducer. Underdampening implies a gain of energy, and in this case, systolic blood pressure would be higher, whereas diastolic would be slightly lower. Overdampening implies a loss in energy, which results in a lower reading of systolic blood pressure peak, and the diastolic blood pressure is higher. Dampening of the arterial pressure waveform can be fixed by recalibrating/zeroing the circuit, with a heparinized saline flush in case there is a small occlusion, or it might require re-inserting a new catheter in the artery. A gain in energy or excessive whip, which is exaggerated high systolic pressure, can also manifest on arterial line waveform if the tip of the arterial catheter keeps hitting the vessel wall with each transmitted pulsation. To prevent dampening in the circuit, the arterial catheter must be well secured. The tubing should be non-compliant; there should be a constant flow of a heparinized solution through the tubing. Go to the next page if you knew the correct answer, or click the link image(s) below to further research the concepts in this question (if desired).

Research Concepts: Arterial Lines

We update eBooks quarterly and Apps daily based on user feedback. Please tap flag to report any questions that need improvement.

Question 440: A 32-year-old male presents to the ED with right lower quadrant abdominal pain of acute onset and persistent nausea and vomiting. He has no significant medical, surgical, or family history and does not take home medications. During the physical exam, he is significantly uncomfortable with severe rebound tenderness over the right lower quadrant. His vital signs are within normal limits except for a low-grade fever, and a temperature of 39 degrees Celsius. His laboratory results revealed leukocytosis with normal hemoglobin and platelet counts. A CMP and Lipase were within normal ranges. A CT scan of the abdomen with contrast revealed signs consistent with acute appendicitis. A surgeon evaluated the patient and while planning for emergent surgery ordered a routine EKG. A consult was placed to internal medicine for evaluation of the rhythm seen below. What is the following rhythm?

Choices: 1. Respiratory sinus arrhythmia 2. Atrial fibrillation 3. Sinus rhythm 4. Third degree AV block

Photo:Contributed by Michael Soos

Answer: 1 - Respiratory sinus arrhythmia Explanations: Sinus arrhythmia is characterized by an irregular rate with R-R interval variation greater than 0.12 seconds. Additionally, P waves are typically monoform and in a pattern consistent with atrial activation originating from the sinus node. Currently, sinus arrhythmia is thought to be a normal physiologic variation of sinus rhythm found in young healthy individuals; recent studies have revealed a reduction in its prevalence in the elderly. Typically this rhythm does not require additional evaluation or workup by a Cardiologist. Respiratory sinus arrhythmia is a common finding. There are 3 types of sinus arrhythmia, the most common of which is respiratory sinus arrhythmia. In nonrespiratory sinus arrhythmia, electrocardiograms will appear similar to the respiratory type. The two differ in that nonrespiratory sinus arrhythmia is not associated with the respiratory cycle. Go to the next page if you knew the correct answer, or click the link image(s) below to further research the concepts in this question (if desired).

Research Concepts: Sinus Arrhythmia

We update eBooks quarterly and Apps daily based on user feedback. Please tap flag to report any questions that need improvement.

Question 441: A 65-year-old male, suffering from diabetes mellitus type 2 and dyslipidemia, presented to the emergency department with complaints of heavy retrosternal chest pain, diaphoresis, and nausea. His EKG was remarkable for ST elevations in leads V1, V2, and V3. The patient was taken to the coronary cath lab in 20 minutes with successful revascularization of the left anterior descending artery. The patient was admitted to the intensive care unit after the successful PCI. Two hours later, the patient developed this rhythm (see below). However, he was asymptomatic with a blood pressure of 115/75 mmHg, a heart rate of 92/min, a respiratory rate of 16/min, and SPO2 of 97%. What is the next best step in the management of this rhythm?

Choices: 1. Watchful waiting 2. Start amiodarone 3. Cardioversion 4. Activate the cath lab

Photo:Contributed by Abdulrahman S Museedi, MD

Answer: 1 - Watchful waiting Explanations: Accelerated idioventricular rhythm is a very common phenomenon observed on EKG after revascularization. Accelerated idioventricular rhythm is a benign rhythm and the risk to develop ventricular tachycardia is very low. No treatment is required for accelerated idioventricular rhythm other than watching. The heart rate is the key to differentiate the accelerated idioventricular rhythm from ventricular tachycardia (accelerated idioventricular rhythm 100 and ventricular tachycardia > 100). Go to the next page if you knew the correct answer, or click the link image(s) below to further research the concepts in this question (if desired).

Research Concepts: Acute ST Elevation Myocardial Infarction

We update eBooks quarterly and Apps daily based on user feedback. Please tap flag to report any questions that need improvement.

Question 442: When can Physician Orders for Life-Sustaining Treatment (POLST) or Medical Orders for Scope of Treatment (MOST) forms be completed?

Choices: 1. Only when an attorney is present for validation 2. Only when the patient is deemed competent 3. When the patient or surrogate can understand the implications doing or not doing a particular treatment 4. When all of the next of kin agree on the course of treatment

Answer: 3 - When the patient or surrogate can understand the implications doing or not doing a particular treatment

Explanations: Advance directives are instructions given by a patient expressing their wishes about their medical care, including living wills, POLST/POST/MOLST/MOST forms, and Durable Power of Attorney (DPOA) for healthcare. For a living will or DPOA to be valid, the patient must have decision-making capacity (determined by a physician) to sign it, i.e., they can understand the risks, benefits, and implications of doing or not doing a particular treatment; however, the patient does not have to be deemed competent (a legal decision made by a judge). A Living Will is a legal document, and the patient must have the decision-making capacity to complete it. An attorney may be used but is not required in most, if not all, states. The POLST, etc. forms may contain the same directives that are in a Living Will, but they are in the form of medical orders that are actionable by healthcare professionals. The patient OR the surrogate must have the decision-making capacity to complete the POLST or MOST with the healthcare provider. In short, the patient must complete a Living Will; whereas, the patient and/or the surrogate may complete a POLST form. The most desirable time for the execution of an advance directive is when the patient has decision-making capacity while in a stable rather than an urgent or emergent condition. However, if the patient has not completed an advance directive and the patient's medical condition warrants, the family/next-of-kin/surrogate/guardian may have to make the medical decisions and can complete a POLST form. Oral advance directives require only that the patient has made an oral statement regarding their wishes. Validity is strengthened if the patient was clear and specific in their wishes and if the decision was repeated to multiple people over time. Go to the next page if you knew the correct answer, or click the link image(s) below to further research the concepts in this question (if desired).

Research Concepts: Advance Directives

We update eBooks quarterly and Apps daily based on user feedback. Please tap flag to report any questions that need improvement.

Question 443: A 72-year-old male is undergoing percutaneous coronary intervention. Due to concerns for hemodynamic instability, a radial arterial line is placed. During the procedure, the cardiologist is able to measure the pressure in the ascending aorta using the angiography catheter. Compared with aortic pressure, radial artery pressure exhibits which of the following?

Choices: 1. Higher mean pressure 2. Wider pulse pressure 3. Decreased resonance 4. Earlier dicrotic notch

Answer: 2 - Wider pulse pressure Explanations: As one moves away from the aorta, there is an increasing number of branch points in the arterial tree. This results in an increase in pressure wave reflections and greater resonance. The BP distal to the aorta will have a higher SBP and lower DBP due to increased resonance. As a result, there is a wider pulse pressure. The distance from the aorta affects the SBP and DBP but does not affect the mean arterial pressure, so this will be the same at the aorta and the radial artery. Moving further away from the heart causes a wider pulse pressure and greater delay in the dicrotic notch. Go to the next page if you knew the correct answer, or click the link image(s) below to further research the concepts in this question (if desired).

Research Concepts: Arterial Pressure Monitoring

We update eBooks quarterly and Apps daily based on user feedback. Please tap flag to report any questions that need improvement.

Question 444: A 65-year-old female with a history of paroxysmal atrial fibrillation and hypertension presents with an episode of 6 hours duration. The patient denies chest pain or shortness of breath. Medications include lisinopril and aspirin. Her blood pressure is 105/75 mmHg. ECG shows atrial fibrillation with a ventricular response rate of 150 without acute changes. Select appropriate management.

Choices: 1. Intravenous metoprolol followed by oral metoprolol 2. Urgent cardioversion 3. Heparinization and then cardioversion 4. Warfarin

Answer: 1 - Intravenous metoprolol followed by oral metoprolol Explanations: The first step in treatment should be to reduce the ventricular rate with a beta blocker. Emergent cardioversion is only needed if the patient is not stable. As the fibrillation has only been present for 6 hours, anticoagulation before elective cardioversion is not needed if it is done immediately. Anticoagulation should be continued after cardioversion for at least 4 weeks. Go to the next page if you knew the correct answer, or click the link image(s) below to further research the concepts in this question (if desired).

Research Concepts: Atrial Fibrillation

We update eBooks quarterly and Apps daily based on user feedback. Please tap flag to report any questions that need improvement.

Question 445: A 16-year-old asthmatic from India who has been stable on inhaled bronchodilators and corticosteroids has been having frequent exacerbations since last year. His provider added a few more drugs to his prescription and advised a close follow up which he failed to have as they moved to the US afterward. Now he has presented to the emergency department in an agitated state complaining of palpitations, chest tightness, abdominal pain, and feeling of the sinking of the heart. He is extremely short of breath and had one episode of vomiting on his way to the hospital. On examination, there is a restless young male with severe dyspnea, tachycardia, tachypnea, and hypotension. His electrocardiogram reveals supraventricular tachycardia. While taking his blood samples he starts yelling that he can see a wolf howling in front of him. Which of the following is the best treatment plan for him?

Choices: 1. Isotonic saline (20 mL/kg), ondansetron, cardiac cardioversion, and involve ICU 2. Hemodialysis, endotracheal intubation, gastric lavage, and cardiac defibrillation 3. Isotonic saline (20 mL/kg), benzodiazepine, haloperidol, and diphenhydramine 4. Just IV hydration is needed

Answer: 1 - Isotonic saline (20 mL/kg), ondansetron, cardiac cardioversion, and involve ICU Explanations: Around the world, theophylline (1,3-dimethylxanthine) is primarily used as a bronchodilator for patients with asthma and chronic obstructive pulmonary disease (COPD). However, in the United States, asthma and COPD are mainly treated with other agents, and theophylline is largely used to treat bradycardia and apnea in premature newborns. Theophylline causes endogenous release catecholamines through indirect stimulation of beta-1 and beta-2 receptors, which at therapeutic levels cause desired bronchodilation. Unfortunately, theophylline has a narrow therapeutic window, and even levels slightly above this therapeutic window can have many adverse effects in the setting of acute and chronic toxicity. Serum levels between 5-15 mcg/mL of theophylline are considered safe Theophylline has an extremely narrow therapeutic window. Theophylline toxicity occurs when serum theophylline levels surpass the levels in the therapeutic range. This can occur by intentional overdose or unintentionally when metabolism and/or clearance of theophylline is altered due to certain physiological stressors. Theophylline has 2 primary mechanisms. One mechanism is that theophylline blocks adenosine receptors, which has both therapeutic and toxic effects such as bronchodilation, tachycardia, cardiac arrhythmias, seizures, and cerebral vasoconstriction. At larger doses, theophylline inhibits phosphodiesterase causing increased cyclic adenosine monophosphate resulting in increased levels of adrenergic activation and catecholamine release. In theophylline toxicity, epinephrine levels can be 4- to 8-times higher than normal, and norepinephrine concentrations can be 4- to 10-times higher than normal. Increased catecholamine concentrations have a variety of adverse effects such as cardiac arrhythmias, metabolic acidosis, hyperglycemia, and hypokalemia. Most patients with theophylline toxicity are successfully managed with supportive care. Airway, breathing, circulation, and hemodynamic monitoring are essential to the care of patients with theophylline toxicity. Intubation with ventilator support may be required for airway protection. Go to the next page if you knew the correct answer, or click the link image(s) below to further research the concepts in this question (if desired).

Research Concepts: Theophylline Toxicity

We update eBooks quarterly and Apps daily based on user feedback. Please tap flag to report any questions that need improvement.

Question 446: A 43-year-old male is found outdoors, several hours following a traumatic deep venous thrombosis after his left lower extremity was compressed by machinery. Following removal of the machinery, he suffers a subsequent pulmonary embolism, is stabilized in transit to the hospital, and transported to the intensive care unit (ICU). He arrives with the following vital signs: pulse 56/minute, blood pressure 110/67 mmHg, respirations 18/minute, temperature 31 C (87.8 F), and pulse oximetry 89%. Upon completion of arterial blood gas processing, it is determined that his partial pressure of oxygen (PO2) values are much higher than anticipated. Which of the following disease states is most likely to be documented as part of this patient’s history?

Choices: 1. Remote renal cell carcinoma status post nephrectomy 2. Iron deficiency anemia 3. Polycythemia vera 4. Sickle cell anemia

Answer: 3 - Polycythemia vera Explanations: Many algorithms related to blood gas correction do not consider the underlying hematocrit (HCT) of the patient. Patients with higher HCT will have a more robust increase of oxygen partial pressure upon warming from 31 to 37 degrees as there is more oxyhemoglobin to release O2 molecules as P50 (hemoglobin-oxygen affinity indicator) increases. Polycythemia vera and other viscous states also predispose patients to thrombosis. Polycythemia vera is a state of erythrocytosis and will demonstrate a proportionally larger increase of partial pressure of oxygen (PO2) upon warming compared to a person with a normal hematocrit. This effect is magnified compared to anemic states. Sickle cell anemia and malignancy can also increase the likelihood of thrombosis. However, sickle cell anemia patients and patients lacking the erythropoietic potential of a kidney are more likely to suffer from low hematocrit. Go to the next page if you knew the correct answer, or click the link image(s) below to further research the concepts in this question (if desired).

Research Concepts: Blood Gas Temperature Correction

We update eBooks quarterly and Apps daily based on user feedback. Please tap flag to report any questions that need improvement.

Question 447: A 45-year-old female is transferred to the floor from the neurosurgery intensive care unit after having a craniotomy for clipping of a ruptured cerebral aneurysm 5 days ago. At transfer, she was awake, alert, able to follow commands. She moves all extremities equally and has no sensory deficit. This patient should be monitored closely for which of the following complications?

Choices: 1. Secondary hemorrhage 2. Cerebral edema 3. Cerebral infarction 4. Meningitis

Answer: 3 - Cerebral infarction Explanations: Delayed ischemia secondary to vasospasm can happen in up to 1/3 of patients with subarachnoid hemorrhage. Vasospasm can lead to ischemic brain injury (delayed ischemia) as a result of the restricted blood flow caused by vessel constriction. Vasospasm usually occurs 5-10 days after subarachnoid hemorrhage. In a setting of acute onset neurologic deficits in a patient with subarachnoid hemorrhage, transcranial Doppler, or cerebral angiography can be used to diagnose vasospasm. Calcium channel blockers like nimodipine or nicardipine are often used to prevent vasospasm. Go to the next page if you knew the correct answer, or click the link image(s) below to further research the concepts in this question (if desired).

Research Concepts: Cerebral Aneurysm

We update eBooks quarterly and Apps daily based on user feedback. Please tap flag to report any questions that need improvement.

Question 448: A 35-year-old lady is brought to the emergency department complaining of shortness of breath and swelling of legs for the past two weeks. She also complains of occasional dysphagia while taking solid food and has also noticed hoarseness of her voice. A detailed history reveals that she is twenty-four weeks pregnant. Her blood pressure is 90/60 mmHg, pulse 102/min and temperature is 36.9 C (98.42 F). ECG was done which showed atrial fibrillation and a biphasic P wave in leads V1 and V2. A chest x-ray was done which shows straightening of the left heart border. Which one of the following is the most likely finding on physical examination of this patient?

Choices: 1. Pan-systolic murmur 2. Crescendo-decrescendo murmur 3. Diastolic murmur with an opening snap 4. Early, blowing diastolic murmur

Answer: 3 - Diastolic murmur with an opening snap Explanations: The clinical scenario is consistent with the diagnosis of mitral stenosis. Pregnancy is a risk factor for the development of mitral stenosis as it is associated with a 50% increase in the plasma volume. Mitral stenosis presents with some unique features like hoarseness and dysphagia due to the enlarged left atrium pressing on the laryngeal nerve and esophagus respectively. On examination, an opening snap is heard followed by a diastolic rumble. Left atrial hypertrophy shows up as a biphasic P wave in leads V1 and V2. Go to the next page if you knew the correct answer, or click the link image(s) below to further research the concepts in this question (if desired).

Research Concepts: Mitral Stenosis

We update eBooks quarterly and Apps daily based on user feedback. Please tap flag to report any questions that need improvement.

Question 449: A male is examined by a provider for heart problems. The provider reclines the patient and holds his arm up vertically. He then notices and feels a tapping impulse transmitted through the bulk of the muscles. Based on these findings, what condition does this patient have?

Choices: 1. Aortic stenosis 2. Aortic regurgitation 3. Thoracic outlet syndrome 4. Raynaud syndrome

Answer: 2 - Aortic regurgitation Explanations: A water hammer pulse is bounding and very forceful. A water hammer pulse is often seen in aortic regurgitation because of the increased stroke volume of the left ventricle. The wide pulse pressure often results in a large amount of blood being pumped out during each heartbeat. In patients with asymptomatic left ventricular dysfunction, more than 25% develop symptoms in less than one year. Cardiac function usually worsens more rapidly after the appearance of symptoms. Mortality may be more than 10% per year. Symptoms can include shortness of breath, chest pain, and palpitation. Other disorders with water hammer pulse include patent ductus arteriosus (PDA), pregnancy, fever, and anemia. Go to the next page if you knew the correct answer, or click the link image(s) below to further research the concepts in this question (if desired).

Research Concepts: Aortic Regurgitation

We update eBooks quarterly and Apps daily based on user feedback. Please tap flag to report any questions that need improvement.

Question 450: A 65-year-old patient had been intubated and managed in ventilatory support following acute exacerbation of chronic obstructive pulmonary disease (COPD) complicated with severe pneumonia. The patient had already failed two extubation trials previously. Now the patient has good self respiratory efforts with minimal ventilatory support of positive end-expiratory pressure (PEEP) of 8 and fraction of inspired oxygen (FiO2) 0.50. He is hemodynamically stable and has also passed the spontaneous breathing trial (SBT). What is the recommended next step in the management of the patient?

Choices: 1. Extubate the patient 2. Tracheostomy 3. Administration of methylprednisolone just prior to extubation 4. Cuff leak test

Answer: 4 - Cuff leak test Explanations: The patients who have been intubated for greater than 6 days, had failed previous extubation, and with traumatic intubation are at high risk for post-extubation stridor and failed extubation. These patients should ideally undergo a cuff leak test prior to extubation. The cuff leak test is performed by the expired tidal volume following the deflation of the tube cuff. The leak is the difference in the tidal volume of the patient with and without the cuff. A significant leak of air above 110 ml ensures a sufficient cross-sectional area of the airway, thereby minimizing the occurrence of post-extubation stridor in the patient. The patient with a low leak in a cuff leak test should only be considered for undergoing tracheostomy. The methylprednisone is given 4 hours prior to the extubation in patients with a high risk of post-extubation stridor as an adjunct to a positive cuff leak test. Go to the next page if you knew the correct answer, or click the link image(s) below to further research the concepts in this question (if desired).

Research Concepts: Extubation

We update eBooks quarterly and Apps daily based on user feedback. Please tap flag to report any questions that need improvement.

Question 451: A 46-years-old male patient brought to the hospital after sustaining a serious burn from an accidental fire at his home. 30 % of his body is affected. He has been shifted to the intensive care unit. 3 days after, he develops cough, fever, 101 degrees Fahrenheit, chest auscultation reveals crackles on the bilateral lung field. A broad-spectrum beta-lactam antibiotic is planned to be used. Along with that, an aminoglycoside antibiotic is planned, which has no synergistic effect with beta-lactam drugs. What is the peak concentration of this antibiotic that is preferred for the treatment of this hospital-acquired infection?

Choices: 1. 10 to 20 mcg/ml 2. 50 to 60 mcg/ml 3. 20 to 30 mcg/ml 4. 40 to 50 mcg/ml

Answer: 4 - 40 to 50 mcg/ml Explanations: This patient developed hospital-acquired pneumonia. Aminoglycoside is added in this condition to cover gram-negative anaerobes. The target serum concentration for traditional dosing for amikacin is a peak of 20 to 30 mcg/ml and a trough of less than 8 mcg/ml (1 to 4 mcg/ml). Higher peak concentrations of 40 to 50 mcg/ml are recommended for serious infections such as hospital-acquired pneumonia with multi-drug resistant organisms. These higher peaks can be achieved with extended-interval dosing. Infusion time is over an hour. The dose for extended-interval dosing is 15 mg/kg. The frequency of subsequent doses depends on the creatinine clearance. Go to the next page if you knew the correct answer, or click the link image(s) below to further research the concepts in this question (if desired).

Research Concepts: Amikacin

We update eBooks quarterly and Apps daily based on user feedback. Please tap flag to report any questions that need improvement.

Question 452: A 35-year-old man with HIV presents with hemoptysis, dyspnea, and fatigue. The client has not been compliant with his antiretroviral therapy. Scattered crackles are heard bilaterally on chest auscultation. His CD4 count is 130 cells/mm3. A CT scan of the chest shows ground-glass opacities in both lungs. Which of the following is the first-line treatment for this client's suspected condition?

Choices: 1. Trimethoprim-sulfamethoxazole 2. Azithromycin-trimethoprim 3. Penicillin 4. Ceftriaxone-azithromycin

Answer: 1 - Trimethoprim-sulfamethoxazole Explanations: This client has developed pneumocystis jiroveci pneumonia (PCP). Clients presenting with PCP may show signs of fever, cough, dyspnea, and, in severe cases, respiratory failure. Pneumocystis is thought to be transmitted from person to person via an airborne route. The first-line treatment for both HIV-infected and uninfected patients is 21 days of trimethoprim-sulfamethoxazole (TMP-SMX). For mild to moderate disease, give TMP 15 to 20 mg/kg/day and SMX 75 to 100 mg/kg/day orally in 3 divided doses or TMP-SMX double strength (DS) 2 tablets 3 times per day. Typically, clients at risk are those with any underlying disease state that alters host immunity such as those with cancer, HIV, transplant recipients, or those being treated with immunosuppressive medications. Go to the next page if you knew the correct answer, or click the link image(s) below to further research the concepts in this question (if desired).

Research Concepts: Prevention Of Opportunistic Infections In HIV/AIDS

We update eBooks quarterly and Apps daily based on user feedback. Please tap flag to report any questions that need improvement.

Question 453: Pneumonectomy is considered a high-risk procedure associated with significant morbidity and mortality. After a pneumonectomy, which of these complications is the most common cause of death?

Choices: 1. Infection 2. Anastomotic dehiscence 3. Non-cardiogenic pulmonary edema 4. Arrhythmia

Answer: 3 - Non-cardiogenic pulmonary edema Explanations: Non-cardiogenic pulmonary edema is infrequent but one of the most serious complications after pneumonectomy. It usually develops in 4% of patients undergoing major resections of lung parenchyma. Pathophysiology of postpneumonectomy pulmonary edema is incompletely understood. One of the proposed mechanisms is increased filtration gradient across pulmonary capillaries together with increased permeability. It presents soon after surgery. The chest x-ray may show fluffy infiltrates and may be confused with acute respiratory distress syndrome. Treatment consists of restriction of fluid intake, diuretics, and respiratory support to maintain adequate oxygenation. Go to the next page if you knew the correct answer, or click the link image(s) below to further research the concepts in this question (if desired).

Research Concepts: Pneumonectomy

We update eBooks quarterly and Apps daily based on user feedback. Please tap flag to report any questions that need improvement.

Question 454: A 17-year-old male is brought in by ambulance to the emergency department. He was found unconscious at a dance nightclub. Paramedics report he has a Glasgow Coma Scale of 10. Vital signs include a temperature of 36.3 degrees Celsius, blood pressure of 110/80 mmHg, heart rate of 48 beats per minute, respiratory rate of 6 breaths per minute, and oxygen saturation of 95% on room air. Fingerstick glucose was 97 mg/dl. The patient is intubated and abruptly awakens 2 hours later. He extubates himself and demands to go home. Which of the following drugs is most likely to be given to this patient?

Choices: 1. Naloxone 2. Atropine 3. Pralidoxime 4. Diazepam

Answer: 2 - Atropine Explanations: The patient in this clinical scenario has most likely developed gamma-hydroxybutyrate (GHB) overdose. The hallmark of gamma-hydroxybutyrate (GHB) overdose is an abrupt onset of coma followed later by an abrupt resolution of symptoms. Signs and symptoms of GHB toxicity include CNS depression, respiratory depression, and bradycardia. Mild hypothermia can also be seen. There is no antidote for GHB toxicity. The mainstay of treatment is airway protection and monitoring. Management should also include cardiorespiratory monitoring, pulse oximetry, and capnography if available. Patients may develop severe respiratory depression or apnea, and therefore immediate evaluation of the airway is paramount. In milder cases, supplemental oxygen with or without a nasopharyngeal airway is sufficient until the patient awakens. In more severe cases, endotracheal intubation may be necessary. GHB-intoxicated patients usually do not require any sedation while mechanically ventilated and will precipitously awaken and potentially extubate themselves or require immediate extubation. Severe bradycardia can be treated with atropine, and hypotension is often sufficiently managed with intravenous (IV) fluids. If there is any concern for opioid coingestion and toxicity, IV naloxone should be strongly considered. Patients can often be safely discharged home once they are awake, symptom-free, and all other co-intoxications or injuries are ruled out. Go to the next page if you knew the correct answer, or click the link image(s) below to further research the concepts in this question (if desired).

Research Concepts: Gamma-Hydroxybutyrate Toxicity

We update eBooks quarterly and Apps daily based on user feedback. Please tap flag to report any questions that need improvement.

Question 455: A 29-year-old male with a history of asthma and prior intubation presents with acute severe episodes of shortness of breath. The exam shows a silent chest. An arterial blood gas is done showing a PaO2 of 50, SpO2 of 80%, and a PaCO2 of 68 with a pH of 7.05. The patient is immediately intubated and admitted to the ICU with the following ventilator settings: tidal volume 400ml, respiratory rate 16, FiO2 60%, and positive end-expiratory pressure (peep) 12. Repeat arterial blood gas (ABG) one hour after intubation shows PaO2 of 73, SpO2 94%, and a PaCO2 of 60 with a pH of 7.13. What is the next best step to take on managing this patient's ventilator?

Choices: 1. Increase the respiratory rate 2. Decrease the FiO2 3. Decrease the tidal volume 4. Increase the inspiratory time

Answer: 1 - Increase the respiratory rate Explanations: The current patient is presenting with acute asthma exacerbation and severe hypercapnic and hypoxic respiratory failure. The appropriate management was done by intubating the patient as he is currently experiencing respiratory collapse with severe acidosis. After intubation, the patient still has severe hypercapnia and acidosis, with now resolved hypoxemia. To improve hypercapnia and respiratory acidosis, the best option is to increase minute ventilation. (MV) = RR x TV. Increasing Respiratory rate or tidal volume would increase minute ventilation and decrease PCO2, improving hypercapnia and acidosis. Our ability to increase Tidal volume will be limited by respiratory mechanics, and increasing this parameter would increase plateau pressures. Increasing Respiratory rate will also achieve decreasing CO2 and improving acidosis. Our ability to increase RR is limited by the expiratory time. Increasing RR, too, will reduce the expiratory time. If the expiratory time is made too short, we risk the patient experiencing auto-PEEP and increasing plateau pressures. Changing FiO2 will only affect oxygenation and no ventilation. Changing inspiratory times can help us use higher RR while preventing auto-PEEP. DECREASING (not increasing) inspiratory time will let us improve RR while permitting longer expiratory times and that way, preventing auto-PEEP. Go to the next page if you knew the correct answer, or click the link image(s) below to further research the concepts in this question (if desired).

Research Concepts: Ventilator Management

We update eBooks quarterly and Apps daily based on user feedback. Please tap flag to report any questions that need improvement.

Question 456: A 45-year-old man is brought to the emergency with lethargy, confusion, abdominal distension, and hematemesis. He migrated from India 2 years ago. He has a 7-year history of hepatitis C infection. His abdominal distention improves with furosemide. Esophagogastroduodenoscopy reveals medium esophageal varices on the distal esophagus. Which of the following is the next best step in the management of this patient?

Choices: 1. Lisinopril 2. Nadolol 3. Transjugular intrahepatic portosystemic shunt (TIPS) 4. Ciprofloxacin

Answer: 2 - Nadolol Explanations: Patients with a medium to large esophageal varices should receive primary prophylaxis to prevent bleeding, which causes hematemesis and is also cause of hepatic encephalopathy. Primary prophylaxis of varices is achieved by nonselective beta-blockers or endoscopic variceal ligation. The choice of beta-blocker or ligation depends on patient preference and size of varices. Ligation is preferred for large varices. Nonelective beta-blocker reduce portal pressure by blocking the adrenergic vasodilatory response of mesenteric arterioles, which result in unopposed alpha-adrenergic vasoconstriction and thus reduced portal blood flow. ACE inhibitors have no role in the management of cirrhosis. They are the mainstay of hypertension, heart failure, and chronic kidney diseases. TIPS is performed when a patient does not respond to diuretics and has recurrent esophageal varices. TIPS causes hepatic encephalopathy in 35% of the patients. Go to the next page if you knew the correct answer, or click the link image(s) below to further research the concepts in this question (if desired).

Research Concepts: Portal-Systemic Encephalopathy

We update eBooks quarterly and Apps daily based on user feedback. Please tap flag to report any questions that need improvement.

Question 457: A patient suddenly collapsed in the ICU. The patient had just had open-heart surgery two days ago. The nurse had just put the temperature probe in his mouth when he arrested. Blood pressure is unrecordable and carotids are not palpable. CPR is started. While the anesthesiologist is attempting to intubate the patient, it is decided to administer epinephrine. What is the correct dose of the drug in this scenario?

Choices: 1. 1 mg IV push every 3 - 5 minutes 2. 0.1 mg IV push every 3 - 5 minutes 3. 0.1 mg/kg IV push every 3 - 5 minutes 4. 10 mg in 10 cc normal saline down the endotracheal tube

Answer: 1 - 1 mg IV push every 3 - 5 minutes Explanations: The standard dose of epinephrine in cardiac arrest is 1 mg every 3-5 minutes. Higher doses are not recommended and may even be harmful. Addition of vasopressin to epinephrine does not offer an advantage. Most experts recommend administering epinephrine ASAP after the cardiac arrest so that the heart becomes responsive to the defibrillator. Go to the next page if you knew the correct answer, or click the link image(s) below to further research the concepts in this question (if desired).

Research Concepts: Epinephrine

We update eBooks quarterly and Apps daily based on user feedback. Please tap flag to report any questions that need improvement.

Question 458: A 65-year-old male patient is brought to the emergency department by his family after an episode of loss of consciousness. The patient did not experience any prodromal features, generalized body movements, involuntary loss of urine, or tongue bite during the episode. He mentions that he had an upper respiratory tract infection a week ago that resolved spontaneously. The patient has a history of hypertension and gastroesophageal reflux disease, for which he takes amlodipine and famotidine. Vitals show a blood pressure of 122/72 mm Hg, which drops to 104/68 mmHg on inspiration, pulse of 64/min, respiratory rate of 15/min, and temperature of 98.6 F (37 C). Physical examination reveals distended neck veins and distant heart sounds. The lungs are clear to auscultation bilaterally. What is the most likely diagnosis?

Choices: 1. Cardiac tamponade 2. Constrictive pericarditis 3. Pleural effusion 4. Pulmonary embolism

Answer: 1 - Cardiac tamponade Explanations: The most likely diagnosis is cardiac tamponade. The patient’s history of an upper respiratory tract infection, current presentation, distant heart sounds, and distended neck veins on examination indicate the diagnosis. Pulsus paradoxus is frequently seen in cardiac tamponade. Pulsus paradoxus is defined as a drop in blood pressure by more than 10 mmHg on inspiration. In cardiac tamponade, there is decreased ventricular compliance, and the increased venous return during inspiration results in the bowing of intraventricular septum towards the left, decreasing stroke volume. It can be seen in constrictive pericarditis, pulmonary embolism, and bilateral pleural effusion but this patient is more likely to have developed cardiac tamponade. Go to the next page if you knew the correct answer, or click the link image(s) below to further research the concepts in this question (if desired).

Research Concepts: Pulsus Paradoxus

We update eBooks quarterly and Apps daily based on user feedback. Please tap flag to report any questions that need improvement.

Question 459: A 55-year-old man is admitted to the intensive care unit with gram-negative sepsis. His clinical condition deteriorates over the next few days. He ultimately develops refractory hypotension and multi-organ failure and dies despite aggressive treatment. Autopsy reveals bilateral enlarged and hemorrhagic suprarenal structures. Histopathological examination shows hyperplasia along with congestion and extensive hemorrhage. Which of the following factors is most likely responsible for the development of hemorrhage in this structure?

Choices: 1. Anatomical location 2. Vascular arrangement 3. Drug toxicity 4. Age-related degeneration

Answer: 2 - Vascular arrangement Explanations: This patient has died secondary to septic shock. The organ described in the autopsy is the adrenal gland. The adrenals are paired endocrine organs located in the retroperitoneum above the kidneys. They have two anatomically and functionally distinct parts each with its unique embryological origin, the outer cortex and the inner medulla. The autopsy findings are consistent with the development of adrenal hemorrhage. Adrenal hemorrhage is an unusual and often underdiagnosed disease characterized by bleeding into the adrenals. Septicemia and critical illness are important causes. The adrenal gland is one of the most well-perfused organs of the body. It has a rich and extensive arterial supply. Usually, 3 sets of arteries, the superior, middle, and inferior adrenal artery which are generally derived from the inferior phrenic, the abdominal aorta and the renal arteries respectively, contribute to the arterial supply. There is a rich subcapsular plexus which drains into medullary sinusoids, the outflow through venules is limited. This pattern of vascular supply has been described as a 'vascular dam.' Furthermore, the outflow is via a single adrenal vein. This unique vascular arrangement can lead to adrenal congestion and subsequent hemorrhagic transformation. In times of stress, there is trophic stimulation of adrenals due to ACTH release from the anterior pituitary. This leads to hyperplasia of the gland and increased vascularity. This increased vascular congestion, together with vascular predisposition, contributes to adrenal hemorrhage. The adrenals are responsible for producing steroid hormones and catecholamines. Their anatomic location and aging do not contribute significantly to hemorrhage. Drug toxicity also is unlikely to contribute to the autopsy findings. Go to the next page if you knew the correct answer, or click the link image(s) below to further research the concepts in this question (if desired).

Research Concepts: Adrenal Hemorrhage

We update eBooks quarterly and Apps daily based on user feedback. Please tap flag to report any questions that need improvement.

Question 460: A patient was admitted after a fall where she hit her upper back and neck. On examination, there are no cranial nerve deficits. The motor examination shows 4/5 in elbow extension and wrist extension of both upper extremities and 3/5 on both lower extremities. There was decreased sensation below the clavicle and the last two digits of both hands. She was diagnosed with a spinal cord injury. Several weeks later, she is receiving therapy when she develops a severe headache, diaphoresis, and hypertension. What is the best treatment for this condition?

Choices: 1. Nitroprusside 2. Remove the offending stimulus 3. Opioid analgesia 4. Restrict fluids

Answer: 2 - Remove the offending stimulus Explanations: Patients with SCI above T6 are at risk for autonomic dysreflexia. A minor noxious stimulus causes a sympathetic discharge causing severe hypertension. The patient should be placed in an upright position. Medications used to treat this are rapid short-acting nitrates (nitroglycerin or nifedipine). Go to the next page if you knew the correct answer, or click the link image(s) below to further research the concepts in this question (if desired).

Research Concepts: Autonomic Dysreflexia

We update eBooks quarterly and Apps daily based on user feedback. Please tap flag to report any questions that need improvement.

Question 461: A 16-year-old patient sustained a severe head injury following a motor vehicle collision. His intracranial pressure (ICP) has been monitored by the placement of an external ventricular drain (EVD) placement. Despite keeping the patient intubated and sedated in mechanical ventilation, his ICP is persistently above 20 mm Hg. His serum electrolytes were normal, and serum osmolarity was 350 mosmol/kg. The treating clinician then plans to start medical therapy for managing his refractory cerebral edema. Which of the following is the most rational approach in managing the patient?

Choices: 1. Hypertonic saline 2. Mannitol 3. Urea 4. Glycerol

Answer: 1 - Hypertonic saline Explanations: The use of hypertonic saline in the management of intracranial hypertension has shown to be of rapid onset, sustained as well as long-lasting effects with collateral improvement in cerebral perfusion as well. 3% hypertonic saline with a loading dose of 5 ml/kg and a maintenance dose of 2 ml/kg every six hourly has shown to be highly efficacious as well as safe in managing refractory intracranial hypertension. The occurrence of side effects such as central pontine myelinolysis and acute tubular necrosis with hypertonic saline is minimal in patients with serum osmolality of above 320 mosmol/kg and normal serum sodium values. Urea and glycerol have low efficacy in managing cerebral edema. Mannitol use leads to a high occurrence of rebound cerebral edema, renal failure, and electrolyte imbalance. Both hypertonic saline, as well as mannitol have comparable all-cause mortality rates. Go to the next page if you knew the correct answer, or click the link image(s) below to further research the concepts in this question (if desired).

Research Concepts: Intracranial Hypertension

We update eBooks quarterly and Apps daily based on user feedback. Please tap flag to report any questions that need improvement.

Question 462: A 65-year-old female is admitted to the hospital for a non-ST-segment elevation myocardial infarction. During the initial assessment by the night shift, she appears pale and diaphoretic. She reports shortness of breath and dizziness. A bedside cardiac monitor reveals monomorphic wide-complex ventricular tachycardia (VT). Before further evaluation, the patient becomes unresponsive and becomes pulseless. The cardiac monitor continues to show VT. Cardiopulmonary resuscitation (CPR) and subsequent biphasic defibrillation with 200 joules is given. After 2 minutes of CPR, the patient remains pulseless, and the rhythm now shows ventricular fibrillation (VF). She is defibrillated again with 300 joules, and 1 mg of epinephrine is given intravenously with continued CPR. After 2 minutes, the patient remains in VF. Which of the following is the next best step in the management of this patient?

Choices: 1. Continue CPR, defibrillate with 360 joules and give 150 mg of amiodarone intravenously 2. Continue CPR, defibrillate with 300 joules and give 300 mg of amiodarone intravenously 3. Continue CPR, defibrillate with 360 joules and give 1 mg/kg of norepinephrine intravenously 4. Continue CPR, defibrillate with 200 joules and give 1 mg of epinephrine intravenously

Answer: 2 - Continue CPR, defibrillate with 300 joules and give 300 mg of amiodarone intravenously

Explanations: During cardiopulmonary resuscitation for VF or VT, each successive defibrillation should be of equal or greater value until the maximum dose is achieved. After the first unsuccessful defibrillation, CPR should be continued with a second defibrillation and 1 mg of epinephrine bolus intravenously. The CPR should be continued for 2 minutes after the epinephrine and the shock has been delivered. If the patient remains in VT or VF at the next rhythm and pulse check, CPR should be continued with another dose of defibrillation. At this time acceptable antiarrhythmic agents include 1 mg of epinephrine or 300 mg of amiodarone bolus. Lidocaine is no longer preferred for cardiopulmonary resuscitation of VT or VF. Go to the next page if you knew the correct answer, or click the link image(s) below to further research the concepts in this question (if desired).

Research Concepts: Defibrillation

We update eBooks quarterly and Apps daily based on user feedback. Please tap flag to report any questions that need improvement.

Question 463: A 65-year-old woman presents to the hospital with two days of fever, headache, and sore throat. She was seen at an urgent care center the day before and was given azithromycin without relief. Her vital signs reveal blood pressure 88/50 mmHg, temperature 103 F (39.4 C), pulse 115 beats/min, and SpO2 97% on room air. She appears lethargic. Her pharynx is normal and she is swallowing and speaking normally. Lungs are clear to auscultation. She has one cold sore on her lip. The abdomen is soft and non-tender. What is the next best step in the management of this patient?

Choices: 1. IV ceftriaxone, vancomycin, and acyclovir 2. CT head 3. Lumbar puncture 4. Acetaminophen

Answer: 1 - IV ceftriaxone, vancomycin, and acyclovir Explanations: This patient most likely has meningitis. The definitive test of choice is a lumbar puncture to send CSF for necessary analysis. If the patient is hemodynamically unstable, it is appropriate to start treatment immediately before lumbar puncture is performed. Cold sores may indicate that the patient may have a herpes simplex virus. Herpes simplex virus can cause aseptic meningitis, which should be treated with acyclovir. Go to the next page if you knew the correct answer, or click the link image(s) below to further research the concepts in this question (if desired).

Research Concepts: Aseptic Meningitis

We update eBooks quarterly and Apps daily based on user feedback. Please tap flag to report any questions that need improvement.

Question 464: An 18-year-old male presents with a long history of easy bruising and recurrent nosebleeds, for which he has required multiple admissions. He has a family history of nosebleeds. Now he has been complaining of a headache, gait difficulty with weakness, paresthesias, confusion, and nausea. Examination reveals that he has numerous telangiectasias over his face and lips. MRI reveals that he has a ring-enhancing lesion in the brain. What is the cause of the brain lesion?

Choices: 1. Bleeding in the brain 2. AVMs in the lung 3. Fragile blood-brain barrier 4. Use of IV drugs

Answer: 2 - AVMs in the lung Explanations: Patients with hereditary hemorrhagic telangiectasia develop arteriovenous malformations of the liver, lungs, brain, and spine. The pulmonary shunts commonly associated with this disorder put patients at significant risk for the development of brain abscesses. When there are AVMs in the lung, the normal filtering ability of the pulmonary circulation is lost and septic emboli go to the brain. These patients also experience paradoxical cerebral emboli, strokes, and subarachnoid hemorrhage, due to an increased risk of intracranial aneurysms. Brain abscess will present as a ring-enhancing lesion with symptoms of weakness, seizures, headache and even paraplegia. Go to the next page if you knew the correct answer, or click the link image(s) below to further research the concepts in this question (if desired).

Research Concepts: Osler-Weber-Rendu Disease

We update eBooks quarterly and Apps daily based on user feedback. Please tap flag to report any questions that need improvement.

Question 465: A 69-year-old man is brought to the emergency department with chest pain, diaphoresis, and dyspnea. An acute ST-elevation myocardial infarction is diagnosed. He had no prior cardiovascular symptoms. Past history is significant for hypertension and diabetes mellitus. On the fifth day of the hospitalization, the patient develops shock with a blood pressure of 60/40 mg and a pulse of 118/ min. Left ventricular rupture is suspected, and echocardiography is being done for evaluation. Which of the following is the best determinant of mortality in this patient?

Choices: 1. Early diagnosis and surgical intervention 2. The time period from acute myocardial infarction and the onset of symptoms 3. The severity of clinical symptoms 4. Etiology of the rupture

Answer: 1 - Early diagnosis and surgical intervention Explanations: Type and size of the injury, rapidity of transfer to the hospital, patient's hemodynamic stability, and the time till intervention play an important role in in-hospital mortality. Early detection of symptoms and signs is the key to decreasing mortality, especially in the acute critical condition of myocardial rupture. Early diagnosis and immediate surgical intervention are critical as the mortality is extremely high without intervention. The severity of symptoms, onset of symptoms, or etiology does not affect mortality. It depends on the time of initiation of the intervention. Go to the next page if you knew the correct answer, or click the link image(s) below to further research the concepts in this question (if desired).

Research Concepts: Left Ventricular Rupture

We update eBooks quarterly and Apps daily based on user feedback. Please tap flag to report any questions that need improvement.

Question 466: A 16-year-old boy with newly diagnosed Wolff-Parkinson-White syndrome is admitted to the hospital for laparoscopic appendectomy. He is not currently taking any medications and is otherwise healthy. Upon insufflation of the abdomen, he develops a wide complex tachycardia with a heart rate of 170/min. His blood pressure is 110/70 mmHg. Which of the following is the next best step in the management of this patient?

Choices: 1. Adenosine 2. Procainamide 3. Esmolol 4. Verapamil

Answer: 2 - Procainamide Explanations: Wolff-Parkinson-White (WPW) is a syndrome in which the heart contains an anomalous bundle of conducting tissue called an accessory pathway (AP), which bypasses the normal AV nodal conduction system. A tachyarrhythmia resulting from forming a circuit between the AV conducting system and the AP is called an atrioventricular reciprocating tachycardia (AVRT). There are two types of AVRTs: orthodromic and antidromic. In the setting of WPW with wide complex tachycardia, agents such as procainamide are preferred since they will slow conduction through the accessory pathway. Administration of AV node blocking agents in antidromic AVRT can increase conduction through the AP and potentially lead to ventricular fibrillation. Orthodromic AVRT consists of antegrade conduction through the normal AV conducting system and retrograde conduction through the AP. Consequently, the tachycardia will have a narrow QRS and can be treated with most agents that slow conduction through the AV node (e.g., adenosine, verapamil, esmolol). Go to the next page if you knew the correct answer, or click the link image(s) below to further research the concepts in this question (if desired).

Research Concepts: Wolff Parkinson White Syndrome

We update eBooks quarterly and Apps daily based on user feedback. Please tap flag to report any questions that need improvement.

Question 467: A 20-year-old man is brought to the emergency department with a history of being found falling suddenly unconscious on the roadside. On examination, his face and tongue are swollen. He is drowsy, and his pulse rate is 112/ minute, and his blood pressure is 82/56 mmHg. His blood investigations reveal elevated levels of angiotensin II. Which of the following enzymes present in the patient's body might have caused elevated levels of this chemical?

Choices: 1. Tryptase 2. Chymase 3. Trehalase 4. Dihydropeptidase

Answer: 2 - Chymase Explanations: The presence of facial swelling and hypotension suggests the diagnosis of an anaphylactic reaction. Elevated levels of angiotensin II may be seen following anaphylaxis. Mast cell degranulation releases chymase. Chymase facilitates the conversion of angiotensin I to angiotensin II. Go to the next page if you knew the correct answer, or click the link image(s) below to further research the concepts in this question (if desired).

Research Concepts: Anaphylaxis

We update eBooks quarterly and Apps daily based on user feedback. Please tap flag to report any questions that need improvement.

Question 468: An elderly homeless man presents with altered mental status, seizures, and vomiting. Exam reveals multiple dental caries, and focal neurological deficit, with no fever, head trauma, or neck stiffness. What should the work up include next?

Choices: 1. Lumbar puncture 2. Non-contrast head CT 3. Contrast-enhanced head CT 4. Placement of ICP monitor

Answer: 3 - Contrast-enhanced head CT Explanations: Brain abscess is one consideration and presents sometimes without fever, elevated WBC, and signs of meningeal irritation. Brain abscesses develop by either contiguous spread or hematogenous spread. Common sites of infection include sinuses, mouth/teeth, and ears. Contrast-enhanced CT and MRI reveal a ring-enhancing lesion usually at the gray-white interface with surrounding edema. Treatment involves identification of organism through relief of mass effect with aspiration vs open excision followed by IV antibiotics. Brain abscess today is more common in the immunocompromised, including HIV patients. Go to the next page if you knew the correct answer, or click the link image(s) below to further research the concepts in this question (if desired).

Research Concepts: Brain Abscess

We update eBooks quarterly and Apps daily based on user feedback. Please tap flag to report any questions that need improvement.

Question 469: A 92-year-old man is brought to the emergency department at 2:35 a.m. from his nursing home with pronounced confusion. He has dilated pupils, poorly responsive to light. He was recently prescribed a new medication to treat vomiting. His blood pressure is 134/78 mmHg, and his temperature is 102.9 F (39.4 C). What is the best initial step in the management of this patient?

Choices: 1. Administer doxylamine 2. Draw blood cultures 3. Begin constant cardiac monitoring 4. Order a chest MRI

Answer: 3 - Begin constant cardiac monitoring Explanations: Doxylamine may cause arrhythmias in toxic doses. Doxylamine may cause respiratory failure in toxic doses. In toxic doses, doxylamine may cause arrhythmias, respiratory failure, or hyperthermia that may lead to cardiac dysfunction. Doxylamine toxicity is more common in the young and elderly populations. Doxylamine may cause hyperthermia in toxic doses, which could result in heat-induced cardiac dysfunction. Go to the next page if you knew the correct answer, or click the link image(s) below to further research the concepts in this question (if desired).

Research Concepts: Doxylamine

We update eBooks quarterly and Apps daily based on user feedback. Please tap flag to report any questions that need improvement.

Question 470: A 66-year-old male patient presents in septic shock and does not respond to appropriate fluid resuscitation. The patient requires a central venous line (CVL) placement. However, the patient is taking warfarin for atrial fibrillation and his INR is highly deranged i.e. 7.6. Which of the following measures need to be taken immediately?

Choices: 1. Administer vitamin K through the IV, continue IV hydration through the peripheral lines, place a central line in the femoral vein as this is the most compressible site 2. Administer PO vitamin K, order FFP to be given in the ICU, continue IV hydration, call the intensivist and let them know that they need to call surgery to place the line once INR is corrected 3. Administer IV vitamin K and IV FFP. Consider peripheral vasopressor support while administering vitamin K and FFP. Place a central line in the femoral or internal jugular vein using ultrasound to help visualize vessels. 4. Administer IV vitamin K and IV FFP. Consider peripheral vasopressor support while administering vitamin K and FFP. Place a subclavian line slowly and controlled with the patient in the Trendelenburg position to best dilate the subclavian vein making it easier to cannulate in the first attempt.

Answer: 3 - Administer IV vitamin K and IV FFP. Consider peripheral vasopressor support while administering vitamin K and FFP. Place a central line in the femoral or internal jugular vein using ultrasound to help visualize vessels.

Explanations: A compressible site is preferred, however vitamin K alone will not reverse fast enough. This patient requires vasopressor support as fluids alone are not enough for resuscitation. This patient needs access and pressor support as he did not respond to fluid resuscitation. The best way is to rapidly reverse the coagulopathy, discuss options with family, and start peripheral pressor support while correcting coagulopathy. Then place a line in a compressible site with ultrasound to limit the number of attempts needed A patient who is hypotensive but has not received any fluid via a peripheral line should be resuscitated first before deciding to place a CVL and starting them on vasopressors. The subclavian approach is contraindicated. There are no absolute indications for a central venous catheter (CVC). Generally, patients requiring vasoactive medications will need one due to the potential for vein sclerosis and ischemia through peripheral IVs. Patients who require multiple medications and continued resuscitation, have inadequate peripheral access, are critically ill, or require multiple phlebosclerotic medications also candidates for central line placement. Go to the next page if you knew the correct answer, or click the link image(s) below to further research the concepts in this question (if desired).

Research Concepts: Central Line

We update eBooks quarterly and Apps daily based on user feedback. Please tap flag to report any questions that need improvement.

Question 471: A 67-year-old female is receiving mechanical ventilation in the intensive care unit for severe acute respiratory distress syndrome (ARDS) secondary to a motor vehicle accident. The ventilator mode is volume assist control, and the tidal volume is set at 500 mL. Which of the following is a significant disadvantage of volume assist control for patients with ARDS?

Choices: 1. Inability to deliver a constant tidal volume 2. Increased risk for elevated plateau pressures and pulmonary barotrauma 3. Lack of adequate alarms 4. Inability to provide proper oxygenation

Answer: 2 - Increased risk for elevated plateau pressures and pulmonary barotrauma Explanations: Volume assist control is a volume-cycled mode of ventilation. It delivers a fixed amount of tidal volume on every breath provided by the ventilator. Since the ventilator will always deliver the same amount of tidal volume, the peak and plateau pressures generated by the system will depend on lung compliance. Stiffer lungs, such as those in patients with ARDS, will generate higher peak and plateau pressures to deliver the same amount of tidal volume when compared to healthy lungs. Increased plateau pressure, higher than 35 cmH2O, has been associated with an increased risk of pulmonary barotrauma. Go to the next page if you knew the correct answer, or click the link image(s) below to further research the concepts in this question (if desired).

Research Concepts: Barotrauma And Mechanical Ventilation

We update eBooks quarterly and Apps daily based on user feedback. Please tap flag to report any questions that need improvement.

Question 472: A 40-year-old man was involved in a motor vehicle accident. The patient was an unrestrained driver when he lost control of his car and hit a tree head-on. Airbags deployed. The patient was awake when EMS first arrived, and he is brought to the hospital. The ATLS algorithm is followed, and the patient is stabilized. Non-contrast CT of the maxillofacial bones reveals a LeForte 2 fracture with the involvement of the right maxillary sinus. Soon afterward, there is an acute change in the patient's clinical status. The patient opens eyes to pain, is making incomprehensible sounds, and has a decorticate posture. Which of the following is the next best step in the management of this patient?

Choices: 1. Nasotracheal intubation 2. Nonrebreather mask 3. Non-contrast head CT 4. Tracheostomy

Answer: 4 - Tracheostomy Explanations: This patient has a GCS of 7/15 and needs to have their airway secured. Given the patient's unstable facial fractures, a tracheostomy is a good choice in securing the airway. This patient should not undergo nasotracheal intubation given the extent of his facial injuries. Though this patient may eventually need further imaging, it is more important to secure his airway at this time. Go to the next page if you knew the correct answer, or click the link image(s) below to further research the concepts in this question (if desired).

Research Concepts: Maxillary Sinus Fracture

We update eBooks quarterly and Apps daily based on user feedback. Please tap flag to report any questions that need improvement.

Question 473: A 65-year-old man is admitted to the intensive care unit after coronary artery bypass surgery. The respiratory therapist has set the ventilator to synchronized intermittent mandatory ventilation (SIMV) mode, rate of 12, a fraction of inspired oxygen (FiO2) of 50% and tidal volume of 550 milliliters. The patient's respiratory rate reads 22 breaths per minute. Which of the following can be expected with this ventilator mode?

Choices: 1. Increased work of breathing 2. Decreased work of breathing 3. Tidal volume of 550 with spontaneous breaths 4. Evidence of breath stacking

Answer: 1 - Increased work of breathing Explanations: Synchronized intermittent mandatory ventilation (SIMV) has been shown to increase work of breathing in ventilated patients. The addition of pressure support may decrease the work of breathing. The ventilator will synchronize the delivery of mandatory and spontaneous breaths in SIMV to avoid breath stacking. Volumes generated during spontaneous breaths in SIMV mode are 100% patient effort driven. The tidal volumes will vary by breath. Go to the next page if you knew the correct answer, or click the link image(s) below to further research the concepts in this question (if desired).

Research Concepts: Synchronized Intermittent Mandatory Ventilation

We update eBooks quarterly and Apps daily based on user feedback. Please tap flag to report any questions that need improvement.

Question 474: A 67-year-old man with a past medical history of chronic hepatitis C, diabetes mellitus, and hypertension presents to the emergency department with hematemesis and altered mental status. On examination, he is somnolent, has asterixis, ascites, and palmar telangiectasias. His hematemesis is uncontrollable with standard treatment and supportive therapy. Which of the following is the strongest contraindication to the use of molecular adsorbent recirculating system (MARS) in this patient?

Choices: 1. Uncontrolled hemorrhage 2. Hepatic encephalopathy 3. Chronic hepatitis C 4. Uncontrolled sepsis

Answer: 1 - Uncontrolled hemorrhage Explanations: Uncontrolled haemorrhage prohibits the use of MARS and is considered as a relative contraindication. MARS has a very good safety profile and is tolerated well by the patients. Thrombocytopenia has been documented during the therapy, but it is mild and pose no significant threat to the patient. The risks associated with MARS are the same as the ones with conventional haemodialysis and requires the use of anticoagulants to prevent coagulation activation during the procedure. MARS is relatively contraindicated if there is increased risk of coagulopathy (example: disseminated intravascular coagulation) i.e. when platelets are below 50,000/microlitre of blood or when the INR is greater than 2.3. Uncontrolled sepsis also prohibits the use of MARS but is not present in this patient. Go to the next page if you knew the correct answer, or click the link image(s) below to further research the concepts in this question (if desired).

Research Concepts: Molecular Absorbent Recirculating System

We update eBooks quarterly and Apps daily based on user feedback. Please tap flag to report any questions that need improvement.

Question 475: A 26-year-old man presents with a 2-month history of generalized fatigue, night sweats, and 3 kg unintentional weight loss. Following further investigation, he is diagnosed with stage IV Hodgkin lymphoma. The patient is started on a chemotherapy regimen that includes a cytotoxic drug belonging to a family of glycopeptide antibiotics that disrupts DNA synthesis. Which of the following tests is most important to perform regularly in this patient whilst he is taking this drug?

Choices: 1. Lumbar puncture 2. Echocardiogram 3. Chest x-ray 4. Abdominal ultrasound

Answer: 3 - Chest x-ray Explanations: Bleomycin is an antibiotic and antineoplastic agent. Bleomycin's most concerning adverse effect is pulmonary fibrosis. It is advisable to perform chest x-rays at regular intervals, therefore, while a patient is on therapy with bleomycin. Less severe reactions to bleomycin include skin pigmentation changes, itching, hypogeusia, rash, nausea, vomiting, and weight loss. Go to the next page if you knew the correct answer, or click the link image(s) below to further research the concepts in this question (if desired).

Research Concepts: Bleomycin

We update eBooks quarterly and Apps daily based on user feedback. Please tap flag to report any questions that need improvement.

Question 476: A late preterm infant is delivered at 34 weeks of gestational age via emergency cesarean section due to worsening preeclampsia in the mother. The mother received only one dose of betamethasone two hours prior to delivery. The infant emerged with a weak spontaneous cry and required positive pressure ventilation in the delivery room. Six hours later, his respiratory distress worsened, necessitating endotracheal intubation and placement of an umbilical arterial catheter. An umbilical venous catheter was attempted but was unsuccessful. Parenteral nutrition was infused through the umbilical arterial catheter. Over the next few hours, his respiratory status stabilized with improvement in blood gases, but the blood glucose levels start decreasing, requiring an increase in the glucose infusion rate to 15 mg/kg/min. At this point, what would be the best course of management to address the underlying cause of the problem?

Choices: 1. Perform glucagon stimulation test 2. Increase the glucose infusion rate (GIR) further to maintain euglycemia 3. Perform lumbar puncture to rule out meningitis 4. Consider alternate access for parenteral nutrition

Answer: 4 - Consider alternate access for parenteral nutrition Explanations: Refractory hypoglycemia is a rare complication of umbilical arterial catheters when dextrose is infusing through them. When the catheter tip is in a high position ending near the celiac axis or mesenteric arteries, the dextrose can stimulate beta cells of the pancreas, secreting insulin leading to hyperinsulinemic hypoglycemia. The condition is reversible after discontinuation of the dextrose-containing fluids. The umbilical artery is not usually used for infusion of maintenance glucose/electrolyte solutions or medications. Go to the next page if you knew the correct answer, or click the link image(s) below to further research the concepts in this question (if desired).

Research Concepts: Umbilical Artery Catheterization

We update eBooks quarterly and Apps daily based on user feedback. Please tap flag to report any questions that need improvement.

Question 477: An on-call physician receives a call that he is soon to receive an ICU transfer from an outside facility. The charge nurse informs the physician that the patient is being transferred with anemia, thrombocytopenia, prolonged aPTT, and INR, and schistocytes noted on a peripheral blood smear. Before the patient arrives, the physician develops an action plan for the patient while considering the additional medical problems the patient is likely to have. What is the most likely additional disease process occurring in this patient?

Choices: 1. Placental abruption 2. Aortic aneurysm 3. Malignancy 4. Sepsis

Answer: 4 - Sepsis Explanations: Sepsis is the most common cause of disseminated intravascular coagulation (DIC). DIC occurs in 30% to 50% of cases of severe sepsis. Trauma, malignancy, aortic aneurysms, and obstetrical complications are causes of DIC, though they are less common. DIC has classically been associated with gram-negative sepsis. Go to the next page if you knew the correct answer, or click the link image(s) below to further research the concepts in this question (if desired).

Research Concepts: Disseminated Intravascular Coagulation

We update eBooks quarterly and Apps daily based on user feedback. Please tap flag to report any questions that need improvement.

Question 478: A 50-year-old male is being evaluated in the intensive care unit. He suffered from right cerebellar hemorrhage, four weeks back, which was complicated by aspiration pneumonia. He was admitted and intubated due to respiratory distress. An elective tracheostomy was done due to prolonged anticipated mechanical ventilation. He is seen today in the ICU and is doing well, his chest X-ray has improved, and he was placed on spontaneous mode this morning. His current medications include clopidogrel, enalapril, omeprazole, enoxaparin, and piperacillintazobactam. He developed minor bleeding from the tracheostomy during this morning’s suctioning and pulmonary toilet. During the round, he suddenly deteriorates, and his blood pressure is shown to be 80/50 mmHg and pulse of 150 beats per minute. His spO2 drops to 75%, and he becomes tachypneic. His tracheostomy suctioning demonstrates fresh blood that is seen to spurt. What immediate measures should be taken given the likely diagnosis?

Choices: 1. Overinflate tracheostomy tube 2. Administer protamine sulfate 3. Place the patient in the supine position 4. Administer tissue plasminogen activator

Answer: 1 - Overinflate tracheostomy tube Explanations: This patient has been on prolonged ventilatory support and has had an elective tracheostomy. The patient is improving on intravenous antibiotics and develops massive pulmonary hemorrhage. Sudden massive hemorrhage, along with spurting of blood, indicates bleeding from an arterial site. The likely diagnosis, in this case, is the formation of a trachea-innominate artery fistula (TIAF). TIAF is caused by erosion of anterior tracheal wall by a tracheostomy cuff of the tip and subsequent catastrophic bleed. The best temporary measure is to over inflate the tracheostomy cuff. Overinflation of cuff will lead to pressure over the innominate artery and may stop the bleeding. If bleeding does not stop by this maneuver, digital pressure over the artery can be used to buy time. Diagnostic procedures such as bronchoscopy and angiography can help to diagnose the condition. Surgical management or endoscopic coiling is the definitive management. But temporary stopping bleed should be of topmost priority. Placing patient supine or reversal of anticoagulation will not stop arterial bleeding. Go to the next page if you knew the correct answer, or click the link image(s) below to further research the concepts in this question (if desired).

Research Concepts: Tracheo Innominate Artery Fistula

We update eBooks quarterly and Apps daily based on user feedback. Please tap flag to report any questions that need improvement.

Question 479: A 55-year-old man who underwent an uneventful two-vessel coronary artery bypass continues to have minor bleeding during the closure. The patient is given whole blood rapidly to maintain volume. Soon after, he develops hyperthermia, hypotension, and diffuse bleeding from the incision and suture lines. What is the underlying pathophysiology of this reaction?

Choices: 1. Immune complex-mediated reaction 2. Antibody-mediated reaction 3. Mast cell-mediated reaction 4. T-cell mediated reaction

Answer: 2 - Antibody-mediated reaction Explanations: Type II hypersensitive reactions are the antibody-mediated destruction of cells. This type of reaction is best seen with blood-transfusion reactions, where the host antibodies react with foreign antigens on the incompatible transfused blood cells and mediate the destruction of these cells. In this patient, diffuse bleeding and renal failure developed right after the blood transfusion indicating acute hemolytic reaction. Acute hemolytic reactions are fatal and develop in cardiac patients. These reactions can be prevented by a proper crossmatching between the recipient's and the donor's blood. The cross-matching reveals the presence of the antibodies in the donor or recipient sera that can cause these reactions. Go to the next page if you knew the correct answer, or click the link image(s) below to further research the concepts in this question (if desired).

Research Concepts: Transfusion Reactions

We update eBooks quarterly and Apps daily based on user feedback. Please tap flag to report any questions that need improvement.

Question 480: A 55-years-old man undergoes an emergency laparotomy for intestinal obstruction. The patient is shifted to the intensive care unit, where he is mechanically ventilated with 20 cmH2O of positive end-expiratory pressure (PEEP). On the third day of ICU admission, the patient suddenly deteriorates. His blood pressure falls from 120/75 mmHg to 50/30 mmHg, and the oxygen saturation drops to 70%. Which of the following is the most likely cause of the patient's deterioration?

Choices: 1. Tension pneumothorax 2. Hyperventilation 3. Pulmonary embolism 4. Acute myocardial infarction

Answer: 1 - Tension pneumothorax Explanations: PEEP is produced by applying positive pressure to the mechanical ventilator's exhalation valve after the expiratory phase. It is often used to increase arterial oxygenation when FiO2 exceeds 0.50 to reduce O2 toxicity. There are, however, several potential hazards associated with the use of PEEP. These include decreased cardiac output, pulmonary barotrauma (i.e., tension pneumothorax), increased extravascular lung water, and redistribution of pulmonary blood flow. Barotrauma, such as pneumothorax, pneumomediastinum, and subcutaneous emphysema, occurs due to the overdistention of alveoli by PEEP. Pulmonary barotrauma should be suspected when there is an abrupt deterioration of arterial oxygenation and cardiovascular function during mechanical ventilation with PEEP. If barotrauma is suspected, a chest x-ray film should be obtained, and if tension pneumothorax is present, a chest tube should be placed in the involved chest cavity. Go to the next page if you knew the correct answer, or click the link image(s) below to further research the concepts in this question (if desired).

Research Concepts: Positive End-Expiratory Pressure

We update eBooks quarterly and Apps daily based on user feedback. Please tap flag to report any questions that need improvement.

Question 481: A 45-year-old woman presents to the provider complaining of high fever, left eye chemosis, and headache for the past three days. Past medical history is significant for a history of diabetes mellitus for the past ten years. Vital signs reveal a blood pressure of 120/80 mmHg, a pulse rate of 79 beats per minute, a respiratory rate of 14/min., and a temperature of 39 degrees Celsius. On physical examination, she has bilateral proptosis, and palsy of the sixth cranial nerve on the right side. Which of the following antibiotics should be included in the initial treatment of her underlying condition?

Choices: 1. Vancomycin and cefotaxime 2. Cefotaxime and metronidazole 3. Vancomycin and metronidazole 4. Vancomycin, cefotaxime, and metronidazole

Answer: 4 - Vancomycin, cefotaxime, and metronidazole Explanations: An infected cavernous sinus thrombosis is a medical emergency. Even with early antibiotic therapy, the mortality rate is approximately 20 percent. There are numerous potential sources of infection; therefore, the initial therapy must be broad-spectrum until definitive determination of the causative pathogen. Recent research shows that anticoagulation is beneficial in one study, but thrombolytics have not yet been studied. The dangerous area of the face consists of the triangular area from the corners of the mouth to the nasal bridge, including the lower part of the nose and maxilla. Venous drainage from this area drains directly into the cavernous sinus. Therefore, any infection involving the dangerous area of the face is a potential cause of cavernous sinus thrombosis. Go to the next page if you knew the correct answer, or click the link image(s) below to further research the concepts in this question (if desired).

Research Concepts: Cavernous Sinus Thrombosis

We update eBooks quarterly and Apps daily based on user feedback. Please tap flag to report any questions that need improvement.

Question 482: A 17-year-old male involved in a motor vehicle collision was found alert and stable at the scene but complaining of chest pain and shortness of breath. He was not wearing a seat belt and was suddenly thrown onto the dashboard. His airway is patent, and pulmonary exam reveals diminished breath sounds on the left. There are no signs of penetrating injury, subcutaneous emphysema, or flail chest. His blood pressure is 110/60 mmHg, pulse 100 beats per minute, and oxygen saturation 92%. A portable chest x-ray shows opacification of the left chest with a midline shift to the right. What is the next step in the management of this patient?

Choices: 1. Chest ultrasound 2. Chest CT 3. Rib radiographs 4. Chest tube

Answer: 4 - Chest tube Explanations: Chest x-ray is often the first imaging modality in patients with chest trauma. However, ultrasound is more specific for evaluation of pleural effusions and hemothorax. Ultrasound is usually readily available, rapid, repeatable, and reliable in experienced operators. In this case, a chest x-ray showing tension hemothorax was performed first. The next appropriate step is to stabilize the patient with a chest tube, rather than additional imaging. Lung ultrasound can be used to identify small pleural effusions greater than 20 mL in cases where the chest x-ray findings are equivocal. Chest CT would be the gold standard to identify all possible thoracic injuries. In this patient, who is displaying radiologic evidence of tension hemothorax, and signs of impending decompensation, a chest thoracostomy tube would be the next best step. Further imaging may be obtained once the patient becomes more stable. A CT scan is the gold standard imaging modality to evaluate intrathoracic injuries. However, in an acute trauma setting, a plain radiograph is usually adequate to initiate treatment. A rib series may be appropriate for a more stable, ambulatory patient with a low energy mechanism of injury. Once a traumatic pleural effusion sufficient to opacify an entire hemithorax is identified, chest tube placement should not be delayed for further imaging. Go to the next page if you knew the correct answer, or click the link image(s) below to further research the concepts in this question (if desired).

Research Concepts: Hemothorax

We update eBooks quarterly and Apps daily based on user feedback. Please tap flag to report any questions that need improvement.

Question 483: A 17-year-old boy with a history of Lennox-Gastaut syndrome, chronic pain, and depression is rushed to the emergency department after being found unconscious in his bed at home. Vital signs show temperature 98.4 F, blood pressure 110/88 mmHg, heart rate 58/min, respiratory rate 4/min, and SpO2 94% on room air. On exam, he is unable to open his eyes or follow commands but does withdraw to pain. His father is still recovering from the flu, and the patient the previous night complained of fatigue, but otherwise, the patient has had no known sick contacts. His current medications include buprenorphine, fluoxetine, and lamotrigine. He was recently started on cannabidiol (CBD) for treatment for his seizures, as well as starting the ketogenic diet a few weeks ago. What is the most likely cause of this patient's clinical presentation?

Choices: 1. Medication interaction 2. Nutritional deficiencies 3. Influenza 4. Suicide attempt

Answer: 1 - Medication interaction Explanations: Buprenorphine and cannabidiol (CBD) should not be used together due to potentials serious side effects, including respiratory depression, coma, and death. This patient has a respiratory rate of 4 and is likely suffering from a potentially fatal drug interaction. Currently, cannabidiol (CBD) has only been approved for children ages two and older who suffer from the seizure disorders Lennox-Gastaut syndrome and Dravet syndrome. The ketogenic diet has been proven to be beneficial for patients with seizure disorders. While the patient may have nutritional deficiencies, this is not the cause of the respiratory depression. The patient was complaining of fatigue, which can be a sign of influenza. However, it is unlikely that the patient's respiratory rate is depressed because of an infection by the flu. The patient suffers from depression, but besides this, no other clues are suggesting a suicidal attempt. It is more likely that the patient is suffering from an almost fatal drug interaction between buprenorphine and CBD. Go to the next page if you knew the correct answer, or click the link image(s) below to further research the concepts in this question (if desired).

Research Concepts: Cannabidiol (CBD)

We update eBooks quarterly and Apps daily based on user feedback. Please tap flag to report any questions that need improvement.

Question 484: A 55-year-old male is brought to the emergency department following a high-energy explosion that set fire to a building. The patient is brought in by the emergency medical technicians in a semi-conscious state and is violently coughing. A primary survey reveals a lean, old man in obvious respiratory distress. His vital signs show a heart rate of 100 beats/minute, a blood pressure of 110/70 mmHg, a respiratory rate of 16/minute, and an oxygen saturation of 90% in room air. Which of the following most accurately describes the blast phase injury in this patient?

Choices: 1. Primary blast injury 2. Secondary blast injury 3. Tertiary blast injury 4. Quaternary blast injury

Answer: 4 - Quaternary blast injury Explanations: Quaternary blast injuries include all blast-induced injuries and conditions that are not caused by primary, secondary, and tertiary blast effects. These injuries can be caused by exposure to resulting fire, fumes, radiation, biological agents, smoke, dust, toxins, environmental exposure, and the psychological impact of the event. Exacerbation of underlying medical conditions such as COPD, asthma, unstable angina, anxiety, depression is also a part of quaternary blast injuries and has a major impact on mortality and morbidity. In an intentional explosion, other substances can be added to cause more injury, illness, or to induce more fear and panic in the community. Go to the next page if you knew the correct answer, or click the link image(s) below to further research the concepts in this question (if desired).

Research Concepts: Blast Injuries

We update eBooks quarterly and Apps daily based on user feedback. Please tap flag to report any questions that need improvement.

Question 485: A 66-year-old Hispanic male is brought to the emergency department after being found unconscious at his home. His past medical history is significant for uncontrolled diabetes mellitus type 2 and hypertension. He is currently non-compliant with his medications per the family members at the bedside. Blood glucose was found to be 300 mg/dL. IV fluids, potassium, and insulin were initiated as per the DKA protocol. Four hours later, the patient's blood pressure dropped precipitously to 92/54 mmHg, and his heart rate became elevated at 110 bpm. It was then discovered that the patient had a 5x6 cm sacral ulcer with moderate erythema surrounding the base and moderate warmth on palpation. The ulcer was cleaned and dressed appropriately. Blood cultures were sent. He was then transferred to the intensive care unit (ICU) and started on empiric IV antibiotics for sepsis. 48 to 72 hours after admission into the ICU, the provider orders a serum protein marker that helps distinguish if the antibiotics are working appropriately. Which of the following increases this protein directly?

Choices: 1. Lipopolysaccharide (LPS) 2. Tumor necrosis factor (TNF) 3. Interleukin-1 (IL-1) 4. Interferon-gamma (IFN-Y)

Answer: 1 - Lipopolysaccharide (LPS) Explanations: Procalcitonin (PCT) is a serum biomarker that is upregulated following a bacterial infection. Current guidelines recommend that PCT levels should be ordered 48-72 hours following the initiation of antibiotics to ensure that the correct regimen was initiated. If the antibiotics are effective, the levels should drop accordingly. Lipopolysaccharide (LPS) is a lipid and polysaccharide molecule found in the outer membrane of gram-negative bacteria that increases procalcitonin directly along with inducing a robust immune response. TNF, also known as cachectin, is a pro-inflammatory cytokine that also increases procalcitonin but does so indirectly. Interleukin 1 is a cytokine that is an endogenous pyrogen. It increases procalcitonin indirectly. Interferon gamma is the major macrophage-activating cytokine with antiviral properties. It decreases procalcitonin. Go to the next page if you knew the correct answer, or click the link image(s) below to further research the concepts in this question (if desired).

Research Concepts: Procalcitonin

We update eBooks quarterly and Apps daily based on user feedback. Please tap flag to report any questions that need improvement.

Question 486: A 60-year-old female with a history of hypertension, diabetes, atrial fibrillation presents to the emergency department with altered mental status and new-onset rightsided weakness that started an hour prior to arrival. On presentation, blood pressure was 200/140 mmHg. Examination revealed an NIH stroke scale of 25. Neurological examination was significant for global aphasia, right-sided hemiparesis, and sensory loss. Further laboratory investigations showed a normal complete blood count, serum sodium of 130 mEq/L. Initial computed tomography head without contrast showed a dense middle cerebral artery on the left with the loss of grey-white matter differentiation in the left frontoparietal cortex and sulcal effacement. Which of the following interventions will prevent secondary brainstem hemorrhages from occurring?

Choices: 1. Early involvement of neurosurgery for possible decompressive hemicraniectomy 2. Correction of serum sodium 3. Aggressive blood pressure control 4. Discontinuing anticoagulation

Answer: 1 - Early involvement of neurosurgery for possible decompressive hemicraniectomy Explanations: Secondary brainstem hemorrhages or Duret hemorrhages happen in the clinical setting of transtentorial herniation of any cause. The patient described likely has a malignant left middle cerebral artery infarct. Such patients are at risk of brain edema, which can progress to cause transtentorial herniation and thereby secondary brainstem hemorrhages. Early involvement of neurosurgery for a possible decompressive hemicraniectomy can prevent herniation and secondary brainstem hemorrhage to occur. In malignant middle cerebral artery infarcts, brain edema occurs between day 1 to day 5. These patients require serial computed tomography head. Kinking of posterior cerebral artery against tentorium cerebelli can result in occipital lobe infarcts, which can further increase intracranial pressure and risk of herniation. Aggressive blood pressure control and discontinuation of anticoagulation should be done to prevent the hemorrhagic conversion of the infarct. While serum sodium should be corrected and maintained between above 140 mmol/L but it alone may not prevent herniation. Go to the next page if you knew the correct answer, or click the link image(s) below to further research the concepts in this question (if desired).

Research Concepts: Duret Hemorrhages

We update eBooks quarterly and Apps daily based on user feedback. Please tap flag to report any questions that need improvement.

Question 487: A 36-year-old patient presents to the emergency department with severe epigastric pain and vomiting. He is diagnosed with severe alcoholic pancreatitis and subsequently admitted to the intensive care unit. He requires vasopressors for hemodynamic instability and mechanical ventilation. He is kept nil-per-os (NPO), and total parenteral nutrition supplementation (TPN) is initiated. After 1 week of TPN supplementation, there is no improvement in pancreatitis; therefore, a feeding jejunostomy is placed. Twenty-four hours after initiation of jejunal tube feeds, he is noted to be hypotensive and has high peak pressure changes on the ventilator. An arterial blood gas analysis is suggestive of metabolic alkalosis. Which of the following is associated with the pathophysiology of this patient’s change in condition?

Choices: 1. Increased glycogenolysis 2. Decreased gluconeogenesis 3. Increased secretion from alpha pancreatic cells 4. Extracellular shift of electrolytes like potassium and phosphate

Answer: 2 - Decreased gluconeogenesis Explanations: Refeeding syndrome is associated with a rapid increase in insulin. Insulin acts to reduce blood glucose. It does this by preventing glycogenolysis (the breakdown of glycogen into glucose), preventing gluconeogenesis (formation of new glucose molecules), and increasing glycogenesis (conversion of glucose into glycogen). Insulin drives electrolytes like potassium and phosphate intracellularly. Insulin is sometimes used to treat hyperkalemia as it facilitates the diffusion of potassium intracellularly. The alpha cells of the pancreas secrete glucagon, beta cells secrete insulin, and the delta cells secrete somatostatin. Glucagon promotes the breakdown of glycogen and increases blood glucose. The actions of insulin are discussed previously. Somatostatin is an inhibitory hormone, inhibiting multiple neuroendocrine and gastrointestinal hormones. Go to the next page if you knew the correct answer, or click the link image(s) below to further research the concepts in this question (if desired).

Research Concepts: Feeding Jejunostomy Tube

We update eBooks quarterly and Apps daily based on user feedback. Please tap flag to report any questions that need improvement.

Question 488: A 62-year-old patient with alcohol use disorder is admitted with liver failure. A workup reveals that he is developing severe cognitive deficits and a flapping tremor. It is decided to treat him with an antibiotic. Which of the following should be given?

Choices: 1. Amoxicillin 2. Erythromycin 3. Neomycin 4. Trimethoprim/sulfamethoxazole

Answer: 3 - Neomycin Explanations: Hepatic encephalopathy treatment, in part, is aimed at manipulating the intestinal milieu. Neomycin is an aminoglycoside that is used orally to reduce bowel flora. It can be used before bowel surgery or to reduce ammonia production in patients with hepatic encephalopathy. Neomycin is not given parenterally as it is ototoxic and nephrotoxic. Go to the next page if you knew the correct answer, or click the link image(s) below to further research the concepts in this question (if desired).

Research Concepts: Acute Liver Failure

We update eBooks quarterly and Apps daily based on user feedback. Please tap flag to report any questions that need improvement.

Question 489: A middle-aged male patient presents to the clinic with a Wagner stage 3 diabetic foot ulcer and has a history of impaired wound healing. The clinician decides to proceed with hyperbaric oxygen therapy. The clinician monitors the patient as he "dives" into the hyperbaric oxygen therapy chamber. During decompression, the patient complains of chest pain and shortness of breath. What is the most likely underlying pathology for this complaint?

Choices: 1. The patient is experiencing acid reflux from laying on his back 2. The patient is experiencing normal side effects of hyperbaric oxygen therapy and increased atmospheric pressure 3. The patient is experiencing a pneumothorax 4. The patient is experiencing barotrauma to his ears

Answer: 3 - The patient is experiencing a pneumothorax Explanations: Pneumothorax is a rare but potentially fatal risk of HBO. Pneumothorax is commonly called a collapsed lung. Barotrauma to the ear is a more common but much less morbid side effect of HBOT. It is commonly referred to as a "squeeze". History of air-trapping lung lesion, spontaneous pneumothorax, or mechanical ventilation put one at increased risk for pneumothorax during HBOT. Pneumothorax can progress to tension pneumothorax and decreased venous return to the heart. Treatment usually includes the insertion of a chest tube. Go to the next page if you knew the correct answer, or click the link image(s) below to further research the concepts in this question (if desired).

Research Concepts: Impaired Wound Healing

We update eBooks quarterly and Apps daily based on user feedback. Please tap flag to report any questions that need improvement.

Question 490: A 60-year-old man, with a history of hypertension, obesity, and dyslipidemia, presents with exertional dyspnea and pitting ankle edema. Which of the following therapies would best treat the findings expected on his chest x-ray?

Choices: 1. Continuous positive airway pressure (CPAP) 2. High flow supplemental oxygen 3. Vasodilators 4. Opioids

Answer: 1 - Continuous positive airway pressure (CPAP) Explanations: Congestive heart failure is associated with interstitial and alveolar edema. The determining factor as to whether fluid is filtered out of pulmonary vasculature and into the alveoli is a balance between forces in both the alveoli and the capillaries themselves. Oncotic pressures are determined by protein and other solutes in the alveoli, as well as the capillary hydrostatic pressure, exert forces that will force fluid into the alveoli. On the contrary, oncotic pressures in the pulmonary capillaries and hydrostatic pressure in the alveoli will force fluid back into the pulmonary capillaries. Continuous positive airway pressure (CPAP) increases the pressure within the alveoli, forcing fluid back into pulmonary capillaries. This is the only management strategy or medication listed that actively pushes fluid out of alveoli. Supplemental oxygen is provided through the application of CPAP, which acts to both improve the patient's pulse oximetry and increase the alveolar hydrostatic pressure. This assumes all other factors like hemoglobin content are normal. Additional management considerations include vasodilators such as nitroglycerin and opioids. However, these interventions take time to work. Therefore, the most rapid intervention in the case of congestive heart failure that will do the most good to improve ventilation of the patient is CPAP. Go to the next page if you knew the correct answer, or click the link image(s) below to further research the concepts in this question (if desired).

Research Concepts: Continuous Positive Airway Pressure

We update eBooks quarterly and Apps daily based on user feedback. Please tap flag to report any questions that need improvement.

Question 491: A 51-year-old male is admitted to the hospital for his next chemotherapy dose for a recently diagnosed non-Hodgkin lymphoma. The patient has no other significant past medical history. On his 4th day of hospitalization, the patient suddenly develops difficulty breathing, muscle pain, and a significant worsening of his lower limb edema without any fevers or chills. The patient had a blood pressure of 155/94 mmHg 2 hours ago. When the nurse inflates the blood pressure cuff to check the blood pressure, the patient develops muscle spasms in the same upper limb. He is in mild respiratory distress, and crackles can be auscultated throughout the lung fields. The patient has a respiratory rate of 24 breaths/min. Neck veins seem to be distended. Heart rate is 112 beats/min with a regular rhythm. BIlateral 2+ edema in the lower limb is also noted. Lab reports include serum creatinine of 3.4 mg/dl, leukocyte count of 12,000 cells/mm3, Hgb levels of 9 g/dl, platelet count of 20,000 cells/mm3, sodium levels of 131 mEq/L, potassium levels of 6.3 mEq/L, bicarbonate levels of 15 mEq/L, blood urea of 77 mg/dl, calcium levels of 6.8 mg/dl, phosphorus levels of 8.6 mg/dl, uric acid levels of 19 mg/dl and albumin levels of 4 g/dl. Urinalysis reports trace blood and 2+ protein levels. Urine microscopy revealed two red blood cells/HPF, 4-10 WBC/HPF, uric acid crystals, and granular casts. In addition to hemodialysis, which of the following is the next most appropriate next step in management?

Choices: 1. A drug which is a competitive inhibitor of xanthine oxidase 2. A drug which is a recombinant uricase catalyzing uric acid to allantoin 3. A drug that binds and stabilizes tubulin to inhibit microtubule polymerization, impairing neutrophil chemotaxis and degranulation. 4. A drug that Inhibits reabsorption of uric acid in the distal convoluted tubule

Answer: 2 - A drug which is a recombinant uricase catalyzing uric acid to allantoin Explanations: In a patient with hyperuricemia due to tumor lysis syndrome, the most appropriate way to lower the uric acid levels rapidly is the use of rasburicase, which is a drug that is a recombinant uricase catalyzing uric acid to allantoin which is more water-soluble than uric acid. Because rasburicase also leads to the formation of hydrogen peroxide, treated patients are at risk for methemoglobinemia and hemolytic anemia. Therefore, patients should be checked for G6PD deficiency. Allopurinol and febuxostat work by limiting uric acid formation. Neither of these drugs metabolizes previously generated uric acid. Thus these drugs can be used only for the prevention of uric acid formation which is a little too late in this case. Conditions of accelerated cell breakdown or turnover such as rhabdomyolysis, hemolysis, and tumor lysis can also be a purine source and thus, increase uric acid production. Uric acid is filtered in the glomerulus and is mostly reabsorbed in the proximal convoluted tubule. The distal convoluted tubule is mostly impermeable to uric acid. Go to the next page if you knew the correct answer, or click the link image(s) below to further research the concepts in this question (if desired).

Research Concepts: Hyperuricemia

We update eBooks quarterly and Apps daily based on user feedback. Please tap flag to report any questions that need improvement.

Question 492: A patient is brought to the emergency room because of behavior problems. His friend says that he has a history of using illicit drugs. The individual appears to be hallucinating, euphoric, and extremely agitated. He has reddish skin, slurred speech, and a very unsteady gait. An ocular exam reveals dilated pupils with nystagmus. Which of the following drugs was most likely ingested?

Choices: 1. Cocaine 2. Phencyclidine 3. Amphetamine 4. Marijuana

Answer: 2 - Phencyclidine Explanations: Phencyclidine also is known as angel dust and is a potent hallucinogen. The drug works as an N-methyl-D-aspartate receptor antagonist and is a dangerous chemical. The behavioral effects resemble schizophrenia, and most individuals are euphoric. Other findings of phencyclidine ingestion include rage, reddish skin, dilated pupils, nystagmus, and excitation. . PCP causes mydriasis in 6% of patients and miosis in 2%. The overdose is managed by supportive care and use of sedatives and antipsychotics. Go to the next page if you knew the correct answer, or click the link image(s) below to further research the concepts in this question (if desired).

Research Concepts: Phencyclidine Toxicity

We update eBooks quarterly and Apps daily based on user feedback. Please tap flag to report any questions that need improvement.

Question 493: A 29-year-old woman at 35 weeks gestation is brought in by ambulance to the emergency department after being involved in a motor vehicle collision. She endorses pain only to the left thigh. A pelvic exam reveals only a deformity of the left femur. The patient's vital signs reveal a temperature of 37.1 C, a heart rate of 105 bpm, blood pressure of 90/55 mmHg, the respirations of 18/min, and oxygen saturation of 99%. The abdominal is non-tender, and the gravid uterus is approximately 35 weeks distended. An ultrasound of the uterus reveals posterior placenta, fetal heart rate of 143 bpm, and no evidence of subchorionic hematoma. X-ray of the left lower extremity reveals a mid-shaft fracture of the left femur. After Intravenous fluid administration, the patient's heart rate is 89 bpm, and blood pressure is 110/75 mmHg. Orthopedic surgery is consulted and recommends immediate surgery of the left femur. While in the operation room for an open reduction and internal fixation of the femur, the patient's heart rate spikes to 121 bpm, and blood pressure is 85/49 mmHg. A bolus of intravenous fluid does improve the patient's hemodynamics. What is the next step in the management of the patient's condition?

Choices: 1. Thoracotomy 2. Reposition the patient 3. Vasopressor administration 4. Consult obstetrics and gynecology

Answer: 2 - Reposition the patient Explanations: The pregnant patient who was previously hemodynamically stable prior to the surgical intervention is hemodynamically unstable during the procedure. While a number of etiologies can be potential culprits to this patient's condition, such as fat embolism, pulmonary embolism, hypovolemic shock, or hemorrhagic shock, the most simple explanation is that the patient has inferior vena cava (IVC) syndrome because her inferior cava is being obstructed by the compression of the gravid uterus while she is supine for the surgery. Therefore, the most non-invasive intervention should be tried first, which is fluid resuscitation, and if that does not help, then the patient should be repositioned so that the uterus is away from the IVC. IVC compression is very common in pregnant women and the incidence increases with increased gestational age. Pregnant women should be counseled on laying down and sleeping in the left lateral decubitus position to avoid compression of the IVC. The IVC courses underneath the gravid uterus and repositioning the patient should relieve the obstruction, therefore increasing venous blood return to the heart and improving this patient's hemodynamics. A thoracotomy is indicated only for penetrating trauma leading to cardiac arrest and would do more harm to the patient than benefit. Go to the next page if you knew the correct answer, or click the link image(s) below to further research the concepts in this question (if desired).

Research Concepts: Inferior Vena Cava Syndrome

We update eBooks quarterly and Apps daily based on user feedback. Please tap flag to report any questions that need improvement.

Question 494: A 43-year-old man is brought to the emergency department (ED) for altered mental status. The patient was found in his house by his neighbor. Upon arrival at the scene, EMS report the patient was somnolent and looked yellow and did not respond to 0.4 mg of naloxone. Upon arrival to the ED, the patient has a heart rate of 108/min, blood pressure 93/54 mmHg, respiratory rate 28/min, oxygen saturation 94% on the nasal cannula, and temperature 101.4 F. The patient is noted to have a distended abdomen and upon palpation of the right upper quadrant, winces in pain, and grunts in discomfort. 30 ccs/kg of IV fluid bolus is given and initial labs reveal white blood cell count of 18000/microL, AST 89 IU/L, and ALT 119 IU/L. The patient is taken to the operating room for emergent surgery. Which of the following intubation medications is most likely to worsen this patient's condition?

Choices: 1. Ketamine 2. Fentanyl 3. Midazolam 4. Rocuronium

Answer: 2 - Fentanyl Explanations: The patient is febrile, tachycardic, hypoxic, and has a WBC count of 18000/microL. The patient is in septic shock. In septic shock, the patient has decreased systemic vascular resistance, which causes hypotension. For tracheal intubation, induction and sedation medications should be chosen accordingly. Medications that can cause further hypotension, such as fentanyl, should be avoided. Fentanyl is a synthetic opioid that has a rapid onset of action and is rapidly metabolized. It causes profound analgesia and is best used for sedation after tracheal intubation in patients with intracranial injuries as it can help reduce intracranial pressure. Conversely, it can cause a worsening of systemic hypotension in patients with septic shock. Fentanyl can also cause respiratory depression, and in this patient with hypoxia and respiratory distress, it is important to avoid fentanyl. Ketamine (choice 1) can cause laryngospasm but is not contraindicated in patients with hypotension. Midazolam (choice 3) is a benzodiazepine that can also cause hypotension and respiratory depression but is easily titratable and has less central nervous system depression than fentanyl. Rocuronium (choice 4) is an induction agent that is not contraindicated in patients in septic shock. Go to the next page if you knew the correct answer, or click the link image(s) below to further research the concepts in this question (if desired).

Research Concepts: Tracheal Intubation Medications

We update eBooks quarterly and Apps daily based on user feedback. Please tap flag to report any questions that need improvement.

Question 495: A 75-year-old African American man presents to the emergency department for evaluation of shortness of breath. He has a past medical history of asthma, congestive heart failure with an ejection fraction of 25%, hypertension, and coronary artery disease. He describes a 2-week history of shortness of breath, which has progressively gotten worse. The patient is unable to complete full sentences without getting short of breath. He denies any associated chest pain but states he has been having racing of the heart for the past one week. The patient takes aspirin, lisinopril, metoprolol, spironolactone, atorvastatin, albuterol inhaler as needed for wheezing, and he has been compliant with all his medications. On physical exam, the patient has bibasilar crackles, irregularly irregular heartbeat, and 2+ bilateral lower extremity edema. Vital signs show blood pressure 140/90 mmHg, heart rate 140/min, and SpO2 95% on room air. EKG reveals irregularly irregular rhythm, with variable P-R interval and variable P-wave morphologies. Renal function tests, electrolytes, D-dimer are within normal limits. Serum magnesium is 2.3 mEq/L and potassium 4.2 mEq/L. In the emergency department, a trial of intravenous magnesium is given, which does not help with his heart rate. Which of the following is the next best step in the management of this patient?

Choices: 1. Albuterol/ipratropium nebulizations 2. Repeat intravenous magnesium 3. Verapamil 4. Increase the dose of metoprolol

Answer: 4 - Increase the dose of metoprolol Explanations: The patient in the above clinical vignette presented with possible worsening heart failure caused by multifocal atrial tachycardia (MAT). Increasing the dose of metoprolol to target a heart rate of 110/min would be the ideal next step of management. Metoprolol is a selective beta-blocker that suppresses atrial ectopy and also decreases AV nodal conduction. It is preferred over calcium channel blockers for heart rate control especially in patients with heart failure with reduced ejection fraction. At this time, breathing treatment is not indicated. Patient electrolytes are within normal limits. One dose of magnesium did not help with the heart rate. Studies have shown magnesium replacement in patients with or without hypomagnesemia has improved MAT. Verapamil has been shown to be efficacious in controlling heart rate in patients with multifocal atrial tachycardia by decreasing AV nodal conduction. In this above clinical vignette, the patient has reduced ejection fraction in which case the better option would be to increase dose of metoprolol. Calcium channel blockers are generally avoided in patients with reduced ejection fraction. Go to the next page if you knew the correct answer, or click the link image(s) below to further research the concepts in this question (if desired).

Research Concepts: Multifocal Atrial Tachycardia

We update eBooks quarterly and Apps daily based on user feedback. Please tap flag to report any questions that need improvement.

Question 496: A 42-year-old male presents to the emergency department with altered mental status. The family reports that the patient had recently gone through a divorce and was found with a bottle of pink-colored pills next to him. His past medical history is significant for hypertension, allergic rhinitis, and insomnia. His home prescription medications include lisinopril. His blood pressure is 125/87 mmHg, heart rate is 186 beats/min, respiratory rate is 37 breaths/min, and oxygen saturation (SpO2) is 96% on room air. Physical exam reveals respiratory distress and dry mucous membranes. Electrocardiogram (ECG) demonstrates a QTc interval of 525 milliseconds. Ingestion took place approximately 85 minutes ago. He starts seizing on the gurney, and his SpO2 falls to 79%. What is the best step in the management of this patient?

Choices: 1. Lorazepam bolus and ICU admission for emergent hemodialysis 2. Lorazepam bolus and emergent intubation for airway protection 3. Diazepam bolus and albuterol nebulizer 4. Intravenous naloxone

Answer: 2 - Lorazepam bolus and emergent intubation for airway protection Explanations: Benzodiazepines, such as lorazepam, can be used for symptomatic management of seizures secondary to diphenhydramine toxicity. Given that this patient’s SpO2 is declining, continued respiratory distress can be fatal. Emergent intubation is warranted in this situation to protect the patient’s airway and to provide respiratory support. Naloxone is used for opioid reversal and thus, would be ineffective in reversing the effects of diphenhydramine. Go to the next page if you knew the correct answer, or click the link image(s) below to further research the concepts in this question (if desired).

Research Concepts: Diphenhydramine Toxicity

We update eBooks quarterly and Apps daily based on user feedback. Please tap flag to report any questions that need improvement.

Question 497: A 43-year-old man is brought to the emergency department after a referral for the management of diabetic ketoacidosis. He has a history of type 1 diabetes, being on regular insulin since childhood. His major complaints are high-grade fever and severe headache for the past 5 days. He was prescribed amoxicillin for suspected sinusitis. His girlfriend reports that he has had decreased food intake in the past 3-4 days, and he had also skipped his regular medicines due to extreme lethargy. He also reports severe nausea and vomiting, along with mild breathing difficulty for the past 8 hours. Examination reveals an unconscious patient in a gasping state. He is immediately intubated and artificially ventilated. Further examination reveals proptosis, reddish discoloration, blackish eschar, and swelling of his right eye. His right pupil is fixed and dilated. His bedside blood glucose is 510 mg/dL, and ketones are strongly positive. The base excess is -14 mE/L. Treatment for diabetic ketoacidosis is initiated. He is hemodynamically stable and saturating 98% on room air. A T2 weighted MRI with Fat Sat sequence reveals soft tissue densities in the right sinuses and orbit, and gadolinium contrast evaluation confirms thrombosis in the right cavernous sinus. Sporangia and rhizoids were visualized with lactophenol blue staining of the tissues obtained from debridement. Which of the following is the next best step in the management of this patient?

Choices: 1. Liposomal amphotericin B 2. Amphotericin B 3. Posaconazole 4. Voriconazole

Answer: 1 - Liposomal amphotericin B Explanations: This young patient in diabetic ketoacidosis with evidence of sinus/ orbit and cavernous sinus involvement merits a high consideration for rhino-orbital cerebral mucormycosis. The suspicion or diagnosis of rhino-orbital cerebral mucormycosis is a medical as well as surgical emergency since it carries high morbidity & mortality even with appropriate treatment. IV liposomal amphotericin is initiated immediately at a dose of 5 mg per Kg body weight per day, with many clinicians going up to a dose of 10 mg per Kg body weight/day. Surgical debridement is planned immediately. Extensive surgical debridement is to be planned at the earliest. This will help to arrive at an early tissue diagnosis via frozen section specimens. Tissue PCR techniques also will help in reaching a diagnosis. Surgical debridement is also expected to reduce the fungal load. Extensive exploration and debridement of sinuses along with orbital decompression or exenteration is usually done. Local installation of amphotericin is often done in debrided cavities. The intracranial disease is also debrided as much as feasible. The role of surgical exploration and debridement in intracranial involvement is a bit controversial since many experts believe that mortality is inevitable. However, there are case reports of survival in early intracranial involvement. Liposomal amphoterin B has better tissue penetration and minimal renal toxicities compared to parenteral amphotericin B. IV posaconazole or isavuconazole is used only when the patient is not tolerating amphotericin or is not responding to amphotericin. They are used as a step-down oral continuation therapy agent, only after many weeks of IV amphotericin therapy has resulted in clinical improvement in the given patient. Go to the next page if you knew the correct answer, or click the link image(s) below to further research the concepts in this question (if desired).

Research Concepts: Rhino-orbital Cerebral Mucormycosis

We update eBooks quarterly and Apps daily based on user feedback. Please tap flag to report any questions that need improvement.

Question 498: A 54-year-old woman presents after noticing red spots develop on both her legs just above the ankles. Her past medical history includes beta-thalassemia major that is managed with recurrent blood transfusions. She has no allergies. On examination, she is afebrile, her respiratory rate is 10/minute, heart rate 78 beats per minute, and blood pressure 134/80 mmHg. Lab results show normal WBC and platelet counts, BUN 30 mg/dL (7-20 mg/dL) and creatinine 2.0 mg/dL (0.8-1.3 mg/dL ). C4 is low, and C3 is normal. Autoimmune studies show negative ANA and positive RF and cryoglobulins. Further screening reveals positive HCV antibody and HCV RNA high viral load. Given the likely diagnosis, which of the following patient factors indicate a likely poor prognosis?

Choices: 1. Age under 65-years 2. Renal impairment 3. Low C4 complement 4. Skin lesions above ankle

Answer: 2 - Renal impairment Explanations: In the absence of liver cirrhosis, renal impairment is an important prognostic factor in HCV related mixed cryoglobulinemic vasculitis. Kidney involvement can progress rapidly to renal failure and death. Renal involvement is a poor prognostic factor in non-infectious related mixed cryoglobulinemic vasculitis as well. Poor prognostic factors in the absence of liver failure include proteinuria greater than 1 g/day, serum creatinine greater than 150 micromol/L, cardiomyopathy, gastrointestinal, and CNS involvement. Age more than 65 years is a poor prognostic factor in non-infectious related mixed cryoglobulinemic vasculitis. Palpable purpura in the dependent areas like legs is a common presentation in this disease and not a prognostic factor. C4 can be low due to immune complex-mediated complement activation and does not dictate prognosis. Go to the next page if you knew the correct answer, or click the link image(s) below to further research the concepts in this question (if desired).

Research Concepts: Cryoglobulinemic Vasculitis

We update eBooks quarterly and Apps daily based on user feedback. Please tap flag to report any questions that need improvement.

Question 499: A 16-year-old man is brought to the emergency department due to fever, headache, photophobia, and vomiting for the past two days. His medical history is significant for hemolytic uremic syndrome diagnosed two weeks ago. The temperature is 39.2 C (102.6 F), blood pressure is 120/75 mmHg, the pulse is 100/min, and the respiratory rate is 17/min. Funduscopy was unremarkable. His neck is stiff. White blood cell count is 16,500/mm with neutrophilic predominance. CT scan of the head is unremarkable. Blood and lumbar puncture results are suggestive of bacterial meningitis, but cultures are still pending. Which of the following is responsible for increasing the risk of invasive meningitis?

Choices: 1. Eculizumab 2. Vitamin A supplementation 3. Over the counter antipyretics 4. Meningococcal vaccination

Answer: 1 - Eculizumab Explanations: Eculizumab is a monoclonal antibody that binds complement protein C5 leading to terminal complement cascade inhibition. The late complement system plays the main role in the host defense against invasive bacterial meningitis. (1400- to a 10,000-fold increase in meningococcal disease risk) Terminal complement deficiency is associated with recurrent invasive bacterial infections (in up to 50% of patients) with a recurrence rate of 100-150 times higher than that in the general population. Meningococcal vaccine is a cellular fraction vaccine that is generally safe and doesn't increase the risk of bacterial meningitis. Go to the next page if you knew the correct answer, or click the link image(s) below to further research the concepts in this question (if desired).

Research Concepts: Meningitis

We update eBooks quarterly and Apps daily based on user feedback. Please tap flag to report any questions that need improvement.

Question 500: A 16-year-old boy is brought to the emergency department with complaints of an inability to move his both upper and lower limbs after halfway through a soccer match. He reports no fever, pain, and change in sensation in his limbs. He went to camping 1 month back does not recall any insect bite. He does not recall any recent illness and denies the use of any medication currently. He had a similar episode of muscle weakness a few months back, which resolved spontaneously. His vital signs are within normal limits. Cardiovascular, respiratory, and gastrointestinal examinations reveal no abnormality. On neurological examination, muscle strength is decreased, and knee and ankle reflexes are depressed. His touch, vibration, and proprioception sensations are intact. Laboratory work reveals sodium 140 mEq/L, potassium 2.2 mEq/L, serum urea nitrogen 16 mEq/L, creatinine 0.7 mg/dL, chloride 106 mEq/L, bicarbonate 24 mEq/L, and creatine kinase 1110 U/L. EKG shows depressed ST-segment and tall T waves. What is the best initial therapy for this patient?

Choices: 1. Intravenous glucocorticoids 2. Plasma exchange 3. Oral potassium supplementation 4. Equine serum antitoxin

Answer: 3 - Oral potassium supplementation Explanations: Sudden onset of flaccid paralysis in a young male patient with past medical of a similar episode with hypotonia and depressed deep tendon reflexes without changes in the sensation in the setting of hypokalemia strongly leads to suspicion of the hypokalemic periodic paralysis. Oral potassium solution is the initial management approach for the patient presenting with an attack of muscle weakness in hypokalemic periodic paralysis. Intravenous glucocorticoids, equine serum antitoxin, plasma exchange are ineffective in hypokalemic periodic paralysis. Administration of the oral potassium usually alleviates the symptoms of the hypokalemic periodic paralysis. Absence of fever, no insect bite, or any recent illness, intact sensation on examination, are helpful to rule out the possible causes of flaccid paralysis like acute viral myelitis, GuillianBarre syndrome, or tick paralysis. The occurrence of the symptoms several hours after strenuous exercise, playing in a soccer match in this patient, is more suspicious for hypokalemic periodic paralysis. Prolonged rest after strenuous exercise is an identified triggering factor for flaccid paralysis in hypokalemic periodic paralysis. Go to the next page if you knew the correct answer, or click the link image(s) below to further research the concepts in this question (if desired).

Research Concepts: Hypokalemic Periodic Paralysis

We update eBooks quarterly and Apps daily based on user feedback. Please tap flag to report any questions that need improvement.

Section 6 Question 501: An 86-year-old man with a past medical history of follicular cell lymphoma status post-chemotherapy presents to the emergency department for the fourth time this month with abdominal pain, bloating, and an inability to have a bowel movement for 6 days and pass gas for 2 days. A CT scan of the abdomen is done and is consistent with small bowel obstruction secondary to a large mesenteric mass. The patient is consistently nauseous and vomiting, and he is unable to tolerate anything orally. Which of the following is the most appropriate indication for a percutaneous endoscopic gastrostomy (PEG) tube placement in this patient?

Choices: 1. Mechanical reopening of the obstruction 2. Increasing the caloric intake 3. Stimulating a bowel movement 4. Decompression of the stomach

Answer: 4 - Decompression of the stomach Explanations: A PEG tube provides enteral feeds in a patient with malnutrition. However, enteral feeds do not aid in relieving his distended stomach and symptoms secondary to his bowel obstruction. PEG tubes can also be used to decompress the stomach due to direct percutaneous access to the stomach. PEG tube can be used to decompress the stomach in a patient with a gastric obstruction causing a distended stomach. Percutaneous endoscopic gastrostomy (PEG) tubes serve as the favorable route of feeding and nutritional support in patients with a functional gastrointestinal (GI) system who require long-term enteral nutrition, usually beyond 4 weeks. Go to the next page if you knew the correct answer, or click the link image(s) below to further research the concepts in this question (if desired).

Research Concepts: Percutaneous Endoscopic Gastrostomy Tube

We update eBooks quarterly and Apps daily based on user feedback. Please tap flag to report any questions that need improvement.

Question 502: A 63-year-old male presents with severe shortness of breath and chest pain on inspiration with bluish discoloration of the fingers. On exam, his blood pressure is 100/70 mmHg, pulse 105/min, respiratory rate 26/min, and SpO2 is 81% on room air and 83% on supplemental oxygen. On auscultation, S1 and S2 are normal with no added sounds. Chest auscultation reveals fine inspiratory crepitations. The differentials include cardiogenic edema and acute respiratory distress syndrome. Which of the following differentiates acute respiratory distress syndrome from cardiogenic pulmonary edema?

Choices: 1. ST-segment changes on electrocardiogram 2. Bilateral lung infiltrates on chest x-ray 3. Pulmonary capillary wedge pressure (PCWP) less than 18 mm Hg 4. Hypoxemia

Answer: 3 - Pulmonary capillary wedge pressure (PCWP) less than 18 mm Hg Explanations: Acute respiratory distress syndrome (ARDS) can be differentiated from cardiogenic pulmonary edema by pulmonary capillary wedge pressure (PCWP), which is measured by an invasive technique (Swan Ganz catheter). Cardiogenic edema has PCWP greater than 18 mm Hg, but ARDS has PCWP of less than 18 mmHg. Cardiogenic pulmonary edema increases the pressure in the left atrium and hence an increased PCWP. Cardiogenic pulmonary edema can also be differentiated from ARDS further by B-type natriuretic peptide (BNP)/pro BNP levels. It tends to increase significantly in cardiogenic pulmonary edema as it is released by the walls of the atria and ventricles of the heart. Radiological investigations like chest x-ray and echocardiography can also be used for differentiation between cardiogenic pulmonary edema and ARDS. A chest x-ray will show cardiomegaly, vascular redistribution, and pleural effusion in cases of cardiogenic pulmonary edema, while these findings are less likely in ARDS. Echo can also be used to differentiate the two as it can detect any valvular defects of the heart, ejection fraction, and wall dysfunction. Tachycardia, hypoxemia, and lung infiltrations are the common findings in both cardiogenic edema and ARDS. Still, hypoxemia of cardiogenic cause responds quickly to high flow supplemental oxygen, while hypoxemia of ARDS can be refractory to oxygen therapy. Go to the next page if you knew the correct answer, or click the link image(s) below to further research the concepts in this question (if desired).

Research Concepts: Cardiogenic Pulmonary Edema

We update eBooks quarterly and Apps daily based on user feedback. Please tap flag to report any questions that need improvement.

Question 503: A 68-year-old male with a past medical history of congestive heart failure with recurrent pleural effusions undergoes an indwelling pleural catheter insertion. Three weeks later, the patient presents with fever, difficulty breathing, and cough. The patient says he is unable to drain the pleural catheter. Computed tomography chest reveals large right-sided pleural effusion, and the pleural catheter was in place. Diagnostic thoracentesis shows empyema with loculated effusion. With therapeutic thoracentesis, the patient's lung fails to expand. What is the best next step?

Choices: 1. Repeat thoracentesis 2. Remove the pleural catheter and replace the new one 3. TPA through the pleural catheter 4. Decortication

Answer: 4 - Decortication Explanations: It is characterized by the lung's inability to expand and fill the thoracic cavity due to a restricting fibrous visceral pleural peel. The resulting chronic pleural space is fluid-filled, and the persistence of the fluid is solely due to hydrostatic equilibrium. As the patient's lung is not expanding secondary to the trapped lung, repeated thoracentesis is not helpful. As the patient has empyema, pleural catheters should be removed, but as the patient has extensive, complicated effusion with trapped lung, decortication is the treatment of choice. Decortication is an option for lung re-expansion if symptoms persist 6 months after empyema resolution. TPA through the pleural catheters can be tried to see if catheter drains. Still, as the patient has empyema with trapped lung, it is recommended to remove the existing pleural catheter, followed by decortication. Repeated pleural catheter insertion, especially in the setting of empyema, is not recommended. Go to the next page if you knew the correct answer, or click the link image(s) below to further research the concepts in this question (if desired).

Research Concepts: Trapped Lung

We update eBooks quarterly and Apps daily based on user feedback. Please tap flag to report any questions that need improvement.

Question 504: A 68-year-old man presents with severe respiratory distress. He has a past medical history of chronic obstructive pulmonary disease (COPD), hypertension, and hyperlipidemia. His medications include metoprolol, inhaled fluticasone/salmeterol, and atorvastatin. His vital signs show oxygen saturation 87% on room air, respiratory rate 32 per minute, heart rate 106 bpm, blood pressure 110/72 mmHg, and temperature 101.4 F. A chest xray shows consolidation in the right lung base. Arterial blood gases show pH 7.30 (7.35-7.45), PaO2 56 mmHg (75-100), and PaCO2 48 mmHg (35-45). The patient is started on empiric antibiotics, blood cultures are performed, and he is intubated. After 12 hours, he is noted to have increased intracranial pressure. Which of the following findings would also be expected?

Choices: 1. Hyponatremia 2. Hypernatremia 3. Hyperkalemia 4. QT prolongation

Answer: 4 - QT prolongation Explanations: This patient likely was induced with sevoflurane, which is a halogenated inhalational anesthetic approved for induction and maintenance of general anesthesia used in adults and pediatric patients for inpatient and outpatient surgery. Sevoflurane is a volatile anesthetic. This drug provides hypnosis, amnesia, analgesia, akinesia, and autonomic blockade during surgical and procedural interventions. Sevoflurane can cause QT prolongation. It can rarely cause increased intracranial pressure. Succinylcholine can cause hyperkalemia. Go to the next page if you knew the correct answer, or click the link image(s) below to further research the concepts in this question (if desired).

Research Concepts: Sevoflurane

We update eBooks quarterly and Apps daily based on user feedback. Please tap flag to report any questions that need improvement.

Question 505: A 16-year-old girl is brought to the emergency department with dizziness, weakness, lack of appetite, and lethargy for three weeks. She further states that lately, she has been craving salt, which is unusual for her. She has a history of asthma. Her mother has type 1 diabetes and Hashimoto thyroiditis. On examination, her blood pressure in the lying position is 105/65 mmHg with a pulse rate of 95/min. On standing, she is noted to have a blood pressure of 85/55 mmHg with a pulse rate of 118/min. Her temperature is 99.8 F. An initial set of investigations is shown below. Patient value Reference range Sodium 126 mEq/L 134-144 mEq/L Potassium 5.9 mEq/L 3.6-5.0 mEq/L Chloride 113 mEq/L 98-107 mEq/L Bicarbonate 21 mEq/L 21-28 mEq/L BUN 43 mEq/L 5-20 mg/dL Creatinine 1.9 mg/dL 0.8-1.4 mg/dL Glucose 50 mg/dL 80–140 mg/dL Urine sodium 62 mEq/L >20 mEq/L She is given appropriate hormonal therapy to treat the underlying pathology. Which of the following is the most appropriate fluid therapy for this patient?

Choices: 1. 5% dextrose in water 2. 5% albumin 3. 0.9% saline 4. Lactated ringer

Answer: 3 - 0.9% saline Explanations: Hyponatremia is defined as a serum sodium concentration of less than 135 mEq/L but can vary to some extent depending upon the set values of varied laboratories. Hyponatremia is a common electrolyte abnormality caused by an excess of total body water in comparison to that of the total body sodium content. The etiology of hyponatremia can be classified based upon the volume status of the extracellular fluid. As mentioned earlier, sodium is the major solute of extracellular fluid (ECF). Based upon the volume of ECF, a patient can be classified into hypovolemic, euvolemic, or hypervolemic. Physiological stimuli that cause vasopressin release in adjunct with increased fluid intake can cause hyponatremia. Hypothyroidism and adrenal insufficiency may contribute to an increased release of vasopressin. Physiological stimuli for vasopressin release include loss of intravascular volume (hypovolemic hyponatremia) and the loss of effective intravascular volume (hypervolemic hyponatremia). Symptoms depend upon the degree and chronicity of hyponatremia. Patients with mild-tomoderate hyponatremia (greater than 120 mEq/L) or a gradual decrease in sodium (greater than 48 hours) have minimal symptoms. Patients with severe hyponatremia (less than 120 mEq/L) or rapid decrease in sodium levels have multiple varied symptoms. Symptoms can range from anorexia, nausea and vomiting, fatigue, headache, and muscle cramps to altered mental status, agitation, seizures, and even coma. This patient's orthostatic hypotension, electrolyte abnormalities, and arranged renal profile indicate adrenal insufficiency, likely Addison disease. In addition to hydrocortisone and fludrocortisone administration, the patient requires normal saline to increase total body volume. Both volume depletion and hormone replacement improve blood pressure and electrolytes. Go to the next page if you knew the correct answer, or click the link image(s) below to further research the concepts in this question (if desired).

Research Concepts: Hyponatremia

We update eBooks quarterly and Apps daily based on user feedback. Please tap flag to report any questions that need improvement.

Question 506: A 65-year-old male presents to the emergency department with diarrhea and muscle weakness. The patient states that for the past 5 days, he has been having difficulty walking around the house. His past medical history is significant for gastroesophageal reflux disease, diabetes mellitus, dyslipidemia, and hypothyroidism. The patient states he does not know what medication he takes but that they are all "over-the-counter." The physical exam is unremarkable. However, lab tests reveal an ionized calcium level of 5.2 mg/dL, magnesium level of 16 mEq/L, and phosphate 4.2 mg/dL. The patient's vital signs are abnormal. However, he is hemodynamically stable. Which of the following abnormalities is likely to be found in vital signs?

Choices: 1. Tachycardia 2. Hypotension 3. Tachypnea 4. Bradycardia

Answer: 2 - Hypotension Explanations: Magnesium toxicity that is uncorrected can eventually cause SA/AV node block and eventually cardiac arrest. However, this patient's vital signs are noted to be abnormal but otherwise stable and magnesium toxicity does not cause bradycardia or tachycardia. The toxic effects of magnesium are inherently linked to the levels (mEq/liters) found in the serum. As the levels of magnesium rise, different symptoms start to manifest and the fatality of those symptoms is proportional to the levels of magnesium found. Starting at 5-10 mEq/L, patients will begin to develop ECG changes (prolonged PR interval, widened QRS). At 10 mEq/L, there will be a loss of deep tendon reflexes and muscle weakness. At 15 mEq/L, signs of abnormal conductivity surface as SA/AV node block. Additionally, patients begin to experience respiratory paralysis. At 20 mEq/L or higher, the patient is likely to experience cardiac arrest. One of the most common vital sign findings in magnesium toxicity is hypotension. This is due to the vasodilatory effect of magnesium and so patients who have elevated levels of magnesium can develop severe hypotension. Go to the next page if you knew the correct answer, or click the link image(s) below to further research the concepts in this question (if desired).

Research Concepts: Magnesium Toxicity

We update eBooks quarterly and Apps daily based on user feedback. Please tap flag to report any questions that need improvement.

Question 507: A 65-year-old male comes in with abdominal distension, anorexia, insomnia, and generalized weakness. He has a history of alcohol use disorder. Two days back, he met his primary care provider who started lactulose, multivitamins, furosemide, decreased his daily protein intake, and performed a small volume paracentesis. His son says that for the past 6 hours, the patient has been drowsy and non-responsive. Physical examination reveals trembling of his hands when held at 90 degrees with his arms fully extended. Which of the following could have been the most likely reason for this patient's sudden deterioration from his baseline?

Choices: 1. Procedure performed to relieve his abdominal distension 2. Reduction of protein from his diet 3. Lactulose 4. Anti-hypertensive medication initiation

Answer: 4 - Anti-hypertensive medication initiation Explanations: This patient with alcohol use disorder presents with hepatic encephalopathy (HE). Triggers of HE include renal failure, gastrointestinal bleeding (e.g., esophageal varices), constipation, infection, medication non-compliance, excessive dietary protein intake, dehydration (e.g., fluid restriction, diuretics, diarrhea, vomiting, excessive paracentesis), electrolyte imbalance, consumption of alcohol, or consumption of certain sedatives, analgesics or diuretics all in the setting of chronic liver disease. This patient was started on furosemide, which was the trigger. In some cases, hepatic encephalopathy may occur following the creation of a transjugular intrahepatic portosystemic shunt (TIPS). Under normal conditions, ammonia is produced by bacteria in the gastrointestinal tract (e.g., a breakdown product of amines, amino acids, purines, and urea) followed by metabolism and clearance by the liver. In the case of cirrhosis or advanced liver dysfunction, however, there is either a decrease in the number of functioning hepatocytes, portosystemic shunting, or both, resulting in decreased ammonia clearance and hyperammonemia. Antibiotics (e.g., rifaximin, neomycin/paromomycin/metronidazole, or vancomycin) are often given empirically due to the frequency of infection as an underlying cause. Additional treatment measures include lactulose/lactitol (a non-absorbable osmotic laxative that also helps convert ammonia to non-absorbable ammonium in the gastrointestinal tract), LOLA (L-ornithine and L-aspartate preparation - increases the use of ammonia in the urea cycle to produce urea), zinc (to correct underlying deficiency common in cirrhotic patients) either alone or in combination with each other or antibiotics. Go to the next page if you knew the correct answer, or click the link image(s) below to further research the concepts in this question (if desired).

Research Concepts: Hepatic Encephalopathy

We update eBooks quarterly and Apps daily based on user feedback. Please tap flag to report any questions that need improvement.

Question 508: A 65-year-old African American woman is being evaluated in the intensive care unit. She presented to the emergency department with an acute left lower quadrant abdominal pain 16 days ago. She has a history of recurrent diverticulitis and underwent an urgent segmental resection with primary anastomosis. The patient started receiving parenteral nutrition with glucose and lipid infusion rates of 7 mg/kg/min and 0.1 g/kg/hour. On postoperative day 13, the patient expresses concern over a new right upper quadrant pain. Vital signs show blood pressure 130/95 mmHg, temperature 96.8 F, and respiratory rate 12/min. AST and ALT are 80 IU/L and 95 IU/L, respectively. Which of the following is the next best step in the management of this patient?

Choices: 1. Decreasing glucose infusion rate 2. Discontinue peripheral parenteral nutrition 3. Discontinue total parenteral nutrition 4. Perform a right upper quadrant ultrasound

Answer: 1 - Decreasing glucose infusion rate Explanations: Increased glucose in the blood induces hepatic lipogenesis. Increased glucose also triggers an increase in the insulin level leading to even more lipogenesis. Increased hepatic lipogenesis can lead to eventual steatosis of the liver. This can be managed by decreasing dextrose dosage to under 5 g/kg day and substituting 30% of dextrose energy with lipids. Substituting 30% of dextrose energy with lipids can decrease the amount of insulin in the blood. Go to the next page if you knew the correct answer, or click the link image(s) below to further research the concepts in this question (if desired).

Research Concepts: Total Parenteral Nutrition

We update eBooks quarterly and Apps daily based on user feedback. Please tap flag to report any questions that need improvement.

Question 509: A 75-year-old American Association Anesthesiology class-III (ASA-III) man with a history of coronary artery disease, chronic renal disease, and obstructive sleep apnea is taken to the operating room for inguinal hernia repair. He had been seen by a cardiologist and cleared for surgery. General anesthesia via laryngeal mask airway (LMA) is considered with maintenance using desflurane. Vital signs are normal prior to pre-op, and induction and placement of the LMA are uneventful. Ten minutes later, during surgical prep, the patient's vital signs show HR 115/min, BP 148/96 mmHg, RR 14/min, SpO2 99%, EtCO2 45 mmHg, and T 98.9 F (37.1 C). The end-tidal desflurane value reads 8.5, and the only other medication administered at this point was cefazolin. What is the most likely cause for the patient's clinical condition?

Choices: 1. Rapid titration of desflurane 2. Malignant hyperthermia 3. Insufficient depth of anesthetic 4. Allergic reaction to perioperative antibiotic administered

Answer: 1 - Rapid titration of desflurane Explanations: Desflurane, especially when rapidly titrated, will stimulate the sympathetic nervous system. A higher flow rate on the anesthesia machine can lead to an increase in the fraction of inspired volatile, which will yield more rapid titration of anesthetic. This rapid change in end-tidal volatile, rather than a slow titration, can increase HR and BP transiently. In this patient with a history of CAD, sympathetic stimulation should be avoided to prevent further ischemia. While desflurane is a malignant hyperthermia triggering agent, it is unlikely given the patient's modest elevation in EtCO2, which is more likely explained by his history of OSA. The elevation in HR and BP are non-specific, and it would be much more likely to malignant hyperthermia with a much more rapid elevation in EtCO2. The non-specific nature of isolated tachycardia and hypertension are much more commonly associated with rapid titration of desflurane as opposed to MH in this scenario. Inadequate depth of anesthesia certainly presents as hypertension and tachycardia. However, it is much less likely in this scenario where the patient has no surgical stimulation yet as the prep is ongoing, and it has been 10 minutes since the stimulation of placing the LMA occurred. In addition, the patient's MAC is > 1, which is more than adequate given the lack of stimulation at this time. It is because of the rapid titration of the volatile agent desflurane that likely led to tachycardia and hypertension. Allergic or anaphylactic reactions are common with steroidal non-depolarizing neuromuscular blocking drugs as well as perioperative antibiotics. However, it is much more commonly associated with hypotension, tachycardia, and increased peak airway pressures. The etiology of this patient's tachycardia and hypertension are much more likely caused by sympathetic stimulation from desflurane. The mechanism by which the sympathetic stimulation occurs is not fully understood, but rapid titration tends to increase the incidence of stimulation. Go to the next page if you knew the correct answer, or click the link image(s) below to further research the concepts in this question (if desired).

Research Concepts: Desflurane

We update eBooks quarterly and Apps daily based on user feedback. Please tap flag to report any questions that need improvement.

Question 510: A 16-years-old male presents with sudden onset of severe headache, lowgrade fever, and diplopia. A few days ago he underwent surgical drainage for a furuncle on his midface. On examination, there is a sick looking young male with periorbital edema, chemosis, lateral gaze palsy, and ptosis on the right side. On fundoscopy, there is bilateral disc edema. Lumbar puncture reveals elevated cerebrospinal fluid (CSF) opening pressure. What is the most appropriate initial management option?

Choices: 1. Broad-spectrum antibiotics 2. Low molecular weight heparin 3. Corticosteroids 4. Immediate neurosurgical intervention

Answer: 1 - Broad-spectrum antibiotics Explanations: This is the case of cavernous sinus thrombosis secondary to staphylococcus aureus infection. The patient has developed nerve palsies and raised intracranial pressure. It is advised to start broad-spectrum antibiotics initially. Low molecular weight heparin and neurosurgical intervention have controversial roles. When there is raised intracranial pressure, it is prudent to keep the head elevated to 30 degrees and neutrally positioned to minimize venous outflow resistance and improve cerebral spinal fluid displacement from the intracranial to the spinal compartment. Hypoxia and hypercapnia can increase intracranial pressure (ICP). Controlling ICP through optimal respiratory management is crucial. It is essential to control ventilation to maintain a normal PaCO2 and maintain adequate oxygenation without increasing the positive endexpiratory pressure (PEEP). Go to the next page if you knew the correct answer, or click the link image(s) below to further research the concepts in this question (if desired).

Research Concepts: Intracranial Hypertension

We update eBooks quarterly and Apps daily based on user feedback. Please tap flag to report any questions that need improvement.

Question 511: A 16-year-old boy is being evaluated in the intensive care unit on the fourth day of admission due to pneumonia. He has been requiring multiple sedatives and analgesics while on a pressure-assist ventilator. Due to ongoing agitation, the pharmacist warns an intensivist about the maximum dose of sedatives and opioid use. A plan is made to change the ventilator mode to neurally-adjusted ventilatory assist (NAVA). Following the switch, arterial blood gas analysis shows pH 7.42, pCO2 39 mmHg, and pO2 150 mmHg. He is currently on NAVA of 2 cmH2O/microV and Edi max is 8 microV. Which of the following is the next best step in the management of this patient?

Choices: 1. Decrease NAVA by 0.5 cmH2O/microV 2. Extubate patient now 3. Increase NAVA level by 3 cmH2O/microV 4. Change back to the pressure-assist ventilator

Answer: 1 - Decrease NAVA by 0.5 cmH2O/microV Explanations: As NAVA improves asynchrony during ventilation, it may be helpful in decreasing agitation and lower the need for sedative medications. Arterial or capillary blood gas is an important tool to monitor ventilator requirements. If blood gas is acceptable, NAVA level can be weaned by 0.5-1 cmH2O/microV. It is recommended to consider extubation once the NAVA level reaches 1cmH2O/microV. Go to the next page if you knew the correct answer, or click the link image(s) below to further research the concepts in this question (if desired).

Research Concepts: Neurally Adjusted Ventilatory Assist (NAVA)

We update eBooks quarterly and Apps daily based on user feedback. Please tap flag to report any questions that need improvement.

Question 512: A 65-year-old woman comes to the emergency department because of acute onset shortness of breath and coughing up blood for the past 5 hours. She has hypertension and type 2 diabetes mellitus. She also has a history of developing thrombocytopenia after using an anticoagulant for deep vein thrombosis. On examination, she appears anxious, her pulse is 112 beats/min, and respirations are 24 breaths/min, blood pressure 110/60 mmHg. She has normal breath sounds with no wheezing on auscultation of the chest. Her chest x-ray shows no abnormalities, but her ventilation-perfusion scan shows a small defect in the right middle lobe. The drug most likely to be used for the patient acts on which of the following substances?

Choices: 1. Glycoprotein IIb/IIIa 2. Factor VII 3. Antithrombin 4. Factor Xa

Answer: 4 - Factor Xa Explanations: Novel oral anticoagulants (NAOCs) like rivaroxaban are direct factor Xa inhibitors and can be used as anticoagulants for acute pulmonary embolism and also continued for prevention of recurrent venous thromboembolic events. This is an appropriate choice of anticoagulant in this patient with a history of heparininduced thrombocytopenia where heparin is contraindicated. NOACs are becoming a safe and effective choice of oral anticoagulant in the management of hemodynamically stable patients with acute pulmonary embolism without shock or hypotension. NOACs like rivaroxaban offer a convenient and cost-effective single-drug therapeutic approach. The ease of not requiring frequent clinical appointments for lab monitoring and route of administration makes management more convenient and enables earlier discharge and outpatient treatment in low-risk patients. Warfarin is not the drug of choice for acute pulmonary embolism because the onset of action is typically 24 - 72 hours, and peak therapeutic effect is only seen 5 to 7 days after initiation. Heparin acts by indirectly inactivating thrombin and activated factor X (factor Xa) through binding with antithrombin to enhance its activity and is the initial treatment of choice for most patients with acute pulmonary embolism but given the history of heparininduced thrombocytopenia in this patient, this drug is contraindicated. Antiplatelet drugs like abciximab act as glycoprotein IIb/IIIa inhibitor preventing platelet aggregation and are used during percutaneous coronary intervention. Go to the next page if you knew the correct answer, or click the link image(s) below to further research the concepts in this question (if desired).

Research Concepts: Rivaroxaban

We update eBooks quarterly and Apps daily based on user feedback. Please tap flag to report any questions that need improvement.

Question 513: A 35-year-old female with a past medical history of asthma presented to the emergency department complaining of severe shortness of breath. Patient states since the past week she wasn't feeling well. Seven days ago she first started to have a runny nose and dry cough progressively the symptoms have worsened. She started having shortness of breath which progressively got to the point she was not able to talk 2 words without getting short of breath. She denied any fevers, chills but states she has racing of the heart. She denied any sick contacts. The patient uses an albuterol inhaler as needed for asthma but did not help with her symptoms this time. In the emergency department, her blood pressure was 140/90, heart rate of 140, pulse ox showed 95% on room air. On physical exam, the patient was tachypneic with a bilateral expiratory wheeze, prolonged expiratory phase, tachycardic with an irregular rhythm. A chest xray did not show any significant cardio-thoracic process. EKG showed finding suggestive of multifocal atrial tachycardia. The patient was started on breathing treatments with Lev-albuterol nebulization treatments, IV glucocorticoids were initiated. After a few minutes of treatment the patient's respiratory rate is 23, a heart rate of 135, pulse-ox showed 95% on room air appears to be in moderate discomfort due to her breathing but she denied any tiredness. An arterial blood gas sample was obtained which showed respiratory alkalosis. what is the next step of management?

Choices: 1. Place the patient on Non-invasive positive pressure ventilator support 2. Start patient of albuterol-ipratropium nebulization treatments 3. Start intravenous diltiazem 4. Start intravenous magnesium

Answer: 4 - Start intravenous magnesium Explanations: Non-invasive positive pressure ventilation is not indicated yet. The patient only received initial treatment for asthma exacerbation and needs some time to optimize treatment for her asthma exacerbation. The patient has moderate discomfort due to her breathing but did not appear tired. Her Arterial blood sample showed respiratory alkalosis. The concern should arise in this patient if her PCO2 normalizes or if it gets higher than 45 which indicates respiratory muscle fatigue which can lead to respiratory failure. To prevent respiratory muscle fatigue non-invasive positive pressure ventilation can be used but is premature in this case. In the above patient inhaled albuterol and ipratropium can be started but it increases the patient heart rate even higher. The patient is already on levalbuterol inhalation at this time albuterol/ipratropium inhalation is not indicated. Heart rate control with intravenous diltiazem is not indicated at this time. Asthma exacerbation is indicated before heart rate control. Intravenous magnesium is indicated at this time for the treatment of asthma exacerbation. Studies have shown Magnesium replacement to help to reduce atrial and ventricular ectopic activity. Go to the next page if you knew the correct answer, or click the link image(s) below to further research the concepts in this question (if desired).

Research Concepts: Multifocal Atrial Tachycardia

We update eBooks quarterly and Apps daily based on user feedback. Please tap flag to report any questions that need improvement.

Question 514: A 19-year-old previously healthy camp counselor was admitted to the hospital with two days history of headache and low-grade fever followed by unresponsiveness and generalized seizure. He underwent a lumbar puncture, which showed an increase in cell count and proteins, but it was negative for gram stain and bacterial culture. He was treated with acyclovir, vancomycin, and ceftriaxone. In spite of therapy and supportive management, he died on the second day in the intensive care unit. Autopsy revealed suppurative meningoencephalitis and necrotic tissue, which showed trophozoites of protozoa. What is the most likely route for this patient to have acquired this form of meningoencephalitis?

Choices: 1. Diving or swimming in contaminated water 2. Intravenous drug abuse 3. Using human excrement as vegetable fertilizer 4. Eating raw fish or seafood

Answer: 1 - Diving or swimming in contaminated water Explanations: This is a case of amebic meningoencephalitis. Contaminated water can cause infection with Acanthamoeba or Naegleria which may develop into meningoencephalitis. The organism is thought to acquire access to the CNS through the cribriform plate. Human excrement also known as night soil can be a source of parasitic worm infections, such as ascariasis because the helminth eggs are in feces and can thus be transmitted from one infected person to another person (fecal-oral transmission of disease). Major types of food poisoning that can result from eating raw or undercooked fish and shellfish include Salmonella and Vibrio vulnificus. Go to the next page if you knew the correct answer, or click the link image(s) below to further research the concepts in this question (if desired).

Research Concepts: Amebic Meningoencephalitis

We update eBooks quarterly and Apps daily based on user feedback. Please tap flag to report any questions that need improvement.

Question 515: A 17-year-old male driver is brought in by emergency medical services after a rollover motor vehicle collision. He is hemodynamically stable with a Glasgow coma scale of 15 and denies loss of consciousness. He has a significant crush injury to his right leg, and his lab work is notable for acute kidney injury, hyperkalemia, hypocalcemia, and hyperphosphatemia. Which of the following is the most potent treatment for his hyperphosphatemia?

Choices: 1. Calcium acetate 2. Sevelamer 3. Aluminum hydroxide 4. Continuous dialysis

Answer: 4 - Continuous dialysis Explanations: While aluminum hydroxide is a potent hypophosphatemic, it is not as effective as continuous dialysis. Aluminum hydroxide should be used with caution in the setting of acute kidney injury. Intermittent dialysis is not as potent at phosphate removal as continuous dialysis. The gold standard for phosphate clearance is continuous hemodialysis. Go to the next page if you knew the correct answer, or click the link image(s) below to further research the concepts in this question (if desired).

Research Concepts: Aluminum Hydroxide

We update eBooks quarterly and Apps daily based on user feedback. Please tap flag to report any questions that need improvement.

Question 516: Which diagnostic test should be ordered in the case of a 45-year-old male who works as a painter and is found obtunded at work? The head CT, urine toxicology, and ethanol are all negative. Lab values reveal Na 136 mEq/L, K 5.4 mEq/L, Cl 111 mEq/L, HCO3 11 mEq/L, BUN 16 mg/dL, Creatinine 1.4 mg/dL, Glucose 90 mg/dL, ABG: PO2 88, PCO2 25, and pH 7.18.

Choices: 1. Urine ketones 2. Serum sodium 3. Echocardiogram 4. Plasma osmolality

Answer: 4 - Plasma osmolality Explanations: The patient has a compensated high anion gap metabolic acidosis and with signs of intoxication. The next step is to measure the plasma osmolality and compare it to the estimated osmolality which is 287 (2Na + (BUN/2.8) + (glucose/18) + (blood ethanol/4.6)) to determine the osmolar gap (measured osmolality-calculated osmolality=osmolar gap). This, in normal cases, is less than 10. A higher gap is due to low molecular weight molecules such as ethylene glycol, methanol, acetone, or ketones that are not accounted for in the equation. Methanol is used in paint thinners and is a likely culprit in this case. Ketoacidosis is a cause of anion gap metabolic acidosis, but given the normal blood glucose in this case, is unlikely. Go to the next page if you knew the correct answer, or click the link image(s) below to further research the concepts in this question (if desired).

Research Concepts: Methanol Toxicity

We update eBooks quarterly and Apps daily based on user feedback. Please tap flag to report any questions that need improvement.

Question 517: A 68-year-old woman with a past medical history of alcohol use disorder and hypertension is admitted to the hospital with a diagnosis of grade 3 subarachnoid hemorrhage (Hunt and Hess scale). Few hours following the admission, the patient complains of shortness of breath. A chest x-ray is notable for pulmonary edema. EKG shows for ST-segment elevation in the anterolateral leads. There is no reciprocal ST depression. Troponin T is mildly elevated. An echocardiogram is notable for akinesis of the left ventricle apical segments. Which of the following is the most likely cause of the patient's current presentation?

Choices: 1. Irritation of pericardium 2. Coronary artery disease 3. Catecholamine surge 4. Alcohol use

Answer: 3 - Catecholamine surge Explanations: This scenario describes a case of Takotsubo cardiomyopathy following subarachnoid hemorrhage (SAH). Takotsubo cardiomyopathy, also known as Gebrochenes-Herz syndrome, transient apical ballooning syndrome, apical ballooning cardiomyopathy, stress-induced cardiomyopathy, stress cardiomyopathy, or broken-heart syndrome, is a form of non-ischemic cardiomyopathy. It is characterized by transient regional systolic dysfunction of the left ventricle mimicking acute myocardial infarction but with only minimal release of cardiac enzymes. The cause of Takotsubo cardiomyopathy is due to a catecholamine surge following aneurysm rupture in this patient. The most widely accepted criteria are the Mayo Clinic diagnostic criteria for the identification of stress cardiomyopathy. From the following key features, all are required to meet the diagnosis: 1. Transient hypokinesis, akinesis, or dyskinesis in the left ventricular mid segments with or without apical involvement; regional wall motion abnormalities that extend beyond. 2. A single epicardial vascular distribution; and frequently, but not always, a stressful trigger. 3. The absence of obstructive coronary disease or angiographic evidence of acute plaque rupture. 4. New EKG abnormalities (ST-segment elevation and/or T-wave inversion) or modest elevation in cardiac troponin. 5. The absence of pheochromocytoma and myocarditis. Go to the next page if you knew the correct answer, or click the link image(s) below to further research the concepts in this question (if desired).

Research Concepts: Subarachnoid Hemorrhage

We update eBooks quarterly and Apps daily based on user feedback. Please tap flag to report any questions that need improvement.

Question 518: A 68-year-old woman is brought to the emergency department by an ambulance after a motor vehicle collision. She is unconscious, and a neurological examination is not possible. Her past medical history is significant for hypertension, coronary artery disease, atrial fibrillation on warfarin, and congestive heart failure. CT scan confirms the presence of a subdural hematoma. Which of the following is the next best step in the management of this patient?

Choices: 1. Prothrombin complex concentrate 2. Levetiracetam 3. Mannitol 4. Intravenous fluids

Answer: 1 - Prothrombin complex concentrate Explanations: Warfarin greatly worsens the bleeding associated with a head injury. One of the first priorities is to reverse the effects of the patient's anticoagulation rapidly. The best medication to immediately start to reverse the warfarin is prothrombin complex concentrate. Fresh frozen plasma can also be given, but prothrombin complex concentrate is preferred. Go to the next page if you knew the correct answer, or click the link image(s) below to further research the concepts in this question (if desired).

Research Concepts: Geriatric Head Injury

We update eBooks quarterly and Apps daily based on user feedback. Please tap flag to report any questions that need improvement.

Question 519: A 65-year-old male with end-stage renal disease (ESRD) on hemodialysis complains of right-sided neck pain during his routine outpatient treatment. On examination, he is afebrile with BP 145/90 mmHg, HR 92 bpm. A right internal jugular tunneled hemodialysis catheter is noted. The exit site has purulent drainage and erythema that extends 4 cm cephalad along the subcutaneous tunnel. The catheter cuff is not exposed. On palpation, there is induration and tenderness over the subcutaneous tunnel. A blood sample drawn from the hemodialysis circuit shows a leucocyte count of 13,000/microL with 80% neutrophils and a normal platelet count. Which of the following is the most appropriate management strategy for this patient?

Choices: 1. Culture exit site discharge and do not remove the catheter 2. Culture exit site discharge and remove the catheter 3. Culture exit site discharge and exchange catheter over the guidewire 4. Culture blood and remove the catheter only if evidence of bacteremia

Answer: 2 - Culture exit site discharge and remove the catheter Explanations: The patient has evidence of both exit-site infection and tunnel infection of the tunneled hemodialysis catheter. According to the Infectious diseases society of America (IDSA), tunnel infection is defined as tenderness, hyperemia, or induration that extends >2 cm from the exit site and along the subcutaneous tunnel. It may or may not be associated with bacteremia. If there is purulent drainage, it should be collected and sent for gram staining and culture. The catheter should always be removed in the setting of tunnel infection. Exit site infections can be treated with topical antibiotics in the absence of systemic signs of infection or sepsis. However, tunnel infections require parenteral antibiotics and catheter removal due to the higher risk of bacteremia. Catheter exchange over a wire should be done. After the removal of the catheter, a new catheter must be inserted at an alternate site. Blood cultures should be drawn before starting empiric broad-spectrum antibiotics to cover both gram-positive and gram-negative organisms. Based on culture and sensitivity results, the antibiotic regimen can be titrated. If a catheter-related bloodstream infection (CRBSI) is also present, then the duration of treatment will be determined by the clinical presentation, complications, microorganism isolated, and vascular access options of the patient. Go to the next page if you knew the correct answer, or click the link image(s) below to further research the concepts in this question (if desired).

Research Concepts: Central Line Associated Blood Stream Infections

We update eBooks quarterly and Apps daily based on user feedback. Please tap flag to report any questions that need improvement.

Question 520: A 65-year-old woman has deep vein thrombosis extending from the thigh to the common femoral vein. Overnight catheter-directed therapy with tissue plasminogen activator is prescribed. No heparin is administered. The fibrinogen level is monitored. What is the next step in management if bleeding occurs at the groin puncture site?

Choices: 1. Administer fresh frozen plasma 2. Administer vitamin K 3. Administer aminocaproic acid 4. Administer protamine

Answer: 3 - Administer aminocaproic acid Explanations: Aminocaproic acid is a derivative of lysine and an inhibitor of certain proteolytic enzymes. Aminocaproic acid can treat excessive fibrinolysis. Side effects of aminocaproic acid include nausea, fever, and vomiting. Aminocaproic acid can also cause inflammation of the liver and appendix. Go to the next page if you knew the correct answer, or click the link image(s) below to further research the concepts in this question (if desired).

Research Concepts: Thrombolytic Therapy

We update eBooks quarterly and Apps daily based on user feedback. Please tap flag to report any questions that need improvement.

Question 521: A 52-year-old man presents to the clinic with a nonhealing ulcer on his left foot for the past two weeks. He has a history of alcoholic cirrhosis, for which he received a liver transplant two years ago. Physical examination shows a cold maculopapular lesion with an ulcerated center or a violaceous nodular lesion. A swab from the ulcer does not show any bacterial growth after 72 hours. Three days later, the patient develops fever and a headache. A lumbar puncture analysis performed is shown below. Patient findings Normal range Fibrin webs Appearance Clear visible Protein 0.7 g/L 0.1-0.4 g/L Glucose 37 mg/dL 40-70 mg/dL White cell 70 lymphocytes 0-10 cells count Gram stain Negative Negative Which of the following tests is most likely to confirm the diagnosis in this patient?

Choices: 1. KOH preparation of the ulcer swab 2. Blood cultures 3. Serum polysaccharide antigen 4. MRI

Answer: 3 - Serum polysaccharide antigen Explanations: The clinical scenario describes an immunocompromised patient with possible cyptococcus infection. Cryptococcosis is the third most common invasive fungal infection in organ transplant recipients, after candidiasis and aspergillosis. Cutaneous manifestations can be protean, may mimic an abscess, present as cellulitis that later ulcerates, blisters, and shows necrosis, or mimic panniculitis. The polysaccharide capsule antigen is a large molecule, and assays have been developed to detect the antigen in serum and other body fluids. If cryptococcal polysaccharide antigen (CA) is present in cerebrospinal fluid (CSF), then there is a good chance that the yeast is present as well. The cryptococcal polysaccharide antigen assay is nearly 100% sensitive and 96% to 99.5% specific in serum and 96% to 100% sensitive and 93.5% to 99.8% specific in CSF. Go to the next page if you knew the correct answer, or click the link image(s) below to further research the concepts in this question (if desired).

Research Concepts: Cutaneous Cryptococcus

We update eBooks quarterly and Apps daily based on user feedback. Please tap flag to report any questions that need improvement.

Question 522: A 45-year-old man with adult respiratory distress syndrome secondary to acute pancreatitis is admitted to the intensive care unit for septic shock. The patient is on maximal doses of norepinephrine and vasopressin and thus was started on angiotensin II following which his blood pressure improved, and norepinephrine and vasopressin requirements have come down. The patient is on day two of angiotensin II therapy. Each time the angiotensin II is weaned, the patient's blood pressure drops. According to the hospital policy, this agent cannot be used after two days. Which of the following is the next best step in the management of this patient?

Choices: 1. Continue using angiotensin II 2. Stop the angiotensin II 3. Consult the pharmaceutical manufacturer for advice 4. Palliative care consult

Answer: 1 - Continue using angiotensin II Explanations: Angiotensin II originally studied for only two days however certain patients in the ATHOS3 trial were very dependent on this agent and continue to be on that for seven days. The maximum dose of this agent for 7 days Is 80 nanograms per kg per minute. Any use beyond seven days, this agent would fall under the experimental realm. It potentially may be unethical to stop this agent on a dependent patient in 2 days as originally designed in ATHOS 3 trial; thus, a continuation of this agent for 7 days was allowed. Go to the next page if you knew the correct answer, or click the link image(s) below to further research the concepts in this question (if desired).

Research Concepts: Angiotensin II

We update eBooks quarterly and Apps daily based on user feedback. Please tap flag to report any questions that need improvement.

Question 523: A 66-year-old woman isis admitted to the intensive care unit after undergoing liver transplantation. She remains on a mechanical ventilator postoperatively. She lives alone and has a history of alcohol use disorder and chronic liver disease. Her body mass index is 17 kg/m2. Postoperatively, the patient has a hemoglobin of 10 g/dL, leukocyte count 6000/microL, platelet count 60000/microL, BUN 8 mg/dL, and international normalized ratio (INR) of 4.0. Which of the following is the most suitable for prophylaxis against peptic ulceration in this patient?

Choices: 1. Sucralfate 2. Histamine-2-receptor antagonist 3. Proton pump inhibitor 4. Aluminum hydroxide

Answer: 2 - Histamine-2-receptor antagonist Explanations: Ventilator bundles typically consist of a constellation of various preventative strategies to reduce the risk of ventilator-associated complications. There is no ideal ventilator bundle. They may differ across hospitals, regions, and countries. However, the primary preventative strategies are mostly similar across different institutions. Mechanically ventilated patients are at risk for gastric ulceration due to their predisposition to aerobic Gram-negative bacterial (AGNB) colonization of the stomach. Therefore, peptic ulcer prophylaxis may be administered as part of a ventilator bundle of strategies aimed at preventing ventilator-associated complications. Upper gastrointestinal (GI) bleeding is a known complication of gastric ulceration. In the assessment of mechanically ventilated patients, it is important to identify risk factors for clinically significant upper GI bleeding (including vitamin K deficiency, coagulopathy or bleeding diatheses) to guide the appropriate choice of agent for ulcer prophylaxis. In this scenario, the patient is malnourished (as evidenced by her body mass index of 17 kg/m2) and has a history of alcohol use disorder. It is likely she may have a deficiency of vitamin K. Due to her history of chronic liver disease, she may likely have a clotting factor deficiency as a result of impairment in synthetic hepatic function. This is correlated by her high international normalized ratio (INR) of 4. She also has thrombocytopenia. These aspects indicate coagulopathy. Sucralfate is often preferred for use in mechanically ventilated patients as it protects against gastric ulceration without increasing the gastric pH. Although histamine-2-receptor antagonists (H2-RA) have the effect of increasing gastric pH, which is unfavorable because an increased gastric pH promotes colonization with aerobic Gram-negative bacteria, mechanically ventilated patients with risk factors for severe upper GI bleeding, including those with coagulopathy, should receive prophylaxis with an H2-RA rather than sucralfate. Go to the next page if you knew the correct answer, or click the link image(s) below to further research the concepts in this question (if desired).

Research Concepts: Ventilator Complications

We update eBooks quarterly and Apps daily based on user feedback. Please tap flag to report any questions that need improvement.

Question 524: A 16-year-old female is admitted to the ICU for ventilator management after developing respiratory failure secondary to benzodiazepine use to control agitation. She presented to the emergency department yesterday after an intentional overdose of citalopram. Which laboratory test confirms the diagnosis?

Choices: 1. Serum lactate level 2. Urine drug screen 3. Serum ammonia level 4. No specific test, this is a diagnosis of exclusion

Answer: 4 - No specific test, this is a diagnosis of exclusion Explanations: Serotonin toxicity is a clinical diagnosis initially defined by the Sternbach criteria. Due to the nonspecific nature of the Sternbach criteria, the Hunt serotonin toxicity criteria were developed. Besides signs and symptoms, a history suggestive of serotonin exposure or overdose is required to diagnose serotonin toxicity. Diagnosis of serotonin toxicity is not only clinical but also a diagnosis of exclusion, therefore, even though diagnosis does not rely on any specific laboratory value, thorough testing to rule out other causes of altered mental status is essential. Go to the next page if you knew the correct answer, or click the link image(s) below to further research the concepts in this question (if desired).

Research Concepts: Selective Serotonin Reuptake Inhibitor Toxicity

We update eBooks quarterly and Apps daily based on user feedback. Please tap flag to report any questions that need improvement.

Question 525: A 36-year-old male is brought to the emergency department with hypotension with a blood pressure of 90/60 mmHg, tachycardia with a heart rate of 122/min, and tachypnea with a respiratory rate of 30/min. Pulse oximetry reveals a peripheral capillary oxygen saturation of 100% percent on room air. The patient's skin appears cherry red, and he appears to have soot in his nose and mouth area. His pupils are dilated, and he appears intoxicated. Initial investigations showed sodium levels of 146 mEq/L, potassium levels of 5.5 mEq/L, bicarbonate levels of 12 mEq/L, blood urea nitrogen 8 mg/dL, creatinine 1.96 mg/dL, and anion gap 32.6. His total leucocyte count was 12.9 mg/dL, hemoglobin 13.2 mg/dL, and a platelet count of 106000/mircoliter. Arterial blood gases included a pH of 6.67, carbon dioxide 84 mmHg, PO2 of 157 mmHg, and bicarbonate levels at 9.7 mEq/L. Liver function tests revealed aspartate transaminase levels of 151 U/L. His creatinine kinase is 2095 U/L, and ammonia is 23 U/L. The plasma lactate level was 16.0 mmol/L. Which of the following blood report of the patient have the highest sensitivity in diagnosing the patient?

Choices: 1. A raised anion gap metabolic acidosis 2. Plasma lactate levels of greater than 8mmol/L 3. Raised liver function tests 4. Raised total leucocyte count

Answer: 2 - Plasma lactate levels of greater than 8mmol/L Explanations: Cyanide poisoning presents with bradycardia, hypertension, tachypnea, cherry-red skin, and soot around the face. Pulse oximetry can be high and falsely reassuring. Cyanide can block aerobic metabolism, causing oxygen delivered to cells to go unused. Lactic acidosis can result from the shift from aerobic to anaerobic metabolism. Metabolic acidosis with a reduced arteriovenous difference suggests the diagnosis. Plasma lactate levels greater than eight mmol/L is 94% sensitive and 70% specific for significant cyanide toxicity. Aggressive therapy with oxygen is needed. Cyanide antidotes include hydroxocobalamin. Cyanocobalamin combines with cyanide and allows the kidneys to excrete it. Raised total leukocyte count can also be in infections. Liver function tests are increased in multiple diseases of the liver which is not specific. Go to the next page if you knew the correct answer, or click the link image(s) below to further research the concepts in this question (if desired).

Research Concepts: Cyanide Toxicity

We update eBooks quarterly and Apps daily based on user feedback. Please tap flag to report any questions that need improvement.

Question 526: A 65-year-old male with an unknown past medical history is brought to the emergency department with shortness of breath and lethargy. The patient is found to be confused and hypoxic, with an O2 saturation of 65%. The basic metabolic panel includes sodium 140 mEq/L, potassium 5.8 mEq/L, chloride 100 mEq/L, bicarbonate 19 mEq/L, blood urea nitrogen 65 mg/dL, creatinine 3.2 mg/dL, and blood glucose 132 mg/dL. The patient weighs 100 kilograms. The clinician decides to perform emergency intubation. What induction and paralytic medications should be used?

Choices: 1. Rocuronium 100 mg IV followed by etomidate 30 mg IV 2. Propofol 50 mg IV followed by rocuronium 100 mg IV 3. Etomidate 30 mg IV followed by succinylcholine 100 mg IV 4. Etomidate 30 mg IV followed by rocuronium 100 mg IV

Answer: 4 - Etomidate 30 mg IV followed by rocuronium 100 mg IV Explanations: This patient presents with acute hypoxic respiratory failure requiring rapid sequence intubation. The patient has an anion gap metabolic acidosis as well as an acute kidney injury, causing hyperkalemia. The proper dose of etomidate is 0.1-0.6 mg/kg, and the proper dose of rocuronium is 0.6-1.2 mg/kg, etomidate 30 mg IV, followed by rocuronium 100 mg IV, is the proper dosing and medications for this patient. Paralytic agents should always be given after induction agents to avoid paralyzing a conscious patient. Due to the hyperkalemia, succinylcholine is contraindicated. The proper dosing of propofol as an induction agent is 1.5-2.5 mg/kg; therefore, 50 mg would be an inadequate dosage. Go to the next page if you knew the correct answer, or click the link image(s) below to further research the concepts in this question (if desired).

Research Concepts: Tracheal Rapid Sequence Intubation

We update eBooks quarterly and Apps daily based on user feedback. Please tap flag to report any questions that need improvement.

Question 527: A previously healthy 34-year-old male is in the intensive care unit due to a traumatic brain injury. On physical examination, he is unarousable to painful stimuli, pupils are unresponsive to light bilaterally, and gag reflex testing elicits no response. Diagnostic testing is completed to confirm brain death. Most recent laboratory data include a white blood cell count of 8300/mm3 and a hemoglobin of 12.2 g/dL. He is taken to the operating room for donation of select organs. Which of the following is the best next step for a successful organ donation?

Choices: 1. Inject organs with sirolimus to reduce immunogenicity 2. Cool organs to 4 C (39.3 F) to reduce warm ischemia damage 3. Store organs at 37 C (98.6 F) until transplanted into the host 4. Store organs at 0 C (32 F)

Answer: 2 - Cool organs to 4 C (39.3 F) to reduce warm ischemia damage Explanations: The most common type of injury to organs after procurement is warm ischemia damage. This includes a rush of inflammatory markers and free radicals, which can adversely affect the transplantation process. In an effort to reduce warm ischemia injury, organs are rapidly cooled to 4 C (39.3 F). It is important to note that cellular processes are not halted completely but rather delayed. Another method to reduce warm ischemia damage is flushing with an appropriate preservation solution. This leads to a decreased immune reaction and the formation of oxygen-free radicals upon reperfusion. Warm ischemia times currently tolerated are less than 30 minutes. Storge at a temperature below 4 C can lead to protein denaturation. Go to the next page if you knew the correct answer, or click the link image(s) below to further research the concepts in this question (if desired).

Research Concepts: Tissue and Organ Donation

We update eBooks quarterly and Apps daily based on user feedback. Please tap flag to report any questions that need improvement.

Question 528: A 32-year-old patient with post-traumatic diffuse brain edema was being monitored clinically by the placement of the intraventricular intracranial pressure (ICP) monitoring system. Suddenly his attending nurse sees a gradual rise in the P2 wave going above the P1 and P3 waves as visualized in the ICP waveform analysis. There is also a fall in his motor score from M5 (localizing) to M3 (flexing.) Which of the following is the appropriate immediate next step in the management of the patient?

Choices: 1. Prepare for decompressive hemicraniectomy 2. Take the patient for an emergency repeat CT scan 3. Paralyze and sedate the patient 4. Lower down the external ventricular drain (EVD) below the level of the tragus

Answer: 4 - Lower down the external ventricular drain (EVD) below the level of the tragus Explanations: The clinical scenario, as depicted in the ICP waveform analysis, is typical of rising ICP pressure, and therefore, there is an urgent need for intervention to reduce the same. The intraventricular ICP monitoring system in addition to being the gold standard for measuring the same, allows for the immediate release of CSF, thereby counteracting any rise in ICP. So the best immediate plan of management would be to allow for egress of CSF by lowering the transducer and the EVD bag below the level of the tragus (external auditory canal). After combating this sudden rise in ICP only, we can ask for other interventions like repeating CT scans and preparing for damage control surgery. Go to the next page if you knew the correct answer, or click the link image(s) below to further research the concepts in this question (if desired).

Research Concepts: Intracranial Pressure Monitoring

We update eBooks quarterly and Apps daily based on user feedback. Please tap flag to report any questions that need improvement.

Question 529: A 55-year-old man with a history of hypertension and diabetes presents to the emergency department after 1 day of fever and rigors. The patient's vital signs are a temperature of 102.4 F, a heart rate of 130/min, respiratory rate 25/min, blood pressure 90/50 mmHg, and SpO2 90% on room air. The patient's physical exam reveals an ill-appearing, diaphoretic, and dyspneic male with signs of poor peripheral perfusion. The patient is presumed to be in septic shock, and supplemental oxygen is administered, a peripheral IV is secured, antibiotics are given, and blood drawn for lab tests. An arterial blood gas is performed, while the patient is on room air, with the following results pH of 7.3, paCO2 of 30 mmHg, paO2 of 60 mmHg, HCO3 of 12 mEq/L and SaO2 of 89%. Which of the following is the most accurate interpretation of these results?

Choices: 1. Primary respiratory acidosis with compensation 2. Primary metabolic acidosis with compensation 3. Primary respiratory acidosis without compensation 4. Primary metabolic acidosis without compensation

Answer: 2 - Primary metabolic acidosis with compensation Explanations: Using 7.40 as a cutoff point, the pH is not in the normal range, in which case it is 7.40, so acidosis is present. The PaCO2 is decreased, indicating a respiratory alkalosis, and the HCO3 is decreased, indicating a metabolic acidosis. The value consistent with the pH is the HCO3. Therefore, this is a primary metabolic acidosis. The acid-base that is inconsistent with the pH is the paCO2, as it is decreased, indicating a respiratory alkalosis. Therefore, there is compensation in response to the primary disorder. However, it is ineffective as the pH is still low despite the compensatory response. Go to the next page if you knew the correct answer, or click the link image(s) below to further research the concepts in this question (if desired).

Research Concepts: Arterial Blood Gas

We update eBooks quarterly and Apps daily based on user feedback. Please tap flag to report any questions that need improvement.

Question 530: A 66-year-old man is brought to the emergency department after a motor vehicle accident. He complains of severe left flank abdominal pain. His vital signs reveal blood pressure 90/60 mmHg, heart rate 110/min, and respiratory rate 22/min. He has a past medical history of hypertension, colon cancer diagnosed five months ago, and pulmonary embolism four months ago. He is currently on losartan 50 mg daily and rivaroxaban 15 mg/twice daily. He is compliant with his medication and took it five hours ago before the car accident. The surgeon on call reviews the CT scan and decides to take the patient to the operating room immediately for exploratory laparotomy. What is the most appropriate management strategy to reverse anticoagulation in this patient?

Choices: 1. Vitamin K 2. Idarucizumab 3. Prothrombin complex concentrate 4. Andexanet-alfa

Answer: 4 - Andexanet-alfa Explanations: The FDA approved andexanet-alfa as a specific agent for emergency reversal of patients on rivaroxaban and apixaban anticoagulation therapy. The dose of andexanet-alfa varies depending on the milligrams of rivaroxaban or apixaban that the patient is receiving daily and the time since the last dose. In this case, the previous dose of the patient was 5 hours ago, and he is taking 15 mg of rivaroxaban. By fabricant recommendations, the andexanet dose will be 800 mg infusion. Andexanet-alfa is a recombinant factor Xa, and the onset of action is evident two minutes after starting the infusion. For vitamin K peak of its pharmacological action is around 12-14 hours after administration. As this case is spleen trauma requiring emergent damage control surgery, vitamin K will not reverse the anticoagulation of the patient in the needed time. Additionally, vitamin K is widely used as a specific reversal agent for patients on warfarin with higher INR that does not have significant bleeding. The FDA approved idarucizumab as a specific antidote for dabigatran. This therapy would work in this case. Prothrombin complex concentrate was the first line of treatment for patients on DOACs that required reversal of anticoagulation for many years. However, per recommendations of ASRA guidelines, this first option should be andexanet-alfa as it is the specific antidote. Go to the next page if you knew the correct answer, or click the link image(s) below to further research the concepts in this question (if desired).

Research Concepts: Perioperative Anticoagulation Management

We update eBooks quarterly and Apps daily based on user feedback. Please tap flag to report any questions that need improvement.

Question 531: A 67-year-old male with a recent diagnosis of stage 3 non-small cell carcinoma presents to the emergency department with left leg swelling for the past 3 days. Venous duplex ultrasound is performed and the patient is found to have a left lower limb deep venous thrombosis. What is the next step regarding the treatment of this DVT?

Choices: 1. Low molecular weight heparin 2. Rivaroxaban 3. Intravenous unfractionated heparin 4. IVC filter

Answer: 1 - Low molecular weight heparin Explanations: Patients presenting with VTE and malignancy are ideally treated with lower molecular weight heparin for immediate anticoagulation. Lower extremity DVT is the most common type of DVT in patients presenting with malignancy. Patients with no contraindications for LMW and no renal insufficiency are started on LMW heparin for immediate anticoagulation. Direct oral anticoagulants are also preferred by some experts, rivaroxaban or apixaban, as an alternative. There is not enough data present for the use of DOACs in a setting where immediate anticoagulation is required. However, some experts use it. UFH can be used in patients with renal insufficiency. Go to the next page if you knew the correct answer, or click the link image(s) below to further research the concepts in this question (if desired).

Research Concepts: Hypercoagulability

We update eBooks quarterly and Apps daily based on user feedback. Please tap flag to report any questions that need improvement.

Question 532: A 40-year-old female farmer with a history of hyperlipidemia for the past five years presents to the hospital with a chronic cough for the last three months not suppressed by over-the-counter-cough suppressants. She has not smoked in her life, but her husband has been smoking for the past 25 years. She finds it difficult to work on her farm as she feels tired and becomes dyspneic on mild exertion. She has an increase in weight gain and bilateral pedal edema. On examination, there are no abnormal lung sounds. There is bilateral 2+ pedal edema, elevated JVP, and positive hepatojugular reflux. On cardiac auscultation, S3 is heard on the right lower sternal border. On echocardiography, there is mild tricuspid regurgitation and right ventricular hypertrophy. The left side of the heart shows normal findings. Cardiac catheterization reveals pressures of 28 mmHg in the pulmonary artery and 10 mmHg pulmonary capillary wedge pressure (PCWP). She is started on appropriate medications. What is the most likely mechanism of action of the preferred treatment?

Choices: 1. Blockade of cellular calcium channels 2. Blocking G-protein coupled cell surface receptors 3. Blockade of cGMP signaling pathway 4. Blockade of Phosphodiesterase-5 enzyme

Answer: 2 - Blocking G-protein coupled cell surface receptors Explanations: For a patient with NYHA class, 3 or 4 and diagnostic findings implying pulmonary hypertension is initiated on diuretics, and refractory cases are started on endothelin receptor blockers (ERB). Bosentan is a non-selective blocker of both endothelin A (ETA) and endothelin B (ETB) receptors. Sitaxentan blocks only ETA and is a selective blocker. Bosentan blocks endothelin receptors of which ETA is of greater affluence in its activity compared to ETB. Both are G-protein coupled cell surface receptors found on various parts of the body. ETA and ETB, both are present in the lungs, where ETA is predominantly on vascular smooth muscle cells. ET-1 is seen to be elevated in significant levels among females with pulmonary artery hypertension. The blockade reduces ET-1 activity. In the pathogenesis of pulmonary hypertension, endothelial dysfunction is a reason for the release of excessive ET-1 in circulation that interacts with the vasodilatory function by nitric oxide (NO), furthermore, it activates the increased release of Angiotensin-II, tilting the balance towards vasoconstriction. The blockade restabilizes this balance. Calcium channel blockers block the channels located on the surface of the cell surface. This leads to a decline in the intracellular calcium levels due to reducing the movement of calcium from extra to intracellular spaces. This acts to cause vasodilation and reduced cardiac contractility. It is often used as a first-line drug for systemic hypertension. cGMP pathway is utilized by nitric oxide and atrial natriuretic peptide, both of which are not to be blocked for the treatment of pulmonary artery hypertension. Phosphodiesterase-5 inhibitors are good alternatives to ERBs. Females tend to have elevated ET-1 levels as a significant association of pulmonary hypertension. Recent studies show that females started on ERBs have better outcomes, while males are better when initiated on PDE-5 inhibitors. Go to the next page if you knew the correct answer, or click the link image(s) below to further research the concepts in this question (if desired).

Research Concepts: Bosentan

We update eBooks quarterly and Apps daily based on user feedback. Please tap flag to report any questions that need improvement.

Question 533: A 65-year-old man presents due to the weeping of his legs. He has a past medical history of obesity, non-alcoholic steatohepatitis, and hypertension. His vital signs show oxygen saturation 98% on room air, respiratory rate 26 per minute, heart rate 115 bpm, blood pressure 80/60 mmHg, and temperature 98 F. On abdominal ultrasound, there is mild to moderate ascites with evidence of liver cirrhosis. An echocardiogram shows a hyperdynamic heart, concentric left ventricular thickening, grade 1 diastolic dysfunction, and an ejection fraction of 60-65%. Laboratory investigations rule out infectious and renal pathologies. Which of the following findings would be expected given the most likely cause of this patient's hypotension?

Choices: 1. Increased procalcitonin 2. Decreased stroke volume 3. Low left ventricular end-diastolic pressure 4. Low systemic vascular resistance

Answer: 4 - Low systemic vascular resistance Explanations: High output cardiac failure is characterized by an increase in cardiac output with a low systemic vascular resistance; however, even in this high output state, it is still insufficient for the body's demand, leading to clinical heart failure. Low systemic vascular resistance can be due to several different mechanisms depending on the underlying etiology. To compensate, the stroke volume and heart rate rise to increase cardiac output. If high output cardiac failure continues to progress, it can lead to cardiomyopathy, either secondary to direct cardiac effects of the underlying disease or by tachycardia-mediated cardiomyopathy. Go to the next page if you knew the correct answer, or click the link image(s) below to further research the concepts in this question (if desired).

Research Concepts: High-Output Cardiac Failure

We update eBooks quarterly and Apps daily based on user feedback. Please tap flag to report any questions that need improvement.

Question 534: A 16-year-old female, who had been diagnosed to have chronic otitis media in her right ear for the past one year, presents to the outpatient department with the complaint of right-sided headache for the past five days. She had two episodes of vomiting today. On examination, there is no neurological deficit. She undergoes a magnetic resonance imaging of the brain, which shows an area of restricted diffusion in the right temporoparietal region (T1 hypointense and T2 hyperintense) having perilesional edema and a midline shift of 3 mm. The periphery of the lesion is not enhancing on contrast administration. If she undergoes a surgical biopsy of this lesion, what could be the probable histological finding?

Choices: 1. Perivascular infiltrates 2. Reticular matrix 3. Neovascularity 4. Collagen capsule

Answer: 2 - Reticular matrix Explanations: The patient is having the right temporoparietal brain abscess in the late cerebritis stage (4-9 days). Reticular matrix (collagen precursor) is characteristic of this stage. The imaging findings (restricted diffusion with no peripheral contrast enhancement) show a lack of capsule formation. Perivascular infiltrates suggest early cerebritis (days 1-3), neovascularity suggests early capsule formation (days 10-13), and collagen capsule suggests late capsule stage (>14 days). Go to the next page if you knew the correct answer, or click the link image(s) below to further research the concepts in this question (if desired).

Research Concepts: Brain Abscess

We update eBooks quarterly and Apps daily based on user feedback. Please tap flag to report any questions that need improvement.

Question 535: An 85-year-old woman with congestive heart failure has been admitted to the hospital. She lives independently but as of last few weeks, the family reports inability to bathe herself, needing help with continence, skin break down, several falls, not communicating much, keeping to herself, and confusion. Her frailty score is calculated to be high. For which of the following dverse outcomes during inpatient hospitalization does she have the highest risk?

Choices: 1. Weight gain 2. Delirium 3. Depression 4. Hypertension

Answer: 2 - Delirium Explanations: Frailty is a risk factor for several adverse outcomes during hospital stays, including functional decline, falls, pressure ulcers, and delirium. Frailty is also associated with a higher risk of mortality during and after a hospital stay, admissions to nursing homes. It places a heavy and increasing burden on health and aged care systems. Frailty is the result of cumulative cellular damage from diverse etiologies over the life of the individual. Go to the next page if you knew the correct answer, or click the link image(s) below to further research the concepts in this question (if desired).

Research Concepts: Clinical Frailty Scale

We update eBooks quarterly and Apps daily based on user feedback. Please tap flag to report any questions that need improvement.

Question 536: A 65-year old woman is brought to the healthcare provider. She was taking warfarin for deep vein thrombosis when she developed neurological complaints. CT brain shows right basal ganglia hematoma. She is admitted to the intensive care unit where her INR is 3. The patient is given vitamin K. She has a history of a previous blood transfusion reaction. What other treatment will be ideal to reverse the coagulopathy?

Choices: 1. Fresh frozen plasma (FFP) 2. Prothrombin complex concentrates (PPCs) 3. Platelet concentrates 4. Recombinant activated factor VII (rFVIIa)

Answer: 2 - Prothrombin complex concentrates (PPCs) Explanations: Prothrombin complex concentrates (PPCs) have fewer complications than fresh frozen plasma (FFP). PCCs do not require cross-matching and can be reconstituted and administered rapidly. PCCs contains clotting factors II, VII, IX, and X and can rapidly normalize INR. rFVIIa does not replace all clotting factors. Go to the next page if you knew the correct answer, or click the link image(s) below to further research the concepts in this question (if desired).

Research Concepts: Hemorrhagic Stroke

We update eBooks quarterly and Apps daily based on user feedback. Please tap flag to report any questions that need improvement.

Question 537: A 42-year-old man comes to the clinic complaining of fatigue. He has a history of hypertension, hyperlipidemia, and chronic pancreatitis. He denies fever, dizziness, but reports abdominal pain, vomiting blood twice in the last week, and dark stool. Medications are hydrochlorothiazide and atorvastatin. On physical exam, the patient is afebrile, blood pressure is 110/75 mmHg, pulse rate is 110/min, and respiratory rate is 20/min. No blood is visible on rectal exam, but fecal occult blood is positive. Upper gastrointestinal bleeding is suspected, and the patient is sent for upper gastrointestinal endoscopy. On endoscopy, gastric varices are visible in the absence of esophageal varices. What is the most common cause of this endoscopic presentation?

Choices: 1. Cirrhosis of liver 2. Splenic vein thrombosis 3. Portal vein thrombosis 4. Pancreatitis

Answer: 2 - Splenic vein thrombosis Explanations: Patients with splenic vein thrombosis commonly present with gastric varices in the absence of esophageal varices. They can also have gastric varices that are greater than esophageal varices. Patients with splenic vein thrombosis can be asymptomatic or can present with a variety of symptoms. These symptoms include abdominal pain, variceal bleeding, splenomegaly, and thrombocytopenia. Go to the next page if you knew the correct answer, or click the link image(s) below to further research the concepts in this question (if desired).

Research Concepts: Splanchnic Venous Thrombosis

We update eBooks quarterly and Apps daily based on user feedback. Please tap flag to report any questions that need improvement.

Question 538: A 47-year-old male automobile mechanic is hospitalized after severe lead exposure. He is in the ICU intubated for airway protection and receiving IV chelation therapy. He has not made significant urine output for the past six hours. On evaluation, the patient is intubated and sedated. His skin and mucous membranes are pale and dry. He has a heart rate of 104/min but otherwise has normal vital signs. Which of the following is the next best step to help eliminate lead in this patient?

Choices: 1. Stop IV chelation therapy immediately 2. Administer IV crystalloid fluid 3. Begin renal replacement therapy 4. Administer IV loop diuretic

Answer: 2 - Administer IV crystalloid fluid Explanations: Lead is primarily eliminated through the urine. Most patients who present with acute lead poisoning or encephalopathy will be dehydrated from poor intake and nausea, vomiting, or diarrhea. Adequate IV fluid resuscitation is the first-line treatment to promote sufficient urine output and consequently lead to elimination from the body. IV loop diuretics or renal replacement therapy may be considered as further supportive measures in patients who have been adequately resuscitated. Chelation therapy should not be stopped at least until the patient's symptoms improve. Go to the next page if you knew the correct answer, or click the link image(s) below to further research the concepts in this question (if desired).

Research Concepts: Lead Encephalopathy

We update eBooks quarterly and Apps daily based on user feedback. Please tap flag to report any questions that need improvement.

Question 539: An 80-year old man with a history of hypertension on metoprolol presents to the emergency department via emergency medical services after being in a motor vehicle accident where he was the unrestrained driver who struck a tree head-on. He is currently awake with mild confusion and a Glasgow coma scale of 14 (baseline is 15) with a traumatic contusion to the anterior chest wall and upper abdomen from the steering wheel. He complains of severe abdominal and left shoulder pain. Vital signs reveal pulse 80/minute, respiratory rate 22/minute, blood pressure 100/70 mmHg, and pulse oximetry 92% on room air. A focused assessment with sonography for trauma is performed, which is positive for fluid in the left upper quadrant. A chest x-ray is negative. He undergoes computed tomography of the head, neck, chest, abdomen, and pelvis, which reveals a splenic laceration with small hemoperitoneum but no active extravasation. Which of the following is the next best step in the management of this patient?

Choices: 1. Normal saline intravenous at maintenance rate 2. STAT one-liter intravenous bolus of normal saline followed by blood transfusion 3. STAT two-liter intravenous bolus of normal saline 4. Emergent intubation and resuscitation with intravenous fluid and blood

Answer: 2 - STAT one-liter intravenous bolus of normal saline followed by blood transfusion

Explanations: There are many anatomic and physiologic changes associated with normal aging, which need to be understood to best diagnose and treat geriatric trauma patients. As we age, all our organ systems deteriorate with time and lose their underlying ability to function as they once optimally had at a younger age. This leads to significant considerations that must be undertaken when taking care of the geriatric trauma patient. Many elderly patients also present with a multitude of medical comorbidities that adversely affect their physiologic response and may alter their clinical presentation. Polypharmacy is also very common in the elderly and has a variety of effects on the cardiovascular system in this patient population in which beta-blockade, calcium-channel blockers, and cardiac glycosides are common, which lead to negative inotropic, dromotropic and chronotropic effects. Under normal circumstances, this is the desired effect of the medication; however, with an acute insult, these medications prevent the host from amounting to a normal physiologic response to compensate and maintain homeostasis. Shock, defined as global tissue hypoperfusion, is often clinically defined as a systolic blood pressure less than 90 mmHg. This definition is often incomplete for many patients, especially those who are geriatric trauma patients over the age of 65. Current literature supports a systolic blood pressure of 110 mmHg to be a better benchmark, which should be used for this purpose to identify occult shock. Furthermore, certain indices have been utilized to better detect occult shock as opposed to using one discrete variable such as heart rate, blood pressure, or urine output as they are all insensitive. Utilization of the shock index has been found to be a more sensitive indicator. It is calculated as the quotient of the systolic blood pressure by the heart rate (shock index = heart rate/systolic blood pressure). A shock index of 0.5-0.7 is considered normal, but when it exceeds 0.7, the patient is in shock. Further iterations of this index have also been proposed, which are felt to be more sensitive such as the respiratory adjusted shock index or RASI. RASI is calculated by multiplying the shock index by (respiratory rate/10). When the RASI is greater than 1.3, it suggests that the patient is in occult shock. This patient's shock index is 0.8, and RASI 1.76 is which both indicate that the patient, even though has what would be deemed normal vital signs, is in occult shock. As previously mentioned, shock can be insidious in the elderly geriatric trauma population and should be readily sought. A general rule of thumb is that a patient who presents with an injury and hypotension presumably has traumatic hemorrhagic shock until proven otherwise. Adult patients can only hemorrhage into five spaces enough volume to cause hemorrhagic shock to be present, and each of the following is examined during the primary and secondary resuscitation: chest, abdomen and pelvis, retroperitoneum, long bones, and field or floor. If this triage is negative, then additional etiologies of shock must be considered. This patient presents with left upper quadrant free fluid on ultrasound as well as computed tomography evidence of a splenic injury. Left shoulder pain occurs from diaphragmatic irritation by blood, and this is known as Kerr’s sign. Current advanced trauma life support teaching is that an attempt should be made to stop all

hemorrhage as able. For cases in which the hemorrhage is considered non-compressible such as, in the chest, abdomen, or retroperitoneum, radiologic or surgical approaches are often warranted. Patients who are in hemorrhagic shock should be resuscitated with as little crystalloid as possible, preferably normal saline or Ringer’s lactate up to 1 liter, and then blood products or whole blood should be utilized as better outcomes have been obtained. Permissive hypotension, a concept where resuscitation is withheld or limited to maintain a radial pulse and mentation and not try to achieve physiologically normal systolic blood pressures, is often practiced in the young trauma patient. The thought is that by keeping pressures low, the body won’t “pop the clot,” leading to worsening of the hemorrhage. This practice however should not be utilized in patients with brain injuries or in the geriatric population as the outcomes have been found to be more dismal. Trauma patients who are in respiratory failure and are in hemorrhagic shock are at risk for hemodynamic collapse if not first resuscitated and then intubated as the induction medications and positive pressure ventilation lead to vasodilation with hypotension and increased intrathoracic pressure with a decrease in preload respectively. Consideration to change the standard airway, breathing, circulation (ABC) approach for the patient who presents with hemorrhagic shock to circulation, airway, and breathing (CAB) would be appropriate to prevent cardiovascular collapse. Go to the next page if you knew the correct answer, or click the link image(s) below to further research the concepts in this question (if desired).

Research Concepts: Geriatric Trauma

We update eBooks quarterly and Apps daily based on user feedback. Please tap flag to report any questions that need improvement.

Question 540: A 66-year-old man is admitted to the hospital for acute pancreatitis. He is prescribed subcutaneous heparin for DVT prophylaxis. 5 days later, his platelet count drops from a baseline of 184000/microL at presentation to 63000/microL. The team sends a platelet factor 4 (PF4) ELISA and serotonin release assay (SRA), discontinues heparin, and starts the patient on argatroban infusion. The SRA eventually comes back positive. The patient does not develop any thrombosis, his platelet counts recover, he is ready for discharge, and argatroban is stopped. Which of the following is the most appropriate anticoagulation regimen on discharge for this patient?

Choices: 1. Enoxaparin for 30 days 2. Fondaparinux for 90 days 3. Rivaroxaban for 30 days 4. Warfarin for 90 days

Answer: 3 - Rivaroxaban for 30 days Explanations: Given that the patient has isolated HIT without any thromboembolic complications, he will require 30 days of anticoagulation on discharge. Rivaroxaban is the most appropriate response. Rivaroxaban is one of the recommended agents for HIT, and the treatment duration of 30 days is appropriate. Enoxaparin is not an appropriate choice, as it is a form of heparin and will interact with PF4 and IgG. Fondaparinux is an appropriate choice; however, the treatment duration of 90 days is too long for isolated HIT. Warfarin is an appropriate choice; however, the treatment duration of 90 days is too long for isolated HIT. In addition, if warfarin were to be initiated, it would require bridging (with argatroban or fondaparinux) for at least 5 days and until 2 therapeutic INRs are obtained. Go to the next page if you knew the correct answer, or click the link image(s) below to further research the concepts in this question (if desired).

Research Concepts: Heparin Induced Thrombocytopenia

We update eBooks quarterly and Apps daily based on user feedback. Please tap flag to report any questions that need improvement.

Question 541: A 36-year-old male patient presents for the evaluation of weakness. His symptoms started about a month ago when he noted weakness of the right hand. He has difficulty grabbing small objects and problems with lifting the right hand above his head. Two weeks before the consultation, he noted that his right leg was dragging. About a week ago, his live-in partner noticed some behavioral and memory changes; he was described to be more irritable, impulsive, and can not remember simple words for everyday objects. He had a positive human immunodeficiency virus (HIV) screen six months ago. On examination, his vital signs are within normal limits. His pupils are isocoric, briskly reactive to light, with a visual field cut on the right temporal and left nasal fields. There is peripheral facial palsy on the right side. The motor exam shows 3/5 strength on the right upper and lower extremity, 4/5 on the left upper extremity, and 5/5 on the left lower extremity. There is a 20% deficit to light touch, pain, and temperature on the right. Babinski sign is positive bilaterally. Head MRI reveals T1 hypointense white matter lesions in the parietooccipital region. Which of the following is the most likely cause of this patient's symptoms?

Choices: 1. Herpes simplex virus (HSV) 2. CNS lymphoma 3. John Cunningham (JC) virus 4. Human immunodeficiency virus (HIV)

Answer: 3 - John Cunningham (JC) virus Explanations: Progressive multifocal leukoencephalopathy (PML) is caused by the JC virus that is normal under control of the immune system. Immunosuppressed states lead to the activation of the virus. PML is a rapidly progressive neurological disorder that is fatal. The diagnosis requires a combination of clinical, imaging, and microbiological evidence. The disorder can present with a wide range of symptoms that are often similar to what observes in a patient with multiple sclerosis. Visual symptoms occur in one-third of patients and are often the initial presentation of the infection. MRI is more sensitive in picking up white matter changes. Usual locations include the parieto-occipital region, usually asymmetric, but may also involve the infratentorial region. Go to the next page if you knew the correct answer, or click the link image(s) below to further research the concepts in this question (if desired).

Research Concepts: Progressive Multifocal Leukoencephalopathy

We update eBooks quarterly and Apps daily based on user feedback. Please tap flag to report any questions that need improvement.

Question 542: A 44-year-old man with cerebral palsy and intellectual disability who resides in a group home is brought to the emergency department (ED) with concerns of poor oral intake, weight loss of 10 pounds, and low-grade fevers recorded intermittently for the past two weeks. In the ED, the patient is found to be febrile with a temperature of 101.5 F, respiratory rate 22/min, heart rate 105/min, and blood pressure 100/70 mmHg. On examination, he has a stage 3 sacral pressure ulcer with some discharge. Labs show a white blood cell count of 27000/microL with neutrophilia and left shift. He is given a bolus of IV fluids, and two sets of blood cultures are drawn. He is given empiric antibiotics with IV piperacillin-tazobactam and IV vancomycin. The next day, the microbiology lab notified the primary team that the patient is growing Enterococcus faecium in 4 out of 4 culture bottles with sensitivities still pending. The same evening the patient develops worsening hypoxia and is put on 5 L/min oxygen via nasal cannula. Which of the following is the next best step in the management of this patient?

Choices: 1. Switch piperacillin-tazobactam to daptomycin 2. MRI of the sacrum 3. Echocardiogram 4. CT angiogram of the chest

Answer: 3 - Echocardiogram Explanations: Given the poor appetite, low-grade fevers, weight loss, tachycardia, and Enterococcus bacteremia, there should be a concern for infective endocarditis and obtain a transthoracic echocardiogram to look for a vegetation. Also, in the light of new hypoxia, we are concerned about septic embolization from native valve vegetation. The sacral pressure ulcer seems to be an obvious source of E. faecium infection. The patient also resides in a group home which puts him at risk for getting an infection with resistant species like E. faecium. Although MRI to look for sacral osteomyelitis would help determine the duration of antibiotics, it would not be the next step in managing this patient. Although E. faecium was shown to be 80% vancomycin-resistant and 90% ampicillin-resistant, we ordinarily wait for final sensitivity data. In the case of endocarditis or osteomyelitis, the duration of antibiotics is 6 to 8 weeks. Hence stopping antibiotics and switching to another agent is not warranted. Given that the patient is found to have acute hypoxia, a pulmonary embolism is a good differential. However, in this case, we are leaning towards a septic embolus from a vegetation on the heart valves - likely the tricuspid valve. Hence we would choose an echocardiogram to be the next step in the management of this patient. Go to the next page if you knew the correct answer, or click the link image(s) below to further research the concepts in this question (if desired).

Research Concepts: Enterococcus Infections

We update eBooks quarterly and Apps daily based on user feedback. Please tap flag to report any questions that need improvement.

Question 543: A 45-year-old patient with past history of type 2 diabetes mellitus presents following a fall two days prior. She is now experiencing severe pain and tenderness over the upper arm. On physical exam, her pain seems out of proportion to the overlying erythema and extends proximally. Laboratory studies reveal leukocytosis. Tissue aspiration PCR is positive for streptococcal pyrogenic exotoxin. What initial treatment for this condition, in addition to surgical debridement and antibiotics, has been shown to reduce mortality?

Choices: 1. Leaving wound open after surgical debridement 2. Hyperbaric oxygen therapy 3. Reconstruction of the wound with a skin flap 4. Topical oxygen therapy

Answer: 2 - Hyperbaric oxygen therapy Explanations: In cases of necrotizing fasciitis, amputation may be required, but it is preferable to make that decision after multiple surgical debridements, antibiotic therapy, and hyperbaric oxygen therapy have been used first. Hyperbaric oxygen therapy is FDA approved to treat necrotizing fasciitis. It has been shown to reduce the significant inflammation caused by the condition. It also maintains perfusion of the affected tissues and can exert an antimicrobial effect. While reconstruction of the resulting wound following debridement is often performed in cases of necrotizing fasciitis, this is a late procedure after the infection has been treated. Initially, the wound needs to remain open for frequent irrigation, debridement, and packing with antibiotic dressings. Topical oxygen therapy, though theorized for decades, has not been shown to be effective as there is an inability to create a great enough diffusion gradient for oxygen to move across the skin. Go to the next page if you knew the correct answer, or click the link image(s) below to further research the concepts in this question (if desired).

Research Concepts: Necrotizing Fasciitis

We update eBooks quarterly and Apps daily based on user feedback. Please tap flag to report any questions that need improvement.

Question 544: A 32-year-old woman presents to the emergency department with confusion, vomiting, and abdominal pain. The patient is ill-appearing, restless, and confused. Her husband reports a 2-week history of abdominal pain, followed by poor oral intake, nausea, and vomiting. On physical exam, her blood pressure is 106/70 mmHg, her pulse is 145/min, her temperature is 36.2 °C (97.1 °F), and her respiratory rate is 28/min. Scleral icterus and jaundice are noted with moderate right upper quadrant tenderness. Laboratory testing revealed total bilirubin 5.6 mg/dL, aspartate aminotransferase (AST) level of 3428 U/L, alanine aminotransferase (ALT) level of 7498 U/L, alkaline phosphate level of 310 U/L, international normalized ratio (INR) of 6.8, and an arterial pH of 6.92. Her serum acetaminophen (paracetamol) level is 0 mcg/mL (normal range 10 to 30 mcg/mL). What is the next best step in the management of this patient?

Choices: 1. Start N-acetylcysteine infusion 2. Start ledipasvir/sofosbuvir treatment 3. Start pegylated interferon-alfa plus oral ribavirin treatment 4. Emergent transfer to a liver transplant center

Answer: 4 - Emergent transfer to a liver transplant center Explanations: Acute hepatitis can progress to acute liver failure. Signs and symptoms of acute liver failure include hyperbilirubinemia, hepatic encephalopathy, and coagulopathy. Coagulopathy, defined as INR greater than 1.5, indicates severely impaired liver function and is one criterion used when considering the need for a liver transplant. Patients with acute liver failure can have multiorgan failure. They will need fluid resuscitation and monitoring for hypoglycemia, hypokalemia, hypomagnesemia, hypophosphatemia, bleeding, and encephalopathy. Patients with acute hepatitis that process to acute liver failure are best treated in liver transplant centers. Transfers should be arranged as soon as possible when patients are medically stable for emergent transfer. Acute hepatitis can progress to acute liver failure. In the United Kingdom, the United States, and Australia, drug-induced liver failure from acetaminophen (paracetamol) causes 39-50% of cases of acute liver failure. The other predominant cause is acute viral hepatitis. Go to the next page if you knew the correct answer, or click the link image(s) below to further research the concepts in this question (if desired).

Research Concepts: Acute Hepatitis

We update eBooks quarterly and Apps daily based on user feedback. Please tap flag to report any questions that need improvement.

Question 545: A 17-year-old boy is brought to the hospital after accidentally ingesting the tablets used in cleaning jewelry in the electroplating industry one hour ago. Since the ingestion, the patient has vomited thrice and has become agitated and irritable. Gastric lavage is performed, and the aspirate is sent for analysis. His past medical history is significant for major depressive disorder managed on sertraline. On examination, his Glasgow coma score is 12, heart rate is 126/min, respiratory rate is 34/min, oxygen saturation is 94% on room air, and blood pressure is 104/66 mmHg. Pupils are bilaterally reactive to light. Cherry-red discoloration of the lips and mucous membranes is observed. Blood tests show hemoglobin of 18.2 g/dL, platelet count of 293,000/microliter, sodium of 144 mmol/L, potassium 3.9 mmol/L, magnesium of 2.6 mg/dL, alanine aminotransferase level of 54 IU/L, and creatinine of 1.8 mg/dL. Arterial blood gas analysis shows a severe metabolic acidosis. Intravenous fluids are started, and high flow oxygen therapy is initiated. What is the most appropriate treatment for this patient?

Choices: 1. Sodium thiosulfate 2. Hydroxycobalamin 3. Cyanocobalamin 4. Nitrites

Answer: 2 - Hydroxycobalamin Explanations: This patient took tablets used in the jewelry cleaning industry. This coupled with his presentation of altered mental status, confusion, agitation, irritability, tachypnea, metabolic acidosis, nausea, vomiting, and cherry red mucus membranes leads to the diagnosis of cyanide poisoning. Labs that are pertinent to the initial evaluation in a patient with cyanide poisoning are complete blood count, electrolytes, urinalysis, urine toxicology screen, arterial blood gas, carboxyhemoglobin level (if in a fire), chest x-ray and EKG. Plasma lactate also may be obtained, and a level of greater than 8 mmol/L is 94% sensitive and 70% specific for significant cyanide toxicity in smoke inhalation victims. Narrowing of the venous-arterial oxygen gradient. Cyanide inhibits cellular oxidative phosphorylation resulting in a marked decrease in peripheral tissue oxygen extraction from the blood. This results in elevated central venous oxygenation. Hydroxycobalamin is the antidote of choice for acute cyanide poisoning, especially if the patient has coexisting carbon monoxide poisoning. Other antidotes impair oxygen-carrying capacity and worsen cellular hypoxia and acidosis. The standard dose is 5 grams, given intravenously over 15 minutes. Beware that this antidote turns urine dark red; this is not due to myoglobinuria. Sodium thiosulfate is the second-line treatment for cyanide toxicity, and nitrites may be used to induce methemoglobinemia if both hydroxycobalamin and sodium thiosulfate are not available or when they are contraindicated . Go to the next page if you knew the correct answer, or click the link image(s) below to further research the concepts in this question (if desired).

Research Concepts: Cyanide Toxicity

We update eBooks quarterly and Apps daily based on user feedback. Please tap flag to report any questions that need improvement.

Question 546: A 16-year-old male presents to the emergency room with symptoms consistent of an overdose. When asked what he had been exposed to, he says he does not remember exactly but that he had picked up eye drops from a friend's house and ingested them in a suicide attempt. He has symptoms of tachycardia, flushing, delirium, and upon examination, mydriasis. What is the most appropriate medication given for this overdose?

Choices: 1. Reversible cholinesterase inhibitor 2. Selective serotonin reuptake inhibitor 3. Beta blocker 4. Antipsychotics

Answer: 1 - Reversible cholinesterase inhibitor Explanations: Physostigmine is a reversible cholinesterase inhibitor used to treat atropine poisoning or overdose. It is a tertiary amine that crosses the blood-brain barrier, unlike any other cholinesterase inhibitors, making it the drug of choice. The antimuscarinic toxidrome results from blockade of the neurotransmitter acetylcholine at central and peripheral muscarinic receptors. Physostigmine is a carbamate that acts by reversibly inhibiting acetylcholinesterase. Selective serotonin reuptake inhibitors are most commonly used to treat depression and anxiety disorders. Although this patient has tachycardia and flushing, his symptoms of mydriasis and delirium are not explained by depression or anxiety disorders. Mydriasis and delirium are seen in atropine poisoning. Beta-blockers are used to treat tachycardia but are not the treatment of choice for atropine poisoning. Tachycardia is a symptom of atropine poisoning. Antipsychotics are used to treat symptoms of psychosis, bipolar disorder, and at times, delirium. This would not be a suitable treatment for atropine poisoning. Go to the next page if you knew the correct answer, or click the link image(s) below to further research the concepts in this question (if desired).

Research Concepts: Physostigmine

We update eBooks quarterly and Apps daily based on user feedback. Please tap flag to report any questions that need improvement.

Question 547: A 65-year-old man presents to the emergency department with left lower quadrant abdominal pain of sudden onset. CT shows minimal spots of free air around the sigmoid colon. The patient has stable vital signs. Physical examination shows a nontender abdomen, no masses or guarding, and normal bowel sounds, with no peritoneal signs. What is the most likely diagnosis?

Choices: 1. Diverticulitis with micro-perforation 2. Perforated appendicitis 3. Perforated peptic ulcer 4. Meckel diverticulum

Answer: 1 - Diverticulitis with micro-perforation Explanations: The most likely diagnosis in this age group with these findings is acute diverticulitis. Most patients with acute diverticulitis do not require surgery. A high fiber diet may decrease the incidence of diverticulitis. Most acute episodes of acute diverticulitis are self-limiting. Go to the next page if you knew the correct answer, or click the link image(s) below to further research the concepts in this question (if desired).

Research Concepts: Bowel Perforation

We update eBooks quarterly and Apps daily based on user feedback. Please tap flag to report any questions that need improvement.

Question 548: A 45-year-old woman with no significant past medical history is brought to the emergency department with complaints of fever and severe, new-onset persistent headache for 2-3 days. She reports being vaccinated with the ChAdOx1 nCoV-19 vaccine 15 days ago. Vital signs at admission show blood pressure of 118/70 mmHg, heart rate 65/min, and temperature 98.3°F. Physical examination is unremarkable. Initial blood work demonstrates hemoglobin 11.9 g/dL and platelet count 78000/microL. Liver and renal function are unremarkable. D-Dimer is 10000 ng/mL (reference range 250 ng/mL) at admission. POCT SARS-CoV-2 is negative. Which of the following is the next best step in the management of this patient?

Choices: 1. Electroencephalogram (EEG) 2. CT cerebral venography 3. X-ray of the skull 4. Ibuprofen and discharge

Answer: 2 - CT cerebral venography Explanations: The patient in the above clinical vignette most likely has vaccine-induced immune thrombotic thrombocytopenia (VITT) given her recent history of single-dose vaccination against COVID-19 with ChAdOx1 nCoV-19 vaccine 15 days ago, thrombocytopenia, and severe new-onset headache indicating possible cerebral sinus venous thrombosis (CSVT). The next best step in the management of this patient is to confirm the presence of CSVT with a CT cerebral venography. Diagnosis of CSVT is an integral component to confirm the diagnosis of VITT, and early diagnosis of VITT is important given its therapeutic implications. X-ray of the skull and EEG would not evaluate for the presence of CSVT, and offering supportive management would be futile in this patient. Go to the next page if you knew the correct answer, or click the link image(s) below to further research the concepts in this question (if desired).

Research Concepts: Coronavirus (COVID-19) Vaccine-Induced Immune Thrombotic Thrombocytopenia (VITT)

We update eBooks quarterly and Apps daily based on user feedback. Please tap flag to report any questions that need improvement.

Question 549: A 65-year-old woman is admitted to the hospital with triple vessel disease and is scheduled for urgent CABG. Her medication allergy list states a history of thrombocytopenia with heparin, which occurred 6 months ago. Which antithrombotic strategy is most appropriate during CABG in this patient?

Choices: 1. Bivalirudin 2. Fondaparinux 3. Obtain an SRA and if negative, use heparin 4. Heparin

Answer: 1 - Bivalirudin Explanations: Bivalirudin is the most appropriate choice for use during CABG in patients with a history of HIT. In patients with a remote history of HIT who require CABG, it is appropriate to check a platelet serotonin-release assay (SRA), and if negative, heparin may be used only during CABG, at the lowest dose and shortest duration needed. Given that the patient requires urgent CABG, sending an SRA and waiting for the results would not be appropriate. Therefore, bivalirudin is the best option. Fondaparinux is not approved for use during CABG. Even in patients with a remote history of HIT, heparin products may not be used unless an SRA is checked and is negative. Even if the SRA is negative, heparin may be used only during CABG, at the lowest dose and shortest duration needed. Go to the next page if you knew the correct answer, or click the link image(s) below to further research the concepts in this question (if desired).

Research Concepts: Heparin Induced Thrombocytopenia

We update eBooks quarterly and Apps daily based on user feedback. Please tap flag to report any questions that need improvement.

Question 550: A 37-year-old man with a past medical history of chronic pain syndrome in the lower extremities on a pain regimen, hyperlipidemia on atorvastatin, hypertension on lisinopril, and prior IV drug use is brought to the hospital after he was found at home confused 1 hour ago. The roommate reports that his mental status has significantly changed from 2 days ago and had a few episodes of vomiting this morning. The physical exam is notable for diffuse jaundice, right upper quadrant tenderness, and not being oriented to self. Initial labs reveal BUN 40 mg/dL, creatinine 2.5 mg/dL, bicarbonate 19 mEq/L, AST 15,000 IU/L, ALT 12,000 IU/L, INR 1.5, and the toxicity panel is still pending. An analgesic with which of the following mechanisms of action is most likely responsible for the patient's presentation?

Choices: 1. Dopamine antagonism 2. Increasing serotonin effects in the CNS 3. Inhibition of the hypothalamic heat-regulating center 4. Inhibition of the mu-receptors

Answer: 2 - Increasing serotonin effects in the CNS Explanations: Acetaminophen is widely used in combination with hydrocodone to treat pain. Liver cirrhosis increases the risk of developing acetaminophen toxicity. Analgesia effects of the acetaminophen are obtained via activation of the serotonergic inhibitory pathways in the CNS. Antipyretic properties are obtained by inhibition of the hypothalamic heat-regulating center. Go to the next page if you knew the correct answer, or click the link image(s) below to further research the concepts in this question (if desired).

Research Concepts: Hydrocodone and Acetaminophen

We update eBooks quarterly and Apps daily based on user feedback. Please tap flag to report any questions that need improvement.

Question 551: A 50-year-old man with no significant past medical history undergoes a decompressive craniectomy for an acute subdural hematoma following a traumatic head injury 24 hours ago. He is now on the neurosurgical intensive care unit and remains intubated and ventilated. His urine output is 800 mL over the last 2 hours, despite appropriate intravenous fluid administration. Which of the following additional findings is most consistent with the patient's most likely diagnosis?

Choices: 1. Urine specific gravity >1.005 2. Serum Na+ 135 mmol/L 3. Urine osmolality 300 mOsm/kg 4. Serum osmolality 295 mmol/kg

Answer: 3 - Urine osmolality 300 mOsm/kg Explanations: Neurogenic diabetes insipidus (DI) is associated with traumatic brain injury (incidence up to 35%), subarachnoid hemorrhage, and pituitary surgery. Abnormally low urine osmolality (300 mOsm/kg) indicates the presence of diabetes insipidus. Development of DI following nonpituitary surgery is often associated with severe cerebral edema. DI may be transient or permanent. It occurs as a result of the failure of ADH release from the hypothalamic-pituitary axis. This leads to the production of large volumes of dilute urine (as the body is unable to concentrate the urine) and inappropriate loss of water. The patient becomes clinically dehydrated and hypernatraemic. Typically, patients may have a urine output of more than 6 L/day. Go to the next page if you knew the correct answer, or click the link image(s) below to further research the concepts in this question (if desired).

Research Concepts: Diabetes Insipidus

We update eBooks quarterly and Apps daily based on user feedback. Please tap flag to report any questions that need improvement.

Question 552: An 85-year-old female is brought to the emergency department with severe respiratory distress. On arrival, she is confused and agitated. Vital signs include blood pressure 100/65 mmHg, pulse 105/minute, respiratory rate 23/minute, oxygen saturation 70%, and temperature 38.1 C (100.5 F). The patient is immediately intubated. The respiratory therapist can see a color change on the end-tidal carbon dioxide monitor with condensation on the inside of the endotracheal tube. Breath sounds are equal bilaterally. What is the next best step in the management of this patient?

Choices: 1. Obtain a CT scan 2. Continue to monitor the patient 3. Obtain a chest x-ray 4. Listen for bilateral breath sounds twice per hour for 4 hours

Answer: 3 - Obtain a chest x-ray Explanations: A chest x-ray is necessary to confirm the placement of the endotracheal tube after an intubation procedure. Bilateral breath sounds heard on examination is an indication that the endotracheal tube is placed correctly, but confirming with a chest x-ray is still necessary. Monitoring the patient is not sufficient to ensure that the endotracheal tube is in place. A CT of the chest is unnecessary as a chest x-ray is adequate to confirm placement. Go to the next page if you knew the correct answer, or click the link image(s) below to further research the concepts in this question (if desired).

Research Concepts: Tracheal Rapid Sequence Intubation

We update eBooks quarterly and Apps daily based on user feedback. Please tap flag to report any questions that need improvement.

Question 553: A 56-year-old woman is admitted to the ICU with a diagnosis of viral pneumonia with respiratory failure. She required ventilatory support for 6 days and was extubated on the 7th day as the pneumonia was resolving. On the 8th day, she develops a fever of 101°F. Chest radiograph shows significant clearance of pneumonia. Her vital signs reveal a heart rate of 133/min, respiratory rate of 32/min, and blood pressure of 90/60 mmHg. General examination reveals doubtful redness around the central line site. Which of the following is the next best step in the management of this patient?

Choices: 1. Antibiotic lock 2. Remove the central venous catheter 3. CT chest 4. Acetaminophen

Answer: 2 - Remove the central venous catheter Explanations: The patient seems to have a catheter-related bloodstream infection (CRBSI). CRBSI is defined as the presence of bacteremia arising from the intravenous catheter. Prompt removal of the catheter is indicated when CRBSI is suspected. Two sets of blood cultures should be obtained, one from the central venous line and the other from a peripheral site. Empirical antibiotic therapy should be started immediately. CRBSI is the commonest cause of nosocomial bacteremia. Common pathogens causing CRBSI are Staphylococcus aureus, Pseudomonas aerogenosa, coagulase-negative staphylococci, Escherichia coli, Klebsiella pneumoniae, and Acinetobacter baumanii. Go to the next page if you knew the correct answer, or click the link image(s) below to further research the concepts in this question (if desired).

Research Concepts: Fever In the Intensive Care Patient

We update eBooks quarterly and Apps daily based on user feedback. Please tap flag to report any questions that need improvement.

Question 554: A 90-year-old-man from a nursing home is evaluated after multiple episodes of ventricular tachycardia. The patient had an implantable cardioverter-defibrillator placed 1 year back. Interrogation shows multiple shocks provided for frequent ventricular tachycardia in the last 1 month. The patient received an amiodarone infusion after the recent episode of ventricular tachycardia. He has nonischemic dilated cardiomyopathy, and echocardiography done 1 month back showed left ventricular ejection fraction 20% (the same as 6 months back). The patient also has a history of alcoholism, dementia, chronic kidney disease, diabetes mellitus, dyslipidemia, hypothyroidism, and multiple ischemic infarcts. He has been admitted multiple times in the last 1 year for exacerbation of heart failure. On examination, he is bed-bound and not oriented to time, place, or person. He is afebrile, with a heart rate of 86/min, blood pressure 92/50 mmHg, and oxygen saturation of 92% on 3 L/min of oxygen via nasal cannula. The patient has elevated jugular venous pressure. Lung examination reveals crackles on bilateral lung bases. S3 is heard on auscultation. There is bilateral 2+ pitting edema up to the knees. Laboratory evaluation shows serum sodium 125 mEq/L and serum creatinine 3.2 mg/dL. The patient does not tolerate betablockers due to soft blood pressure and angiotensin-converting enzyme inhibitors or angiotensin receptor blockers due to chronic kidney disease. Which of the following is the next best step in the management of this patient?

Choices: 1. Implantation of left ventricular assist device 2. Digoxin 3. Discussion with next of kin regarding goals of care and advanced directives 4. Dopamine infusion

Answer: 3 - Discussion with next of kin regarding goals of care and advanced directives Explanations: Elderly patients should have discussions about end of life and goals of care early before the health crises, preferably in ambulatory settings. Patients with a terminal illness such as end-stage heart disease, end-stage lung disease, endstage kidney disease, malignancy requiring a third-line therapy, etc., have predicted a life span of less than a year. Such patients for whom the answer to the question - "will you be surprised if the patient died in next 1 year?" is "no," should have a discussion about goals of care and advanced directives. Studies have shown that early initiation of such discussion leads to a better quality of life and satisfaction of family members. Arrhythmias due to end-stage heart failure are difficult to control even with maximal antiarrhythmic agents. The addition of digoxin may not provide significant benefit, improvement in quality or quantity of life, so goals of care should be discussed prior to adding any chronic medication. Intravenous inotropic agents are used as a bridge to the left ventricular assist device. This patient is not a candidate for any major surgery due to multiple co-morbidities. The patient is not a good candidate for major surgeries. Before proceeding with any invasive intervention, a discussion should be held to know about the patient's wishes and goals. Go to the next page if you knew the correct answer, or click the link image(s) below to further research the concepts in this question (if desired).

Research Concepts: Geriatric Care Special Needs Assessment

We update eBooks quarterly and Apps daily based on user feedback. Please tap flag to report any questions that need improvement.

Question 555: A 48-year-old woman presents to the emergency department with one day of nausea and emesis and an inability to pass stool or flatus. Her heart rate is 121/min, blood pressure 100/60 mmHg, respiratory rate 18/min, and SpO2 98% on room air. Her past medical and surgical history is significant only for a Roux-en-Y gastric bypass for obesity three years ago. Two large-bore IVs are inserted for resuscitation and a nasogastric tube is placed. Which of the following is the next best step in the management of this patient?

Choices: 1. Exploratory laparotomy 2. CT abdomen with IV and oral contrast 3. Abdominal x-ray 4. Inpatient observation

Answer: 1 - Exploratory laparotomy Explanations: This patient most likely has an internal hernia secondary to her Roux-en-Y bypass with a gastrojejunostomy. Typical imaging modalities may be unable to detect a small amount of bowel trapped and, therefore cannot be relied upon. This patient should be taken directly to the OR to examine whether they have any bowel trapped in an internal hernia and to have it reduced if possible or resected if it is infarcted. Given that just a small amount of bowel can become trapped and create a closed-loop obstruction a lack of significant findings on abdominal x-ray does not rule out strangulated bowel. Admitting the patient for observation would merely increase the likelihood that incarcerated bowel would become infarcted. Go to the next page if you knew the correct answer, or click the link image(s) below to further research the concepts in this question (if desired).

Research Concepts: Gastrojejunostomy

We update eBooks quarterly and Apps daily based on user feedback. Please tap flag to report any questions that need improvement.

Question 556: A 67-year-old man is recovering from a large intraperitoneal bleed in the intensive care unit after receiving multiple hits from a baseball bat during a mugging. The patient was initially somnolent and difficult to arouse. His initial blood pressure reading was 60/45 mmHg. The patient received a 1 L bolus infusion of 0.9% saline. Soon afterward, his blood pressure rose to 75/55 mmHg. The patient is now somewhat arousable and able to speak in short phrases. What is the next most appropriate management strategy for this patient?

Choices: 1. An additional 2 L bolus infusion of 0.9% saline to reach systolic pressures between 80-90 mmHg 2. Incremental 100 -200 mL infusions of 0.9% saline to reach systolic pressures between 80-90 mmHg 3. Incremental 500 mL infusions of 0.9% saline to reach systolic pressures between 90-100 mmHg 4. An additional 1 L bolus infusion of 0.9% saline and incremental 500 mL infusions to reach systolic pressure between 90-100 mmHg

Answer: 2 - Incremental 100 -200 mL infusions of 0.9% saline to reach systolic pressures between 80-90 mmHg

Explanations: Smaller 100-200 mL infusions of fluid towards acceptable hypotensive systolic levels have shown to have better outcomes on patient survivability. Blunt trauma should be managed with systolic pressures in the 80 -90 mmHg range until the source of the bleeding is repaired. The Advanced Trauma Life Support (ATLS) recommends bolus infusions no greater than 1.5 L in order to rescue a patient from imminent danger. Afterward, smaller infusions may be used to titrate the patient's blood pressure toward appropriate permissive hypotensive levels. A 2 L bolus infusion may lead to an increase in bleeding due to a reflex vasodilatory effect, compared to smaller infusions towards acceptable hypotensive levels. Go to the next page if you knew the correct answer, or click the link image(s) below to further research the concepts in this question (if desired).

Research Concepts: Permissive Hypotension

We update eBooks quarterly and Apps daily based on user feedback. Please tap flag to report any questions that need improvement.

Question 557: A culture is taken from a draining purulent ulcer in the groin of a critically ill patient in the intensive care unit who has been admitted for diabetic ketoacidosis. The patient had initially been improving but then developed groin pain and fever. What antibiotic regimen should the patient be placed on to best treat the underlying pathology of his groin pain?

Choices: 1. Piperacillin-tazobactam 2. Bactrim 3. Metronidazole & ceftriaxone 4. Gentamicin & metronidazole

Answer: 1 - Piperacillin-tazobactam Explanations: Piperacillin-tazobactam, or Zosyn, is the treatment of choice for Fournier's gangrene as it provides antibiotic coverage against the aerobic and anaerobic bacteria that are at the root of Fournier's gangrene. Bactrim would be the appropriate answer to simple cellulitis; however, in Fournier's gangrene, a strong antibiotic with more coverage is required. Metronidazole & a 3rd generation cephalosporin is part of the previously used triple antibiotic therapy that was previously used to treat Fournier's Gangrene. Triple antibiotic therapy included a 3rd generation cephalosporin or an aminoglycoside in addition to penicillin & metronidazole. Go to the next page if you knew the correct answer, or click the link image(s) below to further research the concepts in this question (if desired).

Research Concepts: Fournier Gangrene

We update eBooks quarterly and Apps daily based on user feedback. Please tap flag to report any questions that need improvement.

Question 558: A 45-year-old man presents to the hospital with acute abdominal pain, which started a couple of hours ago. He appears diaphoretic, uncomfortable, and prefers to lie still. While examining, he has an episode of non-bilious vomiting and tries to contract the abdominal wall muscles upon palpation voluntarily. There is no ascites. His vital signs reveal pulse 110 beats/min, respiratory rate 20 breaths/min, blood pressure 142/86 mmHg, and temperature 38.3 C. He reports experiencing an episode of epigastric abdominal pain, nausea, and vomiting over a month ago which self-resolved in 1-2 days. He did not seek medical care at that time. On careful history, he shares struggling with alcohol use disorder, consuming 6-10 drinks of whiskey every day. Liver functions, amylase, and lipase are elevated while renal function shows elevated BUN with normal creatinine. What is the best next step in the management of this patient?

Choices: 1. Reassure and discharge with oral ibuprofen 2. IV fluids and observe inpatient 3. CT of the abdomen and pelvis 4. Urgent upper endoscopy

Answer: 3 - CT of the abdomen and pelvis Explanations: This patient has hallmark signs and symptoms of peritonitis. The patient usually is in tremendous pain, vague, associated with nausea and vomiting. They usually prefer to lie still. Given his history of prior epigastric abdominal pain, nausea, and vomiting in the background of chronic alcohol use, it appears to be an acute episode of self-resolving pancreatitis. A pseudocyst is a common complication seen in patients with alcoholic pancreatitis. Spontaneous regression is usually seen, but in a few cases, complications such as perforation have been reported. Management will include immediate diagnosis, stabilizing the patient, and urgent surgical consultation. Peritonitis requires inpatient management, surgical intervention if needed. To send the patient home will be unethical. Although IV fluids and stabilizing the patient is part of the plan, urgent accurate diagnosis of the location and size of perforation is important. Upper endoscopy has no role in the management of peritonitis. Go to the next page if you knew the correct answer, or click the link image(s) below to further research the concepts in this question (if desired).

Research Concepts: Pancreatic Pseudocyst

We update eBooks quarterly and Apps daily based on user feedback. Please tap flag to report any questions that need improvement.

Question 559: A 33-year-old woman with a BMI of 35 kg/m2 is brought to the emergency department for evaluation of shortness of breath of one-day duration. The patient is 33 weeks pregnant and has a sedentary lifestyle. Her vital signs on evaluation show blood pressure 150/94 mmHg, heart rate 110/min, respiratory rate 22/min, and temperature 100 F. The pulse oximetry is 91% on room air. On physical examination, she is in mild respiratory distress and able to complete full sentences. The chest is clear on examination, heart rate is regular, the abdomen is soft, non-tender, fundal height is 33 cm, and she has 1+ pedal pitting edema. Blood counts, renal function, liver function, electrolytes, and troponin are unremarkable. D-dimer is 1100 ng/ml. EKG shows sinus tachycardia without any significant ST segment or T wave changes. The chest x-ray is unremarkable. Ultrasound doppler for lower extremity is negative. Lung V/Q scan shows a high probability of pulmonary embolism. Which of the following is the next best step in the management of this patient?

Choices: 1. Therapeutic low molecular weight heparin 2. Unfractionated heparin (UFH) infusion 3. Thrombectomy 4. Apixaban

Answer: 1 - Therapeutic low molecular weight heparin Explanations: Treatment of choice of venous thromboembolism in pregnancy is low molecular weight heparin (LMWH). It’s more efficacious and has a better safety profile than intravenous unfractionated heparin. However, LMWH should be discontinued at least 24 hours prior to delivery, and postpartum treatment should be started as soon as hemostasis is achieved. Anticoagulation therapy should be continued for at least 6 weeks postpartum. The goal of the total anticoagulation period is 3-6 months. A subset of pregnant patients who have a higher risk of recurrent PE (acute PE within the last month) cannot be left without anticoagulation for 24-36 hours. These patients may benefit from intravenous unfractionated heparin (UFH) within 24 hours of delivery, which can be discontinued 4-6 hours prior to delivery. It is also preferred in patients with an elevated risk of bleeding or persistent hypotension. The rationale is UFH has a shorter halflife, and a near-complete reversal is possible with protamine. Thrombolytic therapy raises the risk of maternal hemorrhage and should only be reserved for life-threatening acute pulmonary embolism. The safety profile of novel oral anticoagulants (NOACs) has not been studied well for pregnancy. Go to the next page if you knew the correct answer, or click the link image(s) below to further research the concepts in this question (if desired).

Research Concepts: Lung Ventilation Perfusion Scan (VQ Scan)

We update eBooks quarterly and Apps daily based on user feedback. Please tap flag to report any questions that need improvement.

Question 560: A 50-year-old woman is undergoing right knee arthroscopy. The procedure goes as planned, and a postoperative adductor canal block is being given. The area is prepped and draped, and due to the patient's body habitus, visualization proves to be difficult. Bupivacaine is administered, and the patient immediately becomes severely hypotensive and appears to have a wide-complex tachyarrhythmia on the monitor. Which of the following is the best initial step in the management of this patient?

Choices: 1. Initiate immediate intravenous lipid emulsion (ILE) therapy 2. Perform gastric lavage 3. Start advanced cardiac life support 4. Arrange for hemodialysis

Answer: 3 - Start advanced cardiac life support Explanations: ACLS guidelines are the most appropriate initial step to follow in hemodynamically unstable patients. intravenous lipid emulsion (ILE) therapy is the standard treatment for local anesthetic systemic toxicity (LAST) according to the ASRA professional society. ILE may be an adjunct therapy to ACLS for cardiac arrest due to local anesthesia. It has been emerging in emergency rooms and critical care units as potential rescue therapy for many other acute toxicities and poisonings. Drug classes that have been investigated include tricyclic antidepressants, calcium channel blockers, beta-blockers, antipsychotics, insecticides, and organophosphates. Go to the next page if you knew the correct answer, or click the link image(s) below to further research the concepts in this question (if desired).

Research Concepts: Lipid Emulsion Therapy

We update eBooks quarterly and Apps daily based on user feedback. Please tap flag to report any questions that need improvement.

Question 561: A 75-year old woman is being evaluated on post-operative day one following laparoscopic cholecystectomy. Morning labs indicate an increase in her serum creatinine from 0.8 mg/dL to 1.5 mg/dL. Which of the following best approximates the renal mass lost for this much increase in serum creatinine in this patient?

Choices: 1. 10% 2. 20% 3. 50% 4. 70%

Answer: 3 - 50% Explanations: Perioperative acute kidney injury (AKI) is a serious yet under-recognized problem in patients who have recently undergone surgery. Due to increasing age and number of comorbidities, perioperative AKI is increasing in incidence and has significant morbidity and mortality. Serum BUN and creatinine levels increase late in the process of acute kidney injury (AKI) development, with increases in serum creatinine not being recognized until approximately 50% of the renal mass is lost. Innovations in laboratory analysis of AKI has determined the existence of AKI biomarkers that can be detected before renal mass is lost. These biomarkers are surrogates of early stress of the renal tubules. They are mobilized secondary to cardiopulmonary bypass and surgical stimulation and reflect underlying mechanisms of renal injury since different biomarkers are released in response to specific insults. Of note, insults due to G1 cell cycle arrest are associated with TIMP-2 and IGFBP-7 biomarkers and are predictive of major adverse kidney events, including death, persistent renal dysfunction, and need for dialysis. Early identification of high-risk patients is made using these biomarkers, with early application of KDIGO bundles. In patients for whom bundled care is initiated early, there are lower rates of moderate and severe AKI when compared to patients with AKI that are treated with standard therapy. Go to the next page if you knew the correct answer, or click the link image(s) below to further research the concepts in this question (if desired).

Research Concepts: Perioperative Acute Kidney Injury

We update eBooks quarterly and Apps daily based on user feedback. Please tap flag to report any questions that need improvement.

Question 562: A 70-year-old male patient presents to the emergency department (ED) with tetany and seizures. His EKG shows a prolonged QT interval. His laboratory tests are WBC: 6700 cells/mm^3, Hb: 14.8 mg/dl, and platelet: 167000 cells/microliter. Intravenous calcium gluconate is initiated. What is the next preferred management?

Choices: 1. Cardiac catheterization 2. Initiate aspirin 3. Check magnesium level 4. Reassure and discharge home

Answer: 3 - Check magnesium level Explanations: Hypomagnesemia can cause hypocalcemia, and hypocalcemia is difficult to correct without first normalizing magnesium. Replete magnesium level if magnesium concentration is less than 0.8 mEq/L. Hypocalcemia typically presents with tetany, seizures, and prolonged QT interval. Continuous observation after administering IV calcium gluconate every 4 to 6 hours is essential and the patient should not be discharged home. Go to the next page if you knew the correct answer, or click the link image(s) below to further research the concepts in this question (if desired).

Research Concepts: Calcium Gluconate

We update eBooks quarterly and Apps daily based on user feedback. Please tap flag to report any questions that need improvement.

Question 563: A 62-year-old male is brought to the emergency department following a witnessed seizure in the grocery store. He is currently alert, and his orientation is improving. Finger-stick blood glucose upon arrival was 112 mg/dL, with no dextrose administered en route. Upon questioning, the clinician discovers this has never happened before, and he has no other significant medical or family history. He undergoes a CT of the head that demonstrates a frontal lobe parenchymal mass with scattered coarse and gyriform calcifications. After it is determined there isn't any enhancement on MRI, what is the next test that will aid the most in making the diagnosis?

Choices: 1. Magnetic resonance spectroscopy 2. Tumor genetic testing 3. Tumor histologic markers 4. Tumor cytology and architecture

Answer: 2 - Tumor genetic testing Explanations: A nonenhancing, parenchymal, seizure producing central nervous system tumor is most likely a low-grade glioma in a patient without a history of cancer. Of these, a frontal lobe mass with coarse calcifications is most likely an oligodendroglioma. The best way to determine this from other gliomas is through genetic marker testing. Oligodendrogliomas are defined by an isocitrate dehydrogenase (IDH) mutation and a 1p/19q codeletion. This combination is absent in the other gliomas such as astrocytoma and glioblastoma. Genetic testing plays a prognostic role in diagnosing these tumors, as those that have a mutated IDH and 1p/19q codeletion respond better to chemotherapy, and are less aggressive than those with wild-type IDH and lacking the codeletion. Oligodendrogliomas often share many histological markers with the other glial tumors, such as glial fibrillary acidic protein (GFAP) found in astrocytomas. Their cytology and cell architecture is not definitive in diagnosing these tumors. Go to the next page if you knew the correct answer, or click the link image(s) below to further research the concepts in this question (if desired).

Research Concepts: Oligodendroglioma

We update eBooks quarterly and Apps daily based on user feedback. Please tap flag to report any questions that need improvement.

Question 564: A 45-year-old male who works as a painter and is found obtunded at work. He has a negative head CT, negative urine toxicology, negative ethanol and the following lab values: Na 138 mEq/L, K 5.2 mEq/L, Cl 101 mEq/L, HCO3 12 mEq/L, BUN 15 mg/dL, creatinine 1.1 mg/dL, glucose 96 mg/dL, ABG: PO2 89, PCO2 26, pH 7.19. Which diagnostic test should be ordered?

Choices: 1. Ketones 2. Urinary potassium 3. Drug screen 4. Plasma osmolality

Answer: 4 - Plasma osmolality Explanations: The patient has a compensated (PCO2 should be 1.5(HCO3)+8 for compensation which is the case) anion gap (AG=32=Na-(Cl+HCO3)) metabolic acidosis and with signs of intoxication. The next step is to measure the plasma osmolality and compare it to the estimated osmolality which is 287 (2Na+(BUN/2.8)+(glucose/18)+(blood ethanol/4.6)) to determine the osmolar gap (measured osmolality-calculated osmolality=osmolar gap) which, in normal cases, is less than 10 so a higher gap is due to low molecular weight molecules such as ethylene glycol, methanol, acetone, and ketones that are not accounted for in the equation. Methanol is used in paint thinners and is a likely culprit in this case. Ketoacidosis is a cause of anion gap metabolic acidosis, but given the normal blood glucose in this case, it is unlikely. Go to the next page if you knew the correct answer, or click the link image(s) below to further research the concepts in this question (if desired).

Research Concepts: Methanol Toxicity

We update eBooks quarterly and Apps daily based on user feedback. Please tap flag to report any questions that need improvement.

Question 565: A 17-year-old male presents to the emergency department with a history of being involved in a motor vehicle collision. He currently has no external injury or visible signs. He mentions difficulty and pain with swallowing and changes in his voice. What is the best time to perform a CT scan to evaluate his voice changes?

Choices: 1. Within 24 hours 2. After one week of vocal rest 3. After two weeks of vocal rest 4. Never

Answer: 1 - Within 24 hours Explanations: It is recommended to perform CT within 24 hours of trauma as a laryngeal fracture is frequently underdiagnosed. Triad of odynophagia, dysphagia, and dysphonia should always raise a high degree of suspicion. CT scan is the most important diagnostic tool. CT can differentiate between displaced and undisplaced laryngeal fractures and can make a difference in management. Early diagnosis and treatment will significantly help prevent complications. Go to the next page if you knew the correct answer, or click the link image(s) below to further research the concepts in this question (if desired).

Research Concepts: Laryngeal Fracture

We update eBooks quarterly and Apps daily based on user feedback. Please tap flag to report any questions that need improvement.

Question 566: A 56-year-old man with a past medical history significant for Graves disease status post thyroidectomy, type 2 diabetes mellitus, chronic kidney disease stage III, and hypertension is admitted with complaints of confusion, bilateral pedal edema, and abdominal distention. On physical examination, the patient is found to have bilateral pitting edema, severe ascites, and icterus. He is drowsy and lethargic. Labs reveal platelet count of 70,000/microL, PT 27 seconds, INR of 2.1, AST 389 IU/L, ALT 206 IU/L, total bilirubin 6.2 mg/dL, gamma globulin 6.8 mg/dL, and albumin 2.3 mg/dL. Ammonia levels are elevated at 62 mmol/L. Ultrasound of the abdomen shows a nodular liver with a strong suspicion of cirrhosis. Both the serum antinuclear antibody (ANA) and anti-smooth muscle antibody (ASMA) come back positive. The patient is diagnosed to have autoimmune hepatitis. The patient is started on lactulose, which shows only a slight improvement in his ammonia levels and mental status in 72 hours. He is given IV furosemide and has minimal diuresis and continues to have findings of severe ascites and significant pedal edema. Which of the following is the next best step in the management of this patient?

Choices: 1. High-dose prednisone and azathioprine 2. Liver transplantation 3. Continue current treatment and monitor response 4. Obtain liver biopsy

Answer: 2 - Liver transplantation Explanations: The mainstay of treatment is around and has been the use of corticosteroids alone or in combination with azathioprine. The majority of patients respond to initial therapy and go into remission; however, once the treatment is stopped, the majority of patients relapse as well. However, patients who have pre-existing comorbid conditions, and those who have endstage decompensated liver disease are usually referred for liver transplantation and are not started on medications. This patient has significant co-morbid conditions and also appears to have end-stage decompensated liver disease, as evidenced by his severe ascites, hepatic encephalopathy, bilateral lower extremity pedal edema, and lab and imaging abnormalities. He also has not responded very well to the treatment in 72 hours. This patient should be referred for liver transplantation. The initiation of steroids and azathioprine is not indicated at this time. He also has not responded very well to the treatment in 72 hours and is unlikely to respond to the continuation of the same treatment. A liver biopsy can be done to find out the severity of disease, but the patient is unstable to undergo an invasive procedure like a liver biopsy at this time. As evidenced by his severe ascites, hepatic encephalopathy, bilateral lower extremity pedal edema, lab values, and ultrasound of the abdomen showing a strong suspicion for cirrhosis, he appears to have end-stage decompensated liver disease already and liver biopsy would not be very helpful at this time in aiding the diagnosis. Therefore he should be referred for liver transplantation. Go to the next page if you knew the correct answer, or click the link image(s) below to further research the concepts in this question (if desired).

Research Concepts: Hepatitis

We update eBooks quarterly and Apps daily based on user feedback. Please tap flag to report any questions that need improvement.

Question 567: A 75-year-old man presents to the hospital with complaints of generalized fatigue and shortness of breath for the past two weeks. The symptoms started gradually and have become worse with time. A detailed medical history reveals that he has been diagnosed with diabetes mellitus about thirty years ago and is on metformin. His vital signs show blood pressure: 85/65 mmHg, pulse rate: 112 beats per minute, temperature: 39 C (102.2 F), respiratory rate: 28 breaths per minute, and oxygen saturation: 89% at room air. A physical examination was carried out, which reveals a decrease of 20 mmHg of blood pressure on inspiration. Furthermore, abdominal examination reveals hepatomegaly and peripheral edema. Echocardiography was carried out, which shows an echolucent free space between the visceral and parietal pericardium. What physical finding, in addition to these, is most likely to be found in this case?

Choices: 1. Biot respiration 2. Cheyne–Stokes respiration 3. Kussmaul sign 4. Crescendo-decrescendo murmur

Answer: 3 - Kussmaul sign Explanations: The clinical scenario is consistent with the diagnosis of effusive-constrictive pericarditis. Constrictive pericarditis is a disease involving scarring and loss of elasticity of the pericardium surrounding the heart, leading to impaired filling. Effusive-constrictive pericarditis (ECP) is a syndrome involving both constriction of the visceral pericardium and an effusion causing a tamponade-like effect on the heart. It presents with symptoms of volume overload such as shortness of breath, ascites, and peripheral edema. Tachycardia occurs reflexively due to decreased cardiac output. There are many findings on physical examination. One of which is Kussmaul sign, a rise in right atrial pressure, and eventually the jugular venous pressure during inspiration. Other physical findings include dyspnea, hepatomegaly, an added heart sound, and pulsus paradoxus, which refers to a drop in blood pressure of greater than 10mmHg during inspiration. Go to the next page if you knew the correct answer, or click the link image(s) below to further research the concepts in this question (if desired).

Research Concepts: Constrictive-Effusive Pericarditis

We update eBooks quarterly and Apps daily based on user feedback. Please tap flag to report any questions that need improvement.

Question 568: Which of the following is the correct statement about duplex scanning for thrombosis of the inferior vena cava (IVC)?

Choices: 1. The proximity of the IVC to the abdominal wall posteriorly makes the procedure technically easy 2. Respiratory variation and compressibility of the IVC are not as useful as in duplex scanning of the femoral system 3. The large caliber of the IVC makes duplex scanning here more reliable than at the femoral level 4. Only the IVC and iliac systems are visualized with duplex in the abdomen

Answer: 2 - Respiratory variation and compressibility of the IVC are not as useful as in duplex scanning of the femoral system

Explanations: The cardiac cycle has more effect than respiratory variation in the IVC above the renal veins. The IVC is not compressible as the femoral system is. Visualization of dilated collaterals may be of value in diagnosis of IVC thrombosis. The disadvantages of duplex in this situation include anatomic limitations and operator dependence. Go to the next page if you knew the correct answer, or click the link image(s) below to further research the concepts in this question (if desired).

Research Concepts: Inferior Vena Caval Thrombosis

We update eBooks quarterly and Apps daily based on user feedback. Please tap flag to report any questions that need improvement.

Question 569: A 32-year-old man presents to the clinic with a 5-month history of arthralgia, asthenia, decreased libido, cramps, nocturia, dysuria, post-void dribbling, and foamy urine. He has a history of chronic exposition to paraquat and glyphosate for 15 years in his agricultural work in EL Salvador, Central America before migrating to the United States. He takes no medications. An initial set of investigations is shown below. Patient value Reference range Serum sodium 128 mEq/L 134-144 mEq/L Serum potassium 2.7 mEq/L 3.6-5.0 mEq/L Serum chloride 92 mEq/L 98-107 mEq/L Serum 1.1 mg/dL 1.8-3.0 mg/dL magnesium Serum creatinine 2.9 mg/dL 0.6-1.2 mg/dL Serum BUN 98 mg/dL 5-20 mg/dL Urine protein 0.3 g/24 hours 0.15 g/24 hours Which of the following additional lab findings is most likely to be seen in this patient?

Choices: 1. Urine sediment showing dysmorphic erythrocytes 2. Reduced electrolytes in urine and blood 3. Increased osmolality in blood 4. Elevated fractional excretion of sodium and magnesium

Answer: 4 - Elevated fractional excretion of sodium and magnesium Explanations: Electrolyte loss in urine, primarily magnesium, phosphorus, sodium, and potassium— beginning in the early stages of the disease, signifies primarily tubular damage as the initial site of renal damage, which once more points to chronic tubulointerstitial nephropathy. The proximal tubule reabsorbs 60% of electrolytes, which is why this must be the segment most involved. This electrolyte loss explains the electrolyte polyuria and low concentrations of some electrolytes in the blood, as well as the symptoms of cramping and fainting. Elevated fractional excretion of electrolytes is found in CINAC with a low concentration of electrolytes in the blood. Dysmorphic erythrocytes are not described in CINAC. The osmolality in blood in CINAC decreases due to increased fractional excretion of electrolytes in urine. Go to the next page if you knew the correct answer, or click the link image(s) below to further research the concepts in this question (if desired).

Research Concepts: Chronic Interstitial Nephritis in Agricultural Communities (CINAC)

We update eBooks quarterly and Apps daily based on user feedback. Please tap flag to report any questions that need improvement.

Question 570: A 28-year-old male patient presents to the emergency department of tertiary care hospital with complaints of shortness of breath. Upon further inquiry, the patient states that he works in an industry that is involved in manufacturing generators that utilize a gas that consists of three atoms of oxygen (O) linked in a cyclic structure. On physical examination, his vital measurements indicate that he has a blood pressure of 120/80 mmHg, a heart rate of 94 beats per minute, and a breathing rate of 31 breaths per minute. After the initial assessment and evaluation, a chest radiograph has been ordered. Which of the following other tests should be considered in the initial emergency evaluation of this patient?

Choices: 1. Electrocardiogram 2. Complete blood count 3. Arterial blood gases 4. Metabolic panel with an anion gap

Answer: 3 - Arterial blood gases Explanations: The history of working in a generator manufacturing factory that utilizes a gas that consists of three atoms of oxygen (O) linked in a cyclic structure along with respiratory complaints are suggestive of ozone toxicity. Arterial blood gas and chest radiography should be performed in such patients as exposure to other pulmonary irritants are possible. Ozone can lead to the formation of other toxic substances such as formaldehyde when it reacts with other chemicals from a multitude of products. Evaluation of ozone toxicity is similar to the evaluation of any pulmonary irritant. Oxygen saturation monitoring should be implemented and also bedside spirometry. An electrocardiogram should be performed as well in patients with or at risk for the underlying cardiac disease. Go to the next page if you knew the correct answer, or click the link image(s) below to further research the concepts in this question (if desired).

Research Concepts: Ozone Toxicity

We update eBooks quarterly and Apps daily based on user feedback. Please tap flag to report any questions that need improvement.

Question 571: A 17-year-old male presents with severe headaches. He emigrated from Mexico 5 months ago. He noted headaches about a month ago, on the frontal region, initially graded 3/10, and gradually progressed to 7/10. This is more prominent in the mornings and accompanied by nausea. He also claims that for the past week, he has been having vivid visual hallucinations of animals like goats and insects in his room, which he can describe in great detail. On examination, BP 140/90 mmHg, HR 54 beats/min, RR 18 breaths/min, T 99.6 F. His pupils are isocoric, briskly reactive to light, there are no visual field cuts, and fundoscopy reveals bilateral papilledema. There is no facial asymmetry or dysarthria. The motor and sensory examinations are normal. There is positive nuchal rigidity, Brudzinski, and bilateral Babinski. The head CT was normal. The chest x-ray shows pleural apical thickening.The CSF examination reveals: an opening pressure of 23 cm/H2O, WBC 345/mm^3, 85% lymphocytes, glucose 23 mg/dL, and protein 90 mg/dL. Which of the following etiologic agents is most likely?

Choices: 1. Mycobacterium tuberculosis 2. Cryptococcus neoformans 3. Herpes simplex 4. Rickettsia rickettsia

Answer: 1 - Mycobacterium tuberculosis Explanations: Tuberculous meningitis (TBM) is a manifestation of extrapulmonary tuberculosis caused by the seeding of the meninges with the bacilli of Mycobacterium tuberculosis (MTB). Tuberculous meningitis presents 1% of all cases of extra-pulmonary TB. In the developed world where there is a lower prevalence of TB in the population, estimates are that TBM accounts for 6% of all causes of meningitis. In locations with a higher prevalence of MTB in the population, estimates are that TBM accounts for up to one third to one half of all bacterial meningitis. Tuberculous meningitis assessment is by obtaining cerebrospinal fluid (CSF) for analysis. Typically, the CSF reveals low glucose, elevated protein, and modestly elevated WBC count with a lymphocytic predominance. The CSF analysis most closely resembles the CSF analysis of viral meningitis. Go to the next page if you knew the correct answer, or click the link image(s) below to further research the concepts in this question (if desired).

Research Concepts: Tuberculous Meningitis

We update eBooks quarterly and Apps daily based on user feedback. Please tap flag to report any questions that need improvement.

Question 572: A 65-year-old man presents with a complaint of left-sided chest pain radiating to the left arm for the last three hours. His past medical history is significant for diabetes and hyperlipidemia. Initial vitals showed a pulse of 110/min, blood pressure of 80/40 mmHg, respiratory rate of 16/min, and oxygen saturation of 96% on room air. Physical examination is unremarkable. The electrocardiogram shows ST elevation in the lead II and aVF with ST depression in V1-V3. The patient is diagnosed with inferior wall myocardial infarction. After initial stabilization, the patient is rushed to the catheterization lab. A large thrombus is noted in the proximal right coronary artery on the coronary angiogram. What is the most appropriate initial management of hypotension in this patient?

Choices: 1. Epinephrine 2. Dopamine 3. Intravenous fluids 4. Dobutamine

Answer: 3 - Intravenous fluids Explanations: Right ventricular infarction complicates up to 40% of inferior ST-elevation myocardial infarction. Intravenous fluids are essential to improve the right ventricle preload in the right ventricular infarct. Early revascularization is vital to improve the right ventricle infarct. Norepinephrine and dobutamine should be considered for right ventricular failure when cardiogenic shock persists after intravenous fluids but not for first-line management. Go to the next page if you knew the correct answer, or click the link image(s) below to further research the concepts in this question (if desired).

Research Concepts: Coronary Artery Thrombus

We update eBooks quarterly and Apps daily based on user feedback. Please tap flag to report any questions that need improvement.

Question 573: Patient's with mitral stenosis commonly have elevated pressures of which of the following?

Choices: 1. Left ventricle 2. Pulmonary veins 3. Aorta 4. Peripheral arteries

Answer: 2 - Pulmonary veins Explanations: Pulmonary venous pressure is elevated in mitral valve stenosis. In patients with mitral stenosis, the blood from the left atrium is unable to enter the left ventricle. Over time, this results in an enlarged left atrium and the backward pressure results in elevations in pulmonary venous pressure. The Left atrium can be massively enlarged and push on the trachea and lead to a chronic cough. Go to the next page if you knew the correct answer, or click the link image(s) below to further research the concepts in this question (if desired).

Research Concepts: Mitral Stenosis

We update eBooks quarterly and Apps daily based on user feedback. Please tap flag to report any questions that need improvement.

Question 574: A 65-year-old patient on hospice whose family member pages the nurse stat to have a healthcare team member evaluate distressing symptoms after administration of glycopyrronium. Which of the following underlying diseases is least likely to have predisposed this patient to an adverse effect of antisialagogue?

Choices: 1. Unstable ischemic heart disease 2. Narrow-angle glaucoma 3. End-stage renal disease on dialysis 4. Chronic stable atrial fibrillation

Answer: 4 - Chronic stable atrial fibrillation Explanations: Cardiovascular effects (e.g., atrial fibrillation, tachycardia) may occur after administration. Caution must be exercised in patients with a history of long QT syndrome or whose QTc is prolonged at the screening. Patients with narrow-angle glaucoma are at increased risk of adverse effects including halos, headache, severe eye pain, decreased vision, and nausea/vomiting. Patients with severe renal impairment including end-stage renal disease on dialysis are at a highly increased risk of severe systemic exposure to glycopyrrolate. This agent should be used with caution in patients with unstable ischemic heart disease, left ventricular failure, history of myocardial infarction, and arrhythmia (excluding chronic stable atrial fibrillation). Chronic stable atrial fibrillation confers the lowest risk of the choices to precipitate adverse effects. Go to the next page if you knew the correct answer, or click the link image(s) below to further research the concepts in this question (if desired).

Research Concepts: Antisialagogues

We update eBooks quarterly and Apps daily based on user feedback. Please tap flag to report any questions that need improvement.

Question 575: A 35-year old man is brought to the emergency department after an intentional drug overdose. He is unconscious with GCS of 9. His friends report that he had been feeling depressed for the past few weeks. On examination, the patient has a blood pressure of 90/60 mm Hg, rapid, weak pulse, respiratory rate of 7/min, and dilated pupils. Further investigation reveals that the patient has taken a large amount of sedative-hypnotic drug that acts on GABA receptors. Which of the following is the mainstay of treatment for this patient?

Choices: 1. Supportive management focused on blood pressure and respiratory status 2. Administration of naloxone 3. Hyperbaric oxygen therapy 4. Administration of atropine

Answer: 1 - Supportive management focused on blood pressure and respiratory status Explanations: Barbiturate overdose results in cold, clammy skin, shallow respirations, dilated pupils, weak and rapid pulse, sedation, coma, and even death. Treatment should focus on supporting blood pressure and respiratory status. The presentation for barbiturate toxicity is somewhat nonspecific with overlap with many other drugs, including alcohol, benzodiazepines, and anticonvulsants. It is important to perform toxicological studies to determine the concentration of drugs in the body, as well as confirm that a barbiturate was taken. Patients may have taken other drugs in conjunction with the barbiturate, and this may affect treatment. Go to the next page if you knew the correct answer, or click the link image(s) below to further research the concepts in this question (if desired).

Research Concepts: Barbiturate Toxicity

We update eBooks quarterly and Apps daily based on user feedback. Please tap flag to report any questions that need improvement.

Question 576: A 65-year-old male is in the ICU after sustaining an extensive myocardial infarction one week ago. He is making a slow recovery. He complains of a new-onset severe right upper quadrant abdominal pain. He's had very little by mouth during his recovery. His white cell count is 18,000 and total bilirubin is 3. The abdominal exam shows moderate right upper quadrant pain with fullness in that area. The ultrasound shows a distended gallbladder with inflammatory fluid and a thickened gallbladder wall but no gallstones. What is the best treatment for this patient?

Choices: 1. Laparoscopic cholecystectomy 2. Open cholecystectomy 3. Close observation 4. Percutaneous cholecystostomy tube

Answer: 4 - Percutaneous cholecystostomy tube Explanations: The best definitive treatment for a stable patient would be laparoscopic cholecystectomy. Percutaneous cholecystostomy tube will decompress the acute distended gallbladder in an unstable patient who is a poor surgical risk. Often times the acute gallbladder will culture Klebsiella, Strep, Staph, or E. coli. Unstable patients who are poor surgical candidates may recover without incident when a percutaneous cholecystostomy tube is utilized and may not require a future cholecystectomy, especially in the setting of acalculous cholecystitis. Go to the next page if you knew the correct answer, or click the link image(s) below to further research the concepts in this question (if desired).

Research Concepts: Acalculous Cholecystitis

We update eBooks quarterly and Apps daily based on user feedback. Please tap flag to report any questions that need improvement.

Question 577: A 40-year-old African American female is brought in by her husband, complaining that she has been drinking a lot of water recently. She has been urinating numerous times in a day. For two days, she has become drowsy and not been herself as per her husband. She suffered a traumatic brain injury after falling from the back of a truck 4 months ago and has had some cognitive deficits since. She also has a history of sarcoidosis, diabetes mellitus, and hypertension. Her medications include hydrochlorothiazide, metformin, and hydroxychloroquine. She has not had a flare-up of sarcoidosis recently. Her blood pressure is 140/92 mmHg. Lab work is significant only for serum sodium 108 mEq/L. She is admitted for the management of hyponatremia. She is administered 3% hypertonic saline at 50 mL/h in the ICU. Serum sodium is 117 mEq/L 4 hours later. What is the next best step in the management of this patient?

Choices: 1. Infuse sterile water through the intravenous line 2. Infuse dextrose 5% water 3. Continue hypertonic saline but at a decreased rate of 25 mL/h 4. Administer tolvaptan

Answer: 2 - Infuse dextrose 5% water Explanations: The rapid rise in serum sodium in a chronic hyponatremic patient can cause central pontine myelinolysis. Central pontine myelinolysis is caused by dehydration of the brain tissue and separation of the myelin sheath from the axons due to higher osmolality in the serum when compared to the neuron sheaths. The correct answer here is to infuse D5 water, which is an isotonic solution to decrease serum sodium level since the serum sodium level in this patient has increased too quickly by 9 mEq/L in a span of four hours. D5 water has no sodium in it, thereby diluting and decreasing the sodium level in the blood. If the serum sodium is increasing too fast, it should be slowed down or reversed, to decrease the rate of correction. Sodium should not increase by more than 10-12 mEq/L in 24 hours. Central pontine myelinolysis results in neurological deficits like altered mental status, difficulty with the ability to speak and swallow, weakness of body parts, and decrease in sensation, etc. Infusion of sterile water can cause hemolysis due to the hypotonicity of the fluid. Decreasing hypertonic saline will still make the sodium level go up. Administering tolvaptan, which is a vasopressin antagonist, will also make the sodium level go up, which is not desired in this case. Go to the next page if you knew the correct answer, or click the link image(s) below to further research the concepts in this question (if desired).

Research Concepts: Primary Polydipsia

We update eBooks quarterly and Apps daily based on user feedback. Please tap flag to report any questions that need improvement.

Question 578: A 23-year-old man presents to the emergency department with severe right upper abdominal pain and abdominal swelling. Laboratory investigations reveal AST 341 U/L, ALT 455 U/L, and lactic acid 4.1 mg/dL. The obliteration of the hepatic vein is evident in the abdominal ultrasound. The patient was recently discharged from the hospital following an episode of minimal change disease. Which of the following is the most likely responsible for this unusual site of thrombosis?

Choices: 1. Factor V Leiden mutation 2. Protein C deficiency 3. Protein S deficiency 4. Antithrombin III deficiency

Answer: 4 - Antithrombin III deficiency Explanations: This patient is likely suffering from thrombosis of the hepatic vein following the acquired deficiency of antithrombin III due to renal loss. Antithrombin III binds to heparin on endothelial cells and forms a complex with thrombin (thrombin-antithrombin (TAT) complex) thus inhibiting coagulation. Its deficiency may present as early age thrombosis (less than 50 years old) and carries the highest risk for thrombotic events among the inherited thrombophilias. ATIII deficiency can occur as a consequence of reduced synthesis (liver damage) or increased loss (nephrotic syndrome, enteropathy, DIC, sepsis, burn, trauma, microangiopathy, and cardiopulmonary bypass surgery). The prothrombin gene mutation, factor V Leiden mutation, deficiencies in protein S, protein C, and antithrombin account for most of the remaining cases. Go to the next page if you knew the correct answer, or click the link image(s) below to further research the concepts in this question (if desired).

Research Concepts: Hypercoagulability

We update eBooks quarterly and Apps daily based on user feedback. Please tap flag to report any questions that need improvement.

Question 579: A patient is involved in a motor vehicle accident at 100 mph. The patient was a restrained driver and is clutching his chest secondary to chest pain. There are muffled heart sounds, jugular venous distension, pulsus paradoxus, and the impression of the steering wheel on his chest. Blood pressure is 85/55 mmHg. What should be done?

Choices: 1. CT of the chest 2. Pericardiocentesis or a a wide subxiphoid pericardial window 3. Chest x-ray 4. Cardiac enzymes and EKG

Answer: 2 - Pericardiocentesis or a a wide subxiphoid pericardial window Explanations: The patient has pericardial tamponade. There is no time for the confirmative study of transesophageal echocardiography. Beck's triad is jugular venous distension, hypotension, and muffled heart tones. Depending on the skill level of the physician start with pericardiocentesis or if possible evacuation of blood and clots by a wide subxiphoid pericardial window. This is followed by a very quick median sternotomy when repair of the associated cardiac injuries is necessary. Emergency department thoracotomy can be considered if the patient develops cardiac arrest. Cardiac tamponade can be identified during the FAST exam. Go to the next page if you knew the correct answer, or click the link image(s) below to further research the concepts in this question (if desired).

Research Concepts: Cardiac Tamponade

We update eBooks quarterly and Apps daily based on user feedback. Please tap flag to report any questions that need improvement.

Question 580: A 17-year-old male patient presents to the emergency department with a 3hour history of depersonalization, blurry vision, slurred speech, an ataxic gait, and hallucinations. The patient was at a party shortly before his symptoms began. His blood pressure is 170/92 mmHg, pulse 120/min, and temperature 99.9 F (37.7 C). On physical examination, the patient is aggressive, combative, and poorly cooperative. His skin is diffusely erythematous, pupils are 8mm bilaterally, and bilateral nystagmus is present. Urine toxicology confirms the diagnosis. The patient is most likely to also experience which of the following symptoms?

Choices: 1. Homicidal impulses 2. Increased appetite 3. Nosebleed 4. Decreased respiratory rate

Answer: 1 - Homicidal impulses Explanations: Phencyclidine (PCP) is a dissociative anesthetic that is a commonly used recreational drug. PCP is a crystalline powder that can be ingested orally, injected intravenously, inhaled, or smoked. PCP is a non-competitive antagonist to the NMDA receptor, which causes analgesia, anesthesia, cognitive defects, and psychosis. PCP blocks the uptake of dopamine and norepinephrine, leading to sympathomimetic effects such as hypertension, tachycardia, bronchodilation, and agitation. Patients intoxicated with PCP are at increased risk for suicidal or homicidal behaviors and impulses. Aggressive behavior, loss of boundaries, and combativeness are associated with PCP intoxication. Go to the next page if you knew the correct answer, or click the link image(s) below to further research the concepts in this question (if desired).

Research Concepts: Phencyclidine Toxicity

We update eBooks quarterly and Apps daily based on user feedback. Please tap flag to report any questions that need improvement.

Question 581: A 65-year-old male patient received epidural anesthesia for total knee arthroplasty with no complications during the procedure. The next day he started complaining of bilateral leg weakness that progressed to paralysis with no associated pain. What is the cause of the most likely diagnosis in this patient?

Choices: 1. Accidental dural puncture with an epidural needle 2. Nerve injury 3. Epidural space infection 4. Blood accumulation and ischemic compression of the spinal cord

Answer: 4 - Blood accumulation and ischemic compression of the spinal cord Explanations: Accidental puncture of the dura mater with the epidural needle or wet tap presents with a headache. Bilateral leg weakness is more consistent with the physical compression of the spinal cord that can be explained by hematoma formation. Nerve root injury can happen when providing epidural anesthesia. However, the presentation is usually unilateral and symptoms manifest in the muscular and dermatomal region that corresponds to the injured nerve root. The bilateral complete paralysis that the patient presents in this case, can be better explained by a lumbar epidural hematoma. The epidural abscess formation typically takes more than 24 hours and is accompanied by signs of infection. An epidural hematoma can present accurately as evidenced in this patient. An epidural hematoma can be formed by bleeding inside the epidural space. Ischemic compression of the spinal cord can cause permanent neurologic damage. Epidural hematoma is the most likely explanation for this patient considering the acute presentation, paralysis and lack of accompanying infection signs. Go to the next page if you knew the correct answer, or click the link image(s) below to further research the concepts in this question (if desired).

Research Concepts: Epidural

We update eBooks quarterly and Apps daily based on user feedback. Please tap flag to report any questions that need improvement.

Question 582: A 25-year-old woman presents with a severe bilateral headache. The symptoms started two days ago as a mild, unilateral headache, which she tried treating with paracetamol, but the symptoms slowly progressed and spread to include both sides of her head. She has no past medical history and is not prone to headaches or migraines. She takes a combined oral contraceptive pill. An urgent CT head with contrast is ordered, which demonstrates a cavernous sinus thrombosis. What cranial nerve palsy is most commonly associated with this condition?

Choices: 1. 3rd nerve palsy 2. 4th nerve palsy 3. 5th nerve palsy 4. 6th nerve palsy

Answer: 4 - 6th nerve palsy Explanations: Cavernous sinus thrombosis can cause several cranial nerve palsies, most including palsies in nerves 3 to 6. The most common, however, is a 6th nerve palsy. Patients typically present with fever, headache (50% to 90%), periorbital swelling and pain, and visual changes (photophobia, diplopia, loss of vision). In general, antimicrobial and antithrombotic therapies are primary considerations as thrombophlebitis can cause severe infection and death. Anticoagulation with unfractionated heparin (UFH) or low molecular weight heparin (LMWH) for several weeks to several months is recommended. Go to the next page if you knew the correct answer, or click the link image(s) below to further research the concepts in this question (if desired).

Research Concepts: Cavernous Sinus Thrombosis

We update eBooks quarterly and Apps daily based on user feedback. Please tap flag to report any questions that need improvement.

Question 583: A 79-year-old man is hospitalized in ICU for severe pneumonia. In addition to drug therapy, he is given non-invasive ventilation cycles with a face mask. At the beginning of the ventilation, the patient is very cooperative. After a few hours, there is a marked improvement in the respiratory picture with a good increase in oxygenation and reduced dyspnea. However, despite the improvement, the patient begins to be impatient, shakes, and removes the facial mask repeatedly. In a short time, he completely loses his temper, threatens the staff, and removes the venous line because he wants to leave the place that no longer recognizes to be the hospital where he was initially hospitalized. His heart rate is 57/min. Detailed evaluation reveals no obvious cause of the patient's behavior. His wife does, however, report a history of excessive alocohol use. Which of the following is the next best step in the management of this patient?

Choices: 1. Dexmedetomidine 2. Midazolam 3. Haloperidol 4. Rivastigmine

Answer: 2 - Midazolam Explanations: Although benzodiazepines can have a delirogenic effect, they are exclusively useful in patients with alcohol or sedative withdrawal, or for delirium resulting from seizures. Short-acting benzodiazepines such as midazolam, and lorazepam, must be preferred. Apart from managing delirium due to alcohol withdrawal, there is no evidence to support the use of benzodiazepines in the treatment of other types of delirium. Although rivastigmine, donepezil, and physostigmine have been proposed for ICU delirium management, evidence of effectiveness is currently very scarce. Dexmedetomidine is useful in adults under mechanical ventilation when hyperactive delirium can obstacle the weaning, but it is not recommended for this aim. Moreover, bradycardia in a serious side effect. Go to the next page if you knew the correct answer, or click the link image(s) below to further research the concepts in this question (if desired).

Research Concepts: ICU Delirium

We update eBooks quarterly and Apps daily based on user feedback. Please tap flag to report any questions that need improvement.

Question 584: A 24-year-old man with no significant past medical history is admitted to the ICU with severe acute respiratory distress syndrome (ARDS) secondary to influenza. His ABG shows Pa02 of 75 mmHg while on 100% FIO2. He is intubated and sedated with fentanyl and propofol. He is on continuous cisatracurium infusion, and his tidal volume is 6 ccs/kg ideal body weight. He is on inhaled nitric oxide at 40 PPM. He is on airway pressure release ventilation mode of mechanical ventilation. Today is day 5 of his intubation. He has been prone for 2 days with no improvement in his PaO2. Which of the following treatment modalities is most likely to provide the greatest mortality benefit for this patient?

Choices: 1. Use of paralytics 2. Inhaled nitric oxide 3. Airway pressure release ventilation 4. 6 ccs/kg ideal body weight of tidal volume (lung-protective mechanical ventilation strategy)

Answer: 4 - 6 ccs/kg ideal body weight of tidal volume (lung-protective mechanical ventilation strategy)

Explanations: Lung-protective mechanical ventilation strategy ( 6cc/kg IBW) as published in 2000 in NEJM by ARDS-NET group is the only strategy amongst the listed modality which has shown mortality benefit. The mechanism of action of lung-protective ventilation (6 ccs/kg IBW tidal volume) is by preventing further injury/ barotrauma to the already inflamed alveoli in ARDS. You may not see improvement in oxygenation however you still use lung-protective mechanical ventilation. The use of paralytics improves ventilator synchrony; however, in recent trials, it has shown to offer no mortality benefit. Inhaled nitric oxide improves oxygenation in ARDS patients by pulmonary vasodilation. However, it has not shown any mortality benefits in clinical trials. Airway pressure release ventilation is a rescue mode of mechanical ventilation, which has also not shown mortality benefits. Go to the next page if you knew the correct answer, or click the link image(s) below to further research the concepts in this question (if desired).

Research Concepts: Acute Respiratory Distress Syndrome

We update eBooks quarterly and Apps daily based on user feedback. Please tap flag to report any questions that need improvement.

Question 585: A 31-year-old woman presents to the hospital for worsening shortness of breath of 2 weeks duration. She works as a hairdresser and is pretty much on her feet all day. She has no significant past medical history and had cholecystectomy a year ago. She reports some mild parasternal chest pain that is nonpleuritic and nonradiating. Her only medication is an over the counter multivitamin. On examination, her vital signs are normal except for a heart rate of 110/min and oxygen saturation 89%. She also reports some mild left lower extremity pain with swelling and mild tenderness on palpation around the calf. Homan sign is positive. An EKG shows sinus tachycardia. D-dimer is elevated. Lower extremity doppler shows a nonocclusive thrombus involving the left saphenous vein. A stat chest CT angiogram confirms the presence of pulmonary emboli. A heparin infusion is started and the patient is admitted. The patient's oxygenation improves within 24 hours on the floor on supplemental oxygen, and she is transitioned to oral anticoagulation. What is the most appropriate duration of anticoagulation therapy for this patient?

Choices: 1. 3 months 2. 6 months 3. Indefinite 4. 6 to 12 months

Answer: 3 - Indefinite Explanations: Indefinite anticoagulation therapy is indicated in a patient with a first episode of unprovoked DVT, and unprovoked symptomatic pulmonary embolism. The risk of bleeding should be considered when deciding on indefinite anticoagulation treatment. Patients with a low risk of bleed should be anticoagualted indefinitely. For those with a higher risk of bleeding, indefinite anticoagulation is usually not adviced. The estimated lifetime risk for recurrent unprovoked thromboembolism is reduced when anticoagulation therapy is extended. Anticoagulation therapy for 3 months is recommended for patients with a provoked thromboembolic event if the inciting event is eliminated. Go to the next page if you knew the correct answer, or click the link image(s) below to further research the concepts in this question (if desired).

Research Concepts: Anticoagulation

We update eBooks quarterly and Apps daily based on user feedback. Please tap flag to report any questions that need improvement.

Question 586: A 16-year-old male patient presents to the hospital with the difficulty of breathing. His condition started a week ago with fever and cough. It was noticed that he is irritable and with bluish discoloration of his lips. His vital signs show a temperature of 40 C (104 F), pulse rate 110/min, respiratory rate 60 breaths/min, and oxygen saturation of 93%. He is lethargic. Lung examination shows symmetrical chest expansion, with deep intercostal retractions and crackles in both lung fields. Arterial blood gas examination shows pH 7.2, PCO2 60 mmHg, HCO3 24 mmol/L, PaO2 60 mmHg, and O2 saturation 90% on room air. The patient is intubated and is hooked to a mechanical ventilator with the following set-up on assist control mode FiO2 80%, tidal volume 195 ml (6 ml/kg), PEEP of 8 cmH2O. Chest x-ray shows diffuse bilateral opacities and infiltrates. PaO2/FiO2 ratio is 200 mmHg. Which of the following phases of his condition shows the repair process takes place by the restoration of epithelial and endothelial barriers and reestablishing the epithelial integrity with the absorption of the intraalveolar fluid that enhances functional recovery?

Choices: 1. Exudative phase 2. Transudative phase 3. Proliferative phase 4. Fibrotic phase

Answer: 3 - Proliferative phase Explanations: Acute respiratory distress syndrome (ARDS) is a common condition that is caused by pulmonary and extra-pulmonary pathologies, and it carries high rates of mortality and morbidity. The definition is non-cardiogenic pulmonary edema that occurs because of an inflammatory process that increases capillary membranes permeability leading to impairment in oxygenation and gas exchange presenting with hypoxemia and bilateral pulmonary infiltrates. There are three distinct phases in the development of ARDS: exudative, proliferative, and fibrotic phase. The first exudative phase that occurs over the first 7 to 10 days occurred after lung exposure to injury with subsequent activation of an inflammatory cascade leading to the accumulation of protein-rich fluid and hemorrhage secondary to alveolar endothelial and epithelial barriers damage. In the second proliferative phase, the repair process takes place by the restoration of epithelial and endothelial barriers and reestablishing the epithelial integrity with the absorption of the intra-alveolar fluid that enhances functional recovery. The fibrotic phase, which does not occur in all patients, is associated with fibrous tissue formation and linked to increased mortality and prolonged duration of mechanical ventilation. Go to the next page if you knew the correct answer, or click the link image(s) below to further research the concepts in this question (if desired).

Research Concepts: Respiratory Distress Syndrome

We update eBooks quarterly and Apps daily based on user feedback. Please tap flag to report any questions that need improvement.

Question 587: A 30-year-old man is brought to the emergency department after a road traffic accident. He has multiple bruises over the anterior chest and abdominal wall. His blood pressure is 80/40 mmHg, and his pulse rate is 110/min. The trachea is midline. Pulmonary capillary wedge pressure (PCWP) is 12 mmHg. After rapid administration of 1 L crystalloid, the PCWP is 22 mmHg, and the blood pressure is 85/45 mmHg, with a pulse rate of 124/min. Which of the following is most likely responsible for the patient's persistently low blood pressure?

Choices: 1. Myocardial contusion 2. Pneumothorax 3. Hypovolaemic shock 4. Neurogenic shock

Answer: 1 - Myocardial contusion Explanations: Hypotension and tachycardia following trauma are consistent with shock. The most common cause of shock in these settings is hypovolaemia. The PCWP is low in hypovolaemia. Following a fluid challenge, the PCWP increased with no corresponding change in blood pressure. These findings exclude the diagnosis of shock caused by hypovolaemia. Elevated CVP/PCWP with persistent hypotension after fluid bolus should suggest an alternative diagnosis. Myocardial contusion should be suspected in this patient, which can be confirmed with elevated cardiac enzymes and ECG changes. The clinical findings are not suggestive of pneumothorax in this patient. Pulmonary embolism is associated with high pulmonary artery pressure but normal PCWP. Go to the next page if you knew the correct answer, or click the link image(s) below to further research the concepts in this question (if desired).

Research Concepts: Pulmonary Capillary Wedge Pressure

We update eBooks quarterly and Apps daily based on user feedback. Please tap flag to report any questions that need improvement.

Question 588: A 17-year-old patient presented to an emergency department following a sudden onset of severe headache. CT head revealed the presence of an acute subarachnoid hemorrhage within the interhemispheric cisterns with evolving hydrocephalus. The treating physician placed an external ventricular drain for managing acute hydrocephalus. However, ten minutes later, the patient deteriorated, and both his pupils were fixed and dilated. What is the most likely cause of this sudden deterioration in the patient?

Choices: 1. Blockage of the external ventricular drain 2. Rebleeding 3. Vasospasm 4. Pneumocephalus

Answer: 2 - Rebleeding Explanations: The major cause of mortality in patients with ruptured aneurysmal bleed is rebleeding of the aneurysm. The mortality in cases of re-rupture of an aneurysm is as high as 90%. The placement of external ventricular drain and egress of cerebral spinal fluid can reduce the tamponade effect from the hematoma onto the aneurysmal dome. This can paradoxically lead to the re-rupture of the aneurysm. Vasospasm leads to new clinical deficits but seldom leads to acute malignant intracranial hypertension. The onset of pneumocephalus in cases of acute hydrocephalus is minimal. Go to the next page if you knew the correct answer, or click the link image(s) below to further research the concepts in this question (if desired).

Research Concepts: Cerebral Aneurysm

We update eBooks quarterly and Apps daily based on user feedback. Please tap flag to report any questions that need improvement.

Question 589: A 67-year-old man is brought to the emergency department by EMS after the patient was noted to have multiple hematemesis episodes. He was intubated en route to the ED for airway protection. Based on the history obtained from the patient’s wife, the patient has a history of alcohol use disorder and reports drinking one bottle of whiskey daily for the past 30 years. On examination, the patient appears disheveled and is sedated and intubated. His heart rate is 120/min, blood pressure 70/50 mmHg, and respiratory rate is 28/min. Physical examination is suggestive of spider angiomata on the chest wall, scleral icterus, and abdominal distention. An initial set of investigations is shown below. Patient value Reference range Hemoglobin 6.8 g/dL 13.2-17.5 g/dL Platelets 90000/microL 150000-400000/microL Total bilirubin 3.3 mg/dL 0.3-1.0 mg/dL AST 240 IU/L 35 IU/L ALT 180 IU/L 10-35 IU/L ALP 183 IU/L 44-147 IU/L After initial resuscitation, the patient undergoes an esophagogastroduodenoscopy (EGD) examination that shows actively bleeding GOV1 varices not amenable to endoscopy intervention despite multiple attempts, and the performing endoscopist recommends urgent TIPS, which is only available in a hospital a few hours away. Treatment with multiple pressors is initiated due to hemorrhagic shock, and the pressor requirement is being escalated to maintain a MAP of 65 mmHg. Which of the following is the next best step in the management of this patient?

Choices: 1. Open surgical repair 2. Palliative care 3. Liver transplantation 4. Balloon tamponade with Linton-Nachlas tube

Answer: 4 - Balloon tamponade with Linton-Nachlas tube Explanations: This patient presented with acute gastric variceal hemorrhage secondary to active bleeding from underlying GOV1 varices. This patient is in hemorrhagic shock secondary to gastric variceal bleeding based on the requirement of multiple pressors. Considering that the patient failed endoscopy intervention, he should be referred for urgent TIPS placement. However, considering that the current hospital does not have interventional radiology support and the nearest hospital that can perform TIPS is a few hours away, the patient would immensely benefit from tamponade with Linton-Nachlas tube placement as a bridge to definite therapy. A surgical evaluation must be considered as a last resort. Go to the next page if you knew the correct answer, or click the link image(s) below to further research the concepts in this question (if desired).

Research Concepts: Gastric Varices

We update eBooks quarterly and Apps daily based on user feedback. Please tap flag to report any questions that need improvement.

Question 590: A 17-year-old male presents to the emergency department after ingesting slang nut in a suicide attempt. His vital signs include a blood pressure of 128/88 mmHg, pulse of 110/min, a temperature of 99.8 F, and a respiratory rate of 14/min. Treatment for this ingestion predominantly consists of which of the following?

Choices: 1. Gastric lavage and physostigmine 2. Exchange transfusion and methylene blue 3. Hemodialysis and atropine 4. Airway management and benzodiazepines

Answer: 4 - Airway management and benzodiazepines Explanations: Slang nut is the Southeast Asian plant that contains strychnos nux vomica, or strychnine toxin. Strychnine toxicity results in inhibition of the inhibitory neurotransmitter, glycine, causing uncontrolled muscle spasms. If a patient presents early before any clinical signs of ingestion, activated charcoal can be administered. Treatment is with benzodiazepines, decreasing stimulation, intubation, and neuromuscular blockade if spasms are uncontrolled, patients develop hyperthermia or significant rhabdomyolysis, or airway compromise occurs. Go to the next page if you knew the correct answer, or click the link image(s) below to further research the concepts in this question (if desired).

Research Concepts: Strychnine Toxicity

We update eBooks quarterly and Apps daily based on user feedback. Please tap flag to report any questions that need improvement.

Question 591: A 68-year-old female presents to the emergency department with a change in her mental status. Her past medical history is significant for drug use disorder and chronic obstructive pulmonary disease (COPD). Her family reports daily alcohol intake. On physical examination, her vital signs show a heart rate of 110 beats/minute, blood pressure of 160/90 mmHg, and oxygen saturation of 83%. She smells of alcohol and is disoriented. She is breathing without difficulty, but you notice some vomitus on the edge of her mouth. Her electrocardiogram shows ST depressions in leads V3-V5. Which of the following is an indication for the use of airway management?

Choices: 1. Patient's heart rate of 100 beats/minute 2. Patient's oxygen saturation of 83% 3. Patient's disorientation 4. ST depression in leads V3-V5

Answer: 2 - Patient's oxygen saturation of 83% Explanations: Indications for the use of airway management are failure to oxygenate, failure to ventilate, and failure to maintain a patent airway. Her oxygen saturation indicates the failure of oxygenation. The modality of airway management primarily depends on the cause and severity of the patient's condition but is also subject to factors such as environment and clinician skill. The patient is noted to be breathing without difficulty, so she is maintaining a patent airway, and disorientation per se is not an indication. The patient is tachycardic and has ST depressions in the lateral leads (V3-V5), but this would not, by itself, be a reason for airway management/intubation. These clinical issues do not fit the previously stated indications for the use of airway management. Go to the next page if you knew the correct answer, or click the link image(s) below to further research the concepts in this question (if desired).

Research Concepts: Airway Assessment

We update eBooks quarterly and Apps daily based on user feedback. Please tap flag to report any questions that need improvement.

Question 592: A 65-year-old male is hospitalized with persistent atrial fibrillation. He is started on enoxaparin 30 mg IV followed by 1 mg/kg SQ every 12 hourly to minimize the risk of ischemic infarction. How should the level of anticoagulation be followed in the patient so as to minimize the risk of abnormal bleeding?

Choices: 1. Partial thromboplastin time 2. Prothrombin time 3. No monitoring is necessary 4. Bleeding time

Answer: 3 - No monitoring is necessary Explanations: Enoxaparin is derived from the depolymerization of unfractionated heparin. It, however, retains UFH’s ability to activate antithrombin and thereby is effective for anticoagulation therapy. Anti-Xa activity level is recommended by the College of American Pathologists for monitoring the effect of LMWH. Bleeding time is used to assess platelet function. Prothrombin time is widely used to monitor warfarin. Go to the next page if you knew the correct answer, or click the link image(s) below to further research the concepts in this question (if desired).

Research Concepts: Enoxaparin

We update eBooks quarterly and Apps daily based on user feedback. Please tap flag to report any questions that need improvement.

Question 593: A 50-year-old woman from the South-West of the United States presents to the hospital with fever, headache, and malaise for the last three days. She also complains of intense pain and swelling on her inner thigh. Her vital signs show a heart rate of 110 beats per minute, blood pressure of 110/70 mmHg, and temperature of 101 F (38.3 C). Clinical examination also shows warm and non-fluctuant masses involving her inguinal lymph nodes. She reveals that she works for a spy agency and that she suspicious animal remains around her house in the preceding weeks, most likely dead rats. Which of the following is the best initial therapy for this patient?

Choices: 1. Gentamicin 2. Chloramphenicol 3. Amoxicillin 4. Ceftriaxone

Answer: 1 - Gentamicin Explanations: Aminoglycosides such as streptomycin or gentamicin are considered first-line treatment for seven to ten days in bubonic plague. More than 80 to 95% of Y. pestis infections usually present with suppurative adenitis, which is known as the bubonic plague. Symptoms include sudden fever, chills, headache, and malaise. Within a day or so, a bubo develops, starting as intense pain and swelling in the regional lymph node area, commonly inguinal, which is followed by the involvement of axillary or cervical nodes. Chloramphenicol is preferred in case of meningitis. Go to the next page if you knew the correct answer, or click the link image(s) below to further research the concepts in this question (if desired).

Research Concepts: Comprehensive Review Of Bioterrorism

We update eBooks quarterly and Apps daily based on user feedback. Please tap flag to report any questions that need improvement.

Question 594: A 65-year-old female presents to the emergency department with a chief complaint of altered mental status. The patient was found unresponsive. Her heart rate is 110/min; the blood pressure is 90/60 mmHg, respiratory rate 12 breaths per minute, SpO2 98% on room air, finger-stick glucose is 125mg/dL. Initial resuscitative and evaluation measures are appropriately implemented. The patient's home-health attendant states that the patient has been refusing oral intake except for toast and drinks tea exclusively. During the evaluation, the patient suffers a tonic-clonic seizure that is successfully aborted with 4 mg of intravenous lorazepam. The basic metabolic panel shows Na 113 mmol/L. The first-line treatment for this condition is administered, and the patient is eventually admitted to the medical ICU. Several days later, the patient again seizes and is found to have decreased reflexes. Repeat serum sodium is within normal limits. What is the likely iatrogenic cause of the patient's deteriorating condition?

Choices: 1. Bacterial meningitis 2. Hypoglycemia 3. Central pontine myelinolysis 4. Subarachnoid hemorrhage

Answer: 3 - Central pontine myelinolysis Explanations: Overcorrection of serum sodium, particularly when chronically low can lead to the devastating condition of central pontine myelinolysis. The "tea and toast" diet is a common etiology of hyponatremia, other potential causes include beer potomania, cerebral salt wasting, and SIADH. Serum sodium should be corrected at a rate not faster than 6-12 mEq in the first 24 hours and no more than 18 mEq in 48 hours. 3% Sodium Chloride is indicated in this patient given acute neurologic sequelae of severe hyponatremia. Go to the next page if you knew the correct answer, or click the link image(s) below to further research the concepts in this question (if desired).

Research Concepts: Crystalloid Fluids

We update eBooks quarterly and Apps daily based on user feedback. Please tap flag to report any questions that need improvement.

Question 595: A 26-year-old patient is suspected of having overdosed on heroin that was suspected. He required a total of 4 mg of naloxone. An initial 2 mg was given by emergency medical responders and 2 mg in the emergency department. The clinician is called to the bedside by the emergency department nurse as the patient seems to be having increased work of breathing, and his oxygen saturation is 87%. His pulse is 110/min, and his respiratory rate is 25/min. What is the next best step in the management of this patient?

Choices: 1. Administer an additional dose of 4 mg IV naloxone 2. Place the patient on supplemental oxygen 3. Urgent intubation and ventilation 4. Obtain a CT-pulmonary angiogram of the chest to evaluate for pulmonary embolism

Answer: 2 - Place the patient on supplemental oxygen Explanations: Negative pressure pulmonary edema is a known complication of naloxone administration, occurring in up to 3.6% of patients who receive naloxone. The mechanism for developing pulmonary edema in the context of opioid reversal with naloxone is multifactorial. Mechanisms proposed include catecholamine surge, pulmonary capillary leak, and fleeting airway obstruction by a large inspiration against a closed glottis. Despite the possible complication of negative pressure pulmonary edema, this should not deter the use of naloxone in case of overdose. Go to the next page if you knew the correct answer, or click the link image(s) below to further research the concepts in this question (if desired).

Research Concepts: Naloxone

We update eBooks quarterly and Apps daily based on user feedback. Please tap flag to report any questions that need improvement.

Question 596: A 65-year-old undergoes an antrectomy with truncal vagotomy. Four days later, he is found to have mediastinal emphysema, fever, and mild dyspnea. A chest radiograph reveals pleural fluid in the left hemithorax and bilateral fluffy air-space disease. What is the most likely diagnosis?

Choices: 1. Congestive heart failure 2. Pneumonia 3. Pulmonary embolism 4. Esophageal tear

Answer: 4 - Esophageal tear Explanations: When surgery is performed on the esophagus, there is always the chance of an esophageal tear. After a truncal vagotomy, the esophagus is occasionally torn. Tearing of the esophagus can present with rapid acute respiratory distress syndrome and subcutaneous emphysema. The diagnostic test of choice for an esophageal rupture is a contrast esophagram, beginning with water-soluble contrast and following, if no tear is evident, with small amounts of barium. Go to the next page if you knew the correct answer, or click the link image(s) below to further research the concepts in this question (if desired).

Research Concepts: Esophageal Perforation And Tears

We update eBooks quarterly and Apps daily based on user feedback. Please tap flag to report any questions that need improvement.

Question 597: A 17-year-old man on treatment for frequent seizures is brought to the emergency department by his friend after two episodes of myoclonic jerking in the legs, each lasting about 6 minutes. According to his friend, the patient can predict the onset of his seizures and would stare into the distance with a lip-smacking movement for about 1 minute. Sometimes he would start jerking both arms for about 30 seconds, but this time it lasted 6 minutes and happened once again on their way to the emergency department. He did not lose consciousness or urinate on himself. Blood pressure is 100/60 mm Hg, and the pulse rate is 70/min. On neurological exam, a weakness in the lower right limb, with a power of +1, is observed. Laboratory reports are all within normal limits. The blood valproate level is within normal limits. One hour after the initial evaluation, the patient has no recurrence of seizure activity has a power of 4+ in all the limbs. The pupils are bilaterally equal and reactive. Neurology consult recommended a new drug to be added to the patient's regimen to control his condition better. What is the side effect classically associated with this medication?

Choices: 1. Pancreatitis 2. Liver failure 3. Gingival hyperplasia 4. Steven Johnson syndrome

Answer: 4 - Steven Johnson syndrome Explanations: Lamotrigine is the treatment of choice in a case of partial seizures associated with secondary generalization. Lamotrigine acts by blocking voltage-gated sodium channels. Lamotrigine is used in patients who do not tolerate, do not respond to, or have contraindications to valproate. Apart from partial seizures, it is also specifically used for Lennaux Gestaut syndrome. Lamotrigine is also used in treating bipolar disorder. The adverse reaction associated with lamotrigine use is Stevens-Johnson syndrome, especially when used with carbamazepine. Stevens-Johnson syndrome presents with a generalized rash, widespread bullae, and mucosal involvement. These bullae rapidly progress to denude the surface epithelium, potentially leading to severe dehydration and sepsis, among other complications. Treatment involves immediate cessation of the drug and supportive care. Lamotrigine is also associated with toxic epidermal necrolysis and DRESS (Drug rash with eosinophilia and systemic symptoms) syndrome. Lamotrigine can occasionally cause liver problems, but this is uncommon. DRESS (Drug rash with eosinophilia and systemic symptoms) syndrome is a noteworthy side effect of both phenytoin and carbamazepine, while gingival hyperplasia is specific to phenytoin. Pancreatitis is a rare side effect with valproate, not lamotrigine. Go to the next page if you knew the correct answer, or click the link image(s) below to further research the concepts in this question (if desired).

Research Concepts: Partial Epilepsy

We update eBooks quarterly and Apps daily based on user feedback. Please tap flag to report any questions that need improvement.

Question 598: An 18-year-old man is brought to the hospital after a high-speed car accident. He is mildly confused and has a GCS of 14/15, his heart rate is 100/min, and his blood pressure is 110/70 mmHg. Admission labs have been sent and are pending. He has large bruises around each eye and is complaining of significant left flank pain. During the primary survey, he begins to have multiple episodes of emesis. Two large-bore IV lines are placed. Which of the following is the next best step in the management of this patient?

Choices: 1. Place a nasogastric tube 2. Turn the patient on his side to minimize aspiration risk and continue the primary survey 3. Turn the patient on his side and administer supportive care such as ondansetron 4. Proceed emergently to the operating room

Answer: 2 - Turn the patient on his side to minimize aspiration risk and continue the primary survey

Explanations: Turning the patient on his side to avoid aspiration and continuing the survey is correct for this patient. As he is a trauma patient, it is imperative to complete the primary and secondary surveys to ensure no major injury is missed. If the emesis continues and becomes intractable, it should raise suspicion for possible increased intracranial pressure, especially in this patient who was injured in a high-speed accident and has bilateral periorbital ecchymoses. However, symptoms associated with increased intracranial pressure, such as blown pupils, are evaluated in the primary survey, which should be performed in a systematic manner. The bilateral ecchymoses around each eye, also known as raccoon eye sign, should raise suspicion for a basilar skull fracture. Placement of a nasogastric tube under these conditions is contraindicated as it may worsen the fracture, or the tube may even slip inside the skull cavity. Turning the patient on his side to minimize aspiration risk is the correct maneuver, but providing supportive care should wait until the primary and secondary surveys are completed in order to ensure significant injuries are not missed. Given the information provided in the question stem, it is unclear whether this patient needs to go to the operating room. Completing the primary and secondary surveys are imperative to determine the best option for this patient. Go to the next page if you knew the correct answer, or click the link image(s) below to further research the concepts in this question (if desired).

Research Concepts: Nasogastric Tube

We update eBooks quarterly and Apps daily based on user feedback. Please tap flag to report any questions that need improvement.

Question 599: A large group of politicians were at a party and suddenly become ill. Several died having suffered from convulsions. Those that present to the hospital have cyanosis, headache, hyperventilation, hypotension, lightheadedness, nausea, vomiting, and metabolic acidosis. Which of the following should be administered first?

Choices: 1. 100% oxygen 2. Sodium thiosulfate 3. Sodium nitrate 4. Amyl nitrite

Answer: 1 - 100% oxygen Explanations: Initial care for patients with cyanide exposure is airway control, ventilation, and 100% oxygen. Crystalloids and vasopressors are used for hypotension. Sodium bicarbonate is titrated according to arterial blood gas. Administer hydroxocobalamin or sodium thiosulfate and sodium nitrite if the diagnosis is strongly suspected. Do not wait for laboratory confirmation. Go to the next page if you knew the correct answer, or click the link image(s) below to further research the concepts in this question (if desired).

Research Concepts: Cyanide Toxicity

We update eBooks quarterly and Apps daily based on user feedback. Please tap flag to report any questions that need improvement.

Question 600: A 65-year-old farmer is brought to the emergency department of a tertiary care hospital with complaints of chest pain six hours after being bitten by an insect while he was walking in his fields barefoot. Upon physical examination, he is diaphoretic and clutching his chest, and two-bite marks with surrounding erythema are seen at the site of the bite. Cardiovascular examination reveals a regular rhythm and rate along with bilaterally strong and symmetric pulses. Respiratory examination reveals that the lungs are clear bilaterally. An ECG done shows nonspecific ST-T changes in multiple leads. Which of the following is the next best step in the management of this patient?

Choices: 1. Send troponins 2. Give IV fluids 3. Attach oxygen 4. Irrigate the wound

Answer: 1 - Send troponins Explanations: The sign and symptoms in this patient are suggestive of Centipede envenomation. Centipede venom is a pharmacologically diverse and potent substance. Venom can include bioactive proteins, peptides, and other small molecules. These can have myotoxic, cardiotoxic, and neurotoxic effects. Local wound care is the primary management of uncomplicated centipede bites. Monitoring for systemic symptoms is recommended. If the patient complains of specific symptoms such as chest pain, a workup for myocardial ischemia, including troponins and EKG, should be obtained. Treatment of minor centipede bites includes irrigating the site to reduce the risk of infection, applying ice packs also helps as the cold elevates the pain threshold impedes nerve conduction, and vasoconstricts vessels to prevent tissue edema. Some patients report pain improvement with submersion of the extremity in hot water, as it is thought to denature any heat-labile toxins in the venom. However, some patients have also reported increased pain with hot water exposure. Go to the next page if you knew the correct answer, or click the link image(s) below to further research the concepts in this question (if desired).

Research Concepts: Centipede Envenomation

We update eBooks quarterly and Apps daily based on user feedback. Please tap flag to report any questions that need improvement.

Section 7 Question 601: A 22-year-old woman is brought to the hospital with altered mental status, hallucinations and severe rigidity. She had an intrathecal pump implanted 12 months ago for the management of her underlying movement disorder. Her mother states that she has not been able to follow up with her appointments recently. Which of the following is the most likely cause of her presenting symptoms?

Choices: 1. Drinking alcohol during the administration of the intrathecal medication 2. Abrupt withdrawal of the intrathecal medication 3. Overdose of the intrathecal medication 4. Taking erythromycin while being on the intrathecal administration of medication

Answer: 2 - Abrupt withdrawal of the intrathecal medication Explanations: Abrupt withdrawal of intrathecal baclofen may occur in patients using the drug over 2 months and may result in a hypermetabolic state with hyperpyrexia, impaired mental status, muscle rigidity and severe rebound spasticity which may advance to rhabdomyolysis and multi-system failure. The FDA requires a black box warning for baclofen. Abrupt withdrawal of intrathecal baclofen may occur in patients using the drug over 2 months and may result in a hypermetabolic state with hyperpyrexia, impaired mental status, muscle rigidity, and severe rebound spasticity which may advance to rhabdomyolysis and multi-organ system failure. The symptoms improve upon recommencing of baclofen by a similar dose as before the interruption. Withdrawal most commonly occurs because of a delivery system problem. The patient and caregiver must understand the importance of monitoring the pump to prevent withdrawal events. Abrupt discontinuation of oral baclofen therapy might cause seizures and hallucinations. Gradual dose reduction is recommended to prevent withdrawal symptoms. Drinking alcohol in a patient on the administration of intrathecal baclofen will not cause a hypermetabolic state. Overdose of the intrathecal baclofen will not cause a hypermetabolic state. Taking erythromycin while being on the intrathecal administration of baclofen will not cause a hypermetabolic state. Go to the next page if you knew the correct answer, or click the link image(s) below to further research the concepts in this question (if desired).

Research Concepts: Baclofen

We update eBooks quarterly and Apps daily based on user feedback. Please tap flag to report any questions that need improvement.

Question 602: A 65-year-old male presents to the emergency department in respiratory distress. He has obvious tongue swelling, increased work of breathing, and drooling. Vital signs show tachycardia, respiratory rate of 30/minute, and oxygen saturation of 70%. He has no known medical problems and takes no medications. What is the next best step in the management of this patient?

Choices: 1. Intubation 2. Epinephrine 3. Corticosteroids 4. Continuous positive airway pressure

Answer: 1 - Intubation Explanations: This patient is suffering from hereditary angioedema resulting in airway compromise. In the absence of ACE inhibitors or allergen contacts, this patient has a much higher probability of suffering from a C1 esterase inhibitor defect. Patients in respiratory distress with low oxygen saturation and oral swelling should receive a definitive airway sooner rather than later due to obtaining an airway becoming more difficult as swelling worsens. In cases of airway swelling, epinephrine should be attempted as it is more common to have anaphylaxis than hereditary angioedema. Once attempts at epinephrine and other anaphylactic medications have been unsuccessful, and the diagnosis of hereditary angioedema (HAE) is suspected, specific therapies should be attempted. Specific therapy for C1 esterase inhibitor deficiency with laryngeal edema involves replacing the deficient enzyme. The easiest way is to give human plasma, which contains C1 esterase inhibitor (C1INH ). The initial dose of human plasma is two units. C1INH concentrate, which is derived from human plasma, may be used. The initial dose is 1500 units. Go to the next page if you knew the correct answer, or click the link image(s) below to further research the concepts in this question (if desired).

Research Concepts: C1 Esterase Inhibitor Deficiency

We update eBooks quarterly and Apps daily based on user feedback. Please tap flag to report any questions that need improvement.

Question 603: A 58-year-old male with a past medical history of prostate cancer, status post prostate resection 5 years ago, as well as long-standing osteoarthritis and cigarette smoking which is 3 weeks status post left knee replacement presents to the emergency department with a complaint of 3 days of progressive pleuritic chest discomfort, shortness of breath, and a mild cough with blood-tinged sputum. His left lower extremity has been persistently swollen since the surgery. His vital signs include a heart rate of 104 beats per minute, blood pressure of 138/84 mm Hg, pulse oximetry reading of 97% on room air, and an oral temperature of 37.4 C (99.32 F). Which of the following is most accurate regarding his diagnosis?

Choices: 1. The left upper lobe of his lung is the most likely region of the lung to be affected by this condition. 2. Air bronchograms on a CT of the chest are highly suggestive of this diagnosis. 3. An area of radiographic oligemia (increased lucency) on a chest x-ray makes this diagnosis less likely. 4. A peripheral wedge-shaped hyperdensity seen on a chest x-ray in the appropriate clinical setting is a specific finding for this diagnosis.

Answer: 4 - A peripheral wedge-shaped hyperdensity seen on a chest x-ray in the appropriate clinical setting is a specific finding for this diagnosis.

Explanations: Pulmonary embolism is the most common cause of pulmonary infarction and, though there are numerous risk factors and signs of pulmonary embolism, clinical signs and symptoms of deep vein thrombosis, heart rate over 100 beats per minute, immobilization or surgery within the previous 4 weeks, history of previous venous thromboembolism, hemoptysis, malignancy, exogenous hormone use, and age over 50 are frequently cited. Hampton’s hump (wedge-shaped consolidation at the lung periphery), Westermark’s sign (radiographic oligemia or increased lucency), and Fleischer sign (prominent pulmonary artery) are specific chest x-ray findings for pulmonary infarction. Though chest x-rays can show specific findings that are highly suggestive of pulmonary infarction, these findings have poor overall sensitivity, making CT angiogram of the chest the preferred method for diagnosis of pulmonary embolism with or without infarction. CT angiogram findings associated with pulmonary infarction include a feeding vessel or "vessel sign," central lucency, and semicircular shape. Additionally, the finding of air bronchograms on CT makes the presence of a pulmonary infarction less likely. Unilateral infarct occurs most common with pulmonary infarction, with the lower lobes having a strong predilection over the upper lobes, and the right lower lobe is the most frequently affected. Go to the next page if you knew the correct answer, or click the link image(s) below to further research the concepts in this question (if desired).

Research Concepts: Pulmonary Infarction

We update eBooks quarterly and Apps daily based on user feedback. Please tap flag to report any questions that need improvement.

Question 604: A 51-year-old man presents to the hospital with myalgia and dark-colored urine after a crush injury. Vital signs show a heart rate of 100/min and blood pressure 130/100 mmHg. The physical examination is unremarkable except for a laceration over the right leg. Initial labs reveal a creatine phosphokinase level of 6000 IU/L (normal 30-223 IU/L). His troponin level is 0.1 ng/mL ( normal 0.00 to 0.04 ng/mL). EKG shows T wave inversion in lead 1. What is the best initial step in the management of this patient?

Choices: 1. Intravenous normal saline 2. Aspirin and clopidogrel 3. Echocardiogram 4. Hemodialysis

Answer: 1 - Intravenous normal saline Explanations: The diagnosis of rhabdomyolysis is almost certain when creatine phosphokinase levels exceed 5000 IU/L. Rhabdomyolysis is suspected in levels above 1000 IU/L. It is the diagnosis in this patient because there was a crush injury and also dark urine along with muscle pain. Treatment of rhabdomyolysis is fluid resuscitation with isotonic saline. Myocardial infarction would not cause discoloration in urine, and also, an ECG would show more specific findings. Sometimes rhabdomyolysis causes a mild elevation in troponins, but that does not indicate myocardial infarction in the absence of cardiac symptoms. Go to the next page if you knew the correct answer, or click the link image(s) below to further research the concepts in this question (if desired).

Research Concepts: Creatine Phosphokinase

We update eBooks quarterly and Apps daily based on user feedback. Please tap flag to report any questions that need improvement.

Question 605: A 29-year-old woman is brought to the emergency department after having an episode of a seizure lasting for more than 5 minutes. She is intubated for airway protection, and intravenous (IV) phenytoin is given. Her family reports that she has been complaining of severe headaches for the last few days before the onset of a seizure. On examination, she remains sedated, unresponsive, and withdraws to noxious stimuli more prominently on the left side. Contrast CT scan detects a ring-enhancing lesion vasogenic edema in the left paracentral gyrus. Further workup indicates halo-shaped pulmonary infiltrates. Blood investigations show neutropenia. She also has glioblastoma (GBM) multiforme, for which she takes high-dose prednisone. Which of the following is the most likely etiology of the focal lesion seen on CT scan in this patient?

Choices: 1. Aspergillosis 2. Cryptococcus 3. Candidiasis 4. Coccidioides

Answer: 1 - Aspergillosis Explanations: Aspergillus is a fungus found throughout the world that can cause infection in primarily immunocompromised hosts and individuals with the underlying pulmonary disease. Invasive aspergillosis is common only in the immunocompromised population, composed of patients with AIDS, neutropenic patients, those on long-term corticosteroids, and recipients of transplants on anti-rejection medications. The most common initial symptoms include dyspnea, increased sputum production, chills, headache, and arthralgias. As the condition quickly progresses, fever, toxicity, and weight loss manifest. On examination, there may be sinus tenderness, nasal discharge, rales, dermatologic changes, or meningeal signs of the central nervous system. Chest radiographs may show parenchymal opacities of pulmonary aspergilloma (fungus ball). CT imaging of the lungs will show characteristic nodules with surrounding attenuation (“halo sign”), aspergilloma (fungal ball in a pre-existing lung cavity), cavitations, or fibrosis. CT may also be useful to evaluate for sinus involvement such as masses, opacification, or destruction of sinus walls if this manifestation of aspergillosis is suspected. Go to the next page if you knew the correct answer, or click the link image(s) below to further research the concepts in this question (if desired).

Research Concepts: Aspergillosis

We update eBooks quarterly and Apps daily based on user feedback. Please tap flag to report any questions that need improvement.

Question 606: A 53-year-old male industrial worker is brought to the hospital in an acute confusional state. The patient's attendant states that this company is a pharmaceutical company where this patient was handling a bag that contains carbon tetrachloride. He was not wearing a protective face mask. On examination, the patient appears confused. Vital signs show temperature 39°C, blood pressure 130/86 mmHg, pulse 100/min, and respiratory rate 15/min. On laboratory investigation, serum ALT and AST are elevated. Serum creatinine is 1.8 mg/dL. Intravenous cimetidine is given to the patient. Which of the following best explains the underlying mechanism of action of this medication beneficial for this patient?

Choices: 1. Stimulation of CYP450 enzyme 2. Inhibition of CYP450 enzyme 3. Inhibition of glutathione reductase 4. Stimulation of glutathione peroxidase

Answer: 2 - Inhibition of CYP450 enzyme Explanations: Carbon tetrachloride (CCL4) toxicity causes multiple organ dysfunction by lipid peroxidation and some other mechanisms. This lipid peroxidation occurs by free radical formation from CCL4 by the help of CYP450 enzymes. This enzyme can be inhibited by intravenous cimetidine. Cimetidine acts as a CYP450 enzyme inhibitor. When this enzyme is inhibited by cimetidine, free radical formation is decreased from CCL4. So, complications will be less. Intravenous glucose also acts like as intravenous cimetidine to downregulate CYP450 to reduce CCL4 degradation. If an appropriate measure is not taken in a patient with CCL4 intoxication, life-threatening complications may develop. CO2 induced hyperventilation to exhale toxin, enzyme inhibition by cimetidine, or glucose to decrease the free radical formation are commonly available options. Electrolyte imbalance and metabolic acidosis should also be treated if present. Go to the next page if you knew the correct answer, or click the link image(s) below to further research the concepts in this question (if desired).

Research Concepts: Carbon Tetrachloride Toxicity

We update eBooks quarterly and Apps daily based on user feedback. Please tap flag to report any questions that need improvement.

Question 607: A car pulls up to the emergency department entrance, and several teenagers rush in, asking for help. The staff pulls a somnolent 16-year-old girl from the back seat and brings her into the resuscitation room. The patient sent a text message to her friends, stating that she was depressed about a bad breakup and had taken an overdose to "end the pain." Her companions don’t know much about her health history, but one mentions that she has “Tetralogy or something like that” and had surgery as a baby. She believes that she only takes one medication, ”a heart pill.” Her EKG reveals bradycardia and a first-degree heart block. Also, her blood glucose is 368 mg/dL. Her cardiac problem quickly escalates to a 2nd and then a 3rddegree heart block. Having astutely decided what the likely offending agent is, what are the probable treatment options?

Choices: 1. Glucagon, vasopressors, cardiac pacing 2. Sodium bicarbonate, intralipid, urine alkalinization 3. High dose insulin, intralipid, IV calcium, atropine, vasopressors, methylene blue, ECMO 4. Sodium bicarbonate, benzodiazepines, ECMO

Answer: 3 - High dose insulin, intralipid, IV calcium, atropine, vasopressors, methylene blue, ECMO

Explanations: This patient has Tetralogy of Fallot and was placed on calcium channel blockers (CCBs) due to potential rhythm disturbances after the surgery. She overdosed on this medication and already exhibited toxicity when she was found. CCBs interfere with insulin secretion which leads to hyperglycemia. Hemodynamic effects are bradycardia, hypotension, and eventual cardiovascular collapse. This often fatal ingestion requires prompt, aggressive, and multimodal treatment, much of which can have minimal effect in massive intakes. Vasopressors and ECMO are often lifesaving in these extreme cases. Glucagon, vasopressors, and cardiac pacing would be most appropriate for a beta-blocker overdose. Beta-blockers also interfere with gluconeogenesis, usually leading to hypoglycemia. This patient has hyperglycemia. Sodium bicarbonate, intralipid, urine alkalinization would be most appropriate for an overdose of tricyclic antidepressants (TCAs). TCAs typically cause tachycardia followed by QRS prolongation and right axis deviation. Sodium bicarbonate, benzodiazepines, and ECMO would be most appropriate for an overdose of antimalarial drugs. Go to the next page if you knew the correct answer, or click the link image(s) below to further research the concepts in this question (if desired).

Research Concepts: Deadly Single Dose Agents

We update eBooks quarterly and Apps daily based on user feedback. Please tap flag to report any questions that need improvement.

Question 608: A 32-year-old woman is brought to the hospital because of her inability to pass urine for the last 24 hours. Immediate catheterization is done. After taking proper history and past medical records, she has been found as a patient of chronic kidney disease. She also has fatigue, general weakness. Serum electrolytes show a potassium level of 6.4 mEq/L. An oral medication with no systemic absorption is prescribed. While treating with this medication, the patient develops respiratory distress, which becomes more severe while lying on bed and improves while sitting or standing. She also develops swelling in the lower limb. Cardiac auscultation reveals S3 gallop. The medication given for hyperkalemia is stopped. Which of the following is most likely responsible for the patient's new symptoms?

Choices: 1. Hyperkalemia 2. Hypermagnesemia 3. Sodium load 4. Hypercalcemia

Answer: 3 - Sodium load Explanations: Sodium polystyrene sulfonate is an oral drug that can be used for hyperkalemia. Besides lowering the potassium level, it may cause sodium loading. Patients with preexisting fluid overload (respiratory distress, orthopnea, edema, S3) may not be able to tolerate this sodium load. In this condition, SPS should be used with great caution, and electrolytes should be monitored regularly. Go to the next page if you knew the correct answer, or click the link image(s) below to further research the concepts in this question (if desired).

Research Concepts: Sodium Polystyrene Sulfonate

We update eBooks quarterly and Apps daily based on user feedback. Please tap flag to report any questions that need improvement.

Question 609: A 74-year-old male patient is recovering in the surgical ICU after a motor vehicle accident. The patient has been treated for multiple extremity fractures and consistently notes pain level at 8/10. He is currently undergoing a trial of pressure support ventilation. The current ventilator setting is as follows: pressure support mode 12 cmH2O, CPAP 5 cmH2O, and 45 % FiO2. The patient presents with the following monitored parameters 10 min into the PSV mode: VTe 550 ml, spontaneous RR 28 bpm, and BP 160/95 mmHg. The patient presents to be very anxious. Which of the following is the next best step in the management of this patient?

Choices: 1. Sedate the patient with propofol. 2. Return the patient to a mechanically controlled rate. 3. Give fentanyl for analgesia and monitor the respiratory status. 4. Verbally reassure the patient.

Answer: 3 - Give fentanyl for analgesia and monitor the respiratory status. Explanations: When a patient experiences pain, other monitored data such as heart rate and blood pressure deviate from acceptable ranges. Maintaining a patient comfort level while keeping the respiratory drive in mind is an important part of a spontaneous breathing trial. Pain and anxiety should be ruled out before discontinuing SBT. Sedating this patient with propofol will halt the weaning process. The goal is to allow the patient to stay alert and maintain spontaneous respirations. Propofol should only be adminsited when medically appropriate. Go to the next page if you knew the correct answer, or click the link image(s) below to further research the concepts in this question (if desired).

Research Concepts: Pressure Support

We update eBooks quarterly and Apps daily based on user feedback. Please tap flag to report any questions that need improvement.

Question 610: A 68-year-old man is admitted to the hospital with complaints of fever and chills for 1 week. His past medical history is significant for type 2 diabetes mellitus, hypertension, chronic urinary retention with chronic indwelling Foley catheter, and obesity. Upon presentation, the patient's heart rate is 106/min, blood pressure 72/56 mmHg, temperature 102 F, and SpO2 92% on room air. His systemic examination reveals a drowsy person, arousable to touch and to follow commands. Other systems are unremarkable. Initial lab tests revealed a WBC count of 18000/microL and elevated CRP. Urinary analysis reveals pyuria, hematuria, many bacteria, with positive leukocyte esterase and positive nitrites. The patient is given broadspectrum antibiotics, norepinephrine, and admitted to the intensive care unit. Which of the following findings is most likely to be appreciated on right heart catheterization in this patient?

Choices: 1. Elevated pulmonary capillary wedge pressure 2. Normal wedge pressure 3. Elevated right atrial pressure 4. Elevated pulmonary artery pressure

Answer: 2 - Normal wedge pressure Explanations: This patient most likely has septic shock secondary to a urinary tract infection (UTI). The chronic urinary retention and indwelling Foley catheter are common risk factors for a UTI. The urinary analysis also reveals signs of infection. In patients with septic shock, the wedge pressure will be normal as the left ventricular function is uncompromised. In patients with septic shock, the right atrial pressure or CVP can be used to assess the need for fluid resuscitation and responsiveness to treatment. Go to the next page if you knew the correct answer, or click the link image(s) below to further research the concepts in this question (if desired).

Research Concepts: Pulmonary Capillary Wedge Pressure

We update eBooks quarterly and Apps daily based on user feedback. Please tap flag to report any questions that need improvement.

Question 611: An 87-year-old woman is placed on ventilator support after a cardiac arrest and subsequent successful resuscitation. The patient's family has gathered to discuss palliative extubation. The primary provider reviews the patient's condition and options. The patient does not have an advanced directive or a medical power of attorney. During the discussion, several family members begin arguing about the decision. Which of the following is the most appropriate tool to aid with de-escalation?

Choices: 1. Emphasizing the importance of focusing on what the patient would want 2. Restating the provider's medical opinion to guide decision making 3. Stating that the closest living relative to the patient will make the decision 4. Leaving the situation until a decision is made

Answer: 1 - Emphasizing the importance of focusing on what the patient would want Explanations: Providers have a responsibility to help mediate and advocate for their patients, and shared decision-making with mediation is the ideal goal of discussing difficult medical decisions with family members. Often emphasizing that this decision should be made with the patient's wishes in mind can help the patient's family members work together towards a common goal. While providing medical information is important, restating or trying to enforce a provider's preference is not appropriate in this situation. A patient's closest living relative may potentially be used if there are difficulties in agreeing with a decision. However, this option is usually used only after attempting to settle differences in opinion to arrive at a shared decision. While it may be appropriate to give family members space to help their decision, it is not appropriate to be absent from the decision-making process in this situation. Go to the next page if you knew the correct answer, or click the link image(s) below to further research the concepts in this question (if desired).

Research Concepts: Refusal of Care

We update eBooks quarterly and Apps daily based on user feedback. Please tap flag to report any questions that need improvement.

Question 612: A 34-year-old man with a past medical history of hypertension and diabetes is admitted to ICU for acute respiratory distress. He states that he had a cough, nausea, and vomiting last week. Vital signs show blood pressure 164/86 mmHg, pulse 88/min, respiratory rate 25/min, and SpO2 90% on room air. Labs show WBC count within normal limits, decreased lymphocytes, elevated ESR, elevated CRP, elevated LDH, and elevated ferritin. The patient is treated for possible coronavirus pneumonia. Overnight, the patient becomes confused and hypoxic with SpO2 55% and is intubated. Vent settings are tidal volume 370 mL, PEEP 15, respiratory rate 28/min, and FiO2 80%. Three days later, his creatinine increases from initial 1.0 mg/dL on admission to 6.9 mg/dL with potassium 6.0 mEq/L. A hemodialysis catheter is attempted in the right internal jugular vein. During the procedure, the patient starts to desaturate to 82% on 100% FiO2 on the vent. The procedure is aborted. Current vital signs show blood pressure 87/69 mmHg and pulse 147/min. The trachea appears to deviate, the jugular veins are distended, and he has hyperresonant sounds on the right side of the chest. What is the next best step in the management of this patient?

Choices: 1. Place the patient in the prone position 2. Needle decompression 3. Decrease PEEP 4. Start vasopressors

Answer: 2 - Needle decompression Explanations: The patient's hypoxia is due to tension pneumothorax, most likely secondary to central venous catheter placement in the internal jugular. Pneumothorax is more commonly caused by central venous catheter placements in the internal jugular or subclavian as compared to barotrauma. Needle compression is the best management. Tension pneumothorax, which is a lifethreatening emergency. Needle compression must be done immediately. Otherwise, the patient can end up in cardiac arrest or death. Putting the patient in the prone position would be a treatment for acute respiratory distress syndrome. Low PEEP is required for tension pneumothorax. However, tension pneumothorax is a life-threatening emergency that must be treated immediately. High PEEP will worsen the pneumothorax. The drop in blood pressure is due to the tension pneumothorax. When the pressure in the pleural cavity increases, it causes a mechanical obstruction by compressing on the superior vena cava and inferior vena cava, which decreases venous return and ultimately decreases cardiac output. This causes a drop in blood pressure. Vasopressors may keep blood pressure up, but the underlying problem should be resolved. Go to the next page if you knew the correct answer, or click the link image(s) below to further research the concepts in this question (if desired).

Research Concepts: Tension Pneumothorax

We update eBooks quarterly and Apps daily based on user feedback. Please tap flag to report any questions that need improvement.

Question 613: A 39-year-old woman was brought in by paramedics with a gunshot wound in the abdomen. The patient was intubated and had an exploratory laparotomy that showed a small bowel injury with intraperitoneal hemorrhage, which was repaired and contained. Postoperatively, the patient had a central line placement for total parenteral nutrition administration. Which of the following laboratory abnormality is associated with the patient's nutrition source?

Choices: 1. Hyperglycemia 2. Hypocalcemia 3. Hypokalemia 4. Hyponatremia

Answer: 4 - Hyponatremia Explanations: Total parenteral nutrition (TPN) is mainly associated with hyponatremia. Other complications are hyperglycemia and low calcium or potassium. TPN requires a dedicated central line. One of the most common complications of the central line is an infection of the catheter, which can cause sepsis. Long-term TPN also has been linked to nonalcoholic fatty liver. Go to the next page if you knew the correct answer, or click the link image(s) below to further research the concepts in this question (if desired).

Research Concepts: Total Parenteral Nutrition

We update eBooks quarterly and Apps daily based on user feedback. Please tap flag to report any questions that need improvement.

Question 614: A 25-year-old man presents to the emergency department with complaints of anorexia, fatigue, and progressive dyspnea with decreased exercise capacity. Three days back, he developed a fever and cough with mucopurulent expectoration. He denies a prior history of cardiac or pulmonary disease. On admission, he is in acute pulmonary edema with respiratory failure and acidosis. Chest x-ray reveals cardiomegaly with heterogenous diffuse infiltrates sparing lung bases and upper lobes. He is intubated, artificially ventilated, and prescribed IV antibiotics and diuretics. Serum BNP level is elevated. Blood and sputum cultures are negative. He recovers with treatment and is extubated on the 5th day. Echocardiography reveals thickening and trabeculations of the posterior and lateral wall of the left ventricle. The left ventricle is dilated with global hypokinesia and moderately reduced ejection fraction. What is the most likely cause of the patient's symptoms?

Choices: 1. Lower respiratory tract infection 2. Heart failure 3. Coronary artery disease 4. Interstitial lung disease

Answer: 2 - Heart failure Explanations: LV noncompaction is considered congenital cardiomyopathy caused due to intrauterine arrest of myocardial development. Although the left ventricle is commonly affected, the involvement of both ventricles involvement occurs in 22% - 38% of patients. The most common presentation is left ventricular dysfunction, which can progress to heart failure. Treatment is largely nonspecific, mainly dependent on clinical manifestations. Go to the next page if you knew the correct answer, or click the link image(s) below to further research the concepts in this question (if desired).

Research Concepts: Left Ventricular Non-compaction Cardiomyopathy

We update eBooks quarterly and Apps daily based on user feedback. Please tap flag to report any questions that need improvement.

Question 615: A 76-year-old woman with a history of chronic obstructive pulmonary disease (COPD) on home oxygen at 1 L/min presents to the emergency room with dyspnea, chills, and dry cough. On physical examination, she is afebrile and tachypneic, and her oxygen saturation is 95% on 3 L of oxygen. Her pulmonary examination is within normal limits. The chest radiograph reveals bilateral lower lobe consolidation. The patient is admitted and diagnosed with community-acquired pneumonia and a COPD exacerbation. On the morning following admission, she is found to be minimally arousable. The on-call provider notes that her oxygen saturation is 98% and that she is receiving oxygen by nasal cannula at a rate of 4 L/min. Which of the following should be done next?

Choices: 1. Increase nasal cannula oxygen flow from 4 L to 8 L 2. Place the patient on a non-rebreather oxygen mask 3. Transfer patient to ICU and attempt non-invasive positive pressure ventilation and draw arterial blood gas 4. Intubate immediately

Answer: 3 - Transfer patient to ICU and attempt non-invasive positive pressure ventilation and draw arterial blood gas

Explanations: Target O2 saturation in patients with COPD should be between 88%-92%, as higher saturations may cause oxygen-induced hypercapnia. The pathophysiology of COPD is associated with airway outflow obstruction. Ideal ventilator settings in a patient with COPD are assist-control or synchronized intermittent mandatory ventilation (SIMV). Low inspiratory-expiratory (I:E) ratio, thus allowing more time for expiration to avoid auto-peep. High flow oxygen is relatively contraindicated in COPD exacerbation. The increased oxygen levels lead to hypercapnia by inhibiting hypoxic vasoconstriction and increasing alveolar dead space. Go to the next page if you knew the correct answer, or click the link image(s) below to further research the concepts in this question (if desired).

Research Concepts: Oxygen Toxicity

We update eBooks quarterly and Apps daily based on user feedback. Please tap flag to report any questions that need improvement.

Question 616: A 23-year-old man is found unconscious by the riverside. CPR is done by the rescue service, and the patient is shifted to the emergency department. At that time of presentation in the emergency department, his blood pressure is 60/10 mmHg, and his pulse rate is 30/min. His EKG shows progressive type 2 atrioventricular block. Urgent external cardiac pacing is planned to save his life. Which of the following is the most appropriate location for the placement of negative leads in this patient?

Choices: 1. Left parasternal region 2. Right parasternal region 3. Left subscapular area 4. Right subscapular area

Answer: 1 - Left parasternal region Explanations: Multiple variations of pacer pad placement have been shown to be equally effective. Pacing pads are positioned on the patient's chest, either in the anterolateral position of anteroposterior one. The negative electrode is usually placed in the left parasternal region. The negative electrode can also be positioned at the cardiac apex. The positive electrode can be placed in the left subscapular region or right parasternal region. Go to the next page if you knew the correct answer, or click the link image(s) below to further research the concepts in this question (if desired).

Research Concepts: External Pacemaker

We update eBooks quarterly and Apps daily based on user feedback. Please tap flag to report any questions that need improvement.

Question 617: A 16-year-old girl is brought to the hospital with complaints of facial rigidity, muscle spasms, a painful abdomen, and headaches for one week. Recently, she has started feeling palpitations, loss of bladder control, and fecal incontinence. On examination, her blood pressure and heart rate fluctuate three times in thirty minutes. She has a bandage on her right heel, which her mother says is from a piercing wound she got from a rusty nail. Her ECG reveals supraventricular tachycardia. She is given benzodiazepines for rigidity. Which of the following additional therapies is most likely to improve the patient's symptoms?

Choices: 1. IV propofol 2. IV dantrolene 3. IV magnesium 4. IV calcium gluconate

Answer: 3 - IV magnesium Explanations: The clinical features of tetanus include lockjaw, grimace facial expression (risus sardonicus), generalized muscle spasms associated with severe pains, drooling, uncontrolled urination and defecation, and back arching spasm (opisthotonus) that may cause respiratory distress. With the loss of autonomic control, patients can present with labile blood pressure and heart rate, diaphoresis, bradyarrhythmias, tachyarrhythmias, and cardiac arrest. Benzodiazepines are considered a cornerstone therapy for tetanus manifestations, and diazepam is the most frequently studied and utilized drug in this regard. It not only reduces anxiety but also causes sedation and relaxes muscles, thereby preventing lethal respiratory complications. Intravenous magnesium has been shown to prevent muscle spasms. Clinicians must also provide supportive care, especially for patients with autonomic instability (labile blood pressure, hyperpyrexia, hypothermia). Magnesium is often used in combination with benzodiazepines to manage these complications. Go to the next page if you knew the correct answer, or click the link image(s) below to further research the concepts in this question (if desired).

Research Concepts: Tetanus

We update eBooks quarterly and Apps daily based on user feedback. Please tap flag to report any questions that need improvement.

Question 618: A patient initially notices double vision after eating supper. Shortly thereafter, she starts to experience weakness in arms and legs, difficulty with swallowing, extreme trouble focusing her vision, and some minor breathing difficulties. 911 is called and she is brought to the ER. The clinician notices bilateral drooping of the eyelids and dilated fixed pupils. Her respiratory status has become more fragile. Which of the following tests is most likely to be helpful in making the diagnosis?

Choices: 1. Erythrocyte sedimentation rates 2. Bioassays 3. Cerebrospinal fluid analysis 4. WBC counts

Answer: 2 - Bioassays Explanations: Botulism may be confirmed with a mouse neutralization bioassay. This assay identifies the toxin. Routine laboratory testing is generally not helpful in the diagnosis of botulism. CSF analysis may show slight elevations of certain proteins. Go to the next page if you knew the correct answer, or click the link image(s) below to further research the concepts in this question (if desired).

Research Concepts: Botulism

We update eBooks quarterly and Apps daily based on user feedback. Please tap flag to report any questions that need improvement.

Question 619: A 27-year old male patient was involved in a motor vehicle accident. He is alert at the scene but unable to move his legs. He is taken to the emergency department with a Glasgow coma scale (GCS) of 14. A quick survey reveals a pelvic fracture and urethral injury. A total body CT scan does not show any other injury. He has an external fixator placed, and the urologist has been called regarding a suprapubic catheter. Seven hours later, the patient’s mental status changes, and his GCS drops to 8. He is intubated. What should be ordered to search for his sudden changes in mental status?

Choices: 1. Pulmonary angiography 2. Echocardiogram 3. Diagnostic peritoneal lavage 4. CT chest

Answer: 2 - Echocardiogram Explanations: In patients who suffer fat embolism syndrome, the fat particles travel in the veins, chiefly to the lungs. In about 20% of patients, there is a patent foramen ovale, which can allow the fat emboli to cross the pulmonary circulation into the systemic circulation. In a patient with fat embolism syndrome who develops acute central nervous system changes, one should get an echocardiogram. CT scan will show marked edema and white matter changes. The pulmonary angiogram is indicated in pulmonary embolism. Diagnostic peritoneal lavage is indicated in patients with blunt trauma abdomen with suspected hemoperitoneum which is less likely in the patient owing to his normal initial CT scans. Go to the next page if you knew the correct answer, or click the link image(s) below to further research the concepts in this question (if desired).

Research Concepts: Fat Embolism

We update eBooks quarterly and Apps daily based on user feedback. Please tap flag to report any questions that need improvement.

Question 620: A 16-year-old Hispanic male with a past medical history of status epilepticus and asthma is rushed to Emergency Room with suspected drug overdose after girlfriend came home and found a suicide note and an empty bottle of his prescription pills next to the unconscious body. When asked about any change in behavior leading up to the suicide attempt, the girlfriend said in the past couple of weeks she noticed his speech was slurred, he would stumble frequently, lose balance on level terrain, and complained about feeling sleepy. Vitals: Blood pressure 109/65, Heart Rate 45, Temperature 98.6 F, Respiration 6 breaths/minute. Physical exam: No needle track marks or visible wounds on the body. Cardio S1 S2 no murmurs, lung exam exhibited shallow breaths bilateral, and the abdominal exam was nonsignificant. The pupils are of normal size and reactive to light. Intubation was initiated and the patient started on intravenous fluids. Based on the girlfriend's interview and the patient's physical exam, which suspected drug of abuse could be found unchanged in the patient's urine?

Choices: 1. Phenobarbital 2. Amobarbital 3. Thiopental 4. Butalbital

Answer: 1 - Phenobarbital Explanations: Phenobarbital also has a very long half-life and can be found up to 15 days later in the urine. Most barbiturates are fast acting, and their presence in the urine indicates that the patient likely ingested the drug sometime within the past 3 days. Amobarbital is an intermediate-acting barbiturate and is not typically used for status epilepticus. Thiopental is a short-acting barbiturate and is not typically used for status epilepticus. Butalbital is commonly used in combination with other medications for the treatment of headaches. Go to the next page if you knew the correct answer, or click the link image(s) below to further research the concepts in this question (if desired).

Research Concepts: Phenobarbital

We update eBooks quarterly and Apps daily based on user feedback. Please tap flag to report any questions that need improvement.

Question 621: A 56-year-old man taking anticoagulants presents to the emergency department with black tarry stools, a pulse of 130/minute and blood pressure of 60/30 mmHg. Nurses are unable to establish an intravenous peripheral catheter. An interosseous needle is placed in the right medial proximal tibia, and O positive blood is being transfused. After 30 minutes, there is no change in the vital signs of the patient, and the flow rate through the IO is very slow. What is the next best step to manage the low flow rates?

Choices: 1. Apply a pressure bag to the blood 2. Choose a new IO site 3. Remove the blood tubing and advance the IO needle 4. Abandon the site and start an ultrasound-guided central venous catheter

Answer: 1 - Apply a pressure bag to the blood Explanations: A pressure bag is needed to ensure adequate flow rates on all IO infusions. Always rapidly flush 10 ccs of saline in the adult patient and 2 ccs to 5 cc saline in neonates and children before injection and infusion. Aspiration of marrow confirms the correct placement of the IO needle. The sternal IO site has been documented to have the highest flow rates. Two IO needles can be placed, but not in the same extremity. Go to the next page if you knew the correct answer, or click the link image(s) below to further research the concepts in this question (if desired).

Research Concepts: Intraosseous Vascular Access

We update eBooks quarterly and Apps daily based on user feedback. Please tap flag to report any questions that need improvement.

Question 622: A 35-year-old male patient was admitted to the emergency department (ED) due to the recent onset of respiratory distress. The vital signs on admission were blood pressure: 98/60 mmHg, pulse rate: 100 beats per minute, respiratory rate: 25 breath per minute, and temperature: 38.8 C. The chest x-ray revealed a large air-fluid level lesion in the right upper lobe. Following initial medical therapy, the preferred surgical management was selected. What is the preferred airway management during surgery?

Choices: 1. Direct laryngoscopy following double-lumen tube (DLT) insertion 2. Utilization 39 french double-lumen tube 3. Advance the DLT with the tip of the bronchial concave curve facing posteriorly 4. Positioning the DLT with rigid bronchoscopy

Answer: 2 - Utilization 39 french double-lumen tube Explanations: The Robertshaw double-lumen tubes have one lumen in the trachea and one lumen in the mainstem bronchus. They have D-shaped lumens. R-sided tubes have an endobronchial cuff that is donut-shaped, which allows separate access to the right upper lobe. Fiberoptic bronchoscopy is the gold standard for confirmation of the correct placement of a DLT. Auscultation alone is unreliable as an indicator of proper DLT placement. To initiate one-lung ventilation, the bronchial cuff is inflated, the lung to be isolated is clamped off at the corresponding connector, the connector is then opened to the atmosphere to allow lung collapse. Lung collapse is most rapid if initiated at end-expiration. For blind insertion of a DLT, first, perform direct laryngoscopy and visualize the vocal cords. Visualization of the vocal cords is important as these tubes are large and placement more challenging than placing an SLT. Once the vocal cords are visualized, gently advance the DLT with the tip of the bronchial concave curve facing anteriorly through the vocal cords until the bronchial cuff passes through the cords. The DLT can also be positioned using fiberoptic bronchoscopy. Using a fiberoptic bronchoscope through the bronchial lumen and guiding the DLT over fiber-optic scope increases the accuracy of placement. Go to the next page if you knew the correct answer, or click the link image(s) below to further research the concepts in this question (if desired).

Research Concepts: Double Lumen Endobronchial Tubes

We update eBooks quarterly and Apps daily based on user feedback. Please tap flag to report any questions that need improvement.

Question 623: A 23-year-old female patient presents to the emergency department after her roommate found her lying on their living room floor unconscious. Her blood pressure is 100/70 mmHg, pulse 60/min, respirations 6/min, and temperature 98.0 F (36.7 C). The patient is unable to cooperate with a physical examination. Her pupils are contracted bilaterally. A workup shows the patient took an overdose of a medication that increases the duration of chloride channel opening. What is the most appropriate next step in the management of this patient?

Choices: 1. Flumazenil 2. Alkalinize the urine 3. Activated charcoal 4. Naloxone

Answer: 3 - Activated charcoal Explanations: Barbiturates are a class of sedative-hypnotic drugs. They are commonly used as antiepileptics (phenobarbital) and for the induction of general anesthesia (thiopental). Some states administer barbiturates for provider-assisted suicide/euthanasia and use them for capital punishment by lethal injection. Barbituates potentiate GABA-A receptors and increase the duration of chloride channel opening. Even though barbiturates mediate their effects via GABA, flumazenil is not effective in the treatment of barbiturate toxicity. Barbiturates have no antidotes, and the treatment is supportive. Activated charcoal is beneficial in the management of a barbiturate overdose. However, alkalizing the urine has not been shown to be effective. Treatment of barbiturate toxicity remains supportive as there is no specific antidote or overdose. The first step in treatment, as with any overdose, is assessing the patient’s airway, breathing, and circulation. With significant sedation and respiratory depression, intubation and mechanical ventilation may become necessary. Early treatment with activated charcoal may be useful and can be given via a nasogastric tube. Go to the next page if you knew the correct answer, or click the link image(s) below to further research the concepts in this question (if desired).

Research Concepts: Barbiturate Toxicity

We update eBooks quarterly and Apps daily based on user feedback. Please tap flag to report any questions that need improvement.

Question 624: A 59-year-old man with a past medical history of heart failure with reduced ejection fraction, chronic obstructive pulmonary disease, ischemic coronary heart disease, and essential hypertension is seen in the intensive-care unit for respiratory distress with acute respiratory distress syndrome (ARDS). The patient's clinical condition has been progressively deteriorating. He is intubated and sedated. Vital signs include blood pressure 90/46 mmHg, heart rate 118/min, and temperature 37.5 C (99.5 F). The patient's hypoxia has been refractory to multiple ventilator settings. Current ventilator settings include FiO2 100%, tidal volume 460 mL, PEEP 8 cmH2O, plateau pressure 30 cmH2O, and rate 28 breaths/minute. His oxygen saturation is measured at 78% and drops to 75% on a subsequent reading 2 hours later. Which of the following is the next best step in the management of this patient's hypoxia?

Choices: 1. Decrease PEEP, increase plateau pressure 2. Establish extracorporeal membranous oxygenation (ECMO) 3. Increase respiratory rate 4. Increase PEEP, decrease plateau pressure

Answer: 2 - Establish extracorporeal membranous oxygenation (ECMO) Explanations: For cases of refractory hypoxemia and hypoxia in ARDS, ECMO may be the only option to improve oxygenation. This modality is considered a last resort. Recently, extracorporeal membrane oxygenation (ECMO) has been advocated as salvage therapy in refractory hypoxemic ARDS. Despite advances in critical care, ARDS still has high morbidity and mortality. Even those who survive can have a poorer quality of life. Besides the restriction of fluids in high-risk patients, close monitoring for hypoxia by the team is vital. The earlier the hypoxia is identified, the better the outcome. Go to the next page if you knew the correct answer, or click the link image(s) below to further research the concepts in this question (if desired).

Research Concepts: Acute Respiratory Distress Syndrome

We update eBooks quarterly and Apps daily based on user feedback. Please tap flag to report any questions that need improvement.

Question 625: A 34-year-old woman is admitted to labor and delivery for active labor. She has no significant past medical history and only takes a prenatal vitamin. The patient undergoes successful epidural catheterization on the second attempt, and neuraxial analgesia is provided with significant pain relief. Following normal spontaneous vaginal delivery of a healthy female infant, the patient is able to urinate and ambulate, and her catheter is removed on post-partum day 1. She is discharged home the next day, but returns on post-partum day 5 with thoracolumbar back pain, bilateral lower extremity numbness, and gait difficulty. What is the most likely cause of the patient's new symptoms?

Choices: 1. Fibrocartilaginous embolism with spinal cord infarction 2. Aortic dissection with occlusion of the radicular arteries 3. Spinal epidural hematoma 4. Large central disc herniation with cauda equina syndrome

Answer: 3 - Spinal epidural hematoma Explanations: Spinal epidural hematoma is a rare complication of epidural catheterization for neuraxial analgesia. It is more likely to occur in patients with a history of coagulopathy or thrombocytopenia. However, in rare cases, it can occur in a patient without identifiable risk factors. Bleeding from spinal epidural hematoma typically occurs from a venous source following an inadvertent dural puncture, and symptoms occur over the course of days following the procedure. MRI is the standard diagnostic exam for spinal epidural hematoma. Treatment of spinal epidural hematoma involves urgent decompressive laminectomy with the evacuation of blood products, particularly in the setting of new neurologic deficits. Go to the next page if you knew the correct answer, or click the link image(s) below to further research the concepts in this question (if desired).

Research Concepts: Spinal Cord Compression

We update eBooks quarterly and Apps daily based on user feedback. Please tap flag to report any questions that need improvement.

Question 626: A 36-year-old gentleman underwent right donor hepatectomy. His comorbidity includes G6PD deficiency. The procedure was uneventful, and he was transferred to ICU for monitoring. On a postoperative day 6, he has intractable hiccups, his temperature is 101.5 F, heart rate of 110/min, blood pressures of 90/60 mm Hg. Laboratory investigations reveal a total count of 38,390 cells/cumm. What is the next investigation of choice?

Choices: 1. X-ray erect abdomen 2. MRI abdomen 3. CT abdomen 4. Ultrasound abdomen

Answer: 4 - Ultrasound abdomen Explanations: Ultrasound is the gold standard investigation in the detection of a right-sided subphrenic abscess. Ultrasound can also be used in the percutaneous aspiration of the collection and insertion of a drain. Ultrasound may sometimes show debris and gas, which will help to identify if it is contaminated. CT scan of the abdomen may be used to detect left subdiaphragmatic collections. It helps in quantification and source of the collection. Contrast enhancement will suggest an abscess as air and fluid collection in the abdomen is common post-surgery. Go to the next page if you knew the correct answer, or click the link image(s) below to further research the concepts in this question (if desired).

Research Concepts: Subphrenic Abscess

We update eBooks quarterly and Apps daily based on user feedback. Please tap flag to report any questions that need improvement.

Question 627: A 61-year-old male with a history of diabetes mellitus type II, coronary artery disease, and chronic kidney disease presented to the emergency department with chest pain. A coronary angiogram is ordered, which is essentially unremarkable. The following day the patient experienced tachycardia, diarrhea, and increased sweating. This change in status was reported to the on-call clinician, who ordered the thyroid profile of the patient. Thyroid profile shows elevated T3 and T4 and reduced TSH levels. The provider explains the patient is exhibiting an uncommon thyrotoxic phenomenon. Which of the following was responsible for the patient's condition?

Choices: 1. History of thyroid malignancy 2. Exposed to iodine-containing contrast medium 3. History of diabetes 4. History of chronic kidney disease

Answer: 2 - Exposed to iodine-containing contrast medium Explanations: The Jod Basedow effect is hyperthyroidism following administration of iodine either in diet or in contrast medium. The phenomenon is iodine-induced hyperthyroidism. It can occur when a patient with an endemic goiter is relocated to an abundant iodine region. This should not be confused with the Wolff-Chaikoff effect, which exhibits a reduction in thyroid hormone secondary to large iodine ingestion, an autoregulatory phenomenon. The Jod Basedow effect can be seen with small increases of iodine in a diet previously starved of iodine. Jod-Basedow, in addition to being seen with iodine administration in the diet or iodinated intravenous contrast, can also be seen after amiodarone or prolonged use of iodinated antiseptic solutions. History of CKD is a predisposing factor; however, it is not the direct cause of the patient's symptoms. Go to the next page if you knew the correct answer, or click the link image(s) below to further research the concepts in this question (if desired).

Research Concepts: Jod Basedow Syndrome

We update eBooks quarterly and Apps daily based on user feedback. Please tap flag to report any questions that need improvement.

Question 628: A 90-year-old, 40 kg woman presents to the emergency department with severe lower back, bilateral hip, and knee pain. She declines opioids but makes multiple requests for topical lidocaine patches. An hour after placing a total of 4 patches of 5% topical lidocaine patches in the painful areas, she becomes progressively somnolent and hypotensive, requiring vasopressor support, and proceeds to cardiac arrest. Unfortunately, 20% intravenous lipid emulsion is not available, though the pharmacy did recently acquire a novel lipid formation of mixed chain fatty acids for parenteral nutrition therapy. What is the next best step in the management of this patient?

Choices: 1. Initiate advanced cardiovascular life support (ACLS) 2. Administer propofol as an intravenous lipid emulsion (ILE) alternative 3. Prepare for extracorporeal membrane oxygenation (ECMO) 4. Administer the new lipid formula

Answer: 1 - Initiate advanced cardiovascular life support (ACLS) Explanations: ACLS is the appropriate initial management for local anesthetic systemic toxicity (LAST)induced cardiac arrest. The 10% lipid emulsion used in propofol is never an acceptable intravenous lipid emulsion (ILE) alternative. ECMO may require additional personnel and setup, which should not delay ACLS and lipid emulsion therapy. 100% long-chain fatty acid formulations of lipid emulsions are recommended at this time. Go to the next page if you knew the correct answer, or click the link image(s) below to further research the concepts in this question (if desired).

Research Concepts: Lipid Emulsion Therapy

We update eBooks quarterly and Apps daily based on user feedback. Please tap flag to report any questions that need improvement.

Question 629: A 41-year-old man presents to the emergency department with a painful red and purple rash which started 3 days ago and has been getting worse since onset. The patient states that the rash started just as he was recovering from flu-like symptoms, which he had been having for 2 weeks. The patient has several medications that he has been prescribed and is currently taking. Which of the following medications is most likely responsible for the patient's symptoms?\

Choices: 1. Metformin 2. Albuterol 3. Nevirapine 4. Aluminum hydroxide

Answer: 3 - Nevirapine Explanations: Stevens-Johnson syndrome is a rare and unpredictable reaction that can be caused by drugs such as nevirapine, anti-gout medications (e.g., allopurinol), anticonvulsants, antipsychotics, acetaminophen, and ibuprofen. Risk factors for contracting Stevens-Johnson syndrome include an HIV infection, a weakened immune system, and a family history of Stevens-Johnson syndrome. When Stevens-Johnson syndrome onsets secondary to taking nevirapine, it may first present with prodromal, flu-like symptoms such as fever, sore throat, fatigue, and cough. After the consumption of nevirapine, the prodromal symptoms are followed by a painful red or purple rash that blisters and spreads. Recovery time for this syndrome can take weeks to months. Go to the next page if you knew the correct answer, or click the link image(s) below to further research the concepts in this question (if desired).

Research Concepts: Nevirapine

We update eBooks quarterly and Apps daily based on user feedback. Please tap flag to report any questions that need improvement.

Question 630: A 33-year-old woman has recently recovered from Guillain-Barre syndrome, which began 13 days ago. She was intubated for five days and now has inspiratory stridor, which is mild. On the fifth day post-extubation, she failed a swallow study as she was aspirating thin liquids. She has been on a nectar-thick diet since then. Her voice is normal. Her breathing becomes much more audible after climbing two flights of stairs, with frank biphasic stridor but no subjective discomfort. She can eat and drink on a bedside swallow examination everything except for plain water, where she coughs after swallowing. On flexible laryngoscopy, her true vocal folds are in a median position with a glottic airway estimated at 3mm. What is the next step in management?

Choices: 1. Maintain strict NPO and place a gastrostomy tube. 2. Urgent tracheostomy due to profound airway narrowing. 3. Careful observation and serial flexible laryngoscopy. 4. Posterior cordotomy to increase airway but minimize aspiration risk.

Answer: 3 - Careful observation and serial flexible laryngoscopy. Explanations: This patient has bilateral vocal fold paralysis. Guillain-Barre syndrome can rarely present as isolated bilateral vocal fold paralysis without concurrent ascending muscle weakness. As she has recovered and is largely asymptomatic, simple diet modification and close observation are all that is warranted, as full recovery is expected (though it may be a prolonged recovery course). Permanent measures to augment the airway such as cordotomy or tracheostomy are premature in this largely asymptomatic patient, as she has a very good chance of spontaneous recovery. Patients with bilateral vocal fold paralysis tend to have the vocal folds in a median position, yielding a near-normal voice but potential stridor. This can be inspiratory, expiratory, or biphasic. So long as they are comfortable breathing, expectant management is all that is needed. If they are symptomatic, a reversible procedure such as a tracheostomy is preferred. Go to the next page if you knew the correct answer, or click the link image(s) below to further research the concepts in this question (if desired).

Research Concepts: Airway Glottic Insufficiency

We update eBooks quarterly and Apps daily based on user feedback. Please tap flag to report any questions that need improvement.

Question 631: A 59-year-old woman presents with left-sided weakness. The symptoms started 30 mins ago when she was having lunch in a restaurant. The patient has a past medical history of hypertension and type 2 diabetes mellitus. On examination, the patient is awake, alert, able to follow commands, and is oriented to time, person and place. Cranial nerve examination reveals preferential gaze to the right and dysarthria. The patient is unable to recognize left-sided stimulus on double simultaneous stimulation. The motor examination shows 3/5 strength on the left upper and left lower limb. There is a 20% sensory deficit on the left to pain and light touch. Babinski sign is positive on the left. A head CT is performed. Which of the following do you expect to see on this scan?

Choices: 1. Encephalomalacic changes in the right periventricular region 2. A small area of hyperdensity in the pons 3. Insular ribbon sign with obscuration of the lentiform nucleus 4. Hyperdensity at the Sylvian fissure and base of the skull

Answer: 3 - Insular ribbon sign with obscuration of the lentiform nucleus Explanations: Ischemic brain tissue stops working in seconds and suffers necrosis in as soon as 5 minutes after complete lack of oxygen and glucose supply, prompting a myriad of potential symptoms and disability. Cerebral ischemia can be focal or generalized and cause varying clinical pictures. While generalized cerebral ischemia is generally related to syncope and pre-syncope, focal cerebral ischemia usually causes a stroke or transient ischemic attacks. As a rule of thumb, acute cerebral ischemia is never painful. Go to the next page if you knew the correct answer, or click the link image(s) below to further research the concepts in this question (if desired).

Research Concepts: Cerebral Ischemia

We update eBooks quarterly and Apps daily based on user feedback. Please tap flag to report any questions that need improvement.

Question 632: A 55-year-old male patient with no significant past medical history aside from excessive smoking presents with complaints of right lower extremity swelling and pain. Imaging demonstrates a massive iliofemoral venous thrombosis. She undergoes catheter-directed thrombolysis with tissue plasminogen activator (tPA), and the patient is being monitored closely in the ICU. After several hours fibrinogen is checked and is found to be less than 100mg/dL. If the infusion is not stopped, the patient is most at risk for which of the following complications?

Choices: 1. Myocardial rupture 2. Limb loss 3. Hypotension 4. Bleeding complications

Answer: 4 - Bleeding complications Explanations: The major and most feared complication of thrombolytic therapy is bleeding complications, most notably intracranial hemorrhage. Other complications include systemic hemorrhage, immunologic complications, hypotension, and myocardial rupture. Fibrinogen levels need to be monitored closely in patients undergoing thrombolysis to ensure they are above 150mg/dL. When levels drop below 150mg/dL, this poses a significant bleeding risk. Treatment includes reducing the infusion rate and stopping the infusion rate completely once below 100mg/dL. Platelets and hemoglobin levels should also be monitored closely as well. Hypotension is a known risk with "reperfusion"; however, generally, this can be controlled with infusions and will ultimately resolve as the infusion is stopped and patient resuscitated adequately. Myocardial rupture is a known complication when tPA is used to treat patients who suffer from a myocardial infarction but is less commonly associated with catheter-directed thrombolysis for lower extremity deep vein thrombosis (DVT). Go to the next page if you knew the correct answer, or click the link image(s) below to further research the concepts in this question (if desired).

Research Concepts: Phlegmasia Alba And Cerulea Dolens

We update eBooks quarterly and Apps daily based on user feedback. Please tap flag to report any questions that need improvement.

Question 633: A 32-year-old man is brought to the emergency department after his car engine explodes in a motor vehicle collision. The total body burn area is estimated to be around 18%. On presentation, the patient is in pain and somewhat disoriented. The Glasgow Coma Scale (GCS) score is 14/15. Nasoendoscopy shows swollen upper respiratory tract and slightly edematous vocal cords. Bronchoscopy shows mild erythema throughout and a normal lower airway. After this type of injury, which long-term complication is most likely to develop in this patient?

Choices: 1. Pneumonia 2. Atelectasis 3. Bronchiectasis 4. Pulmonary edema

Answer: 3 - Bronchiectasis Explanations: Long-term complications from smoke inhalation injury are much less common than shortterm complications. They include subglottic stenosis, bronchiectasis, bronchiolitis obliterans, interstitial fibrosis, and reactive airways dysfunction syndrome (RADS). Short-term complications are seen in more severe injuries within 4 to 5 days, and the most common complication is pneumonia. Acute respiratory distress syndrome and pulmonary edema are also seen in the short term. These patients will often demonstrate changes in pulmonary function testing and may require ventilatory support. Complication rates are higher in patients with a history of an underlying lung disease, such as chronic obstructive pulmonary disease (COPD) or asthma. Go to the next page if you knew the correct answer, or click the link image(s) below to further research the concepts in this question (if desired).

Research Concepts: Inhalation Injury

We update eBooks quarterly and Apps daily based on user feedback. Please tap flag to report any questions that need improvement.

Question 634: A 68-year-old man presents to the clinic from a nursing home with worsening dyspnea for three months. He was diagnosed with subacute pulmonary embolism three months ago and has been on anticoagulation therapy with minimal symptom improvement. CT pulmonary angiography at that time showed extensive distal emboli. His past medical history is significant for hypertension, diabetes mellitus, and cerebrovascular accident with residual leftsided weakness. Vital signs are a blood pressure of 146/88 mmHg, pulse 88/minute, and SpO2 94% on room air. Electrocardiogram (ECG) and chest x-ray are unremarkable. An echocardiogram reveals a mean pulmonary artery pressure of 35 mmHg. What is the best initial treatment for this patient?

Choices: 1. Pulmonary endarterectomy 2. Bosentan and sildenafil 3. Epoprostenol 4. Riociguat

Answer: 4 - Riociguat Explanations: Medical treatment of chronic thromboembolic pulmonary hypertension (CTEPH) is based primarily on PAH-targeted therapy, including endothelin receptor antagonists, phosphodiesterase type-5 inhibitors, and prostanoids. Riociguat, a guanylate cyclase stimulator, is used primarily in patients ineligible for surgery or persistent disease after pulseless electrical activity (PEA). Treprostinil and macitentan have also been extensively studied and proven to be beneficial in these patients. Pulmonary endarterectomy is the best treatment to improve hemodynamics and exercise capacity in operable patients. This patient with limited functional capacity and distal emboli is not considered operable. Bosentan and sildenafil are alternatives to riociguat, but riociguat is preferred in mild to moderate disease. IV epoprostenol therapy is only considered in patients who are severely ill. Go to the next page if you knew the correct answer, or click the link image(s) below to further research the concepts in this question (if desired).

Research Concepts: Chronic Thrombo-Embolic Pulmonary Hypertension

We update eBooks quarterly and Apps daily based on user feedback. Please tap flag to report any questions that need improvement.

Question 635: A 38-year-old man presents to the emergency department with lightheadedness and dizziness. His physical examination shows a blood pressure of 78/40 mmHg. He is given 2 liters of Ringer's lactate (RL) and is admitted for dehydration. All of his other labs are within normal limits. At night, a rapid response is called as the patient is confused. He is shifted to the intensive care unit and started on dexmedetomidine for a possible diagnosis of delirium tremens (DTs). His condition gets worse, and he is sedated with propofol and is intubated. An orogastric tube is also inserted, and the patient is started on tube feeds. For the next two days, the patient remains in DTs. Overnight, he has a seizure, and his oxygen requirements go up from 30% to 60%. On chest x-ray, he has bilateral basal haziness. His hemoglobin drops from 10 g/dL to 6.8 g/dL. His creatinine is elevated from 1 mg/dL to 1.8 mg/dL. His CK levels are 6000 U/L. Blood glucose level is 200 mg/dL. ABG shows a pH of 7.32, pCO2 of 48 mmHg, bicarbonate of 24 mEq/L, and pO2 of 80 mmHg. What is the next best step in the management of this patient?

Choices: 1. Continuous electroencephalogram 2. Check echocardiogram 3. Check the electrolytes panel 4. Check beta-hydroxybutyrate level

Answer: 3 - Check the electrolytes panel Explanations: The patient in the scenario has hypophosphatemia. He is at a very high risk of developing electrolyte disturbances due to alcohol use disorder and poor nutritional status. The effects of hypophosphatemia are broad and impact nearly every system. Effects primarily are due to intracellular depletion; however, chronic effects can be seen in the bone structures. Prolonged hypophosphatemia leads to osteopenia, osteoporosis, rickets, or osteomalacia due to decreased bone mineralization. The central nervous system may manifest with metabolic encephalopathy as a result of ATP depletion and may include altered mental state, irritability, paresthesias, numbness, seizures, or coma. Cardiac function is impacted by ATP depletion. In addition to possible systolic heart failure, the myocytes become less stable, and arrhythmias are possible. The decreased diaphragmatic function impacts pulmonary function with subsequent hypoventilation. Ventilator dependent patients have been shown to have longer hospital courses and worse outcomes when hypophosphatemia is present. Gastrointestinal dysfunction occurs as a result of ATP deficiency, also with dysphagia or ileus possible. Generalized muscular weakness can occur. Go to the next page if you knew the correct answer, or click the link image(s) below to further research the concepts in this question (if desired).

Research Concepts: Hypophosphatemia

We update eBooks quarterly and Apps daily based on user feedback. Please tap flag to report any questions that need improvement.

Question 636: A 60-year-old female was found in a semi-comatose condition and brought to the emergency department. On physical exam, her height was 66 inches (167.6 cm), and her weight was 123 lb (55.8 kg) (BMI=19.9 kg/m2). Her blood pressure was 80/40 mmHg; her pulse rate was 150 beats/min with a temperature of 99 F (37 C). She was disoriented and lethargic with normal deep tendon reflexes. Her home medications include aspirin and ketoconazole prescribed for onychomycosis of her toenails 6 months ago.Laboratory test results demonstrate: hemoglobin 9 g/dl; hematocrit 25%; WBC count 9840 cells/mm3; platelet count 297,000 cells/mm3; creatinine 1.09 mg/dl sodium 128 meq/l; potassium 5.2 mEq/L; chloride 93 mEq/L; and bicarbonate 18 mEq/L. Chest radiography and CT brain didn’t reveal any abnormalities. Blood and urine cultures were sent. The patient was started on intravenous fluid resuscitation with broad-spectrum antibiotics. Which of the following is true regarding this medical condition?

Choices: 1. Ketoconazole is a precipitating factor 2. The patient can present with hypocalcemia 3. Hyperglycemia is a common manifestation 4. Lymphopenia is a common manifestation

Answer: 1 - Ketoconazole is a precipitating factor Explanations: The patient has an adrenal crisis precipitated by ketoconazole use. Manifestations include hypotension, tachycardia, hyponatremia, hyperkalemia with confusion. Ketoconazole interfere with steroidogenesis and clinicians should be aware that it can precipitate adrenal crises Confusion and lethargy are common manifestations of adrenal crises. The altered mental status can be exacerbated in the setting of hyponatremia and hypoglycemia due to the inhibition of gluconeogenesis. Slow correction of hyponatremia is required to avoid precipitation of osmotic demyelination syndrome Other medications that can precipitate adrenal crises are anti-adrenal medications, mitotane, metyrapone, anticancer medications, immune checkpoint inhibitors, tyrosine kinase inhibitors(sunitinib, imatinib), fluconazole, etomidate, rifampicin, cyproterone acetate, diuretics, and megestrol acetate. In adrenal crises, the patient usually presents with hypercalcemia secondary to dehydration, lymphocytosis, and eosinophilia. Go to the next page if you knew the correct answer, or click the link image(s) below to further research the concepts in this question (if desired).

Research Concepts: Adrenal Crisis

We update eBooks quarterly and Apps daily based on user feedback. Please tap flag to report any questions that need improvement.

Question 637: A 70-year-old female with a prior history of diabetes and hypertension, who is also a former smoker, is being evaluated for dyspnea. The patient underwent an echocardiogram that revealed normal left ventricular ejection fraction. The patient was referred for pulmonary artery catheterization as echocardiogram revealed moderately elevated pulmonary artery pressures. At baseline, she has a heart rate of 78 beats per minute with a blood pressure of 148/70 mmHg. While performing right heart catheterization, the patient suddenly becomes dyspneic and complains of chest pain. Her heart rate is now 115 beats per minute. The patient is able to speak and tell you he is uncomfortable. The nurse checks the fluid-filled transducer and finds a large air column that had not been completely evacuated. What is the next best step?

Choices: 1. Remove the pulmonary artery catheter immediately 2. Flush the transducer to evacuate the air from it 3. Place the patient in Trendelenburg position and administer oxygen 4. Intubate the patient

Answer: 3 - Place the patient in Trendelenburg position and administer oxygen Explanations: Removing the pulmonary catheter is not necessary as the suspected issue is air embolism from the air column in the fluid system. Flushing the air column is a necessary step before inserting the catheter into the vein and also before proceeding again. However, as the patient has already experienced an air embolism episode, this would not be the next best step. Placing the patient in the Trendelenburg position and administering high flow oxygen should be the next best step as it helps improve reabsorption of the injected air into the systemic circulation. The patient is still alert and oriented even though he is experiencing symptoms. If the respiratory distress worsens and the patient is unable to protect her airways, intubation should be considered. But intubation would not be the immediate management. Go to the next page if you knew the correct answer, or click the link image(s) below to further research the concepts in this question (if desired).

Research Concepts: Right Heart Cardiac Catheterization

We update eBooks quarterly and Apps daily based on user feedback. Please tap flag to report any questions that need improvement.

Question 638: A 28-year-old, previously healthy male presents with a three-day history of fever, headache, nausea, and vomiting. There are nuchal rigidity and fever, but otherwise, the exam and CT of the head are normal. Lumbar puncture shows mild elevation of protein, lymphocytic pleocytosis, and normal glucose. Select the most likely diagnosis.

Choices: 1. Bacterial meningitis 2. Herpes simplex encephalitis 3. Herpes simplex meningitis 4. Enterovirus meningitis

Answer: 4 - Enterovirus meningitis Explanations: The clinical picture is that of viral meningitis although polymerase chain reactions and cultures should be done. Eighty-five percent of viral meningitis is caused by enteroviruses, usually Coxsackie and echovirus. In developed countries, herpes simplex is the next most common pathogen in adolescents and adults. Treatment with fluids and antipyretics is usually sufficient. Go to the next page if you knew the correct answer, or click the link image(s) below to further research the concepts in this question (if desired).

Research Concepts: Viral Meningitis

We update eBooks quarterly and Apps daily based on user feedback. Please tap flag to report any questions that need improvement.

Question 639: A 45-year-old male is evaluated in intensive care. He was admitted 7 days back into with road traffic acceded and extensive injuries. He was intubated due to deteriorating GCS. His condition has been worsening over the past several hours. His vitals reveal a blood pressure of 80/40 mm Hg, a pulse of 135 beats per minute, spO2 of 95 %. He is on ventilatory support with FiO2 of 0.5. His investigations reveal a total leukocyte count of 21,000 per microL, hemoglobin of 10 gm/dl, and PaO2 95 %. A sample is obtained from his central line and is subjected to blood gas analysis and reveals an oxygen saturation of 82 %. What is the likely cause of this patient’s deterioration?

Choices: 1. Myocardial depression 2. Hypovolemia 3. Pulmonary embolism 4. Sepsis

Answer: 4 - Sepsis Explanations: This patient is in critical care and has been deteriorating with hypotension and tachycardia. The laboratory investigations reveal a high WBC count and a normal arterial oxygen saturation. The oxygen saturation from the blood obtained from the central line represents central venous oxygen saturation. This is normally 65-70 %. This patient’s oxygen extraction is only 15 %, representing a reduction in oxygen extraction in the peripheral tissues. This can occur in individuals with blood redistribution, such as in sepsis. Sepsis leads to the micro-capillary obstruction, which leads to the shunting of arterial blood across the capillary bed and thus leads to an increase in venous oxygen concentration. Venous oxygen concentration can be used to guide therapy in these individuals. Venous oxygen saturation can also be measure continuously. Mixed venous oxygen saturation decreases in conditions such as heart failure, hypovolemia, and pulmonary embolism. Increased extraction or decreased oxygenation of blood will lead to reduced oxygen saturation in the venous system. Go to the next page if you knew the correct answer, or click the link image(s) below to further research the concepts in this question (if desired).

Research Concepts: Venous Oxygen Saturation

We update eBooks quarterly and Apps daily based on user feedback. Please tap flag to report any questions that need improvement.

Question 640: A 17-year-old male patient was admitted to the emergency department following an abrupt onset striking pattern of chest pain 30 minutes prior to the admission. His vitals were blood pressure: 68/40 mmHg, pulse rate: 110 beats per min, respiratory rate: 22 /min, and normal temperature. He is 2.1 m (7 ft.) in height and 77 kg (170 lbs.). Following intravenous fluid resuscitation with 1000 cc Ringer's lactate, his vitals were blood pressure: 69/50 mmHg, pulse rate: 108 bpm, respiratory rate: 20/min. His ECG and troponin levels were unremarkable, while his portable chest x-ray was significant for a wide mediastinum. What is the next preferred management?

Choices: 1. Obtaining CT-angiography 2. Obtaining MR-angiography 3. Transfer to the operating room 4. Request transesophageal echocardiography in the emergency department

Answer: 3 - Transfer to the operating room Explanations: The above suggests a diagnosis of Marfan syndrome and dissection of the aorta. Patients with a history of Marfan syndrome have a high risk of aortic complications that occur at a young age. As the patient is unstable, he should be taken to the operating room where transesophageal echocardiography (TEE) can be done. This is the modality of choice for diagnosis. TEE assesses both the aorta and the valves for pathology. Currently, the preferred modality for the diagnosis of aortic dissection is intravenous contrast CT-scan. Although the sensitivity of chest CT-scan in obtaining the diagnosis of aortic dissection is almost 100%, it is limited to those with stable hemodynamics. Go to the next page if you knew the correct answer, or click the link image(s) below to further research the concepts in this question (if desired).

Research Concepts: Aortic Dissection

We update eBooks quarterly and Apps daily based on user feedback. Please tap flag to report any questions that need improvement.

Question 641: An 82-year-old man with a history of hypertension, hyperlipidemia, and chronic obstructive pulmonary disease is brought to the emergency department from his nursing home after a medical assistant noticed a change in the patient's mental status. The nursing home states that the patient is typically alert and oriented. Upon examination, the patient is obtunded and is only aroused by painful stimulation. Accessory muscles of respiration are visibly recruited. Vital signs upon arrival show blood pressure 86/48 mmHg, heart rate 118/min, temperature 102.3 F, and respiratory rate 36/min. Chest radiography in the emergency department reveals bilateral opacities in the lung fields and a normal cardiac silhouette. The patient is intubated for airway protection. Which of the following modes of mechanical ventilation is likely to provide the highest benefit to this patient?

Choices: 1. Tidal volume 6 mL/kg, plateau pressure > 45 cmH2O 2. Tidal volume 6 mL/kg, plateau pressure 30 cmH2O 3. Tidal volume 12 mL/kg, plateau pressure 30 cmH2O 4. Tidal volume 12 mL/kg, plateau pressure > 45 cmH2O

Answer: 2 - Tidal volume 6 mL/kg, plateau pressure 30 cmH2O Explanations: Studies support the use of low tidal volume and plateau pressure less than 30 cmH2O for the management of patients with ARDS over other settings. Patients are mechanically ventilated, guarded against fluid overload with diuretics, and given nutritional support until evidence of improvement is observed. The mode in which a patient is ventilated has an effect on lung recovery. Evidence suggests that some ventilatory strategies can exacerbate alveolar damage and perpetuate lung injury in the context of ARDS. Care is placed in preventing volutrauma (exposure to large tidal volumes), barotrauma (exposure to high plateau pressures), and atelectrauma (exposure to atelectasis) A lung-protective ventilatory strategy is advocated to reduce lung injury. The NIH-NHLBI ARDS Clinical Network Mechanical Ventilation Protocol (ARDSnet) sets the following goals: Tidal volume (V) from 4 to 8 mL/kg of ideal body weight (IBW), respiratory rate (RR) up to 35/min, SpO2 88% to 95%, plateau pressure (P) less than 30 cmH2O, pH goal 7.30 to 7.45, and inspiratory-to-expiratory time ratio less than 1. To maintain oxygenation, ARDSnet recognizes the benefit of PEEP. The protocol allows for a low or a high PEEP strategy relative to FiO2. Either strategy tolerates a PEEP of up to 24 cm HO in patients requiring 100% FiO2. The inspiratory-to-expiratory time ratio goal may need to be sacrificed and an inverse inspiratory-to-expiratory time ratio strategy instituted to improve oxygenation in a certain clinical situation. Go to the next page if you knew the correct answer, or click the link image(s) below to further research the concepts in this question (if desired).

Research Concepts: Acute Respiratory Distress Syndrome

We update eBooks quarterly and Apps daily based on user feedback. Please tap flag to report any questions that need improvement.

Question 642: A 45-years-old man presents to the clinic for evaluation. He has noted more fatigue, headaches, shortness of breath, and swelling in the legs over the past few weeks. He was diagnosed with systemic sclerosis 1 year ago when he had Raynaud syndrome with digital ulcers, thickening of the skin in the upper and lower extremities, joint pain and joint swelling in the hands, heartburn, and dysphagia. He takes hydroxychloroquine, omeprazole, and amlodipine. His Raynaud syndrome, heartburn, and joint pain improved, although the skin thickening and tightening persist. His past medical history is otherwise unremarkable. Vital signs show blood pressure 198/108 mmHg, pulse 102/min, and temperature 98 F. On exam, he has 2+ pitting edema in the lower extremities. The skin of the entire bilateral upper and lower extremities and the face is thick and tight. He does not have any active digital ulcerations. He is unable to make a fist in both hands but denies any pain. The range of motion of bilateral wrists is limited with palpable crepitus on the dorsal wrists with the range of motion. Cardiac and pulmonary exams are normal. Labs reveal hemoglobin 9.1 g/dL and platelets 101000/microL. His CBC was normal 2 months ago. Which of the following is the most appropriate next step in the management of this patient?

Choices: 1. Anti-centromere, anti-SCL-70 and anti-RNA-polymerase-III antibodies 2. C-reactive protein 3. Serum creatinine and urinalysis 4. Iron and total iron-binding capacity panel

Answer: 3 - Serum creatinine and urinalysis Explanations: Clinically, this patient has diffuse systemic sclerosis (DcSSc). Severe elevation in blood pressure and new-onset anemia and thrombocytopenia suggest scleroderma renal crisis and renal function with serum creatinine and urinalysis shall be performed immediately in this patient. Risk factors for scleroderma renal crisis include diffuse systemic sclerosis, AfricanAmerican race, pericardial effusion, tendon friction rubs, new-onset anemia, anti–RNA polymerase III antibody and use of high dose (or chronic low-dose) corticosteroids. Patients with a serum creatinine of more than 3.0 mg/dL have poor outcomes. Other factors indicating poor outcome include male sex and older age of onset. Immediate treatment with high-dose ACE-inhibitors is indicated and can improve outcomes significantly. Go to the next page if you knew the correct answer, or click the link image(s) below to further research the concepts in this question (if desired).

Research Concepts: Systemic Sclerosis

We update eBooks quarterly and Apps daily based on user feedback. Please tap flag to report any questions that need improvement.

Question 643: An 82-year-old man is being evaluated in the intensive care unit. He was admitted following an emergency aortic aneurysm repair 5 days ago. Despite his vital signs being stable throughout the day, he has become increasingly confused and appears to be having visual hallucinations. He has pulled out his nasogastric tube and an arterial line and is trying to get out of bed, despite reassurance. Which of the following is the next best step in the management of this patient?

Choices: 1. Intramuscular haloperidol 2. Intravenous lorazepam 3. Oxygen via a non-rebreathe mask 4. Physical restraints

Answer: 1 - Intramuscular haloperidol Explanations: Delirium in critically ill patients is a common occurrence. Delirium is associated with increased mortality, infection rates, and increased length of hospital stay. The cause is usually multifactorial. The Society of Critical Care Medicine and the American Psychiatric Association recommends haloperidol as the first-line drug of choice or olanzapine if haloperidol is not tolerated. Risk factors for delirium include old age, alcoholism, smoking, visual/ hearing impairment, sepsis/pyrexia, sleep disturbance, immobilization, and medication. Benzodiazepines should be avoided in delirium as they may worsen symptoms and be ineffective. Go to the next page if you knew the correct answer, or click the link image(s) below to further research the concepts in this question (if desired).

Research Concepts: Postoperative Delirium

We update eBooks quarterly and Apps daily based on user feedback. Please tap flag to report any questions that need improvement.

Question 644: A 66-year-old male patient is diagnosed with non-STEMI and is treated with low molecular weight heparin. The initial lab test is as follows: WBC:8000/µl, HCT:41%, Plt:180,000/mm3.The daily physical examination on the fifth day of admission was significant for swelling and tenderness in the passive movements of his left leg. The lab tests on the day 8 are as follows; WBC:9000/µl, HCT:39%, Plt:80,000/mm3, Total serum bilirubin: 2.5 mg/dl,AST: 250 U/L,ALT: 100 U/L,Urea: 80,and Creatinine: 2 mg/dl. What is the preferred management?

Choices: 1. Stop heparin therapy and platelet transfusion 2. Emergent hemodialysis and platelet transfusion 3. Stop heparin therapy and the addition of argatroban 4. Start argatroban treatment and taper the heparin therapy within 5 following days

Answer: 3 - Stop heparin therapy and the addition of argatroban Explanations: Argatroban is an FDA approved agent for prophylaxis and treatment of thrombosis in patients with HIT (heparin-induced thrombocytopenia and HITTS (heparin-induced thrombocytopenia and thrombosis syndrome). Argatroban doesn’t need dose adjustment in elderly patients, in patients with impaired renal function or based on gender. Argatroban is metabolized primarily by hepatic cytochrome P450 enzymes and excreted mainly via feces. Argatroban should be used with caution in patients with hepatic dysfunction and a lower dose should be used. The treatment of choice in heparin-induced thrombocytopenia is stopping heparin and begin an alternative anticoagulant. Stopping heparin without the addition of another anticoagulant is not adequate to prevent thrombosis in this setting. Alternative anticoagulants are primarily thrombin inhibitors. Go to the next page if you knew the correct answer, or click the link image(s) below to further research the concepts in this question (if desired).

Research Concepts: Argatroban

We update eBooks quarterly and Apps daily based on user feedback. Please tap flag to report any questions that need improvement.

Question 645: A 65-year-old female with alcohol use disorder is transported by paramedics to the emergency department with severe agitation. On examination, she is confused and irritable. The pulse rate is 110 bpm, blood pressure is 170/100 mm Hg and no fever. She does not have any neurological deficits. She is admitted to the psychiatry ward and treated with thiamine, intravenous fluids, and benzodiazepines. Her serum sodium is found to be 112 mEq/L and is corrected with 3 percent saline over 24 hours, and the repeat sodium value is 136 mEq/L. Her condition deteriorated, and she became non-responsive with agonal breathing. She could not be revived after cardiopulmonary resuscitation. What is the most likely risk factor that led to the unfortunate demise of this patient?

Choices: 1. Serum sodium 112 mEq/L for less than 6 hours before admission 2. Serum sodium 112 mEq/L for 6-12 hours before admission 3. Serum sodium 112 mEq/L for 12-24 hours before admission 4. Serum sodium 112 mEq/L for more than 48 hours before admission

Answer: 4 - Serum sodium 112 mEq/L for more than 48 hours before admission Explanations: Central pontine myelinolysis (CPM) is an acute demyelinating condition that primarily affects the pons, although some cases have concomitant extra pontine myelinolysis. It commonly occurs in malnourished patients or patients with alcohol use disorder who undergo rapid correction of chronic hyponatremia. The clinical picture includes seizures, dysphagia, dysarthria, pseudobulbar palsy, behavioral abnormalities, hyperreflexia, quadriplegia, and coma. Serum sodium of less than 120 mEq/L for more than 48 hours is a risk factor for CPM. Go to the next page if you knew the correct answer, or click the link image(s) below to further research the concepts in this question (if desired).

Research Concepts: Central Pontine Myelinolysis

We update eBooks quarterly and Apps daily based on user feedback. Please tap flag to report any questions that need improvement.

Question 646: A 32-year-old woman is intubated for the last 5 days in an ICU setting after being hit by a car in a motor vehicle collision. She develops a high-grade fever, and vital signs show a blood pressure of 90/60 mmHg, a heart rate of 110 beats per minute, and a temperature of 102oF. Cultures are taken, and within 48 hours of incubation, Gram-positive cocci are found in clusters. The tests for catalase and coagulase are both positive. She is started on vancomycin, but her condition keeps on worsening despite the treatment. Which of the following is the most appropriate next step in the management of this patient?

Choices: 1. Stop vancomycin and start daptomycin 2. Stop vancomycin and start piperacillin/tazobactam 3. Stop vancomycin and start linezolid 4. Stop vancomycin and start rifampin

Answer: 3 - Stop vancomycin and start linezolid Explanations: The findings in this patient suggest infection with a methicillin-resistant Staphylococcus aureus (MRSA). MRSA is a leading cause of hospital-acquired and ventilator-associated pneumonia. Hospital-acquired pneumonia (HAP), or nosocomial pneumonia, is characterized as pneumonia developing 48 hours or more after hospital admission, indicating that it was not incubating at the time of admission. Ventilator-associated pneumonia (VAP) is defined as pneumonia developing 48 hours or more after implementation of endotracheal intubation and mechanical ventilation and was not present before intubation. Linezolid is the drug of choice for pneumonia caused by MRSA that has failed vancomycin treatment. Daptomycin is not a choice because, despite covering MRSA, it is deactivated by surfactants in the lungs. MRSA is not affected by piperacillin/tazobactam. Rifampin is effective against MRSA, but it should not be used alone. Go to the next page if you knew the correct answer, or click the link image(s) below to further research the concepts in this question (if desired).

Research Concepts: Methicillin Resistant Staphylococcus Aureus

We update eBooks quarterly and Apps daily based on user feedback. Please tap flag to report any questions that need improvement.

Question 647: A 16-year-old male presents with spiking fevers, diaphoresis, and progressive dyspnea for the past week. He was shot in the right chest and had a chest tube placed a month ago. He was discharged home after a five-day admission. X-ray, ultrasound, and CT scan of the chest reveal pleural effusion. Which of the following is the most likely cause of this patient's condition?

Choices: 1. Chest tube placement 2. Pneumonia 3. Trauma 4. Atelectasis

Answer: 3 - Trauma Explanations: Around 20% of patients with pneumonia will develop a parapneumonic effusion that can lead to empyema. About 70% of patients with empyema have parapneumonic effusion. The other 30% are related to trauma, post-thoracic surgery, esophageal rupture, cervical infection, or, in small numbers, a primary empyema (not associated with previous pneumonia or intervention). If the blood is not adequately evacuated from the chest cavity, it can clot and get infected, resulting in empyema. An infected hemothorax usually presents as fever after a few weeks. Chest x-ray and CT scan are diagnostic. Delayed presentation means an open thoracotomy, but the surgeon may attempt videoassisted thoracoscopic surgery to clean the chest. Go to the next page if you knew the correct answer, or click the link image(s) below to further research the concepts in this question (if desired).

Research Concepts: Empyema

We update eBooks quarterly and Apps daily based on user feedback. Please tap flag to report any questions that need improvement.

Question 648: A 52-year-old man is scheduled for a Whipple procedure for pancreatic carcinoma. An ultrasound-guided right internal jugular cannulation is attempted, but unfortunately, the carotid is punctured twice. Another proceduralist decides to try cannulation of the left internal jugular vein. Which of the following complications is most likely to occur in this patient now, compared to a right-sided attempt?

Choices: 1. Brachial plexus injury 2. Hemopneumothorax 3. Chylothorax 4. Infection

Answer: 3 - Chylothorax Explanations: The thoracic duct usually starts from the level of the twelfth thoracic vertebrae (T12). It extends to the root of the neck. It drains into the venous system at or near the junction of the left internal jugular vein and the left subclavian veins at the commencement of the brachiocephalic vein. It can be injured when placing a central line in the left internal jugular vein. Thoracic duct injury is a very rare complication of central line placement, but it is a unique injury occurring during left-sided placement. The thoracic duct collects lymph in the body except for the right thorax, upper limbs, head, and neck. It transports fats from the intestinal lymphatics to the venous system. It also transports protein and T-lymphocytes back to the venous circulation. Injury to the thoracic duct causes chylothorax. Chylothorax can be diagnosed by pleural fluid analysis. The pleural fluid will reveal high-fat content, specifically triglycerides, and high T-lymphocyte count in chylothorax. Central line placement is associated with a lot of complications including infection, pneumothorax, and brachial plexus injury. However, the rates of these complications are similar between the left and right sides. The carotid puncture may be more common on the left side as there is more overlap of the carotid artery and internal jugular vein on the left side as compared to the right side. Go to the next page if you knew the correct answer, or click the link image(s) below to further research the concepts in this question (if desired).

Research Concepts: Internal Jugular Vein Central Venous Access

We update eBooks quarterly and Apps daily based on user feedback. Please tap flag to report any questions that need improvement.

Question 649: An incident involving a hazardous chemical in an industrial facility is reported by the local authorities. A 40-year-old male who was present at the facility is brought in by his co-worker minutes after the incident. The patient complains of irritation of the eyes, throat, and skin. His vital signs are within normal limits. The patient is not in acute distress, and he can speak in full sentences. The hospital you work at does not have a toxicology service, intensive care unit, or a specialized HAZMAT team available. What is the next step in the management of this patient?

Choices: 1. Put personal protective equipment on the patient and emergency department staff. Transfer the patient to the closest facility with a toxicology service, a HAZMAT team, and an intensive care unit available 2. Isolate the patient in a negative pressure room in the emergency room and consult a poison control 3. Establish intravenous access, apply oxygen by non-rebreather, and start fluid resuscitation 4. Remove the patient’s clothing, perform copious water irrigation, place on a cardiac monitor, and apply oxygen

Answer: 4 - Remove the patient’s clothing, perform copious water irrigation, place on a cardiac monitor, and apply oxygen

Explanations: Personal protective equipment is used to minimize exposure of medical staff and first responders. Transferring the patient to another facility without previous decontamination entails an Emergency Medical Treatment and Active Labor Act (EMTALA) violation. Contaminated victims should be placed in a negative pressure room where the decontamination process is performed and where basic life support can be provided. Intravenous access and fluid resuscitation are part of medical care after exposure to hazardous chemicals. However, these measures should not delay decontamination. Decontamination is the priority in HAZMAT victims. This is most effectively achieved with the removal of clothes and copious water irrigation. After decontamination is done, the patient can be transferred to a higher-level facility. Go to the next page if you knew the correct answer, or click the link image(s) below to further research the concepts in this question (if desired).

Research Concepts: Toxic Exposure Hazardous Materials

We update eBooks quarterly and Apps daily based on user feedback. Please tap flag to report any questions that need improvement.

Question 650: A 67-year-old man presents to the hospital with a chief complaint of chest pain. The patient is found to have severe aortic valve insufficiency. He is taken emergently to the operating room, where he undergoes replacement of his aortic valve. He is found to have a severely reduced ejection fraction and is unable to come off cardiopulmonary bypass. His chest remains open for 24 hours until he can be placed on a veno-arterial ECMO for cardiac support, and he undergoes a left-sided heart pump system placement. The patient undergoes a cardiac transesophageal echocardiogram and is found to have an ejection fraction of 12%. Vital signs show blood pressure of 94/64 mmHg, heart rate of 64/min, and oxygen saturation of 98%. Which of the following is the primary strategy for weaning this patient from VA ECMO?

Choices: 1. Norepinephrine infusion 2. Resolution of the underlying cause of cardiogenic shock 3. Decrease oxygenation 4. Removal of the left-sided heart pump system

Answer: 2 - Resolution of the underlying cause of cardiogenic shock Explanations: ECMO support cannot be withdrawn until the source of the initial problem is resolved. VA ECMO support is performed to aid in arterial support of the body for propulsion when there is cardiogenic shock. Regarding this patient who has a hard time weaning from cardiopulmonary bypass, they can be placed on VA ECMO until the source of the cardiogenic shock can be resolved. Weaning from ECMO cannot be performed on this patient at this time due to his continued low ejection fraction. Serial transesophageal echoes can be completed to look for signs of improvement in heart function daily or even more frequently. Weaning from VA ECMO can usually occur when the ejection fraction has improved to 25%-30%. When placing a patient on VA ECMO, a left-sided heart pump system can be placed to help offload the left ventricle. VA ECMO can cause the left ventricle to form a thrombus due to congestion of blood in the left ventricle. The left-sided heart pump system can be placed into the left ventricle to help propel blood forward to help offload the work of the left ventricle. When weaning VA ECMO, the pump can be left in place to aid the left side of the heart during weaning. When weaning VA ECMO, the patient will need an EF greater than 25% to 30%. Therefore he is not ready to be weaned with heavy pressor support. O2 saturations are important with regards to weaning VA ECMO, but with this patient's low EF, this takes precedence when weaning trials are going to be attempted. The left-sided heart pump system does not need to be removed prior to VA ECMO weaning attempts. Go to the next page if you knew the correct answer, or click the link image(s) below to further research the concepts in this question (if desired).

Research Concepts: Extracorporeal Membrane Oxygenation Weaning

We update eBooks quarterly and Apps daily based on user feedback. Please tap flag to report any questions that need improvement.

Question 651: A 65-year-old immigrant from India presents to establish primary care. He had tuberculosis at the age of 50 and received several years of treatment for that. He brings a "declared cured" report card from a tuberculosis control program from India. He reports having an effusion of the right side for which he underwent several thoracenteses. He states that "they never found any bacteria in that." He also reports that he has some pleural thicking around his lung and was told it would persist for a long time. CT scan of this chest showed right pleural thicking with mild effusion without any parenchymal changes. CT guided thoracentesis shows a milky white grossly exudative fluid, which is lymphocyte-rich (90%) fluid. The fluid is negative for acid-fast bacilli (AFB) stain and polymerase chain reaction (PCR) assay to detect Mycobacterium tuberculosis (TB). He is also negative for interferon-gamma release assay (IGRA). Which of the following is the next best step in the management of this patient?

Choices: 1. Start treatment with second-line treatment for TB due to exudative and lymphocytic pleural fluid 2. Order a tuberculin skin test to look for reaction and if positive, treat with 4 drugs first-line therapy 3. Send pleural fluid for triglycerides and cholesterol levels to rule out pseudo-chylous effusion 4. Thoracic surgery consultation for decortication

Answer: 3 - Send pleural fluid for triglycerides and cholesterol levels to rule out pseudochylous effusion

Explanations: The patient is presently asymptomatic and likely to have trapped lung with chronic effusion. The patient has been adequately treated for tuberculosis in the past and has negative TBPCR assay with negative TB-gold test making active tuberculosis unlikely. Long-standing exudative fluid can accumulate cholesterol from membranes of lysed cells leading to pseudo-chylothorax. True chylothorax, which is caused by the accumulation of chyle, is rich in triglycerides and deficient in cholesterol. The triglycerides will be more than 120 mg/dl, and the cholesterol will be less than 100 mg/dl in true chylous effusion. In pseudo-chylous effusion, triglycerides are less than 100 mg/dl, and cholesterol is more than 200 mg/dl. The presence of chylomicrons detected through electrophoresis will confirm the true chylothorax, and the presence of cholesterol crystals confirms the diagnosis of pseudochylous effusion. As the patient is from a country with endemic tuberculosis with BCG vaccinations, the tuberculin test would have very high false-positive rates. As the patient is asymptomatic, neither the antituberculosis treatment nor the surgical decortication is needed. However, the patient needs to closely follow up due to the high risk of relapse. Go to the next page if you knew the correct answer, or click the link image(s) below to further research the concepts in this question (if desired).

Research Concepts: Chylothorax

We update eBooks quarterly and Apps daily based on user feedback. Please tap flag to report any questions that need improvement.

Question 652: A 76-year-old man is brought to the emergency department for an altered mental status for the last two days. His past medical history is significant for heart failure with reduced ejection fraction, type 2 diabetes mellitus, hypertension, coronary artery disease statuspost coronary artery bypass graft (CABG) 2 years ago, and chronic obstructive pulmonary disease (COPD). On examination, he appears fatigued but is easily arousable. He is not oriented to time, place, or person. Systemic examination reveals prominent pulsations in his neck, bilateral basal inspiratory crackles, and bilateral pitting pedal edema. His extremities are cold to touch, but he is moving all his limbs. No focal deficits are noted. A computed tomography (CT) scan of the brain is unremarkable. Vital signs reveal a pulse of 106 beats per minute, blood pressure of 70/52 mmHg, and pulse oximetry of 88% on room air. A chest x-ray reveals bilateral pulmonary congestion. An electrocardiogram (EKG) shows normal sinus rhythm with an old left bundle branch block. The patient is started on norepinephrine and transferred to the coronary care unit for further management. A procedure to check his left-sided filling pressures is being considered. Which of the following complications is most likely to occur as a consequence of this procedure in this patient?

Choices: 1. Infection 2. Pulmonary artery rupture 3. Complete heart block 4. Pulmonary hypertension

Answer: 3 - Complete heart block Explanations: This patient is in cardiogenic shock, as evidenced by the hypotension, need of pressors, and pulmonary edema. The anticipated procedure is a right heart catheterization to measure pulmonary capillary wedge pressure, which is a surrogate marker of left-sided filling pressures. A relative contraindication is the presence of left bundle branch block. If this is the case, due to the risk of the right bundle branch block during catheter insertion, external or transvenous pacer should be placed at the moment of the procedure to prevent complete heart block. Though infection and pulmonary artery rupture are possible complications, they are less likely in this patient if appropriate precautions and protocols are followed. Go to the next page if you knew the correct answer, or click the link image(s) below to further research the concepts in this question (if desired).

Research Concepts: Pulmonary Capillary Wedge Pressure

We update eBooks quarterly and Apps daily based on user feedback. Please tap flag to report any questions that need improvement.

Question 653: A 65-year-old female patient with a significant history of hypertension presented to the clinic for evaluation of dyspnea upon exertion. She has no other complaints. She is compliant with her lisinopril 10 mg and furosemide 20 mg every day. The blood pressure at the visit was noted to be 120/80 mmHg. Physical examination revealed a diastolic murmur at the apex of the heart. Breath sounds were clear on both lungs. An echocardiogram revealed an LVEF of 55-60% by visual estimation. Left atrial enlargement and severe mitral stenosis were noted. What is the next best step in the management of the patient?

Choices: 1. Increase the dose of lisinopril 2. Refer for percutaneous mitral balloon valvuloplasty 3. Increase the dose of furosemide 4. Perform a chest X-ray

Answer: 2 - Refer for percutaneous mitral balloon valvuloplasty Explanations: Left atrial enlargement (LAE) is most commonly due to conditions caused by pressure overload, volume overload, or both. Mitral valve stenosis is one of the several underlying causes of LAE. Patients with LAE might present with a variety of symptoms, including palpitations, dyspnea, syncope, peripheral edema, fatigue, and weight gain. No known medical therapy is available to reverse the left atrial remodeling. The focus of care and medical therapy in patients with LAE is the identification and treatment of underlying pathologies. Go to the next page if you knew the correct answer, or click the link image(s) below to further research the concepts in this question (if desired).

Research Concepts: Left Atrial Enlargement

We update eBooks quarterly and Apps daily based on user feedback. Please tap flag to report any questions that need improvement.

Question 654: A 29-year-old woman presents to the emergency department for presyncope and persistent hypotension. She reports a family history of pulmonary embolism secondary to protein C deficiency. Acute pulmonary embolism is noted on CT pulmonary angiography. Anticoagulation with IV heparin and a vitamin K antagonist is initiated. If protein C deficiency is confirmed in this patient, what is the preferred duration of anticoagulant therapy?

Choices: 1. 4-6 weeks 2. 6-12 weeks 3. 3-6 months 4. Lifelong

Answer: 4 - Lifelong Explanations: In patients with an acute pulmonary embolism (PE), anticoagulation therapy is generally continued for three to six months if there is an identifiable provoking, transient, or reversible risk factor (recent trauma, orthopedic procedure, surgery, prolonged immobilization). In patients with inherited thrombophilias or persistently elevated risk for PE, lifelong anticoagulation is preferred given their high risk for recurrence. Patients with protein C or S deficiency may require lifelong anticoagulation. Placement of an inferior vena cava filter should be considered if anticoagulants are contraindicated, or a pulmonary embolism occurs in a patient who is being anticoagulated. Go to the next page if you knew the correct answer, or click the link image(s) below to further research the concepts in this question (if desired).

Research Concepts: Acute Pulmonary Embolism

We update eBooks quarterly and Apps daily based on user feedback. Please tap flag to report any questions that need improvement.

Question 655: A 72-year-old man is admitted to the hospital with urosepsis. He has a past medical history of hypertension. He is receiving intravenous fluoroquinolones. Blood tests show creatinine 1.8 mg/dl (0.8-1.3). On the fourth day of admission, his creatinine is 1.3 mg/dl, and he is clinically improving. He develops a mild maculopapular rash on both lower extremities on the fifth day of hospitalization, and his creatinine goes up to 2 mg/dl and to 2.6 mg/dl the next day. There is no new onset of fever, dysuria, or cough. A kidney biopsy is performed and, pending the result, his antibiotic is held. His creatinine started trending down after three days of withholding the antibiotic. What is the pathophysiological mechanism of this patient's disease?

Choices: 1. Delayed hypersensitivity reaction 2. Ischemic renal injury 3. Glomerulitis and tubulitis associated with the infection 4. Podocyte injury

Answer: 1 - Delayed hypersensitivity reaction Explanations: This scenario is suggestive of drug-induced acute interstitial nephritis (AIN). New onset of renal failure associated with the rash. The classic triad of low-grade fever, rash, and eosinophilia may not be present in all patients. Drug-induced AIN is an idiosyncratic delayed hypersensitivity reaction. It is not related to the dose or previous exposure of the offending agent. Go to the next page if you knew the correct answer, or click the link image(s) below to further research the concepts in this question (if desired).

Research Concepts: Interstitial Nephritis

We update eBooks quarterly and Apps daily based on user feedback. Please tap flag to report any questions that need improvement.

Question 656: A 65-year-old male patient presents to the emergency department with a one-day history of fever, sharp substernal chest pain, and difficulty breathing. He states that the chest pain is worse when lying down and improves when leaning forward. He recently had a myocardial infarction five weeks ago and is concerned about a reoccurrence. He has a past medical history of diabetes and hypertension. His blood pressure is 138/82 mmHg, pulse 88/min, respirations 18/min, and temperature 101.0 F (38.3 C). The patient has a pericardial friction rub on examination. His EKG shows diffuse ST elevation concave upward with PR depression. What is the most appropriate next step in management?

Choices: 1. Administer naproxen 2. Administer aspirin, morphine, and nitrate 3. Urgent cardiac catheterization 4. Discharge and recommend follow up in 3 weeks

Answer: 1 - Administer naproxen Explanations: This case is likely due to Dressler syndrome, also known as post-myocardial infarction syndrome, which is a benign, autoimmune pleuritis or pericarditis that can occur weeks after the event. Dressler syndrome (DS) is an immune-related inflammatory response after this patient's myocardial infarction five weeks prior. Therefore medications that would be appropriate to treat DS would typically include NSAIDs, such as naproxen. Dressler syndrome is a form of secondary pericarditis. Pericarditis is a condition in which the pericardium (a fibroelastic sac surrounding the heart and comprised of a parietal and visceral layer which is separated by a potential space) becomes inflamed. Under normal circumstances, the pericardial cavity holds 15 to 50mL of pericardial fluid (an ultrafiltrate of plasma). The exact cause of DS is unknown, though it is thought to be immune-mediated. Antimyocardial antibodies have been shown to be elevated in the blood of patients with DS, but it is unclear whether these antibodies are the cause or occur as a result of the syndrome. These antimyocardial antibodies are thought to target antigens that have become exposed through damage to the pericardium. Go to the next page if you knew the correct answer, or click the link image(s) below to further research the concepts in this question (if desired).

Research Concepts: Dressler Syndrome

We update eBooks quarterly and Apps daily based on user feedback. Please tap flag to report any questions that need improvement.

Question 657: A 60-year-old man is admitted with severe cardiogenic shock. A pulmonary artery catheter is placed to gauge the volume status of the patient. After 48 hours, the patient develops a fever (101°F). Blood tests show a leucocyte count of 16000 per microliter (4500 to 11000) with 80% neutrophils and normal platelet count. An infection of the pulmonary artery catheter is suspected. What is the most appropriate next step in the management of this patient?

Choices: 1. Remove the catheter 2. Remove the catheter and introducer 3. Remove the catheter and introducer, culture the catheter tip 4. Remove the catheter and introducer, and culture the introducer tip

Answer: 4 - Remove the catheter and introducer, and culture the introducer tip Explanations: When a pulmonary artery catheter infection is suspected, the highest yield is to culture the introducer rather than the catheter itself. For infected central venous catheters, it is necessary to culture the tip and not the subcutaneous segment. When a catheter-related infection is suspected, and there is catheter exit site exudate, swab the drainage to obtain samples for gram stain and culture. A diagnosis of a catheter-related infection requires one of the following, the same organism is cultured from a peripheral vein and the catheter tip, the same organism is cultured from a peripheral vein and the catheter hub, the same organism is cultured from samples from two catheter lumens, or one sample is quantitatively three times greater than the other. Go to the next page if you knew the correct answer, or click the link image(s) below to further research the concepts in this question (if desired).

Research Concepts: Central Line Associated Blood Stream Infections

We update eBooks quarterly and Apps daily based on user feedback. Please tap flag to report any questions that need improvement.

Question 658: A 16-year-old female patient restrained passenger with lap and shoulder belt is involved in a rear-end accident with another car. The patient now arrives in the trauma bay and is emergently evaluated. Her vital signs are blood pressure: 108/70 mmHg, pulse rate: 90 beats per minute, respiratory rate: 19 breath per minute, and temperature: 99.6 F. She is hemodynamically stable, denies any loss of consciousness, denies neck pain, and only complains of mild pain in her lower abdomen. On clinical exam, the patient is alert and oriented, and breath sounds are equal bilaterally, the abdomen is tender with voluntary guarding without peritoneal signs. She maintains movement in her lower extremities with normal sensation, anal sphincter tone is intact, and denies any bowel or bladder incontinence. She only takes birth control pills. The patient has a chest and pelvic x-ray, which are both normal, and undergoes focused abdominal sonography for trauma evaluation, which is unable to view her bladder. Blood is noted at her urethral meatus, and a retrograde urethrogram is performed, which shows an extraperitoneal bladder rupture. What would be the next step management at this time?

Choices: 1. Obtain CT of her brain and cervical spine as well as chest, abdomen, and pelvis with intravenous contrast 2. Obtain CT of her abdomen and pelvis with intravenous contrast with delays and if no intraperitoneal injury, place an intra-urinary bladder catheter and consult a surgeon 3. Proceed emergently to the operating room for repair of the extra-peritoneal bladder rupture 4. Place an emergent supra-pubic urinary catheter to drain the bladder and prevent renal injury

Answer: 2 - Obtain CT of her abdomen and pelvis with intravenous contrast with delays and if no intra-peritoneal injury, place an intra-urinary bladder catheter and consult a surgeon

Explanations: A common injury pattern seen in motor vehicle crashes in patients who wear lap and shoulder restraints improperly is mesenteric and hollow viscus injuries. Due to the nature of the bladder's intra-pelvic position, it is often not frequently injured. However, patients who have a full bladder who are subject to this mechanism of injury are at high risk for bladder rupture. Furthermore, young children have not developed enough that their intra-abdominal organs have migrated to be protected by the lower thoracic cage or pelvis and thus remain more susceptible to blunt force injuries. Obtaining a computed tomography (CT) of the abdomen and pelvis would be a reasonable diagnostic study to look for other occult injuries to the intra-abdominal organs. Computed tomography with delays is adequate for examining the remainder of the urogenital system and to determine the extent of urogenital injuries further. It also allows for the definitive classification of a bladder injury being either intra or extra-peritoneal in nature. This is important as both are treated differently. If no other intra-abdominal injury is found that requires operative therapy, an extraperitoneal bladder rupture is managed with intra-urinary bladder catheter drainage with eventual contrast cystogram and removal under a urologist's care. Suppose an intraperitoneal injury is found that warrants operative exploration, directed repair of an extraperitoneal bladder rupture can be carried out. Suprapubic urinary catheters have fallen out of favor as primary treatment but are sometimes used in major urogenital trauma when directed intra-urinary bladder catheter drainage is not possible. In pediatric trauma patients, all attempts are made to limit potentially unnecessary ionizing radiation exposure. Low energy imaging protocols are specially designed and utilized in this patient group. In patients who are alert and oriented without concern for the loss of consciousness or altered sensorium, computed tomography of the brain may be withheld. By PECARN (Pediatric Emergency Care Applied Research Network) algorithms, this patient would have a less than 0.05% risk of a clinically significant traumatic brain injury. Also, patients who have a normal neurologic exam are not intoxicated, have no midline cervical tenderness, neurologic deficit, or distracting injury qualify for clinical cervical spine clearance based on the National Emergency X-Radiography Utilization Study (NEXUS criteria). This prevents the real but small risk of ionizing radiation to a pediatric patient with the possible future development of a malignancy. Go to the next page if you knew the correct answer, or click the link image(s) below to further research the concepts in this question (if desired).

Research Concepts: Blunt Force Trauma

We update eBooks quarterly and Apps daily based on user feedback. Please tap flag to report any questions that need improvement.

Question 659: A 32-year-old male is admitted to the ICU with the diagnosis of septic shock, intubated, sedated, and with a central venous catheter. Closer inspection of the patient reveals a right calf that is tense and a right foot that is cool and pulseless. Further proximal inspection of the right lower extremity reveals two bandaged puncture sites to the proximal medial tibia 2 cm medial to the tibial tuberosity. What is the next best step in the management of this patient?

Choices: 1. Venous ultrasonography of the right lower extremity 2. Arterial ultrasound of the affected extremity 3. Orthopedic consult 4. Obtain compartment pressures on the distal right lower extremity

Answer: 4 - Obtain compartment pressures on the distal right lower extremity Explanations: One complication of Intraosseous (IO) needles is compartment syndrome. This can result from penetration through the posterior cortex of the bone causing infusion of medication and fluids into the muscular compartments of the lower extremity. Two IO needle punctures in the same bone can cause extravasation of medications through the previous puncture site into the tissues. Necrosis of tissue can occur with extravasation of medication and fluids. Every IO needle needs to be stabilized to help ensure it is not dislodged. Go to the next page if you knew the correct answer, or click the link image(s) below to further research the concepts in this question (if desired).

Research Concepts: Intraosseous Vascular Access

We update eBooks quarterly and Apps daily based on user feedback. Please tap flag to report any questions that need improvement.

Question 660: A 65-year-old woman presents to the hospital complaining of productive cough, fever with rigors and chills, and pleuritic type chest pain for five days. She has been suffering from diabetes mellitus and hypertension for ten years. On examination, her blood pressure is 160/90 mmHg, the pulse is 91/min regular, and she has a high-grade fever. On chest auscultation, there are coarse crackles in the right lower chest with absent breath sounds in the base. A chest radiograph shows right-sided infiltrates and basal opacity with lateral upward sloping of a meniscus-shaped contour. A pleural tap is done which shows frank pus. What is the most appropriate next step?

Choices: 1. Chest drain insertion 2. Therapeutic pleurocentesis 3. Pleurodesis 4. Bronchoscopy

Answer: 1 - Chest drain insertion Explanations: This patient appears to be suffering from pneumococcal pneumonia along with empyema which is a grave complication of the disease. Frank pus in the pleural tap is one of the indications of passing a chest drain as soon as possible. The mainstay of treatment for patients with S. pneumonia is antibiotic therapy and supportive care including mechanical ventilation if necessary. Treatment of CAP varies based on the area of practice and severity of the disease. Patients with low-risk CAP are typically treated as an outpatient with macrolide monotherapy. Respiratory fluoroquinolones are used in outpatient patients at higher risk. Non-ICU inpatients are treated with a dual therapy of a b-lactam plus a macrolide or respiratory fluoroquinolone monotherapy. Go to the next page if you knew the correct answer, or click the link image(s) below to further research the concepts in this question (if desired).

Research Concepts: Streptococcus Pneumoniae

We update eBooks quarterly and Apps daily based on user feedback. Please tap flag to report any questions that need improvement.

Question 661: A 51-year-old man develops adult respiratory distress syndrome (ARDS) after being admitted for a pelvic fracture. On the third ICU day, he is sedated, intubated, and ventilated with PEEP (positive end-expiratory pressure) of 12 cmH2O and a FiO2 of 0.6 (60%). Suddenly, his pulse increases from 100/min to 140/min, systolic blood pressure drops from 120 to 90 mmHg, central venous pressure increases from 25 to 45 cmH2O, and jugular veins are distended. Which of the following is the most likely cause of the patient's sudden worsening?

Choices: 1. Shifting of the endotracheal tube into the right main bronchus 2. Pulmonary thromboembolism 3. Myocardial infarction 4. Tension pneumothorax

Answer: 2 - Pulmonary thromboembolism Explanations: Pulmonary embolism (PE) occurs when there is a disruption to the flow of blood in the pulmonary artery or its branches by a thrombus that originated somewhere else. PE usually occurs when a part of this thrombus breaks off and enters the pulmonary circulation. Very rarely, PE can occur from the embolization of other materials into the pulmonary circulation such as air or fat following a trauma; in this case, pelvic fracture. The most common symptoms of PE include dyspnea, pleuritic chest pain, cough, hemoptysis, presyncope, or syncope. Blood clot travels through the blood to any remote site such as lungs, causing pulmonary embolism, which causes acute respiratory distress, which is life-threatening. Go to the next page if you knew the correct answer, or click the link image(s) below to further research the concepts in this question (if desired).

Research Concepts: Acute Pulmonary Embolism

We update eBooks quarterly and Apps daily based on user feedback. Please tap flag to report any questions that need improvement.

Question 662: A 60-year-old female was found in a semi-comatose condition and brought to the emergency department. On physical exam, her height was 66 inches (167.6 cm), and her weight was 123 lb (55.8 kg) (BMI=19.9 kg/m2). Her blood pressure was 80/40 mm; her pulse rate was 50 beats/min with a temperature of 32 C (89.6F). She was disoriented and lethargic. There was no palpable thyroid enlargement, but there was a scar on her neck. Laboratory test results demonstrate: hemoglobin 9 g/dl; hematocrit 25%; WBC count 9840 cells/mm3; platelet count 297,000 cells/mm3; creatinine 1.09 mg/dl sodium 126 meq/l; potassium 3.8 mEq/L; chloride 93 mEq/L; and bicarbonate 24.5 mEq/L. Chest radiography and CT brain didn’t reveal any abnormalities. Blood and urine cultures were sent. The patient was started on intravenous fluid resuscitation with broad-spectrum antibiotics. Which of the following is most likely to be seen in the ECG of this patient?

Choices: 1. ST depression and short QT interval 2. Peaked T wave and short QT interval 3. Low voltage ECG and long QT interval 4. Low voltage ECG and peaked T wave

Answer: 3 - Low voltage ECG and long QT interval Explanations: The patient has myxedema coma with hypotension, bradycardia, and hyponatremia. Given the scar in her neck on physical exam, the most likely precipitating factor is untreated hypothyroidism acquired after thyroidectomy. Complete heart block, bradycardia, flattened T waves, low voltage, and bundle branch blocks are common ECG findings in these patients. Low voltage on EKG can be representative of pericardial effusion due to the accumulation of fluid rich in mucopolysaccharides. Fatal arrhythmias are important to recognize in myxedema coma. The patients can develop QT interval prolongations leading to “Torsades De Pointes” which resolves with treatment of the myxedema. Myxedema causes decreased myocardial contractility and reduced cardiac output which leads to hypotension. Go to the next page if you knew the correct answer, or click the link image(s) below to further research the concepts in this question (if desired).

Research Concepts: Myxedema

We update eBooks quarterly and Apps daily based on user feedback. Please tap flag to report any questions that need improvement.

Question 663: A 40-year-old woman is admitted to the intensive care unit following a highspeed motor vehicle collision, in which she sustained head, chest, and lower limb injuries. She is intubated and placed on a ventilator. On the fourth day of admission, she suddenly desaturates to 80% on FiO2 100%. The endotracheal tube is in position and suctioning is done, but she further deteriorates. Her pulse is 110/min, blood pressure 100/70 mmHg, and oxygen saturation 75% on 100% FiO2. ECG shows sinus tachycardia. She is not receiving low molecular weight heparin. Which of the following is the most appropriate next step?

Choices: 1. Portable chest x-ray 2. D-dimer 3. Bedside echocardiogram 4. CT pulmonary angiogram

Answer: 3 - Bedside echocardiogram Explanations: The most likely diagnosis is pulmonary embolism. Classification of pulmonary thromboembolism (PTE) can be based on physiological grounds; a differentiation between massive, submassive, and non-massive PE has been proposed. Massive PE (5%) can cause hemodynamic instability (as in this patient) and precipitate cardiopulmonary arrest. Echocardiography is helpful in assessing cardiac function at the bedside. In massive PE, thrombus can occasionally be seen proximally in the main pulmonary artery. Echocardiography should be reserved for unstable patients with suspected massive or submassive PE who cannot be safely transported to the radiology department. Computerized tomographic pulmonary angiography (CTPA) is the radiological investigation of choice in stable patients with suspected non-massive PE. ECG, chest x-ray, and D-dimer have insufficient sensitivity or specificity, even when used in association with a clinical assessment to exclude PE. The classic S1Q3T3 pattern on ECG is a nonspecific sign of acute right heart strain and occurs in 20% of patients with PE. Go to the next page if you knew the correct answer, or click the link image(s) below to further research the concepts in this question (if desired).

Research Concepts: Acute Pulmonary Embolism

We update eBooks quarterly and Apps daily based on user feedback. Please tap flag to report any questions that need improvement.

Question 664: A patient suffers a bullet wound to the abdomen and undergoes emergency surgery. After surgery, there is redness around the wound which begins to spread quickly. The area becomes swollen, discolored, and warm to the touch with the superficial appearance of blisters and ulcers. The patient develops tachycardia and is now febrile. Which of the following is the best management choice for this condition?

Choices: 1. It is a superficial soft tissue infection that usually resolves with IV antibiotics 2. It is a deep soft tissue infection, best treated with hyperbaric oxygen therapy and IV antibiotics 3. It is a deep soft tissue infection best treated by local surgical exploration, debridement, and IV antibiotics 4. It is best treated by wide surgical excision and debridement along with broad-spectrum antibiotics

Answer: 4 - It is best treated by wide surgical excision and debridement along with broadspectrum antibiotics

Explanations: This disorder is sometimes referred to as wound gangrene, Meleney ulcer, suppurative fasciitis, or hospital gangrene, but it is most widely known as necrotizing fasciitis or Fournier's gangrene. Necrotizing fasciitis can occur after trauma or around foreign bodies in surgical wounds. The point of bacterial inoculation may be relatively small and may be relatively distant from the affected area. Necrotizing fasciitis is a dangerous and rapidly spreading, inflammatory, infectious condition that is often located in the deep fascia, with secondary involvement of the subcutaneous tissues. Subcutaneous air may be seen on x-rays. To reduce mortality and morbidity, the condition must be emergently treated with aggressive surgical debridement, hemodynamic support, and broad-spectrum antibiotics. Repeated surgeries to remove additional necrotic material may be necessary. Go to the next page if you knew the correct answer, or click the link image(s) below to further research the concepts in this question (if desired).

Research Concepts: Necrotizing Fasciitis

We update eBooks quarterly and Apps daily based on user feedback. Please tap flag to report any questions that need improvement.

Question 665: A 60-year-old man is brought to the hospital by emergency medical services after choking at a restaurant. His wife indicates that he was a chronic smoker, has a desk job, and does not exercise routinely. The neurological examination shows dysphagia, Horner's syndrome, loss of pain and temperature for the left side of the face, nystagmus, and a loss of the left corneal reflex. What is the most likely diagnosis?

Choices: 1. Inferior medial pontine syndrome 2. Lateral medullary syndrome 3. Medial medullary syndrome 4. Lateral pontine syndrome

Answer: 2 - Lateral medullary syndrome Explanations: Sudden onset in symptoms is typical for ischemic events. An ischemic stroke is supported by the patient's hypertension, overweight status, and sedentary lifestyle. This can be confirmed with a non-contrast CT. Non-contrast is utilized as a first imaging modality to ensure that the brain is not subjected to the contrast agent if hemorrhage is observed. All of the symptoms can originate from damage to the lateral medulla, hence the diagnosis of lateral medullary syndrome (Wallenberg Syndrome). Dysphagia and hoarseness can occur due to damage to the nucleus ambiguus in the medulla. Loss of pain and temperature occurs with damage to the spinal trigeminal nucleus. Damage to the inferior vestibular nucleus will cause nystagmus. Damage to the sympathetic fibers will cause Horner's syndrome. The facial nerve is the motor output for the corneal reflex. The facial nerve exits the brainstem at the pontomedullary junction. Damage to the posterior inferior cerebellar artery can affect the nerve at this point, causing a loss of the corneal reflex. Go to the next page if you knew the correct answer, or click the link image(s) below to further research the concepts in this question (if desired).

Research Concepts: Corneal Reflex

We update eBooks quarterly and Apps daily based on user feedback. Please tap flag to report any questions that need improvement.

Question 666: A 16-year-old patient was rushed to the emergency department following a motor vehicle collision. The patient was complaining of a moderate headache. His Glasgow coma scale was 15/15. The patient had no sensory or motor deficits. His brain computed tomographic (CT) scan revealed a punctate anterior left temporal contusion of 1 cm x 1.5 cm. The patient was admitted to the intensive care unit for observation and was managed symptomatically. His blood panel studies at admission were within the normal ranges. His repeat brain CT showed significant progression in the size of the contusion to 3.5 cm x 2.4 cm. He had mild expressive aphasia as well. What is the most likely explanation for the adverse event seen in the patient?

Choices: 1. Vitamin K deficiency 2. Idiopathic thrombocytopenic purpura 3. Protein C and S deficiency 4. Release of tissue factor

Answer: 4 - Release of tissue factor Explanations: The contused brain is one of the major sources of the release of the tissue factor, a component of the coagulation cascade, following traumatic brain injury. The release of the tissue factor leads to the activation of the coagulation cascade, thereby predisposing the risk of consumptive coagulopathy. Trauma-induced coagulopathy is one of the significant factors determining prognostic outcome in patients with a head injury. Its incidence has been reported as high as 97% of the patients with traumatic brain injury. The patient with protein C and S deficiency will have a history of bleeding diathesis in the past. Go to the next page if you knew the correct answer, or click the link image(s) below to further research the concepts in this question (if desired).

Research Concepts: Cerebral Contusion

We update eBooks quarterly and Apps daily based on user feedback. Please tap flag to report any questions that need improvement.

Question 667: A 65-year-old man with a history of hypertension, tobacco use, and stage III chronic kidney disease suffers an out-of-hospital cardiac arrest. He is intubated at the scene and transported to the nearest emergency department. After being successfully resuscitated, he is admitted to the intensive care unit for therapeutic hypothermia. His core temperature is maintained at 33 C (91.4 F) for 24 hours, after which he undergoes passive rewarming. His temperature increases from 33 C (91.4 F) to 36 C (96.8 F) within 2 hours. Which of the following clinical findings is most likely to be seen in this patient?

Choices: 1. Triphasic waves on the EEG 2. Peaked T-waves on ECG 3. Elevated peak airway pressures 4. Hyperbilirubinemia on morning labs

Answer: 2 - Peaked T-waves on ECG Explanations: During therapeutic hypothermia, potassium shifts intracellularly. During the rewarming phase, this process is reversed. This can result in hyperkalemia, particularly if the rewarming process is accelerated. Hyperkalemia could be consistent with the EKG changes described. In patients undergoing therapeutic hypothermia, the rewarming phase is critical. Patients can experience increased cerebral edema, shivering, and electrolyte abnormalities, especially when rewarming occurs too quickly. In this example, the patient's body temperature increased by 3 degrees over a 2 hour period, which considerably exceeds the 0.25-0.5 degree/hour rate, which is recommended during rewarming. Hyperkalemia typically produces peaked T waves early in the course, which is especially notable in the precordial leads. If left untreated, this can progress to PR prolongation and widening of the QRS complex. Fatal arrhythmias can result. EEG can be very useful for evaluating neurological function in the post-arrest patient. Triphasic waves are a common finding in patients with metabolic encephalopathies, including hepatic encephalopathy. They are not characteristic of patients with anoxic brain injury. Go to the next page if you knew the correct answer, or click the link image(s) below to further research the concepts in this question (if desired).

Research Concepts: Anoxic Encephalopathy

We update eBooks quarterly and Apps daily based on user feedback. Please tap flag to report any questions that need improvement.

Question 668: A 56-year-old male was admitted with acute hemorrhagic stroke. His past medical history is significant for hypertension, diabetes mellitus, atrial fibrillation, and a history of anaphylaxis to penicillin. The patient is found to have significant dysphagia, and the decision to place a feeding tube is made. Which of the following should be administered prior to the procedure?

Choices: 1. Clindamycin 900 mg 3 hours prior to the procedure 2. Clindamycin 900 mg 1 hour prior to the procedure 3. Cefazolin 2 gm 3 hours prior to the procedure 4. Cefazolin 2 gm 1 hour prior to the procedure

Answer: 2 - Clindamycin 900 mg 1 hour prior to the procedure Explanations: Antibiotic prophylaxis is required prior to the placement of a percutaneous gastrostomy tube. Given the patient's severe allergy to penicillin, clindamycin is preferred. Antibiotics should be administered 1 hour prior to the procedure. Antibiotics help prevent the development of postprocedural infections. Given the patient's severe penicillin allergy, cefazolin is not recommended. Most patients with minor reactions to penicillin tolerate cefazolin. Go to the next page if you knew the correct answer, or click the link image(s) below to further research the concepts in this question (if desired).

Research Concepts: Percutaneous Gastrostomy And Jejunostomy

We update eBooks quarterly and Apps daily based on user feedback. Please tap flag to report any questions that need improvement.

Question 669: A 26-year-old man who underwent surgery for a craniopharyngioma a week ago presents to the clinic with symptoms of increased thirst and urination. He reports going to the bathroom 8 times last night and an increase in the urine volume. Laboratory evaluation shows a serum sodium level of 155 mEq/L and a urine osmolarity of 125 mOsm/L. Which of the following is the next best step in the management of this patient?

Choices: 1. IV Ringer's lactate 2. Vasopressin nasal spray 3. Repeat surgery 4. Low sodium diet

Answer: 2 - Vasopressin nasal spray Explanations: The patient in this scenario has developed diabetes insipidus, which is a common complication after pituitary surgery in patients with craniopharyngioma. The signs of diabetes insipidus (DI) are increase thirst and urination. Biochemically patients will have a low urine osmolality and an elevated plasma sodium level. DI following surgery is usually temporary, but in patients with craniopharyngioma, DI can be permanent. Vasopressin supplementation is indicated to normalize sodium and relieve the patient's symptoms. Go to the next page if you knew the correct answer, or click the link image(s) below to further research the concepts in this question (if desired).

Research Concepts: Craniopharyngioma

We update eBooks quarterly and Apps daily based on user feedback. Please tap flag to report any questions that need improvement.

Question 670: A young male with tetanus is in the intensive care unit on a ventilator because of difficulty with breathing. While on the ventilator he continues to develop generalized spasms and trismus. He already is on a midazolam drip, but the spasms persist. What other agent may be of benefit in this patient?

Choices: 1. Magnesium 2. Calcium 3. Gabapentin 4. Ethylenediaminetetraacetic acid

Answer: 1 - Magnesium Explanations: Magnesium infusion often is used to break up tetanus spasms in patients who fail to respond to benzodiazepines. Magnesium also helps stabilize autonomic instability. A drip of 2 to 3 grams, intravenously administered, every 12 hours may help. It is important to monitor magnesium levels to avoid hypermagnesemia. Go to the next page if you knew the correct answer, or click the link image(s) below to further research the concepts in this question (if desired).

Research Concepts: Tetanus

We update eBooks quarterly and Apps daily based on user feedback. Please tap flag to report any questions that need improvement.

Question 671: A 65-year-old male with a history of hypertension, diabetes mellitus, and congestive heart failure is hospitalized for pneumonia and a day later develops acute dyspnea and chest pain. He is tachycardic and tachypneic. His lungs are clear to auscultation and there is no peripheral edema. Which diagnostic test should be ordered?

Choices: 1. CT pulmonary angiogram 2. D-dimer level 3. Echocardiogram 4. Ventilation/perfusion scan

Answer: 1 - CT pulmonary angiogram Explanations: CT pulmonary angiogram should be obtained to diagnose pulmonary embolus. V/Q scans are more challenging to interpret in patients with pneumonia, give probabilistic rather than definitive answers, and if negative provide no alternative diagnoses. They are now largely restricted to those who have a contraindication to CT pulmonary angiography. D-dimer levels may be elevated in this patient without pulmonary embolus. CT pulmonary angiography is recommended as the procedure of choice to diagnose or exclude acute pulmonary embolism even in pregnant women. Go to the next page if you knew the correct answer, or click the link image(s) below to further research the concepts in this question (if desired).

Research Concepts: Acute Pulmonary Embolism

We update eBooks quarterly and Apps daily based on user feedback. Please tap flag to report any questions that need improvement.

Question 672: A 66-year-old male is brought to the emergency department with complaints of difficulty breathing. He also complains of increased sputum production and change in the color of the sputum. The patient has a past medical history of COPD, chronic heart failure, diabetes mellitus, sleep-disordered breathing, smoking, and alcohol use disorder. Physical examination is notable for cyanosis, crackles in the right middle lung fields, bilateral wheezing, and peripheral edema (2+) of the extremities. Vitals reveal a temperature of 99.8 F, respiratory rate of 22/min, blood pressure of 90/60 mmHg, and pulse rate of 108/min. Which of the following is a contraindication to the use of non-invasive ventilation with average volumeassured pressure support modality in this patient?

Choices: 1. Sleep-disordered breathing 2. Acute exacerbation of COPD 3. Hemodynamic compromise 4. History of smoking

Answer: 3 - Hemodynamic compromise Explanations: Average volume-assured pressure support (AVAPS) is a relatively newer modality of noninvasive ventilation that integrates the characteristics of both volume and pressurecontrolled non-invasive ventilation. AVAPS is contraindicated in patients with cardiac arrest/hemodynamic compromise/unstable arrhythmia. AVAPS can cause significant hypotension in patients with cardiac dysfunction due to increased intrathoracic pressure and decreased preload. AVAPS has been successful in the treatment of COPD-associated acute hypercapnic respiratory failure. Significant improvement was seen in arterial blood gases, pH, respiratory rate, and the Glasgow Coma Scale. Nonetheless, it is important to note that the patients with a higher APACHE II score, calculated during ICU admission, had a higher risk of treatment failure. Go to the next page if you knew the correct answer, or click the link image(s) below to further research the concepts in this question (if desired).

Research Concepts: Average Volume-Assured Pressure Support

We update eBooks quarterly and Apps daily based on user feedback. Please tap flag to report any questions that need improvement.

Question 673: A 50-year-old male with past medical history significant for hypertension, diabetes mellitus type 2, and colon cancer resected 2 months ago presents to the emergency department with complaints of right upper quadrant pain, nausea, vomiting, fatigue, malaise, and yellow discoloration of the skin for one week. The patient appears lethargic and is slurring his speech. Physical examination shows yellow sclera, yellow skin, and right upper quadrant tenderness without peritoneal or Murphy's sign. He is also found to have a flapping tremor. Laboratory work showed marked elevation of liver enzymes and bilirubin, elevated international normalized ratio (INR) of 2.3, and thrombocytopenia. Computed tomography scan of the abdomen and pelvis with intravenous contrast shows hepatic congestion, minimal flow in the hepatic veins, a collapsed inferior vena cava, and splenomegaly, without any other liver parenchymal, renal, pancreatic, or intestinal abnormality. Apart from definitive treatment, what additional steps should be taken?

Choices: 1. Magnetic resonance imaging of the liver 2. Ursodeoxycholic acid 3. Emergent laparotomy 4. Evaluation for a liver transplant

Answer: 4 - Evaluation for a liver transplant Explanations: This patient has acute liver failure secondary to Budd-Chiari syndrome. Acute onset of symptoms and a history of colon cancer, along with results of the imaging studies, help establish the diagnosis. A patient with Budd-Chiari syndrome who develops acute liver failure should be referred for evaluation for a liver transplant. Magnetic resonance imaging of the liver is unlikely to provide any further information. Ursodeoxycholic acid has no role in the treatment of acute liver failure or Budd-Chiari syndrome. Go to the next page if you knew the correct answer, or click the link image(s) below to further research the concepts in this question (if desired).

Research Concepts: Budd Chiari Syndrome

We update eBooks quarterly and Apps daily based on user feedback. Please tap flag to report any questions that need improvement.

Question 674: A 57-year-old female patient presents to the emergency department with high-grade fever, arthralgias, and an itchy skin rash for three days. She has also experienced chest pain for the past 1 day, worse with deep breathing. She has a history of rheumatoid arthritis diagnosed one year ago, which has been under good control with adalimumab (4 months) and methotrexate (1 year). Other significant past medical history includes a history of hypertension for which she is on hydrochlorothiazide. On examination, she has a high-grade fever, heart rate is 100/minute, blood pressure is 134/88 mmHg. She appears in distress because of pain. The skin exam reveals diffuse maculopapular rash on her lower extremities. The chest exam reveals bibasal decreased breath sounds. The musculoskeletal exam is significant for tenderness and swelling with effusion in bilateral knees, and few proximal interphalangeal joints in the hands. Laboratory evaluation reveals complete blood count with hemoglobin 10.0 mg/dL, platelet count 105,000, and WBC 3000. The erythrocyte sedimentation rate is 78 mm/hr. The chest Xray reveals bilateral pleural effusions. Further serological workup is pursued, which reveals a positive anti-nuclear antibody, anti-dsDNA antibody, rheumatoid factor, and anti-CCP antibody. The rest of the autoimmune profile is normal. Serological work-up one year ago at the time of her rheumatoid arthritis diagnosis was significant for positive anti-CCP and rheumatoid factor, and a negative anti-nuclear antibody. Which of the following is the most likely diagnosis?

Choices: 1. New-onset systemic lupus erythematous 2. Drug-induced lupus secondary to hydrochlorothiazide 3. Drug-induced lupus secondary to adalimumab 4. New extra-articular manifestation of rheumatoid arthritis

Answer: 3 - Drug-induced lupus secondary to adalimumab Explanations: New-onset fever, arthralgia, rash, serositis, and pancytopenia in a patient who was recently started on an anti-TNF agent should raise a strong suspicion of drug-induced lupus. Anti-TNF agents can cause positive ANA and positive anti-dsDNA antibodies. Anti-histone antibodies may not always be present. Management is the withdrawal of the anti-TNF agent (and avoiding all anti-TNF agents in the future). NSAIDs or a short course of corticosteroids may be considered if symptoms do not resolve within a few weeks of discontinuing the anti-TNF agent. It may take several months for the serologies to turn negative. Hydrochlorothiazide induced lupus is usually associated with a positive anti-histone antibody and not an anti-dsDNA antibody. New-onset lupus is possible, but the timeline and typical serologies/symptoms are more characteristic of anti-TNF induced lupus. While rash, arthralgias, serositis, and pancytopenia can be seen as extra-articular manifestations of rheumatoid arthritis, fevers are unusual secondary to rheumatoid arthritis and the serological workup now is consistent with new-onset drug-induced lupus secondary to an anti-TNF agent in this patient with rheumatoid arthritis. Go to the next page if you knew the correct answer, or click the link image(s) below to further research the concepts in this question (if desired).

Research Concepts: Tumor Necrosis Factor Inhibitors

We update eBooks quarterly and Apps daily based on user feedback. Please tap flag to report any questions that need improvement.

Question 675: An 85-year-old male with a past medical history of advanced Alzheimer dementia and hypertension is admitted to the intensive care unit with acute hypoxic respiratory failure and septic shock. He is intubated and placed on mechanical ventilation. Broad-spectrum IV antibiotics are initiated. A chest radiograph reveals a large right pleural effusion. Follow up computed tomography of the chest shows a dense right lower lobe consolidation with air bronchograms and a large effusion. Thoracentesis is consistent with the exudative neutrophilic type of effusion. The patient undergoes tube thoracostomy. Due to poor drainage by hospitalization day 3, a repeat CT scan reveals loculation in the pleural space. Which of the following is the next best step in the management of this patient?

Choices: 1. Tissue plasminogen activator (TPA) alone 2. TPA + deoxyribonuclease 3. Normal saline irrigation 4. No further intervention

Answer: 2 - TPA + deoxyribonuclease Explanations: The use of adjunctive intrapleural medications can be beneficial in patients with inadequate chest tube drainage. The MIST-2 Trial showed combination therapy of fibrinolytic and mucolytics (vs. double placebo, fibrinolytic + placebo, or mucolytics + placebo) improved fluid drainage in patients with pleural infection, reduced the frequency of surgical referral, and the duration of the hospital stay. Of note, there was no proven mortality benefit noted with this intervention. Adverse effects between all groups were not significant. Go to the next page if you knew the correct answer, or click the link image(s) below to further research the concepts in this question (if desired).

Research Concepts: Thoracic Empyema

We update eBooks quarterly and Apps daily based on user feedback. Please tap flag to report any questions that need improvement.

Question 676: A 26-year-old woman presents to the clinic for discussion regarding her HIV care. Two weeks ago, she had an emergency department visit for shortness of breath, cough, and an unintentional weight loss of 10 lbs in the preceding 1 month. During the encounter, a CT chest demonstrated right upper lobe cavitary disease and hila lymph node enlargement. Sputum samples were obtained during the visit. She was then prescribed amoxicillin-clavulanic acid and doxycycline for 5 days and asked to follow up as an outpatient. Currently, she continues to complain of nonproductive cough, lack of appetite, low-grade fever, and chills. Her sputum AFB stain reports show Mycobacterium tuberculosis (TB). She is an immigrant from Nigeria and contracted HIV working as a sex worker but has never taken HAART (highly active antiretroviral therapy). Her CD4 count was 34/microL during the ED visit. Which of the following management strategies is most likely to minimize the risk of TB-related immune reconstitution inflammatory syndrome (IRIS) in this patient?

Choices: 1. Start antitubercular regimen now, then prednisone, and then HAART 4 weeks later 2. Start antitubercular regimen now, then HAART after completion of TB treatment 3. Start antitubercular therapy and HAART now 4. Start HAART only, defer antitubercular regimen till CD4 counts improved to >100 /microL

Answer: 1 - Start antitubercular regimen now, then prednisone, and then HAART 4 weeks later

Explanations: The patient has a simultaneous diagnosis of HIV and TB. It is important to treat both. However, the risk of IRIS is significantly high, with TB in HIV patients compared to nonHIV patients. In patients with CD4 counts of 50/microL, early initiation of HAART (i.e., 1-4 weeks) after initiation of anti-TB treatment has been found to have improved mortality outcomes with an increased risk of nonfatal TB-IRIS. Due to this mortality benefit, early initiation of HAART is recommended along with the use of prophylactic steroids to prevent IRIS symptoms. Waiting for the antitubercular regimen to be completed before starting HAART would delay antiretroviral therapy by at least 6 months and is not recommended. Starting antitubercular medication and HAART at the same time is also not recommended due to the risk of TBIRIS. HAART should be initiated ideally after 8-12 weeks of TB medications (delayed ART initiation) or after 1-4 weeks (early ART initiation) if they have CD4 counts 50/microL. Starting HAART only and not antitubercular medication is not the correct response as it does not treat the opportunistic infections associated with HIV. Treatment of latent or active infections in HIV should be done either prior to or during HAART therapy not after. Additionally, it is not recommended to wait till we see CD4 count improvement to initiate treatment against opportunistic infections. Go to the next page if you knew the correct answer, or click the link image(s) below to further research the concepts in this question (if desired).

Research Concepts: Immune Reconstitution Inflammatory Syndrome

We update eBooks quarterly and Apps daily based on user feedback. Please tap flag to report any questions that need improvement.

Question 677: A 30-year-old woman with a past medical history of anxiety and alcohol use disorder is brought to the emergency department by ambulance after a motor vehicle accident. The patient is unconscious and breathing at seven breaths per minute. The airway is secured, and fluid resuscitation is initiated. The patient is given chlordiazepoxide for possible alcohol withdrawal. After a few hours, her husband arrives and informs the medical team that she has recently been started on chlordiazepoxide by her primary care team. She has therefore received an accidental overdose of this medication. What is the mechanism of action of the most appropriate drug to reverse this?

Choices: 1. Competitive inhibition of the alpha-beta subunit of GABA binding site 2. Competitive antagonist of beta-1-adrenergic receptors 3. Carbonic anhydrase inhibitor 4. Inhibition of conversion of angiotensin I to angiotensin II

Answer: 1 - Competitive inhibition of the alpha-beta subunit of GABA binding site Explanations: Flumazenil acts by competitive inhibition of GABA receptors. It is sometimes used for a partial or complete reversal of the sedative property of benzodiazepines, such as chlordiazepoxide. After the administration of flumazenil, clinicians should monitor the patient for the risk of seizure associated with this medication, especially in patients with chronic benzodiazepine use. In the case of chlordiazepoxide toxicity or overdose, the patient may present with confusion, diminished reflexes, comatose, or somnolent. Competitive antagonist of beta-1-adrenergic receptors is the mechanism of action of betablockers such as propranolol. Inhibition of conversion of angiotensin I to angiotensin is the mechanism of action of ACE inhibitors, such as captopril, which reduces blood pressure. Therefore, it is not helpful in this patient. Go to the next page if you knew the correct answer, or click the link image(s) below to further research the concepts in this question (if desired).

Research Concepts: Chlordiazepoxide

We update eBooks quarterly and Apps daily based on user feedback. Please tap flag to report any questions that need improvement.

Question 678: A patient in the intensive care unit has a mixed venous oxygen saturation of 55%. His hemodynamics reveal a cardiac index of less than 1.7, a blood pressure of 100/64 mm Hg, and a heart rate of 110 beats per minute. His Pa02 is 90 on an FIO2 of 50%. His hematocrit is 31%. Which of the following is the most appropriate next step in this patient's management?

Choices: 1. Blood transfusion 2. Increase FIO2 3. Inotropic support 4. Vasopressor administration

Answer: 3 - Inotropic support Explanations: It is important to know that mixed venous oxygen is affected by anemia, oxygen, cardiac output, and metabolic activity. In the above case, the cardiac index is low. Therefore, inotropic support is required. It is helpful to distinguish low cardiac index from failure versus vasodilation seen in distributive or septic shock. Other useful modalities to recognize acute heart failure, including pulmonary edema or enlarged cardiac silhouette on x-ray, decreased ejection fraction on ultrasound. Go to the next page if you knew the correct answer, or click the link image(s) below to further research the concepts in this question (if desired).

Research Concepts: Cardiogenic Shock

We update eBooks quarterly and Apps daily based on user feedback. Please tap flag to report any questions that need improvement.

Question 679: A 28-year-old man is brought to the hospital by his family for altered mental status. They report a history of depression with psychotic features and recent suicidal thoughts. He was found with an empty bottle of risperidone, which appears to have been filled less than a week prior. On examination, he is altered and ill-appearing. His vitals are notable for tachycardia, tachypnea, and a temperature of 40 C. The presence of which of the following features best distinguishes this disorder from serotonin syndrome?

Choices: 1. Anhidrotic hyperthermia 2. Clonus 3. Rigidity 4. Hyperpnea

Answer: 3 - Rigidity Explanations: Neuroleptic malignant syndrome (NMS) is a syndrome of altered mental status, autonomic instability, hyperthermia, and “lead pipe” rigidity associated with the use of certain medications. NMS is most commonly secondary to dopaminergic antagonists but, more rarely, can occur due to withdrawal from dopaminergic medications. Haloperidol and fluphenazine, both first-generation neuroleptic agents, are the most commonly associated with NMS. However, NMS can also occur with second-generation or "atypical" neuroleptic agents such as risperidone. Risk factors for the development of NMS include high doses of neuroleptic medications, recent or rapid escalation in dosing, a switch from one agent to another, and parenteral administration of the agent causing the syndrome. The treatment of neuroleptic malignant syndrome is largely supportive and includes maintenance of euvolemia and normovolemia as well as prevention of complications, which include deep venous thrombosis, rhabdomyolysis, renal failure, and cardiac dysrhythmias. In cases of severe muscle rigidity not improved with supportive measures, dantrolene and bromocriptine can be used. The distinguishing feature of neuroleptic malignant syndrome is the classic "lead pipe" rigidity. This can help distinguish NMS from other conditions that present with altered mental status, including sepsis, serotonin syndrome, sympathomimetic toxicity, anticholinergic syndrome, salicylate overdose, and thyrotoxicosis. Go to the next page if you knew the correct answer, or click the link image(s) below to further research the concepts in this question (if desired).

Research Concepts: Neuroleptic Agent Toxicity

We update eBooks quarterly and Apps daily based on user feedback. Please tap flag to report any questions that need improvement.

Question 680: A 26-year-old man presents with painful bullous skin lesions and dyspnea after exposure to a chemical warfare agent. On examination, there are severe ocular burns, redness, and bullae filled with yellow fluid on the face, axilla, and groin. There is pulmonary edema. Other victims recall the smell of horseradish just before developing symptoms. What metabolite can be detected in the urine up to 2 weeks after exposure to this agent?

Choices: 1. Glutathione 2. Thiodiglycol 3. Soman 4. Phosphite

Answer: 2 - Thiodiglycol Explanations: Patients that have been exposed to sulfur mustard usually describe the smell of onions or garlic at the scene of exposure or they have a recent interaction with old artillery shells. Thiodiglycol is a metabolite of sulfur mustard that can be detected in the urine. Thiodiglycol has low utility in clinical settings due to delay in obtaining results and it is often rapidly enzymatically transformed. Glutathione is depleted in significant sulfur mustard systemic exposure, resulting in a decreased ability of the cells to protect against oxidative stress. It is not excreted in the urine. Go to the next page if you knew the correct answer, or click the link image(s) below to further research the concepts in this question (if desired).

Research Concepts: Blister Agents

We update eBooks quarterly and Apps daily based on user feedback. Please tap flag to report any questions that need improvement.

Question 681: A 23-year-old college student is admitted to the ICU for fever, headache, vomiting, and neck stiffness. Examination shows blood pressure 110/70 mmHg, a pulse of 120 beats per minute, a respiratory rate of 22/min, and a temperature of 101 degrees Fahrenheit. There is a diffuse purpuric eruption on his trunk and extremities. He is diagnosed as a case of meningococcal meningococcemia, and appropriate therapy is instituted. On the third postadmission day, he develops new abdominal pain and hypotension. His investigations show a sodium level of 130 mmol/l, a potassium level of 6.1 mmol/l, and a bicarbonate level of 15 mEq/l. Which of the following describes the most likely finding to be seen on his abdominal CT scan?

Choices: 1. Hyperdense adrenal enlargement without contrast enhancement 2. Hypodense adrenals with calcifications 3. Unilateral adrenal enlargement with a fatty component 4. Bilateral adrenal masses with contrast enhancement

Answer: 1 - Hyperdense adrenal enlargement without contrast enhancement Explanations: This patient has been diagnosed with disseminated meningococcal infection. A gramnegative diplococcus is an important cause of meningitis, especially in individuals living in dormitories. Neisseria meningitides can cause disseminated infection and can present with a purpuric rash. An important complication that occurs is bilateral adrenal hemorrhage, termed Waterhouse Frederichson syndrome. Waterhouse Frederichson syndrome can present with diffuse abdominal pain and the development of hypotension in a patient with meningococcemia. Other signs of adrenal insufficiency may be present. Investigations may reveal a fall of hemoglobin/hematocrit and the classic hyponatremia, hyperkalemia, and hypoglycemia associated with adrenal insufficiency. Low serum cortisol and elevated ACTH further aids in the diagnosis. A CT scan is important in the diagnosis of adrenal hemorrhage. It shows heterogeneous round to ovoid lesions without enhancement. There are often peri-adrenal inflammatory changes and possibly fat stranding. The adrenals appear hyperdense but without any contrast enhancement. The hematoma may also extend into the perinephric space. Over time, the hematoma appears less dense on subsequent CT scans and may develop calcifications. This explains the CT findings of reduced density of the adrenals and calcifications, which are seen in chronic adrenal hematomas. Overall, adrenal hemorrhage is associated with a 15% mortality rate, which varies according to the severity of the bleed. This increases to a 55 to 60% mortality rate in patients with Waterhouse-Friderichsen syndrome (adrenal hemorrhage can be due to meningococcal sepsis). Treatment would be dependent on the severity, but in general, corticosteroid replacement should be used for adrenal insufficiency. Adrenal enlargement with a significant fatty component is seen in angiomyolipomas. Adrenal lesions with contrast enhancement are seen in cases of neoplastic deposits. Go to the next page if you knew the correct answer, or click the link image(s) below to further research the concepts in this question (if desired).

Research Concepts: Adrenal Hemorrhage

We update eBooks quarterly and Apps daily based on user feedback. Please tap flag to report any questions that need improvement.

Question 682: A 16-year-old male born in Central America patient presents to the hospital due to a new-onset seizure. He has never had a seizure before. The patient's brother states that the patient almost exclusively eats pork. Based on history and symptoms, an MRI is ordered that reveals the presence of larvae. He is commenced on dexamethasone and another specific drug for the underlying diagnosis. What is the mechanism of action of the drug in the discussion?

Choices: 1. Inhibits parasitic actin filament formation 2. Disrupts parasitic microtubule polymerization 3. Generates reactive oxygen species to damage parasite DNA 4. Decreases production of inflammatory mediators and enzymes

Answer: 2 - Disrupts parasitic microtubule polymerization Explanations: This is a case of neurocysticercosis, and the drug in the discussion is albendazole. The mechanism of action of albendazole involves the binding of its active metabolite to the btubulin subunit of microtubules. This stops the polymerization of microtubules within the parasite and stops its motility, leading to its death. Neurocysticercosis is more common in patients who eat pork products, especially undercooked or raw pork products. A new unprovoked seizure in a Latino patient should almost always have neurocysticercosis ruled out due to the high incidence of pork consumption in this population. Albendazole, steroids, and anticonvulsants are usually the mainstay of treatment. Albendazole inhibits microtubular function and can also inhibit parasitic glucose utilization, which depletes its ATP and also can lead to its death. Steroids decrease inflammation by decreasing the inflammatory response. No current drug inhibits parasitic actin filament formation, and albendazole does not regenerate reactive oxygen species to damage DNA. Go to the next page if you knew the correct answer, or click the link image(s) below to further research the concepts in this question (if desired).

Research Concepts: Albendazole

We update eBooks quarterly and Apps daily based on user feedback. Please tap flag to report any questions that need improvement.

Question 683: A 35-year-old man is brought to the emergency department with complaints of headache, blurring of vision, and weakness of the right side of his body. He has been experiencing daily headaches for the past 6 days. The headaches are worse in the morning and are aggravated by coughing and bending forwards. He complains that his right arm and leg are numb and weak. He has also been experiencing intermittent fevers for the past 2 weeks. He was diagnosed with HIV 10 years ago but did not seek treatment. His examination reveals a blood pressure of 130/70 mmHg, pulse of 110 bpm, a temperature of 100 F (37.7 C), and a respiratory rate of 22/min. His neurological examination reveals a weakness of 3/5 in the right upper and lower arm with brisk reflexes. All sensations are diminished on the right side. Investigations reveal a WBC of 3,000/microL, hemoglobin 10 gm/dl, platelets 120,000/microL, CD4+ count of 10/microL, serum ALT 65 IU/l, serum AST 40 IU/l, serum LDH 700 IU/l, CRP 75 mg/l, and serum creatinine 1.0 mg/dl. CSF analysis reveals a cell count of 75 cells/microL, a protein of 110 mg/dl and glucose of 45 mg/dl. A Giemsa stain reveals the presence of crescent-shaped organisms with blue cytoplasm and red staining nucleus. What is the appropriate management of this patient?

Choices: 1. Amphotericin B 2. Pyrimethamine/sulfadiazine 3. Acyclovir 4. Ampicillin

Answer: 2 - Pyrimethamine/sulfadiazine Explanations: This individual is an HIV positive patient with advanced immunosuppression, CD4+ count less than 50 cells/micro. He has presented with signs of raised intracranial pressure and focal neurology. CSF examination confirms the presence of tachyzoites of Toxoplasma gondii. Cerebral toxoplasmosis is seen in advanced HIV infection with reactivation of latent Toxoplasma infection or rarely because of primary infection. The management is with drugs that demonstrate ready penetration into the CSF. Pyrimethamine and sulfadiazine are the treatment of choice due to the ability to cross the blood-brain barrier. The usual dose of pyrimethamine is 50-75 mg daily, and sulfadiazine is 1000-1500 mg daily for 4-6 weeks. The addition of leucovorin prevents the development of hematological side effects associated with pyrimethamine use. Adjunctive glucocorticoids may be added to reduce inflammatory reaction. Treatment duration is 4-6 weeks. Antiretroviral therapy must be instituted to improve CD4+ count. Amphotericin B is used in cases of fungal CNS infections, and acyclovir is used for viral encephalitis. Isolation of listeria would warrant the addition of ampicillin. Go to the next page if you knew the correct answer, or click the link image(s) below to further research the concepts in this question (if desired).

Research Concepts: HIV-1 Associated Toxoplasmosis

We update eBooks quarterly and Apps daily based on user feedback. Please tap flag to report any questions that need improvement.

Question 684: A 90-year-old gentleman is referred for medical decision-making capacity to consent to a percutaneous endoscopic gastrostomy (PEG) tube for feeding following a stroke. Which of the following statements is correct regarding the evaluation of this patient's capacity to consent to this procedure?

Choices: 1. A cognitive screen, such as the mini mental state examination (MMSE), will suffice 2. A full neuropsychological evaluation is always required 3. The individual is assumed to have capacity unless proven otherwise 4. The individual’s ability to write a will is also determined at the same time as the capacity assesment

Answer: 3 - The individual is assumed to have capacity unless proven otherwise Explanations: Although cognitive impairment and psychiatric disturbances can impair one’s capacity to make decisions, people are presumed to have capacity unless this comes into question. A capacity assessment always involves an interview with the patient and any collaterals, as well as an assessment of functional abilities. A screen would not be sufficient. A full neuropsychological assessment is not always required, although some evaluation of cognition is usually performed. One type of capacity does not equal another type. Medical decision-making does not apply to writing a will, which is civil capacity. Go to the next page if you knew the correct answer, or click the link image(s) below to further research the concepts in this question (if desired).

Research Concepts: Neuropsychological Assessment

We update eBooks quarterly and Apps daily based on user feedback. Please tap flag to report any questions that need improvement.

Question 685: A 43-year-old man with a past medical history of hypertension recently returned from Cuba. He presented to the emergency department with abdominal pain that is localized to his right lower quadrant with associated watery diarrhea. He states that he has fevers intermittently that have been controlled with acetaminophen. The patient states that these symptoms began less than 12 hours ago. The patient has a past surgical history of cholecystectomy. What is the best initial step in the management of this patient?

Choices: 1. CT abdomen 2. Blood cultures 3. Administer gentamicin and doxycycline 4. IV fluids, bowel rest, and fever control

Answer: 4 - IV fluids, bowel rest, and fever control Explanations: The question stem highlights important aspects of the diagnosis of pseudoappendicitis secondary to Yersinia enterocolitica infection. First, the stem highlights in the patient's history, travel abroad to Cuba, which may expose the patient to the consumption of meat from pigs and cattle, which may be infected with Y. enterocolitica. Secondly, most patients infected with Y. enterocolitica never develop symptoms, but advanced signs of infection include symptoms such as diarrhea, fever, chills. The patient presents with signs of symptomatic pseudoappendicitis likely secondary to Y. enterocolitica. The patient's right lower quadrant pain is also likely secondary to mesenteric adenitis due to bacterial collection in the terminal ileum. As most cases of Y. enterocolitica are self-limiting, presentation of advanced infection and symptoms warrants treatment. The question stem asks what the initial best step in management is. Elucidating and confirming the patient's diagnosis with other lab tests and imaging can wait, but symptomatic management takes priority. Therefore, the best initial step in management is controlling symptoms with IV fluids, bowel rest, and fever control. The first step in management is controlling symptoms. After the patient's symptoms are controlled, then can other lab tests and imaging modalities be used to make the diagnosis, control of symptoms should not be delayed to make a definitive diagnosis. Obtaining blood cultures would be important if the patient demonstrated signs and symptoms of septic shock. However, the question stem does not indicate any signs of shock. The patient does demonstrate symptoms of fever, diarrhea, and abdominal pain that require management and control. Therefore the first step in management is control of symptoms. Pseudoappendicits secondary to Y. enterocolitica infection is often a self-limiting condition, and antibiotics are rarely required. However, in cases that do not respond to symptomatic therapy, and especially in patients who are immunocompromised, antibiotics should be initiated, which include doxycycline and an aminoglycoside such as gentamicin. The question stem does not indicate an immunocompromised state of the patient. The question asks what the best initial step of management is, which is control of symptoms with IV fluids, bowel rest, and fever control. Control of symptoms should precede antibiotic initiation. Go to the next page if you knew the correct answer, or click the link image(s) below to further research the concepts in this question (if desired).

Research Concepts: Pseudoappendicitis

We update eBooks quarterly and Apps daily based on user feedback. Please tap flag to

report any questions that need improvement.

Question 686: A 72-year-old female was admitted with a newly diagnosed cerebrovascular accident and is evaluated for swallowing function. It is recommended to keep her NPO and place a feeding tube. The patient is unable to provide a surgical history, but her spouse states that she had a weight loss surgery, and part of her stomach was removed. What is the next best step?

Choices: 1. Further imaging to delineate the anatomy 2. Proceed with percutaneous endoscopic gastrostomy tube placement 3. Proceed with open gastrostomy tube placement 4. Discharge the patient with a nasoenteric feeding tube

Answer: 3 - Proceed with open gastrostomy tube placement Explanations: Though additional imaging may be helpful, it is safest to assume that the patient had a bariatric procedure that altered the anatomy, such as a gastric sleeve or roux-en-y gastric bypass. The safest procedure for the patient is an open procedure, either gastrostomy or jejunostomy. This allows the surgeon to evaluate the anatomy during the procedure and make the best decision for tube placement. The patient will need long term feeding access, and a nasoenteric feeding tube is not the most appropriate. Go to the next page if you knew the correct answer, or click the link image(s) below to further research the concepts in this question (if desired).

Research Concepts: Feeding Tube

We update eBooks quarterly and Apps daily based on user feedback. Please tap flag to report any questions that need improvement.

Question 687: A 21-year-old man with a past medical history of metastatic testicular cancer with extensive metastasis to bone, brain, and spine is brought to the emergency department with altered mental status. Bedside physical exam is limited due to his mental state but reveals that he is unresponsive to a verbal or painful stimulus. He has no nuchal rigidity and has normal and reactive pupils. The patient is found to having tongue lacerations due to selfinflicted bites. A CT head is suggestive of multiple metastatic lesions. The patient appears obtunded and intermittently continues to have violent shaking episodes. His mother is at the bedside and is tearful given her son's lack of improvement. She wants him to be made comfortable. Which of the following is the next best step in the management of this patient?

Choices: 1. Olanzapine 2. Soft restraints 3. IV midazolam 4. MRI brain

Answer: 3 - IV midazolam Explanations: The patient presents with an emergent need for sedation for the alleviation of symptoms. The patient has generalized tonic-clonic seizures, thus administration of IV benzodiazepines such as midazolam would control seizure activity and achieve palliative sedation. Neurosurgey should also be consulted. However, the patient presents with seizure-like activity and thus needs treatment for it immediately. The etiology of this patient's seizures is likely due to his intracranial metastasis. Thus administering antipsychotics is not the correct answer as it is reserved for patients with catatonia and underlying psychosis presenting with altered mental status. Likewise placing the patient on restraints would not be a part of his treatment as he is having seizures and not merely presenting with agitations. While getting an MRI of the brain for further evaluation of the mass is a good decision, it should not delay the administration of medication that would achieve symptoms control and relief to the patient. Palliative sedation can be offered before or while he is being evaluated by neurosurgery. Go to the next page if you knew the correct answer, or click the link image(s) below to further research the concepts in this question (if desired).

Research Concepts: Palliative Sedation In Patients With Terminal Illness

We update eBooks quarterly and Apps daily based on user feedback. Please tap flag to report any questions that need improvement.

Question 688: A 31-years-old man presents to the hospital with a chief complaint of productive cough. He also reports nausea and malaise. His vital signs show a temperature of 38.6 C and respiratory rate of 32/min. He has a history of one-time cocaine use with his friends 4 days before. He also has a history of gingivitis a week ago and had not resolved. What is the most likely risk factor responsible for the patient's condition?

Choices: 1. Esophageal dysmotility 2. Cocaine use 3. Immunosuppression 4. Dental infection

Answer: 4 - Dental infection Explanations: Productive cough and fever increase the likelihood of lung abscess as one of the differential diagnoses. Gingivitis is a dental infection that may predispose to lung abscess infection. Lung abscesses are more common in immunocompromised hosts (HIV-AIDS, posttransplantation, or those receiving prolonged immune suppressive therapy). Lung abscesses are also common in those patients with high risk for aspiration: seizures, bulbar dysfunction, and cognitive impairment. Lung abscess is in the group of lung infections such as lung gangrene and necrotizing pneumonia which is characterized by multiple abscesses The patient has no risk factors for immunosuppression. Go to the next page if you knew the correct answer, or click the link image(s) below to further research the concepts in this question (if desired).

Research Concepts: Lung Abscess

We update eBooks quarterly and Apps daily based on user feedback. Please tap flag to report any questions that need improvement.

Question 689: A 68-year-old man is brought to the emergency department with hematemesis. He has a heart rate of 116/min blood pressure 98/63 mmHg. He has a history of type 2 diabetes, hypertension, and alcohol use disorder. He recently recovered from a traumatic fracture of his femur. His hemoglobin is found to be 9 g/dL. Urgent endoscopy reveals a circumferential blackened appearance to the distal third of the esophagus. Which of the following is the most likely cause of the patient's condition?

Choices: 1. Portal hypertension 2. Esophageal malignancy 3. Esophageal necrosis 4. Esophageal stricture

Answer: 3 - Esophageal necrosis Explanations: Chronic vascular disease and acute ischemia can lead to the development of acute oesophageal necrosis. The distal third of the esophagus has the poorest blood supply in relation to the other twothirds. This makes it the most common region to develop necrosis. Traumatic femoral fracture often results in significant blood loss creating an acute ischaemic state that can lead to the development of acute oesophageal necrosis. Acute oesophageal necrosis is the only answer that would give circumferential blackening of the esophagus in an acutely unwell patient. Go to the next page if you knew the correct answer, or click the link image(s) below to further research the concepts in this question (if desired).

Research Concepts: Esophageal Necrosis

We update eBooks quarterly and Apps daily based on user feedback. Please tap flag to report any questions that need improvement.

Question 690: A 70-year-old man with a history of hypertension and 45 pack-year smoking is brought to the emergency department with acute onset chest pain. The pain is described as "sharp," and it radiates to his lower back. On physical examination, he is looking uncomfortable and in significant pain. His blood pressure is 202/90 mmHg in both arms; the pulse is 118/min, respiratory rate 26/min, and temperature 37.2 C (99 F). The lungs are clear to auscultation. The heart examination is unremarkable for murmur or gallop. An electrocardiogram shows sinus tachycardia without ST segments elevation or depression. A chest x-ray shows a normal aortic silhouette. CT of the chest/abdomen/pelvic with intravenous contrast shows acute aortic dissection extending from proximal descending aorta distal to the left subclavian and extending distally to the right renal artery. Which of the following is the next best step in the management of this patient?

Choices: 1. Intravenous hydralazine 2. Intravenous nifedipine 3. Intravenous sodium nitroprusside 4. Intravenous labetalol

Answer: 4 - Intravenous labetalol Explanations: In acute aortic dissection, the initial treatment goal is to control heart rate and blood pressure. Beta-blockers, such as intravenous labetalol or esmolol, are the preferred first-line medication. The medical therapy goal is to lower the heart rate to a goal of 60/min and systolic blood pressure of 120 mmHg. Beta-blockers are the preferred agent owing to their negative chronotropic and lowering blood pressure effects. Other blood pressure medications do not reliably lower the heart rate and are at risk of increasing aortic wall shear stress. Go to the next page if you knew the correct answer, or click the link image(s) below to further research the concepts in this question (if desired).

Research Concepts: Systolic Hypertension

We update eBooks quarterly and Apps daily based on user feedback. Please tap flag to report any questions that need improvement.

Question 691: A 55-year-old woman presents to the hospital for 1 day if experiencing recurrent episodes of palpitations that led to shortness of breath and lightheaded. She has a past medical history of atrial myxoma resection 20 years ago, nonobstructive coronary artery disease, type 2 diabetes, hypertension, and hyperlipidemia. She also has a history of atrial flutter refractory to optimal drug therapy with the maximum dose of beta-blockers, rhythm control with dronedarone. Her vital signs are within normal limits. EKG shows atrial flutter with a variable conduction block. Due to the inefficacy of non-invasive intervention, a plan was made to cardiovert the patient. Which of the following is the most appropriate relation of paced cycle length compared to the tachycardia cycle length for entraining this patient's rhythm?

Choices: 1. 10 to 20 ms shorter 2. 20 to 30 ms shorter 3. Equal 4. 10 to 20 ms longer

Answer: 1 - 10 to 20 ms shorter Explanations: Waldo formulated three criteria for entrainment. These are fixed fusion with constant pacing rate, progressive fusion at faster pacing, and resumption of tachycardia with captured not fused beat on termination of pacing. Through entrainment, Waldo et al. differentiated between atrial flutter type I and II. Atrial flutter type I can be transiently entrained and interrupted by rapid atrial pacing since it is caused by reentry and has an excitable gap. Type I atrial flutter can show concealed entrainment with a zone of slow conduction. In contrast, type II atrial flutter has not been entrained, and the mechanism remains speculative. Entrainment with fusion proves that reentry is the cause of the investigated arrhythmia. The PPI-TCL duration gives information on whether the place of stimulation is inside our outside the reentry circuit. PPI response pattern characterizes different electrophysiologic properties of the tissues (excitable gap, excited tissue, refractory tissue). To entrain the tachyarrhythmia, the paced cycle length needs to be 10 to 20 ms shorter than the tachycardia cycle length (TCL). Go to the next page if you knew the correct answer, or click the link image(s) below to further research the concepts in this question (if desired).

Research Concepts: Reentry Arrhythmia

We update eBooks quarterly and Apps daily based on user feedback. Please tap flag to report any questions that need improvement.

Question 692: A 65-year-old male with a past medical history of atrial fibrillation on warfarin and metoprolol, gastroesophageal disease, and type 2 diabetes on metformin presents to the emergency department after the onset of word slurring and left facial droop 3 hours ago. He is still unable to articulate a complete sentence. After a comprehensive neurologic exam and imaging, an area of decreased perfusion is found in the territory supplied by the middle cerebral artery, and tPA is administered. The patient's symptoms partially improve. Several hours after treatment, the patient develops worsening confusion and arm weakness. Repeat computed tomography scan shows a new large area of increased attenuation in the lateral parietal lobe. What is the appropriate next step?

Choices: 1. Repeat tPA 2. Computed tomography scan with IV contrast 3. Neurosurgical intervention 4. Magnetic resonance angiogram

Answer: 3 - Neurosurgical intervention Explanations: Post-tPA intracranial hemorrhage should be considered if the patient develops a worsening neurologic exam, new headache, change in the level of consciousness, increased weakness, or sudden change in blood pressure. Risk factors include older age, greater stroke severity, hyperglycemia/diabetes, hypertension, atrial fibrillation, heart disease, and anticoagulant use. This pathology usually happens within 36 hours after thrombolytic infusion. Only about half of the patients with this condition are diagnosed 5–10 hours after the administration of tPA. The first step taken should be to immediately stop the infusion of tPA if it is still running. Computed tomography scan of the brain is the first imaging step in the evaluation of possible stroke or when there is suspicion of conversion to hemorrhagic after administration of thrombolytic agents. If hemorrhage is found (seen on computed tomography scan as the previous ischemic infarct with superimposed hyperintense cerebral hemorrhage), the next steps would be to collect baseline labs and reverse the anticoagulation using cryoprecipitate or other products. In addition, neurosurgery should be called for potential surgical treatment. Go to the next page if you knew the correct answer, or click the link image(s) below to further research the concepts in this question (if desired).

Research Concepts: Cerebral Ischemia

We update eBooks quarterly and Apps daily based on user feedback. Please tap flag to report any questions that need improvement.

Question 693: Which of the following patients in cardiogenic shock would benefit the most from an intra-aortic balloon pump?

Choices: 1. A patient with a history of cardiogenic shock who has new Q waves and pulmonary edema 2. A trauma patient with hypotension, distended neck veins, and muffled heart tones 3. A patient with left mainstem artery disease and poor response to IV dopamine 4. A patient with an inferoposterior myocardial infarction and new onset of a cooing, systolic murmur

Answer: 4 - A patient with an inferoposterior myocardial infarction and new onset of a cooing, systolic murmur

Explanations: Intra-aortic balloon pump is a device that increases myocardial oxygen perfusion through the dilation of coronaries during diastole and also augments cardiac output. A balloon pump is most useful to stabilize a patient with a surgically correctable lesion that is causing heart failure, such as a ruptured papillary muscle with acute mitral regurgitation. Patients with a new myocardial infarction, in addition to severe, preexisting cardiac disease, have a poor prognosis. The balloon pump is not indicated for pericardial tamponade, which may occur in a trauma patient. Go to the next page if you knew the correct answer, or click the link image(s) below to further research the concepts in this question (if desired).

Research Concepts: Intra-Aortic Balloon Pump

We update eBooks quarterly and Apps daily based on user feedback. Please tap flag to report any questions that need improvement.

Question 694: A patient develops meningitis after head trauma. The patient expires and the autopsy reveals softening of brain tissue and diffuse edema. The patient most likely had what type of necrosis?

Choices: 1. Coagulation necrosis 2. Liquefactive necrosis 3. Wedge infarct 4. Fat necrosis

Answer: 2 - Liquefactive necrosis Explanations: Liquefactive necrosis is transformation of solid tissue into fluid. Liquefactive necrosis is typical of bacterial or fungal infections. The necrosis is due to a release of hydrolytic enzymes. Liquefactive necrosis often results in abscess formation. Go to the next page if you knew the correct answer, or click the link image(s) below to further research the concepts in this question (if desired).

Research Concepts: Cell Liquefactive Necrosis

We update eBooks quarterly and Apps daily based on user feedback. Please tap flag to report any questions that need improvement.

Question 695: A 65-year-old female undergoes a hip replacement after a fall. Two days later, she develops sudden shortness of breath and tachycardia. On examination, she is tachypneic with a heart rate of 120 beats per minute. Her blood pressure is 120/57 mm Hg. A blood gas reveals a pH of 7.38, CO2 of 35, O2 of 72 and bicarbonate of 23. A duplex of the extremities reveals a large clot in the iliofemoral veins. For a patient with these findings, what is the most frequent finding on an x-ray?

Choices: 1. Normal 2. Hampton hump: a cone-shaped area of opacification 3. Westermark sign: a dilated pulmonary artery with distal oligemia 4. Pleural effusion

Answer: 1 - Normal Explanations: It is virtually impossible to diagnose a pulmonary embolism based on a chest x-ray. The most common presentation of a patient with a pulmonary embolism will be a normal chest x-ray. When the x-ray is normal and suspicion is still high, the next step is to obtain a spiral CT. If the CT is normal with a normal x-ray, one can exclude the diagnosis of pulmonary embolism. When a pulmonary embolus occurs, it may occlude one or a few branches of the pulmonary artery. It may present on an x-ray as a cone-shaped area of opacification, representing an area of atelectasis or infraction. This sign is seen in only a few patients. Occasionally there is a large pulmonary embolus that blocks the pulmonary artery. The proximal artery then appears dilated with distal collapse. This is seen as an area of decreased vascular markings on the chest x-ray. The chest x-ray must be compared to a previous normal x-ray to decipher these fine changes. Again, this sign is seen in only a few patients with pulmonary embolism. Go to the next page if you knew the correct answer, or click the link image(s) below to further research the concepts in this question (if desired).

Research Concepts: Acute Pulmonary Embolism

We update eBooks quarterly and Apps daily based on user feedback. Please tap flag to report any questions that need improvement.

Question 696: A 50-year-old man is brought into the emergency department. The patient had flu-like symptoms in the past few days. On physical examination, the patient is tachycardic, febrile, and tachypneic. The patient's past medical history is unremarkable. While admitted at the hospital, the patient was placed in a room with the air conditioner set at 55 degrees Fahrenheit. The next day the patient is found hypothermic with a new irregular irregular heart rhythm. Which conduction pathway in the heart is most likely affected?

Choices: 1. Sinoatrial node 2. Atrioventricular node 3. Coronary sinus 4. Bundle of His

Answer: 1 - Sinoatrial node Explanations: Atrial fibrillation may develop from low body temperature and can successfully be treated by warming the patient to normal body temperature. Mild hypothermia may have a presentation of shivering and intact mentation. Act quickly to rewarm before arrhythmias, or other complications develop. Remove wet clothing, dry the patient, and raise the temperature in the room. Give warmed oral liquids containing glucose if no contraindications exist such as vomiting. Warmed oxygen and intravenous fluids may be needed. Atrial fibrillation will show an absence of p waves which are derived from the SA node. Mild hypothermia is classified as a core temperature of 89.6 degrees F to 95 degrees F or 32 degrees C to approx 35 degrees C. Moderate hypothermia is 82.4 degrees F to 89.6 degrees F or 28 degrees C to 32 degrees C, and severe hypothermia is 82.4 degrees F or 28 degrees C. Atrial fibrillation and ventricular fibrillation are common in hypothermic patients. The risk for ventricular fibrillation rises as hypothermia worsens. Under 28 degrees C or 82 degrees F, it is essential to prepare for ventricular fibrillation (vfib). Vfib is not treated the same way in the presence of hypothermia. Rewarming must take place for shocks to be effective. One shock and one round of drugs are administered. If no effect, rewarm the patient to a core temperature of >86 degrees F or 30 degrees C before more attempts are made. Even the slightest movement can cause vfib in a profound hypothermic patient. Go to the next page if you knew the correct answer, or click the link image(s) below to further research the concepts in this question (if desired).

Research Concepts: Hypothermia

We update eBooks quarterly and Apps daily based on user feedback. Please tap flag to report any questions that need improvement.

Question 697: A 47-year-old male is brought to the emergency department after being found unconscious in the snow. His blood pressure is 90/55 mmHg, the pulse is 56 beats per minute, respiratory rate is 13 breaths per minute, and a temperature of 31 C. He is semiconscious and arousable to sternal pressure. The paramedic is only able to get interosseous cannulation in the left tibia after attempting for IV access multiple times. Central venous line placement is planned. Which of the following is the best possible site of central venous line placement in this patient?

Choices: 1. Brachiocephalic vein 2. Internal iliac vein 3. Femoral vein 4. Internal jugular vein

Answer: 3 - Femoral vein Explanations: Hypothermia can cause slowed impulses conduction through potassium channels resulting in prolonged ECG intervals. Patients with moderate to severe hypothermia will likely exhibit bradycardia and are at increased risk for ventricular arrhythmias. Therefore, care should be taken to not stimulate the heart due to the increased irritability of the myocardium. If central access is required in the setting of hypothermia, a femoral central venous line (CVL) is preferred due to less irritation of myocardium and a lower risk of dysrhythmia. The ideal central venous access is through the femoral vein, which leads to external iliac then to the common iliac and finally ending at the inferior vena cava. Internal jugular and brachiocephalic veins both lead to the superior vena cava. Both are not ideal sites for venous access compared to femoral vein cannulation due to higher myocardial irritability and dysrhythmia. Internal iliac vein empties into the common iliac vein without directly connecting to the femoral vein. Go to the next page if you knew the correct answer, or click the link image(s) below to further research the concepts in this question (if desired).

Research Concepts: Hypothermia

We update eBooks quarterly and Apps daily based on user feedback. Please tap flag to report any questions that need improvement.

Question 698: A 46-year-old male with ARDS from aspiration pneumonia is admitted to the intensive care unit. He has been intubated and sedated for the last several days with increasing oxygen requirement. The patient has been sedated with propofol infusion and fentanyl infusion, however, the patient has been over-breathing the ventilator. It is decided to start the patient on a cisatracurium drip. The patient received 10 mg of bolus followed by infusion of cisatracurium at 4 mcg/kg/min. He had RASS goal of - 4 and train of four was 2, and the patient was completely synchronous with the ventilator. Vital signs were Blood pressure 100/60 mmHg, heart rate 66 beats per minute, Temp 98.4 F at the time of initiation of the medication. 12 hours later, the attending clinician noticed a significant increase in the drip rate of cisatracurium from 4 mcg/kg/min to 10 mcg/kg/min. The patency of the intravenous line and IV tubing was assessed, and no issue was identified. The only change in patient's clinical condition was the fact that he had been persistently febrile overnight with Tmax of 101.4 F. What could be a possible explanation for this increased requirement of the medication?

Choices: 1. Tachyphylaxis 2. Fever 3. Worsening ARDS 4. Malfunctioning intravenous line

Answer: 2 - Fever Explanations: Cisatracurium is eliminated by Hoffman's elimination which is a process independent of liver and kidney. Factors that affect Hoffman elimination process include temperature and pH changes. Patient had significant fever thus increasing the elimination process of the medication and needing more medication to achieve the same degree of paralysis Tachyphylaxis to such an extent is not usually seen this early with this medication. Worsening ARDS has no bearing on increasing requirements of paralytic dose. It was ascertained that the intravenous lines were working fine so there is no reason to doubt that statement. Go to the next page if you knew the correct answer, or click the link image(s) below to further research the concepts in this question (if desired).

Research Concepts: Cisatracurium

We update eBooks quarterly and Apps daily based on user feedback. Please tap flag to report any questions that need improvement.

Question 699: A 65-year old female diagnosed with a lung abscess two weeks ago. The patient was started on intravenous (IV) piperacillin and tazobactam, but her symptoms did not improve and she had a fever, productive cough, and shortness of breath. Her past medical history is significant for congestive heart failure (CHF) with reduced ejection fraction with an automated implantable cardioverter-defibrillator (AICD) and left-sided hemiplegia. She has been bed-bound for the past year. Her vitals show a blood pressure of 100/90 mmHg, a temperature of 103 Fahrenheit, a pulse rate of 110 beats per minute, and a respiratory rate of 21 breaths per minute. A chest x-ray revealed a cavity lesion in the posterior segment of the right middle lobe. Computed tomography (CT) of the chest showed a lung abscess with a size of around 8 cm. The size of the abscess has increased as compared to the previous CT scan. Which of the following is the next step in the management of the patient's current condition?

Choices: 1. Continue with the antibiotic coverage 2. Surgical resection 3. Endoscopic or percutaneous drainage 4. Bronchoalveolar lavage

Answer: 3 - Endoscopic or percutaneous drainage Explanations: Depending on the patient's immune system, a lung abscess can be indolent or symptomatic. Surgical intervention is considered in patients who fail to respond to medical therapy. A segmentectomy or lobectomy of the lung is done. Patients who are poor surgical candidates percutaneous and endoscopic drainage can be considered. Lung abscesses are common in patients with high risk for aspiration, who have seizures, bulbar dysfunction, and cognitive impairment. Go to the next page if you knew the correct answer, or click the link image(s) below to further research the concepts in this question (if desired).

Research Concepts: Lung Abscess

We update eBooks quarterly and Apps daily based on user feedback. Please tap flag to report any questions that need improvement.

Question 700: A 68-year-old woman presents with sudden onset difficulty walking. The symptoms started about 6 hours ago upon awakening. She has a past medical history of type 2 diabetes mellitus, which is well controlled. Her vital signs show oxygen saturation 98% on room air, respiratory rate 17 per minute, heart rate 87 beats per minute, blood pressure 170/95 mmHg, and temperature 98 F. On examination, she is, alert, able to follow commands, and is orientated to time, place, and person. A CT scan of the head reveals a hypodense area in the right inferior cerebellum. Which of the following neurological exam findings should be investigated in this patient?

Choices: 1. Dysdiadochokinesia 2. Dysphasia 3. Peripheral facial palsy 4. Hemiparesis

Answer: 1 - Dysdiadochokinesia Explanations: The patient has a lesion in the cerebellum. Cerebellar lesions can present with instability and imbalance in the absence of weakness or sensory deficits. The absence of cognitive deficits points to a lesion below the cerebrum. The posterior inferior cerebellar artery supplies the inferior cerebellum. It is the most likely artery affected in this patient. In the acute presentation of neurological deficits, in a vulnerable population, a plain cranial CT followed by a computed tomography angiography (CTA) would be helpful to direct stroke treatment. Go to the next page if you knew the correct answer, or click the link image(s) below to further research the concepts in this question (if desired).

Research Concepts: Cerebellar Dysfunction

We update eBooks quarterly and Apps daily based on user feedback. Please tap flag to report any questions that need improvement.

Section 8 Question 701: A 70-year-old man with a history of prior myocardial infarction with the placement of drug-eluting stents and diabetes presents to the emergency department with crushing chest pressure starting 2 hours ago. His electrocardiogram is notable for ST elevations in leads I, aVL, and reciprocal depressions in leads II, III, aVF. The patient is emergently taken for a left heart catheterization. He is noted to have an embolus in the proximal left anterior descending artery. His prior stent in the right coronary artery is patent. An aspiration thrombectomy is performed to remove the embolus. He is then referred for a transthoracic echocardiogram, which reveals a patent foramen ovale, and a 40 mm mass on the aortic valve on the left coronary cusp. What is the next best step in the management of this patient?

Choices: 1. Send the patient for emergent cardiac surgery to fix the patent foramen ovale (PFO) 2. Obtain transesophageal echocardiogram to evaluate the cardiac mass better 3. Implant an internal cardiac defibrillator for risk of sudden cardiac death 4. Reassurance

Answer: 2 - Obtain transesophageal echocardiogram to evaluate the cardiac mass better Explanations: Transesophageal echocardiogram (TEE) is the correct answer because it provides better spatial resolution than a TTE and helps narrow the differential diagnosis of an aortic valve mass, which includes valve thrombus, endocarditis, and papillary fibroelastoma (PFE). On TEE, PFE should have a distinct appearance of a mass on a stalk with a shimmering border. Before surgery, you need to be sure of the diagnosis, which requires further imaging evaluation. PFO is a distractor. While a venous side thrombus can traverse potentially to the left atrium via a PFO, the probability is lower with a 40 mm mass sitting much closer to the coronaries in the aortic root. While PFEs can increase the risk of sudden cardiac death, there is no data to support the routine use of ICD implantation. The patient became symptomatic from PFE and does not need further evaluation and intervention. Go to the next page if you knew the correct answer, or click the link image(s) below to further research the concepts in this question (if desired).

Research Concepts: Papillary Fibroelastoma

We update eBooks quarterly and Apps daily based on user feedback. Please tap flag to report any questions that need improvement.

Question 702: A 29-year-man with a history of polysubstance abuse and IV drug use is brought to the hospital by ambulance after being found unresponsive in a back alley. On his last admission, he was found to have methicillin-sensitive staphylococcus aureus bacteremia without complication. He completed antibiotic therapy two months ago. Vital signs reveal temperature 95 F, blood pressure 90/50 mmHg, pulse 98/min, respiratory rate 4/min, and SpO2 86% on ambient air. Physical exam shows a comatose well-developed male with poor oral hygiene. Toxicology is positive for benzodiazepines, opiates, marijuana, and alcohol. He has mildly elevated transaminases. He is given naloxone and flumazenil with minimal response. He is subsequently intubated and admitted to the intensive care unit. After two days of admission, he develops a fever. Urinalysis and physical examination are unrevealing. The chest radiograph shows the correct positioning of the endotracheal tube and a new infiltrate in the left lower lung. Which of the following factors is most strongly associated with an infection due to a multi-drug resistant organism in this patient?

Choices: 1. Hospitalization within the last 90 days 2. Poor oral hygiene 3. Positive sepsis criteria on admission 4. Prior IV antibiotic use within the last 90 days

Answer: 4 - Prior IV antibiotic use within the last 90 days Explanations: This patient has developed ventilator-associated pneumonia (VAP). VAP presents after 48 hours of intubation and mechanical ventilation. The most common pathogens seen in VAP are Staphylococcus aureus, Pseudomonas, Candida species, Klebsiella oxytoca and pneumoniae, Streptococcus species, and Enterobacter species. Multidrug-resistant organisms (MRDO) are commonly seen in VAP. Risk factors for VAP with MDRO are: prior IV antibiotic use within the last 90 days, need for ventilatory support, septic shock at the time of VAP, acute respiratory distress syndrome preceding VAP, greater than five days of hospitalization before VAP onset and need for acute renal replacement therapy. This patient received IV antibiotics for his S. aureus bacteremia infection two months ago and is the strong risk factor for developing MDRO infection. Go to the next page if you knew the correct answer, or click the link image(s) below to further research the concepts in this question (if desired).

Research Concepts: Nosocomial Infections

We update eBooks quarterly and Apps daily based on user feedback. Please tap flag to report any questions that need improvement.

Question 703: A 45-year-old man with a history of irritable bowel syndrome who has undergone appendectomy and cholecystectomy in the past is admitted to the emergency department with abdominal pain. The patient states that the pain has been ongoing over the last month. The pain is poorly localized and has been relieved by his home opioid pain medications until three days ago when the medications “just don’t seem to cut it.” It feels different from his previous flares of irritable bowel syndrome, for which morphine has helped to resolve. He has been taking oxycodone 10 mg twice daily for the pain. A review of the state drug prescription database shows that he was previously taking hydrocodone 10 mg until approximately one month ago when he was prescribed oxycodone. The patient’s vital signs reveal blood pressure: 128/76 mmHg, temperature: 37 C, and pulse rate: 82 beats/min. Physical examination reveals a well-appearing male in no acute distress and a mildly-tender abdomen. A CT scan does not show any acute intra-abdominal pathology. His bloodwork is unremarkable. A review of his medical record reveals multiple negative CT scans over the past 3 months. What is the next preferred step in management?

Choices: 1. Administer IV morphine 2. Confront the patient regarding drug-seeking behavior 3. Place a nasogastric tube and consult surgery 4. Symptomatic control with non-narcotic therapy

Answer: 4 - Symptomatic control with non-narcotic therapy Explanations: This patient with chronic abdominal pain despite multiple surgeries an unremarkable workup in the ED, including normal vital signs, normal laboratory findings, and negative imaging, has a diagnosis most consistent with narcotic bowel syndrome (NBS). Although this patient has a history of irritable bowel syndrome, these two disease entities can occur together. The symptoms of NBS can easily be erroneously attributed to IBS, as both are funcitonal abdominal pain disorders. This patient who states that the pain is different in quality from his previous IBS flares should prompt clinicians to consider an alternative diagnosis. NBS is a hyperalgesic syndrome caused by opioids. Thus, the mainstay of treatment is narcotic cessation. This can be done through the use of a multi-disciplinary approach, with gradual tapering of opioids. It is important to first counsel the patient that opioids are likely worsening his overall condition. The administration of opioid analgesics will likely worsen his condition, and may cause him to require an escalation of opioid therapy in the future to temporarily relieve the pain. Although a multi-disciplinary approach, including psychotherapy and/or cognitive behavioral therapy, will likely be used for the definitive management of this patient's condition, the patient must first be counseled on the condition before this can begin. It is easy to attribute this patient's behavior to drug-seeking behavior. However, the fact that the patient has undergone multiple surgical procedures and multiple ED workups should prompt one to believe that the patient has been genuinely seeking treatment, rather than just medications. Go to the next page if you knew the correct answer, or click the link image(s) below to further research the concepts in this question (if desired).

Research Concepts: Narcotic Bowel Syndrome

We update eBooks quarterly and Apps daily based on user feedback. Please tap flag to report any questions that need improvement.

Question 704: A 65-year-old woman presents to the clinic with swelling over the bilateral lower limbs and dyspnea for seven months. She says that the swelling initially started around the ankle and progressed to the knee. She also started experiencing shortness of breath around the same time, which has progressed over the last few months. Now she can only walk 100 m before she starts panting. She has no history of alcohol consumption, smoking, or illicit drug use. Blood pressure is 100/90 mmHg, the pulse rate is 90/min, and the jugular venous pulse (JVP) is 10 cm. Examination reveals pitting edema over the bilateral lower limb. The cardiovascular exam reveals a diastolic murmur over the right sternal border. S2 is split, and S1 is normal. A 2DEchocardiogram reveals normal left-sided cardiac function but does reveal a pulmonary and tricuspid regurgitation. Laboratory investigations reveal prothrombin time (PT) 10 seconds, activated partial thromboplastin time (aPTT) is 30 seconds; the D-dimer study is normal. Pulmonary arterial pressure is 25 mmHg, and capillary wedge pressure is 10 mmHg. There is mild tenderness over the right subcostal region, but there is no organomegaly observed. What is the treatment of choice for the primary cause of this patient's symptoms?

Choices: 1. Enoxaparin 2. Bosentan 3. Right ventricular assist device 4. Levosimendan

Answer: 2 - Bosentan Explanations: This patient with bilateral pitting edema till the knee, dyspnea for seven months, a wide split S2 and a tricuspid regurgitation on 2D echocardiogram is a case of primary pulmonary hypertension. Right heart failure in this patient is secondary to back pressure from primary pulmonary hypertension, causing an abnormal function at both tricuspid and pulmonary valve. With the backpressure, the right atrium becomes dilated. Subsequently, it causes a reduction in the venous return, further leading to a raised JVP and edema in the organs upstream from the right heart. Bosentan is an endothelin receptor antagonist and aids in pulmonary vasodilation. Chronic right ventricular failure (RVF) secondary to primary pulmonary hypertension can be treated by causing pulmonary vasculature to dilate. This is done by endothelin receptor antagonists (ERAs) and phosphodiesterase inhibitors (PDE5-inhibitors). They are pulmonary vasodilators that reduce PAP, reduce pulmonary venous resistance (PVR), and improve cardiac output in patients with RVF. ERAs block the endothelin-A and endothelinB receptors in endothelial and vascular smooth muscle cells, reducing the vasoconstrictive, proliferative, and proinflammatory effects of endothelin. The use of ERAs in the ICU is limited by their longer half-life and hepatotoxicity (bosentan). PDE5-inhibitors block degradation of cGMP. Enoxaparin is low molecular weight heparin used that could be used in cases of right heart failure secondary to a massive pulmonary embolus. However, a normal D-dimer study and the chronic nature of this patient's complaints make this diagnosis less likely. Levosimendan is used to increase left ventricular (LV) output. It is an inotrope. However, this patient has a normal left ventricular function as per 2D-Echocardiogram, and her right heart failure is not secondary to left heart failure. This treatment would not be effective. A right ventricular assist device may be an option for the patient with isolated RVF awaiting transplant. However, a transplant without addressing pulmonary hypertension would lead to a recurrence in right heart failure post-transplant. Go to the next page if you knew the correct answer, or click the link image(s) below to further research the concepts in this question (if desired).

Research Concepts: Right Heart Failure

We update eBooks quarterly and Apps daily based on user feedback. Please tap flag to report any questions that need improvement.

Question 705: A 65-year-old man presents to the emergency department with dyspnea and palpitations. He has a past medical history significant for congestive heart failure and moderately severe asthma. He is allergic to diltiazem. He drinks alcohol occasionally and denies smoking. His vital signs show blood pressure 118/80 mmHg and heart rate 150/min with an irregular rhythm. EKG shows absent P waves along with an irregular QRS complex. Which of the following is the next step in the management of this patient?

Choices: 1. Quinidine 2. Amiodarone 3. Digoxin 4. Metoprolol

Answer: 3 - Digoxin Explanations: The initial goal in this patient is rate control. Beta-blockers and diltiazem are first-line therapies. Metoprolol should be used with caution in patients with asthma. Digoxin is indicated in the treatment of heart failure with concomitant atrial fibrillation. Cardiac glycosides such as digoxin work by inhibiting the sodium-potassium ATPase (NaK ATPase) pump. Digoxin has a narrow therapeutic index ranging between 0.5 and 0.9 ng/mL. Digoxin toxicity manifests when the serum drug level reaches more than 2.0 ng/mL. Digoxin overdose can happen due to myriad of reasons, including iatrogenic causes such as prescribing higher doses of the drug to patients, noncompliance by the patient, accidental overdosing by taking more pills as prescribed, consumption of plants containing cardiac glycosides, chronic kidney disease, and hypokalemia can also predispose patients to digoxin excess. Go to the next page if you knew the correct answer, or click the link image(s) below to further research the concepts in this question (if desired).

Research Concepts: Digoxin

We update eBooks quarterly and Apps daily based on user feedback. Please tap flag to report any questions that need improvement.

Question 706: A 65-year-old patient presents with numerous complaints. He claims that over the past few months, he has no appetite, intermittent nausea, weakness, headache, muscle cramps, and confusion. He denies any trauma and has no medical issues except for being a smoker. The physical exam is unremarkable. Blood work reveals sodium of 120 mEq/L, serum osmolality is 230, and high urine osmolality. The urine sodium is more than 40 mEq/L. The blood urea nitrogen is low, but the potassium levels are normal. CT scan reveals the presence of cerebral edema. What should the initial treatment be in this patient?

Choices: 1. Demeclocycline 2. Fluid restriction 3. Low dose lithium 4. Intravenous hypertonic saline

Answer: 4 - Intravenous hypertonic saline Explanations: The treatment of the syndrome of inappropriate antidiuretic hormone secretion (SIADH) depends on whether the patient is symptomatic, the degree of hyponatremia, and whether the condition is acute or chronic. As this patient has sodium of 120 mEq/L, hypertonic saline is indicated. Correcting hyponatremia very quickly can lead to central pontine myelinolysis. In patients where the sodium is not as low, the first-line treatment is the restriction of fluid intake. The use of lithium or ADH antagonists is not recommended. Go to the next page if you knew the correct answer, or click the link image(s) below to further research the concepts in this question (if desired).

Research Concepts: Syndrome of Inappropriate Antidiuretic Hormone Secretion

We update eBooks quarterly and Apps daily based on user feedback. Please tap flag to report any questions that need improvement.

Question 707: A 17-year-old male patient is 48 hours into treatment for septic shock. He is currently ventilated with the following settings: FiO2 is 0.6, peak inspiratory pressure/positive end-expiratory pressure ratio is 28:8, and the rate is 18 breaths per minute. He is receiving a combination of epinephrine (0.1 mcg/kg per min) and norepinephrine (0.1 mcg/kg per min) for circulatory support, and dexmedetomidine for sedation. His vital signs include a temperature of 38.3 C (100.9 F), heart rate of 110 beats per minute, blood pressure of 98/65 mmHg, and central venous pressure (CVP) of 10 mmHg. Peripheral perfusion is characterized by 1+ to 2+ pulses and 2 to 3 second capillary refill. Urine output is 0.4 mL/kg per hour and has not changed since a bolus of normal saline increased his central venous pressure from 8 to 10 mmHg. His acidosis has resolved. The clinician is about to start a milrinone infusion. Which of the following is most accurate regarding this patient’s management?

Choices: 1. The low urine output most likely is related to inadequate intravascular volume 2. Decreasing his norepinephrine dose is more likely to improve peripheral perfusion than decreasing the epinephrine infusion rate 3. Heart rate and contractility will be affected to about the same degree with a decrease in either the epinephrine or norepinephrine 4. The low urine output most likely will improve with antibiotics

Answer: 2 - Decreasing his norepinephrine dose is more likely to improve peripheral perfusion than decreasing the epinephrine infusion rate

Explanations: Although the initial response to distributive shock is a hyperdynamic state with elevated heart rate to maintain cardiac output despite low stroke volume, the ongoing capillary leak from inflammation leads to inadequate end-organ perfusion with resultant altered mental status, oliguria, and lactic acidosis. Initial steps to re-establish end-organ perfusion in shock should include adequate fluid resuscitation followed by the administration of vasoactive medications. However, practitioners should be careful not to over-resuscitate patients. A central venous pressure goal of 10, in the absence of heart failure, indicates the adequate establishment of intravascular volume. Norepinephrine is an endogenous catecholamine that is derived directly from dopamine, which is, in turn, derived from the amino acids, phenylalanine, or tyrosine. The cardiovascular effects of norepinephrine are mediated primarily through alpha-1 receptors in the peripheral vasculature, leading to vasoconstriction. As a sympathomimetic medication, it is an excellent first choice in volume-refractory sepsis states, i.e., severe sepsis or septic shock, characterized by a capillary leak. However, high doses of norepinephrine (> 0.1 mcg/kg/min) can lead to elevated systemic vascular resistance that can compromise end-organ perfusion. In this scenario, the high doses of norepinephrine may be worsening the glomerular filtration rate, leading to oliguria despite the adequate intravascular volume. Epinephrine is an endogenous catecholamine derived from the methylation of norepinephrine. Its cardiovascular effects are mediated through beta-1, beta-2, and alpha-1 receptors. When administered as a sympathomimetic drug, it leads to elevations in heart rate (chronotropy) and contractility (inotropy). Milrinone is a selective inhibitor of phosphodiesterase-3. Phosphodiesterase-3 is an enzyme that degrades cAMP and cGMP. With inhibition of this enzyme, there is a reduction in the degradation of cAMP, and therefore its effects are potentiated. The effects include an increase in inotropy, vasodilation, and minimal chronotropy. Milrinone seems to improve diastolic function, possibly by having a direct effect on the myocardium via enhancement of sarcomere uptake of calcium ions, that is, lusitropy. Go to the next page if you knew the correct answer, or click the link image(s) below to further research the concepts in this question (if desired).

Research Concepts: Septic Shock

We update eBooks quarterly and Apps daily based on user feedback. Please tap flag to report any questions that need improvement.

Question 708: A 67-year-old man with is being treated with proton pump inhibitors due to gastroesophageal reflux disease has weakness and nausea. Laboratory tests show a chronic kidney disease. The clinical evaluation demonstrates only a loss of deep tendon reflexes and apart from weakness and nausea the patient is asymptomatic. Which of the following is most likely to be seen on serum electrolyte profile of this patient?

Choices: 1. Mild Hypermagnesemia 2. Moderate Hypermagnesemia 3. Mild Hyponatremia 4. Moderate Hyponatremia

Answer: 1 - Mild Hypermagnesemia Explanations: In individuals with chronic kidney failure, some conditions including proton pump inhibitors, malnourishment, and alcohol use disorder can increase the risk of Hypermagnesemia. The earliest clinical sign of mild hypermagnesemia is the loss of deep tendon reflexes. The most frequent symptoms of mild hypermagnesemia include weakness, nausea, dizziness, and confusion. Moderate hypermagnesemia is characterized by the worsening of the confusional state and sleepiness, bladder paralysis, flushing, headache, paralytic ileus as well as a slight reduction in blood pressure and blurred vision caused by diminished accommodation and convergence. Go to the next page if you knew the correct answer, or click the link image(s) below to further research the concepts in this question (if desired).

Research Concepts: Hypermagnesemia

We update eBooks quarterly and Apps daily based on user feedback. Please tap flag to report any questions that need improvement.

Question 709: A 65-year-old male patient underwent a microsurgical clipping following a ruptured anterior communicating artery aneurysm. The patient, on his third post-operative day, became slightly drowsy, along with the worsening of the motor power in his bilateral lower limbs. What is the most likely cause of the deterioration in this patient?

Choices: 1. Meningitis 2. Rebleed 3. Vasospasm 4. Seizure

Answer: 3 - Vasospasm Explanations: There is a high risk of vasospasm in subarachnoid hemorrhage following aneurysmal bleed, owing to the release of bilirubin. In cases of anterior communicating artery aneurysmal (ACOM) bleed, there is an onset of vasospasm within the territories of the distal anterior cerebral arteries (DACA). The distal anterior cerebral arteries supply paracentral lobule that controls power in the bilateral lower limbs. Therefore, vasospasm in this region leads to weakness of bilateral lower limbs. Rebleeding, despite microsurgical clipping, is rare. Since the patient did not have a highgrade fever, chills, and neck rigidity, the possibility of meningitis is also low. The patient also did not have any seizurogenic activities. Go to the next page if you knew the correct answer, or click the link image(s) below to further research the concepts in this question (if desired).

Research Concepts: Anterior Cerebral Artery Stroke

We update eBooks quarterly and Apps daily based on user feedback. Please tap flag to report any questions that need improvement.

Question 710: A 20-year-old man presents with hoarseness and drooling after another player elbowed him in the anterior neck while playing basketball. His heart rate is 90 beats per minute, blood pressure 120/80 mmHg, respiratory rate 27 breaths per minute, and he is saturating 93% on room air. Which of the following is true regarding the management of this patient?

Choices: 1. The treatment of choice is humidified oxygen 2. Early orotracheal intubation should be avoided 3. A cricothyroidotomy kit should be at the bedside when intubation is attempted 4. Awake intubation is contraindicated

Answer: 3 - A cricothyroidotomy kit should be at the bedside when intubation is attempted Explanations: Following blunt laryngotracheal trauma, early control of the airway is advisable as edema may rapidly progress to airway compromise. This patient presents with signs and symptoms concerning for impending airway loss given the mechanism of his injury. This patient's airway should be secured early via intubation, but equipment to perform a cricothyroidotomy should be readily available in case the patient loses his airway during the attempted intubation. Tracheal trauma may present with no symptoms, or with symptoms including hoarseness. The mildest, usually minimally symptomatic, cases may be treated with supplemental oxygen and observation. Awake intubation is preferred over rapid sequence intubation when intubation and ventilation are predicted to be difficult. In this patient with potential impending airway loss, awake intubation would be ideal. Regardless of the type of intubation technique planned, equipment necessary for a surgical airway, such as a cricothyroidotomy, should be readily available in case intubation fails due to loss of patency of the airway. Go to the next page if you knew the correct answer, or click the link image(s) below to further research the concepts in this question (if desired).

Research Concepts: Tracheal Trauma

We update eBooks quarterly and Apps daily based on user feedback. Please tap flag to report any questions that need improvement.

Question 711: A 50-year-old man is brought to the hospital in a state of altered mentation at home by his close relatives. He has a history of HIV for the past 8 years and has been on regular antiretroviral therapy. He also has a history of hypertension and diabetes, well-controlled with his regular medications. An urgent MRI reveals bilateral diffuse cortical changes with patchy brain stem involvement. A toxicology screen is negative. CSF cytology is within normal limits. CSF for HSV PCR comes back positive for HSV-1. What is the best initial therapy for this patient?

Choices: 1. Ganciclovir 2. Foscarnet 3. Cidofovir 4. Acyclovir

Answer: 4 - Acyclovir Explanations: This immunocompromised person is diagnosed to have herpes simplex encephalitis on the basis of positive HSV-1 PCR of CSF. HSV-1 is the most common sporadic and fatal encephalitis worldwide in adults. He needs to be immediately initiated on IV acyclovir which is the first-line agent in herpes simplex encephalitis. Acyclovir prevents viral multiplication. Herpes simplex encephalitis has got significant morbidity & mortality despite antiviral treatment. The mortality and morbidity are much higher in immunocompromised. The notable side effects of acyclovir include thrombophlebitis and crystal-induced nephropathy; the latter is prevented by adequate hydration. Atypical or subtle clinical presentations are common in immunocompromised patients which could delay the diagnosis and treatment. Diffusion-weighted MRI with flair sequences can usually detect the classic temporal lobe/limbic system changes in herpes simplex encephalitis (HSE). However, a predominantly extra-temporal presentation with more diffuse involvement of the brain can occur in immunocompromised. CSF pleocytosis may not occur in immunocompromised. IV ganciclovir can be considered if IV acyclovir preparation is not available. Acyclovir resistance is higher in immunocompromised patients than immunocompetent. Foscarnet or cidofovir is considered in such cases. Go to the next page if you knew the correct answer, or click the link image(s) below to further research the concepts in this question (if desired).

Research Concepts: Herpes Simplex Encephalitis

We update eBooks quarterly and Apps daily based on user feedback. Please tap flag to report any questions that need improvement.

Question 712: A 65-year-old female presents with fever, chills, and confusion. Her respiratory rate and heart rate are elevated and her blood pressure is 90/60. Urine output is diminished. Which treatment will most likely increase systemic vascular resistance and be the best immediate treatment option in this patient?

Choices: 1. Epinephrine 2. Dopamine 3. Intravenous fluid at keep vein open 4. Dobutamine

Answer: 2 - Dopamine Explanations: Dopamine and IV fluid boluses increase preload, systemic vascular resistance, and circulating blood volume in patients with shock. Dopamine at high doses increases vasoconstriction and increases blood pressure. The effects of high doses of dopamine between 2 to 10 micrograms/kg/min on vasoconstriction are the result of its actions on the alpha- and beta-adrenergic receptors in the peripheral vasculature Lower doses of dopamine have opposite effects, including increasing excretion by the kidneys and increasing heart rate. Go to the next page if you knew the correct answer, or click the link image(s) below to further research the concepts in this question (if desired).

Research Concepts: Dopamine

We update eBooks quarterly and Apps daily based on user feedback. Please tap flag to report any questions that need improvement.

Question 713: A 45-year-old male is presented to the ER after a road traffic accident. On arrival to ER, he has a pulse of 134/min, blood pressure of 90/60 mmHg, and a respiratory rate of 28/min. On examination, his extremities are cold. Urethral catheterization is done in the ER. IV fluids are started. What is the best indicator of adequate capillary perfusion in response to treatment in this patient?

Choices: 1. Urine output 2. Blood pressure 3. Skin temperature 4. Pulse

Answer: 1 - Urine output Explanations: The best indicator that resuscitation is going well is urine output. However, there are also other laboratory parameters that one can use to assess the adequacy of resuscitation. Both base deficit and lactate levels can be used to follow resuscitation. The base deficit reflects the intensity of shock and oxygen delivery. Sublingual capnometry is a new tool used to assess organ perfusion. The device consists of a C02 sensor that detects levels of C02. Lactate levels are a reliable marker for hypoperfusion and indicate anaerobic metabolism. A failure to clear lactate levels within 24 hours after shock and resuscitation is a predictor of increased mortality. Blood pressure and pulse are not reliable indicators of capillary perfusion. Go to the next page if you knew the correct answer, or click the link image(s) below to further research the concepts in this question (if desired).

Research Concepts: Hemorrhagic Shock

We update eBooks quarterly and Apps daily based on user feedback. Please tap flag to report any questions that need improvement.

Question 714: A 75-year-old female patient presents from a nursing home after pulling out her feeding tube, which was placed approximately 6 months ago. Her medical history is significant for osteoporosis, unstable angina, and advanced dementia. Her vital signs are stable. What is the first procedure that should be attempted to give the patient a new feeding tube?

Choices: 1. Percutaneous endoscopic gastrostomy tube placement 2. Fluoroscopic gastrostomy tube placement 3. Bedside insertion of a new gastric tube 4. Open gastrostomy tube placement on the operating room

Answer: 3 - Bedside insertion of a new gastric tube Explanations: When a patient has had a feeding tube for a long period of time, usually more than a few months, the track is well established a new tube with an inflatable balloon is the first thing that should be attempted. If successful with bedside replacement of the feeding tube, many providers will obtain an xray where contrast is placed into the feeding tube to confirm placement into the stomach. There are many additional techniques that can be used to help successfully replace a feeding tube at the bedside. If unsuccessful bedside replacement, the patient will likely need to have a percutaneous endoscopic or fluoroscopically placed feeding tube. Go to the next page if you knew the correct answer, or click the link image(s) below to further research the concepts in this question (if desired).

Research Concepts: Feeding Tube

We update eBooks quarterly and Apps daily based on user feedback. Please tap flag to report any questions that need improvement.

Question 715: A 53-year-old male with a history of lung cancer presents to the emergency department with severe respiratory distress. Endotracheal intubation is performed. After successful intubation, the capnography waveform is reviewed. Which of the following values would indicate the correct endotracheal tube placement?

Choices: 1. 25 mmHg 2. 37 mmHg 3. 55 mmHg 4. 48 mmHg

Answer: 2 - 37 mmHg Explanations: The normal range of capnography is 35 mmHg to 45 mmHg. If the value falls into this range, post-intubation correlates with a properly positioned tube. If the capnography value were zero, this would indicate that the endotracheal tube was not in the airway. Most likely, this would indicate esophageal intubation, and repeat endotracheal intubation would need to be performed. Capnography assesses ventilation, which is the movement of air in and out of the lungs. It does not assess oxygenation. If a capnography reading is above 45 mmHg, the endotracheal tube is in the correct position. However, the patient will need more ventilation, and there will be changes in the ventilator machine. Go to the next page if you knew the correct answer, or click the link image(s) below to further research the concepts in this question (if desired).

Research Concepts: Airway Management

We update eBooks quarterly and Apps daily based on user feedback. Please tap flag to report any questions that need improvement.

Question 716: A 23-year-old man presents to the emergency department with high-grade fever, confusion, headache, and photophobia. He has recently come back from a trip to Southeast Asia. His symptoms were mild for the past three days but have flared up in the past couple of hours. He has had HIV for the last 5 years. On examination, the patient is visibly uncomfortable due to the bright lights. His thighs flex every time his head is lifted. His right eye is in the "down and out" position. A lumbar puncture is performed. Which of the following findings is most likely to be seen in this patient's cerebrospinal fluid (CSF)?

Choices: 1. A glucose level of 20 mg/dL 2. Neutrophil count of 1000 cells/microL 3. A glucose level of 120 mg/dL 4. A protein level of 20 mg/dL

Answer: 1 - A glucose level of 20 mg/dL Explanations: This patient most likely has tuberculous meningitis. (TBM), as evidenced by his symptoms of meningitis, possible exposure to tuberculosis (TB), and a history of HIV. TBM often presents with cranial nerve palsies as well. Tuberculous meningitis assessment is by obtaining cerebrospinal fluid (CSF) for analysis. Typically, the CSF reveals low glucose, elevated protein, and modestly elevated WBC count with a lymphocytic predominance. The CSF analysis most closely resembles the CSF analysis of viral meningitis. First-line antituberculous treatments have excellent CSF penetration. Treatment for TBM consists of two months of daily isoniazid (INH), rifampin (RIF), pyrazinamide (PZD), and either streptomycin (SM) or ethambutol (EMB). This regimen is then followed by 7 to 10 months of INH and RIF. Confirming the diagnosis of TB is a challenging diagnostic dilemma; this is especially true in resource-poor areas. Definitive diagnosis results from the identification of mycobacterium tuberculosis (MTB) in the CSF. Standard Ziehl-Neelsen acid-fast bacilli (AFB) identification smears from CSF are highly unreliable. A neutrophil count of 1000 cells/microL is seen in bacterial meningitis. Go to the next page if you knew the correct answer, or click the link image(s) below to further research the concepts in this question (if desired).

Research Concepts: Tuberculous Meningitis

We update eBooks quarterly and Apps daily based on user feedback. Please tap flag to report any questions that need improvement.

Question 717: A 16-year-old female is brought to the emergency department with the complaint of generalized paralysis. History taken from the friend reveals that she had some seafood in the evening. Otherwise, her past medical history is insignificant. Vital signs are pulse rate 95/min, blood pressure 135/85 mmHg, respirations 27/min, and temperature 98.6 F (37 °C). Physical examination reveals a generalized decreased muscle tone. What is the next step in the management of this patient?

Choices: 1. Sodium bicarbonate 2. Intravenous immunoglobulin 3. Respiratory support and supportive care 4. Calcium gluconate

Answer: 3 - Respiratory support and supportive care Explanations: Tetrodotoxin is a neurotoxin that is most commonly found in marine animals. The toxin blocks sodium channels leading to gastrointestinal, neurological, and cardiac symptoms in poisoned patients. There is currently no known antidote. A monoclonal antibody targeted against TTX (Anti-TTX) is available. No studies on efficacy have been published. Since there is no known antidote for tetrodotoxin, the management is interprofessional. All symptomatic patients must be monitored closely in the ICU and resuscitated according to the trauma ABCDE protocol. Specialists should be involved depending on the organ involved. Respiratory support and supportive care is the mainstay of treatment for TTX poisoning. There is no specific antidote. TTX is excreted through the urine. Hemodialysis may be required for patients with renal pathology. There is no specific treatment for TTX poisoning. For suspected tetrodotoxin poisoning, patients should be observed in the intensive care unit (ICU) for 24 hours due to some patients having a delayed onset of symptoms of up to 20 hours. Go to the next page if you knew the correct answer, or click the link image(s) below to further research the concepts in this question (if desired).

Research Concepts: Tetrodotoxin Toxicity

We update eBooks quarterly and Apps daily based on user feedback. Please tap flag to report any questions that need improvement.

Question 718: A 32-year old male commercial marine diver is evaluating a sailing vessel for repairs at 4-8 meters below sea level. He has no past medical history and takes no medications. He drinks alcohol socially and does not smoke tobacco products, vape, or use illicit drugs. His SCUBA equipment becomes entangled, and his regulator hose is damaged. He coughs, loses his regulator, and his face mask starts to fill with water. He rapidly ascends to the surface without stopping or exhaling. At the surface, he is unconscious, seizing, and lifeless. Advanced cardiac life support is initiated, and he is immediately transported to the local hospital. He regains consciousness, is confused, dizzy, and dysarthric. Chest x-ray reveals pneumomediastinum and bibasilar infiltrates, but no pneumothorax. Electrocardiogram reveals sinus tachycardia without ischemia. An intravenous crystalloid solution is infusing, and supplemental oxygen at 100% non-rebreather is in place. Which of the following is the next best step in the management of this patient?

Choices: 1. Broad-spectrum antibiotic therapy 2. Hyperbaric oxygen recompression therapy 3. Computed tomography (CT) scan of the head 4. Electroencephalogram (EEG) and intravenous levetiracetam

Answer: 2 - Hyperbaric oxygen recompression therapy Explanations: Air-gas emboli are formed when pneumomediastinum air is forced into the capillaries, pulmonary veins, and central circulation. These bubbles collect in the heart or brain and obstruct blood flow. Initial presentation of cerebral arterial gas embolism is the loss of consciousness, seizure, and/or cardiac arrest due to lack of blood flow in the large vessels or from air gas emboli in the brainstem. Air-gas emboli can also cause headaches, confusion, numbness, tingling, paresthesia, fatigue, hemiplegia, paralysis, aphasia, or any stroke symptom. The damaged epithelial and endothelial cells release inflammatory mediators. This damage to the cells leads to leaky vessels that cause swelling and edema. In the brain, this manifests as cerebral edema. The need for recompression and hyperbaric oxygen therapy needs to efficiently and accurately assessed for the patient. Prompt treatment within the first two hours provides the most benefit and resolution of symptoms. Delays in recompression and hyperbaric oxygen therapy of more than 6 hours are associated with worse outcomes. Go to the next page if you knew the correct answer, or click the link image(s) below to further research the concepts in this question (if desired).

Research Concepts: Pulmonary Barotrauma

We update eBooks quarterly and Apps daily based on user feedback. Please tap flag to report any questions that need improvement.

Question 719: A 27-year-old woman with a history of depression, insomnia, anemia, and fibromyalgia is brought to the emergency department after reported ingestion of an unknown toxin about 45 minutes before arrival. She texted her family to say she was going to kill herself by taking a bottle of one of her medications. Her family called 911, and she was promptly taken to the emergency department. On arrival, her blood pressure is 150/85 mmHg, temperature 37.0 C (98.6 F), and her pulse is 102/min. She is awake but uncooperative with providing a history of what she ingested. She is quiet with poor eye contact, but she is speaking clearly and appears to be protecting her airway. Chart review of a recent outpatient visit provides a medication list, including acetaminophen, amitriptyline, ferrous sulfate, paroxetine, and tramadol. For which of the following medications of hers would the administration of activated charcoal be ineffective in reducing absorption?

Choices: 1. Acetaminophen 2. Amitriptyline 3. Ferrous sulfate 4. Paroxetine

Answer: 3 - Ferrous sulfate Explanations: Activated charcoal best adsorbs toxins in their nonionized forms. Polar, water-soluble molecules are less likely to be adsorbed. Due to the pharmacodynamics of activated charcoal, it best absorbs nonpolar, poorly water-soluble organic toxins. Most ingested toxins will have decreased systemic absorption in the presence of activated charcoal, including acetaminophen, aspirin, barbiturates, tricyclic antidepressants, theophylline, phenytoin, and a majority of inorganic and organic materials. Activated charcoal does not effectively adsorb alcohols, metals such as iron and lithium, electrolytes such as magnesium, potassium, or sodium, and acids or alkalis due to the polarity of these substances. In this case, iron supplementation (ferrous sulfate) would not be bound by activated charcoal as it is an ionized toxin. There would be no benefit to activated charcoal administration in the setting of an iron overdose. Go to the next page if you knew the correct answer, or click the link image(s) below to further research the concepts in this question (if desired).

Research Concepts: Activated Charcoal

We update eBooks quarterly and Apps daily based on user feedback. Please tap flag to report any questions that need improvement.

Question 720: A 22-year-old man with a history of cystic fibrosis-associated liver disease presents with worsening abdominal distension and jaundice. On physical examination, the patient's temperature is noted to be 37.7 °C, heart rate of 88/min and blood pressure 100/60mmHg. The patient is noted to have scleral icterus, palmar erythema, and moderate ascites. Laboratory results show hemoglobin 10.0 g/dL (13-17 g/dL), platelet count 69 x 10^9/L (150-400 x 10^9/L), serum creatinine 2.4 mg/dL (0.8-1.3 mg/dL), sodium 133 mmol/L (135-145 mmol/L), total bilirubin 4.2 mg/dl (2-20 µmol/L), albumin 2.4 g/dL (35-50 g/L), AST 110 U/L (5-30 U/L), ALT 100U/L (5-30 U/L) ,ALP 289 U/L (50-100 U/L) and INR of 2.1. An ultrasound examination of the abdomen shows significant ascites and nodular liver consistent with cirrhosis. A CT scan of the abdomen is negative for biliary ductal dilatation or biliary obstruction. What is the most appropriate next step in the management of this patient?

Choices: 1. Arrange an urgent magnetic resonance cholangiopancreatography (MRCP) examination 2. Schedule the patient for an outpatient endoscopic retrograde cholangiopancreatography (ERCP) examination 3. Transfer the patient to ICU for close supportive management 4. Refer the patient to a transplant center for liver transplantation

Answer: 4 - Refer the patient to a transplant center for liver transplantation Explanations: Liver transplantation confers a substantial survival advantage in cystic fibrosis liver disease (CFLD) patients but the clinical criteria and timing of liver transplantation in CFLD are not well established. Debray et al recommend liver transplantation should be considered if CFLD patients demonstrate progressive hepatic failure characterized by progressive hypoalbuminemia and coagulopathy, worsening jaundice and ascites, variceal bleeding not controlled by conventional means, hepatopulmonary and portopulmonary syndromes, severe malnutrition despite intensive nutritional provision, worsening pulmonary function (FEV1/FVC 50%) and declining quality of life linked to liver disease. This patient would benefit from a referral to a transplant center considering he has progressive hepatic dysfunction characterized by a high MELD score (30). Combined lung and liver transplantation (CLLT) should be considered for patients with advanced pulmonary and liver disease. Go to the next page if you knew the correct answer, or click the link image(s) below to further research the concepts in this question (if desired).

Research Concepts: Cystic Fibrosis And Liver Disease

We update eBooks quarterly and Apps daily based on user feedback. Please tap flag to report any questions that need improvement.

Question 721: A 40-year-old male presents to the clinic with worsening shortness of breath (stage IV NYHA). He has been admitted twice in the last one year for ST-segment elevation myocardial infarction. During both the admissions, work-up with coronary artery catheterization showed normal coronary arteries, blood alcohol levels > 0.2 g/dL, and urine toxicology positive for cocaine. An echocardiogram done in the last admission, three months ago, showed left ventricular ejection fraction (LVEF) 15%. The rest of the laboratory work-up showed creatinine 0.9 mg/dL, hemoglobin 10 g/dL, hematocrit 30%, platelets 150,000/mircoliter. INR 1.2. Treatment was initiated with lisinopril and spironolactone on discharge. On evaluation in the clinic today, he mentions he is compliant with his medications and wants to explore other treatment options. An echocardiogram is repeated, which continues to show LVEF of 17%. He continues to endorse the intermittent use of alcohol and recreational drugs. Which of the following is a contraindication to LVAD placement in the patient?

Choices: 1. Creatinine 0.9 mg/dL 2. Hemoglobin 10 g/dL 3. INR 1.2 4. Blood alcohol level 0.2 and positive urine toxicology for cocaine

Answer: 4 - Blood alcohol level 0.2 and positive urine toxicology for cocaine Explanations: End-stage renal disease (glomerular filtration rate 30 or creatinine clearance 30) is a contraindication to LVAD placement. Active severe bleeding, chronic thrombocytopenia, and refusal to blood transfusions is a contraindication to LVAD placement. Severe liver disease (bilirubin 2.5 or international normalized ratio > 2.0 with cirrhosis or portal hypertension) is a contraindication to LVAD placement. Psychosocial elements such as evidence of ongoing alcohol, smoking or drug use or dependency, inability to adhere to medical regimen, active mental illness, and poor social support are contraindications to LVAD placement. Go to the next page if you knew the correct answer, or click the link image(s) below to further research the concepts in this question (if desired).

Research Concepts: Left Ventricular Assist Devices

We update eBooks quarterly and Apps daily based on user feedback. Please tap flag to report any questions that need improvement.

Question 722: A 44-year-old man is evaluated for dry cough and progressive dyspnea for the last six days. Dyspnea is limiting his ability to exercise. He is also reporting fevers, myalgia, and headache. His temperature is 100.4 F (38 C), O2 saturation is 92% on 3 liters via nasal cannula, heart rate is 122/min, and respiratory rate is 26/min. The chest radiograph reveals bilateral interstitial infiltrates. On physical exam, he has bilateral crackles all over the lung field. A high-resolution CT scan of the chest shows extensive ground-glass changes bilaterally. One hour after the initial presentation, his breathing gets worse, and O2 saturation drops. The patient gets intubated and placed on mechanical ventilation. What is the best next diagnostic test?

Choices: 1. Test for influenza A and B 2. Bronchoalveolar lavage 3. Bronchoscopy 4. Schedule for lung biopsy

Answer: 1 - Test for influenza A and B Explanations: Acute interstitial pneumonia (AIP) is an acute, rapidly progressive idiopathic pulmonary disease that often leads to fulminant respiratory failure and acute respiratory distress syndrome (ARDS). Symptoms usually start as viral-like prodrome followed by shortness of breath with cough, and fever which then progress rapidly to acute respiratory distress. The physical exam is non-specific with hypoxia, tachypnea, and bilateral diffuse crackles. Many of these patients are severely hypoxemic and require mechanical ventilation. It is an idiopathic disease with unknown etiology, and diagnosis is only confirmed after excluding other causes of ARDS. Influenza screening is important, as well as serology for atypical organisms and fungus. Excluding cardiac causes of pulmonary edema is essential. Echocardiography is needed to rule out underlying cardiomyopathy or valve dysfunction. Microbiologic workup should include blood and sputum culture. Legionella is an atypical organism that can be detected on a urine antigen test. Bronchoscopy with bronchoalveolar lavage (BAL) is needed to exclude diffuse alveolar hemorrhage (DAH), eosinophilia, or malignant infiltrates but is not the initial step. If all the previous workup fails to achieve an alternative diagnosis, lung biopsy might be indicated. Go to the next page if you knew the correct answer, or click the link image(s) below to further research the concepts in this question (if desired).

Research Concepts: Acute Interstitial Pneumonia

We update eBooks quarterly and Apps daily based on user feedback. Please tap flag to report any questions that need improvement.

Question 723: A 35-year old female is brought to the emergency department due to an altered level of consciousness after an intentional drug overdose. She was diagnosed with multiple sclerosis 5 years ago and was recently prescribed a medication that acts on central alpha 2 receptors to treat her symptoms of muscle spasticity. The physical examination is significant for a pulse rate of 50/min, a blood pressure of 90/50 mm Hg, respiratory rate of 10/min, and diminished deep tendon reflexes. Despite initial resuscitation, her vitals fail to improve. Administration of which of the following is the next best step?

Choices: 1. Intralipid 2. Sodium bicarbonate 3. Naloxone 4. Norepinephrine

Answer: 4 - Norepinephrine Explanations: Alpha-agonist toxicity may occur accidentally or intentionally. The primary mechanism of the toxicity of alpha agonists is through central alpha-2 agonism. Stimulation of central alpha-2 receptors causes decreased secretion of catecholamines through a negative feedback mechanism. Norepinephrine is indicated for bradycardia and hypotension secondary to central alpha-2 agonist toxicity that does not respond to stimulation and intravenous fluids. Norepinephrine provides direct replacement of the inhibited catecholamines in this setting. Go to the next page if you knew the correct answer, or click the link image(s) below to further research the concepts in this question (if desired).

Research Concepts: Alpha Receptor Agonist Toxicity

We update eBooks quarterly and Apps daily based on user feedback. Please tap flag to report any questions that need improvement.

Question 724: A 75-year-old man with chronic atrial fibrillation presents to the clinic for a follow-up. He has been taking digoxin 0.25 mg per day for 10 years. He recently had a diagnosis of peptic ulcer disease with positive H. pylori and was prescribed clarithromycin and omeprazole. He has had increased nausea, anorexia, fatigue, and exercise intolerance. Which of the following is the most likely cause of his complaints?

Choices: 1. Inhibition of liver function by omeprazole 2. Decreased protein-binding of digoxin 3. Decreased inactivation of digoxin by enteric bacteria 4. Decreased fluid intake

Answer: 3 - Decreased inactivation of digoxin by enteric bacteria Explanations: In 10% to 15% of patients receiving digoxin, a significant amount of the drug can get inactivated in the gastrointestinal tract by enteric bacteria, primarily Eubacterium lentum. Classic drug-drug interaction occurs with the macrolides, which sterilize the gastrointestinal tract of this bacterium and cause increased digoxin absorption. Of the macrolides, clarithromycin is the most cited offending medication and is often part of the tripe cocktail for H. pylori. Malnutrition can lead to decreased protein and albumin levels, but digoxin is not highly protein-bound. Other drugs that can potentially interact with digoxin include quinidine, verapamil, diltiazem, amiodarone, and spironolactone. Omeprazole does not interact with digoxin to cause toxicity. Providers prescribing medications to patients taking digoxin should be aware of the possible drug interactions that exist and have an understanding of the pharmacokinetics of digoxin. Fluid intake is important with patients on digoxin, but often with chronic heart failure, fluid restriction is needed. Go to the next page if you knew the correct answer, or click the link image(s) below to further research the concepts in this question (if desired).

Research Concepts: Cardioactive Steroid Toxicity

We update eBooks quarterly and Apps daily based on user feedback. Please tap flag to report any questions that need improvement.

Question 725: A 65-year-old man with a past medical history of hypertension and diabetes mellitus presents to the emergency department with an inability to move either of his lower extremities and loss of pain and temperature sensation below his waist. His symptoms came on suddenly approximately three hours ago. A full neurologic exam shows 0/5 power in both lower extremities, but his proprioceptive and fine touch sensations to be normal. However, after an hour in the emergency department, he begins to complain of suprapubic discomfort, and an overly distended bladder on ultrasound is discovered. Which of the following additional complications is most likely to occur in this patient?

Choices: 1. Tachycardia 2. Ankle clonus 3. Hoffman sign 4. Hypotension

Answer: 4 - Hypotension Explanations: This patient’s clinical presentation is characteristic of anterior cord syndrome, caused by ischemia or infarction of the anterior spinal artery supplying the anterior two-thirds of the spinal cord. The lateral horns of the spinal cord span from T1 to L2, and can be affected in anterior cord syndrome depending on the location of the lesion. The lateral horns house the neuronal cell bodies of the sympathetic nervous system. Damage to this region of the spinal cord can lead to autonomic dysfunction. This patient has an overdistended bladder, which indicates that he is experiencing autonomic dysfunction. Another common manifestation of autonomic dysfunction in anterior cord syndrome is hypotension. Go to the next page if you knew the correct answer, or click the link image(s) below to further research the concepts in this question (if desired).

Research Concepts: Anterior Cord Syndrome

We update eBooks quarterly and Apps daily based on user feedback. Please tap flag to report any questions that need improvement.

Question 726: A 45-year-old man with a history of alcohol use disorder is brought to the hospital for severe epigastric pain and nausea. He has had eight similar admissions in the past two years. Vital signs show a blood pressure of 102/64 mmHg, pulse 94/min, respiratory rate 20/min, and SpO2 95% on room air. Initial labs show hemoglobin 11.3 g/dL, WBC count 17000/microL, platelets 71000/microL, blood glucose 284 mg/dL, serum lipase 290 IU/L, serum AST 475 IU/L, serum calcium 8.1 mg/dL, PO2 80 mmHg, base deficit 3 mEq/L, serum creatinine 1.1 mg/dL, serum BUN 16 mg/dL, and serum LDH 640 IU/L. The patient is given IV normal saline and admitted to the hospital. Ranson criteria are being used to assess the patient's mortality. Which of the following is the minimum time required to reassess the patient's labs to complete the criteria?

Choices: 1. 12 hours 2. 24 hours 3. 48 hours 4. 72 hours

Answer: 3 - 48 hours Explanations: Ranson criteria are used to predict the severity and mortality of acute pancreatitis. Five parameters are assessed on admission, and the other six are assessed at 48 hours postadmission. One point is given for each positive parameter for a maximum score of 11. The modified criteria have a max score of 10. Five parameters assessed on admission and the other 5 at the 48-hour mark. The criteria with 11 parameters are used to assess the severity of alcoholic pancreatitis. The 5 parameters on admission are age older than 55 years, WBC count greater than 16,000 cells/mm^3, blood glucose greater than 200 mg/dL (11 mmol/L), serum AST greater than 250 IU/L, and serum LDH greater than 350 IU/L. At 48 hours, the remaining 6 parameters are: serum calcium less than 8.0 mg/dL (less than 2.0 mmol/L), hematocrit fall greater than 10%, PaO2 less than 60 mmHg, BUN increased by 5 or more mg/dL (1.8 or more mmol/L) despite intravenous (IV) fluid hydration, base deficit greater than 4 mEq/L, and sequestration of fluids greater than 6 L. Score Interpretation: 0 to 2 points: mortality 0% to 3%, 3 to 4 points: 15%, 5 to 6 points: 40%, 7 to 11: nearly 100%. Go to the next page if you knew the correct answer, or click the link image(s) below to further research the concepts in this question (if desired).

Research Concepts: Ranson Criteria

We update eBooks quarterly and Apps daily based on user feedback. Please tap flag to report any questions that need improvement.

Question 727: A 51-year-old patient with alcohol use disorder is seen in the ER with complaints of a fever, chills, night sweats and anorexia. He says he has had productive thick sputum for the past 2 days and has a persistent cough. He describes his sputum as red and viscous. He has a WBC of 18 and a left shift. The rest of the exam is unremarkable. Chest x-ray reveals a possible consolidation. What organism is most likely causing this infection?

Choices: 1. Mycobacterium tuberculosis 2. Staphylococcus aureus 3. Klebsiella pneumoniae 4. Aspergillosis

Answer: 3 - Klebsiella pneumoniae Explanations: Patients with Klebsiella typically present with an acute onset of high fever, chills, and flulike symptoms. Most patients with Klebsiella will develop a productive cough with abundant, thick, tenacious, and blood-tinged sputum sometimes called currant jelly sputum. In the U.S., 66% of patients have alcohol use disorder. In some parts of Asia it causes up to 40% of community acquired pneumonia in the elderly. Mortality is from 50 to 100%. Go to the next page if you knew the correct answer, or click the link image(s) below to further research the concepts in this question (if desired).

Research Concepts: Bacterial Pneumonia

We update eBooks quarterly and Apps daily based on user feedback. Please tap flag to report any questions that need improvement.

Question 728: A 65-year-old man with a past medical history of heart failure with reduced ejection fraction (HFrEF) and morbid obesity is brought to the emergency department after a head trauma after a fall at home. The patient is poorly responsive. Vital signs show blood pressure 110/60 mmHg, pulse 110/min, respiratory rate 8/min, temperature 98.6 F, and SpO2 94% on room air. The patient is admitted to the ICU. Transthoracic echocardiography is planned to assess the patient's fluid status. However, the patient is found to have poor cardiac windows, and cardiac function and volume status could not be assessed. Which of the following is the next best step in the management of this patient?

Choices: 1. Transesophageal echocardiography 2. Endotracheal intubation 3. Fluid resuscitation 4. Vasopressors

Answer: 2 - Endotracheal intubation Explanations: The patient should be intubated due to poor mental status with respiratory depression to help ventilation. Transesophageal echocardiography is absolutely contraindicated in a patient with respiratory depression. The airway should be secured before making any attempts for TEE. The decision between IVF and vasopressors can be made once the patient has a secure airway. Go to the next page if you knew the correct answer, or click the link image(s) below to further research the concepts in this question (if desired).

Research Concepts: Intraoperative Echocardiography

We update eBooks quarterly and Apps daily based on user feedback. Please tap flag to report any questions that need improvement.

Question 729: A 72-year-old man is admitted to the emergency department (ED) with a complaint of tearing chest pain that started about an hour ago and has progressively increased in severity since that time. He states he has never experienced pain like this before. He rates his pain as a 9 on a scale of 0-10. He has a past medical history of well-controlled hypertension, atherosclerosis, and renal failure for which he undergoes dialysis. His BMI is 43. Imaging obtained in the ED confirms a diagnosis of aortic dissection, and the patient is prepared for transfer to the operating room for emergent repair. The anesthetist completes his pre-operative assessment and designates this patient as ASA-PS VE. Which aspect of this patient’s history most likely classifies him as ASA-PS VE?

Choices: 1. Diagnosis of acute aortic dissection 2. History of renal failure and dialysis 3. Well-controlled hypertension 4. BMI of 43

Answer: 1 - Diagnosis of acute aortic dissection Explanations: Acute aortic dissection is a constant threat to life and an emergency. The patient is not expected to live without surgical intervention. This diagnosis would designate this patient as ASA-PS V, while the emergency presentation justifies the "E" added to the classification. This is the most significant active problem this patient is experiencing and is the top priority when assessing this patient. History of renal failure would cause this patient to be classified incorrectly as ASA-PS III. The acute aortic dissection is the most significant active problem, therefore the patient would be more correctly classified according to the presence of aortic dissection. Wellcontrolled hypertension alone would designate this patient as ASA-PS II. The presence of an emergent aortic dissection takes precedence when assessing a patient and assigning the proper ASA-PS classification. BMI of 43 is considered morbid obesity, which alone would classify a patient as ASA-III. Although this patient is morbidly obese, the aortic dissection is emergent and takes precedence as the most significant active problem. It would be more correct to classify the patient as ASA VE. Go to the next page if you knew the correct answer, or click the link image(s) below to further research the concepts in this question (if desired).

Research Concepts: American Society of Anesthesiologists Staging

We update eBooks quarterly and Apps daily based on user feedback. Please tap flag to report any questions that need improvement.

Question 730: A 26-year-old female developed seizures in the recovery room following her wisdom teeth extraction under general anesthesia. She was febrile and was hemodynamically unstable. She developed tachycardia and showed minimal respiratory efforts. She was intubated and given was lorazepam. Paramedics transferred her to the emergency room from the surgical center. Examination showed blood pressure of 155/90 mm Hg, heart rate of 135, and temperature of 42.2 degrees Celsius. She was unresponsive to pain. Her pupils were 3 mm equal and reactive to light. There were increased muscle tone and hyperreflexia. Her ECG showed sinus tachycardia with peaked T-waves. Which is the next plan in managing the patient?

Choices: 1. Paralyze and ventilate 2. Creatinine Kinase 3. Dantrolene sodium 4. Escalate her antiepileptic medication

Answer: 3 - Dantrolene sodium Explanations: The patient most likely has malignant hyperthermia, which is a hypermetabolic response to some potent volatile anesthetic gases. In susceptible individuals, with a mutation in the Ryanodine receptor 1 (RYR-1) gene, some medications induce the excessive release of stored calcium ions within muscle cells. The resulting increase in calcium concentrations within the cells leads to unopposed muscle contractions. This leads to hyperthermia and metabolic acidosis. Dantrolene depresses excitation-contraction coupling in skeletal muscle by acting as a receptor antagonist to the ryanodine receptor and decreasing free intracellular calcium concentration. Hypothermia and antiepileptics are only adjunctive treatment in malignant hyperthermia. Go to the next page if you knew the correct answer, or click the link image(s) below to further research the concepts in this question (if desired).

Research Concepts: Malignant Hyperthermia

We update eBooks quarterly and Apps daily based on user feedback. Please tap flag to report any questions that need improvement.

Question 731: A 60-year-old man with an unknown past medical history is brought to the emergency department after being found unconscious on a park bench. On examination, the patient is lethargic but arousable to sternal rub. Vital signs reveal a blood pressure of 100/60 mmHg, pulse rate of 60 per minute, respiratory rate of 24 per minute, and temperature of 33 C (91.4 F). Physical exam shows a middle-aged gentleman with moist clothing that smells of alcohol. There are no signs of trauma, and the rest of his physical exam is unremarkable. His wet clothing is removed, and he is started on warm IV fluids and placed in a warming blanket. After three hours, the patient's temperature remains 33 C (91.4 F). Which of the following is the best next step in the management of this patient?

Choices: 1. Pleural irrigation 2. Venoarterial extracorporeal membrane oxygenation 3. Antibiotics and glucocorticoids 4. Cardiopulmonary bypass

Answer: 3 - Antibiotics and glucocorticoids Explanations: Mild hypothermia is defined by an estimated core temperature of 32 to 35 C (90 to 95 F). Moderate hypothermia is an estimated core temperature of 28 to 32 C (82 to 90 F). Severe hypothermia is an estimated core temperature of 28 C (82 F). Passive rewarming is the modality of choice for mild hypothermia. It may also be used as an adjunct for a patient undergoing aggressive rewarming for moderate to severe hypothermia. Active external rewarming is indicated in moderate to severe hypothermia and for a patient with mild hypothermia who is unstable, lacks physiological reserves, or failed initial treatment. Active internal rewarming is indicated in severe hypothermia or a patient with moderate hypothermia who failed to respond to less aggressive techniques. Venoarterial extracorporeal membrane oxygenation is used in extreme cases (cardiac arrest, frozen limbs) or if all other measures before have failed. A patient that does not adequately respond to rewarming or has symptoms that do not match the severity of their hypothermia should have further testing to rule out other possible causes of hypothermia such as infection, hypothyroidism, hypoglycemia, etc. Empirically treating with antibiotics and glucocorticoid may provide more benefits than harm as infection and adrenal insufficiency are possible etiologies of hypothermia that do not respond to adequate rewarming. The patient in this question stem has mild hypothermia that is not responding to adequate treatment. Pleural irrigation is an active internal rewarming technique reserved for patients with moderate or severe hypothermia who failed to respond to less aggressive techniques. Venoarterial extracorporeal membrane oxygenation and cardiopulmonary bypass are highly invasive, so they are reserved for extreme cases or if all other measures before have failed. Go to the next page if you knew the correct answer, or click the link image(s) below to further research the concepts in this question (if desired).

Research Concepts: Hypothermia

We update eBooks quarterly and Apps daily based on user feedback. Please tap flag to report any questions that need improvement.

Question 732: A construction worker fell 20 feet (6 m) and suffered a head injury. In the emergency department, he is unresponsive and posturing with his forearms flexed close to his body and legs rigidly extended with internal rotation and plantar flexion. What is the Glasgow coma scale motor score allocated for such a response seen in the patient?

Choices: 1. Two 2. Three 3. Four 4. Five

Answer: 2 - Three Explanations: Decorticate posturing involves the arms flexed close to the body with internal rotation and the legs stiffly extended. Decorticate posturing indicates an injury above the red nucleus in the midbrain. The red nucleus governs the flexion of the upper limbs and extension of the lower limbs. The decorticate posturing is given a score of three while assessing motor response. The decerebrate posturing has a motor score of two. The motor score of one indicates no response to painful stimuli in the patient. Go to the next page if you knew the correct answer, or click the link image(s) below to further research the concepts in this question (if desired).

Research Concepts: Decorticate Posturing

We update eBooks quarterly and Apps daily based on user feedback. Please tap flag to report any questions that need improvement.

Question 733: A 16-year-old male patient is brought to the hospital by his parents with complaints of persistent cough for one week and excessive use of his blue inhaler. Previously, he had been well on fluticasone/salmeterol inhaler and montelukast daily. His parents are concerned that he may not have taken his medication because of a domestic quarrel. On examination, his respiratory rate is 32/min, peak expiratory flow rate (PEFR) is 235 L/min, (predicted is 590 L/min), and oxygen saturation is 89%. He is started on oxygen therapy, salbutamol and ipratropium bromide, nebulizers, and IV hydrocortisone. Later in the evening, he is found to have pO2 of 7.6 kPa, pCO2 of 5.9 kPa, pH of 7.36, respiratory rate of 15/min, and PEFR of 230 L/min. What is the most appropriate next step in the management of this patient?

Choices: 1. IV aminophylline 2. Consult ICU team 3. IV magnesium 4. Back to back salbutamol nebulization

Answer: 2 - Consult ICU team Explanations: If a patient has received 3 doses of an inhaled bronchodilator and shows no response, the factors that should be used to determine admission are the severity of airflow obstruction, duration of asthma, response to medications, adequacy of home support, any mental illness. The most appropriate next step, in this case, would be to involve the ICU team. Despite management, his PEFR has declined further and he is making a less respiratory effort now. There is evidence of hypoxemia with hypercapnia on arterial blood gases which should be taken as an indication to include ICU physicians. While back to back nebulizations, IV magnesium and IV aminophylline are all needed, calling the ICU team is of utmost importance. Mortality is higher when the ICU team is involved late, such as in peri-arrest situations. Go to the next page if you knew the correct answer, or click the link image(s) below to further research the concepts in this question (if desired).

Research Concepts: Asthma

We update eBooks quarterly and Apps daily based on user feedback. Please tap flag to report any questions that need improvement.

Question 734: A 32-year-old right-handed lady with a past medical history of systemic lupus erythematosus (SLE) presents to the emergency department with the symptoms of nausea, dizziness, headache, confusion, vertigo, dysarthria, and bilateral central vision loss with the onset of symptoms two hours ago. On examination, the temperature is 37 C, pulse rate is 90/min, blood pressure is 165/73 mmHg, and the respiratory rate is 36/min breaths per minute. The patient is alert but confused and complaining of a headache. Her blood glucose level is 93 mg/dL, and the National Institute of Health stroke scale is 7. CT scan of the head without contrast is negative for an acute bleed, and subsequent CT angiogram of head and neck shows a large vessel occlusion (LVO) at the basilar tip. Which of the following steps is recommended at this time?

Choices: 1. MRI scan of head and neck to characterize the infarct 2. CT perfusion scan to measure the extent of the infarct 3. Mechanical thrombectomy 4. Antiplatelet medications

Answer: 3 - Mechanical thrombectomy Explanations: MRI scan of head and neck is not indicated at this time, and the patient should immediately receive tissue plasminogen activator (tPA) such as alteplase and transferred to an angiography suite for mechanical thrombectomy (MT). At this point, there is no need to delay the treatment with unnecessary testing, and the patient should immediately receive alteplase. Mechanical thrombectomy (MT) is recommended in this case based on the recent DAWN and DEFUSE-3 trials for anterior circulation. Everyone should be considered for alteplase administration as per NINDS trial results published in 1993, which showed significant mortality and outcomes benefits of alteplase administration if the patient has arrived in the emergency department (ED) within the three hours window and has no contraindication to alteplase. The patient should first receive alteplase as she is within the tPA window. Although after tPA administration, the patient should be moved to an angiography suite for MT as the symptom onset is within 24 hours, and there is an LVO at the basilar tip, which would require MT. Go to the next page if you knew the correct answer, or click the link image(s) below to further research the concepts in this question (if desired).

Research Concepts: Basilar Artery Infarct

We update eBooks quarterly and Apps daily based on user feedback. Please tap flag to report any questions that need improvement.

Question 735: A patient diagnosed is diagnosed with diabetes insipidus. Appropriate fluids are started. Which of the following medications is given?

Choices: 1. Glipizide 10 mg intravenously, daily 2. Glipizide 10 mg by mouth, daily 3. Vasopressin 10 units intravenously, daily 4. Vasopressin 5-10 units intramuscularly or subcutaneously, every 8 to 12 hours

Answer: 4 - Vasopressin 5-10 units intramuscularly or subcutaneously, every 8 to 12 hours Explanations: Vasopressin is an antidiuretic hormone replacement drug. It may be given intramuscularly, subcutaneously, and nasally to treat diabetes insipidus, a disorder of the pituitary gland. Adult dosing is 5 to 10 units intramuscularly or subcutaneously every 8 to 12 hours in the acute management of diabetes insipidus. Pediatric dosing is 2.5 to 10 units intramuscularly, subcutaneously, or intranasally every 8 to 12 hours. The dose is titrated based on serum sodium and osmolarity, urine output, and overall fluid balance in both adults and pediatric clients. Normally, the hypothalamus produces antidiuretic hormone, which accumulates in the posterior pituitary. Glipizide is a therapy for diabetes mellitus, not diabetes insipidus. Vasopressin is not given intravenously. Go to the next page if you knew the correct answer, or click the link image(s) below to further research the concepts in this question (if desired).

Research Concepts: Diabetes Insipidus

We update eBooks quarterly and Apps daily based on user feedback. Please tap flag to report any questions that need improvement.

Question 736: A 17-year-old man is brought to the emergency department by his friends with bizarre behavior. They mention that the patient has been saying that he has been selected by the “divine authorities to fight the evil in this world.” The patient has stopped attending classes and stays in his room. He has not been eating or sleeping in the past week. They mention that he has not been using drugs. The patient's interview supports this history. Vitals are within normal limits. On examination, the patient appears disorganized, disheveled, has a pressured speech, and is pacing across the room. Laboratory investigations reveal no abnormalities. He is admitted after a diagnosis is made and started on appropriate treatment. Over the next week, the patient becomes calmer. Eleven days after admission, he complains of a sudden increase in sweating and stiffness of his muscles. Vital signs are temperature 102 F (38.8 C), blood pressure 162/91 mm Hg, pulse 126/min, and respiratory rate of 24/min. All medications are stopped and intravenous fluids are started. What medication should be given next if the patient does not improve with these interventions?

Choices: 1. Cyproheptadine 2. Benztropine 3. Diphenhydramine 4. Dantrolene

Answer: 4 - Dantrolene Explanations: The patient initially had features of mania and was most likely treated with antipsychotics. The patient has most likely developed the neuroleptic malignant syndrome. Discontinuing the offending agent is paramount, followed by supportive therapy. This includes aggressive cooling and correction of volume deficits and any electrolyte imbalances. Dantrolene is a muscle relaxant and acts on the ryanodine receptors. It is specific for the neuroleptic malignant syndrome. Cyproheptadine is used for serotonin syndrome. Benztropine and diphenhydramine are useful for antipsychotic-induced extrapyramidal symptoms. Go to the next page if you knew the correct answer, or click the link image(s) below to further research the concepts in this question (if desired).

Research Concepts: Neuroleptic Malignant Syndrome

We update eBooks quarterly and Apps daily based on user feedback. Please tap flag to report any questions that need improvement.

Question 737: A 36-year-old man is brought by paramedics after being hit by a car. After securing the airway and breathing, two large-bore intravenous lines are attached, and the patient is started on normal saline. Vital signs after 3 liters of normal saline include respiratory rate 18 per minute, heart rate 110 beats per minute, and blood pressure 80/55 mmHg. Examination shows a Glasgow coma scale of 9 with cold extremities and a weak tachycardic pulse. A FAST scan shows intraabdominal fluid. What is the most appropriate next step in the management of this patient's hypovolemia?

Choices: 1. Give packed red blood cells instead of normal saline 2. Start the patient on colloids 3. Continue IV normal saline 4. Perform emergency laparotomy

Answer: 4 - Perform emergency laparotomy Explanations: This patient is suffering from acute hypovolemia. Acute blood loss results in peripheral vasoconstriction causing cold extremities. After an acute trauma, patients who do not respond to normal saline bolus are called nonresponders. Emergency laparotomy is necessary for non-responders, so the bleeding source may be identified and repaired. Go to the next page if you knew the correct answer, or click the link image(s) below to further research the concepts in this question (if desired).

Research Concepts: Hypovolemic Shock

We update eBooks quarterly and Apps daily based on user feedback. Please tap flag to report any questions that need improvement.

Question 738: A 32-year-old woman with a past medical history of factor V Leiden presents to the emergency department with complaints of sudden-onset weakness in her right arm and leg for the past 3 hours. Glasgow coma scale score is 15/15, and the NIH stroke scale is 9. The patient has recently started taking combination oral contraceptive pills. Vitals show a temperature of 38.4 C, blood pressure 130/80 mmHg, heart rate 98/min, and respiratory rate 17/min. Non-contrast CT imaging of the brain shows no hemorrhage. Initial labs are shown below. Patient value Reference range Hemoglobin 6.9 g/dL 11.5-16 g/dL 4100WBC count 8800/microL 10900/microL 150000Platelet count 38000/microL 400000/microL Creatinine 1.7 mg/dL 0.8-1.4 mg/dL Total bilirubin 5.8 mg/dL 0.3-1.0 mg/dL LDH 520 IU/L 90-230 IU/L Sodium 139 mEq/L 134-144 mEq/L Potassium 4.8 mEq/L 3.6-5.0 mEq/L Glucose 142 mg/dL 65-100 mg/dL Which of the following is the next best step in the management of this patient?

Choices: 1. Intravenous alteplase 2. Aspirin 3. Plasma exchange therapy 4. Nicardipine

Answer: 3 - Plasma exchange therapy Explanations: This scenario describes a case of thrombotic thrombocytopenic purpura (TTP). TTP is a type of microangiopathic hemolytic anemia that classically has been characterized by the pentad of fever, thrombocytopenia, hemolytic anemia, renal dysfunction, and neurologic dysfunction. TTP can be either congenital or acquired. Acquired TTP is more common than the congenital type and is caused by autoantibodies targeting ADAMTS13. Antiplatelet drugs, immunosuppressive agents, HIV, estrogen-containing birth control, and pregnancy are the most commonly listed triggers for ADAMTS13 autoantibody formation, causing acquired TTP. Factor V Leiden has been implicated as a risk factor, although rare, for TTP. Although renal failure is part of the classic presentation, in practice, it is rare to have renal dysfunction. The CNS is the most commonly affected end-organ, with manifestations that can include a headache, focal neurologic deficits (ischemic stroke and TIA), seizures, confusion, and vertigo. The mainstay of treatment in TTP is plasma exchange with high dose steroids (1 mg/kg of prednisone). Go to the next page if you knew the correct answer, or click the link image(s) below to further research the concepts in this question (if desired).

Research Concepts: Ischemic Stroke

We update eBooks quarterly and Apps daily based on user feedback. Please tap flag to report any questions that need improvement.

Question 739: A 16-year-old female presents to the emergency department following a front impact motor vehicle collision. She is extremely short of breath and cannot talk, and seems to be gasping for air. Physical examination reveals bruising in a linear pattern across the low neck and chest. There is also a crunching sensation felt on palpation of the neck and the chest around the sternal notch. What should be the next step in management?

Choices: 1. Needle decompression of possible pneumothorax 2. Portable chest x-ray 3. Awake fiberoptic intubation 4. Awake tracheostomy

Answer: 3 - Awake fiberoptic intubation Explanations: The patient is gasping for air and is unable to talk, indicating the possibility of laryngeal injury. Impending airway loss due to suspected laryngeal injury must be managed with direct visualization of the airway. Tracheostomy may be required and should be prepared for, but awake fiberoptic intubation is a safe option to both assess the airway and proceed to secure it. Signs of laryngeal injury include hoarseness, stridor, subcutaneous air, seatbelt sign, or other signs of an anterior neck injury. Airway management of laryngeal injury without direct visualization of the airway above and below the vocal cords may lead to the false passage of the endotracheal tube. This patient may have pneumothorax as well, but the impending airway obstruction must be managed first. A chest x-ray may be helpful to diagnose a pneumothorax, but should not delay emergent airway management. Go to the next page if you knew the correct answer, or click the link image(s) below to further research the concepts in this question (if desired).

Research Concepts: Laryngeal Injury

We update eBooks quarterly and Apps daily based on user feedback. Please tap flag to report any questions that need improvement.

Question 740: A previously healthy 52-year-old man presents to the hospital with substernal chest pain, severe worsening dyspnea with associated dry cough, abdominal pain, and low-grade fever. He reports he recently traveled to North Africa but denies sick contacts. A chest radiograph reveals diffuse bilateral opacities concerning for pulmonary edema. Abdominal ultrasound reveals hepatosplenomegaly. Labs reveal unexplained pancytopenia, and arterial blood gas shows profound hypoxemia. EKG reveals an acute ST-elevation myocardial infarction. He undergoes cardiac catheterization, which shows multi-vessel coronary artery disease requiring emergent surgical revascularization. During the procedure, a lung biopsy is performed, which reveals foamy macrophages with a peribronchial thickening. A diagnosis of lipoid pneumonia is made, and the patient improved with antibiotics, corticosteroids, and whole-lung lavage. Which of the following is the most likely etiology of this patient's underlying condition?

Choices: 1. Accumulation of glucocerebroside in alveolar macrophages 2. Sphingomyelinase deficiency 3. Dysregulated immune response causing persistent stimulation of phagocytic cells 4. Acquired clonal stem cell disorder

Answer: 2 - Sphingomyelinase deficiency Explanations: Niemann-Pick disease is a rare, autosomal recessive, congenital lipid storage disorder caused by a deficiency of the enzyme sphingomyelinase. Lack of this enzyme inhibits the breakdown of sphingomyelin, which is a fat found in cell membranes. When the lung is involved, endogenous lipoid pneumonia can be a presenting feature of this disease. There are multiple variants of this disease. Patients with Niemann-Pick Type A are severe with central nervous system involvement leading to death in early childhood. Niemann-Pick type B is a less severe form without CNS involvement, and patients often survive into adulthood. Interstitial lung disease is one of the major causes of morbidity and mortality due to the accumulation of lipid-laden phagocytes throughout the lung parenchyma. Characteristic histological findings in Niemann-Pick disease include foamy phagocytes (lipid-laden macrophages) caused by intracellular deposition of sphingomyelin in various tissues throughout the body. Bone marrow, liver, or lung biopsies often make the diagnosis. Symptoms are dependent upon the organs that accumulate these toxic lipids. Common clinical manifestations of Niemann-Pick disease include hyperlipidemia, hepatosplenomegaly, thrombocytopenia, and interstitial lung disease. Go to the next page if you knew the correct answer, or click the link image(s) below to further research the concepts in this question (if desired).

Research Concepts: Lipoid Pneumonia

We update eBooks quarterly and Apps daily based on user feedback. Please tap flag to report any questions that need improvement.

Question 741: A 65-year-old male patient with a history of chronic obstructive pulmonary disease (COPD) controlled on inhaled corticosteroids presents to the hospital complaining of sudden onset dyspnea and left-sided pleuritis type chest pain. A chest radiograph is performed which shows a 1.5 cm large area of air on the left side without lung markings. On aspiration, it reduces to 0.75 cm. What is the most appropriate next step in management?

Choices: 1. Admit overnight for oxygen supplementation 2. Discharge with follow up after 24 hours 3. Admit for chest drain insertion 4. Repeat aspiration

Answer: 1 - Admit overnight for oxygen supplementation Explanations: A pneumothorax is defined as a collection of air outside the lung but within the pleural cavity. It occurs when air accumulates between the parietal and visceral pleurae inside the chest. The air accumulation can apply pressure on the lung and make it collapse. The degree of collapse determines the clinical presentation of pneumothorax. There are two types of pneumothorax: traumatic and atraumatic. The two subtypes of atraumatic pneumothorax are primary and secondary. A primary spontaneous pneumothorax (PSP) occurs automatically without a known eliciting event, while a secondary spontaneous pneumothorax (SSP) occurs subsequent to an underlying pulmonary disease. In secondary spontaneous pneumothorax, if size/depth of pneumothorax is less than 1cm and no dyspnea then the patient is admitted, high flow oxygen is given and observation is done for 24 hours. If size/ depth is between 1-2cm, needle aspiration is done, then the residual size of pneumothorax is seen, if the depth after the needle aspiration is less than 1cm management is done with oxygen inhalation and observation and in case of more than 2cm, tube thoracostomy is done. In case of depth more than 2cm or breathlessness, tube thoracostomy is done. Go to the next page if you knew the correct answer, or click the link image(s) below to further research the concepts in this question (if desired).

Research Concepts: Pneumothorax

We update eBooks quarterly and Apps daily based on user feedback. Please tap flag to report any questions that need improvement.

Question 742: A 69-year-old man with a past medical history of hypertension, diabetes, hyperlipidemia, and an abdominal aortic aneurysm is found to be unconscious. The patient was in the hospital waiting room, visiting his family when he suddenly felt lightheaded and collapsed. In the hospital, the rapid response is called. His vital signs show a blood pressure of 80/60 mmHg, heart rate of 140/min, and SpO2 95% on room air. The patient appears to be pale, minimally responsive, tachycardic, having labored breathing, and bilateral abdominal ecchymosis. Large wide bore intravenous access is obtained subsequently, and IV fluid resuscitation is started. He is also intubated. Complete blood count and basic metabolic panels are sent. What is the next best step in the management of this patient?

Choices: 1. CT abdomen/pelvis 2. Abdominal ultrasound 3. Close monitoring in the intensive care unit 4. Emergency laparotomy

Answer: 4 - Emergency laparotomy Explanations: The likely diagnosis given the patient's presentation is acute abdominal aortic aneurysmal rupture. Since the patient's hemodynamical instability, he would need urgent open surgical repair. Delay in surgery carries high mortality in these patients. CT of the abdomen and pelvis are not indicated for this patient as he is hemodynamically unstable. Abdominal ultrasound is not indicated since the patient is thermodynamically unstable. Bedside fast ultrasound can be done in patients with no known history of an abdominal aneurysm but if clinical suspicion for aortic rupture is high. It is not appropriate to transfer to ICU at this time as the patient needs emergent surgery. Go to the next page if you knew the correct answer, or click the link image(s) below to further research the concepts in this question (if desired).

Research Concepts: Abdominal Aortic Aneurysm Rupture

We update eBooks quarterly and Apps daily based on user feedback. Please tap flag to report any questions that need improvement.

Question 743: A 45-year-old male patient is brought to the hospital after a car accident. The patient complains of severe back pain on arrival. A foley catheter is inserted and dark red urine can bee seen in the tubing. He is sent for a CT scan with contrast to assess the status of his kidneys. Eight hours later, the patient is anuric. A urinalysis is done which reveals the fractional excretion of sodium to be 3% and the urinary sodium concentration to be 50 mEq/l. The creatinine and BUN are elevated. What additional finding will be seen in the urine of this patient?

Choices: 1. Hyaline casts 2. Muddy brown casts 3. Waxy casts 4. White blood cell casts

Answer: 2 - Muddy brown casts Explanations: This patient is suffering from acute tubular necrosis. Muddy casts in the urine are typical of acute tubular necrosis. He was in a hypovolemic state as well as exposed to nephrotoxic contrast during the CT scan. The urinalysis of acute tubular necrosis shows muddy brown casts or renal tubular epithelial cells secondary to the sloughing of tubular cells into the lumen due to ischemia or toxic injury. The urinary sodium excretion determines that the kidney is sodium avid in hypovolemic states (prerenal) where kidneys try to conserve sodium. Also, they lose sodium due to tubular injury with values more than 40 to 50 mEq/L indicating acute tubular necrosis and less than 20 mEq/L suggestive of prerenal disease. Fractional excretion of sodium is a good test to differentiate between acute tubular necrosis and prerenal disease with a value of less than 1% favoring prerenal disease and more than 2% supporting acute tubular necrosis. However, these values are not always accurate as in chronic prerenal states such as congestive heart failure and cirrhosis in which there is an overlap between both (ATN and prerenal AKI) the value is less than 1%. Numerous biomarkers have evolved to detect AKI/acute tubular necrosis early as compared to serum creatinine. These biomarkers include serum cystatin C to be an early and reliable marker of renal injury as compared to serum creatinine which is often witnessed 48 to 72 hours after the initial insult. In prerenal disease, the urinalysis microscopy is normal or may contain hyaline casts. White blood cell casts are seen in glomerulonephritis. Go to the next page if you knew the correct answer, or click the link image(s) below to further research the concepts in this question (if desired).

Research Concepts: Acute Renal Tubular Necrosis

We update eBooks quarterly and Apps daily based on user feedback. Please tap flag to report any questions that need improvement.

Question 744: A 28-year-old male presents with nausea and headache. He started complaining of headaches about a week ago. Yesterday, the headaches are now accompanied by nausea and double vision. He is positive for HIV but refused treatment. Examination shows BP 140/90 mmHg, HR 64 beats/min, respiratory rate, and temperature are normal. Cranial nerve examination shows isocoric pupils, briskly reactive to light, bilateral papilledema on fundoscopy, and bilateral lateral rectus palsy. The motor and sensory examinations are normal. Babinski is positive bilaterally. The CT scan is normal. A lumbar puncture is done. Which of the following pathogens is most likely?

Choices: 1. Gram-negative diplococci 2. Gram-positive diplococci 3. Mucicarmine positive encapsulated budding yeast cells 4. Strongly acid-fast tubercles

Answer: 3 - Mucicarmine positive encapsulated budding yeast cells Explanations: Cryptococcus is the most likely CNS fungal infection. Patients who are HIV positive may have minimal or nonspecific symptoms at presentation. The patient commonly presents with neurological symptoms such as a headache, altered mental status, and other signs and symptoms include lethargy along with fever, stiff neck (both associated with an aggressive inflammatory response), nausea, and vomiting. CSF usually presents low glucose and high protein levels. White cell count can be normal or higher than 20 microL and have a lymphocyte predominance. Nevertheless, CSF can be normal and have positive results on India ink stain and antigen testing, especially in HIVpositive patients who do not have an adequate inflammatory response. Go to the next page if you knew the correct answer, or click the link image(s) below to further research the concepts in this question (if desired).

Research Concepts: Cryptococcal Meningitis

We update eBooks quarterly and Apps daily based on user feedback. Please tap flag to report any questions that need improvement.

Question 745: A 24-year-old man with a history of active Crohn disease and antiphospholipid syndrome undergoes a multi-visceral transplant, which is complicated by bleeding and requiring intravenous prothrombotic therapies. Due to the recent procedure, the transplant team felt it would be best to hold some of the patient’s home medicines. In the immediate post-operative period, he does well and tube feeds are started. On postoperative day two, the patient develops excruciating abdominal pain with associated nausea. Vital signs show temperature 99.8 F, pulse rate 131/min, blood pressure 153/97 mmHg, and respiratory rate 22/min. His abdomen is soft without any guarding. Incisions appear unremarkable. Labs are obtained, which show a worsening leukocytosis to 23,000/microL, lactic acid of 7.1 mmol/L, and a creatinine level of 0.9 mg/dL. Which of the following is the next best step in the management of this patient?

Choices: 1. Fluid bolus 2. CT mesenteric angiography 3. Mesenteric duplex 4. Hold tube feeds and re-evaluate patient in a few hours

Answer: 2 - CT mesenteric angiography Explanations: The patient is most likely suffering from post-operative mesenteric ischemia, secondary to his hypercoagulable state, and unable to be safely placed on anticoagulation. His laboratory values show an elevated WBC count and evidence of anaerobic metabolic stress. Acute mesenteric ischemia in the setting of intestinal transplant is a surgical emergency and requires immediate diagnosis and return to the operating room for management. CT scan is the best imaging modality for mesenteric ischemia as it is readily available and a rapid test to image the mesenteric vessels. Mesenteric duplex is a modality that can diagnose mesenteric ischemia but is not as rapid and is user dependant. It is highly specific but does have lower sensitivity than CT due to overlying bowel gas disrupting the images. Go to the next page if you knew the correct answer, or click the link image(s) below to further research the concepts in this question (if desired).

Research Concepts: Intestinal And Multivisceral Transplantation

We update eBooks quarterly and Apps daily based on user feedback. Please tap flag to report any questions that need improvement.

Question 746: A 16-year-old male brought to the emergency department by lawenforcement for “bizarre behavior.” The patient is extremely agitated and uncooperative requiring physical restraint by four police officers. The patient has a heart rate of 135 bpm, blood pressure 177/82 mm of Hg, respiratory rate 26 breaths/minute, oxygen saturation 92% on room air. Fingerstick blood sugar 110 mg/dL. The patient is given 4 mg IM lorazepam for sedation. The patient is warm to the touch. Eye exam shows miotic pupils and rotary nystagmus. Police state that witnesses report that the patient was smoking “angel dust” before exhibiting his “bizarre behavior.” Which of the following is false pertaining to the substance that the patient has been using?

Choices: 1. Dopamine reuptake is blocked 2. Acetylcholine receptor stimulation is increased 3. Sigma receptor stimulation is decreased 4. Gama-aminobutyric acid (GABA) receptor stimulation is increased

Answer: 3 - Sigma receptor stimulation is decreased Explanations: Sigma receptor stimulation is increased by phencyclidine causing lethargy and coma. Phencyclidine inhibits reuptake of dopamine and norepinephrine leading to sympathomimetic effects such as hypertension, tachycardia, bronchodilation, and agitation. Phencyclidine binds to gamma-aminobutyric acid (GABA) receptors causing sedation. Phencyclidine binds to acetylcholine receptors to cause muscarinic and nicotinic effects. Go to the next page if you knew the correct answer, or click the link image(s) below to further research the concepts in this question (if desired).

Research Concepts: Phencyclidine Toxicity

We update eBooks quarterly and Apps daily based on user feedback. Please tap flag to report any questions that need improvement.

Question 747: A 32-year-old man is brought to the emergency department by paramedics after being found unconscious. He was found with a needle in his arm in an alleyway. He was breathing slowly, and he was unresponsive with pinpoint pupils. On the scene, his heart rate was 55 beats/minute, blood pressure 127/84 mmHg, a respiratory rate of about four breaths/minute, and his temperature was 98.5 degrees F (36.9 C). Before administering naloxone, the paramedic noted that with the patient's eyelids open, moving his head side to side caused his eyes to move in the opposite direction, like a doll's eyes. This doll's eyes reflex involves several cranial nerves and pathways within the brainstem. One well-known route within this reflex connects cranial nerve III and VI. What disease is known to cause a lesion in this pathway?

Choices: 1. Huntington's chorea 2. Muscle eye brain disease 3. Claude syndrome 4. Multiple sclerosis

Answer: 4 - Multiple sclerosis Explanations: Multiple sclerosis is a demyelinating disorder that can cause pathology in the cranial nerves to the eye. Multiple sclerosis can cause internuclear ophthalmoplegia (INO) by causing demyelination of the medial longitudinal fasciculus. The medial longitudinal fasciculus among other things connects cranial nerve III and VI. The medial longitudinal fasciculus is critical for the connection of cranial nerves in the doll's eyes reflex (vestibulo-ocular reflex). Go to the next page if you knew the correct answer, or click the link image(s) below to further research the concepts in this question (if desired).

Research Concepts: Doll's Eyes

We update eBooks quarterly and Apps daily based on user feedback. Please tap flag to report any questions that need improvement.

Question 748: A 67-year-old man is being evaluated on postoperative day 1 following a right hemicolectomy. The nurse reports the patient has had low-normal blood pressures with a mean arterial pressure of 65-68 mmHg for approximately 1 hour. Emergent labs reveal no leukocytosis, but his lactate level is 2.1 mmol/L, and his serum creatinine has increased from 0.8 mg/dL to 1.9 mg/dL. On examination, his mucous membranes are moist, and his pain is wellcontrolled. He denies bowel movement or flatus as of yet but has no complaints otherwise. He has been receiving 0.9% sodium chloride solution via IV at 125 mL/h. Which of the following is the next best step in the management of this patient?

Choices: 1. 1 L bolus of crystalloid solution over 60 minutes 2. Initiation of pressor support 3. Initiation of hydroxyethyl starch-containing solution 4. Initiate fenoldopam

Answer: 1 - 1 L bolus of crystalloid solution over 60 minutes Explanations: Studies have shown that the development of lactic acidosis or hypotension intraoperatively are late indicators of reduced renal perfusion. As such, cardiac output and renal blood flow should be maintained with aggressive fluid resuscitation, and when fluids alone are not sufficient, inotropes are necessary. It is most appropriate to start with crystalloid solutions. Pressor support is indicated if fluid resuscitation is ineffective, or if hemodynamic instability develops. The use of hydroxyethyl starches, when compared to colloids, has been linked to increased rates of AKI and increased need for RRT in some investigations, while it has not been shown to increase AKI in other studies. Fenoldopam is a selective dopamine-1 agonist that has been shown to reduce the need for RRT. However, this has only been consistently demonstrated in patients having undergone cardiac surgery, and its use is limited by its propensity to cause systemic hypotension. Go to the next page if you knew the correct answer, or click the link image(s) below to further research the concepts in this question (if desired).

Research Concepts: Perioperative Acute Kidney Injury

We update eBooks quarterly and Apps daily based on user feedback. Please tap flag to report any questions that need improvement.

Question 749: A 58-year-old man is brought to the hospital after a witnessed unresponsive spell while seated at a dinner theater. There was no prodrome of headache or chest pain. Witnesses report that he slumped to the floor and had brief generalized motor movements lasting perhaps ten seconds. He was alert and oriented a few seconds later. He has a history of hypertension but no other significant medical history. He was incontinent during the event, but there was no tongue-biting. He states he had some wine with dinner but was not clinically intoxicated. Physical examination is unremarkable. Which of the following best identifies the type of event?

Choices: 1. Provoked seizure 2. Unprovoked seizure 3. Convulsive concussion 4. Convulsive syncope

Answer: 4 - Convulsive syncope Explanations: The differential diagnosis of brief dissociative spells is complicated. The lack of any postictal confusional state perhaps is the strongest factor weighing against a seizure and suggesting syncope. All historical factors should be considered in the evaluation. Brief symmetric motor movements commonly occur with syncope and are given the term convulsive syncope. Tongue-biting, when present tends, involves the anterior tongue with syncope and the lateral tongue with seizures. Convulsive syncope was studied in blood banks. Patients typically had a prodrome of yawning, sweating, slight diaphoresis, and then a typical faint. Brief motor movements were noted in many patients. Go to the next page if you knew the correct answer, or click the link image(s) below to further research the concepts in this question (if desired).

Research Concepts: Seizure

We update eBooks quarterly and Apps daily based on user feedback. Please tap flag to report any questions that need improvement.

Question 750: A 61-year-old patient with stage IIIB squamous cell carcinoma presents to the emergency department with confusion and a disoriented state. Laboratory investigations revealed that he has a calcium level of 18.5 mg/dL. In addition to that, his serum parathyroid hormone levels are seven pg/mL (normal = 10-55 pg/mL ), but the parathyroid-like hormone is positive. Treatment with large volumes of intravenous fluids and furosemide was done, but it only lowered the calcium to 17.0mg/dL. Which of the following is the most appropriate treatment for this patient?

Choices: 1. Continue fluids and furosemide 2. Start calcitonin 3. Start pamidronate 4. Start calcitonin and pamidronate

Answer: 4 - Start calcitonin and pamidronate Explanations: Paraneoplastic syndrome is one of several causes of hypercalcemia. Squamous cell carcinoma of the lung is the most likely cause of parathyroid hormonerelated peptide. Calcitonin can drop the calcium acutely but tachyphylaxis develops. Intravenous pamidronate, a bisphosphonate, takes 1 to 2 days for effect; therefore, for acute reduction, calcitonin is given along with pamidronate. Go to the next page if you knew the correct answer, or click the link image(s) below to further research the concepts in this question (if desired).

Research Concepts: Paraneoplastic Syndromes

We update eBooks quarterly and Apps daily based on user feedback. Please tap flag to report any questions that need improvement.

Question 751: A fourth-year medical student is about to start his sub-internship in the general surgery service. He decides to get involved in the surgery boot camp that his medical school offers. The first day they are taught how to perform central venous catheters to get prepared for his critical care rotations as a resident. What would be the best method to teach this student?

Choices: 1. Verbal repetition of the procedure following a checklist with one simulated case at the end of the day 2. Repetition of the procedure in several different models with verbal feedback at the end of the camp 3. Repetition of the procedure in one model several times with feedback at the end of the day 4. Repetition of the procedure with a partner with constant feedback of the technique as well as feedback at the end of the day by the trainer

Answer: 4 - Repetition of the procedure with a partner with constant feedback of the technique as well as feedback at the end of the day by the trainer

Explanations: In the skills assessment and learning, the more feedback a trainee gets, the better for him. The more repetitions a trainee has, the more self-consciousness is developed. When learning new skills, the best way to improve and creating self-consciousness of details is by comparing yourself to another trainee. It helps develop skills faster than doing a self-evaluation. Reinforcement of the same technique under the same conditions and using a checklist to establish a pattern to follow, this will make every repetition similar to the previous one, improving the technique. Verbal repetition or written tests have been less effective in teaching any skill, especially in the critical care setting. Go to the next page if you knew the correct answer, or click the link image(s) below to further research the concepts in this question (if desired).

Research Concepts: Simulation Training and Skill Assessment in Critical Care

We update eBooks quarterly and Apps daily based on user feedback. Please tap flag to report any questions that need improvement.

Question 752: A 65-year-old female immigrant from Pakistan presents with the chief complaints of severe shortness of breath, productive cough, and inability to carry out activities of daily living. She has a history of biomass fuel exposure for twenty years in her hometown. On examination, a restless female with severe respiratory distress, blood pressure 110/70 mmHg, pulse 112/min regular, respiratory rate 26/min, and oxygen saturation of 88% on room air. On auscultation, her chest is largely silent with decreased air entry bilaterally. Her arterial blood gases suggest a type II respiratory failure. Which of the following should be considered besides supportive care?

Choices: 1. Elective endotracheal intubation 2. High flow oxygen therapy 3. Theophylline 4. Bilevel positive airway pressure

Answer: 4 - Bilevel positive airway pressure Explanations: In the case of chronic obstructive pulmonary disease (COPD) with type II respiratory failure, it is recommended to put the patient on bilevel positive airway pressure. The nonpharmacological approach includes vaccination (influenza and pneumococcal), smoking cessation, and pulmonary rehabilitation. Pulmonary rehabilitation is indicated in all stages of COPD. Bronchodilators are the primary pharmacologic treatment used. Short-acting bronchodilators provide immediate relief, and long-acting bronchodilators are used for maintenance in patients with advanced disease. A long-acting beta2 agonist alone or in combination with an inhaled corticosteroid has shown to reduce exacerbation. A long-acting antimuscarinic agent has also been shown to improve dyspnea and reduce exacerbations. A combination of two different classes of long-acting bronchodilators can be used in patients with moderate-to-severe COPD when monotherapy is insufficient. Continuous oxygen therapy is indicated for hypoxemic patients with COPD when arterial oxygen saturation is equal to, or less than 88% or partial pressure of oxygen is equal to or less than 55 mmHg. Based on current estimates, COPD will be the third leading cause of death worldwide by 2030. In the United States, it is estimated that more than 12 million people are affected by COPD and is more prevalent in women than men. The death rate for COPD has declined in men from 1999 to 2010, but it is steady among women. Typical symptoms of COPD include a chronic, productive cough, shortness of breath, and wheezing. Acute exacerbation of COPD is usually defined as at least two of the following three conditions: worsening of shortness of breath, change in purulence, or quantity of sputum. Physical findings include barrel-shaped chest, diminished breath sounds, and hyperresonance on percussion. In advanced disease with cor pulmonale, jugular venous distention, accentuated second heart sound, hepatomegaly, and pedal edema can be seen. Lung auscultation may show a prolonged expiratory phase with or without wheezing. Respiratory symptoms are often relieved with pursed-lip breathing and tripod position. Pulmonary function test (PFT) is recommended to diagnose COPD. Airflow limitation is diagnosed when forced expiratory volume in one second (FEV1) to forced vital capacity ratio is less than 70%. According to Global Initiative for Chronic Obstructive Lung Disease classification system, the severity of obstructive defect is mild, moderate, severe, or very severe when FEV 1 percent predicted is greater than 80%, 50% to 79%, 30% to 49%, and less than 30% respectively. Diffusion capacity for carbon monoxide is reduced in COPD. A chest radiograph often shows hyperinflation and flattening of the diaphragms, especially in advanced disease. High-resolution CT of the chest may demonstrate emphysema. An echocardiogram is used to assess secondary pulmonary hypertension which is due to hypoxemic vasoconstriction of the pulmonary blood vessels. A walking or nocturnal desaturation study identifies patients who would benefit from oxygen therapy with activities or during sleep. Testing for alpha-1 antitrypsin level should be considered in age younger than 40 years and non-smokers if a significant obstructive defect is found on PFTs. Go to the next page if you knew the correct answer, or click the link image(s) below to further research the concepts in this question (if desired).

Research Concepts: Chronic Obstructive Pulmonary Disease

We update eBooks quarterly and Apps daily based on user feedback. Please tap flag to report any questions that need improvement.

Question 753: A 23-year-old female who is 33 weeks pregnant presents to the emergency department after a motor vehicle collision. The patient is supine on a backboard with a cervical collar in place. Given the patient's mechanism of injury, there is concern for a possible cervical spine injury. On arrival, the patient is hypotensive with a blood pressure of 80/60 mmHg and a heart rate of 95 beats per minute. The evaluating clinician is concerned about the aortocaval compression syndrome and knows the patient needs to be re-positioned immediately to help treat the hypotension. What is the most appropriate next step of management?

Choices: 1. Isolated elevation of the left hip 2. Manual displacement of the uterus to the left of the midline 3. Tilt the patient's bed into a reverse Trendelenburg position 4. Tilt the backboard 45 degrees to the right

Answer: 2 - Manual displacement of the uterus to the left of the midline Explanations: Since there is a concern for a cervical spine injury, manual displacement of the uterus to the left side of the patient's abdomen. This will decrease potential compression of the inferior vena cava by the uterus and is one of the accepted initial treatments when aortocaval compression is suspected. This is done by placing the provider's hand on the right side of the abdomen, lateral to the gravid uterus, and shifting the uterus to the left, and thus off of the vena cava. This maneuver is simple, requires no expertise, and is not harmful to the uterus or fetus. Once other causes of hypotension are treated and/or ruled out, then the physical movement of the uterus off of the spine (and inferior vena cava) is necessary. When trauma is not involved, simply placing the patient in the left lateral position may be done. However, when trauma is present, immobilization of the spine must simultaneously occur, and different maneuvers are indicated. Isolated elevation of the right hip alone may alleviate the compression. According to the advanced trauma life support guidelines, tilting the backboard 15 to 30 degrees to the left (right side upwards) is an additional option. This may be done either manually or with elevators, including premade elevators or towel rolls placed under the board. This option is sometimes difficult as the weight of the gravid abdomen gravitationally pulls the patient to the left side, potentially compromising spinal immobilization. Though difficult, in-line immobilization is a priority. Go to the next page if you knew the correct answer, or click the link image(s) below to further research the concepts in this question (if desired).

Research Concepts: Aortocaval Compression Syndrome

We update eBooks quarterly and Apps daily based on user feedback. Please tap flag to report any questions that need improvement.

Question 754: A 16-year old is seen in the hematology clinic with complaints of fatigue. Examination reveals mild jaundice and splenomegaly. Blood work reveals a white blood cell (WBC) count of 2.9, hematocrit of 22, and hemoglobin of 8. The mean corpuscular hemoglobin concentration (MCHC) is markedly increased. Which of the following is the most threatening long-term complication for a patient treated for this condition?

Choices: 1. Overwhelming post-splenectomy sepsis (OPSS) 2. Infection of the Incision site 3. Vulnerability to infection 4. Reaction to medications

Answer: 1 - Overwhelming post-splenectomy sepsis (OPSS) Explanations: Overwhelming post-splenectomy sepsis (OPSS) is a life-threatening complication for those undergoing splenectomy. Infection of the incision site is always a concern for those undergoing any surgery. Antibiotic therapy until the age of sixteen (possibly longer) Is a standard treatment because of the increased vulnerability to infection following splenectomy. A two-year course of penicillin or long-term treatment with ampicillin is often recommended by physicians for patients undergoing a splenectomy. Vaccination therapy is the standard of care prior to surgery, including a booster dose of pneumococcal vaccine five to 10 years after undergoing a splenectomy. Go to the next page if you knew the correct answer, or click the link image(s) below to further research the concepts in this question (if desired).

Research Concepts: Hereditary Spherocytosis

We update eBooks quarterly and Apps daily based on user feedback. Please tap flag to report any questions that need improvement.

Question 755: A 76-year-old woman with a history of hypertension, hyperlipidemia, and a 40 pack-year smoking history presents to the hospital with fluctuating left-sided weakness and dysarthria. On examination, the patient was noted to have a horizontal gaze palsy but a preserved vertical gaze. Her National Institute of Health Stroke Scale fluctuated between 4 and 7. CT brain imaging did not show any acute pathology, and vessel imaging with a CT head and neck angiography was negative for a large vessel occlusion and a patient basilar artery, but with extensive atherosclerotic disease in multiple vascular territories. She was normal 4 hours before the presentation. Blood pressure ranges were between systolic of 140-160 mmHg and diastolic of 90-100 mmHg. Other vital signs are within normal limits. Which of the following is the next best step in the management of this patient?

Choices: 1. Intravenous tissue plasminogen activator, if no contraindications 2. Aspirin 3. Diagnostic angiography to evaluate the atherosclerotic disease with an intention to treat the intracranial stenosis with angioplasty +/- stenting 4. Heparin infusion and warfarin

Answer: 1 - Intravenous tissue plasminogen activator, if no contraindications Explanations: The patients presented with a pontine warning syndrome with fluctuating clinical features and is within the therapeutic window for the use of a thrombolytic agent. Thrombolysis is indicated in patients presenting within a 4.5-hour time frame with an ischemic stroke presentation with no clear contraindications. This stroke presentation is likely due to atherosclerotic disease likely large artery atherosclerotic disease or small vessel disease based on the extensive vascular risk factors and the CT angiography showing extensive atherosclerotic disease. Irrespective of the etiology of the stroke, thrombolysis has been shown to be beneficial if used within a reasonable time frame. If the patient presented outside of the therapeutic window of 4.5 hours from the clinical presentation, aspirin would have been a reasonable choice. Go to the next page if you knew the correct answer, or click the link image(s) below to further research the concepts in this question (if desired).

Research Concepts: Pontine Infarction

We update eBooks quarterly and Apps daily based on user feedback. Please tap flag to report any questions that need improvement.

Question 756: A 61-year-old woman is placed on veno-arterial (VA) ECMO due to low ejection fraction after a long clamp time on her on-pump CABG. The patient is taken to the intensive care unit after placement of the VA ECMO catheters. Over the course of her ICU stay, her EF shows signs of recovery. When attempting to wean the VA ECMO circuit, which of the following best identifies the minimum ejection fraction and flow rate that would allow weaning the patient from VA ECMO?

Choices: 1. Ejection fraction 10%; flow rate 3 L/min 2. Ejection fraction 15%, flow rate 4 L/min 3. Ejection fraction 30%, flow rate 4 L/min 4. Ejection fraction 25%, flow rate 1.5 L/min

Answer: 4 - Ejection fraction 25%, flow rate 1.5 L/min Explanations: When weaning a patient from VA ECMO, the cause of the cardiogenic shock must be resolved prior to weaning. Attempts at weaning before restoring the ejection fraction should not be attempted until the heart can function without the aid of maximum pressor support. When weaning from VA ECMO, transesophageal echocardiography is essential in looking at the function of the heart. Echo is often performed on patients to look for progress for resolution of the cardiogenic shock. VA ECMO should not be weaned until the ejection fraction has recovered to a minimum of 25%. This number has shown that the heart is strong enough to function for weaning trials. When decreasing the flow rates on the VA ECMO circuit, a more accurate echo of the heart's ejection fraction can be obtained. The flow rate should be reduced as low as possible, with the minimum flow rate being 1.5 L/min as this allows to see the heart working with little support. It is recommended not to go with lower flow rates as this can cause thrombosis to the ECMO circuit. Weaning from VA ECMO should not be attempted until resolution of the cardiogenic shock, ejection fractions less than 25% are not recommended for weaning until the heart is stronger. Flow rates greater than 1.5 L/min give the heart more support and can falsely alert the physician to know the true function of the heart. Go to the next page if you knew the correct answer, or click the link image(s) below to further research the concepts in this question (if desired).

Research Concepts: Extracorporeal Membrane Oxygenation Weaning

We update eBooks quarterly and Apps daily based on user feedback. Please tap flag to report any questions that need improvement.

Question 757: A 45-year-old male presents with complaints of progressive shortness of breath. He has had the symptoms for the past year. His shortness of breath was gradual in onset but is progressive. The patient now feels breathless even on mild exertion. He has been finding it difficult to perform his daily activities. He also reports occasional episodes of dizziness and loss of consciousness. There is no history of cough or fever. His examination reveals a blood pressure of 130/50 mmHg, a pulse of 100 beats per minute, a respiratory rate of 27 per minute, and a temperature of 98 F. His cardiac auscultation reveals the presence of an early diastolic grade 4/6 murmur best heard over the left 3rd intercostal space. The murmur increases on expiration. His respiratory, gastrointestinal, and neurological examinations are unremarkable. His investigations revealed a WBC count of 5,000 per microL, hemoglobin of 12 gm/dl, platelet count of 200,000 per microL, serum ALT 35 IU/l, serum AST 35 IU/l, and serum creatinine of 1.1 mg/l. Serologies are performed, which are anti- HCV negative, anti-Hbs positive, HbsAg negative. Anti-Hbc IgM negative, RPR negative, TPHA positive, ANA negative, and RA factor negative. An X-ray reveals the presence of mediastinal widening and linear calcifications of the ascending aorta. What is the most likely etiology for this patient’s shortness of breath?

Choices: 1. Mitral valve stenosis 2. Pericardial effusion 3. Aortic inflammation, regurgitation, and insufficiency 4. Valvular calcification

Answer: 3 - Aortic inflammation, regurgitation, and insufficiency Explanations: This patient presents with symptoms of progressive dyspnea. His history of dizziness and loss of consciousness suggests a cardiovascular cause. The presence of an early diastolic murmur suggests the presence of aortic regurgitation. The serology investigation reveals positivity for treponema pallidum (TPHA) and his x-ray suggests aortic dilation. Linear calcifications of the ascending aorta is closely associated with the most likely diagnosis which is cardiovascular syphilis. Cardiovascular syphilis is a form of tertiary syphilis characterized by aortitis, aortic valve regurgitation, and aortic dilatation eventually resulting in heart failure. The usual presentation is 15-30 years after the primary infection. Morphologically, the aorta appears dilated with patchy areas of fibrosis and necrosis referred to as "tree bark aorta". Histologically, it is characterized by a chronic perivascular inflammatory, lymphoplasmacytic infiltrate. There is obliterative endarteritis of the vasa vasorum and subsequent necrosis. There is co-existent aortic insufficiency, which may lead to left ventricular failure. Cardiovascular syphilis may cause coronary ostial lesions but does not typically cause atherosclerosis. Valvular calcification and stenosis may cause shortness of breath, such as in aortic and mitral stenosis. Go to the next page if you knew the correct answer, or click the link image(s) below to further research the concepts in this question (if desired).

Research Concepts: Syphilis

We update eBooks quarterly and Apps daily based on user feedback. Please tap flag to report any questions that need improvement.

Question 758: A 50-year-old woman with a past medical history of diabetes presents to the office with left lower extremity cellulitis. She is prescribed sulfamethoxazole-trimethoprim and follows up 48 hours later, reporting new-onset fever and persistent erythema. She is admitted to the hospital and started on vancomycin and piperacillin/tazobactam. The next morning, her vital signs are stable, but there are bullae on her leg with no regression of erythema. What is the next best step in the management of this patient?

Choices: 1. Change vancomycin to linezolid 2. Add levofloxacin for improved pseudomonal coverage 3. Consult infectious disease 4. Obtain a stat CT scan of the leg

Answer: 4 - Obtain a stat CT scan of the leg Explanations: Necrotizing fasciitis is a rare infection of the fascia that leads to necrosis of the subcutaneous tissue. It is characterized by fevers, erythema, edema, pain out of proportion to the exam, and crepitus. If necrotizing fasciitis is suspected and the patient is unstable, crepitus is noted, and/or symptoms have worsened, surgery should be called immediately. In a stable patient without crepitus or rapid progression of erythema, edema, etc. a CT scan is often the imaging modality used to assess for gas. If you're suspicious of necrotizing fascitis, surgical consultation is recommended over an infectious disease consultation. Changing from vancomycin to linezolid is not necessary. Double coverage for pseudomonas is not typically recommended. Go to the next page if you knew the correct answer, or click the link image(s) below to further research the concepts in this question (if desired).

Research Concepts: Cellulitis

We update eBooks quarterly and Apps daily based on user feedback. Please tap flag to report any questions that need improvement.

Question 759: A 34 years old patient presented at the ED for weakness. On examination, pupils were isochoric and equally reactive to light, there was impaired lateral and medial gaze, the was able to close his eyes tightly, there was bilateral facial droop, the patient was unable to stick out his tongue, and the speech was severely dysarthric. The manual muscle testing shows 3/5 on the right upper and lower extremity, 4/5 on the left upper and lower extremity, sensory exam shows deficit more on the left than the right. Babinski was positive bilaterally. Where is this patient's lesion?

Choices: 1. Cerebellar vermis 2. Right parietotemporal region 3. Bilateral occipital lobe 4. Anterior pontine region

Answer: 4 - Anterior pontine region Explanations: Occlusion in the vertebrobasilar system can result in locked-in syndrome. As the basilar artery gets occluded, it will affect cranial nerve nuclei and tracts in the pons, causing bilateral facial palsy, as well as problems with eye movement due to the involvement of the medial longitudinal fasciculus and paramedian pontine reticular formation. The involvement of the corticospinal tracts in this level will cause weakness of both sides of the body to varying degrees, and quadriplegia as an endpoint. The involvement of the spinothalamic tracts causes sensory impairment on both sides. Lock-in syndrome is the full syndrome in basilar artery occlusion, causing quadriplegic and cranial nerve deficits. The sparing of the ascending reticular formation in the pons causes the patient to be awake. Go to the next page if you knew the correct answer, or click the link image(s) below to further research the concepts in this question (if desired).

Research Concepts: Locked-in Syndrome

We update eBooks quarterly and Apps daily based on user feedback. Please tap flag to report any questions that need improvement.

Question 760: A 3-year old girl is admitted to the recovery after undergoing an open reduction and fixation of her midshaft radius and ulna fractures. On exam, she is responding to voice, her blood pressure 140/85 mmHg, heart rate 98 bpm, respiratory rate 12 cycles per min, and saturation 92% on 5L of O2. Her past medical exam is significant for tonsillar hypertrophy. She is put on ventilation support. Which of the following is an absolute contraindication for her ventilation support?

Choices: 1. Epistaxis 2. Pneumothorax 3. Bullous lung disease 4. Inability to protect their airway

Answer: 4 - Inability to protect their airway Explanations: Continuous positive airway pressure therapy (CPAP) is a form of noninvasive positivepressure ventilation. It is routinely used in individuals with obstructive sleep apnea and chronic obstructive pulmonary disease. Epistaxis, pneumothorax, bullous lung disease, and apnea are contraindications to CPAP. This patient has obstructive sleep apnea from tonsillar hypertrophy. She will need CPAP in the recovery room as she is used to chronic hypercapnia and hypoxia. The inability to protect the airway is an absolute contraindication for CPAP. It may lead to aspiration of gastric contents. Go to the next page if you knew the correct answer, or click the link image(s) below to further research the concepts in this question (if desired).

Research Concepts: Continuous Positive Airway Pressure

We update eBooks quarterly and Apps daily based on user feedback. Please tap flag to report any questions that need improvement.

Question 761: A 16-year-old boy who was recently diagnosed as HIV-positive presents with a 6-hour history of fever, headache, and blurred vision. A head CT scan shows no intracranial abnormality. A lumbar puncture is performed, and CSF testing reveals encapsulated organisms. Besides amphotericin, which drug should be given to this patient?

Choices: 1. Trimethoprim 2. Ketoconazole 3. Flucytosine 4. Nystatin

Answer: 3 - Flucytosine Explanations: Amphotericin B is an essential part of induction treatment for cryptococcal meningitis. The most effective combination for the induction therapy is flucytosine and amphotericin B. Unfortunately, flucytosine is frequently unavailable because of its cost. The infectious diseases society of America (IDSA) guidelines do not include the use of ketoconazole, nystatin, or trimethoprim for the treatment of cryptococcal meningitis. Go to the next page if you knew the correct answer, or click the link image(s) below to further research the concepts in this question (if desired).

Research Concepts: Cryptococcal Meningitis

We update eBooks quarterly and Apps daily based on user feedback. Please tap flag to report any questions that need improvement.

Question 762: A 75-year-old Caucasian man presents to the emergency department with one episode of a small amount of hemoptysis. He has also had a dry cough and decreased urine output for the last 2 days. He denies fever, chills, or dysuria. He denies frank hematuria. On examination, his respiratory rate is 24/min, heart rate 88/min, and blood pressure 130/78 mmHg. Labs reveal serum creatinine 2.4 mg/dL and BUN of 26 mg/dL. His urinalysis is positive, with 1+ protein and moderate blood. His chest x-ray is suggestive of bilateral fluffy infiltrates. He never had kidney disease in the past, and his most recent serum creatinine during his visit was 1 mg/dL 3 months back. He denies the recent use of NSAIDs other medications. A renal biopsy is planned. Soon after admission, he develops progressive respiratory discomfort, requiring mechanical ventilatory support. What is the best initial therapy for this patient?

Choices: 1. IV methylprednisone 2. Oral prednisone 3. Plasmapheresis 4. Oral azathioprine

Answer: 1 - IV methylprednisone Explanations: The clinical vignette is suggestive of anti-GBM disease. Timely treatment is the key to preserving renal function. The recommended guideline is to start pulse dose steroids as soon as anti-GBM disease is suspected. It is not advisable to wait for the biopsy result to confirm anti-GBM disease. Pulse dose methylprednisone 500- 1000 mg IV for 3-5 days is recommended. Once the diagnosis is confirmed plasmapheresis can be initiated. Go to the next page if you knew the correct answer, or click the link image(s) below to further research the concepts in this question (if desired).

Research Concepts: Rapidly Progressive Glomerulonephritis

We update eBooks quarterly and Apps daily based on user feedback. Please tap flag to report any questions that need improvement.

Question 763: What is the pathology behind cardiac tamponade? Choices: 1. Accumulation of 250 ml of fluid 2. Rate of fluid accumulation 3. Afterload 4. Increased venous return

Answer: 2 - Rate of fluid accumulation Explanations: The rate of accumulation of fluid is more important than the volume in the pathogenesis of cardiac tamponade. As little as 150 ml of fluid can cause clinically significant symptoms while as much as a liter can slowly accumulate if the pericardium is distensible. In acute cases, even 100 ml of fluid accumulation can cause symptoms but in chronic cases, the pericardial sac can tolerate as much as 1,000 ml. The other factor that also affects development of tamponade is the compliance of the pericardium. If the pericardium is pliable, it may initially stretch and prevent symptoms. Go to the next page if you knew the correct answer, or click the link image(s) below to further research the concepts in this question (if desired).

Research Concepts: Cardiac Tamponade

We update eBooks quarterly and Apps daily based on user feedback. Please tap flag to report any questions that need improvement.

Question 764: A 45-year-old man presents to the emergency department with a complaint of breathlessness and dry cough. This is his third admission this year with similar complaints. On arrival, his vitals are blood pressure 100/70 mmHg, pulse 90/min, respiratory rate 18/min and a temperature of 99 F. His spO2 is 86%. Laboratory evaluation reveals an LDH level of 750 IU/L, CD4 count 140/microL and the chest x-ray shows bilateral interstitial infiltrates. What is the best initial therapy for this patient?

Choices: 1. Trimethoprim/ sulfamethoxazole 2. Zidovudine (AZT) 3. Pentamidine isethionate 4. Ceftriaxone/azithromycin

Answer: 1 - Trimethoprim/ sulfamethoxazole Explanations: Trimethoprim/sulfamethoxazole is the first-line therapy for Pneumocystis jiroveci pneumonia (PCP). This drug is an anti-infective/amebicide/antiprotozoal. The organism responsible for PCP is the P jiroveci protozoon. PCP is normally present in the lungs of humans and various animals. However, it becomes an aggressive pathogen in the immunocompromised patient. Pentamidine is among the drugs of choice for treating pneumocystis carinii and used for patients with sulfa allergies. PCP does not respond to antifungal treatment. Second-line drugs include dapsone, pentamidine, or atovaquone. In HIV patients, the treatment duration is for 21 days. Go to the next page if you knew the correct answer, or click the link image(s) below to further research the concepts in this question (if desired).

Research Concepts: Pneumocystis Jirovecii Pneumonia

We update eBooks quarterly and Apps daily based on user feedback. Please tap flag to report any questions that need improvement.

Question 765: A 65-year-old man with a history of hypertension, coronary artery disease, ischemic cardiomyopathy, and pacemaker implantation secondary to complete heart block presents to the emergency department with worsening shortness of breath. He also reports orthopnea and lower extremity edema but no chest pain. His home medications include aspirin, atorvastatin, metoprolol, and lisinopril. He reports compliance with medications and a low salt diet. Vital signs show blood pressure 125/80 mmHg, heart rate 110/min, and SpO2 97% on 2 L/min oxygen. Examination shows diffuse crackles bilaterally, and +2 pitting edema but is otherwise unremarkable. EKG shows an atrial sensed ventricular paced rhythm at a rate of 110/min. A chest X-ray shows pulmonary edema. Labs are remarkable for normal CBC, normal kidney function, negative troponin, and BNP of 2200 pg/mL. Which of the following is the most appropriate next step in evaluating the cause of his acute decompensated heart failure?

Choices: 1. Coronary angiogram 2. Right heart catheterization 3. Transesophageal echocardiogram 4. Pacemaker interrogation

Answer: 4 - Pacemaker interrogation Explanations: Patients with chronic heart failure may have acute decompensation due to a variety of reasons, including non-compliance with diet or medications, an ischemic event, arrhythmia, or any other factor that increases cardiac demand (anemia, thyrotoxicosis, etc.). In this case, the patient has a tachycardic rhythm, most likely pacemaker-mediated tachycardia, which resulted in the worsening of his heart failure symptoms. The reason why tachycardia can result in acute decompensation of heart failure symptoms is two-fold. First, tachycardia increases the oxygen demand of cardiac muscle, which may cause a supply-demand mismatch. Second, tachycardia causes a decrease in diastolic filling time, which can cause decreased preload, which lowers the cardiac output. Patients who develop pacemaker-mediated tachycardia and stay in that rhythm are prone to develop a variety of symptoms. Classically, they present with palpitations or dizziness. However, in cases where the tachycardia lasts for a long time, in a patient with underlying heart failure, this may cause acute decompensation of the heart failure symptoms. In cases of acute decompensated heart failure in a patient with a permanent pacemaker, careful examination of the EKG, and subsequently, device interrogation should be one of the first steps in the evaluation. Go to the next page if you knew the correct answer, or click the link image(s) below to further research the concepts in this question (if desired).

Research Concepts: Pacemaker Mediated Tachycardia

We update eBooks quarterly and Apps daily based on user feedback. Please tap flag to report any questions that need improvement.

Question 766: A 17-year-old male was brought to an emergency department after he was struck by a car. On arrival, his blood pressure was 112/74 mmHg, heart rate is 114/min, respiratory rate is 14 breaths/min, and oxygen saturation is 90% at room air. On physical examination, the patient had bruises and abrasion all over the body with a laceration of the abdomen. His Glasgow coma scale was 15/15. Examination of the left ear revealed purplish colored tympanic membrane. X-rays of upper and lower limbs demonstrated fractures of the left radius and the left femur. CT scan of the abdomen showed irregular linear areas of hypoattenuation. The patient was immediately shifted to the operating room. He had several surgeries and was improving but still required morphine for pain control. One week after the surgery, he develops a fever and altered mental status. His Glasgow coma scale becomes 8/15. What is the next best step in the management of this patient?

Choices: 1. CT of head and lumbar puncture to rule out meningitis 2. Order a CT scan to rule out a pulmonary embolism 3. Check blood cultures and start antibiotics for sepsis 4. Chest x-ray to evaluate for atelectasis secondary to hypoventilation

Answer: 1 - CT of head and lumbar puncture to rule out meningitis Explanations: A basilar skull fracture allows communication between the nasopharynx or middle ear and the meningeal space. Fractures that involve the petrous ridge of the temporal bone will cause blood to pool behind the tympanic membrane causing it to appear purple. This usually appears within hours of injury and maybe the earliest clinical finding. Altered mental status and fever with known basilar skull fracture require evaluation of the CSF. CSF leak is not easy to diagnose and the fluid should be sent for analysis of beta transferrin. The majority of CSF leaks resolve spontaneously within 5-10 days but some can persist for months. Meningitis may occur in less than 5% of patients but the risk increases with the duration of the CSF leak. Pulmonary emboli are rare in children but a fat embolus could occur. Hypoventilation and atelectasis would have more respiratory symptoms. Go to the next page if you knew the correct answer, or click the link image(s) below to further research the concepts in this question (if desired).

Research Concepts: Basilar Skull Fractures

We update eBooks quarterly and Apps daily based on user feedback. Please tap flag to report any questions that need improvement.

Question 767: A 65-year-old male patient presents to the emergency department due to lower gastrointestinal bleeding after attempting suicide by ingesting an unknown amount of his prescribed medications 6 hours ago. He takes metoprolol and rivaroxaban for atrial fibrillation and thyroxine for hyperthyroidism. On examination, the patient looks drowsy, and his pulse is 98/min, respiratory rate is 16 breaths/min, and blood pressure is 100/70 mm hg. His hemoglobin level is 9.0 g/l; platelet count is 135,000/mcL, PT is 10 seconds, aPTT 38 seconds, and INR 3.3. His capillary blood glucose level is 80 mg/dL. After hospital admission, he is managed with IV fluids and oxygen, but he still complains of shortness of breath and lightheadedness and has ongoing lower gastrointestinal bleeding. Which of the following blood components is the most appropriate choice to control the bleeding in this patient?

Choices: 1. Platelet concentrate 2. Packed red blood cells 3. 4-factor prothrombin complex concentrate 4. Idarucizumab

Answer: 3 - 4-factor prothrombin complex concentrate Explanations: Patients with life-threatening/major bleeding should be managed in an intensive care setting with appropriate hemodynamic support, an antifibrinolytic agent like tranexamic acid, oral activated charcoal when appropriate, and drug discontinuation. Reversal of anticoagulation is generally thought to be desirable in a patient with serious or life-threatening bleeding who remains actively anticoagulated. Patients with refractory bleeding even after massive transfusion or with history of taking direct factor Xa inhibitor like rivaroxaban need an inactivated 4-factor prothrombin complex concentrate comprising of factor II, VII, IV, X or the newer FDA approved reversal agents such as Andexanet Alfa which is a recombinant coagulation factor Xa. There is insufficient evidence to strongly favor either 4-factor PCC or andexanet alfa over the other. If both are unavailable, then a 3-factor PCC can be used in supplementation with FFP to supply factor VII. Platelet concentrate transfusion is not used to reverse the anticoagulant effect of DOACs in a patient with a normal platelet count like this patient. But, thrombocytopenic patients with bleeding should be treated for the underlying cause of thrombocytopenia and receive platelet transfusions if it is severe and bleeding is major or life-threatening. Packed red blood cell is not required in this patient, but RBC transfusion may be required, in case of severe anemia depending on the hemoglobin level, rate of bleeding, and amount of blood loss. Idarucizumab is a humanized monoclonal antibody used in patients treated with dabigatran when a reversal is needed for emergency surgery or procedures, or in the event of life-threatening uncontrolled bleeding. Go to the next page if you knew the correct answer, or click the link image(s) below to further research the concepts in this question (if desired).

Research Concepts: Rivaroxaban

We update eBooks quarterly and Apps daily based on user feedback. Please tap flag to report any questions that need improvement.

Question 768: A 70-year-old, 85 kg man with a history of diabetes and hypertension is admitted to the emergency department with a history of high-grade fever, burning micturition, and altered sensorium. His blood pressure is 70/40 mmHg, pulse 102/min, respiratory rate 24/min. Despite receiving 1.5 L of 0.9% saline over the last hour, his blood pressure has only increased minimally from 70/40 mmHg to 80/50 mmHg, and he has passed 10 mL of urine. He has received appropriate antibiotics. Which of the following is the next best step in the management of this patient?

Choices: 1. A further 500 mL of crystalloid 2. 250 mL boluses of 6% hydroxyethyl starch 3. Intravenous hydrocortisone 100 mg 4. Noradrenaline infusion

Answer: 1 - A further 500 mL of crystalloid Explanations: The latest Surviving Sepsis Guidelines recommends the use of crystalloids, such as 0.9% saline, as the initial fluid for resuscitation for people with severe sepsis. They suggest that patients with sepsis-induced hypoperfusion with suspicion of hypovolaemia can be challenged with as much as 30 mL/kg of crystalloid greater amounts may be needed in some patients. Fluid administration should be continued as long as there is a hemodynamic improvement based on either dynamic (change in pulse pressure or stroke volume variation) or static (heart rate, arterial pressure) variables. Evidence is growing against the use of starches, with recent studies showing an association with renal impairment and the need for renal replacement therapy. The guidelines also discourage intravenous corticosteroid therapy in patients where fluid resuscitation and vasopressors can restore adequate blood pressure. Noradrenaline is the first choice for vasopressor therapy for septic shock refractory to fluid administration, and adrenaline may be added to maintain adequate blood pressure. Go to the next page if you knew the correct answer, or click the link image(s) below to further research the concepts in this question (if desired).

Research Concepts: Urosepsis

We update eBooks quarterly and Apps daily based on user feedback. Please tap flag to report any questions that need improvement.

Question 769: A 65-year-old male was foraging for mushrooms in November in the forest. He had no training in mushroom identification. He thought he picked a white to beige mushroom, with a cap, gills, and annulus that he had eaten previously. It tasted normal in a salad uncooked. Eight hours after eating the mushroom, he developed vomiting, and 12 hours after eating, he had crampy abdominal pain and profuse watery diarrhea. In the emergency department, his initial vital signs include a heart rate of 135 bpm, blood pressure 95/65 mmHg, respiratory rate 20 breaths/min, temperature 98.6 F at 24 hours post-ingestion. On the abdominal exam, the bowel sounds are normal, and there are no peritoneal signs. An intravenous line is started, and a fluid bolus of ringer lactate is given. Labs show he had elevated blood urea nitrogen (35 mg/dl) and creatinine ( 1.4 mg/dl) consistent with dehydration but normal liver transaminases. The next day despite IV fluids, he had rising transaminases (AST and ALT > 2000 IU). After two days of supportive care, he was placed on the liver transplant list for an urgent transplant, and on day four, he had a liver transplant. What is the histopathologic manifestation of liver injury that would be found on his native liver on pathology?

Choices: 1. Centrilobular necrosis 2. Periportal necrosis 3. Veno-occlusive disease 4. Hepatic steatosis

Answer: 1 - Centrilobular necrosis Explanations: The amatoxin of Amanita phalloides is implicated in this hepatic injury. It takes about 24 hours before any signs or laboratory indicators of liver injury begin to appear. The first stage does not begin until six to 12 hours after ingestion. There are no signs of a problem for at least 6 hours. After this silent phase, it is followed by the onset of nausea, abdominal cramps, profuse watery diarrhea, and signs of dehydration. A physical exam may reveal dry mucosal membranes and tachycardia, and given sufficient dehydration, hypotension. After the GI phase, the second stage appears where the patient seems to cover transiently, and GI symptoms resolve, but ongoing liver damage continues. This stage may last two to three days and is characterized by rising liver function transaminase, bilirubin, the development of coagulopathy, and eventually hepatic encephalopathy. In the third stage, both liver and renal function become compromised. Hepato-renal syndrome and hepatic encephalopathy may occur rapidly after laboratory signs of liver injury, and death can occur in three to seven days. The mechanism of action of amatoxin is by inhibiting RNA polymerase, causing disruption of transcription of mRNA. As a result, hepatocytes cannot synthesize key protein-coding genes, leading to the disintegration of nucleoli and pathologically centrilobular hepatic necrosis. The veno-occlusive disease occurs with some toxic plants and herbal teas associated with pyrrolizidine alkaloids. Periportal necrosis is more common in viral liver disease, especially chronic hepatitis. Hepatic steatosis rarely leads to fulminant liver disease but is highly prevalent in a high-fat diet, alcohol misuse, and metabolic syndrome. Go to the next page if you knew the correct answer, or click the link image(s) below to further research the concepts in this question (if desired).

Research Concepts: Amatoxin Mushroom Toxicity

We update eBooks quarterly and Apps daily based on user feedback. Please tap flag to report any questions that need improvement.

Question 770: A 65-year-old male is brought to the emergency department with complaints of difficulty in breathing. His medical history includes chronic obstructive pulmonary disease (COPD), diabetes mellitus, hypertension, sleep-disordered breathing, smoking, and alcohol use. An arterial blood gas (ABG) performed at the time of presentation revealed a pH of 7.25, PaCO2 of 70 mmHg (FiO2 28%), PaO2 of 87 mmHg, and bicarbonate of 42 mmol/L. The patient is admitted to the ICU and is placed on non-invasive ventilation. The usage of AVAPS (Average volume-assured pressure support) modality is associated with which of the following advantages in comparison to the BiPAP (Bilevel positive airway pressure) modality in this patient?

Choices: 1. Low risk of aspiration 2. Rapid clinical improvement 3. Low risk of air leaks 4. Decreased rate of treatment-failure

Answer: 2 - Rapid clinical improvement Explanations: AVAPS has been successful in the treatment of COPD-associated acute hypercapnic respiratory failure. Although both AVAPS and BiPAP are reliable in the treatment of acute hypercapnic respiratory failure, AVAPS is associated with a speedy improvement of the patient clinically. The patients also experience greater comfort and satisfaction comparatively, and therefore, improved treatment-compliance. Rather than having one fixed IPAP setting, the AVAPS mode can set a range of values for the IPAP, a maximum and a minimum IPAP. The pressure-support is no longer fixed as the IPAP changes by itself within the set range. The ventilator does this based on the targeted tidal volume, a pre-set value. It uses a feedback loop to either increase or decreases the inspiratory pressure from breath to breath to ensure the pre-set tidal volume is delivered. Remember that both the therapies (AVAPS and BiPAP) are effectual, and there was no difference in the duration of hospital stay or the time-period that the patient was on noninvasive ventilation. Similar complications and contraindications exist for the varying modes of NIPPV, including AVAPS and BiPAP. Go to the next page if you knew the correct answer, or click the link image(s) below to further research the concepts in this question (if desired).

Research Concepts: Average Volume-Assured Pressure Support

We update eBooks quarterly and Apps daily based on user feedback. Please tap flag to report any questions that need improvement.

Question 771: A 58-year-old man is about to undergo a lower extremity orthopedic surgery. He has an epidural indwelling catheter to manage his pain intraoperatively. It is also planned to leave in his epidural catheter after surgery and connect him to a patient-controlled analgesia machine to manage his pain postoperatively. During the procedure, the patient suddenly reports blurry vision, numbness and tingling around his mouth and a sense of impending doom. What is the next best step in the evaluation of this patient?

Choices: 1. Order a stat potassium level 2. Echocardiogram 3. Electrocardiogram 4. Head CT

Answer: 3 - Electrocardiogram Explanations: Local anesthetic systemic toxicity is a complication of any sodium channel blocker that is being injected. Local anesthetic systemic toxicity occurs when the injection is placed into an artery or vein, or so much has been placed in the local tissue that toxic doses have been absorbed into the cardiovascular system. Symptoms of local anesthetic systemic toxicity are tinnitus, blurred vision, dizziness, tongue parathesias, circumoral numbness, nervousness, agitation, restlessness, and muscle twitching. High concentrations can lead to prolonged PR intervals and widened QRS complexes, and sinus/brady arrest. When local anesthetic systemic toxicity presents in a patient, an electrocardiogram needs to be performed immediately to monitor the effects on the heart and help decide if treatment with Intralipid is necessary. Go to the next page if you knew the correct answer, or click the link image(s) below to further research the concepts in this question (if desired).

Research Concepts: Patient Controlled Analgesia

We update eBooks quarterly and Apps daily based on user feedback. Please tap flag to report any questions that need improvement.

Question 772: A 56-year-old man, post office worker, presents to the emergency department with difficulty breathing and a dry cough with fatigue that has been worsening for the last six days. His son called 911 today after he fell on the floor. He has a medical history of diabetes and hyperlipidemia, for which he takes metformin and simvastatin. He never smoked cigarettes or used illicit drugs. In the emergency department, he becomes severely hypoxemic and requires immediate mechanical ventilation. He gets admitted to the intensive care unit. Work up is negative for infectious, toxic, or autoimmune etiologies. X-rays reveal an interstitial pattern bilaterally. HRCT shows ground-glass opacification. Lung biopsy reveals diffuse alveolar damage with edema in the interstitium and alveolus. What is the definitive management option in such a disorder?

Choices: 1. Rotating antibiotics 2. Glucocorticoids 3. Supportive care 4. Posaconazole

Answer: 3 - Supportive care Explanations: Acute Interstitial Pneumonia (AIP) is an acute, rapidly progressive idiopathic pulmonary disease that often leads to fulminant respiratory failure and acute respiratory distress syndrome (ARDS). Symptoms usually start as viral-like prodrome followed by shortness of breath with cough, and fever, and progress rapidly to acute respiratory distress. There is no proven treatment in acute interstitial pneumonia. Management is mainly based on supportive care. Adequate oxygenation often cannot be achieved without invasive mechanical ventilation. As in acute respiratory distress syndrome, arterial blood gases can identify the severity of respiratory distress based on Berlin Criteria (PaO2/FiO2), which may guide the treatment. There is no proven treatment in acute interstitial pneumonia. Management is mainly based on supportive care. Adequate oxygenation often cannot be achieved without invasive mechanical ventilation. Go to the next page if you knew the correct answer, or click the link image(s) below to further research the concepts in this question (if desired).

Research Concepts: Acute Interstitial Pneumonia

We update eBooks quarterly and Apps daily based on user feedback. Please tap flag to report any questions that need improvement.

Question 773: A 65-year-old farmer was admitted to the hospital following exposure to nitrogen dioxide after entering a silo and inhaling a reddish-brown cloud. The patient was admitted for observation. After 24 hours, he got progressively worsening dyspnea, new bilateral infiltrates on x-ray, and required supplemental oxygenation in the form of bilevel positive airway pressure (BiPAP). Why was this patient's dyspnea and hypoxemia delayed?

Choices: 1. Nitrogen dioxide is highly water-soluble and has delayed effects on the lungs 2. Nitrogen dioxide is highly water-soluble and has an immediate effect on the lungs 3. Nitrogen dioxide has a low water-solubility and therefore can have delayed effects on the lungs 4. The farmer had a concomitant exposure that delayed the effects of nitrogen dioxide

Answer: 3 - Nitrogen dioxide has a low water-solubility and therefore can have delayed effects on the lungs

Explanations: Nitrogen dioxide has a low water-solubility with little to no immediate warning signs. A patient may have prolonged exposure without signs or symptoms. When someone is exposed to nitrogen dioxide, they should be observed for delayed pulmonary effects. The lower the water-solubility, the greater the effect on the lower airways. Go to the next page if you knew the correct answer, or click the link image(s) below to further research the concepts in this question (if desired).

Research Concepts: Nitrogen Dioxide Toxicity

We update eBooks quarterly and Apps daily based on user feedback. Please tap flag to report any questions that need improvement.

Question 774: A 45-year-old man has been placed on venovenous (VV) extracorporeal membrane oxygenation using femoral and internal jugular veins for cannulation for severe refractory hypoxemia secondary to acute respiratory distress syndrome. His extracorporeal membrane oxygenation flow is at 3.5L/min, and cardiac output is 6 L/min. Arterial blood gas showed pH7.38, PaCO2 45 mmHg, PaO2 55 mmHg, HCO3 22 mEq/L. The FiO2 at the membrane is 100%. His current ventilator settings include RR 20/min, tidal volume of 300 ml, PEEP of 5, and FiO2 of 40%. The oxygen saturation of the blood entering the oxygenator is 90%, and the oxygen saturation leaving the oxygenator is 100%, and the oxygen saturation in the venous blood in the superior vena cava is 65%, and the patient’s current arterial oxygen saturation is 85%. How can his hypoxemia be improved?

Choices: 1. Increase the extracorporeal membrane oxygenation blood flow to 6L/min 2. This is the oxygenator problem and must be changed 3. Increase the ventilator settings to a PEEP of 10 and increase the FiO2 to 80% 4. Get an X-ray to look for malposition of cannulas

Answer: 4 - Get an X-ray to look for malposition of cannulas Explanations: This is a clear-cut scenario of recirculation during venovenous extracorporeal membrane oxygenation, and it occurs when well-oxygenated blood returning from the extracorporeal membrane oxygenation circuit gets sucked in by the venous drainage cannula again instead of going through the lungs. The classic signs of recirculation are low patient’s oxygen saturation and high preoxygenator saturations and can be identified by the proximity of the drainage and the reinfusion cannulae on the X-ray. Increasing the pump flow might improve the oxygen saturations but will eventually increase recirculation and causing further hypoxemia. The complication of recirculation is very exclusive to venovenous extracorporeal membrane oxygenation and can be minimized by increasing the distance between the drainage and reinfusion cannulae. Go to the next page if you knew the correct answer, or click the link image(s) below to further research the concepts in this question (if desired).

Research Concepts: Refractory Hypoxemia and Venovenous ECMO

We update eBooks quarterly and Apps daily based on user feedback. Please tap flag to report any questions that need improvement.

Question 775: A 70-year-old man is brought to the hospital with confusion. He is windowed and stays alone at home, but is able to perform day to day activities quite independently. His kids visit him once a week or so. He had an intermittent low-grade fever for the past week, for which his primary care provider had prescribed acetaminophen tablets. His neighbor noticed subtle behavioral changes since the past week. Grandchildren who visited him today found him irrelevant, abusive and aggressive. They also noticed that he has had passed urine and stools in the living room. CT brain is unremarkable. A urine toxicology screen is negative. CSF study shows RBCs 150 microL, WBCs 300/mmicroL (90% lymphocytes). The protein is 136 mg/dL and glucose 50 mg/dL (corresponding blood glucose is 85 mg/dL). Remaining CSF reports are awaited. Which of the following evaluations is the gold standard to confirm the diagnosis in this patient?

Choices: 1. Brain biopsy 2. MRI brain with diffusion-weighted images and FLAIR sequences 3. Clinical symptoms and signs 4. CSF polymerase chain reaction (PCR)

Answer: 4 - CSF polymerase chain reaction (PCR) Explanations: The most sensitive and specific way to diagnose HSV encephalitis is to do an HSV PCR in CSF. HSV PCR has got a sensitivity of 98% and a specificity of 94-100%. HSV PCR could be false negative in the initial 72 hours and needs to be repeated after 3-7 days if the suspicion is high. RBCs in the CSF can also give a false negative test. Herpes simplex virus (HSV) can be type 1 or type 2. Herpes simplex virus type 1 (HSV-1) is the most common cause of life-threatening sporadic encephalitis in the world. It causes encephalitis in adults and children beyond the neonatal age. Neonatal herpes encephalitis could be either due to type 1 or type 2, the latter being more common. Herpes simplex encephalitis is associated with significant morbidity and mortality in children and adults. A high index of suspicion, rapid diagnostic workups, and early diagnosis with early initiation of acyclovir can further improve morbidity and mortality. Clinical signs and symptoms are highly non-specific. Immunocompromised or elderly can have atypical or subtle presentations. Cognitive, behavioral and personality changes could be easily misdiagnosed as a psychiatric disorder. Classic radiologic features that have been described include asymmetric involvement of the limbic system, abnormalities in the inferior medial temporal lobes and inferolateral frontal lobes, and the insular cortices. Diffusion-weighted MRI with flair sequences is 90% sensitive in detecting changes in HSE (herpes simplex encephalitis). CT can detect changes only in half to two-thirds of patients. Brain biopsy is very rarely performed nowadays, and considerations include undiagnosed brain pathology where the patient is further deteriorating, or not responding to antiviral treatment in spite of a positive HSV PCR. Go to the next page if you knew the correct answer, or click the link image(s) below to further research the concepts in this question (if desired).

Research Concepts: Herpes Simplex Encephalitis

We update eBooks quarterly and Apps daily based on user feedback. Please tap flag to report any questions that need improvement.

Question 776: A 65-year-old woman with a history of diabetes mellitus presents to the emergency room with complaints of dysuria, nausea, vomiting and abdominal pain. Her vitals are blood pressure 85/45 mmHg, heart rate 110/minute, temperature 101.5 F (36.6 C). After 3 liters of intravenous ringer lactate, there is no improvement in blood pressure. She is started on norepinephrine infusion. The next day her creatinine is noted to be 2.1 mg/dL from baseline of 0.9 mg/dL. A urinalysis is sent and microscopic slide prepared. What is the most likely findings to be seen in the urine sediment?

Choices: 1. Bland sediment, few renal tubular cells 2. Coarse, granular casts, renal tubular epithelial cells 3. Dysmorphic red blood cells 4. White blood cells, white blood cell casts

Answer: 2 - Coarse, granular casts, renal tubular epithelial cells Explanations: The patient is presenting with septic shock due to urosepsis, blood pressure is low and has signs of infection. Renal injury associated with septic shock is acute tubular necrosis. In acute tubular necrosis tubular cells are injured so reabsorption can not take place. Hence the fractional excretion of sodium >1%, sediment shows renal tubular epithelial cells along with casts, coarse, granular casts, also known as "muddy brown" casts. Dysmorphic red blood cells and casts are seen in glomerulonephritis (example lupus nephritis, anticytoplasmic antineutrophilic antibody (ANCA) vasculitis. White blood cells and casts are seen more in acute interstitial nephritis. Bland sediment with few renal tubular epithelial cells are seen in prerenal azotemia. Other findings are fractional excretion of sodium 1%, BUN/ Cr ratio >20. These can be seen in volume depletion due to gastrointestinal losses Go to the next page if you knew the correct answer, or click the link image(s) below to further research the concepts in this question (if desired).

Research Concepts: Renal Failure

We update eBooks quarterly and Apps daily based on user feedback. Please tap flag to report any questions that need improvement.

Question 777: A 65-year-old man is brought to the emergency department by his son for chest pain and difficulty breathing for the past 8 days. He mentions that he developed headaches, watery diarrhea, and abdominal pain 2 days ago. He recently returned from a cruise. The patient smokes a pack of cigarettes for the past 36 years. Vital signs show blood pressure of 98/62 mmHg, pulse of 59/min, respiratory rate of 24/min, and temperature of 101 F (38.3 C). He appears confused. Examination reveals lung crackles. Laboratory investigations reveal a sodium level of 129 mEq/L, blood urea nitrogen level of 34 mg/dL, and elevated liver enzymes. Presence of which of the following features in this patient most strongly relates to disease severity?

Choices: 1. Blood urea nitrogen level of 34 mg/dL 2. Sodium level of 129 mEq/L 3. Respiratory rate of 24/min 4. Pulse of 59/min

Answer: 1 - Blood urea nitrogen level of 34 mg/dL Explanations: This patient most likely has Legionnaire's disease. The most important clinical infection caused by Legionella pneumophila in both children and adults is pneumonia. In patients of severe disease, expect respiratory failure, bilateral pneumonia, pulmonary infiltrates, and the presence of at least two of the following: diastolic blood pressure lower than 60 mm Hg; respiratory rate greater than 30/min; and blood urea nitrogen greater than or equal to 30 mg/dL. The morbidity is highest in very young and very old patients, those who have chronic lung disease, are immunosuppressed, patients with diabetes mellitus, and have delayed antimicrobial therapy. Bradycardia and hyponatremia are features of the disease and are not linked to disease severity. Go to the next page if you knew the correct answer, or click the link image(s) below to further research the concepts in this question (if desired).

Research Concepts: Legionnaires' Disease

We update eBooks quarterly and Apps daily based on user feedback. Please tap flag to report any questions that need improvement.

Question 778: This 45-year old Indian lady has been ventilated for severe acute respiratory distress syndrome secondary to COVID-19 pneumonia. Her oxygen requirement has gone up in the past few minutes. Air entry was equal on both sides, and a bedside chest ultrasound revealed the presence of pleural slides bilaterally, with bibasal consolidation and copious B lines. The echocardiography was normal. Her current ventilator settings are show volume control ventilation FiO2 0.7, tidal volume 350 cc, respiratory rate 32 per minute, PEEP 12 cms H2O. Her predicted body weight (PBW) is 60 kg. Her latest ABG revealed PaO2 56 mm Hg, PaCO2 68 mm Hg, pH 7.273, HCO3 38 mEQ/L. The peak pressure is 38 cms H20, and the plateau pressure is 30 cms. What should be the next best step in the management of this patient?

Choices: 1. Increase FiO2 and PEEP 2. Recruitment maneuvers 3. Proning 4. Diuresis

Answer: 3 - Proning Explanations: This lady has severe ARDS with a P/F ratio of 80 is very likely to get benefit from prone position ventilation. Prone position ventilation is not only known to improve hypoxemia in ARDS patients but also known to minimize lung injury by increasing homogeneity of ventilation. The latter aspect has been shown in animal studies. Prone position ventilation has been shown to improve the mortality in ARDS patients, as shown in a 2010 metanalysis, and also at the landmark PROSEVA trial on severe ARDS patients published in 2013. The reduced mortality is believed to be partly contributed by decreased risk for ventilatory induced lung injury since prone position ventilation improves the homogeneity of ventilation. Prone position ventilation improves oxygenation by improving the end-expiratory lung volume and by rectifying the ventilation-perfusion mismatch. Also, it reduces the mass effect of the heart and mediastinum on the lower lobes and improves regional ventilation. Prone position ventilation is relatively avoided in hemodynamically unstable patients and patients with unstable spinal cord or abdominal injuries. Increased incidence of pressure ulcers, increased endotracheal tube obstruction, and Intercostal tube displacements are reported with prone position ventilation. ECMO is an alternate option to be considered when there is a strong contraindication/inability to perform prone positioning. Go to the next page if you knew the correct answer, or click the link image(s) below to further research the concepts in this question (if desired).

Research Concepts: Ventilator-Induced Lung Injury (VILI)

We update eBooks quarterly and Apps daily based on user feedback. Please tap flag to report any questions that need improvement.

Question 779: A 66-year-old female presents to the emergency department in severe respiratory distress. She has been having trouble breathing for the last two hours. She has had chronic obstructive pulmonary disease (COPD) for eight years and is on home oxygen therapy. Sometimes, she experiences acute exacerbations, but she describes her current symptoms as being severe. Her respiratory rate is 42/min, heart rate is 98, BP is 118/70 mmHg, and her temp is 98 F. Pulmonary examination shows decreased breath sound on both sides; the right side is higher. She was resuscitated immediately, intubated, and mechanically ventilated. Her bedside xray shows airspace disease suggestive of collapsed alveoli. What is the best way to monitor the progress of her lung disease and allow for the optimization of mechanical ventilation settings in the intensive care unit?

Choices: 1. Continuous pulse-oximetry and monitoring of her percentage oxygen saturation 2. Continuous monitoring of arterial blood pressure to guide judicious fluid administration and minimize pulmonary edema 3. Continuous monitoring of respiratory compliance using continuous flow technique and examining pressure-volume curve measurements 4. Daily bed-side x-ray and assessing evolution of the airspace disease

Answer: 3 - Continuous monitoring of respiratory compliance using continuous flow technique and examining pressure-volume curve measurements

Explanations: The patient is intubated as a result of acute respiratory distress syndrome (ARDS) and it is vital to monitor the progress of her lung disease. Respiratory compliance is a measure of the change in volume relative to the change in pressure at any given time. The ventilator can measure pulmonary compliance continuously. This measurement permits analysis of the static mechanical properties of the respiratory system. The static pressure-volume curve properties are impaired in ARDS. Continuous flow technique is a simple and reliable tool to measure respiratory compliance and helps in monitoring lung injury in patients with ARDS in ICU ventilator settings. Go to the next page if you knew the correct answer, or click the link image(s) below to further research the concepts in this question (if desired).

Research Concepts: Pulmonary Compliance

We update eBooks quarterly and Apps daily based on user feedback. Please tap flag to report any questions that need improvement.

Question 780: A 74-year-old male has been admitted to the hospital with malignant hypertension. His medical history is remarkable for congestive heart failure, hypercholesterolemia, alcoholic cirrhosis, and hypertension. He is started on an infusion of sodium nitroprusside at a rate of 0.5 mcg/kg/minute that is titrated up to 2 mcg/kg/minute over a period of 24 hours. The next day, he develops persistent bradycardia and confusion. O2 saturation is 89% on room air. Venous Blood gas demonstrates a pH of 7.05, pO2 of 102 mm Hg, and pCO2 of 20 mm Hg, and bicarbonate of 8 mmol/L. What is the most appropriate treatment for this patient?

Choices: 1. Dialysis 2. Thiosulfate infusion 3. Activated charcoal 4. Hyperbaric oxygen

Answer: 2 - Thiosulfate infusion Explanations: Nitroprusside is an effective intravenous antihypertensive medication that has a rapid onset of action. It is approved for use in acute decompensated heart failure and acute hypertension. Prolonged use of nitroprusside can result in excess cyanide production. Cyanide toxicity results in lactic acidosis and altered mental status and can develop in a matter of hours following administration. Close attention must be paid to acidemia and dropping serum bicarbonate levels (although these parameters are not always reliable indicators of toxicity in early stages). The cyanide toxicity can be treated with infusion of sodium thiosulfate, which detoxifies cyanide into thiocyanate by acting as a sulfur donor. Aside from cyanide toxicity, other important side effects of sodium nitroprusside are profound hypotension with resultant end-organ ischemia, increased intracranial pressure, methemoglobinemia, and thiocyanate toxicity. Go to the next page if you knew the correct answer, or click the link image(s) below to further research the concepts in this question (if desired).

Research Concepts: Sodium Nitroprusside

We update eBooks quarterly and Apps daily based on user feedback. Please tap flag to report any questions that need improvement.

Question 781: A 67-year-old man with a history of chronic obstructive pulmonary disease (COPD) was transferred to the medical floor from the intensive care unit (ICU). He was admitted to ICU three days ago due to acute hypoxic respiratory failure secondary to community-acquired pneumonia requiring intubation and mechanical ventilation. On the medical floor, he has new symptoms of abdominal discomfort, nausea, and two episodes of vomiting containing a small amount of blood over the past several hours. He has no prior medical problems other than COPD and has never had similar symptoms. The patient smokes a pack of cigarettes daily and drank 12 cans of beer before presentation to the emergency department. Vital signs currently are normal. Abdominal examination reveals mild epigastric tenderness, but no masses, rebound or guarding. Which of the following is the most likely cause of this patient's hematemesis?

Choices: 1. Partial esophageal tear 2. Pancreatic autodigestion Injury 3. Gastric mucosal erosion 4. Dilated esophageal vessels

Answer: 3 - Gastric mucosal erosion Explanations: Acute erosive gastropathy Is characterized by the development of severe hemorrhagic lesions after the exposure of gastric mucosa to various injurious agents or after a substantial reduction in blood flow. Presenting symptoms include abdominal discomfort, nausea, vomiting, and hematemesis. Mechanical ventilation for 48-72 hours is a risk factor for stress-related gastritis and gastric ulcer. Mallory-Weiss tear is a partial tear in the esophagus that occur in the distal esophagus at the gastroesophageal junction after repeat bouts of retching and vomiting. This patient’s vomiting is not severe enough to cause an esophageal tear. Go to the next page if you knew the correct answer, or click the link image(s) below to further research the concepts in this question (if desired).

Research Concepts: Curling Ulcer

We update eBooks quarterly and Apps daily based on user feedback. Please tap flag to report any questions that need improvement.

Question 782: A 65-year-old woman with a past medical history of a cerebral vascular accident two weeks ago is brought to the emergency department with shortness of breath and fever. She is diagnosed with septic shock and the decision is made to intubate her. Which of the following is the most appropriate paralytic for intubation?

Choices: 1. Succinylcholine 2. Ketamine 3. Propofol 4. Rocuronium

Answer: 4 - Rocuronium Explanations: Rocuronium is the correct answer. Depolarizing muscle agents, such as succinylcholine, are contraindicated in cases of neurologic injury due to post-synaptic receptor up-regulation that typically occurs within three to five days. A cerebral vascular accident two weeks ago places the patient at risk for life-threatening hyperkalemia. The risk of hyperkalemia is not associated with decreased potassium clearance however attention should be given to those who have chronically elevated potassium levels such as renal failure patients. Ketamine and Propofol are commonly used induction agents for both intubation and procedural sedation. They are not neuromuscular blocking drugs. The conjunctive use of induction agents is vital to rapid sequence intubation to reduce the sympathetic reflexes, improve intubating conditions, and avoid the unwarranted effect of paralyzing a conscious patient. The most well-known depolarizing neuromuscular blocking agent is succinylcholine. It is the only such drug used clinically and is considered by many the drug of choice for emergency department rapid sequence intubation. It provides the fastest optimal conditions during intubation of critically ill patients. However, it has several contraindications that need to be considered before administration. Advocates of rocuronium cite its lack of contraindications and avoidance of depolarization in the middle of an intubation attempt. Go to the next page if you knew the correct answer, or click the link image(s) below to further research the concepts in this question (if desired).

Research Concepts: Depolarizing Neuromuscular Blocking Drugs

We update eBooks quarterly and Apps daily based on user feedback. Please tap flag to report any questions that need improvement.

Question 783: A 76-year-old woman presents to the emergency department with abdominal pain that radiates to her back. On admission, her heart rate is 110/min, with blood pressure 87/62 mmHg. Her past medical history includes coronary artery disease and documented abdominal aortic aneurysm. An ultrasound reveals free intraperitoneal fluid and abdominal aortic aneurysm with a diameter of 6 cm. What is the best next step in the management of this patient?

Choices: 1. Exploratory laparotomy 2. Emergent CT scan 3. Cardiological evaluation for elective repair 4. Endovascular stent-graft

Answer: 1 - Exploratory laparotomy Explanations: A ruptured abdominal aortic aneurysm is an emergent situation. The risk of death from a ruptured aortic aneurysm is as high as 90%. Patients should be divided into 2 categories; stable and unstable (as in this case), according to hemodynamic factors, such as blood pressure, hurt rate, etc. A prompt aortic repair should be performed. Since abdominal aortic rupture is an emergent situation, diagnosis is made clinically or with the support of ultrasound. Abdominal aortic aneurysm rupture most commonly presented with a combination of the 3; hypotension, pulsatile mass, and abdominal/back pain. Ultrasound may demonstrate free intraperitoneal fluids or abdominal aortic aneurysm rupture. In the case of an unstable patient with suspected abdominal aortic aneurysm rupture stabilizing and immediate open surgery should be performed. In case of a stable patient, a CT aortogram should be performed, to confirm the diagnosis and assesses suitability for emergency endovascular stent-graft repair. Even though before an elective repair of abdominal aortic aneurysm all patients should overgo preoperative cardiological evaluation, in case of an unstable patient with a ruptured abdominal aortic aneurysm, preoperative cardiological evaluation is not held. Even if a patient carries a high risk for cardiological related complication, surgery should not be delayed. Go to the next page if you knew the correct answer, or click the link image(s) below to further research the concepts in this question (if desired).

Research Concepts: Abdominal Aortic Repair

We update eBooks quarterly and Apps daily based on user feedback. Please tap flag to report any questions that need improvement.

Question 784: A 17-year-old boy admitted to the hospital complains of a new-onset chest pain that started a few hours ago. It is associated with mild shortness of breath. He also complains of fever and a cough. On further questioning, he reveals that the cough is associated with yellow-colored sputum production. A detailed medical history reveals that he had met with an accident and underwent nephrectomy a week ago. He has no known medical conditions. His vital signs show blood pressure 100/70 mmHg, respiratory rate 22 breaths/min, heart rate 98 beats/min, temperature 102 F (38.8 C), and oxygen saturation of 88% on room air. A chest examination performed reveals late inspiratory crackles on auscultation. A chest x-ray performed shows an area of opacity in the left lung. What is the most appropriate next step in management?

Choices: 1. Prescribe piperacillin-tazobactam 2. Prescribe vancomycin 3. Prescribe vancomycin and levofloxacin 4. Prescribe linezolid

Answer: 1 - Prescribe piperacillin-tazobactam Explanations: Pneumonia that occurs 48 -72 hours after hospital admission is known as hospital-acquired pneumonia (HAP) and usually presents with signs and symptoms of cough with sputum, fever, and shortness of breath. Patients with HAP without any risk factors for MRSA (methicillin-resistant staph aureus) infection such as prior intravenous antibiotic use within three months, hospitalization in a unit with 20% or above of methicillin-resistant Staphylococcus aureus isolates, or unknown prevalence of MRSA, and low risk of mortality are prescribed antibiotics with activity against MSSA (methicillin-sensitive staph aureus) like piperacillin-tazobactam. A physical examination reveals dullness on percussion, an increase in vocal fremitus, and crackles on auscultation. Other treatment options include cefepime, levofloxacin, imipenem, or meropenem. Vancomycin and linezolid are given in case of MRSA infection and should not be used in MSSA infections. Go to the next page if you knew the correct answer, or click the link image(s) below to further research the concepts in this question (if desired).

Research Concepts: Nosocomial Pneumonia

We update eBooks quarterly and Apps daily based on user feedback. Please tap flag to report any questions that need improvement.

Question 785: A 60-year-old woman is ready to be discharged on the sixth postoperative day following an uneventful total hip replacement. A physical exam shows localized necrosis of skin at the sites of subcutaneous injection of enoxaparin on the abdominal wall. Labs show a fall in the platelet count from 216000/microL (five days ago) to 64000/microL (today). Which of the following is the next best step in the management of this patient?

Choices: 1. Reduce the dose of enoxaparin by half and administer two units of FFP 2. Switch enoxaparin to fondaparinux 3. Stop enoxaparin and transfuse two units of adult platelets 4. Hold enoxaparin only

Answer: 2 - Switch enoxaparin to fondaparinux Explanations: This patient’s history is suggestive of heparin-induced thrombocytopenia (HIT). HIT is an adverse drug reaction to heparin. The incidence of HIT is estimated at 1% with low-molecular-weight heparins (LMWH) and about 5% with unfractionated heparins (UFH). HIT has been described with every route of heparin administration. Where clinical suspicion of HIT is intermediate to high, it is essential to stop UFH or LMWH. There should be no delay in commencing anticoagulation with alternative agents like bivalirudin or fondaparinux while awaiting confirmatory tests, as the risk of thrombosis remains as high as 50% even after stopping heparin. Adverse reactions are either non-immune-mediated (type I) or immune-mediated (type II). The non-immune mediated reaction typically has an earlier onset and seldom leads to a drop in the platelet count below 100000/microL. The immune-mediated reaction is clinically more significant as it is associated with thrombosis. It occurs between 5 to 14 days postheparin exposure, known as ‘typical’ HIT. Go to the next page if you knew the correct answer, or click the link image(s) below to further research the concepts in this question (if desired).

Research Concepts: Heparin Induced Thrombocytopenia

We update eBooks quarterly and Apps daily based on user feedback. Please tap flag to report any questions that need improvement.

Question 786: A 65-year-old male with unknown past medical history is brought to the emergency department by ambulance for chest pain that began 20 minutes ago. On arrival, the patient is non-verbal but does groan upon sternal chest rub. His heart rate is 150 bpm, blood pressure 65/48 mm Hg, and SpO2 88 %. His EKG shows a QRS width of 140 ms. What is the next step in management?

Choices: 1. Obtain an EKG and place on a cardiac monitor 2. Start cardiopulmonary resuscitation 3. Obtain IV access and start aggressive fluid resuscitation 4. Proceed to intubate

Answer: 4 - Proceed to intubate Explanations: Providers should always remain vigilant when managing wide complex tachycardia as these patients can quickly become unstable and experience sudden cardiovascular collapse. It is critical to first evaluate both the patient’s ABCs (airway, breathing, circulation) and hemodynamic status because it will guide your management in the emergency setting. In this scenario, the patient is hemodynamically unstable and does not appear to be protecting his airway; therefore, he should be intubated. Obtaining an electrocardiogram or maintaining an intravenous line is not the best course of action. The patient should be intubated. Go to the next page if you knew the correct answer, or click the link image(s) below to further research the concepts in this question (if desired).

Research Concepts: Wide QRS Complex Tachycardia

We update eBooks quarterly and Apps daily based on user feedback. Please tap flag to report any questions that need improvement.

Question 787: An 80-year-old woman is admitted to the hospital with progressively worsening dyspnea of one-month duration. For one week, she has been sleeping on a recliner. She states that she can hardly walk one block without getting short of breath. She also complains of worsening lower extremity swelling. Currently, she feels depressed as she is unable to lead an active life, which is a big change compared to a month ago. She has a 3-year history of ischemic cardiomyopathy and takes aspirin, bisoprolol, digoxin, spironolactone, and furosemide. On examination, the patient is slightly anxious and has cold peripheries. Bilateral lower extremity pitting edema is noted up to the mid-shins. She is afebrile with a pulse rate of 110/min (regular), blood pressure of 80/60mmhg, and respiratory rate of 30/min. Jugular venous distention is present. Auscultation is significant for an S3 and bilateral basal rales. Laboratory studies are significant for mildly deranged liver enzymes (AST- 160 U/L, ALT- 173 U/L), elevated creatinine (3.1mg/dl), hyponatremia (130meq/l). Serum digoxin levels are low (0.2ng/ml). EKG shows sinus tachycardia. Which of the following is the next best step in the management of this patient?

Choices: 1. Enroll the patient for the transplant list 2. Increase the dose of digoxin 3. Start on dobutamine 4. Start her on milrinone

Answer: 3 - Start on dobutamine Explanations: The cardiogenic shock is defined by persistent symptomatic hypotension and end-organ dysfunction. Symptoms suggesting this include worsening exertional dyspnea and orthopnea. Cold peripheries (suggesting poor peripheral perfusion) with leg edema, S3, and basal crepitations are some signs of shock. End-organ damage which is evident with elevated liver enzymes and deranged renal functions can be noted in patients with cardiogenic shock. Heart failure leading to cardiogenic shock is treated with ionotropic agents like milrinone or dobutamine. Dobutamine acts on beta1 receptors of the heart and induces the ionotropic effect. Even though milrinone has a similar effect, it is renally excreted and could get accumulated in patients with renal failure. Beta-blockers are usually not recommended for patients in cardiogenic shock with volume overload. This has a negative ionotropic effect and should be restarted once the patient tides over this crisis situation and is being diuresed adequately. The primary goal should be to improve cardiac function. Digoxin is known to reduce the hospitalizations in patients with heart failure, but cannot be increased in patients with deranged renal functions. Go to the next page if you knew the correct answer, or click the link image(s) below to further research the concepts in this question (if desired).

Research Concepts: Cardiogenic Shock

We update eBooks quarterly and Apps daily based on user feedback. Please tap flag to report any questions that need improvement.

Question 788: A 19-year-old woman presents to the clinic after having been found to have dysarthria, ataxia, drooling, and Parkinsonism. Her husband reports no recent hospitalizations, but she did see her obstetrician a few months ago for placement of an intrauterine device (IUD) for emergency contraception as they were not ready to start a family. He explains that she has been told she has a defective gene on chromosome 13 that was prevalent in her family. Which of the following is the next best step in the management of this patient?

Choices: 1. Emergent plasmapheresis 2. Remove her intrauterine device (IUD) 3. Initiate trientine therapy 4. Genetic analysis

Answer: 2 - Remove her intrauterine device (IUD) Explanations: Patients with Wilson disease are very sensitive to increases in total body copper levels. Wilson disease is an autosomal recessive disorder caused by a mutation in the ATP7B gene on chromosome 13, leading to impaired copper excretion into bile, leading to excess copper accumulation within hepatocytes and subsequently extrahepatic copper deposition. The copper IUD is routinely used as an emergency contraception method. Removal of sources of excess copper and subsequent chelation with a first-line agent such as penicillamine should be pursued. Go to the next page if you knew the correct answer, or click the link image(s) below to further research the concepts in this question (if desired).

Research Concepts: Copper Toxicity

We update eBooks quarterly and Apps daily based on user feedback. Please tap flag to report any questions that need improvement.

Question 789: A 65-year-old patient in the intensive care unit (ICU) develops abdominal pain and diarrhea. He was admitted to the ICU for acute infective exacerbation of chronic obstructive pulmonary disease (COPD) 7 days back. He was started on intravenous ceftriaxone and moxifloxacin for his exacerbation. His condition seemed to be improving; however, today, he has developed abdominal pain and has had 6 episodes of loose stools since morning. He has also spiked a fever and complains of crampy abdominal pain. Examination reveals a pulse 105 beats per minute, a temperature of 100 F, respiratory rate of 22 per minute, and blood pressure of 100/70 mmHg. His oxygen saturation is 93 % on room air, and he is not dyspneic. Abdominal examination reveals tenderness over the left lower quadrant and increased bowel sounds. His investigations show a total leucocyte count of 20,000 per microL, serum creatinine of 2.1 mmol/l, and a C-reactive protein of 85 mg/L. What is the appropriate treatment for the patient's condition?

Choices: 1. Intravenous piperacillin-tazobactam 2. Intravenous hydrocortisone 3. Oral vancomycin 4. Oral loperamide

Answer: 3 - Oral vancomycin Explanations: This is an elderly patient who has been admitted to intensive care with acute exacerbation of chronic obstructive airway disease. He has been on broad-spectrum antibiotics and has now developed fever and diarrhea, making the diagnosis of antibiotic-associated Clostridium difficile colitis likely. The treatment of choice for Clostridium difficile colitis is oral vancomycin. Development of diarrhea with greater than 3 loose stools in 24 hours in a patient who is at risk (hospitalization, broad-spectrum antibiotic use, elderly) should raise suspicion for Clostridium difficile colitis induced colitis. The management options include oral vancomycin, oral metronidazole, or oral fidaxomicin. The initial dose recommendation for vancomycin is 125 mg every 6 hours, with a duration of treatment of 10 days. Intravenous vancomycin is not effective as it is not secreted in the colon. Occasionally, patients may present as ileus and rectal vancomycin as a retention enema can be administered. Vancomycin can also be used for recurrence and is administered in a pulsed and tapered regime. For prevention, unnecessary broad-spectrum antibiotic use should be avoided, and shifting to narrow-spectrum agents should be instituted as early as possible. Go to the next page if you knew the correct answer, or click the link image(s) below to further research the concepts in this question (if desired).

Research Concepts: Clostridium Difficile

We update eBooks quarterly and Apps daily based on user feedback. Please tap flag to report any questions that need improvement.

Question 790: A 67-year-old man presents with generalized abdominal pain for one week. He also complains of worsening edema in both of his lower limbs and gaining about 10 pounds (4.5 kg) in the last month. His past medical history includes multiple myeloma and alcohol use disorder. Vitals reveal oxygen saturation 97% on 2 liters oxygen, respiratory rate 16 per minute, heart rate 95 beats per minute, blood pressure 105/70 mmHg, and temperature 37 C. On physical examination, his abdomen is distended with 2+ edema in both lower limbs. On palpation, his abdomen is tense and diffusely tender. Lung and heart sounds are normal. Laboratory studies show WBC 2 x 10^9/L (4-10), hemoglobin 8.5 g/dL (13-17), platelets 65 x 10^9/L (150-400), sodium 128 mmol/L (135-145), potassium 5.5 mmol/L (3.5-5.0, chloride 101 mmol/L (95-105), blood urea nitrogen mg/dL 70 (8-21), creatinine 7.2 (baseline 0.9), calcium 1.0 mmol/L (1.031.23), AST 55 U/L (5-30), ALT 34 U/L (5-31). What is the next best step in the management of this patient?

Choices: 1. Kidney biopsy 2. Intravenous hydration 3. Kidney ultrasound 4. Bladder pressure measurement

Answer: 4 - Bladder pressure measurement Explanations: It is essential to rule out intraabdominal hypertension and abdominal compartment syndrome in patients presenting with abdominal distension and signs of multiorgan failure. The normal intra-abdominal pressures range between 0 to 5 mmHg, while pressures between 12 to 20 mmHg are indicative of intra-abdominal hypertension (IAH). Abdominal compartment syndrome (ACS) is defined as an intra-abdominal pressure of more than 20 mmHg along with signs of organ failure. The increased abdominal pressure decreases the compliance of the lungs, leading to hypoxemia and increased afterload due to a decrease in venous return. Eventually, this can lead to multi-organ failure. Although ultrasound is recommended, the bladder pressure should be done immediately after the abdominal examination findings reveal a firm or tense abdomen in the setting of decreased urine output or renal failure. If the patient has ascites, it needs to be drained, and a surgical consult is needed in cases with acute abdomen or perforated viscus. Go to the next page if you knew the correct answer, or click the link image(s) below to further research the concepts in this question (if desired).

Research Concepts: Abdominal Compartment Syndrome

We update eBooks quarterly and Apps daily based on user feedback. Please tap flag to report any questions that need improvement.

Question 791: Which of the following is the best initial management of the patient with diabetic ketoacidosis who becomes comatose after 12 hours of aggressive treatment?

Choices: 1. Hyperventilation 2. Stop insulin therapy 3. Stop K+ supplementation 4. Oxygen therapy

Answer: 1 - Hyperventilation Explanations: Hyperventilation reduces the blood flow through the brain by causing cerebral vasoconstriction. Stopping insulin therapy is not recommended. Other modes of treatment would be mannitol to promote diuresis. The most basic definition of cerebral edema is swelling of the brain. It is a relatively common phenomenon with numerous etiologies. Cerebral edema categorizes into either vasogenic, cellular, osmotic, and interstitial causes. It can arise from a variety of causes, including head trauma, vascular ischemia, intracranial lesions, or obstructive hydrocephalus resulting in interstitial edema. The consequences of cerebral edema can be devastating, even fatal, if untreated. The explanation of the mechanism of injury arising from cerebral edema comes via the Monroe-Kellie doctrine. The Monroe-Kellie doctrine states that space of the cranial cavity is fixed in volume and contains fixed proportions of brain matter (approximately1400 ml), blood (approximately 150 ml) and cerebrospinal fluid (approximately 150 ml). Because of this fixed space, an increase in the volume of one of these components must, therefore, result in the loss of another component in equal amounts. In cerebral edema, the relative volume of brain tissue increases as the brain tissues swells with edema. This increased relative brain volume decreases perfusion (blood) to the brain, and the pressure can cause further damage to both the edematous and non-edematous brain. Clinical presentation of cerebral edema is variable, ranging from asymptomatic to severe autonomic dysregulation, coma, and death. Symptoms appear as the intracranial pressure (ICP) rises above 20 cm H2O in most patients. Treatment for cerebral edema targets the underlying cause and any life-threatening complications. Treatments include hyperventilation, osmotherapy, diuretics, corticosteroids, and surgical decompression. Go to the next page if you knew the correct answer, or click the link image(s) below to further research the concepts in this question (if desired).

Research Concepts: Cerebral Edema

We update eBooks quarterly and Apps daily based on user feedback. Please tap flag to report any questions that need improvement.

Question 792: A 17-year-old male with a past medical history of asthma and depression is found unconscious by a bystander and taken to an emergency department. He is minimally responsive, so a detailed history cannot be obtained. On examination, his blood pressure is 100/70 mmHg, pulse 112/min, respiratory rate 12/min, and oxygen saturation 99% on room air. His pupils are reactive to light, and his skin is flushed. Abdominal examination reveals decreased bowel sounds and suprapubic fullness. An EKG shows QRS complex widening. Lab results, including liver function tests, are unremarkable. His blood-alcohol level is less than 5%, and the urine toxicology screen is negative. What is the best next step in management?

Choices: 1. Check an acetaminophen level 2. Observation 3. Administer IV sodium bicarbonate 4. Administer flumazenil

Answer: 3 - Administer IV sodium bicarbonate Explanations: Considering the patient is relatively unresponsive and the liver function tests are normal, an acetaminophen level would be one of the tests to consider, but not necessarily the best next step based on the presentation. If the patient was stable and responsive, observation could be the next appropriate step. At this time, an intervention is needed as the patient is minimally responsive. The question stem indicates the patient presents with likely unknown intoxication, widening of the QRS complex, reduced bowel sounds, and urinary retention which is indicative of tricyclic antidepressant toxicity. The best next step would be administering IV sodium bicarbonate. This would increase the sodium load and alkalinization, thereby being cardioprotective. Flumazenil is used in benzodiazepine intoxication. Patients with benzodiazepine intoxication are sedated, and present with decreased respiratory rate but do not have specific EKG findings as mentioned in the question stem. Go to the next page if you knew the correct answer, or click the link image(s) below to further research the concepts in this question (if desired).

Research Concepts: Amitriptyline

We update eBooks quarterly and Apps daily based on user feedback. Please tap flag to report any questions that need improvement.

Question 793: Pulmonary capillary wedge pressure increases in which cardiac disorder? Choices: 1. Tricuspid stenosis 2. Right ventricle failure 3. Pulmonary hypertension 4. Mitral stenosis

Answer: 4 - Mitral stenosis Explanations: Left sided heart failure can increase PCWP. The normal left ventricular diastolic pressure is 5 mm Hg. A pressure gradient across the mitral valve of 20 mm Hg due to severe mitral stenosis will cause a left atrial pressure of about 25 mm Hg. This left atrial pressure is transmitted to the pulmonary vasculature resulting in pulmonary hypertension. As left atrial pressure remains elevated, the left atrium will increase in size. As the left atrium increases in size, there is a greater chance of developing atrial fibrillation. If atrial fibrillation develops, the atrial kick is lost. Thus, in severe mitral stenosis, the left ventricular filling is dependent on the atrial kick. With loss of the atrial kick, there is a decrease in cardiac output and sudden development of congestive heart failure. Go to the next page if you knew the correct answer, or click the link image(s) below to further research the concepts in this question (if desired).

Research Concepts: Mitral Stenosis

We update eBooks quarterly and Apps daily based on user feedback. Please tap flag to report any questions that need improvement.

Question 794: A 38-year-old woman with a past medical history of obesity and 10 packyear smoking is brought to the hospital with symptoms of hallucinations, disorganized behavior, and catatonia. Her husband mentions that the patient complained of headache and fever 10 days prior to admission. Physical examination is notable for new-onset tachycardia and fluctuating blood pressure. Anti-NMDAR IgG antibodies are detected by indirect immunofluorescence assay in the patient's serum. Which of the following is the next best step in the evaluation of this patient?

Choices: 1. Transvaginal ultrasound 2. Chest x-ray 3. Blood culture 4. Urine toxicology

Answer: 1 - Transvaginal ultrasound Explanations: This scenario describes a case of Anti-NMDA receptor encephalitis. The disease starts with a prodromal state mimicking common viral infections. But within weeks to a few months (less than 3 months), complex neuropsychiatric features emerge rapidly during the psychotic phase. The psychotic phase evolves into the unresponsive phase characterized by mutism, decreased motor activity, and catatonia. Following the unresponsive phase, a hyperkinetic phase with autonomic instability and prominent movement disorders becomes evident. Anti-NMDAR IgG antibodies, detected by indirect immunofluorescence assay in the serum and the CSF, are diagnostic for the disease. After confirmation of the diagnosis, a comprehensive evaluation should be done to detect underlying malignancy. Transvaginal ultrasonography is the most crucial test for young women presenting with the illness due to the high incidence of underlying ovarian teratoma. Go to the next page if you knew the correct answer, or click the link image(s) below to further research the concepts in this question (if desired).

Research Concepts: Anti-NMDA Receptor Encephalitis

We update eBooks quarterly and Apps daily based on user feedback. Please tap flag to report any questions that need improvement.

Question 795: A 30-year-old male athlete presents to the clinic with a history of syncope and palpitations. He also reports a history of the loss of his father at a younger age. On cardiac examination, he has a double pulse, and on auscultation, a systolic ejection murmur is heard, which increases on Valsalva maneuver. The electrocardiogram (ECG) is suggestive of left ventricular hypertrophy. What is the gold standard test to confirm the diagnosis in this patient?

Choices: 1. 2D echocardiography 2. Cardiac magnetic resonance imaging (MRI) 3. Cardiac catheterization 4. Radionuclide imaging

Answer: 2 - Cardiac magnetic resonance imaging (MRI) Explanations: Hypertrophic obstructive cardiomyopathy (HOCM) is an autosomal dominant condition characterized by obstruction of the left ventricular outflow tract and asymmetric septal hypertrophy. It is typically seen in young athletes who die of sudden cardiac death. The evaluation includes ECG, 2D-echocardiography, cardiac MRI, radionuclide imaging, and cardiac catheterization. Cardiac MRI is established as a gold standard test in patients with HOCM. It not only helps in anatomical evaluation and risk stratification but also diagnoses complications of HOCM, such as aneurysm and fibrosis. Though initial evaluation is always with 2D-echocardiography (which will diagnose 80% of patients with HOCM), it still can miss HOCM in 20% of the patients. 2D-echocardiography assesses ventricular function, septal hypotrophy, cardiac dimensions and helps in ruling out other valvular heart diseases. Cardiac catheterization and nuclear scanning may not be required in most cases. In selective cases, it may be used to assess outflow tract obstruction, the severity of mitral valve regurgitation, and the patency of the coronary arteries. Go to the next page if you knew the correct answer, or click the link image(s) below to further research the concepts in this question (if desired).

Research Concepts: Pulsus Bisferiens

We update eBooks quarterly and Apps daily based on user feedback. Please tap flag to report any questions that need improvement.

Question 796: A 65-year-old woman with a history of hypertension is brought to the clinic for ongoing memory issues. The patient is accompanied by her husband, who reports that the patient has had trouble with short-term memory for the past year. She is able to carry out her activities of daily living but is now requiring assistance to walk and has to wear diapers because she had a few episodes of incontinence. Upon examination, her Mini-Mental State Examination score is 24/30, significant for 0/3 recall in 5 minutes. The neurological exam is significant for a wide-based gait with short steps. A CT head without contrast shows dilated ventricles. Which of the following is the next best step in the management of this patient?

Choices: 1. High volume lumbar puncture 2. Urinalysis 3. Thyroid function tests 4. MRI brain without contrast

Answer: 1 - High volume lumbar puncture Explanations: The patient's history and physical likely suggest normal pressure hydrocephalus. The next step should be lumbar puncture with gait assessment before and after. Another more effective method of evaluation is external lumbar drainage. A larger amount of CSF was removed, and brain volume change was greater than with lumbar puncture. Cognitive decline, magnetic gait, and urinary incontinence form the triad of normalpressure hydrocephalus. CT head shows dilated ventricles. MRI brain will not help with the diagnosis or the management. Frequent incontinence and use of diapers predispose to urinary tract infections. However, the patient does not complain of any symptoms suggestive of infection or fluctuating course of confusion. Go to the next page if you knew the correct answer, or click the link image(s) below to further research the concepts in this question (if desired).

Research Concepts: Differentiating Delirium Versus Dementia in the Elderly

We update eBooks quarterly and Apps daily based on user feedback. Please tap flag to report any questions that need improvement.

Question 797: A 17-year-old college student is brought by his family to the emergency department with complaints of fever, headache, and confusion. He has recently returned from China, where he worked on a pig farm over the summer. He did not receive any particular vaccination before the trip. Family remembers he was troubled by mosquito bites. Computed tomography (CT) scan of the head shows bilateral thalamic hemorrhages. Cerebrospinal fluid studies show increased lymphocytes, elevated protein, normal glucose, and raised opening pressure. What is the most appropriate treatment for this condition?

Choices: 1. Doxycycline 2. Supportive care with intravenous fluids and antipyretics 3. Acyclovir 4. Penicillin

Answer: 2 - Supportive care with intravenous fluids and antipyretics Explanations: Japanese encephalitis is the most common form of preventable mosquito-borne encephalitis. It is common in countries in South-East Asia and the Western Pacific. Transmission is highest in rural areas, especially near pig farms or rice paddies with standing water during the summer and rainy seasons. There is an effective vaccine for prevention that is recommended for travelers to endemic areas. However, there is no effective treatment beyond supportive care with IV fluids and antipyretics. Bilateral thalamic edema, lesions, or hemorrhage are common in Japanese encephalitis, and their presence in an appropriate clinical context should suspect this disease. Go to the next page if you knew the correct answer, or click the link image(s) below to further research the concepts in this question (if desired).

Research Concepts: Japanese Encephalitis

We update eBooks quarterly and Apps daily based on user feedback. Please tap flag to report any questions that need improvement.

Question 798: A 90-year-old patient woman is transferred to the hospital from her nursing home for evaluation of a declining neurological status. CT scan showed a collection of blood in her left lateral rectus muscle. Which of the following conditions is most likely to be associated with this finding?

Choices: 1. Factor V Leiden mutation 2. Atrial fibrillation 3. Alzheimer disease 4. Fanconi anemia

Answer: 2 - Atrial fibrillation Explanations: Spontaneous muscular hematomas have an increased incidence in patients who are on anticoagulation therapy. Patients with atrial fibrillation are commonly treated with anticoagulation therapy. Spontaneous muscular hematomas almost exclusively occur in elderly patients on anticoagulation therapy. Factor V Leiden mutations lead to hypercoagulability, and would not increase the incidence of spontaneous muscular hematomas. Go to the next page if you knew the correct answer, or click the link image(s) below to further research the concepts in this question (if desired).

Research Concepts: Muscular Hematoma

We update eBooks quarterly and Apps daily based on user feedback. Please tap flag to report any questions that need improvement.

Question 799: A 70-year-old man noted left side weakness when she woke up at 7:30 am, and he presented to the emergency department at 10:00 am; he was completely fine when he went to sleep at 11:00 pm the night before. On exam, he has left visual field cut, left hemineglect, left facial palsy, and left side weakness that involves the upper limb more than the lower limb. CT head shows right dense MCA sign and no evidence of hemorrhage, CTA head and neck shows proximal (M1) right middle cerebral artery occlusion, and CT head perfusion study shows a substantial region of salvageable hypo-perfused tissue, blood pressure is 135/85 mmHg, which of the following treatments is indicated?

Choices: 1. IV labetalol 2. Mechanical thrombectomy 3. IV heparin 4. IV recombinant tissue plasminogen activator (rtPA)

Answer: 2 - Mechanical thrombectomy Explanations: Embolic strokes may result in large vessel occlusion and cause severe strokes. Such patients may benefit from mechanical thrombectomy, which can be done within the first 6 hours as demonstrated in five different randomized controlled clinical trials including MR CLEAN, ESCAPE, REVASCAT, SWIFT PRIME, and EXTEND-1A, and up to 24 hours from symptoms onset in selected patients to save the viable brain tissues at risk as demonstrated in the DAWN and DEFUSE 3 trials such as this patient with a substantial region of salvageable hypo-perfused tissue at risk. If endovascular therapy is contemplated, a noninvasive intracranial vascular study such as CTA head and neck is strongly recommended but should not delay intravenous r-tPA for patients who should be administered first. However, this patient presented more than 4.5 hours from symptoms onset and, therefore, not a candidate for IV r-tPA. Data shows a systolic blood pressure target of less than 140 mm Hg was safe and associated with better outcomes than the guideline target of 180 mm Hg. This patient systolic BP is less than 140 mm Hg, and there is no need for IV labetalol. IV heparin is not indicated for acute stroke management before mechanical thrombectomy. Go to the next page if you knew the correct answer, or click the link image(s) below to further research the concepts in this question (if desired).

Research Concepts: Embolic Stroke

We update eBooks quarterly and Apps daily based on user feedback. Please tap flag to report any questions that need improvement.

Question 800: A 16-year-old patient with status epilepticus following severe head injury is being managed with paralyzing agents and kept on a ventilator. During clinical examination, the treating provider notices synchronous rhythmic constriction and dilatation of both his pupils. What is the next step in the management of the patient?

Choices: 1. Ocular ultrasound 2. CT head 3. MRI scan 4. Bedside EEG

Answer: 4 - Bedside EEG Explanations: The presence of synchronous rhythmic constriction and dilatation of the pupils is termed as pupillary hippus. The pupillary hippus signifies the presence of refractory nonconvulsive status epilepticus and is a reliable bedside neurological sign in monitoring epilepsy in paralyzed patients. When paralyzing agents are used in treating a patient with refractory status epilepticus, EEG monitoring should be performed to assess the effectiveness of anticonvulsant therapy and to rule out subclinical seizures. CT, MRI scans, and ocular ultrasound only help in monitoring any changes in the intracranial pressure and identifying probable causes for epilepsy. Go to the next page if you knew the correct answer, or click the link image(s) below to further research the concepts in this question (if desired).

Research Concepts: Status Epilepticus

We update eBooks quarterly and Apps daily based on user feedback. Please tap flag to report any questions that need improvement.

Section 9 Question 801: A 70-year-old African American male with a past medical history of hypertension and diabetes presents to the emergency department complaining of severe abdominal pain. The patient reports the pain started 2 hours ago and has been progressively worsening. He describes the pain as throbbing, 9/10 in severity, and located in the left-sided flank region. He denies any associated burning urination, nausea, vomiting, or any history of urinary tract infection or renal stones. The patient has a 35 pack-year smoking history and drinks three to four 20 ounce cans of beer daily but denies illicit drug use. On physical exam, the patient appears to be diaphoretic, pale, and in severe distress due to pain. The abdominal exam is significant for tenderness and ecchymosis. Vital signs show a blood pressure of 80/60 mmHg, heart rate of 140 beats per minute, and SpO2 95% on room air. EKG shows sinus tachycardia. Wide bore catheter access is obtained, and fluid resuscitation is started immediately. Basic labs, including complete blood counts and basic metabolic panel, are sent. What is the next best step in the management of this patient?

Choices: 1. CT angiography of thorax and abdomen 2. Departmental abdominal ultrasound 3. Exploratory laparotomy 4. Focused bedside abdominal ultrasound

Answer: 4 - Focused bedside abdominal ultrasound Explanations: The likely diagnosis in this patient is an abdominal aortic aneurysmal rupture, which is a surgical emergency. The next best step of management for this patient is to perform a bedside ultrasound of the abdomen. Bedside ultrasound should be done quickly to save precious time in this patient. Once the diagnosis is confirmed, the patient should be immediately transferred to the operating room for an emergent repair. The patient has never been diagnosed with an abdominal aortic aneurysm in the past, so it is important to perform a quick bedside ultrasound to confirm the diagnosis before transferring the patient to the operating room. CT angiography is not indicated at this time since it takes longer to perform and would delay surgery. Departmental abdominal ultrasound requires the patient to fast, and it is time-consuming to transport the patient to the designated ultrasound area. Go to the next page if you knew the correct answer, or click the link image(s) below to further research the concepts in this question (if desired).

Research Concepts: Abdominal Aortic Aneurysm Rupture

We update eBooks quarterly and Apps daily based on user feedback. Please tap flag to report any questions that need improvement.

Question 802: A 65-year-old male comes to the emergency department with the complaint of sudden onset of weakness in his right arm and leg. He noticed these symptoms one hour ago. Currently, he is unable to walk, and he has slurred speech. On examination, his blood pressure is 100/60 mmHg and pulse 98/min regular. The neurologist is consulted, and he recommends an imaging study that shows compressed basal cisterns and midline shift. Which of the following is the initial emergent treatment of his condition?

Choices: 1. Withdrawing cerebrospinal fluid via lumbar puncture 2. Mannitol 3. Acetazolamide 4. Phenobarbital coma

Answer: 2 - Mannitol Explanations: When intracranial hypertension is suspected, immediate intervention is necessary. Mannitol is commonly used as a hyperosmolar agent and is usually given as a bolus of 0.25 to 1 g/kg body weight. Serum osmolality should be kept less than 320 mOsm to avoid side effects of therapy like renal failure, hypokalemia, and hypoosmolarity. Mannitol works immediately, and a urinary catheter should be inserted to monitor urine output. In the meantime, a neurosurgery consult should be obtained to determine the benefits of a ventriculostomy. If the patient is on a ventilator, hyperventilation can be performed. The low PCO2 can also cause vasoconstriction of the vessels in the brain. In addition to positioning the patient upright with a straight neck, initiate hyperosmolar therapy. Go to the next page if you knew the correct answer, or click the link image(s) below to further research the concepts in this question (if desired).

Research Concepts: Intracranial Hypertension

We update eBooks quarterly and Apps daily based on user feedback. Please tap flag to report any questions that need improvement.

Question 803: A 16-year-old boy is brought to the emergency department after a motor vehicle collision. He loses a lot of blood and requires a red blood cell (RBC) transfusion. After the initiation of transfusion, he develops facial swelling and has difficulty breathing. On examination, he has audible expiratory wheezing, laryngeal edema, and his blood pressure is 80/35 mmHg. He is emergently treated with epinephrine. On further questioning, he reports a past medical history of recurrent otitis media, sinusitis, and lung infections. On exam, he is found to have atopic dermatitis. Patients with this condition can be safely transfused with which RBCs?

Choices: 1. Washed RBCs 2. IgM deficient donor RBC 3. Frozen RBCs 4. Leukoreduced RBCs

Answer: 1 - Washed RBCs Explanations: IgA deficient individuals can develop an anaphylactic reaction to blood products even with the first transfusion. The suitable products for patients with selective IgA deficiency are those prepared from other IgA-deficient individuals or saline-washed red blood cells or Deglycerolized, previously frozen RBCs. Patients with IgA deficiency who develop anaphylaxis during infusion or transfusion of blood products will require laboratory testing to determine the presence of anti-IgA antibodies. Deglycerolization is equivalent to washing. Go to the next page if you knew the correct answer, or click the link image(s) below to further research the concepts in this question (if desired).

Research Concepts: IgA Deficiency

We update eBooks quarterly and Apps daily based on user feedback. Please tap flag to report any questions that need improvement.

Question 804: A 65-year-old man is admitted to the intensive care unit with septic shock. Two liters of fluid has been infused, but blood pressure is still low. He requires central venous access for vasopressor administration. He has a history of diabetes mellitus, hypertension, and chronic renal failure. He is being dialyzed thrice per week through a right forearm arteriovenous fistula. Which of the following locations is most appropriate for central line placement in this individual?

Choices: 1. Left internal jugular vein 2. Right subclavian vein 3. Left femoral vein 4. Right internal jugular vein

Answer: 4 - Right internal jugular vein Explanations: The internal jugular vein is a common site for central line placement. The right internal jugular vein has the straightest course to the right atrium. It is relatively easy to place a central line in the right IJV with a high success rate with or without ultrasound. It is an easily compressible site in the case of bleeding or accidental arterial puncture and it has relatively low pneumothorax risk as compared to the subclavian vein. Due to ease of placement and relatively fewer complications, right IJV is often the site chosen by anesthesiologists for central line placement. The left internal jugular has a smaller caliber than the right internal jugular vein. It has a tortuous path to the right atrium. Because of the tortuous path, left internal jugular vein canulation is associated with increased malposition. The incidence of arterial puncture is higher with the left internal jugular vein as it is smaller in size and overlays the internal carotid artery more often than the right internal jugular vein. The presence of AV fistula in the right arm makes subclavian vein undesirable for central line placement as it may cause interference with the outflow from the arm and potentially may compromise the patient’s dialysis access. The femoral veins carry a higher risk of thrombosis and infection. Go to the next page if you knew the correct answer, or click the link image(s) below to further research the concepts in this question (if desired).

Research Concepts: Internal Jugular Vein Central Venous Access

We update eBooks quarterly and Apps daily based on user feedback. Please tap flag to report any questions that need improvement.

Question 805: A 65-year-old male patient who just moved to the area and is setting up a primary care facility presents to the hospital with complaints of low-grade fever, cough, and shortness of breath. The patient's previous medical record shows that he was diagnosed with HIV more than five years ago; however, the patient has been only intermittently compliant with the treatment. The examination of the chest reveals bilateral crepitations. The patient is evaluated with a chest x-ray, which shows bilateral lower lobe pneumonia. What is the minimum CD4 count required to start him on co-trimoxazole?

Choices: 1. 20 2. 40 3. 200 4. 500

Answer: 3 - 200 Explanations: Pneumocystis Carinii Pneumonia (PCP), now referred to as Pneumocystis Jirovecii Pneumonia is a fungal infection that most commonly affects the immunocompromised and, in some cases, can be severely life-threatening. Typically, patients at risk are those with any underlying disease states that alter host immunity such as those with cancer, HIV, transplant recipients, or those taking immunosuppressive therapies and medications. Individuals with HIV infection whose CD4 count falls below 200, and who are not on Pneumocystis jiroveci (PCP) prophylaxis, are at increased risk for opportunistic infections. Other patients prone to PCP include those with an immunosuppressive disorder like severe combined immunodeficiency syndrome. Individuals who are malnourished are also susceptible to PCP. Finally, patients receiving long term immunosuppressive therapy following a transplant are also prone to PCP. Go to the next page if you knew the correct answer, or click the link image(s) below to further research the concepts in this question (if desired).

Research Concepts: Pneumocystis Jirovecii Pneumonia

We update eBooks quarterly and Apps daily based on user feedback. Please tap flag to report any questions that need improvement.

Question 806: A 66-year-old woman presents to the emergency department with an onset of dark-colored urine that began last night. Her past medical history is positive for hypertension, hyperlipidemia, and osteoarthritis in both knees. Her chart indicates a prescription was given one week ago for cephalexin following staph-induced cellulitis in her left axilla. What protein is most likely to be found in the highest in concentration in her urine?

Choices: 1. Hemoglobin 2. Albumin 3. Myoglobin 4. Insulin

Answer: 1 - Hemoglobin Explanations: A potential side effect of cephalexin is hemolytic anemia, which results in the lysis of red blood cells. Red blood cells contain high concentrations of hemoglobin. This can present as dark-colored urine. Cephalexin can also cause nausea and interstitial nephritis. Go to the next page if you knew the correct answer, or click the link image(s) below to further research the concepts in this question (if desired).

Research Concepts: Cephalexin

We update eBooks quarterly and Apps daily based on user feedback. Please tap flag to report any questions that need improvement.

Question 807: A 17-year old patient has been brought to the emergency department. She is minimally responsive and when the monitor is placed she has a very wide QRS with sinus tachycardia. What class of medications has she likely ingested?

Choices: 1. Opioids 2. Tricyclic antidepressants 3. Analgesics 4. Beta blockers

Answer: 2 - Tricyclic antidepressants Explanations: Tricyclic antidepressants cause prolonged QRS complexes on ECG and this is an early sign of toxicity. Tricyclic antidepressants have a narrow therapeutic window, increasing the likelihood of toxicity. Sinus tachycardia is a very common feature of TCA toxicity and a wide QRS supports the diagnosis. The serum levels of tricyclic antidepressants do not correlate with the symptoms and one must use the ECG and other blood work to support the diagnosis. Go to the next page if you knew the correct answer, or click the link image(s) below to further research the concepts in this question (if desired).

Research Concepts: Tricyclic Antidepressant Toxicity

We update eBooks quarterly and Apps daily based on user feedback. Please tap flag to report any questions that need improvement.

Question 808: A 29-year-old parturient patient at 38 weeks of gestation is undergoing epidural catheter placement for labor and delivery pain management. Loss of resistance is noticed, and the catheter is placed. A combination of lidocaine 2% and epinephrine 1:200,000 is pushed through the catheter. The patient develops hypotension, respiratory depression, aphonia, and inability to move arms and legs. What is the cause of the patient's current condition?

Choices: 1. Valvular heart disease 2. Familial dysautonomia 3. Drug toxicity 4. Medication wrong-route administration

Answer: 4 - Medication wrong-route administration Explanations: Heart valves abnormalities and heart function are very important considerations in pregnant women undergoing neuraxial anesthesia due to the risk of sympathetic depression caused by the anesthetics and associated complications. However, the patient's presentation is more consistent with a total spinal (acute aphonia and high anesthetic level are not be associated with a heart valve dysfunction). Familial dysautonomia symptoms include the inability to produce tears, frequent lung infections, difficulty maintaining the right temperature, bed-wetting, and poor balance. The initial symptoms of local anesthetics toxicity include tinnitus, dizziness, circumoral numbness, tongue paresthesias, and blurred vision. The foramen magnum is the superior boundary of the epidural space. However, the spinal space extends superiorly reaching the cranial nerves. Total spinal occurs when local anesthetics reach the cranial subarachnoid space potentially manifesting symptoms such as fixed dilated pupils, aphonia, and loss of consciousness. In this patient, the local anesthetic was meant to be administered in the epidural space but was injected in the intrathecal space instead. Go to the next page if you knew the correct answer, or click the link image(s) below to further research the concepts in this question (if desired).

Research Concepts: Epidural

We update eBooks quarterly and Apps daily based on user feedback. Please tap flag to report any questions that need improvement.

Question 809: A 59-year-old woman presents to the emergency department with a rapidly worsening vision and an acute excruciating, debilitating headache, described as the worst in her life. She denies having any focal neurologic deficits. Shortly after arrival, her blood pressure is measured to be 190/90 mmHg, and she develops a generalized tonic-clonic seizure for one minute. She loses consciousness and does not follow commands thereafter. Although no further seizures are witnessed, she begins to deteriorate quickly, and her Glasgow Coma Scale (GCS) drops to 7. She is intubated, and the intensivist is consulted for admission into the intensive care unit. Which of the following is the next best step in the management of this patient?

Choices: 1. Control blood pressure emergently 2. Obtain an electroencephalogram (EEG) and start an antiepileptic drug 3. CT of the brain without IV contrast 4. MRI of the brain with and without IV contrast

Answer: 3 - CT of the brain without IV contrast Explanations: This individual’s presentation is concerning for a subarachnoid hemorrhage, possibly due to an aneurysmal rupture. It is crucial to differentiate an emergent diagnosis, such as subarachnoid hemorrhage, from posterior reversible encephalopathy syndrome (PRES), especially since both diseases can present clinically in a similar way: visual disturbance, seizures, acutely altered mentation, and headaches. A CT of the head without IV contrast is obtained to help assess for intracranial hemorrhage, including subarachnoid hemorrhage, and this is done as soon as the patient is stabilized. A CT of the head without IV contrast can help to distinguish intracranial hemorrhage from hypodense cerebral edema. In PRES, cerebral edema is typically located in the parietal and occipital lobes, although other regions can be involved. Sometimes, a CT of the head cannot detect cerebral edema, and an MRI of the brain is recommended to observe the edema. The MRI, however, can take a long time to obtain. In the clinical scenario above, assessing for acute intracranial hemorrhage with a CT of the head is the next critical step. This question also helps to illustrate the importance of relying on taking a good clinical history. A subarachnoid hemorrhage can be distinguished clinically from PRES, as the former is acute in origin, sudden, and associated with an immediately peaking excruciating worst headache of life called thunderclap headache. Go to the next page if you knew the correct answer, or click the link image(s) below to further research the concepts in this question (if desired).

Research Concepts: Posterior Reversible Encephalopathy Syndrome

We update eBooks quarterly and Apps daily based on user feedback. Please tap flag to report any questions that need improvement.

Question 810: A 65-year-old male patient presents to the emergency department complaining of muscle spasms and diarrhea. On examination, he has a dry mouth, tachycardia, hypertension, hyperreflexia, diaphoresis, and dilated pupils. His friend says he takes “some medicine for something”. What is likely to be in his medical history and medicine cabinet?

Choices: 1. Gout/Colchicine 2. Chronic Diarrhea/Lomotil (Diphenoxylate and Atropine) 3. Depression/amitriptyline 4. Schizophrenia/chlorpromazine

Answer: 4 - Schizophrenia/chlorpromazine Explanations: Colchicine (plant toxin/secondary metabolite) usually causes GI symptoms and CNS changes like confusion, delirium, and hallucinations. Lomotil has biphasic toxicity (anticholinergic and then opioid-like) and this presentation may be consistent with the first phase. Hyperreflexia is not a usual feature of this overdose. Tricyclic overdoses often cause tachycardia but hypotension is often present. Also, CNS depression is common, not excitation. Thorazine and other antidepressants can have extrapyramidal effects (a dopamine antagonist) and in overdose cause a serotonin syndrome differentiated from other conditions (like Neuroleptic Malignant Syndrome) by hyperreflexia. Basic treatment involves abstinence from or removal of the drug (GI decontamination), benzodiazepines, and supportive care. Go to the next page if you knew the correct answer, or click the link image(s) below to further research the concepts in this question (if desired).

Research Concepts: Deadly Single Dose Agents

We update eBooks quarterly and Apps daily based on user feedback. Please tap flag to report any questions that need improvement.

Question 811: A 65-year-old woman with hypertension, hyperlipidemia, and toxic multinodular goiter undergoes total thyroidectomy. Morning bloodwork the day after surgery reveals a calcium level of 6.2 mg/dL, phosphorus 2.0 mg/dL, albumin 4.1 mg/dL and PTH 72.0 pg/mL. An EKG is obtained, and her QTc is 480 ms. Which of the following is the best initial therapy for this patient?

Choices: 1. Calcium carbonate 1000 mg by mouth daily starting right away 2. Calcium citrate 4000 mg by mouth once a day starting right away 3. Calcium chloride 1g IV STAT 4. Calcium gluconate 2g IV STAT followed by continuous infusion

Answer: 4 - Calcium gluconate 2g IV STAT followed by continuous infusion Explanations: Symptomatic or severe hypocalcemia, i.e. calcium level less than 7.6 mg/dL, requires IV calcium treatment. Calcium gluconate is less toxic if it extravasates into surrounding tissues. Calcium chloride has higher elemental calcium content but requires central venous access for administration. Oral calcium should be initiated as soon as the patient can tolerate diet. Go to the next page if you knew the correct answer, or click the link image(s) below to further research the concepts in this question (if desired).

Research Concepts: Hungry Bone Syndrome

We update eBooks quarterly and Apps daily based on user feedback. Please tap flag to report any questions that need improvement.

Question 812: A 55-year-old man is brought in by emergency medical services (EMS) after being involved in a motor vehicle accident. On examination, the patient responds inconsistently and specifically to external stimuli, and he withdraws to painful stimuli. Which of the following would also be expected in this category of the rancho los amigos scale (RLAS)?

Choices: 1. Responds more to his wife 2. Responds more to the medical staff 3. Ability to follow simple commands 4. Tremor, athetosis, and involuntary movements

Answer: 1 - Responds more to his wife Explanations: A patient with a rancho los amigos score of 3 demonstrates only localized responses to external stimuli inconsistently. Voluntary motor testing will be unavailable for the therapist. The patients will respond more to familiar people (friends and family) versus strangers. When assessing motor control in these patients, only reflexes, rigidity, and tone can be assessed. Withdrawal secondary to pain would be a sensory reflex evaluation. The patient is unlikely to be able to follow simple commands. Go to the next page if you knew the correct answer, or click the link image(s) below to further research the concepts in this question (if desired).

Research Concepts: Ranchos Los Amigos

We update eBooks quarterly and Apps daily based on user feedback. Please tap flag to report any questions that need improvement.

Question 813: A medical director of the adult intensive care unit (ICU) started using angiotensin II one year back in the ICU. In the last one-year angiotensin II was administered to 50 patients. It was observed to work best in patients with a subset of acute respiratory distress syndrome (ARDS) and septic shock. The director earmarks a fellow to study the potential relationship between the finding. What is the reason for the improved efficacy of this medication in ARDS and septic shock patients?

Choices: 1. Angiotensin II works best in patients with a high mortality rate 2. Reason for improved efficacy is unknown 3. Angiotensin II improves lung endothelium function 4. Procalcitonin is high in patients with ARDS and septic shock

Answer: 3 - Angiotensin II improves lung endothelium function Explanations: Angiotensin I is converted into angiotensin II by the angiotensin-converting enzyme, which is produced by the lung endothelium. In acute respiratory distress syndrome (ARDS), there is an injury of the endothelium leading to a decrease in the production of this enzyme, resulting in potentially angiotensin II deficiency state. In patients with ARDS and septic shock, angiotensin II seems to work better because of this potential hypothesis of angiotensin II deficiency state. Any other condition which impacts the lungs and causes vasoplegic shock would potentially benefit from angiotensin II based on a similar mechanism. Patients with ARDS and septic shock are certainly a very sick group of patients, and they may have high procalcitonin. However, that is not the reason for the efficacy of this medication in that group if patient. Go to the next page if you knew the correct answer, or click the link image(s) below to further research the concepts in this question (if desired).

Research Concepts: Angiotensin II

We update eBooks quarterly and Apps daily based on user feedback. Please tap flag to report any questions that need improvement.

Question 814: A 43-year-old man with a past medical history of end-stage renal disease is brought to the emergency department for altered mental status. The patient’s initial vital signs show heart rate 39/min, blood pressure 94/51 mmHg, respiratory rate 19/min, and oxygen saturation 83% on room air. The patient withdraws his right arm when pinched, opens his eyes spontaneously, and is speaking in clear sentences but is oriented only to self. The patient displays an apneic pattern of breathing. An immediate decision is made to secure the patient’s airway. As assessment of the patient’s airway shows an overbite of the jaw, a short neck, and morbid obesity. While the patient is being pre-oxygenated, a point of care glucose shows a level of 94 mg/dL, and an electrocardiogram reveals peaked T-waves with widened QRS-complexes. The patient is pre-oxygenated and achieves an oxygen saturation of 95%. After rapid sequence intubation via direct laryngoscopy, the patient continues to desaturate and is now 79% despite being mechanically ventilated. What is the most likely cause of this patient’s desaturation?

Choices: 1. Hyperkalemia 2. Pulmonary embolism 3. Esophageal intubation 4. Myocardial Infarction

Answer: 3 - Esophageal intubation Explanations: The patient who has a history of end-stage renal disease and presents to the emergency department with altered mental status can have any number of reasons as to why he is altered. However, once intubated, the most common complication of tracheal intubation of difficult airways is esophageal intubation (choice 3). Esophageal intubation can cause the desaturation of a patient despite mechanical ventilation. The patient achieved adequate oxygenation prior to intubation via a bag-valve-mask, and rapid desaturation after intubation is likely to be esophageal intubation. In morbidly obese patients, auscultation is ineffective in determining breath sounds after mechanical ventilation, and it is hard to see abdominal distention. Therefore it is safe to assume the patient’s esophagus is intubated. Esophageal intubation is common in patients with difficult airways. Intubations should be confirmed with end-tidal carbon dioxide levels or color change monitors to ensure successful cannulation of the trachea. In patients with pulmonary embolism (choice 2), an increase in oxygen saturation would not be expected; in addition, patients might be tachycardic and might complain of chest pain. Patients with myocardial infarction (choice 4) can also cause a patient to desaturate. However, the electrocardiogram shows evidence of hyperkalemia (choice 1), which will cause bradycardia and cardiac arrest but does not typically cause rapid desaturation after intubation. Go to the next page if you knew the correct answer, or click the link image(s) below to further research the concepts in this question (if desired).

Research Concepts: Difficult Airway

We update eBooks quarterly and Apps daily based on user feedback. Please tap flag to report any questions that need improvement.

Question 815: A 30-year-old primigravid female presents to the emergency department at 36 weeks of gestation with headache, vomiting, and right upper quadrant pain for the last three hours. Her medical history is significant for gestational hypertension, which was being controlled with labetalol. She has no other medical condition and has no known allergies. She is up-to-date with her prenatal care and was tested GBS positive. Her body mass index is 28 kg/m2, and she has gained about 16 kg during the pregnancy. Her temperature is 99 F, her blood pressure is 170/95 mmHg, and her pulse is 77/min. Physical examination a confused pregnant female with right upper quadrant tenderness and reduced bowel sounds. Fetal heart rate tracing shows a baseline of 150/min with moderate variability, no decelerations, and no accelerations. There is moderate bilateral pedal edema. Complete blood count reveals a hemoglobin of 9 g/dl, platelet count of 90,000/m3, and leukocyte count of 9000/m3. Liver function tests are deranged. Coagulation studies are unremarkable. Proteinuria is also noted on urinalysis. Which of the following correctly explains her presentation?

Choices: 1. Distended hepatic capsule 2. Obstruction of bile flow 3. Acute fatty liver 4. Liver capsule inflammation and adhesion

Answer: 1 - Distended hepatic capsule Explanations: The patient has presented with preeclampsia with severe features. Preeclampsia is characterized by the triad of hypertension, proteinuria, and edema. Severe features include right upper quadrant pain, elevated liver enzymes, thrombocytopenia, clonus, hyperactive reflexes, etc. Right upper quadrant pain is consistent with HELLP syndrome, one of the severe features of preeclampsia. Further laboratory testing shows hemolytic anemia, thrombocytopenia, and elevated liver enzymes. Systemic inflammation and activation of the coagulation cascade, which leads to microangiopathic hemolytic anemia, subsequently causes hepatocellular necrosis and thrombi in portal circulation. This causes liver swelling, distension of liver capsule, and right upper quadrant pain. Acute fatty liver of pregnancy is also a complication of the third trimester or early postpartum period. However, this setting is more consistent with HELLP syndrome. Option four is consistent with pelvic inflammatory disease resulting in the right upper quadrant due to Fitz-Hugh-Curtis syndrome. Go to the next page if you knew the correct answer, or click the link image(s) below to further research the concepts in this question (if desired).

Research Concepts: Hypertension In Pregnancy

We update eBooks quarterly and Apps daily based on user feedback. Please tap flag to report any questions that need improvement.

Question 816: A 37-year-old man with no past medical history is admitted to the trauma intensive care unit following a car accident with polytrauma and traumatic brain injury. After resuscitation and appropriate continued management, he has absent pupillary and corneal reflexes, with intact gag and cough reflexes. The legal surrogate is told that he cannot be declared brain dead but has little chance of meaningful neurological recovery. The surrogate expresses that the patient would have wanted to 'die peacefully' and not be attached to artificial life support but asks if organ donation is contradictory to comfort care?

Choices: 1. Contact the organ donation team for immediate organ harvesting 2. Inform the surrogate that organ donation is possible (donation after circulatory death) where discontinuation of life support and comfort/end-of-life care can be provided in the operating room prior to organ procurement 3. Continue life support indefinitely 4. Tell the surrogate that organ donation is not possible

Answer: 2 - Inform the surrogate that organ donation is possible (donation after circulatory death) where discontinuation of life support and comfort/end-of-life care can be provided in the operating room prior to organ procurement

Explanations: 'Termination of life support' involves several steps and can be conducted prior to organ procurement in the appropriate clinical scenario. Communication with legal surrogate must be detailed to explain the process The patient's wishes should be respected, and attempt made to follow them as closely as possible in the given clinical condition Contacting the organ transplant team is not unethical and is part of the Donation after Circulatory Death (DCD) process. Comfort measures are an essential component of that process Go to the next page if you knew the correct answer, or click the link image(s) below to further research the concepts in this question (if desired).

Research Concepts: Termination of Life Support

We update eBooks quarterly and Apps daily based on user feedback. Please tap flag to report any questions that need improvement.

Question 817: A 70-year-old man is brought to the emergency department after being found in an altered mental status, requiring endotracheal intubation in the field. Post-intubation assessment reveals that the pulse-oximetry remains at 60%, and the capnography waveform is flat, and the value is 0. What is the next best step in the airway management of this patient?

Choices: 1. Cricothyrotomy 2. Remove the endotracheal tube and repeat endotracheal intubation 3. King tube placement 4. Laryngeal mask airway

Answer: 2 - Remove the endotracheal tube and repeat endotracheal intubation Explanations: A flat capnography waveform with a value of zero and the patient remaining hypoxic would indicate that the endotracheal tube is not in the trachea and is likely in the esophagus. The tube would need to be removed, and repeat intubation would need to be performed to obtain proper placement of the endotracheal tube. Appropriate placement of the tube would be noted with a good (nonflat) capnography waveform and a range value of 35-45 mmHg. The gold standard for assessing the placement of an ET is direct visualization with the help of a laryngoscope. Additional ways to assure proper confirmation of endotracheal tube placement include carbon dioxide, capnography waveform, chest x-ray, ultrasound, and clinical assessment. The AHA (American Heart Association) recommends continuous waveform capnography besides clinical assessment as the most reliable method of confirming and monitoring correct placement of an ET tube. Bedside mobile ultrasound is another resource that some emergency departments have to confirm the position of the ET tube. Many physicians frequently use a chest x-ray to assess the placement of the ET tube. Clinically, abnormal tube placement can be diagnosed with absent breath sounds on the left chest, if right mainstem intubation occurred, and no bilateral breath sounds bilaterally if an esophageal intubation occurred. Additionally, with esophageal intubation air may be auscultated in the mid-epigastric region upon ventilation administration. Lastly, low oxygen saturation will be noted. Go to the next page if you knew the correct answer, or click the link image(s) below to further research the concepts in this question (if desired).

Research Concepts: Airway Management

We update eBooks quarterly and Apps daily based on user feedback. Please tap flag to report any questions that need improvement.

Question 818: A 72-year-old Caucasian male with a history of quadriparesis, posttraumatic stress disorder (PTSD), diabetes, insomnia, and a chronic indwelling Foley catheter presents for follow up with his primary care clinician after being discharged from the hospital with sepsis secondary to pyelonephritis. He was told that he had an acute kidney injury while hospitalized. His baseline glomerular filtration rate (GFR) is 46ml/minute. He takes baclofen for muscle spasticity, prazosin for his PTSD, metformin for his diabetes, and diphenhydramine as needed to help him sleep. Today, in the office, his GFR is 26ml/minute and his liver enzymes are normal. He is concerned that he may need to stop medications given his decreased kidney function. What is the preferred next management?

Choices: 1. Continue prazosin and diphenhydramine, then discontinue metformin 2. Continue prazosin, discontinue metformin and diphenhydramine 3. Continue metformin and diphenhydramine, then decrease the dose of prazosin 4. Continue diphenhydramine, baclofen, metformin, and prazosin at their respective current dose

Answer: 1 - Continue prazosin and diphenhydramine, then discontinue metformin Explanations: Prazosin and diphenhydramine are hepatically metabolized and require no adjustment of the dose based on his GFR. Metformin is contraindicated with a GFR less than 30ml/min and should be avoided with a GFR between 31-45ml/min. Prazosin and diphenhydramine require no adjustment of dose based on GFR. Metformin is contraindicated with a GFR less than 30ml/min. Prazosin and Diphenhydramine require no adjustment of dose based on GFR. Metformin is contraindicated with a GFR less than 30ml/min. All medications should not be continued as metformin is currently contraindicated. Baclofen has a risk of toxicity with renal impairment and dose should be decreased or discontinued. Some clinicians recommend discontinuation of baclofen with a GFR less than 30ml/min. Prazosin and diphenhydramine may be continued at their current dose. Go to the next page if you knew the correct answer, or click the link image(s) below to further research the concepts in this question (if desired).

Research Concepts: Prazosin

We update eBooks quarterly and Apps daily based on user feedback. Please tap flag to report any questions that need improvement.

Question 819: A 48-years-old male with Marfan syndrome presented to the emergency department after a motor vehicle collision. He was wearing a seat belt, and he decelerated rapidly to save a pedestrian and swerved into a ditch. He complained of severe chest pain. Physical examination reveals several bruises on the chest. His respiratory examination is unremarkable. Which is most likely on a chest x-ray?

Choices: 1. Multiple right-sided rib fractures 2. A left pulmonary contusion 3. Hemomediastinum 4. Widening of the mediastinum

Answer: 4 - Widening of the mediastinum Explanations: Traumatic aortic injuries are usually deceleration injuries because of a huge differential force to the fixed and mobile parts of the thoracic aorta. The most common sites of injury are the following: aortic isthmus; just distal to the origin of the left subclavian artery as this is the transition zone between the mobile ascending aorta and the relatively fixed descending aorta; tethered site of the aorta at the ligamentum arteriosum, and the ascending aorta, just proximal to the origin of the brachiocephalic vessels. Patients with Marfan disease are more prone to these injuries. A chest x-ray suggestive of thoracic aortic rupture may show loss of aortic knob, widening of the mediastinum, deviation of nasogastric tube in the esophagus, an apical cap, or sternal fracture. If there is a high clinical suspicion, then further diagnostic workup should be pursued, which includes aortography, CT angiography, and transesophageal echocardiography. A contrast-enhanced helical CT scan is the screening diagnostic investigation of choice for aortic trauma. Go to the next page if you knew the correct answer, or click the link image(s) below to further research the concepts in this question (if desired).

Research Concepts: Aortic Trauma

We update eBooks quarterly and Apps daily based on user feedback. Please tap flag to report any questions that need improvement.

Question 820: A 43-year-old male is brought in to the emergency department with concerns of epigastric pain with radiation to the back. He has a long history of daily alcohol consumption and recently loss his job because of this. Labs are remarkable for a lipase level of 3,545 U/L. The patient is admitted to the regular medicine floor and started on aggressive fluid resuscitation, pain control with morphine and he is kept NPO. On the second day of hospitalization, the patient is found lethargic and hypotensive. Vital signs show blood pressure of 74/36 mmHg, which does not improve with 2L bolus of intravenous normal saline, hear rate of 114 bpm, respiratory rate of 14 breaths per minute and temperature of 97.6 F. What is the next best step in management for this patient?

Choices: 1. Another bolus 2 L of intravenous normal saline 2. Initiate norepinephrine infusion 3. Hydrocortisone 4. Vancomycin and meropenem

Answer: 2 - Initiate norepinephrine infusion Explanations: Hypovolemic shock is a serious complication of hypovolemia. It refers to persistent low arterial blood pressure after appropriate fluid resuscitation. The etiology of hypovolemia is of utmost importance to properly treat and manage this complication. In the acute setting, the etiology may not be readily apparent. In this case, fluid resuscitation and vasopressor support may be required to prevent further organ damage. In this patient, pancreatitis appears to be the most likely etiology of hypovolemia and shock. Go to the next page if you knew the correct answer, or click the link image(s) below to further research the concepts in this question (if desired).

Research Concepts: Hypovolemia

We update eBooks quarterly and Apps daily based on user feedback. Please tap flag to report any questions that need improvement.

Question 821: A 35-year-old man comes into the hospital after being stranded on a top of a mountain during the winter. The man was found by a search team after going missing for five days. The patient's past medical history is unremarkable. On physical exam, the patient is hypothermic and found to have frostbite on all his fingers and toes. The fingers and toes appear dry, shrunken, and gangrenous. The decision is made to remove the frostbite fingers, but after the patient is stabilized. The patient is admitted to the hospital for observation before surgery. On day 4 in the hospital, the fingers and toes are found to be swollen with purulent discharge. Upon palpation of the fingers and toes, crepitus is found. The provider suspects an infection of the fingers and toes. What is the most likely microbe causing the infection?

Choices: 1. Gram-negative anaerobic bacteria 2. Gram-positive aerobic bacteria 3. Gram-positive anaerobic bacteria 4. Gram-negative aerobic bacteria

Answer: 3 - Gram-positive anaerobic bacteria Explanations: When extremities become necrotic and gangrenous, they may become infected with Clostridium perfringens. Frostbite is a form of dry gangrene. When gangrene becomes infected it becomes wet gangrene. Wet gangrene that has crepitus indicates a gas-producing organism. C. Perfringens classically cause superinfections with crepitus. This microbe is a gram-positive sporeforming anaerobic bacteria. It classically metabolizes carbohydrates into gas creating crepitus along with infection. Dry gangrene is a form of necrosis due to ischemia. Due to the prolonged cold temperatures, the blood supply to the extremities is constricted. The body tries and redirect the blood supply to the most vital organs. This protective mechanism can lead to organ failure and limb loss. Go to the next page if you knew the correct answer, or click the link image(s) below to further research the concepts in this question (if desired).

Research Concepts: Hypothermia

We update eBooks quarterly and Apps daily based on user feedback. Please tap flag to report any questions that need improvement.

Question 822: A 16-year-old male patient presents to the hospital with complaints of general malaise, weight loss, night sweats, and arthralgias for more than three weeks. He had a holiday abroad where he got a tattoo, which later became itchy and red. On examination, he has a high-grade fever, the heart rate is 52/min, and there are petechial hemorrhages in his conjunctiva and oral mucosa. On auscultation, there are no cardiac murmurs, and the chest is clear. Urine dipstick shows hematuria. His white cell count is 16000/microliter, and C-reactive protein is 140 mg/dl. An electrocardiogram shows that p waves are unrelated to QRS complexes. What is the most likely diagnosis?

Choices: 1. Rheumatic fever 2. Infective endocarditis 3. Viral hemorrhagic fever 4. Pyelonephritis

Answer: 2 - Infective endocarditis Explanations: This patient appears to be suffering from infective endocarditis, which should be considered first in patients with chronic fever, weight loss, malaise, and absence of murmurs. The history of skin infection makes the staphylococcus infection the more likely cause. The presence of a third-degree cardiac block shows the involvement of conducting tissue. During a physical examination, attention should be paid to any signs of infection or skin rashes, such as rheumatic fever, Lyme disease, and endocarditis, which cause heart blocks. Conjunctival petechial hemorrhages are much more common than the classical features of splinter hemorrhages, Osler nodes, and Janeway lesions. Go to the next page if you knew the correct answer, or click the link image(s) below to further research the concepts in this question (if desired).

Research Concepts: Third-Degree Atrioventricular Block

We update eBooks quarterly and Apps daily based on user feedback. Please tap flag to report any questions that need improvement.

Question 823: A 72-year-old woman with a past medical history significant for myasthenia gravis, hypertension, hyperlipidemia, and acid reflux presents to the hospital with complaints of generalized weakness and difficulty breathing. Her home medications include physostigmine, lisinopril, atorvastatin, and omeprazole. She is diagnosed with myasthenia flare and treated with plasmapheresis and albumin as the replacement fluid. After initiating plasmapheresis, she develops flushing, bradycardia, and hypotension. Which of the following is most likely to have triggered this event?

Choices: 1. Inadequate volume replacement 2. Hyperviscosity syndrome 3. Vasovagal reflex 4. Lisinopril

Answer: 4 - Lisinopril Explanations: The use of ACE inhibitors has been associated with atypical reactions during plasmapheresis. Anaphylactic or atypical reactions including flushing, hypotension, dyspnea, and bradycardia have been reported. The reactions were worse when albumin was used as replacement fluid, and the patient had ACE exposure in the last 24 hours. A possible mechanism is increased kinin production. It can be minimized by avoiding the use of an ACE inhibitor or by using a different replacement fluid. Go to the next page if you knew the correct answer, or click the link image(s) below to further research the concepts in this question (if desired).

Research Concepts: Angiotensin Converting Enzyme Inhibitors (ACEI)

We update eBooks quarterly and Apps daily based on user feedback. Please tap flag to report any questions that need improvement.

Question 824: A 65-year-old man with a past medical history of hyperlipidemia, migraines, and osteoarthritis presents to the hospital with new-onset atrial fibrillation. His heart rate generally runs low between 40/min and 50/min. In an attempt to cardiovert the patient, a transesophageal echocardiogram is performed without signs of thrombus, and electric cardioversion is attempted with 300 Joules. The patient's atrial fibrillation is persistent, and he fails electric cardioversion. The patient is advised about the long term effects of atrial fibrillation and is interested in attempting antiarrhythmic therapy. Treatment with dofetilide is being considered. Which of the following complications is most likely to occur in this patient with treatment?

Choices: 1. Ventricular arrhythmia 2. Worsening bradycardia 3. Tachycardia 4. Right axis deviation

Answer: 1 - Ventricular arrhythmia Explanations: Dofetilide may induce or worsen ventricular dysrhythmias, producing life-threatening polymorphic ventricular tachycardia. Patients with a history of torsades de pointes, prolonged QTc (greater than 440 ms), hypomagnesemia, or serum potassium below 4.0 mEq/L are at increased risk of developing ventricular dysrhythmias when placed on dofetilide. Patients are generally hospitalized when starting dofetilide to be able to actively monitor for dosing related ventricular arrhythmias. Dofetilide and sotalol are higher risk for torsades de pointes than amiodarone within the class III antiarrhythmic medication. Go to the next page if you knew the correct answer, or click the link image(s) below to further research the concepts in this question (if desired).

Research Concepts: Dofetilide

We update eBooks quarterly and Apps daily based on user feedback. Please tap flag to report any questions that need improvement.

Question 825: A 49-year-old African American male with a past medical history of hypertension and tobacco use presents to the emergency department complaining of left-sided tearing chest pain radiating to his back. The patient states that he ran out of his hypertensive medication approximately two weeks ago. The physical exam is pertinent for a patient in severe distress with a grade IV/VI diastolic decrescendo murmur heard best at the left 3rd intercostal space, which was not present at the previous outpatient visit one month ago. Vital signs are notable for a heart rate of 106/min, blood pressure 205/115 mmHg in the left arm, and 199/105 mmHg in the right arm. A bedside transthoracic echocardiogram is performed, showing a left ventricular ejection fraction of 55-60% and valvular heart diasease. Once hemodynamically stabilized, what is the definitive treatment plan for this patient?

Choices: 1. Medical management of blood pressure with close observation for the next 24-48 hours 2. Surgical repair of ascending aortic dissection with transcatheter aortic valve replacement 3. Surgical repair of ascending aortic dissection with surgical aortic valve replacement 4. Surgical repair of ascending aortic dissection with aortic valve repair via replacement of damaged aortic sinus

Answer: 4 - Surgical repair of ascending aortic dissection with aortic valve repair via replacement of damaged aortic sinus

Explanations: The patient presents with classic signs and symptoms of aortic dissection, noncompliant with antihypertensives, and daily tobacco use place the patient at high risk for dissection. Descending (type B) aortic dissections can be observed with medical management of blood pressure if the patient remains hemodynamically stable, ascending (type A) aortic dissections require emergent surgical repair. The patient has evidence of moderate to severe aortic regurgitation on transthoracic echo, often the treatment modality of choice would be valve replacement. Since this patient is at low risk it could be argued open surgical repair would be the treatment of choice. The patient does not have a previous echocardiogram to compare with, however, AR murmur was not present one month ago, and there is evidence of dissection extending into the L ventricle. It can be assumed the patient has a structurally normal aortic valve and the regurgitation is secondary to the dissection. This is one of the few instances where repair of disrupted sinuses is the treatment of choice rather than valve repair. Go to the next page if you knew the correct answer, or click the link image(s) below to further research the concepts in this question (if desired).

Research Concepts: Aortic Valve Disease

We update eBooks quarterly and Apps daily based on user feedback. Please tap flag to report any questions that need improvement.

Question 826: A 32-year-old man with a history of asthma, heavy alcohol use is being treated for Rickettsia conorii infection in the hospital. On the 3rd day of treatment, his clinical course begins to worsen, and he develops new-onset atrial fibrillation, worsening liver function studies, thrombocytopenia, and spreading purpuric rash over his body. He dies on the 5th day after a progressively worsening clinical course. Which of the following patient factors most likely worsened the patient's clinical course?

Choices: 1. Age 2. Sex 3. History of asthma 4. History of alcohol use

Answer: 2 - Sex Explanations: Alcohol use disorder is known to cause worsening progression of Boutonneuse fever. More severe manifestations of the disease can occur in select patient populations. These include advanced age, immunocompromise, chronic alcoholism, glucose-6-phosphate dehydrogenase [G6PD] deficiency, prior prescription of an inappropriate antibiotic, and delay of treatment. The other listed factors for this patient do not affect the clinical course. Go to the next page if you knew the correct answer, or click the link image(s) below to further research the concepts in this question (if desired).

Research Concepts: Boutonneuse Fever

We update eBooks quarterly and Apps daily based on user feedback. Please tap flag to report any questions that need improvement.

Question 827: A 16-year-old boy is intubated for adult respiratory distress syndrome (ARDS) secondary to pancreatitis. He has a right radial arterial line placed. The patient is currently on peak inspiratory pressure (PIP) 30 cm H2O, positive end-expiratory pressure (PEEP) 10, and FiO2 80%. He has a SpO2 of 93% on the ventilator. Which of the following best describes the primary indication for keeping an arterial line for this patient?

Choices: 1. Measurement of cerebral perfusion pressure 2. Separate access for infusion of medications which are not compatible together 3. Accurate measurement of oxygenation index 4. As an access point for cardiac catheterization

Answer: 3 - Accurate measurement of oxygenation index Explanations: In this patient who has hypoxemic respiratory failure, the arterial line would allow for the assessment of the oxygenation index (OI). This is a measurement of the severity of lung disease and is calculated by mean airway pressure/ partial pressure of oxygen in arterial blood x FiO2. The partial pressure of arterial blood requires an indwelling arterial catheter, especially because the trend in the OI is more important than a single value. Oxygenation index of more than 8 implies acute respiratory distress syndrome (ARDS), which moderate and more than 16 implies severe ARDS. This patient with pancreatitis would require closer monitoring of blood pressure and blood gases and not of cerebral perfusion pressure. Medications should never be administered in the arterial line. The question does not give any description of cardiac dysfunction, and hence cardiac catheterization is not needed. Go to the next page if you knew the correct answer, or click the link image(s) below to further research the concepts in this question (if desired).

Research Concepts: Arterial Lines

We update eBooks quarterly and Apps daily based on user feedback. Please tap flag to report any questions that need improvement.

Question 828: An 86-year-old man is brought to the emergency department after a fall reported by EMS. No history could be obtained from the patient. There is a visible contusion on his forehead. His gaze appears to deviate to the left on the exam. What is the best initial step in the management of this patient?

Choices: 1. Levetiracetam 2. Haloperidol 3. Phenobarbital 4. Aspirin

Answer: 1 - Levetiracetam Explanations: The deviated gaze with the history of the fall indicates that the patient likely had a traumatic head injury with a seizure and is possibly in status epilepticus. Levetiracetam is typically the anti-epileptic of choice in a traumatic brain injury in the elderly. Levetiracetam has a lower side effect profile when compared to other anti-seizure medications in the elderly. The patient is likely having a seizure and needs treatment for the seizure, such as levetiracetam. An antipsychotic such as haloperidol is not indicated at this time. Aspirin is initially contraindicated as there is a known traumatic head injury and giving aspirin could make an intracranial bleed worse. Go to the next page if you knew the correct answer, or click the link image(s) below to further research the concepts in this question (if desired).

Research Concepts: Geriatric Head Injury

We update eBooks quarterly and Apps daily based on user feedback. Please tap flag to report any questions that need improvement.

Question 829: A 17-year-old male patient is being managed in an intensive care unit following a severe head injury. His bodyweight is 55 kilograms. The patient has now developed persistent hyponatremia with serum sodium below 120 mEq/l. His urine output is also lower than 25ml per hour. The patient also has gained weight in the past few days, as reported by the attending clinician. What is the next step in the fluid management of the patient for managing his hyponatremia?

Choices: 1. Administration of 500 ml of 0.9% NaCl 2. Administration of 500 ml of 3% NaCl 3. Fluid restriction and frequent re-evaluation 4. Administration oral fludrocortisone

Answer: 3 - Fluid restriction and frequent re-evaluation Explanations: The patient has clinical signs and symptoms suggestive of a syndrome of inappropriate antidiuretic hormone (SIADH) secretion. The patient has hyponatremia, with decreased urine output, and features suggestive of water retention reflected from his weight gain. This is all compatible with the diagnosis of SIADH. The pivotal step in the management of SIADH is the fluid restriction in the patient. Urine output in a normal adult patient is more than 0.5 ml/kg/hour. Hypertonic saline is indicated in the management of cerebral salt wasting (CSW) wherein the patient will have hyponatremia associated with characteristic polyuria. Go to the next page if you knew the correct answer, or click the link image(s) below to further research the concepts in this question (if desired).

Research Concepts: Fluid Management

We update eBooks quarterly and Apps daily based on user feedback. Please tap flag to report any questions that need improvement.

Question 830: A 50-year-old man is admitted to the ICU for ventilatory support after sustaining a 45% total body surface area (TBSA) burn to his right side. On the 7th day of his admission, he develops a temperature of 102.1°F (38.9°C) and has a blood pressure of 110/80 mmHg; however, no signs of end-organ dysfunction are present. Laboratory findings reveal a white blood cell (WBC) count of 10,900 cells/mm3 and lactate levels of 1.1 mmol/L. He has a Foley catheter in place. Urinalysis reveals 4 WBC/HPF, traces of RBC, a few bacteria, and positive nitrites, but no leukocyte esterase, glucose, protein, ketones, or casts. Which of the following is the next best step in the management of this patient?

Choices: 1. Start vancomycin and piperacillin/tazobactam for suspected sepsis 2. Start ceftriaxone for catheter-associated urinary tract infection (CAUTI) 3. Initiate sulfamethoxazole/trimethoprim for CAUTI 4. Change the Foley and take a new sample for urinalysis

Answer: 4 - Change the Foley and take a new sample for urinalysis Explanations: Approximately 75% of urinary tract infections (UTIs) acquired in the hospital are linked to the use of a urinary catheter. The Infectious Disease Society of America (IDSA) definition of catheter-associated urinary tract infection (CAUTI) requires all of the following components: 1,000 colony-forming units per ml or more of one bacterial species in the culture grown, signs and symptoms of a UTI (e.g., fever, costovertebral angle tenderness, and hypotension), despite proper evaluation, there is no other reason for these symptoms, the patient has a Foley's catheter in place or catheter removed within less than 48 hours. This patient has a Foley in position and is not showing any signs of systemic infection. A new urine sample should be taken after the Foley catheter is replaced. Prolonged use of the urinary catheter is by far the most significant risk factor for having a catheter-associated UTI (CAUTI). As a result, catheters should only be used when absolutely necessary and must be removed as soon as possible. Go to the next page if you knew the correct answer, or click the link image(s) below to further research the concepts in this question (if desired).

Research Concepts: Urinary Tract Infection

We update eBooks quarterly and Apps daily based on user feedback. Please tap flag to report any questions that need improvement.

Question 831: A 65-year-old male patient is brought by his caretaker with the chief complaints of headache, nausea, vomiting, blurring of vision, abdominal pain, and drowsiness. He has been having headaches since the winter started, but the frequency and intensity have gotten worse. He lives in his house alone since his wife passed away last year. On examination, his blood pressure is 110/70 mmHg, pulse 91 beats per minute and regular, respiratory rate 22 breaths per minute, and his Glasgow coma scale (GCS) is 11/15. Fundoscopy reveals bilateral disc edema. During his time in the emergency department, his GCS declines further and oxygen saturation starts to drop. He is immediately intubated and put on mechanical ventilation. Which of the following should be the target for mechanical ventilation in this case?

Choices: 1. Set PaCO2 between 45 to 50 mmHg 2. Set PaCO2 between 35 to 40 mmHg 3. Set PaCO2 between 30 to 35 mmHg 4. Set PaCO2 less than 30 mmHg

Answer: 3 - Set PaCO2 between 30 to 35 mmHg Explanations: This is a case of raised intracranial pressure secondary to carbon monoxide poisoning. Hypocapnia can reduce cerebral blood flow by 4% for each mmHg change in PaCO2. Experts recommend a PaCO2 between 30 to 35. This decreased PaCO2 can reduce intracranial pressure by vasoconstriction of blood vessels. Hyperventilation can decrease cerebral oxygenation and may induce brain ischemia in patients with traumatic brain injury and acute stroke. The effects of hyperventilation last about 48-72 hours. Once hyperventilation is used, the PaCO2 levels should return to normal gradually. The rebound vasodilatation can be detrimental. Below a PaCO2 of 30, cerebral blood flow reduction is minor. Low PaCO2 can induce hypoxemia in brain tissues and so is not recommended. While some advocate normocarbia, a short period of hypocarbia does work and is beneficial. Go to the next page if you knew the correct answer, or click the link image(s) below to further research the concepts in this question (if desired).

Research Concepts: Intracranial Hypertension

We update eBooks quarterly and Apps daily based on user feedback. Please tap flag to report any questions that need improvement.

Question 832: A 65-year-old woman with a past medical history of heart transplant due to dilated cardiomyopathy presents with progressive dyspnea on exertion, orthopnea, and weight loss. The patient underwent the transplant 8 months ago. For the past two months, her symptoms have worsened gradually. Physical examination reveals pedal edema, bilateral crepitations at the lung bases, and the presence of a new heart murmur. Which of the following will most likely be present in this patient on further evaluation?

Choices: 1. Bradycardia 2. Asystole 3. Atrial flutter 4. Ventricular tachycardia

Answer: 3 - Atrial flutter Explanations: This patient most likely has a heart transplant rejection. A sign of cardiac allograft rejection can be an atrial arrhythmia. Due to vagal denervation post heart transplant, allograft recipients tend to have elevated heart rate at baseline. Besides atrial arrhythmias, sudden death also has been reported following a heart transplant. These patients usually have no chest pain because the donor's heart is not innervated by the recipient's nerves. During an acute rejection, the patient may be completely asymptomatic or maybe in florid heart failure. Go to the next page if you knew the correct answer, or click the link image(s) below to further research the concepts in this question (if desired).

Research Concepts: Heart Transplantation Rejection

We update eBooks quarterly and Apps daily based on user feedback. Please tap flag to report any questions that need improvement.

Question 833: A 50-year-old man with a history of diabetes, hypertension, and recurrent alcoholic pancreatitis presents to the emergency department due to worsening shortness of breath and abdominal discomfort for the past week. Vital signs include a temperature of 98.6 F, blood pressure 145/90 mmHg, heart rate 115/min, respiratory rate 18/min, and oxygen saturation 92% on 3 L/min oxygen by nasal cannula. Physical examination shows a dry oral mucosa, no murmurs on the cardiac exam, absent breath sounds at the left hemithorax, and mild generalized abdominal tenderness. A chest x-ray shows a large left pleural effusion occupying more than 90% of the left hemithorax. Which of the following is the next best step in the management of this patient?

Choices: 1. Paracentesis 2. Thoracentesis 3. Magnetic resonance cholangiopancreatography 4. Echocardiogram

Answer: 2 - Thoracentesis Explanations: Thoracentesis is a procedure to evacuate pleural fluids such as that seen in pancreatic pleural effusion and allows for further analysis of the fluid content. A pleural fluid sample is tested for amylase, protein, albumin, glucose, lactate dehydrogenase, cytology, gram stain, culture, cell count and differential, and adenosine deaminase to help determine the etiology of the fluid accumulation. Patients with pancreatic pleural effusion can present with pulmonary symptoms such as dyspnea, cough, wheezing, or pleuritic chest pain. Paracentesis is a procedure to evacuate fluids from the peritoneal cavity and allows for further analysis of the fluid content. Go to the next page if you knew the correct answer, or click the link image(s) below to further research the concepts in this question (if desired).

Research Concepts: Pancreatic Fistula

We update eBooks quarterly and Apps daily based on user feedback. Please tap flag to report any questions that need improvement.

Question 834: A 65-year-old male patient is brought to the emergency department with complaints of sudden onset of headache and inability to speak for three hours. On his way to the hospital, he has vomited twice. His previous record reveals that he is suffering from diabetes and hypertension. He has a metallic valve in place because he got mitral valve replacement done 6 years ago and since then he is on anticoagulants. Recently he consulted his family physician for a fungal infection over his skin for which he received a course of antifungals. On examination, blood pressure 180/110 mmHg, pulse 92/min regular, maintaining oxygen saturation on room air, and respiratory rate of 21/min. Nuchal rigidity is positive. Fundoscopy reveals bilateral papilledema. What needs to be administered to counteract the underlying pathology?

Choices: 1. Tenecteplase 2. Low molecular weight heparin 3. Vitamin K 4. Protamine sulfate

Answer: 3 - Vitamin K Explanations: This is a case of intracerebral hemorrhage leading to raised intracranial pressure. The patient is on warfarin, and he has been given fluconazole which is an inhibitor of cytochrome P450 C29, an enzyme that is responsible for the metabolism of warfarin. This drug interaction leads to warfarin over-anticoagulation. In the cases of raised intracranial pressure, the eyes should always be examined because the pupils may become dilated and unresponsive to light. The patient may also develop the Cushing reflex with elevations in blood pressure and bradycardia. Go to the next page if you knew the correct answer, or click the link image(s) below to further research the concepts in this question (if desired).

Research Concepts: Intracranial Hypertension

We update eBooks quarterly and Apps daily based on user feedback. Please tap flag to report any questions that need improvement.

Question 835: A 65-year-old male patient with a history of emphysema presents to the hospital complaining of sudden onset dyspnea and pleuritic type chest pain on the left side. He is aa smoker of 25/day. On examination, he looks very restless, pale and sweaty. His blood pressure is 140/85 mmHg, the pulse is 120/min regular, the respiratory rate is 32/min, and oxygen saturation on room air is 88%. He has reduced chest expansion, hyper resonant note, and decreased air entry on the left side. He is commenced on oxygen supplementation and analgesia. What is the next best step in management?

Choices: 1. Urgent surgical intervention 2. Urgent CT scan of the chest 3. Chest drain insertion 4. Chest aspiration

Answer: 3 - Chest drain insertion Explanations: There are two types of pneumothorax: traumatic and atraumatic. The two subtypes of atraumatic pneumothorax are primary and secondary. A primary spontaneous pneumothorax (PSP) occurs automatically without a known eliciting event, while a secondary spontaneous pneumothorax (SSP) occurs subsequent to an underlying pulmonary disease. On examination, the following findings are noted respiratory discomfort, increased respiratory rate, asymmetrical lung expansion, decreased tactile fremitus, hyper resonant percussion note, decreased intensity of breath sounds or absent breath sounds. In secondary spontaneous pneumothorax, if size/depth of pneumothorax is less than 1 cm and no dyspnea then the patient is admitted, high flow oxygen is given and observation is done for 24 hours. If size/depth is between 1-2 cm, needle aspiration is done, then the residual size of pneumothorax is seen, if the depth after the needle aspiration is less than 1 cm management is done with oxygen inhalation and observation and in case of more than 2cm, tube thoracostomy is done. In case of depth more than 2 cm or breathlessness, tube thoracostomy is done. Go to the next page if you knew the correct answer, or click the link image(s) below to further research the concepts in this question (if desired).

Research Concepts: Pneumothorax

We update eBooks quarterly and Apps daily based on user feedback. Please tap flag to report any questions that need improvement.

Question 836: A 74-year-old woman was brought to the emergency department due to constant chest pain accompanied by shortness of breath. She reported that the pain decreases in intensity when she leans down. The patient has a history of hypercholesterolemia and diabetes mellitus type 2. Her vital signs included temperature 38 C (100.4 F), blood pressure 110/75 mmHg, heart rate 95 beats/min, and respiratory rate 20/min. Cardiac auscultation revealed friction rub, with no murmurs or added sounds. The patient was admitted and started on pain medications. On day 2, her blood pressure is 95/64 mmHg. Her neck veins are elevated, and cardiac auscultation reveals muffled S1 and S2. What findings are most likely to be found in this patient's electrocardiogram?

Choices: 1. QRS complexes alternating in height 2. Prolonged QT interval 3. ST-segment elevation in leads V3-V6 4. Widened QRS complexes and loss of P wave

Answer: 1 - QRS complexes alternating in height Explanations: The patient was admitted with pericarditis. Pericardial effusion is a well-known complication of pericarditis. In patients with large pericardial effusion, ECG may show electrical alternans. Electrical alternans refers to consecutive QRS complexes of varying amplitudes corresponding to a swinging motion of the heart within the surrounding fluid. ECG may show diffuse ST-segment elevation in patients with pericarditis. ST-segment elevation in specific leads is a sign of localized myocardial injury. Go to the next page if you knew the correct answer, or click the link image(s) below to further research the concepts in this question (if desired).

Research Concepts: Pericardial Effusion

We update eBooks quarterly and Apps daily based on user feedback. Please tap flag to report any questions that need improvement.

Question 837: A 58-year-old male patient is brought to the hospital with altered mental status after being found on the floor of his bathroom by his adult daughter. He lives alone, and the last time she spoke to him was two days ago. He does not have any medical conditions that she knows of and does not take any medications. She reports he drinks alcohol regularly. He was well during their last conversation. On exam, his temperature is 38.5 C (101.3 F), the heart rate is 110/min regular, the blood pressure is 180/90 mmHg, the respiratory rate is 16, and O2 saturation is 98%, and he is agitated and anxious. EKG shows sinus tachycardia. Complete blood count, complete metabolic panel, and urinalysis are all within normal limits. CT scan of the head, brain, chest, abdomen, and pelvis is without pathology. What is the next most appropriate step in the management of this patient?

Choices: 1. Administer IV crystalloid fluids at 20 mL/kg and start broad antibiotic coverage 2. Obtain TSH and administer beta-blockers after oral rehydration 3. Administer benzodiazepines for symptom management and seizure prophylaxis 4. Administer phenoxybenzamine for noncompetitive alpha-receptor antagonism

Answer: 3 - Administer benzodiazepines for symptom management and seizure prophylaxis Explanations: Intoxication and withdrawal from prescribed and recreationally consumed substances are common causes of tachycardia. Benzodiazepines are the first-line treatment for both alcohol withdrawal and for agitation management in amphetamine intoxication. Differentiating between sepsis and intoxication/withdrawal is a critical skill for clinicians. Blood pressure, mental status, lab work, and infectious source can be important clues. The early differential diagnoses for patients with sympathomimetic toxidrome are ingestion, withdrawal, infection, pulmonary embolism, and hyperthyroidism. Go to the next page if you knew the correct answer, or click the link image(s) below to further research the concepts in this question (if desired).

Research Concepts: Sinus Tachycardia

We update eBooks quarterly and Apps daily based on user feedback. Please tap flag to report any questions that need improvement.

Question 838: A 36-year-old female with no known medical history is admitted to the ICU with acute respiratory distress syndrome (ARDS) after a near-drowning event. The patient is receiving volume cycled mechanical ventilation. On rounds, you note that the alarm for high peak inspiratory pressure is going off. An inspiratory hold maneuver reveals elevated plateau pressures as well. Which of the following complications is most likely to be seen in this patient?

Choices: 1. Bronchospasm 2. Barotrauma 3. Increased venous return 4. Increased mucus

Answer: 2 - Barotrauma Explanations: Patients with ARDS are at increased risk of developing barotrauma. Barotrauma describes the manifestations of extra-alveolar air during mechanical ventilation. Patients with ARDS have decreased compliance in the lung tissue. The decreased compliance results in a significant elevation in both peak inspiratory pressures as well as plateau pressures. Plateau pressure is the pressure applied to the alveoli and other small airways during ventilation. Elevation of plateau pressures may lead to rupture of alveoli units and result in barotrauma. The clinical presentation varies from absent symptoms with the subtle radiographic findings of pulmonary interstitial emphysema to respiratory or cardiac distress due to a tension pneumothorax. Other manifestations include pneumopericardium, pneumomediastinum, pneumoperitoneum, and subcutaneous emphysema. An increase in alveolar pressure mediates lung injury associated with barotrauma. Go to the next page if you knew the correct answer, or click the link image(s) below to further research the concepts in this question (if desired).

Research Concepts: Barotrauma And Mechanical Ventilation

We update eBooks quarterly and Apps daily based on user feedback. Please tap flag to report any questions that need improvement.

Question 839: A 72-year-old man with a past medical history of alcoholic liver disease, diabetes, hypertension, chronic renal failure, and smoking presents to the hospital with abdominal pain, grossly bloody diarrhea, hematemesis, and altered mental status. The patient's history reveals an intake of wild mushrooms on the day before presentation. The patient is unresponsive to standard medical and supportive therapy for his condition. Which of the following is the strongest indication for the use of molecular adsorbent recirculating system in this patient?

Choices: 1. Alcoholic liver disease 2. Amanita phalloides intoxication 3. Chronic renal failure 4. Age > 65 years

Answer: 2 - Amanita phalloides intoxication Explanations: Amanita phalloides, also known as death cap, is a deadly poisonous mushroom. It is frequently mistaken for edible mushrooms and causes acute hepatic failure due to the amatoxins. The amanitin toxin is heat stable, remaining toxic whether eaten raw or cooked. The mechanism of action of amatoxin is by inhibiting RNA polymerase, causing disruption of transcription of mRNA. As a result, hepatocytes cannot synthesize key protein coding genes, leading to the disintegration of nucleoli and pathologically centrilobular hepatic necrosis. The course of Amanita toxicity has three phases: The first stage does not begin until six to 12 hours after ingestion; often, foragers comment on how good the food made from Amanita species tastes, and there are no signs of a problem for at least 6 hours. After this silent phase, it is followed by the onset of nausea, abdominal cramps, profuse watery diarrhea, and signs of dehydration. A physical exam may reveal dry mucosal membranes and tachycardia, and given sufficient dehydration, hypotension. After the GI phase, the second stage appears where the patient appears to recover transiently, and GI symptoms resolve, but ongoing liver damage continues. This stage may last two to three days and is characterized by rising liver function transaminase, bilirubin, the development of coagulopathy, and eventually hepatic encephalopathy. In the third stage, both liver and renal function become compromised. Hepatorenal syndrome and hepatic encephalopathy may occur rapidly after laboratory signs of liver injury, and death can occur in three to seven days. Molecular adsorbent recirculating system has been used for the treatment of liver failure from Amanita phalloides poisoning with positive results. Go to the next page if you knew the correct answer, or click the link image(s) below to further research the concepts in this question (if desired).

Research Concepts: Molecular Absorbent Recirculating System

We update eBooks quarterly and Apps daily based on user feedback. Please tap flag to report any questions that need improvement.

Question 840: A 68-year-old male patient was referred to the emergency department due to severe bilateral lower limb coldness and pallor. His physical examination revealed the absence of bilateral lower limb pulses from femoral to distal. Both limbs were severely cold. His vital signs were blood pressure: 68/40 mmHg, pulse rate: 110 beats per minute, respiratory rate: 19/min, and temperature: normal. The CT-angiography disclosed infrarenal aortic thrombosis. He underwent the standard surgical procedure. Four hours later, his vital signs were blood pressure: 80/60 mmHg, pulse rate: 120 bpm, respiratory rate: 18/min, and normal temperature. His total urine output for the last 4 hours was 30 mL. What is the best next step in management?

Choices: 1. Perform CT-angiography w/o contrast 2. Perform CT-angiography w/ contrast 3. Give a bolus of isotonic fluid 4. Start furosemide to increase urine output

Answer: 3 - Give a bolus of isotonic fluid Explanations: This patient most likely has oliguria secondary to hypovolemia. Vascular surgery is a high-risk procedure and high volume blood loss should be anticipated. Normal urine output is around 1-2 mL/kg/hour. The patient should be given adequate isotonic fluids. If volume replacement does not remedy the situation, the other studies may be of help. Obtaining CT-angiography with contrast without prior adequate hydration might result in renal shutdown and should be prevented. Administration of furosemide in lower limits of blood pressure might result in aggravating hypotension. Go to the next page if you knew the correct answer, or click the link image(s) below to further research the concepts in this question (if desired).

Research Concepts: Peripheral Vascular Bypass

We update eBooks quarterly and Apps daily based on user feedback. Please tap flag to report any questions that need improvement.

Question 841: An 88-year-old man with mild dementia and metastatic pancreatic cancer is admitted to the medical ward from his nursing home for abdominal pain and fever. Though disoriented, he is pleasant and cooperative. He is found to have a urinary tract infection and started on appropriate antibiotics and intravenous (IV) fluids. He shows clinical improvement for his presenting complaints but gradually becomes hypoxic, which is treated with supplemental oxygen. By the fourth day of hospitalization, he becomes acutely short of breath and confused. Chest x-ray and arterial blood gas are ordered. His blood gas shows an elevated carbon dioxide level. The chest x-ray shows pulmonary edema and volume overload. Unintentionally, his IV fluid orders from admission had been continued for the entire hospital course to date. His advanced directives preclude intubation and cardiopulmonary resuscitation (CPR). Which of the following is the most appropriate management strategy for this patient?

Choices: 1. Reverse his code status and intubate 2. Comfort measures 3. Obtain vascular access and initiate extracorporeal carbon dioxide removal 4. Non-invasive ventilation and administer diuretics

Answer: 4 - Non-invasive ventilation and administer diuretics Explanations: Acute hypercarbia developing after several days of hospitalization may be iatrogenic. In this case, the initial picture suggested sepsis. The patient appeared to be recovering, but then he developed respiratory failure. As the chest x-ray shows pulmonary edema, diuretics should be given to address the underlying cause. The initial treatment for hypercarbic respiratory failure should be non-invasive ventilator (NIV) support if no contraindications exist. In patients without treatment limitations, care can be escalated to intubation and mechanical ventilation if NIV fails. In a setting of volume overload, hypoxic respiratory failure usually develops before hypercarbia. If pulmonary edema worsens to the point of impaired gas exchange and respiratory distress, hypercarbia can result. Patient autonomy should be respected, even though the hypercarbia is likely reversible. Extracorporeal carbon dioxide removal is not widely available at hospitals in the United States. Go to the next page if you knew the correct answer, or click the link image(s) below to further research the concepts in this question (if desired).

Research Concepts: Hypercarbia

We update eBooks quarterly and Apps daily based on user feedback. Please tap flag to report any questions that need improvement.

Question 842: An 82-year-old female with a history of heart failure with reduced ejection fraction is admitted for evaluation of lightheadedness. The patient reports that she is feeling lightheaded and generalized weak for the last 1 week. Lightheadedness is more pronounced when the patient stands up from a sitting position. The patient feels thirsty and reports having a weight loss of 4 pounds (2 kg) over the last 1 week. No nausea, vomiting, spinning sensation of head, changes in hearing, tinnitus, or ear discharge is reported. The patient denied syncope, palpitation, chest pain, and shortness of breath. Vitals include blood pressure 110/80 mmHg, pulse 100 bpm, respiratory rate 20/min, pulse ox 93% on room air, temperature 97 °F. Orthostatic vitals are positive. The patient appears dry with no leg edema and oral mucosa is dry. The chest is clear to auscultation. S1 plus S2 plus no additional sound. No bilateral lower extremity edema. No focal motor or sensory deficit. The patient is taking aspirin, carvedilol, atorvastatin, lisinopril, furosemide. EKG shows normal sinus rhythm and no acute ischemia. Echocardiogram performed showed an ejection fraction of 37% with no new changes compared to the prior study performed within the last 6 months. Basic metabolic profiles show serum creatinine of 1.7 mg/dL and basic urea nitrogen of 40. Baseline serum creatinine is 1.1 mg/dL. Serum potassium was 2.9 milliequivalents per liter. What is the most common drug contributing to the current clinical condition?

Choices: 1. Carvedilol 2. Furosemide 3. Atorvastatin 4. Lisinopril

Answer: 2 - Furosemide Explanations: The patient is on carvedilol, and no recent changes in medications have been made. Although carvedilol like any other antihypertensive medication can lead to hypotension, but for the patient in clinical scenario no recent changes in dose of carvedilol are made so it is less likely for carvedilol to contribute to orthostasis and kidney injury. Loop diuretics such as furosemide, bumetanide, torsemide, and ethacrynic acid tend to cause hypovolemia and free water loss and can cause kidney injury due to dehydration. In addition, the elderly patient population is more predisposed to these side effects and caution should be practiced when prescribing diuretics along with close monitoring to observe for development of any adverse effect is recommended. In the current patient, furosemide is most likely the agent causing excessive diuresis leading to dehydration and acute kidney injury. Due to hypovolemia and electrolyte loss, the patient is experiencing generalized weakness and postural hypotension (orthostasis). The primary adverse effects of furosemide include hypokalemia, hyponatremia, hypocalcemia, hypomagnesemia, hyperglycemia, hyperuricemia, azotemia, and metabolic alkalosis. Monitor blood pressure, daily weight, urine output, serum electrolytes, renal function (blood urea nitrogen and serum creatinine), serum uric acid, and blood glucose levels in patients taking furosemide. Statin therapy can lead to myalgias, muscle spasms, myopathy, arthralgias, fatigue, hyperglycemia, abnormal hepatic function test. However, it is less likely to cause orthostatic hypotension and kidney injury. Lisinopril can cause dizziness, hypotension, increased serum creatinine, hyperkalemia, and syncope. In the patient under discussion, furosemide is the most likely cause of orthostatic hypotension and dehydration as evident from recent weight loss, elevated BUN, dry oral mucosa, and other signs of dehydration. Lisinopril should be kept on hold till the kidney injury is resolved, and then lisinopril should be resumed gradually at a lower dose and the dose should be titrated according to the response and tolerance of the patient. Go to the next page if you knew the correct answer, or click the link image(s) below to further research the concepts in this question (if desired).

Research Concepts: Furosemide

We update eBooks quarterly and Apps daily based on user feedback. Please tap flag to report any questions that need improvement.

Question 843: A 47-year-old man is brought to the hospital after police found him unconscious near a local bar. The patient regains consciousness shortly after arrival but appears confused and disoriented. He is unkempt and smells of alcohol. Physical examination reveals ataxic gait and difficulty moving his eyes horizontally. Horizontal nystagmus is noted. Which of the following will occur if intravenous thiamine is not administered promptly?

Choices: 1. Lactic acidosis 2. Permanent memory loss 3. Paralysis 4. Liver failure

Answer: 2 - Permanent memory loss Explanations: Wernicke encephalopathy occurs early in the disease course and characteristically demonstrates non-inflammatory brain lesions. It can present with ataxia, ophthalmoplegia, punctate hemorrhages in the brain, altered mental status, and balance abnormalities. If left untreated, Wernicke encephalopathy can eventually evolve to include Korsakoff psychosis. Individuals will now present with delirium and permanent memory loss. Treatment should commence emergently to prevent disease progression and irreversible brain damage. Empirically, WKS treatment is with at least 500 mg thiamine hydrochloride per 100 mL of normal saline given over 30 minutes. This regimen should be repeated three times a day for 2 to 3 days. Thiamine administration should be before or alongside glucose. WKS affects the central nervous system, which involves the brain and spinal cord. Its most common cause is alcohol misuse, seen alongside poor nutrition, but can present in individuals who are at risk for thiamin deficiency. Go to the next page if you knew the correct answer, or click the link image(s) below to further research the concepts in this question (if desired).

Research Concepts: Vitamin B1 (Thiamine)

We update eBooks quarterly and Apps daily based on user feedback. Please tap flag to report any questions that need improvement.

Question 844: A 60-year-old female with a history of hypertension, diabetes, chronic obstructive pulmonary disease (COPD), and end-stage renal disease on hemodialysis presents for a routine visit. She has no acute symptoms. She is concerned that one of her sisters died of a myocardial infarction a few months ago and is worried about her risk of the same thing happening to her. Her medications include metoprolol 12.5 mg twice a day insulin glargine 30 units at bedtime, and 15 units of insulin lispro with meals. Her vitals are all normal. She brings with her an echocardiogram performed a month ago after one of her dialysis sessions that reveals a region of wall motion abnormality in the left ventricle. What is the most likely explanation of her echocardiogram finding?

Choices: 1. History of diabetes 2. History of hypertension 3. History of end-stage renal disease requiring hemodialysis 4. History of COPD

Answer: 3 - History of end-stage renal disease requiring hemodialysis Explanations: Diseased or ischemic myocardium, when reperfused, leads to stunning of the myocardium, a phenomenon that is transient. Myocardium eventually regains its full contractile function. Along with reperfusion, a few other causes of stunned myocardium include stress cardiomyopathy (Takotsubo cardiomyopathy), neurogenic stunned myocardium, and LV abnormalities associated with dialysis. Patients who receive hemodialysis have an associated cardiovascular mortality and morbidity burden. They are more prone to experiencing demand myocardial ischemia and myocardial stunning. In patients with end-stage renal disease, dialysis itself can induce left ventricular abnormalities, a common feature of stunned myocardium. Go to the next page if you knew the correct answer, or click the link image(s) below to further research the concepts in this question (if desired).

Research Concepts: Stunned Myocardium

We update eBooks quarterly and Apps daily based on user feedback. Please tap flag to report any questions that need improvement.

Question 845: An 84-year-old woman presents with acute decompensated heart failure. After an inadequate response to diuresis and systolic blood pressure values persisting over 150/98 mmHg, she is started on an infusion of a vasodilator that is known to cause lactic acidosis if given for prolonged periods or at high doses. What is the other FDA-approved indication for this drug?

Choices: 1. Hypertension during acute ischemic stroke 2. Hypertensive crisis 3. Arteriovenous shunting 4. Aortic coarctation

Answer: 2 - Hypertensive crisis Explanations: Nitroprusside is a potent vasodilator of both arteries and veins that acts quickly to lower systemic vascular resistance. It must be used with extreme caution in patients with known inadequate cerebral or coronary circulation. Nitroprusside has labeled indications in the treatment of acute decompensated heart failure, management of hypertensive crises, and controlled hypotension during surgery. Off-label uses of nitroprusside include the management of hypertension in the setting of acute ischemic stroke and medical management of acute mitral regurgitation in preparation for surgical intervention. Nitroprusside is contraindicated for the treatment of compensatory hypertension (aortic coarctation or AV shunting). Go to the next page if you knew the correct answer, or click the link image(s) below to further research the concepts in this question (if desired).

Research Concepts: Sodium Nitroprusside

We update eBooks quarterly and Apps daily based on user feedback. Please tap flag to report any questions that need improvement.

Question 846: An otherwise healthy 16-year-old boy from Honduras presents to the hospital with new-onset partial seizures. In terms of medical history, he reached his developmental milestones appropriately and did well throughout the school. He now works as a mechanic after having immigrated to the United States last year. He does not drink or smoke, and he does not use any illicit or recreational drugs. His physical exam is non-focal, but brain MRI reveals numerous ring-enhancing lesions. What is the best initial therapy for this patient?

Choices: 1. Ceftriaxone 2. Acyclovir 3. Albendazole and dexamethasone 4. Phenobarbital

Answer: 3 - Albendazole and dexamethasone Explanations: The likely diagnosis in this case is neurocysticercosis, the result of accidental ingestion of eggs of Taenia solium (i.e., pork tapeworm). It is more common in Central and South America due to ingestion of uncooked egg-infected pork. Seizures are the most common presentation of this infectious disease, occurring in up to 70% of affected patients. In fact, in developing countries, it is the most common parasitic disease of the nervous system, representing the chief etiology of acquired epilepsy in endemic regions. Seizures can be generalized or partial. Other common presentations include headache, stroke, and neuropsychiatric dysfunction. Additionally, 10-30% of affected patients develop communicating hydrocephalus secondary to inflammation and fibrosis of arachnoid villi and/or the meninges. Noncommunicating hydrocephalus may also transpire in the setting of intraventricular cysts. Brain imaging often reveals ring-enhancing lesions in multiple, perhaps bilateral locations. Single lesions are also possible. Treatments comprises anti-helminthic medications (e.g., albendazole) and steroids (e.g., dexamethasone) to suppress the inflammatory response induced by destruction of the live cysticerci in the brain. In conclusion, specific regimens depend on the life cycle of the parasite. First-line anti-seizure medications like phenytoin and carbamazepine are helpful in controlling epileptic activity, and if the infection results in epilepsy in the chronic setting, anticonvulsant therapy is directed in the same way as any other epilepsy syndrome. Go to the next page if you knew the correct answer, or click the link image(s) below to further research the concepts in this question (if desired).

Research Concepts: Tapeworm

We update eBooks quarterly and Apps daily based on user feedback. Please tap flag to report any questions that need improvement.

Question 847: A 17-year-old girl presents to the emergency department with complaints of headache, body aches, fever, nausea, and vomiting for the past day. She is a known case of acute lymphoblastic leukemia who is receiving maintenance chemotherapy. She is admitted to the intensive care unit from the emergency department. Her temperature is 39.3 C, heart rate is 188 beats/min, respiratory rate is 56/min, blood pressure is 67/21 mmHg, and arterial oxygen saturation is 93% on room air. She received a total of 80 mL/kg of crystalloid fluid resuscitation in the emergency department and is now receiving another 20 mL/kg of intravenous fluid. Which of the following is the next best step in the management of this patient?

Choices: 1. Dobutamine is the vasopressor of choice at this point in the therapy 2. Intravenous antibiotic therapy should be initiated within 1 hour of presentation 3. Lactate levels are late indicator so they should be checked at this point 4. Lumbar puncture should be performed before administration of antibiotics

Answer: 2 - Intravenous antibiotic therapy should be initiated within 1 hour of presentation Explanations: Initiation of broad-spectrum antibiotics within 1 hour of presentation has been consistently associated with a reduction in mortality from sepsis syndromes. The vasopressor of choice would depend on physical examination and assessment of warm or vasodilatory shock versus cold or vasoconstrictive shock. Warm shock is characterized by flash cap refill, bounding pulses, and warm extremities. Cold shock is characterized by prolonged capillary refill, thready pulses, and cool extremities. Lactate levels are an early indicator of inadequate oxygen delivery to end organs resulting in anaerobic respiration and subsequent metabolic acidosis. If a septic patient is hemodynamically stable, it may be reasonable to wait to obtain cultures prior to the initiation of antimicrobial therapy. However, if a patient has progressed along the continuum, that is from severe sepsis versus septic shock, practitioners should not wait for diagnostic evaluations to be completed prior to administering life-saving therapeutics, such as broad-spectrum antimicrobials, immediately. Go to the next page if you knew the correct answer, or click the link image(s) below to further research the concepts in this question (if desired).

Research Concepts: Septic Shock

We update eBooks quarterly and Apps daily based on user feedback. Please tap flag to report any questions that need improvement.

Question 848: A 65-year-old male presents to the emergency department with complaints of chest pain. He has a history of diabetes, hypertension, and obesity. He developed the symptoms 30 minutes ago. His 12-lead ECG reveals the presence of ST elevation in leads 2,3 and aVF. He is planned for thrombolysis and with alteplase. The patient is attached to a cardiac monitor, and intravenous alteplase is started. The patient suddenly becomes unresponsive, and his cardiac monitor shows the presence of disorganized, high-frequency ECG tracings without a discernable p wave, QRS complex, or a T wave. CPR is instituted, and a defibrillator is attached. The defibrillator is being charged to deliver a shock. Which patient factor would require the defibrillator to be charged at a higher energy setting?

Choices: 1. Age of patient 2. History of obesity 3. ECG findings on arrival 4. Treatment with alteplase

Answer: 2 - History of obesity Explanations: This patient has presented to the emergency department with complaints of chest pain. His ECG is consistent with the development of inferior ST-elevation myocardial infarction. Unfortunately, the patient crashes during treatment and requires CPR. The rhythm strip is consistent with the development of ventricular fibrillation, which requires prompt defibrillation. The energy select on the defibrillator is 120-200 joules biphasic and 360 joules monophasic. The lowest energy settings to achieve a defibrillatory threshold should be used. The presence of obesity and a thick chest wall required a selection of higher energy settings. Individuals with thick chest walls have higher resistance and therefore, will require higher energy selection to achieve defibrillatory threshold. Shaving chest hairs and the application of conductive gel to skin surface increases conductivity. Other factors that affect the selection of energy levels during defibrillation are the duration of cardiac arrest, paddle size, previous defibrillatory shocks, and contact pressure. The aim is to achieve adequate defibrillation using the lowest energy and thus prevent myocardial injury. Go to the next page if you knew the correct answer, or click the link image(s) below to further research the concepts in this question (if desired).

Research Concepts: Ventricular Fibrillation

We update eBooks quarterly and Apps daily based on user feedback. Please tap flag to report any questions that need improvement.

Question 849: A 28-year-old adventure traveler presents to the hospital with headache, back pain, fever, and jaundice after returning from backpacking through multiple countries. Based on his presentation, yellow fever is considered. Which of the following pieces of historical data makes the diagnosis of yellow least likely?

Choices: 1. Travel to a monkey rescue unit in sub-Saharan Africa 2. Vaccination to prevent yellow fever 10-years prior to presentation 3. Nosebleeds and black emesis 4. Self-limited course with a full recovery

Answer: 2 - Vaccination to prevent yellow fever 10-years prior to presentation Explanations: Since there is no effective therapy for yellow fever, the focus should be on prevention by avoiding mosquito bites and vaccination to travelers to endemic areas. Vaccination confers lifelong immunity within 30 days in 99% of patients. Monkeys may be a reservoir for yellow fever and mosquitos serve as the vector that transmits the disease to humans from other primates. Patients with yellow fever may develop hepatic dysfunction leading to coagulopathy and bleeding from the nose and mucous membranes. Most cases of yellow fever are subclinical and have an excellent prognosis. About 15% of symptomatic patients will develop severe disease. Go to the next page if you knew the correct answer, or click the link image(s) below to further research the concepts in this question (if desired).

Research Concepts: Yellow Fever

We update eBooks quarterly and Apps daily based on user feedback. Please tap flag to report any questions that need improvement.

Question 850: A 19-year-old male military recruit is lagging behind his squadmates during a 12-mile ruck march on a hot and humid day. It is the culminating event for a day-long training exercise. He is brought to the side of the march course by his squad leader, who forces him to drink a 1 L canteen of water, his 5th canteen since starting the march. He continues on for an additional 3 miles and then collapses. He is brought to the medical tent, where he is found to be profoundly altered and combative. He is flushed, tachycardic, hypertensive, and normothermic, with an otherwise unrevealing physical exam. Initial lab evaluation reveals sodium 128 mEq/L, glucose 89 mg/dL, and otherwise normal electrolytes, LFTs, and blood counts. Which of the following is the next best step in the management of this patient?

Choices: 1. Administration of 1 L IV normal saline now 2. Administration of 100 ml IV 3% saline now 3. Active cooling and reassessment of core temperature 4. Administration of oral sodium solution of 4 broth bouillon cubes in 125 ml of water now

Answer: 2 - Administration of 100 ml IV 3% saline now Explanations: Exercise-associated hyponatremia (EAH) must be correctly differentiated from other exertional illnesses (such as heat exhaustion, heatstroke, and dehydration) prior to treatment, as their treatments are often contradictory. Treatment is dictated by the presence or absence of neurologic symptoms. The goal of treatment is the correction of serum sodium and improvement in symptoms. If a patient does not demonstrate neurologic symptoms, EAH is considered mild, and the patient is eligible for oral fluid restriction. Patients restrict their total fluid intake to induce osmotic ADH suppression and, therefore, free water excretion. Patients may also be given concentrated oral sodium repletion. However, this may be poorly tolerated due to taste or nausea and vomiting—proposed regimens include 100 mL of hypertonic saline, 3-4 broth bouillon cubes in 125 mL of water, or a serving of salted pretzels. Oral hypertonic saline is as effective as IV hypertonic saline in mild EAH, with one study demonstrating faster time to recovery and shorter duration of hospital stay for athletes who receive oral treatment. This patient is NOT eligible for oral treatment given his mental status. If a patient demonstrates neurologic symptoms (altered mental status, seizure, coma), EAH is considered severe. The patient should receive a bolus of 100 ml of IV hypertonic saline. This can be repeated every 10 minutes for a total of 3 doses. Up to 600-1000 mL over 1 hour of hypertonic saline has been given without complication in case studies, though larger volumes of sodium repletion increase the risk for rapid sodium correction and osmotic demyelination. This patient MUST be treated with IV hypertonic saline given his mental status. IV isotonic fluids of any type or volume are not recommended in EAH as this will worsen hyponatremia. Go to the next page if you knew the correct answer, or click the link image(s) below to further research the concepts in this question (if desired).

Research Concepts: Exercise-Associated Hyponatremia

We update eBooks quarterly and Apps daily based on user feedback. Please tap flag to report any questions that need improvement.

Question 851: A 42-year-old homeless woman is brought to the hospital after being found unresponsive on the side of the road. She appears disheveled and unkempt. She has no known medical records. Vital signs reveal a temperature of 39 C, blood pressure 108/68 mmHg, pulse 98/min, and respiratory rate 23/min. GCS is 12/15. Physical examination is significant for multiple vesicles in the genital area, some of which appear to be crusted. Bilateral inguinal lymphadenopathy is also seen. Lumbar puncture reveals white blood cells 250/microL (normal 8/microL) with lymphocytic predominance, protein 320 mg/dL (normal 15-45 mg/dL), and glucose 50 mg/dL (normal 50-80 mg/dL). Which of the following is the most common side effect of the best initial therapy for this patient?

Choices: 1. Elevated transaminases 2. Elevated creatinine 3. Leukopenia 4. Seizures

Answer: 2 - Elevated creatinine Explanations: The clinical vignette is most consistent with a diagnosis of herpes simplex encephalitis. While HSV-1 often affects the perioral region and can be known to cause genital lesions, HSV-2 is more commonly the consideration when patients present with genital lesions. Aseptic meningitis occurs in 36% of women and 13% of men leading to the hospitalization of a percentage of inflicted persons. During the prodrome of genital herpes and herpetic eruption, affected individuals may experience more systemic symptoms such as headaches, neck stiffness, and low-grade fever. When these symptoms are associated with altered mentation, encephalitis is most likely. Acyclovir is the most appropriate initial therapy for HSV encephalitis. One of the most common adverse effects associated with acyclovir is crystalline nephropathy, which results in transient renal impairment. This risk can be reduced with adequate hydration and a slow infusion rate. Go to the next page if you knew the correct answer, or click the link image(s) below to further research the concepts in this question (if desired).

Research Concepts: Herpes Simplex Type 2

We update eBooks quarterly and Apps daily based on user feedback. Please tap flag to report any questions that need improvement.

Question 852: A 74-year-old male with a history of coronary artery disease, hypertension, and paroxysmal atrial fibrillation presents with several days of mild chest discomfort and fatigue. On arrival, his blood pressure is 135/88 mmHg, the pulse is 87/minute, and the respiratory rate is 20/minute. On physical exam, he appears tachypneic, and mile volume overload is noted. An echocardiogram shows a reduced ejection fraction of 30% and alternating beat-to-beat left ventricular arterial waveforms. What is the most common electrocardiogram finding for patients with this condition?

Choices: 1. Atrial fibrillation 2. Sinus rhythm 3. Supraventricular tachycardia 4. Electrical alternans

Answer: 2 - Sinus rhythm Explanations: Sinus rhythm or sinus tachycardia is the most common ECG findings with pulsus alternans. Do not confuse pulsus alternans with electrical alternans. Pulsus alternans is identified as variable beat-to-beat ventricular arterial waveforms. Pulsus alternans is most commonly associated with heart failure and considered a poor prognostic indicator. Electrical alternans is the ECG finding of variable R wave voltage. It is associated with pericardial effusion. Go to the next page if you knew the correct answer, or click the link image(s) below to further research the concepts in this question (if desired).

Research Concepts: Pulsus Alternans

We update eBooks quarterly and Apps daily based on user feedback. Please tap flag to report any questions that need improvement.

Question 853: A 20-year-old male was extubated about 20 minutes ago. He had been intubated two days prior for status epilepticus requiring emergency intubation with a 7.0 mm endotracheal tube. He complains of a choking sensation in his throat, and his voice is muffled. Physical examination reveals an anxious appearing patient with a respiratory rate of 28 breaths/min. He is hemodynamically stable, and oxygen saturation is 93% on mask oxygen at 5 L/min. There is a subtle, high-pitched sound noted on inspiration. Which of the following is a risk factor for developing this condition?

Choices: 1. Intubation for more than six days 2. Male sex 3. Cuff pressure of 20 cmH2O 4. Undersized endotracheal tube

Answer: 1 - Intubation for more than six days Explanations: This 20-year-old male has post-extubation stridor, for which post-intubation laryngeal edema is a major contributor. The 2017 American Thoracic Society and American College of Chest Physicians guidelines recommend performing a bedside cuff leak test on all highrisk patients to predict the chance of developing post-extubation stridor. Known risk factors include duration of intubation greater than six days, female sex, traumatic intubation, oversized endotracheal tubes, and reintubation after unplanned extubation. The cuff leak test has varying sensitivity but an excellent specificity of more than 90% in most studies evaluating the prediction of post-extubation stridor. For this reason, it has a high negative predictive value in high-risk patients. A negative cuff leak test with a leak volume of fewer than 110 ml or less than 24% of the delivered tidal volume predicts a higher likelihood of post-extubation stridor. About half of patients with confirmed postextubation stridor require reintubation, and every reintubation event increases morbidity and mortality. Intravenous methylprednisolone 40 mg as a single dose may be given at least 4 hours prior to extubation to help prevent the development of post-intubation laryngeal edema or stridor. If the patient develops post-extubation stridor with or without the above measures (i.e., cuff leak test in high-risk patients and prophylactic methylprednisolone), the treatment is intravenous steroids for 24 to 48 hours. Some clinicians add adrenaline nebulization, though the clinical benefit in adults is yet to be proved. Close monitoring is continued, and reintubation should not be delayed beyond an hour if it is necessary. Go to the next page if you knew the correct answer, or click the link image(s) below to further research the concepts in this question (if desired).

Research Concepts: Post Intubation Laryngeal Edema

We update eBooks quarterly and Apps daily based on user feedback. Please tap flag to report any questions that need improvement.

Question 854: A 76-year-old man with chronic kidney disease presents to the emergency department. He is found to have hyperkalemia with correlating abnormalities on his EKG. The clinician recommends initiation of dialysis, but the patient refuses. His daughter, also the patient's medical power of attorney, is at the bedside and says that he has refused medical interventions in the past. The patient says that he doesn't want to die, but does not want to live on machines. He states that he knows that without dialysis, he will likely die from the potassium levels. He would rather go home to be with his family and "let nature take its natural course." Who has the decision making power in this situation?

Choices: 1. The patient 2. The medical power of attorney 3. The medical provider 4. The hospital ethicist

Answer: 1 - The patient Explanations: The patient can communicate his refusal, understands the link between dialysis and his potassium levels that he will likely die without treatment. His decision is in line with previous decisions, and it does not appear to have altered cognition from substances or delirium. While the daughter has a power of attorney (POA), the patient has the capacity to make decisions. A POA cannot make decisions in lieu of a patient with capacity. The medical provider's input is important for a patient to understand a treatment plan, but a paternalistic approach to medicine does not respect patient autonomy and could border on assault. There is no role for an ethicist in this scenario. Go to the next page if you knew the correct answer, or click the link image(s) below to further research the concepts in this question (if desired).

Research Concepts: Competency and Capacity

We update eBooks quarterly and Apps daily based on user feedback. Please tap flag to report any questions that need improvement.

Question 855: A 75-year-old woman with a past medical history of chronic kidney disease 3A, hypertension, COPD, and type 2 diabetes presents to the hospital with complaints of flu-like symptoms for 4-6 days. She reports she tested positive for SARS-CoV-2 a day back at a drivethrough testing center. At admission, the patient is noted to be febrile (100.8 °F) and hypoxic with SpO2 87% on 4 L/min nasal cannula oxygen. The initial laboratory workup at admission is unremarkable. CT scan of the chest without contrast shows bilateral ground-glass opacities. The patient is admitted to low-level monitoring floors, given remdesivir 200 mg at admission, and scheduled to receive 100 mg daily for the next nine days. The next morning, the patient’s eGFR is 27 mL/min compared to 45 mL/min at admission. Which of the following is the next best step in the management of this patient?

Choices: 1. Continue remdesivir infusion as scheduled 2. Reduce the dose of schedule remdesivir infusion from 100 mg daily to 50mg daily 3. Add dexamethasone 4. Discontinue remdesivir

Answer: 4 - Discontinue remdesivir Explanations: The patient's presentation of flu-like symptoms, acute hypoxic respiratory failure with positive SARS-CoV-2 PCR in the setting of COPD, is consistent with moderate to severe COVID-19. Given the patient's deterioration in eGFR compared to admission, remdesivir should be discontinued immediately with close monitoring of renal function. Remdesivir is contraindicated in adult patients with eGFR 30 mL/min. Considering remdesivir is an investigational drug permitted for emergency use in hospitalized patients with moderate to severe COVID-19, its safety profile has not been evaluated. Currently, there are no recommendations regarding dosages based on renal function. Go to the next page if you knew the correct answer, or click the link image(s) below to further research the concepts in this question (if desired).

Research Concepts: Remdesivir

We update eBooks quarterly and Apps daily based on user feedback. Please tap flag to report any questions that need improvement.

Question 856: A 72-year-old woman with a past medical history of heart failure with reduced ejection fraction (HFrEF) and diabetes is admitted to the ICU for continued hemodynamic monitoring after presenting to the emergency department with severe bradycardia with a heart rate of 35/min and hypotension with a blood pressure of 86/35 mmHg. An initial set of labs is shown below. Patient value Reference range Sodium 130 mEq/L 134-144 mEq/L Potassium 6.7 mEq/L 3.6-5.0 mEq/L Creatinine 2.5 mg/dL 0.8-1.4 mg/dL BUN 58 mg/dL 5-20 mg/dL Bicarbonate 9 mEq/L 21-28 mEq/L Glucose 157 mg/dL 80–140 mg/dL The patient has received atropine, calcium gluconate, insulin/dextrose, furosemide and high-dose albuterol with improvement of her hyperkalemia and bradycardia. However, hypotension persists, requiring the initiation of dobutamine. Despite initial interventions, the patient continues to have bradycardia and refractory hyperkalemia and subsequently requires dialysis. Discontinuation of which of the following patient's medications is most likely to improve her clinical status?

Choices: 1. Atorvastatin 2. Ranolazine 3. Dapagliflozin 4. Methimazole

Answer: 2 - Ranolazine Explanations: Efficacy of standard ACLS management of bradycardia with atropine is unreliable in BRASH syndrome as the bradycardia is not parasympathetically driven. Medications that directly or indirectly decrease the calcium concentration or block the AV node, increase the risk of developing BRASH syndrome. Early recognition and aggressive treatment of this syndrome can decrease the likelihood of requiring dialysis. Complete cessation of these offending medications is often unrealistic as they are crucial in the management of other comorbidities. However, it is important to review home medications because understanding the patients increased risk will lead to a more rapid diagnosis and treatment. Go to the next page if you knew the correct answer, or click the link image(s) below to further research the concepts in this question (if desired).

Research Concepts: BRASH Syndrome

We update eBooks quarterly and Apps daily based on user feedback. Please tap flag to report any questions that need improvement.

Question 857: A 69-year-old male is admitted to the intensive care unit with a hemorrhagic stroke. On neurologic examination absence of brainstem reflexes is noted and the apnea test does not reveal any spontaneous respiratory drive in the patient. What is the next best step in the management of this patient?

Choices: 1. Pronounce brain stem death 2. Take consent for organ donation from the relatives 3. Stop supportive management in the patient 4. Repeat examination after 6 hours

Answer: 4 - Repeat examination after 6 hours Explanations: Diagnosing brain stem death requires complete examination and documentation from at least two qualified physicians with a minimum interval of at least 6 hours. The team advocated for the same include the medical superintendent of the hospital, the treating physician with one neurologist or a neurosurgeon with no inclusions of any members from organ donation or transplant team. The relatives should be given the utmost respect and some time for bereavement from the loss of their beloved before embarking upon the aspect of organ donation. The supportive treatment needs to be continued until the relatives' consent of the termination of the care. Supportive management is also justified for the preservation of other organs during the probable aspect of organ donation. Go to the next page if you knew the correct answer, or click the link image(s) below to further research the concepts in this question (if desired).

Research Concepts: Brainstem Death

We update eBooks quarterly and Apps daily based on user feedback. Please tap flag to report any questions that need improvement.

Question 858: A 45-year-old man presents with progressive exertional dyspnea, fever, and dry cough. He has a past medical history of HIV. Blood tests show CD4 count 150 cell/mm3 (500-1500). A chest x-ray reveals bilateral interstitial infiltrates. The patient is started on trimethoprim-sulfamethoxazole but does not improve and shows signs of severe infection. Which of the following is the most effective method of administration of the drug that should be prescribed for this patient's new symptoms?

Choices: 1. Intramuscular 2. Nebulized 3. Intravenous 4. Oral

Answer: 2 - Nebulized Explanations: Pentamidine is an antiprotozoal drug for pneumocystis jiroveci pneumonia and other related infections. It can be administered IV, IM, orally, or as an inhaled dose. It can be given therapeutically or prophylactically to HIV-positive patients. A nebulized dose is more commonly used prophylactically and in treatment. Go to the next page if you knew the correct answer, or click the link image(s) below to further research the concepts in this question (if desired).

Research Concepts: Pentamidine

We update eBooks quarterly and Apps daily based on user feedback. Please tap flag to report any questions that need improvement.

Question 859: A 65-year-old male presents with complaints of watery diarrhea for three days and decreased urine output for one day. He reports extreme lethargy and generalized weakness. On examination, there is a pale looking male with signs of dehydration, his blood pressure is 100/60 mmHg, pulse 88/min regular, respiratory rate 22/min, and decreased strength in all four limbs. His arterial blood gases reveal pH 7.28, HCO3 15 mmol/L, and pCO2 33 mmHg. His serum sodium is 148 mEq/L, potassium 4 mEq/L, and chloride 126 mmol/L. Having calculated his anion gap, how can one determine what system is leading to this acid-base imbalance?

Choices: 1. By measuring calcium 2. By measuring albumin and correcting the gap 3. By calculating urinary anion gap 4. It cannot be determined

Answer: 3 - By calculating urinary anion gap Explanations: Determining exact etiology of a narrow anion gap hyperchloremic acidosis requires another test, the urine anion gap. The urine anion gap is calculated. Urine anion gap = (Na + K) - Cl where Na is urine sodium, K is urine potassium, and Cl is urine chloride. The urine anion gap provides an estimate of urinary ammonium (NH4) excretion. The normal renal response to metabolic acidosis is to increase acidic NH4 excretion renally. A positive urine anion gap between 20 and 90 mEq/L is indicative of low or normal NH4 excretion, such as is seen in distal renal tubular acidosis. A negative urine anion gap between -20 and -50 mEq/L is indicative of increased NH4 excretion. This occurs in patients with metabolic acidosis generated by profuse watery diarrhea. A urine anion gap approaching 0 is indeterminate. Go to the next page if you knew the correct answer, or click the link image(s) below to further research the concepts in this question (if desired).

Research Concepts: Hyperchloremic Acidosis

We update eBooks quarterly and Apps daily based on user feedback. Please tap flag to report any questions that need improvement.

Question 860: A previously healthy 37-year-old male develops acute chest pain associated with dyspnea. He has no cardiac risk factors and denies illicit drug use. His condition deteriorates, and he develops cardiogenic shock. Electrocardiogram demonstrates ST elevation in the lateral leads and repolarization abnormalities. Cardiac enzymes are elevated. Select the most important diagnostic test.

Choices: 1. CT of the chest 2. Coronary angiography 3. Endomyocardial biopsy and viral titers 4. Transesophageal echocardiogram

Answer: 2 - Coronary angiography Explanations: With a cardiogenic shock, coronary artery disease must be identified. Even though the patient may have acute myocarditis presenting with pseudoinfarction, myocardial biopsies do not provide guidance on treatment. The patient should also have an echocardiogram to determine the extent of cardiac dysfunction and also look for any valvular problem. Surgery for cardiogenic shock carries very high mortality even in the best of hands. An intra aortic balloon pump should be inserted before proceeding with any procedure or surgery. Go to the next page if you knew the correct answer, or click the link image(s) below to further research the concepts in this question (if desired).

Research Concepts: Cardiogenic Shock

We update eBooks quarterly and Apps daily based on user feedback. Please tap flag to report any questions that need improvement.

Question 861: A 65-year-old male with a history of coronary artery disease, hypertension, chronic kidney disease stage 3, severe aortic valve stenosis is undergoing exploratory laparotomy for suspected intestinal obstruction under general anesthesia. The surgeon is requesting paralysis for the duration of the surgery. Which of the following paralytics should be avoided in this patient given the patient's comorbidities?

Choices: 1. Rocuronium 2. Pancuronium 3. Vecuronium 4. Gantacurium

Answer: 2 - Pancuronium Explanations: Pancuronium is a nondepolarizing muscle relaxant used for endotracheal intubation and general anesthesia, and if often associated with tachycardia. Pancuronium is primarily excreted unchanged by the kidneys and may accumulate in patients with kidney disease or injury. The liver does metabolize an insignificant amount. It should not be used in patients with renal insufficiency. Pancuronium may cause tachycardia and may increase myocardial oxygen demand, which may be detrimental in a patient with coronary artery disease. The provider must also be cognizant that tachycardia is not desired in patients with severe aortic valve stenosis as this may precipitate arrhythmias and potentially sudden cardiac death. Rocuronium, gantacurium, and vecuronium are not associated with tachycardia and are, therefore, not contraindicated in this patient. Go to the next page if you knew the correct answer, or click the link image(s) below to further research the concepts in this question (if desired).

Research Concepts: Nondepolarizing Paralytics

We update eBooks quarterly and Apps daily based on user feedback. Please tap flag to report any questions that need improvement.

Question 862: A 66-year-old man with a past medical history of alcohol use disorder underwent a right chronic subdural hematoma using a burr hole 10 days ago and discharged home two days later. He now presents to the emergency department after suffering from a seizure for the first time. The patient is drowsy. In addition to the seizure episode, he also complains of headaches, fever, and chills. The initial exam shows right upper extremity weakness. Which of the following is the most likely diagnosis?

Choices: 1. Epilepsy 2. Recurrent subdural hematoma 3. Subdural infectious collection 4. Alcohol intoxication

Answer: 3 - Subdural infectious collection Explanations: Infected subdural hematomas after surgical drainage may develop into an empyema. Predisposing factors associated with the development of subdural empyema include prior cranial surgery. Drainage of a subdural hematoma can cause a subdural empyema. The patient can present with a seizure, headaches, fever, and decreased level of consciousness. Patients may be intoxicated from ethanol abuse and may develop seizures, but should not develop a focal neurological deficit unless they have an intracranial pathology. The history of headaches, fever, chills, and recent surgery points to an infected subdural hematoma. Go to the next page if you knew the correct answer, or click the link image(s) below to further research the concepts in this question (if desired).

Research Concepts: Subdural Empyema

We update eBooks quarterly and Apps daily based on user feedback. Please tap flag to report any questions that need improvement.

Question 863: A 65-year-old male smoker is diagnosed with community-acquired pneumonia. He is febrile, respiratory rate 32/min, alert, and blood pressure 116/80 mmHg. Lab tests show WBC at 15,000/microliter with a left shift. Serum electrolytes are normal, BUN 32 mg/dL and serum creatinine 1.5 mg/dL. Oxygen saturation on room air is 90 percent. Which of the following is correct?

Choices: 1. Admit to inpatient service and treat with ceftriaxone and azithromycin 2. Discharge to home on levofloxacin 3. Admit and treat with vancomycin and gentamicin 4. Discharge to home on amoxicillin/clavulanate

Answer: 1 - Admit to inpatient service and treat with ceftriaxone and azithromycin Explanations: The CURB-65 criteria include age 65 years or older, confusion, urea > 19.6 mg/dL, respiratory rate > 30/min, and blood pressure systolic 90 mmHg and diastolic 60 mmHg. CURB-65 score estimates mortality based on the score achieved by using these measures. The patient's age, respiratory rate, and BUN are elevated indicating giving a score of 3, conferring a 30-day mortality of 9.2 percent. A score of 2 or more requires inpatient treatment and he should be treated with a betalactam and a macrolide or a respiratory quinolone. Amoxicillin/clavulanate will not cover atypical pathogens. Go to the next page if you knew the correct answer, or click the link image(s) below to further research the concepts in this question (if desired).

Research Concepts: Community-Acquired Pneumonia

We update eBooks quarterly and Apps daily based on user feedback. Please tap flag to report any questions that need improvement.

Question 864: A 72-year-old man who suffered a large stroke two days ago is recovering in the neurological ICU. He has a dense hemiplegia bilaterally but is otherwise stable. His medical history and surgical history is significant only for poorly controlled hypertension. A nasogastric tube is placed for feeds, and the clinician placing it insufflates air and auscultates under the left costal margin to determine its position. When feeds are started, the patient desaturates, dropping to a SaO2 of 88% on room air, and he becomes tachycardic to 140. What is the appropriate next step in management?

Choices: 1. Stop the feeds and send a full electrolyte panel 2. Send the patient for an emergent CT pulmonary embolism protocol and begin therapeutic heparin 3. Send the patient for an emergent CT scan of the head 4. Stop the feeds, withdraw the nasogastric tube, and consider bronchoscopy

Answer: 4 - Stop the feeds, withdraw the nasogastric tube, and consider bronchoscopy Explanations: Given that this patient is only two days out from his stroke, refeeding syndrome is an unlikely diagnosis. The most likely diagnosis is the nasogastric tube was improperly placed, and the patient had tube feeds placed directly into his lungs. Stopping the feeds is the right option, but sending an electrolyte panel is unnecessary. It is unlikely that this patient could develop a thrombus and then embolize it after only two days of paralysis, although it is not impossible. Of course, with this presentation, a PE should be considered. However, in the setting of a recent stroke, therapeutic heparin is contraindicated. While this patient is at risk for a recurrent stroke given his history of poorly controlled hypertension, the proximity of his decomposition to the starting of tube feeds should raise suspicion that the two are related. The placement of the tube was not confirmed via Xray and only by auscultation, a known ineffective technique. This should put intrapulmonary feeds as the highest diagnosis on the differential. Intrapulmonary feeding is the most likely explanation for this patient's acute decompensation after. Additionally, the improper technique used to identify if the tube was properly placed should raise concern. The feeds should be immediately stopped and the tube withdrawn. If only a small amount of feed was instilled, then observation may be acceptable. If a large amount was given, then bronchoscopy should be considered to suction out excess. Also, there should be no reluctance to begin antibiotics if the patient develops a fever as these intrapulmonary feeds will predispose to pneumonia. Go to the next page if you knew the correct answer, or click the link image(s) below to further research the concepts in this question (if desired).

Research Concepts: Nasogastric Tube

We update eBooks quarterly and Apps daily based on user feedback. Please tap flag to report any questions that need improvement.

Question 865: A 65-year-old man with a history of human immunodeficiency virus (HIV) infection, hypothyroidism, hepatitis C infection, and hypertension presents with a productive cough and fever. His initial vital signs showed a temperature of 38.1 C (100.5 F), heart rate 111 bpm, oxygen saturation 88%, and blood pressure 103/54 mmHg. A chest radiograph reveals right lower lobe opacity. His labs show a white blood cell count of 11,000 cells/mm3, creatinine of 1.8 mg/dL, troponin level of 0.4 ng/mL, and brain natriuretic peptide of 550 pg/ml. The patient admits to nonadherence with his HIV treatment. His labs one month ago showed creatinine of 0.9 mg/dL, troponin level was 0.03 ng/mL, and brain natriuretic peptide 75 pg/ml. Bedside echocardiogram shows reduced ejection fraction compared to the last transthoracic echocardiogram he had nine months ago. His condition deteriorates in the emergency department, he is requiring more oxygen and eventually gets intubated secondary to worsening respiratory failure. What would most likely improve his cardiac function in the long term?

Choices: 1. Heparin infusion protocol 2. 40 mg of intravenous furosemide followed by 10mg/hr continuous infusion 3. 30 ml/kg/hr lactated ringers fluid resuscitation followed by broad-spectrum antibiotics 4. Continuous dobutamine infusion

Answer: 3 - 30 ml/kg/hr lactated ringers fluid resuscitation followed by broad-spectrum antibiotics

Explanations: Patients that are non-compliant with their HIV medications are at a higher risk to develop a severe infection due to their immunocompromised status. This patient presented with sepsis most likely secondary to pneumonia. He met severe sepsis criteria and had a new onset of acute kidney injury and decreased cardiac function which is suggestive of cardiorenal syndrome type 5. Patients can develop type 5 cardiorenal syndrome in the setting of sepsis, systemic lupus erythematosus (SLE), diabetes mellitus, decompensated cirrhosis, or amyloidosis; all of these disorders can lead to disease in both the heart and the kidney. The cornerstone treatment of type 5 cardiorenal syndrome is the treatment of underlying causes. Early fluid resuscitation and antibiotics administration remain the gold standard level of care for sepsis that would improve both heart and kidneys function. The patient did not have active chest pain, and his elevated troponin might be secondary to demanding ischemia due to sepsis. Starting heparin will not be necessary now. Moreover, the patient needs to be fluid resuscitated now rather than to be started on a diuretic. He most likely was intubated due to worsening infection and not due to worsening heart function. Go to the next page if you knew the correct answer, or click the link image(s) below to further research the concepts in this question (if desired).

Research Concepts: Cardiorenal Syndrome

We update eBooks quarterly and Apps daily based on user feedback. Please tap flag to report any questions that need improvement.

Question 866: A 40-year-old female is brought to the emergency department with a gunshot wound to the abdomen. The patient is emergently intubated and was taken for an exploratory laparotomy. She was found to have a small bowel injury with intraperitoneal hemorrhage. The patient received six units of blood intraoperatively. After the procedure, she was moved to the intensive care unit (ICU) for monitoring. The next day the patient undergoes central line placement for total parenteral nutrition (TPN) administration. Which of the following possible complication is most likely in this patient?

Choices: 1. Sepsis 2. Cholelitathsis 3. Hypocalcemia 4. Refeeding Syndrome

Answer: 1 - Sepsis Explanations: TPN has an increased risk of hyperglycemia, especially in the first 1-2 days, and is associated with which would be a risk factor for sepsis in this patient. This association is because hyperglycemia impairs leukocyte function and thus decreases the patient’s immune function. Cholelithiasis occurs weeks after the initiation of TPN. Hypocalcemia ould be possible due to citrate in blood transfusions but unlikely as the TPN contains calcium. While TPN administration can be complicated by refeeding syndrome, which results in electrolyte abnormality. Patients on TPN often have their electrolytes checked daily to assess any changes. Go to the next page if you knew the correct answer, or click the link image(s) below to further research the concepts in this question (if desired).

Research Concepts: Total Parenteral Nutrition

We update eBooks quarterly and Apps daily based on user feedback. Please tap flag to report any questions that need improvement.

Question 867: A 17-year-old boy is brought to the emergency department by paramedics with chest pain for 2 hours. He has associated diaphoresis and nausea. The patient is a university student and was at a local in-house party when he suddenly fell to the ground because of the debilitating chest pain. An accompanying friend says that he saw a packet of “white powder” in his room the other day. His blood pressure is 160/94 mmHg, and the pulse is 144/min. Eye examination reveals dilated pupils bilaterally. ECG shows ST-segment elevations in leads II, III, and AVF. What is the drug of choice for the stabilization of this patient?

Choices: 1. Diazepam 2. Metoprolol 3. Citalopram 4. Heparin

Answer: 1 - Diazepam Explanations: A white powder that presents as dilated pupils, increased blood pressure, tachycardia, and chest pain is most likely cocaine. Cocaine increases the concentrations of catecholamines, such as epinephrine and norepinephrine. Increased concentrations of catecholamines potentiate activation of the alpha 1, beta 1, and beta 2 receptors. In some patients, these increased concentrations can contribute to the onset of ventricular fibrillation (V-fib) along with myocardial ischemia. Diazepam reduces the sympathetic outflow by decreasing beta 1 and alpha 1 receptor activity. Metoprolol is not used for the management of acute myocardial infarction due to cocaine use. Go to the next page if you knew the correct answer, or click the link image(s) below to further research the concepts in this question (if desired).

Research Concepts: Beta 1 Receptors

We update eBooks quarterly and Apps daily based on user feedback. Please tap flag to report any questions that need improvement.

Question 868: A 70-year-old woman is readmitted to the ICU with respiratory failure requiring intubation due to chronic obstructive pulmonary disease (COPD) exacerbation. She has a history of COPD, type 2 diabetes, coronary artery disease (CAD), and a 7-day hospitalization 12 days ago. On the 4th day of her admission, she develops increased sputum production, a temperature of 101.9°F (38.8°C), deteriorating oxygenation, and a recent infiltrate in the base of the left lung. Which of the following empiric antibiotic regimen is most appropriate for this patient?

Choices: 1. Azithromycin plus moxifloxacin 2. Linezolid plus tobramycin 3. Cefepime plus vancomycin 4. Ceftriaxone plus azithromycin

Answer: 3 - Cefepime plus vancomycin Explanations: Ventilator-associated pneumonia (VAP) is a term used to describe pneumonia (lung infection) that develops in a patient who has been on mechanical ventilation for more than 48 hours. Ventilator-associated pneumonia is suspected in the setting of fever, a change in auscultation exam, a change in the chest x-ray, and an increasing requirement for respiratory support. Despite the fact that this patient has early-onset ventilator-associated pneumonia (VAP) for the recent admission, a history of the previous hospitalization is a risk factor for multidrugresistant (MDR) organisms. Initial broad-spectrum therapy with coverage of gram-negative bacilli, including Pseudomonas aeruginosa and possibly methicillin-resistant S. aureus, is generally most appropriate. Therefore, cefepime plus vancomycin should be given as empiric therapy in such patients. Go to the next page if you knew the correct answer, or click the link image(s) below to further research the concepts in this question (if desired).

Research Concepts: Ventilator-associated Pneumonia

We update eBooks quarterly and Apps daily based on user feedback. Please tap flag to report any questions that need improvement.

Question 869: A 66-year-old woman is admitted to the intensive care unit for diagnosis of septic shock due to right lower lobe pneumonia. She is given broad-spectrum antibiotics, and fluid resuscitation is begun. Despite aggressive fluid resuscitation, she continues to be hypotensive. A right subclavian line is attempted and inserted after 2 attempts. A few minutes later, the patient started to have worsening shortness of breath and hypotension with increasing pressor requirements. Which of the following is the next best step in the management of this patient?

Choices: 1. Lung ultrasound 2. Broaden antibiotic coverage 3. tPA 4. CT angiogram of the chest

Answer: 1 - Lung ultrasound Explanations: This patient had a right-sided subclavian line placed after multiple attempts leading to pneumothorax. Pneumothorax is one of the common complications of subclavian line placement. It can present as hypoxemia and shock. Urgent decompression of the pleural space is needed with the placement of a chest tube. Lung ultrasound can be performed quickly and efficiently at the bedside, which can show absence of pleural sliding and a lung point. Although a CT scan may be helpful, logistically, it can take time leading to delayed diagnosis and potentially catastrophic complications. Go to the next page if you knew the correct answer, or click the link image(s) below to further research the concepts in this question (if desired).

Research Concepts: Central Venous Access of The Subclavian Vein

We update eBooks quarterly and Apps daily based on user feedback. Please tap flag to report any questions that need improvement.

Question 870: A 56-year-old man with hypertension presented to the emergency department with central chest pain, radiating to the jaw for an hour. On arrival in the emergency department, he was found to have a heart rate of 110 beats per minute, blood pressure of 190/120 mmHg, and the rest of the examination was unremarkable. A 12 leads electrocardiogram shows sinus tachycardia with subtle ST depressions in inferior leads. What is the next step of management?

Choices: 1. Intravenous esmolol 2. Intravenous hydralazine 3. Thrombolysis 4. Primary percutaneous coronary intervention

Answer: 1 - Intravenous esmolol Explanations: Hypertensive emergency is characterized by uncontrolled blood pressure (BP) lead to progressive or impending end-organ dysfunction. Cardiovascular end-organ damage may include myocardial ischemia/infarction, acute left ventricular dysfunction, acute pulmonary edema, and/or aortic dissection. The patient has presented with a hypertension emergency, resulting in acute coronary syndrome. Beta-blockers are the preferred agents in this situation. Thrombolytics are not recommended in non-ST elevation myocardial infarction (NSTEMI) and are contraindicated in a hypertensive emergency. Go to the next page if you knew the correct answer, or click the link image(s) below to further research the concepts in this question (if desired).

Research Concepts: Esmolol

We update eBooks quarterly and Apps daily based on user feedback. Please tap flag to report any questions that need improvement.

Question 871: A 66-year-old male with a past medical history of chronic kidney disease, hypertension, and diabetes presents to the emergency room with complaints of nausea, anorexia, chest pain, and shortness of breath. His pain is sharp and gets worse with deep breathing and coughing. His pain gets better when he leans forward. His vitals are suggestive of blood pressure of 150/80 mmHg, pulse rate of 80 beats per minute, and temperature of 99.4 F. A pericardial friction rub is present on auscultation. The patient has 2+ pitting edema in his lower extremities. Laboratory data shows creatinine of 5.2 mg/dl, BUN of 110 mg/dl, and potassium of 4.8 mmol/L. His echocardiography shows a small amount of pericardial effusion. What is the best treatment option for this patient's condition?

Choices: 1. Ibuprofen 2. Colchicine 3. Pericardiocentesis 4. Hemodialysis

Answer: 4 - Hemodialysis Explanations: Uremia is an important cause of pericarditis. It occurs due to the inflammation of the pericardium due to toxic metabolites. Most patient with uremic pericarditis responds to dialysis and have rapid improvement in chest pain and pericardial effusion. Pleuritic chest pain, pericardial friction rub, and fever are major signs and symptoms of uremic pericarditis. End-stage renal disease patients on renal replacement therapy can also develop uremic pericarditis due to inadequate dialysis. Go to the next page if you knew the correct answer, or click the link image(s) below to further research the concepts in this question (if desired).

Research Concepts: Fibrinous Pericarditis

We update eBooks quarterly and Apps daily based on user feedback. Please tap flag to report any questions that need improvement.

Question 872: A 71-year-old woman is being evaluated in the intensive care unit. She was diagnosed with advanced-stage lung cancer with bony and brain metastasis six months ago. The patient was treated with aggressive chemotherapy and radiotherapy; however, she did not respond to the treatment. The surgeon decides not to operate on the patient as the surgery will not alleviate the suffering. The physician recommends terminating care and allowing the patient to die. The patient has a GCS of 8/15. Which of the following is primarily responsible for deciding on the physician's recommendation in this scenario?

Choices: 1. Ethics committee 2. Power of attorney 3. Physician 4. Patient

Answer: 2 - Power of attorney Explanations: When symptoms are no longer manageable, or patient suffering is too great, termination of care must be considered. Termination of care is not something decided on a whim; rather it is part of the initial endof-life discussions. When the treatment the healthcare team members are providing is medically futile or prolonging patient suffering, terminating the care and allowing the patient to die is necessary. If the patient can participate in these decisions, it should be the patient’s decision when they want to terminate care. If the patient is developmentally or cognitively capable of making such decisions, this responsibility falls to a power of attorney. Go to the next page if you knew the correct answer, or click the link image(s) below to further research the concepts in this question (if desired).

Research Concepts: End of Life Care

We update eBooks quarterly and Apps daily based on user feedback. Please tap flag to report any questions that need improvement.

Question 873: A 16-year-old girl is brought to the hospital with three weeks history of confusion, unusual behavior, as well as difficulty in fluent speech and being forgetful. This was preceded by flulike symptoms for about a week. During diagnostic workup, her CSF is found to be positive for anti-NMDAR IgG antibodies. Which infectious disease in the preceding weeks is most consistent with the most likely diagnosis?

Choices: 1. Herpes simplex encephalitis 2. Cat scratch disease 3. Staphylococcal skin infection 4. Streptococcal throat infection

Answer: 1 - Herpes simplex encephalitis Explanations: After HSV encephalitis, approximately one-third of patients develop autoimmune encephalitis after 2-16 weeks of primary encephalitis. Younger patients (less than 4 years) may develop autoimmune encephalitis in a shorter interval than older patients. The outcome of these autoimmune encephalitides is generally poor compared with the overall outcome of anti-NMDAR encephalitis. Cat scratch disease, staphylococcal skin infection, and streptococcal throat infection do not have an association with anti-NMDAR encephalitis. Go to the next page if you knew the correct answer, or click the link image(s) below to further research the concepts in this question (if desired).

Research Concepts: Anti-NMDA Receptor Encephalitis

We update eBooks quarterly and Apps daily based on user feedback. Please tap flag to report any questions that need improvement.

Question 874: A 24-year-old male presents to the emergency department (ED) after an opioid overdose. He required 0.6 mg of naloxone in the ED upon arrival, which he returned to baseline. During the observation period, he becomes more somnolent with a decreased respiratory rate and a falling oxygen saturation. He requires an additional 0.6 mg of naloxone with improvement. Given his course, the clinician planned to admit him on a naloxone infusion. What is the rate of the naloxone drip that should be ordered in this patient?

Choices: 1. 0.3 mg/hour 2. 0.4 mg/hour 3. 0.5 mg/hour 4. 0.6 mg/hour

Answer: 2 - 0.4 mg/hour Explanations: Naloxone is a competitive opioid antagonist that inhibits the mu-opioid receptor. In a patient who requires multiple boluses of naloxone, an infusion can be started. The initial dose per hour of naloxone is 2/3 of the bolus reversal dose of naloxone. This patient required 0.6 mg of naloxone to have a reversal of his opioid toxicity. 2/3 of 0.6 is 0.4 mg. Therefore, the initial infusion rate should be 0.4 mg/hour of naloxone via continuous infusion. Go to the next page if you knew the correct answer, or click the link image(s) below to further research the concepts in this question (if desired).

Research Concepts: Naloxone

We update eBooks quarterly and Apps daily based on user feedback. Please tap flag to report any questions that need improvement.

Question 875: A 23-year-old male patient is admitted to the emergency department after a motor vehicle collision. He has a blood pressure of 120/78 mm Hg, the respiration rate of 22 breaths per minute, a pulse of 123 beats per minute, and he is afebrile. He is immediately started on 125 ml/hr of normal saline. Over the next 3 hours, he has a urine output of 10 cc/hr, 13 ml/hr, and 18 ml/hr. His central venous pressure is 3 cm H2O, and his blood pressure remains the same. The Foley catheter appears to be draining well. What is the next step in his management?

Choices: 1. Administer furosemide intravenously 2. Continue present fluid 3. Bolus patient with 500 mL of normal saline 4. Add dopamine to the drip

Answer: 3 - Bolus patient with 500 mL of normal saline Explanations: A low central venous pressure and urine output are suggestive of a lack of fluids. The patient needs extra fluids until the central venous pressure is around 6 to 9 cm H2O. Normal saline is the fluid of choice. Pressure support is not needed. Go to the next page if you knew the correct answer, or click the link image(s) below to further research the concepts in this question (if desired).

Research Concepts: Fluid Resuscitation

We update eBooks quarterly and Apps daily based on user feedback. Please tap flag to report any questions that need improvement.

Question 876: A 65-year-old male patient with a past medical history of ulcerative colitis and multiple hospitalizations for pancreatitis presents for a scheduled bowel resection. On postoperative day two, nursing staff notice that he appears to be confused, and he complains of anxiety, insomnia, nausea, and abdominal pain. His blood pressure is 166/90 mmHg, pulse 99/min, respirations 16/min, and temperature 100.0 F (37.8 C). The patient has an alanine aminotransferase (ALT) of 209 IU/L and an aspartate aminotransferase (AST) of 422 IU/L. What is the most concerning consequence of this patient’s condition if he does not receive treatment?

Choices: 1. Agitation 2. Hallucinations 3. Seizures 4. Elevated body temperatures

Answer: 3 - Seizures Explanations: This patient has a history of multiple hospitalizations for pancreatitis, and an AST:ALT ratio of about 2:1, suggesting a history of alcohol abuse. More than 50% of those with a history of alcohol abuse can exhibit alcohol withdrawal symptoms on discontinuing or decreasing their alcohol use. However, only a few (3% to 5%) exhibit symptoms of severe alcohol withdrawal with profound confusion, autonomic hyperactivity, and cardiovascular collapse. This is defined as alcohol withdrawal delirium, more commonly known as delirium tremens (DT). The duration of the last drink becomes critical in recognizing the severity of symptoms. The initial minor withdrawal symptoms are characterized by anxiety, insomnia, palpitations, headache, and gastrointestinal symptoms. These symptoms usually occur as early as 6 hours after cessation of alcohol use. After 12 hours, minor withdrawal symptoms can progress to alcohol hallucinosis, a condition characterized by visual hallucinations. It can typically resolve in 24 to 48 hours, and may also be associated with auditory and tactile hallucinations. One of the more serious complications of alcohol withdrawal is the potential for withdrawal seizures. An alcohol withdrawal seizure is followed by alcohol hallucinosis, though it may present as early as 6 hours before alcohol hallucinosis. It can present as the only withdrawal symptoms but typically resolves 24 to 48 hours later. Seizures can recur, though rarely lead to status epileptics. Uncharacteristic signs of seizure activity should warrant further workup. If withdrawal symptoms remain untreated, this can typically lead to DT. DT is characterized by visual hallucinations, profound confusion, tachycardia, hypertension, hyperthermia, agitation, and diaphoresis. Go to the next page if you knew the correct answer, or click the link image(s) below to further research the concepts in this question (if desired).

Research Concepts: Delirium Tremens

We update eBooks quarterly and Apps daily based on user feedback. Please tap flag to report any questions that need improvement.

Question 877: Which is the most appropriate statement regarding patients with severe burns?

Choices: 1. The addition of colloid-containing solution during the first 24 hours post-burn has shown to decrease fluid requirements in critical burn patients 2. Use of the Parkland formula for fluid resuscitation is contraindicated in patients with congestive heart failure 3. Lactated Ringer's is the resuscitation fluid of choice during the first 24 hours post-burn 4. The single best indicator of adequate fluid resuscitation in major burn patients is blood pressure

Answer: 3 - Lactated Ringer's is the resuscitation fluid of choice during the first 24 hours post-burn

Explanations: Recent practice guidelines for burn shock resuscitation consider supplementation of colloidcontaining fluid after the first 24 hours post-burn, as it has shown to decrease overall fluid requirements. The American Burn Association recommends a colloid infusion of 5% albumin at the amount of (0.3-1.0)/16 mL/kg/% TBSA/hr after the initial 24-hour crystalloid resuscitation. There are no true contraindications to use of the Parkland formula. Patients with congestive heart failure and renal failure must be adequately resuscitated, but also monitored carefully for signs of fluid overload. Lactated Ringer's is the most popular choice of crystalloid solution, as it effectively treats both hypovolemia and extracellular sodium deficits caused by burn injury, and it is isotonic, inexpensive, readily available, and easily stored. The single best indicator of adequate fluid resuscitation in major burn patients is hourly urine output. The fluid rate should be adjusted to maintain urine output between 0.5 and 1 mL/kg/hour in adults, and between 1.0 and 1.5 mL/kg/hour in children. Go to the next page if you knew the correct answer, or click the link image(s) below to further research the concepts in this question (if desired).

Research Concepts: Parkland Formula

We update eBooks quarterly and Apps daily based on user feedback. Please tap flag to report any questions that need improvement.

Question 878: A 16-year-old otherwise healthy female on no medications presents complaining of swelling and is found to have a BP of 150/93, periorbital edema, 3+ peripheral edema, normal cardiovascular exam and the following lab values: Creatinine 0.7 mg/dL, albumin 2.2 g/L, Urinalysis 3+ protein, 0 RBC/HPF, 0 WBC/HPF, 1+oval fat bodies. What should be the first diagnostic test ordered?

Choices: 1. Renal ultrasound 2. Measurement of urine albumin 3. Serum protein electrophoresis 4. Test for anti-neutrophil cytoplasmic antibodies (ANCA)

Answer: 2 - Measurement of urine albumin Explanations: This is a case of nephrotic syndrome as indicated by low serum albumin resulting in hypertension and edema. Nephrotic range proteinuria is 3+ to 4+ protein by dipstick urinalysis or a urine albumin: creatinine ratio over 3.5 mg/g and the nephrotic syndrome causes oval fat bodies to be visible on microscopic analysis of urine specimens. Autoimmune diseases such as systemic lupus can cause nephrotic syndrome but are typically accompanied by other systemic signs and symptoms. Nephrotic syndrome (NS) is a clinical syndrome defined by massive proteinuria (greater than 40 mg/m2 per hour) responsible for hypoalbuminemia (less than 30 g/L), with resulting hyperlipidemia, edema, and various complications. It is caused by increased permeability through the damaged basement membrane in the renal glomerulus especially infectious or thrombo-embolic. It is the result of an abnormality of glomerular permeability that may be primary with a disease specific to the kidneys or secondary to congenital infections, diabetes, systemic lupus erythematosus, neoplasia, or certain drug use. Go to the next page if you knew the correct answer, or click the link image(s) below to further research the concepts in this question (if desired).

Research Concepts: Nephrotic Syndrome

We update eBooks quarterly and Apps daily based on user feedback. Please tap flag to report any questions that need improvement.

Question 879: A 31-year-old man of Middle Eastern descent who recently returned from a safari adventure in rural Kenya presents to the emergency department for fever, fatigue, and abdominal pain. He is found to be febrile to 102.4 F with an unremarkable physical exam. Thick and thin smears displayed intraerythrocytic trophozoite forms consistent with P. ovale infection. Complete metabolic panel is unremarkable. He is admitted to the hospital and treated with hydroxychloroquine and primaquine phosphate per the CDC protocol. On the second day of treatment, the patient reports worsening of his abdominal pain and fatigue. He attempted to walk to the bathroom and reports significant lightheadedness, palpitations, and weakness. His urine is dark. His exam is significant for continued fever, tachycardia to 120/min, scleral icterus, conjunctival pallor, right upper quadrant tenderness to palpation, and hepatosplenomegaly. Which of the following is most likely to be found on further evaluation of this patient?

Choices: 1. Direct hyperbilirubinemia; sickled cells 2. Direct hyperbilirubinemia; bite cells and Heinz bodies 3. Indirect hyperbilirubinemia; bite cells and Heinz bodies 4. Indirect hyperbilirubinemia; sickled cells

Answer: 3 - Indirect hyperbilirubinemia; bite cells and Heinz bodies Explanations: P. ovale infection is treated with chloroquine or hydroxychloroquine with either primaquine phosphate or tafenoquine. Administration of primaquine may cause a hemolytic reaction in patients with G6PD deficiency. G6PD deficiency hemolytic reaction will show bite cells and Heinz bodies on a blood smear. Hemolytic reaction results in an indirect hyperbilirubinemia. Go to the next page if you knew the correct answer, or click the link image(s) below to further research the concepts in this question (if desired).

Research Concepts: Malaria

We update eBooks quarterly and Apps daily based on user feedback. Please tap flag to report any questions that need improvement.

Question 880: A 58-year-old man presents to the emergency department with complete paralysis of the left side, new-onset facial drooping, and slurred speech. He has a history of endstage renal disease on hemodialysis, liver cirrhosis, history of gastrointestinal bleeding while on warfarin, diabetes mellitus, hyperlipidemia, hypertension, atrial fibrillation, and coronary artery disease. He has a sedentary lifestyle, eats fast food daily, and smokes 1 pack-per-day. However, he did quit drinking over 2 years ago. CT head does not show any acute abnormality. He has an NIH Stroke Scale (NIHSS) of 20. He receives a single dose of IV-tPA, which results in significant symptom improvement. His weight is 120 kg, height is 5' 3", BP is 166/90 mmHg, HbA1c 10.1%, hemoglobin 12 g/dL, AST 593 U/L, ALT 650 U/L, albumin 2.5 gm/dL, and INR 1.5. Which of the following best describes his prognosis?

Choices: 1. He is at increased risk of bleed, as he has a HAS-BLED score of 6 and a CHA2DS2-VASc score of 5 2. He has an equal chance of bleed and clotting, as she has a HAS-BLED score of 5 and a CHA2DS2-VASc score of 5 3. He is at increased risk of clots, as she has a HAS-BLED score of 5 and a CHA2DS2-VASc score of 7 4. He has an equal chance of bleed and clotting, as she has a HAS-BLED score of 6 and a CHA2DS2-VASc score of 6

Answer: 1 - He is at increased risk of bleed, as he has a HAS-BLED score of 6 and a CHA2DS2-VASc score of 5

Explanations: The risk versus benefit for assessment of initiating anticoagulation for patients with atrial fibrillation can be calculated by using validated calculations. CHA2DS2-VASc can be used to assess clot and stroke risk. MA patient has a CHA2DS2VASc score of 5 points. Age :0 (65), sex: 0 (male), CHF history: 0 (no), hypertension history: 1 (yes), stroke/TIA/thromboembolism history: 2 (yes), vascular disease history: 1 (yes), diabetes history: 1 (yes). HAS-BLED calculation can be used to assess bleeding risk. Patient's HAS-BLED score is 6 points. Hypertension: 1 (yes), renal disease: 1 (yes), liver disease: 1 (yes), stroke history: 1 (yes), prior major bleeding or predisposition to bleeding: 1 (yes), labile INR: 1 (yes), age >65: 0 (no), medication usage predisposing to bleeding: 0 (no), alcohol use: 0 (no). Comparing the patient's HAS-BLED and CHA2DS2-VASc, the patient is at increased risk of bleed especially due to recent administration of IV-tPA and high NIHSS score. Go to the next page if you knew the correct answer, or click the link image(s) below to further research the concepts in this question (if desired).

Research Concepts: Stroke Anticoagulation

We update eBooks quarterly and Apps daily based on user feedback. Please tap flag to report any questions that need improvement.

Question 881: A 34-year-old diver suffered a thoracic fracture. The most common complication is spinal shock. After ensuring airway, breathing, circulation, and resuscitation are completed, what step should be taken to help prevent complications?

Choices: 1. Begin chemical deep vein thrombosis prophylaxis within the first 72 hours 2. Begin intermittent straight cath for volumes greater than 700 ml 3. Begin medications to slow bowel transit to prevent diarrhea 4. Begin skincare and turning at least every 6 hours

Answer: 1 - Begin chemical deep vein thrombosis prophylaxis within the first 72 hours Explanations: Lack of mobility puts those with spinal cord injured patients at a higher risk of developing a deep vein thrombosis (DVT). Clinically apparent DVT occurs in approximately 15% of patients with acute spinal cord injury, and pulmonary embolism develops in approximately 5% of these patients. The risk of venous thromboembolism rises sharply if treatment is delayed beyond 72 to 96 hours. Deep vein thrombosis (DVT) is a common complication of traumatic spinal cord injury, occurring in 50 to 100 percent of untreated patients. Go to the next page if you knew the correct answer, or click the link image(s) below to further research the concepts in this question (if desired).

Research Concepts: Spinal Shock

We update eBooks quarterly and Apps daily based on user feedback. Please tap flag to report any questions that need improvement.

Question 882: A 55-year-old is being evaluated after deteriorating in the intensive care unit. He was admitted with aspiration pneumonia three weeks back and underwent elective tracheostomy for anticipated prolonged ventilation. He was started on broad-spectrum antibiotics and was demonstrating clinical and radiological signs of recovery. The patient’s condition deteriorated this morning, and he developed extensive bleeding from the tracheostomy. The nursing staff reported minor bleed while suctioning yesterday, but bleeding today has been extensive. Tracheostomy suctioning shows bright red blood from the tracheostomy site. His vitals are a blood pressure of 80/50 mmHg, a pulse of 135 beats per minute, and SpO2 80%. Overinflating the tracheostomy cuff improves the bleeding. What is the definitive management in this patient, given the underlying diagnosis?

Choices: 1. Bronchoscopy and cauterization 2. Percutaneous pulmonary artery thrombectomy 3. Arterial ligation and bypass grafting 4. Activated prothrombin complex concentrate transfusion

Answer: 3 - Arterial ligation and bypass grafting Explanations: This hospitalized patient has been in the intensive care unit for several weeks. He initially improves on broad-spectrum antibiotics but deteriorates suddenly and develops bleeding from tracheostomy. The previous history of minor bleed, the presence of fresh blood, and improvement by over inflation of cuff suggests the development of trachea-innominate artery fistula (TIAF). TIAF is a potentially fatal disease if not diagnosed and managed timely. Definitive surgical management includes either interposition grafting preserving flow in the innominate artery or ligation of the artery and distal bypass grafting to ensure flow to the subclavian and carotid artery. Innominate artery ligation and bypass are generally preferred as it reduces the risk of mortality and rebleeding. Catheterization and coiling of the fistula may be appropriate in some conditions. Prompt recognition and treatment are essential to reduce mortality in cases of TIAF. Thrombectomy can be considered in pulmonary embolism. Cauterization and prothrombin complex concentrate administration will be inappropriate in this patient. Go to the next page if you knew the correct answer, or click the link image(s) below to further research the concepts in this question (if desired).

Research Concepts: Tracheo Innominate Artery Fistula

We update eBooks quarterly and Apps daily based on user feedback. Please tap flag to report any questions that need improvement.

Question 883: What would you expect to see on prothrombin time (PT), partial thromboplastin time (PTT), and bleeding time (BT) in a patient with vWF disease?

Choices: 1. Normal PT, prolonged PTT, prolonged BT 2. Normal PT, prolonged PTT, normal BT 3. Prolonged PT, prolonged PTT, prolonged BT 4. Prolonged PT, prolonged PTT, prolonged BT

Answer: 1 - Normal PT, prolonged PTT, prolonged BT Explanations: vWF disease will have normal PT, prolonged PTT, and prolonged BT. This is due to vWF normally increasing the half-life of clotting factor VIII. vWF also binds to GpIb, and when this does not occur, platelet dysfunction occurs, hence the prolonged BT. vWF disease is the most common coagulopathy and often presents in the out-patient setting with an adolescent female expressing concern about heavy menstruation. Ristocetin agglutination assay induces platelet expression of GpIb. In vWF disease, you will have impaired platelet aggregation due to vWF deficiency as vWF binds to GpIb. Bernard-Soulier will have similar results on ristocetin agglutination assaying compared to vWF disease. The difference between these two conditions in coagulation studies is that vWF will also have prolonged PTT. Go to the next page if you knew the correct answer, or click the link image(s) below to further research the concepts in this question (if desired).

Research Concepts: Von Willebrand Disease

We update eBooks quarterly and Apps daily based on user feedback. Please tap flag to report any questions that need improvement.

Question 884: A 26-year-old male patient is being evaluated in the hospital for progressively worsening kidney function. He was admitted five days ago and was diagnosed with methicillin-resistant staphylococcus aureus infection. Intravenous vancomycin was immediately started and has been adjusted as needed to maintain a level of 15 to 20 mcg/ml. His fever has resolved, but his serum creatinine has gradually increased. His past medical history includes two prior episodes of staphylococcal endocarditis treated with antibiotics. He has a history of occasionally injecting heroin for the past four years. His only medication currently is vancomycin. On physical examination, the temperature is 37.3 C (99.1 F), blood pressure is 110/70 mmHg, heart rate is 92/min, and respiratory rate is 18 breaths/min. His BMI is 22 kg/m2. Cardiac examination reveals a soft diastolic murmur along the left sternal border and trace lower extremity edema bilaterally. No skin rash or arthritis is seen. Labs show: C3 low, C4 normal, creatinine 2.8 mg/dL from 1.5 mg/dL on admission, cryoglobulins negative, urinalysis- 3+ blood; 2+ protein; 30-40 erythrocytes per high power field (HPF); 10-15 leukocytes/HPF; and erythrocyte casts. A transthoracic echocardiogram shows moderate aortic regurgitation without vegetations. Renal ultrasound shows normal-sized, mildly echogenic kidneys. Doppler studies of the renal arteries and veins were normal. What is the most appropriate management?

Choices: 1. Initiate glucocorticoids 2. Schedule a kidney biopsy 3. Switch vancomycin to daptomycin 4. Continue current therapy

Answer: 3 - Switch vancomycin to daptomycin Explanations: The differential diagnoses include infection-related glomerulonephritis, drug-induced nephrotoxicity (from vancomycin), acute interstitial nephritis (AIN), and septic emboli. Findings of nephritic urine sediment in a patient with an active infection in the presence of therapeutic levels of vancomycin suggest infection-related glomerulonephritis rather than other etiologies. Glucocorticoids are not typically required for management of infection-related glomerulonephritis as it typically improves with control of underlying infection. A kidney biopsy is also not often required for accurate diagnosis in this setting. Vancomycin-related tubular toxicity typically occurs after 7 to 10 days and usually has bland urine sediment. Antibiotic-associated AIN is typically seen with beta-lactams and can lead to leukocyte casts but not erythrocyte casts. Go to the next page if you knew the correct answer, or click the link image(s) below to further research the concepts in this question (if desired).

Research Concepts: Acute Kidney Injury

We update eBooks quarterly and Apps daily based on user feedback. Please tap flag to report any questions that need improvement.

Question 885: A 70-year-old woman is brought to the hospital with a fever. She had a surgical procedure for the management of severe aortic stenosis 6 months ago. She is also noted to have new nail infarcts. Peripheral blood cultures are positive for Streptococcus. A PICC line is requested for long-term antibiotics. She takes warfarin daily. Which of the following is the strongest contraindication for fluoroscopic PICC line placement in this patient?

Choices: 1. Raised INR 2. Age 3. Active infection 4. Mechanical valve surgery 6 months ago

Answer: 1 - Raised INR Explanations: Mechanical aortic valves require anticoagulation to a target INR of 2.5 (2.0 - 3.0). Anticoagulation/raised INR >1.5 is a contraindication for PICC line placement, due to the increased risk of hemorrhage. Other contraindications to fluoroscopic PICC line placement include damage to the area of skin to be cannulated, e.g., burns, trauma, contamination, and pregnancy (relative risk of ionizing radiation to the fetus). Following discussion with the local hematology department and using local trust policy guidance, this patient should undergo briding low molecular weight heparin (LMWH) treatment to facilitate a reduced INR, thus allowing the fluoroscopic PICC line placement procedure on a scheduled, elective date. Go to the next page if you knew the correct answer, or click the link image(s) below to further research the concepts in this question (if desired).

Research Concepts: Fluoroscopic PICC Line Placement

We update eBooks quarterly and Apps daily based on user feedback. Please tap flag to report any questions that need improvement.

Question 886: A 46-year-old male patient with acute respiratory distress syndrome who is mechanically ventilated (tidal volume 7 ml/kg, Continuous positive airway pressure (CPAP) 12.5 cm H2O, and a fraction of inspired oxygen (FIO2) 75% becomes hypoxic (SaO2 40%) and hypotensive. Quick evaluation and physical examination of the patient is done, which shows jugular venous distension and that the breath sounds are absent over the left lung field with right tracheal deviation. Which of the following actions is most likely to be taken immediately in this patient?

Choices: 1. Increase FIO2 to 100%, get a portable chest radiograph, and await results 2. Pull the endotracheal tube back 2 centimeters 3. Assess endotracheal tube position, increase FIO2 to 100%, suction the airway, obtain an emergent chest radiograph, and consider emergent chest tube placement 4. Increase continuous positive airway pressure to 20 and start a dopamine drip

Answer: 3 - Assess endotracheal tube position, increase FIO2 to 100%, suction the airway, obtain an emergent chest radiograph, and consider emergent chest tube placement

Explanations: The most likely diagnosis is tension pneumothorax. The practitioner should rule out intubation of the right mainstem bronchus, mucus plug obstructing the left bronchus, and left hemothorax. A left chest tube should be placed if oxygen saturation of arterial oxyhemoglobin and blood pressure do not rapidly respond. The patient will likely have rapid deterioration and cardiovascular compromise leading to death if the tension pneumothorax is not recognized and corrected. Go to the next page if you knew the correct answer, or click the link image(s) below to further research the concepts in this question (if desired).

Research Concepts: Tension Pneumothorax

We update eBooks quarterly and Apps daily based on user feedback. Please tap flag to report any questions that need improvement.

Question 887: A 35-year-old male is rushed to the emergency department following a motor vehicle collision. His presenting Glasgow Coma Scale score is 9. Urgent CT brain shows diffuse cerebral edema with obliteration of all cisterns. Which of the following is the most appropriate approach in the initial management of this patient's traumatic brain injury?

Choices: 1. Methylprednisolone 2. Hypertonic saline 3. Furosemide 4. Mannitol

Answer: 2 - Hypertonic saline Explanations: Hypertonic saline is the mainstay of medically managing traumatic intracranial hypertension. It has been shown to effectively reduce intracranial pressure and improve cerebral and systemic perfusion. It is safe, has a fast onset, sustained, and protracted action with minimal morbidities. Mannitol has a risk of acute renal failure and rebound intracranial hypertension. Go to the next page if you knew the correct answer, or click the link image(s) below to further research the concepts in this question (if desired).

Research Concepts: Cerebral Edema

We update eBooks quarterly and Apps daily based on user feedback. Please tap flag to report any questions that need improvement.

Question 888: A 17-year-old male was brought to an emergency department after a motorbike accident. On arrival, his blood pressure was 98/62 mmHg, pulse was 124 bpm, respiratory rate was 11 breaths/min, and oxygen saturation was 84% at room air. His Glasgow coma scale (GCS) score was 7/15. On physical examination, there is purplish discoloration of his left tympanic membrane with bloody otorrhea. There was bruising of both eyes with tarsal plate sparing and behind the left ear. Left ear drainage is also noted, and fluid is mixed with blood. After initial management in the emergency department, a CT-scan of the head was taken, which reveal pneumocephalus. What emergency intervention could have lead to this radiological finding?

Choices: 1. Nasogastric tube intubation 2. Endotracheal tube intubation 3. Nasal intermittent positive pressure ventilation 4. Air embolization via a central line

Answer: 3 - Nasal intermittent positive pressure ventilation Explanations: Hemotympanum usually appears within hours of injury and maybe the earliest clinical finding. Periorbital ecchymosis (raccoon eyes) is typically not present during the initial evaluation and is delayed by 1 to 3 days. If bilateral, this finding is highly predictive of a basilar skull fracture. Retroauricular or mastoid ecchymosis (Battle sign) is highly predictive of basilar skull fracture. Nasal intermittent positive pressure ventilation (NIPPV) should be avoided as it may induce pneumocephalus. Basilar skull fractures are usually due to significant trauma. A thorough trauma evaluation with interventions to stabilize the airway, ventilation and circulatory issues is the priority. Associated cervical spine injury is common, so attention to cervical spine immobilization, particularly during airway management is necessary. Nasogastric tubes and nasotracheal intubation should be avoided because of the risk for inadvertent intracranial tube placement. Go to the next page if you knew the correct answer, or click the link image(s) below to further research the concepts in this question (if desired).

Research Concepts: Basilar Skull Fractures

We update eBooks quarterly and Apps daily based on user feedback. Please tap flag to report any questions that need improvement.

Question 889: A 42-year-old male was brought to an emergency department via ambulance following an alleged physical assault. He is lying on a stretcher with a cervical collar around his neck. At first glance, the health care provider noticed bruising to both eyes and behind the left ear. Left ear drainage is noted, and fluid is mixed with blood. He is resting with eyes closed with vomit in his mouth. His skin color is pale, and respirations are shallow. The Glasgow coma scale (GCS) score is 6/15. The paramedic states that the patient vomited as they arrived in the ambulance bay and became unconscious. The paramedic also relates that the patient was unconscious on the scene, then became lucid and complained of a headache, and now is unconscious again. What will be most likely finding on a CT-scan of the head?

Choices: 1. Oblique fracture of the left temporal bone along with a lentiform hyperdense area 2. Transverse fracture of the temporal bone along with crescent-shaped homogeneously hyperdense area 3. Discontinuity of the frontal bone along with a an osseous defect in the cribriform plate 4. There will be no abnormal finding on a CT-scan

Answer: 1 - Oblique fracture of the left temporal bone along with a lentiform hyperdense area

Explanations: Hemotympanum: Fractures that involve the petrous ridge of the temporal bone will cause blood to pool behind the tympanic membrane causing it to appear purple. This usually appears within hours of injury and maybe the earliest clinical finding. Basilar skull fractures are often associated with other CNS pathology like epidural hematoma due to the weakness of the temporal bone and the close proximity of the middle meningeal artery. The epidural or extradural hematoma will show a lentiform hyperdense area on a CT-scan. Patients with intracranial hemorrhage require emergent neurosurgical evaluation. Otherwise, skull base fractures are often managed expectantly. Surgical management is necessary for cases complicated by intracranial bleeding requiring decompression, vascular injury, significant cranial nerve injury, or persistent cerebrospinal fluid leak. Go to the next page if you knew the correct answer, or click the link image(s) below to further research the concepts in this question (if desired).

Research Concepts: Basilar Skull Fractures

We update eBooks quarterly and Apps daily based on user feedback. Please tap flag to report any questions that need improvement.

Question 890: A 76-year-old woman presents to the emergency department with weakness, fever, and confusion. She has a past medical history of chronic kidney disease requiring dialysis, hypertension, and osteoarthritis. She has a pacemaker. On examination, she has a temperature of 39 C, and there is a new systolic ejection murmur noted at the right upper sternal border. Conjunctival hemorrhages are noted on examination. Two blood cultures are positive for viridans streptococci. Which of the following additional dermatologic finding is most likely to be found in this patient?

Choices: 1. Warm, erythematous, painful plaques on the lower legs 2. Hemorrhagic, non-tender macules on the thenar eminences 3. Erythematous pustules on the chest 4. Scattered, brown patches with a waxy surface on the back

Answer: 2 - Hemorrhagic, non-tender macules on the thenar eminences Explanations: This patient’s presentation is infective endocarditis, which is associated with Janeway lesions. Janeway lesions are typically described as non-tender, erythematous, hemorrhagic macules commonly found on the palms and soles. Janeway lesions are considered a vascular phenomenon, and minor criteria, of infective endocarditis as per the Modified Duke criteria. Janeway lesions may or may not culture positive for micro-organisms on biopsy. The next step in treating this patient will be urgent intravenous antibiotics. An urgent echocardiogram should also be ordered to confirm the diagnosis. Go to the next page if you knew the correct answer, or click the link image(s) below to further research the concepts in this question (if desired).

Research Concepts: Osler Node and Janeway Lesions

We update eBooks quarterly and Apps daily based on user feedback. Please tap flag to report any questions that need improvement.

Question 891: A 67-year-old man is brought to the hospital with difficulty breathing for 4 hours and not subsided by the repeated use of inhalers. 3 days ago, he had developed a mild fever with a runny nose, which increased his frequency of coughing over time. The patient has a history of smoking for the past 30 years, which he quit after the diagnosis of COPD about 2 years ago. He was started on medications that he claims he is not adherent to. He is given IV hydrocortisone, noninvasive ventilation (NIV), and is transferred to the intensive care unit (ICU) for continual management. Antibiotics are also given due to unrelenting fever. He mentions that he is 'reluctant to be intubated and would rather die than having a tube down his throat.' His vital signs show no improvement despite NIV, and a blood picture demonstrating rising respiratory acidosis. Which of the following is the next best step in the management of this patient?

Choices: 1. Midazolam and invasive mechanical ventilation (IMV) 2. Conservative management 3. Extracorporeal carbon dioxide removal 4. Obtain consent for IMV from the patient's next of kin

Answer: 3 - Extracorporeal carbon dioxide removal Explanations: Extracorporeal carbon dioxide removal (ECCO2R) is a novel strategy used in patients with non-invasive ventilation (NIV) failure despite 'ultra-protective' settings. Its use has demonstrated a steady improvement of prognosis over the years and reduces the length of hospital stay. It also allows for better lung recovery by reducing the work required by the respiratory system for carbon dioxide removal. Of the possible strategies, invasive mechanical ventilation would be discarded due to patient preference, whereas ECCO2R can be considered. ECCO2R can be best used for patients not desiring invasive mechanical ventilation or as to bridge into IMV. The use of ECCO2R has demonstrated a 56% reduction in intubation rates. In conditions using IMV and ECCO2R as an adjunct to it, it is possible to allow earlier extubation. Sedating the patient with the intent to provide IMV despite his reluctance would be lawfully unethical. The patient is having an acute exacerbation of COPD, which began the downward spiral. Appropriate intervention is necessary before culminating in decompensated respiratory failure. Although IMV is highly beneficial in this case and is a valid option, it would be again unethical to move against the patient's choices with negotiating with relatives. Go to the next page if you knew the correct answer, or click the link image(s) below to further research the concepts in this question (if desired).

Research Concepts: Extracorporeal Carbon Dioxide Removal

We update eBooks quarterly and Apps daily based on user feedback. Please tap flag to report any questions that need improvement.

Question 892: A 79-year-old woman with a past medical history of long-standing hypertension, smoking, and coronary artery disease is brought to the emergency department by the caregivers for sudden onset drooling, difficulty chewing, and dysphonia. She has an abnormal expression of emotions and cries frequently. On examination, she has difficulty in tongue protrusion and exaggerated facial reflexes. What is the most likely cause of these symptoms in this patient?

Choices: 1. Progressive supranuclear palsy 2. Posterior fossa tumor 3. A previous episode of stroke 4. Bilateral thalamic infarction

Answer: 3 - A previous episode of stroke Explanations: Pseudobulbar palsy is due to an upper motor lesion caused by bilateral disturbance of the corticobulbar tracts. The corticobulbar tracts exert supranuclear control over brainstem motor nuclei and are involved in the muscular movement of the head and neck. Consecutive lacunar strokes can involve both corticobulbar tracts and cause pseudobulbar palsy. Complete history including any previous history of stroke is required. Pseudobulbar palsy is one of the severe complications of cerebrovascular diseases. Many pathological conditions can lead to pseudobulbar palsy. These include traumatic brain injury, neoplasm, or metabolic abnormality, and neurological disorders, such as amyotrophic lateral sclerosis and Parkinson disease. Other causes include post-stroke pseudobulbar palsy, demyelinating conditions such as multiple sclerosis or congenital malformations. Bilateral thalamic infarction is extremely rare. Patients often have dysphagia, drooling, dysphonia, dysarthria. They have spontaneous or inappropriate emotional outbursts. There is a brisk jaw jerk. Other facial reflexes also become exaggerated. There is paresis of the tongue and facial muscles. Go to the next page if you knew the correct answer, or click the link image(s) below to further research the concepts in this question (if desired).

Research Concepts: Pseudobulbar Palsy

We update eBooks quarterly and Apps daily based on user feedback. Please tap flag to report any questions that need improvement.

Question 893: A 65-year-old man with a past medical history of hypertension and aortic regurgitation who underwent surgical replacement of aortic valve 2 weeks ago presents to the emergency department few days after being discharged with a chief complaint of fever, severe body weakness and mild shortness of breath. His vital signs show blood pressure 112/64 mmHg, pulse rate 98/min, temperature 102.6 F, and respiratory rate 22/min. On physical exam, he is illappearing, in mild respiratory distress and has a grade IV systolic murmur. The patient is admitted to the hospital, empiric antibiotic treatment is started, and blood cultures are obtained before the initiation of treatment. Which of the following organisms is most likely to grow on the patient's blood cultures?

Choices: 1. Streptococcus viridans 2. Enterococcus faecium 3. Staphylococcus aureus 4. Candida albicans

Answer: 3 - Staphylococcus aureus Explanations: Staphylococcus is the most common microorganism to cause prosthetic valve endocarditis in the first 2 months after the valvular surgery, followed by streptococcus. Staphylococcus induced endocarditis is associated with high morbidity and mortality. Staphylococcus aureus endocarditis is associated with a higher need for surgical treatment of endocarditis. Enterococcus faecium and Candida albicans are less likely to cause prosthetic valve endocarditis in the first 2 months after the valve replacement. Go to the next page if you knew the correct answer, or click the link image(s) below to further research the concepts in this question (if desired).

Research Concepts: Prosthetic Valve Endocarditis

We update eBooks quarterly and Apps daily based on user feedback. Please tap flag to report any questions that need improvement.

Question 894: A 58-year-old man with a past medical history of diabetes mellitus, hypertension, and alcohol use disorder is admitted to the hospital with a grade 2 subarachnoid hemorrhage (Hunt and Hess scale). 5 days post-endovascular treatment of the aneurysm, the patient starts complaining of muscle cramps, nausea, and headache. His skin turgor is normal. CT imaging of the brain does not show new findings. A current set of labs is shown below. Patient value Reference range Sodium 130 mEq/L 134-144 mEq/L Potassium 4 mEq/L 3.6-5.0 mEq/L Chloride 98 mEq/L 98-107 mEq/L Bicarbonate 26 mEq/L 21-28 mEq/L Treatment with fluid restriction in this patient is most likely to increase the risk of which of the following complications?

Choices: 1. Vasospasm 2. Rebleeding 3. Seizures 4. Cerebral herniation

Answer: 1 - Vasospasm Explanations: This scenario describes a case of hyponatremia following subarachnoid hemorrhage (SAH). Hyponatremia following SAH is due to increased secretion of anti-diuretic hormone (ADH) in a majority of the cases. If the hyponatremia is due to cerebral salt wasting, there is clear evidence of hypovolemia that presents in the form of hypotension or decreased skin turgor. Hyponatremia, here, is best treated with hypertonic saline (3%). Some clinicians prefer isotonic normal saline if they feel hyponatremia is due to CSW alone. Fluid restriction leads to hypovolemia, which aggravates the risk of cerebral vasospasm. Fluid restriction should be avoided. Go to the next page if you knew the correct answer, or click the link image(s) below to further research the concepts in this question (if desired).

Research Concepts: Subarachnoid Hemorrhage

We update eBooks quarterly and Apps daily based on user feedback. Please tap flag to report any questions that need improvement.

Question 895: An 87-year-old man is brought to the emergency department from a nursing home with 1-day duration of diaphoresis, agitation, delirium, disorientation, hyperthermia, nausea, loss of appetite, and tremor. The patient has a history of major depressive disorder, hyperlipidemia, prostate cancer, and chemotherapy-induced nausea and vomiting (CINV) for which he takes ondansetron. His medication list is not available. Which of the following clinical criteria is most appropriate to use for the diagnosis of this patient?

Choices: 1. McDonald diagnostic criteria 2. SLICC classification criteria 3. Modified Duke criteria 4. Hunter criteria

Answer: 4 - Hunter criteria Explanations: Serotonin syndrome is associated with increased serotonergic neurotransmission. Many drugs can precipitate serotonin toxicity, including but not limited to fentanyl, tramadol, metoclopramide, ondansetron, lithium, buspirone, levodopa, SSRIs, SNRIs and linezolid. The diagnosis of serotonin syndrome is made using the Hunter serotonin toxicity criteria. The patient must be on a serotonergic drug and exhibit one of the following: spontaneous clonus, inducible clonus with agitation or diaphoresis, ocular clonus with agitation or diaphoresis, tremor with hyperreflexia, and hypertonia with temperature > 100.4 F and ocular or inducible clonus. Differential diagnosis includes neuroleptic malignant syndrome (4-14 days after the start of a neuroleptic drug), malignant hyperthermia (during operation or early postoperative period), anticholinergic toxicity, meningitis, and encephalitis. Management involves discontinuation of all serotonergic drugs, supportive care, benzodiazepines (agitation), and serotonin antagonists (e.g., cyproheptadine). Hyperthermic patients (>105 F) will require administration of neuromuscular blocking agents (NMBA) and endotracheal intubation. Go to the next page if you knew the correct answer, or click the link image(s) below to further research the concepts in this question (if desired).

Research Concepts: Drug Interactions In Palliative Care

We update eBooks quarterly and Apps daily based on user feedback. Please tap flag to report any questions that need improvement.

Question 896: A 65-year-old woman with sepsis secondary to pyelonephritis is admitted to an intensive care unit. The lab report shows the growth of Proteus mirabilis carrying an inosine monophosphate (IMP) carbapenemase on a nucleic acid amplification test (NAAT). The patient is intubated and sedated. She has a persistent fever for which she was started on meropenem empirically three days ago but her clinical status has worsened. Her complete blood count shows a white cell count of 22,000. What is the most appropriate antibiotic regimen for this patient?

Choices: 1. Add ciprofloxacin to meropenem 2. Piperacillin/tazobactam + gentamicin 3. Colistin + ceftazidime-avibactam 4. Tigecycline + gentamicin

Answer: 2 - Piperacillin/tazobactam + gentamicin Explanations: Proteus mirabilis has intrinsic resistance to colistin. It is also resistant to tigecycline and often to quinolones. Piperacillin/tazobactam and gentamicin should be considered. Susceptibility testing should guide therapy. Gentamicin is the best second-line treatment and rifampin can be used at third-line treatment. Using combination therapy of two or more antibiotics is associated with improved outcomes in carbapenem-resistant infections. Monotherapy with meropenem-voribactam has less efficacy than dual or triple therapy. Combination therapy with colistin and ceftazidime-avibactam may be useful, but these are not considered integral to an antibiotic regimen. Since the patient has had worsening of symptoms on meropenem and the isolate has shown carbapenemase activity, therapy must be switched. Go to the next page if you knew the correct answer, or click the link image(s) below to further research the concepts in this question (if desired).

Research Concepts: Carbapenem Resistant Enterobacteriaceae

We update eBooks quarterly and Apps daily based on user feedback. Please tap flag to report any questions that need improvement.

Question 897: A 42-year-old male patient was referred to the emergency department(ED) following an episode of massive upper gastrointestinal bleeding. His vital signs on arrival to the ED were blood pressure: 66/40 mmHg, pulse rate: 130 beats per minute, respiratory rate: 19/min, and normal temperature. his past medical history is significant for 10-year cirrhosis due to hepatitis C infection. His laboratory tests were WBC: 9800 cells/mm^3, Hb: 5 m/dl, platelet: 90000 cells/microliter. Four units packed cells were prepared for transfusion. Which one of the following is among the next step management?

Choices: 1. Administration 1 gram intravenous ceftriaxone 2. Administration 4 units recombinant factor VII 3. Application of Sengstaken Blakemore tube in the ED 4. Request an emergent upper GI endoscopy

Answer: 1 - Administration 1 gram intravenous ceftriaxone Explanations: The management of upper GI bleeding initiates with resuscitation. Several endpoints have been described to establish an adequate resuscitation, however, urine out of equal or more than 0.5 cc/kg/hour in the adult population and lowering the lactate and base deficit level might be utilized in the vast majority of institutes. The resuscitation process should initiate with isotonic serum therapy and followed by blood products in a timely manner. The management of upper GI bleeding due to esophageal varices should include prophylactic antibiotic therapy to prevent the possibility of encephalopathy. The efficacy of the resuscitation process should be evaluated frequently to prevent overcorrection as it might disturb the formed clot and aggravate the bleeding. The Sengstaken tube application demands ICU admission, induction of general anesthesia, and orotracheal intubation. Moreover, it might be utilized for less than 36 hours. The tube works with a tamponade mechanism and might result in severe necrosis during prolonged use. Although several studies reviewed the possible beneficial role of recombinant factor VII, it has not been established yet. Go to the next page if you knew the correct answer, or click the link image(s) below to further research the concepts in this question (if desired).

Research Concepts: Esophageal Varices

We update eBooks quarterly and Apps daily based on user feedback. Please tap flag to report any questions that need improvement.

Question 898: A 72-year-old man with a history of diastolic congestive heart failure is brought to the hospital with fever, dry cough, anosmia, ageusia, and dyspnea for 4 days. His wife had similar symptoms one week ago. On exam, he has a temperature of 100 F, heart rate 98/min, respiratory rate 20/min, blood pressure 134/80 mmHg, and pulse oximetry of 95% on room air. The respiratory exam shows diminished air entry bilaterally, and the rest of the exam is unremarkable. His rapid SARS CoV-2 test is positive. His CT chest shows mild ground-glass opacities bilaterally in both lungs. Which of the following is the next best step in the management of this patient?

Choices: 1. IV bamlanimuvab only 2. IV bamlanimuvab and etesevimab 3. IV dexamethasone only 4. Oral azithromycin only

Answer: 2 - IV bamlanimuvab and etesevimab Explanations: Emergency use authorization for bamlanimuvab alone was revoked due to the increased frequency of resistant variants. Combination therapy of anti-SARS-CoV-2 antibodies such as bamlanimuvab plus etesevimab or casirivimab plus imdevimab is given emergency use authorization for use in patients with COVID-19 who are not hospitalized for COVID-19 and not requiring oxygen. Corticosteroids such as dexamethasone are not indicated in patients with mild to moderate COVID-19 who are not requiring oxygen. Azithromycin and other antibacterial therapy are not recommended for patients with COVID-19 who do not have evidence of concurrent or superimposed infection. Go to the next page if you knew the correct answer, or click the link image(s) below to further research the concepts in this question (if desired).

Research Concepts: Best Practices For Administering Monoclonal Antibody Therapy For Coronavirus (COVID-19)

We update eBooks quarterly and Apps daily based on user feedback. Please tap flag to report any questions that need improvement.

Question 899: A workman is assigned to clean the outside entrance to an office complex where a suspicious letter with a white powder in it had been discovered the day before. He wears a paper facemask and latex gloves. Two of the cleaning fluids he is using are ammonia and bleach. After mixing these two agents to save time, he begins pressure washing the sidewalk and an enclosed alcove at the front entrance. After only a few minutes he begins to complain of burning in his eyes and throat and develops shortness of breath. He is taken by ambulance to a hospital, where the administration of albuterol via nebulization fails to improve his pulmonary symptoms. Which of the following causes of this scenario is the most likely?

Choices: 1. There was anthrax in the letter and the patient is suffering from inhalational pulmonary anthrax 2. The patient has occupational asthma due to the cleaning solution 3. The patient is experiencing a latex allergy to the gloves 4. The mixture of ammonia and hypochlorite generated chloramine gas, a highly water-soluble pulmonary irritant.

Answer: 4 - The mixture of ammonia and hypochlorite generated chloramine gas, a highly water-soluble pulmonary irritant.

Explanations: Bleach (hypochlorite) and ammonia react generating a highly irritant gas causing immediate pulmonary symptoms unresponsive to usual asthma medications. The bleach decomposes to form hydrochloric acid. It reacts with ammonia to form toxic chloramine fumes. This reaction was used during WWI as a weapon. Latex is more likely to cause full anaphylaxis. Occupational asthma should respond to albuterol. Anthrax would take several days to show the onset of symptoms. Go to the next page if you knew the correct answer, or click the link image(s) below to further research the concepts in this question (if desired).

Research Concepts: Chlorine Gas Toxicity

We update eBooks quarterly and Apps daily based on user feedback. Please tap flag to report any questions that need improvement.

Question 900: A 45-year-old woman is admitted to the ICU with suspicion of severe acute pancreatitis. The critical care team decides to asses the severity of the disease using the APACHE-II scoring system instead of Ranson criteria. Which of the following factors best explains why the use of Ranson criteria is less preferable in emergency settings?

Choices: 1. Fewer data available for Ranson criteria 2. Time required to fulfill the criteria 3. History of previous similar episodes 4. Etiology of pancreatitis

Answer: 2 - Time required to fulfill the criteria Explanations: Ranson criteria are used to predict the severity and mortality of acute pancreatitis. There are eleven parameters for assessment. Five parameters are assessed on admission, and the other six are assessed at 48 hours post admission. The score and severity of acute pancreatitis cannot be determined completely until 48 hours have passed since admission. This limits its utility in time-sensitive situations like the emergency department One point is given for each positive parameter for a maximum score of 11. A score of 3 or greater predicts severe acute pancreatitis and possible mortality. APACHE-II and the Bedside Index of Severity in Acute Pancreatitis (BISAP) are other scoring systems that can be used at any time and is utilized by emergency medicine physicians. Go to the next page if you knew the correct answer, or click the link image(s) below to further research the concepts in this question (if desired).

Research Concepts: Ranson Criteria

We update eBooks quarterly and Apps daily based on user feedback. Please tap flag to report any questions that need improvement.

Section 10 Question 901: A rapid response is called for a patient due to acute hypoxic hypercapnic respiratory failure. The patient is a 67-year-old male with a past medical history of obesity hypoventilation syndrome and chronic obstructive pulmonary disorder. The nurse reports that over the past one hour, the patient has become increasingly lethargic, and his arterial blood gas (ABG) analysis revealed the patient's pCO2 went from 65 mm Hg to 100 mm Hg. The patient was prepared to intubate. Upon review of his airway, there is a Mallampati score of 4, and the patient has a submental distance of 1 finger length. What would be the most critical step to ensure successful intubation in this patient?

Choices: 1. Use a tracheal tube inducer 2. Extend lower cervical spine, and flex upper cervical spine 3. Flex lower cervical spine and extend upper cervical spine 4. Attempt a surgical airway

Answer: 2 - Extend lower cervical spine, and flex upper cervical spine Explanations: Positioning the patient correctly will help mitigate a difficult airway. Positioning the patient with C6-C7 flexed and C1-C2 extended will allow the clinician the best chance of visualizing the glottis in this situation. This position is called the "sniffing position." The epiglottis serves as the primary landmark that will direct successful endotracheal intubation. The origin of the epiglottis can be found at the base of the tongue. The valley between the tongue and the epiglottis is called the vallecula. At least two to three medical professionals are required to manage the airway for rapid sequence induction. A clear delineation of roles is necessary for successful endotracheal intubation. The lead clinician is responsible for directing the team, managing the patient's airway, and intubating the patient. A nursing staff member is responsible for providing induction and paralytic medications at the onset of the procedure. At this point in time, a surgical airway is not necessary. Go to the next page if you knew the correct answer, or click the link image(s) below to further research the concepts in this question (if desired).

Research Concepts: Tracheal Rapid Sequence Intubation

We update eBooks quarterly and Apps daily based on user feedback. Please tap flag to report any questions that need improvement.

Question 902: A 65-year-old male patient is brought to the emergency department after an episode of syncope. He reports no prodromal symptoms before the episode and no involuntary loss of urine or tongue bite after the episode. The patient’s wife mentions that he did not experience any generalized body movements during the episode. He mentions that he has not experienced this before. He reports a history of episodic, substernal chest pain that lasts for 5 to 10 minutes each time and resolves with rest. He reports an upper respiratory tract infection a week ago that resolved with two tablets of over-the-counter antipyretic. The patient has a history of severe hypertension and gastroesophageal reflux disease, for which he takes lisinopril, chlorthalidone, and famotidine. He does not smoke or drink alcohol. Vitals show a blood pressure of 158/92 mm Hg, with no change noted on inspiration, a pulse of 72/min, respiratory rate of 14/min, and a temperature of 98.6 F (37 C). Physical examination reveals distended neck veins. Cardiac auscultation reveals distant heart sounds and an ejection systolic murmur heard on the right second intercostal space. Pulmonary examination reveals clear lungs bilaterally. Electrocardiogram reveals varying QRS amplitudes from beat to beat. A diagnosis is made, and appropriate treatment is begun. What is the cause of the absence of a change in blood pressure on inspiration?

Choices: 1. Valvular abnormality 2. Old age 3. History of angina 4. Medication side effect

Answer: 1 - Valvular abnormality Explanations: The most likely diagnosis is cardiac tamponade. The patient’s history of an upper respiratory tract infection, current presentation, distant heart sounds, and distended neck veins on examination and findings on the electrocardiogram indicate the diagnosis. This patient also has aortic stenosis, as suggested by the patient’s history, and ejection systolic murmur heard on the right second intercostal space. Pulsus paradoxus refers to an exaggerated fall in a patient’s blood pressure during inspiration by greater than 10 mm Hg and is frequently seen in cardiac tamponade. Cardiac tamponade without pulsus paradoxus can be seen in aortic stenosis and hypertension. Any coexisting disease that significantly elevates left ventricular diastolic pressure would contribute to this finding. Old age, history of angina, and side effects of medications that the patient takes do not contribute to this finding. Go to the next page if you knew the correct answer, or click the link image(s) below to further research the concepts in this question (if desired).

Research Concepts: Pulsus Paradoxus

We update eBooks quarterly and Apps daily based on user feedback. Please tap flag to report any questions that need improvement.

Question 903: A 25-year-old man presents to the hospital with fever, headache, and cough for 5 days. His chest x-ray is suggestive of atypical pneumonia, and the microbiology lab confirms the infection with gram-negative bacteria. Serology confirms the diagnosis and doxycycline is used for treatment. The patient wishes to leave against medical advice, but he is counseled about possible complications like osteomyelitis, endocarditis, and pseudotumors of lungs and spleen if this infection is not treated properly. Working at which of the following places is the strongest risk factor for acquiring this disease?

Choices: 1. Animal farm 2. Trekking industry 3. Fishing industry 4. Ship manufacturing factory

Answer: 1 - Animal farm Explanations: Q fever is usually seen among the workers of animal farms. Chronic manifestations of Q fever include endocarditis, osteomyelitis, and pseudotumors of lungs and spleen. The diagnosis of Q fever is made with serology. Doxycycline is the treatment of choice for Q fever. Go to the next page if you knew the correct answer, or click the link image(s) below to further research the concepts in this question (if desired).

Research Concepts: Coxiella Burnetii

We update eBooks quarterly and Apps daily based on user feedback. Please tap flag to report any questions that need improvement.

Question 904: A 35-year-old woman with a history of lupus nephritis who was on tacrolimus presented to the emergency department with a history of sudden onset headache, vomiting, disorientation, and seizure. On examination, the patient was disoriented, with a pulse of 64/min, and blood pressure of 220/120 mmHg. Her pupils were equal in size and reacted normally to light. Once she was stabilized, she complained of homonymous hemianopsia. What should be the next best step in the evaluation of this patient?

Choices: 1. Blood tacrolimus level estimation 2. MRI brain 3. Complement assay 4. EEG

Answer: 2 - MRI brain Explanations: This patient has PRES (posterior reversible encephalopathy syndrome). Tacrolimus is a risk factor for PRES. MRI brain is the diagnostic test of choice for PRES. Complement assay helps in the diagnosis of a lupus flare. EEG helps in the diagnosis of seizures. Go to the next page if you knew the correct answer, or click the link image(s) below to further research the concepts in this question (if desired).

Research Concepts: Cortical Blindness

We update eBooks quarterly and Apps daily based on user feedback. Please tap flag to report any questions that need improvement.

Question 905: A 42-year-old woman presents to the hospital for abdominal pain and fever. She was brought to the emergency department two weeks ago, following abdominal trauma. Her blood pressure is 118/68 mmHg, temeperature 100.4 F, and the pulse is 122/min. CBC reveals a WBC of 15300/microL. During her prior visit, she required embolization of the short gastric arteries for internal bleeding. A CT is ordered to confirm the diagnosis. What is the most appropriate treatment strategy for this patient?

Choices: 1. Pain control 2. IV antibiotics 3. Ultrasound-guided drainage and IV antibiotics 4. Observation

Answer: 3 - Ultrasound-guided drainage and IV antibiotics Explanations: This patient underwent embolization of the short gastric arteries which feed part of the spleen leading to splenic infarction. A rare complication of a splenic infarct is the formation of a splenic abscess. A splenic abscess will require IR drainage and IV antibiotics. Pain management and IV antibiotics are both indicated but not the mainstay of treatment. Go to the next page if you knew the correct answer, or click the link image(s) below to further research the concepts in this question (if desired).

Research Concepts: Splenic Infarcts

We update eBooks quarterly and Apps daily based on user feedback. Please tap flag to report any questions that need improvement.

Question 906: A 63-year-old man with metastatic lung cancer is being evaluated in the ICU. He is intubated for acute respiratory failure. The patient appears in respiratory distress and is tachypneic on the respirator. He is on a continuous dose of fentanyl via the intravenous route. On examination of the ventilator, the inhaled tidal volume is 200 mL higher than the exhaled tidal volume. The respiratory rate as seen on the ventilator is 40/min. Which of the following is the next best step in the management of this patient?

Choices: 1. Increase the dose of fentanyl 2. Push 2-3 mL of air in the endotracheal tube balloon 3. Intravenous rocuronium 4. Intravenous midazolam

Answer: 2 - Push 2-3 mL of air in the endotracheal tube balloon Explanations: Intubated patients should be evaluated for ventilator-patient asynchrony to mitigate pain. A discrepancy in the inhaled and exhaled tidal volume can depict an air leak, and air must be instilled into the endotracheal tube balloon to abolish the leak. Patients who are mechanically ventilated should have respiratory therapists performing frequent adjustments to the ventilator, changing the ventilatory circuit, and suction. Pulmonary medicine should be involved in abolishing ventilator-patient synchrony, which can cause severe distress. Go to the next page if you knew the correct answer, or click the link image(s) below to further research the concepts in this question (if desired).

Research Concepts: End Of Life Evaluation And Management Of Pain

We update eBooks quarterly and Apps daily based on user feedback. Please tap flag to report any questions that need improvement.

Question 907: A 67-year-old man presents to the hospital with substernal chest pain. He has a history of hypertension and ischemic heart disease. He recently underwent percutaneous coronary intervention (PCI) of the right coronary artery for >90% stenosis. EKG shows STdepressions in leads II, III, and aVF. Intravascular ultrasound shows stent under-expansion. Which of the following conditions is most likely to have caused this complication?

Choices: 1. Thrombus 2. Calcified plaque 3. Lipidic plaque 4. Spontaneous coronary artery dissection (SCAD)

Answer: 2 - Calcified plaque Explanations: Calcified plaque, when severe, can be associated with stent under-expansion, defined as failure of achieving adequate stent expansion. This can be mitigated with lesion preparation techniques before stent implantation which includes cutting balloons, scoring balloons, high-pressure balloon inflations, atherectomy (rotational, orbital, or laser) and intravascular lithotripsy (IVL). Thrombus is not associated with stent under-expansion. However, thrombus may be associated with complications including distal embolization and slow/no-reflow. Lipidic plaque is not closely associated with stent under-expansion. Lipidic plaque however at the site of a stent landing zone can be associated with edge dissections. Spontaneous coronary artery dissection (SCAD) is not associated with stent under-expansion and can often be managed medically without stent implantation. Go to the next page if you knew the correct answer, or click the link image(s) below to further research the concepts in this question (if desired).

Research Concepts: Calcified Plaque

We update eBooks quarterly and Apps daily based on user feedback. Please tap flag to report any questions that need improvement.

Question 908: A 40-year-old patient with HIV/AIDS has not been on any therapy for 18 months due to economic hardship. Four months ago, he was treated successfully for avium complex (MAC). For the past four days, he has had a fever, dry cough, and progressive dyspnea. Vital signs show a temperature of 38.9 C, blood pressure of 88/58 mmHg, heart rate of 140/min, and a respiration rate of 40 breaths per minute. Oxygen saturation is 80% on room air. The neck shows bilateral lymphadenopathy, the lungs show bilateral rales, and the abdomen is non-tender. Oxygen by a non-rebreather mask does not improve the condition of the patient. Which of the following is appropriate management for this patient?

Choices: 1. Bronchoalveolar lavage for direct immunofluorescence 2. Rapid sequence intubation and intravenous fluids 3. Complete blood count, CD4+ count, viral load, and chest x-ray 4. Sputum culture, complete blood count, and chest x-ray

Answer: 2 - Rapid sequence intubation and intravenous fluids Explanations: A further clinical evaluation must wait as the condition of the patient is worsening. The patient has impending respiratory failure most likely due to Pneumocystis jiroveci pneumonia. Arterial blood gas analysis will aid the provider in determining if steroids are required for a patient with Pneumocystis pneumonia (PCP.) Chest radiography may also be useful for cardiac presentations or for those patients who need evaluation for pulmonary infections. If chest radiography does not reveal an obvious pulmonary process and there is still concern for lung pathology computed tomography of the chest may be a consideration for further evaluation. Go to the next page if you knew the correct answer, or click the link image(s) below to further research the concepts in this question (if desired).

Research Concepts: Acquired Immune Deficiency Syndrome

We update eBooks quarterly and Apps daily based on user feedback. Please tap flag to report any questions that need improvement.

Question 909: A 35-year-old man presents to the emergency department with complaints of rigidity and difficulty walking. On examination, the patient has fine tremors and a shuffling gait. On a fundoscopic examination of the patient, disc edema and hyperemia are seen. The patient works at a factory that manufactures house paint. He says that he mistakenly consumed a liquid that looked and smelled like alcohol six days ago. Which toxic metabolite of the liquid consumed is most likely responsible for the patient's symptoms?

Choices: 1. Glycolaldehyde 2. Formic acid 3. Glycolic acid 4. Oxalic acid

Answer: 2 - Formic acid Explanations: Methanol is broken down by ADH into formaldehyde and subsequently converted to formic acid by the action of ALDH. Formic acid accumulates in the putamen of the brain and causes parkinsonian symptoms. Formic acid also causes ocular toxicity, which can be seen on fundoscopy. Glycoaldehyde is a breakdown product of ethylene glycol when acted upon by ADH in the liver. Glycolic acid and oxalic acid are the final metabolites of ethylene glycol. Both oxalic acid and glycolic acid are the end products of ethylene glycol metabolism. And the history and exam findings of the patient, it's evident that he is a case of methanol poisoning. Go to the next page if you knew the correct answer, or click the link image(s) below to further research the concepts in this question (if desired).

Research Concepts: Ethanol

We update eBooks quarterly and Apps daily based on user feedback. Please tap flag to report any questions that need improvement.

Question 910: A 22-year-old woman has just returned from a backpacking trip across Southeast Asia. She had taken loperamide for the last week for symptoms of traveler's diarrhea. She now presents to the emergency department with severe abdominal pain, abdominal distention, intractable vomiting, and an inability to pass gas. Which of the following is the most likely underlying pathophysiologic process for the patient's symptoms?

Choices: 1. Delayed gastric emptying causing toxic megacolon 2. Mesenteric vein thrombosis causing ischemia 3. Parasite infection causing intestinal obstruction 4. Constipation from dehydration

Answer: 1 - Delayed gastric emptying causing toxic megacolon Explanations: The most common side effects of loperamide are dry mouth, abdominal cramps, and nausea. Rare gastrointestinal side effects include toxic megacolon. Toxic megacolon is a known but rare complication of loperamide. The patient is unlikely to have mesenteric ischemia given her young age and lack of risk factors for vascular disease. Parasitic infections, while possible, are more likely to cause crampy abdominal pain and diarrhea. Go to the next page if you knew the correct answer, or click the link image(s) below to further research the concepts in this question (if desired).

Research Concepts: Loperamide

We update eBooks quarterly and Apps daily based on user feedback. Please tap flag to report any questions that need improvement.

Question 911: A 65-year-old man living in rural Ghana presents to the municipal hospital with intermittent fever for three days, severe fatigue, shortness of breath, and decreased urination. He reports a history of multiple bouts of P. falciparum malaria throughout his life, including three episodes in the last year diagnosed at his local hospital. He has been treated intermittently with quinine sulfate over the years. In the emergency department, his temperature is 104.5 F, blood pressure is 150/92 mmHg, heart rate is 124 bpm, and respiratory rate is 22 breaths/min with an O2 saturation of 99%. His exam is significant for scleral icterus, conjunctival pallor, jaundice, a flow murmur on cardiac auscultation, and diffuse abdominal tenderness. CBC is significant for a hemoglobin of 5.4 and a platelet count of 112,000. His creatinine is 3.0, potassium 5.8, bilirubin 4.6, and direct bilirubin is less than 0.2. Urine collected via straight catheter is dark; urinalysis is pending. The blood smear is significant for ring forms of P. falciparum and fragmented red blood cells. The pathophysiology occurring in this patient will most likely lead to which of the following?

Choices: 1. Hemoglobinuria and subsequent renal failure 2. Septic shock 3. Pulmonary edema and subsequent acute respiratory distress syndrome 4. Fatty liver infiltration

Answer: 1 - Hemoglobinuria and subsequent renal failure Explanations: Blackwater fever is severe anemia with hemoglobinuria and renal failure in the context of massive intravascular hemolysis; this patient is displaying signs of Blackwater fever via evidence of hemolysis and renal failure, given his decreased urine output, dark urine concerning for hemoglobinuria, elevated creatinine, indirect hyperbilirubinemia, and profound anemia. Blackwater fever is most often seen in the setting of repeat P. falciparum infections treated with chronic quinine; it is rare and thought to be associated with G6PD deficiency. Western Africa has high rates of P. falciparum malaria, which has the highest morbidity and mortality of the Plasmodia species. Severe anemia in malaria stems from TNF-alpha-mediated mechanisms involving both increased destruction and decreased the production of erythrocytes including cell lysis as parasites replicate and exit erythrocytes; splenic removal and autoimmune lysis of immune marked erythrocytes; poor iron incorporation into new heme molecules; and bone marrow suppression during severe infection leading to decreased production. In patients with malaria, complete blood count reveals thrombocytopenia in 60-70% of all cases and varying degrees of anemia in 29% of adults and 78% of children. Go to the next page if you knew the correct answer, or click the link image(s) below to further research the concepts in this question (if desired).

Research Concepts: Malaria

We update eBooks quarterly and Apps daily based on user feedback. Please tap flag to report any questions that need improvement.

Question 912: A 54-year-old lady presents to the emergency department with complaints of abdominal discomfort and vomiting of 2 days duration. On examination, she has a heart rate of 120 beats/min, blood pressure of 100/60 mmHg, and peripheries are cold and clammy. The abdominal examination reveals the right hypochondriac tenderness. CT abdomen was done, which revealed a perforated duodenum with a subdiaphragmatic collection. She was taken up for exploratory laparotomy, intraoperatively she required multiple vasopressors and she was shifted to ICU past Durham's patch closure of the perforation. Over the next 4 days, she was weaned off vasopressors support and mechanical ventilation. However, on a postoperative day 7, she had a new fever spike (102 F) and hypotension (80/60 mm Hg) in spite of fluid resuscitation and hence started on vasopressor support. Blood cultures revealed branching yeast. What is the medication of choice?

Choices: 1. Fluconazole 2. Amphotericin B 3. Anidulafungin 4. Flucytosine

Answer: 3 - Anidulafungin Explanations: Echinocandins (eg. anidulafungin) is the antifungal of choice in critically ill patients with fungal septicemia. Candida is a commensal in the upper gastrointestinal tract but may result in life-threatening sepsis in immunocompromised patients. The duration of therapy is 14 days after the last negative blood culture. It is recommended to send blood cultures 48 hours after starting therapy. Once the patient has stabilized azoles, or other antifungals can be initiated according to culture sensitivity. Go to the next page if you knew the correct answer, or click the link image(s) below to further research the concepts in this question (if desired).

Research Concepts: Antifungal Antibiotics

We update eBooks quarterly and Apps daily based on user feedback. Please tap flag to report any questions that need improvement.

Question 913: A 17-year-old male patient has been brought to the emergency department with acute confusion. The patient was at a house party, and his friends described that he was drinking alcohol and might have taken other drugs. He also has a history of panic disorder and takes medications for it. They are unable to provide any further information on the patient’s medical history. His blood pressure is 134/82 mmHg, heart rate is 116 beats per minute, respiratory rate is 18 breaths/min, and the temperature is 37.2 C (98.9F). He is lethargic, oriented to time and name, but not to place. His physical examination reveals dry mucus membranes, pupils are 4 mm, equal in size, reactive to light, and there is conjunctival hyperemia. His cardiovascular and respiratory examination is normal. What is the most appropriate next step in the management of this patient?

Choices: 1. Administer fomepizole 2. Administer naloxone 3. Supportive management 4. Gastric lavage

Answer: 3 - Supportive management Explanations: Tetrahydrocannabinol (THC) is one of the main cannabinoids derived from the cannabis plant that can produce euphoria through the binding of CB1 receptors in the central nervous system (CNS). Marijuana intoxication presents with dry mouth, conjunctival injection or hyperemia, and tachycardia. THC use can very rarely induce seizures in those with risk factors or a past history of seizures. Treatment for marijuana intoxication is supportive management. Go to the next page if you knew the correct answer, or click the link image(s) below to further research the concepts in this question (if desired).

Research Concepts: Tetrahydrocannabinol (THC)

We update eBooks quarterly and Apps daily based on user feedback. Please tap flag to report any questions that need improvement.

Question 914: An 82-year-old man is brought to the emergency department by his daughter who describes that the patient has recently been weak, lethargic, confused, and nauseous. He was recently prescribed sertraline three weeks ago. Her vital signs are within normal limits. Which of the following is the best initial step in the evaluation of this patient?

Choices: 1. Serum potassium 2. Serum sodium 3. TSH 4. Serum lipase

Answer: 2 - Serum sodium Explanations: This patient presents with weakness, confusion, nausea, and lethargy in the context of recently starting an SSRI, suggesting syndrome of inappropriate antidiuretic hormone (SIADH). Syndrome of inappropriate antidiuretic hormone ADH release is a condition defined by the unsuppressed release of antidiuretic hormone (ADH) from the pituitary gland or nonpituitary sources or its continued action on vasopressin receptors. SIADH is characterized by impaired water excretion leading to hyponatremia with hypervolemia or euvolemia. A number of drugs that are associated with SIADH act by enhancing the release or effect of ADH. The most common drugs include carbamazepine, oxcarbazepine, chlorpropamide, cyclophosphamide, and selective serotonin reuptake inhibitors (SSRIs). Carbamazepine and oxcarbazepine act in part by increasing the sensitivity to ADH. Chlorpropamide increases the number of V2 receptors in collecting tubules. As high-dose intravenous cyclophosphamide is given with a fluid load to prevent hemorrhagic cystitis, SIADH in such patients is a particular problem, leading to potentially fatal hyponatremia. SSRIs cause SIADH by an unknown mechanism, but people above 65 years of age are more at risk. "Ecstasy" (methylenedioxymethamphetamine)\ is particularly associated with the direct release of ADH (It also stimulates thirst, which further worsens hyponatremia). Less commonly, non-steroidal anti-inflammatory drugs (NSAIDs), opiates, interferons, methotrexate, vincristine, vinblastine, ciprofloxacin, haloperidol, and high dose imatinib have been linked with SIADH. Other causes of SIADH include stroke, hemorrhage, infection, trauma, mental illness, psychosis, surgery, pneumonia, asthma, atelectasis, acute respiratory failure, pneumothorax, hypopituitarism, and hypothyroidism, exogenous hormone administration, and HIV infection. Clinical manifestations of SIADH can be due to hyponatremia and decreased ECF osmolality, which causes water to move into the cells causing cerebral edema. Signs and symptoms depend upon the rate and severity of hyponatremia and the degree of cerebral edema. The earliest clinical manifestations of acute hyponatremia include nausea and malaise, which may be seen when the serum sodium concentration falls below 125 to 130 mEq/L (normal 135 to 145mEq/L). Vomiting is an ominous sign for patients with acute hyponatremia. With a more severe and acute fall in sodium concentration, headache, lethargy, obtundation, and eventually, seizures can occur. Coma and respiratory arrest can occur if the serum sodium level falls below 115 to 120 mEq/L. Acute hyponatremic encephalopathy may be reversible, but permanent neurologic damage or death can occur, particularly in premenopausal women. Go to the next page if you knew the correct answer, or click the link image(s) below to further research the concepts in this question (if desired).

Research Concepts:

Syndrome of Inappropriate Antidiuretic Hormone Secretion

We update eBooks quarterly and Apps daily based on user feedback. Please tap flag to report any questions that need improvement.

Question 915: A 45-year-old man with a history of hypertension, chronic bronchitis, and diabetes presents to the emergency department with generalized fatigue, malaise for the last six months. He reports a low-grade intermittent fever, anorexia, unintentional weight loss of 20 lbs with abdominal fullness. He is monogamous and lives with his girlfriend. He denies any substance abuse and has no pets. He is compliant with his medications with an HbA1c of 6.5%. For the past two weeks, he has had a fever, chills, night sweats, myalgia, and chest pain. On clinical examination, vitals are within normal limits with hepatosplenomegaly and clubbing of fingers. White cell count is 13,000/microL, and platelet count is 110,00/microL. Blood urea nitrogen is 36 mg/dL, creatinine 1.2 mg/dL, aspartate aminotransferase 68 IU/L, alanine aminotransferase 72 IU/L, alkaline phosphatase 130 IU/L, and total bilirubin 1.6 mg/dL. Urine analysis is normal. ESR is 100 mm/g and CRP 35 mg/L. A chest X-ray reveals no acute abnormalities. Blood cultures reveal no growth. A transesophageal echocardiogram reveals normal cardiac function. An immunofluorescence based phase I IgG antibody titer is 1:1200. What is the best initial therapy for this patient?

Choices: 1. Doxycycline and hydroxychloroquine for 18 months 2. Doxycycline and ciprofloxacin for 18 months 3. Doxycycline and rifampin for 18 months 4. Doxycycline and hydroxychloroquine for 24 months

Answer: 1 - Doxycycline and hydroxychloroquine for 18 months Explanations: Q fever endocarditis therapeutic regimen includes doxycycline and hydroxychloroquine for 18 months in native valve endocarditis and 24 months for prosthetic valve endocarditis. Valve surgery is a must in prosthetic valve involvement. Endocarditis is a typical presentation of chronic Q fever affecting aberrant valves and prosthetic valves due to a dysfunctional immune response. A fourfold decrease in IgG antibody levels implies treatment completion and an antibiotics halt. In cases of intolerance or contraindication, ciprofloxacin or rifampin can substitute hydroxychloroquine. Go to the next page if you knew the correct answer, or click the link image(s) below to further research the concepts in this question (if desired).

Research Concepts: Q Fever

We update eBooks quarterly and Apps daily based on user feedback. Please tap flag to report any questions that need improvement.

Question 916: A 65-year-old woman with alcoholic cirrhosis is admitted to the hospital for the evaluation of frequent dark stools. She reports having a history of esophageal varices that have been repaired. Vitals appear to be stable. Examination shows scleral icterus, mild puffiness of the face, mild ascites, and pedal pitting edema. She is coherent but appears to have impaired attention and intermittent jerks of the right arm, left arm, and right foot asynchronously. The patient undergoes a neurophysiological test to differentiate the intermittent jerks from seizures. What are the characteristic waveforms that may be seen in this patient?

Choices: 1. 3 Hz spike and biphasic waves 2. 3 Hz spike and wave 3. Triphasic waves 4. Generalized paroxysmal fast activity

Answer: 3 - Triphasic waves Explanations: The patient most likely has myoclonus secondary to alcoholic cirrhosis. electroencephalogram (EEG) is the best test to determine that these symptoms are typically not associated with any abnormal epileptogenic discharges. The commonly associated EEG findings in hepatic encephalopathy are diffuse slowing with abundant waveforms with a triphasic morphology and an anterior to posterior phase lag associated with arousal. 3 Hz spike and wave patterns are associated with primary generalized epilepsies like juvenile myoclonic epilepsy and absence seizures. Generalized paroxysmal fast activity is commonly associated with Lennox–Gastaut (LGS) or late LGS (LLGS) syndrome related to tonic-axial seizures, medication intractability, and poor prognosis, including mental deterioration. These are also associated with generalized epilepsies. Go to the next page if you knew the correct answer, or click the link image(s) below to further research the concepts in this question (if desired).

Research Concepts: Electroencephalogram

We update eBooks quarterly and Apps daily based on user feedback. Please tap flag to report any questions that need improvement.

Question 917: A 67-year-old man is admitted to the intensive care unit for pneumonia and sepsis. He has a radial artery catheter in place, from which blood has been drawn for labs. Labs show WBC count 10200/microL, hemoglobin/ hematocrit 10.1 g/dL /29%, serum sodium 139 mEq/L, serum potassium 4.5 mEq/L, CO2 19 mg/dL, BUN 14 mg/dL, INR 1.1 and activated partial thromboplastin time 105 seconds. Which of the patient's labs is most likely to be erroneous?

Choices: 1. Hemoglobin/ hematocrit 2. Serum sodium level 3. INR 4. Activated partial thromboplastin time

Answer: 4 - Activated partial thromboplastin time Explanations: Arterial lines usually have heparin-containing solutions running through them. This can affect the extrinsic pathway of coagulation and cause alteration in activated partial thromboplastin time. A common reason for coagulation profile error on drawing blood from the arterial line may be due to inadequate waste (of initially withdrawn blood), which can contain significant residual heparin. Heparin is strongly acidic in nature, sometimes can cause falsely low bicarbonate because of neutralization if there is too much residual heparin in the arterial line. Instead of heparin-containing solutions, normal saline can be continuously infused through the arterial lines. There has been no difference shown in the incidence of thrombosis with the use of heparin vs. nonheparinized solutions. In patients where there are concerns for bleeding or systemic exposure to heparin with preexisting derangement of coagulation profile, heparin-containing solutions can be avoided. Any solution being used should be infused at 1-3 mL/hour (depending on the age of the patient) through arterial lines to prevent clotting in the catheter. Go to the next page if you knew the correct answer, or click the link image(s) below to further research the concepts in this question (if desired).

Research Concepts: Arterial Lines

We update eBooks quarterly and Apps daily based on user feedback. Please tap flag to report any questions that need improvement.

Question 918: A 30-year-old man who is a gymnast by profession presents with a threeday history of pain in his nape after he hit it against a pole. He says the pain radiates along his right arm and forearm. On examination, there is reduced power in right biceps (4/5), and the biceps tendon reflex is diminished. A very well known provocative test for the assessment of the cervical spine is performed. Which of the following statements is true regarding the use of this test in the evaluation of cervical radiculopathy?

Choices: 1. It has a relatively high potential for false-positive results 2. It has a relatively high potential for false-negative results 3. It is best used as a solitary screening test for cervical radiculopathy 4. It is contraindicated in patients presenting with cervical radiculopathy

Answer: 2 - It has a relatively high potential for false-negative results Explanations: The Spurling test has high specificity but low sensitivity. This means it has a relatively high potential for false-negative results. The low sensitivity of the Spurling test means that in the evaluation of cervical radiculopathy, it is not an adequate solitary screening test and is best used in conjunction with other physical exam tests, in addition to detailed patient history. Cervical radiculopathy is the primary indication to perform the Spurling test. The high sensitivity of the Spurling test means that a positive result is highly indicative of cervical nerve root compression, and further diagnostic testing is indicated. Go to the next page if you knew the correct answer, or click the link image(s) below to further research the concepts in this question (if desired).

Research Concepts: Spurling Test

We update eBooks quarterly and Apps daily based on user feedback. Please tap flag to report any questions that need improvement.

Question 919: A 45-year-old African American man presents with a two-hour history of swelling of his tongue and lips and difficulty breathing. His past medical history includes type 2 diabetes mellitus and hypertension. His regular medications include metformin, and he was recently started on captopril six weeks ago. The patient is immediately intubated, and all medications stopped. What substance affected by captopril is responsible for this disease process?

Choices: 1. C1 esterase 2. Potassium 3. Bradykinin 4. Angiotensin II

Answer: 3 - Bradykinin Explanations: Angiotensin-converting (ACE) inhibitors inhibit the conversion of angiotensinogen to angiotensin I. Overall, ACE inhibitors reduce the levels of angiotensin II, and aldosterone while increasing the levels of renin and bradykinin. The main use of ACE inhibitors is to prevent remodeling and reduce cardiovascular mortality in heart failure, reduce proteinuria, and prevent the progression of diabetic nephropathy. Angiotensin II, which is decreased by captopril, causes vasoconstriction of precapillary arterioles and postcapillary venules, inhibits the reuptake of norepinephrine, and causes the release of catecholamines from the adrenal medulla. It also reduces the excretion of sodium and water, stimulates synthesis and release of aldosterone, and causes hypertrophy of vascular smooth muscle cells and cardiac myocytes. Some side effects include paroxysmal cough (1% to 10%), angioedema which can affect any region, hypotension, and hyperkalemia. In terms of angioedema, it can occur in any region such as the intestine, but angioedema of the tongue, glottis, or larynx can obstruct the airway. The prevalence of angioedema is higher in the African-American population. Treatment for angioedema involving the airways involves immediate stabilization with an endotracheal tube until the swelling resolves. Many pharmacological agents like diphenhydramine, methylprednisolone, epinephrine, or bradykinin blocking agents have been tried as treatment without definitive results. Go to the next page if you knew the correct answer, or click the link image(s) below to further research the concepts in this question (if desired).

Research Concepts: Captopril

We update eBooks quarterly and Apps daily based on user feedback. Please tap flag to report any questions that need improvement.

Question 920: A 56-year-old female patient with a malignant pleural effusion (MPE) due to metastatic breast cancer has been admitted for pleurodesis. A thoracostomy tube is placed, and 3.3 liters is rapidly evacuated from the right chest. The patient immediately begins coughing violently and complaining of pleuritic chest pain. The chest x-ray shows unilateral haziness, and fluffy infiltrates. Which of the following would prevent this condition from occurring?

Choices: 1. Thoracostomy in the operating room 2. Slow removal of fluid with intermittent clamping of the tube 3. Fluid administration at the same time that fluid is withdrawn from the chest 4. A maximum removal of 750 ml per day

Answer: 2 - Slow removal of fluid with intermittent clamping of the tube Explanations: Re-expansion pulmonary edema can be a lethal condition if not recognized. It is safer to transfer these patients to a monitored bed. Pain control should be adequate. These patients are frail, to begin with, and do not tolerate any more stress. Removal of pleural fluid by thoracostomy should be done with intermittent clamping. Go to the next page if you knew the correct answer, or click the link image(s) below to further research the concepts in this question (if desired).

Research Concepts: Chest Tube

We update eBooks quarterly and Apps daily based on user feedback. Please tap flag to report any questions that need improvement.

Question 921: A 65-year-old man with a history of prostate cancer presents to the emergency department with severe tetany and numbness around his mouth and tips of fingers. Examination reveals a GCS of 15/15, with no focal neurological deficit. Computed tomography rules out metastasis, intracranial bleeding, and ischemia. His blood tests are unremarkable except for a calcium (ionized) level of 2.5 mg/dL (reference range: 4.4 mg/dL - 5.2 mg/dL). He does not take calcium supplements, nor vitamin D. He has just been started on intravenous zoledronic acid for bone metastases. Which of the following associated conditions is most likely present in this patient?

Choices: 1. Hypoparathyroidism 2. Hypothyroidism 3. Sarcoidosis 4. Peripheral neuropathy

Answer: 1 - Hypoparathyroidism Explanations: Bisphosphonates infusion can precipitate hypocalcemia. Hypoparathyroidism, hypomagnesemia, and vitamin D deficiency are associated with hypocalcemia. Extra caution needs to be taken while starting bisphosphonates in these patients. Sarcoidosis is associated with hypercalcemia, not hypocalcemia. Hypothyroidism and peripheral neuropathy are not usually associated with hypo/hypercalcemia. Go to the next page if you knew the correct answer, or click the link image(s) below to further research the concepts in this question (if desired).

Research Concepts: Bisphosphonate Toxicity

We update eBooks quarterly and Apps daily based on user feedback. Please tap flag to report any questions that need improvement.

Question 922: A 26-year-old male presents following intentional ingestion of approximately 30 grams of copper sulfate 4 hours before admission. On admission, he complains of epigastric and right upper quadrant pain, severe nausea, and melaena. There are no other bleeding manifestations. His urine output is reduced and noted to be dark in color. He does not complain of shortness of breath but does have chest pain. What is the most appropriate next step in the management of this patient?

Choices: 1. Induce emesis and initiate penicillamine therapy 2. Induce emesis and initiate plasmapheresis therapy 3. Initiate antiemetics to avoid emesis and initiate trientine therapy 4. Initiate antiemetics to avoid emesis and initiate penicillamine therapy

Answer: 4 - Initiate antiemetics to avoid emesis and initiate penicillamine therapy Explanations: Emesis must be avoided in cases of copper ingestion as repeated exposure of the esophagus to this corrosive agent may inflict further damage on the mucosa. Symptom management and augmenting excretion, thereby reducing the total body copper, are the mainstays of treatment in acute copper toxicity. Antiemetics should be readily available for this patient to avoid possible sequela related to corrosion and loss of integrity of the esophageal mucosa. Penicillamine is the first-line therapy in patients with copper toxicity and trientine should only be used as a second-line agent. Go to the next page if you knew the correct answer, or click the link image(s) below to further research the concepts in this question (if desired).

Research Concepts: Copper Toxicity

We update eBooks quarterly and Apps daily based on user feedback. Please tap flag to report any questions that need improvement.

Question 923: A 70-year-old male has a known history of alcohol-related cirrhosis, coronary artery disease, hypertension, hyperlipidemia, diabetes mellitus type 2, and obesity is brought to the emergency department. He also has an alcohol use disorder. He developed leg edema and ascites over the past two months. Yesterday he was found confused in the park. In the emergency department, he had altered mental status, scleral icterus, tense abdomen, and leg edema. His blood pressure was 85/55 mmHg, heart rate 95 bpm, and temperature 38.6 C. He was admitted to the hospital for further management. His blood work showed anemia, low platelets, impaired kidney function, and increased INR 2.5. He had increased liver enzymes and total bilirubin 4.5 mg/dL. Abdominal ultrasound showed a nodular appearance of the liver, splenomegaly, and large ascites. A paracentesis was completed, and peritoneal fluid analysis showed increased neutrophil count (>250 cells/µL), and the culture is pending. What is the best next step in management?

Choices: 1. Initiate a liver transplant evaluation 2. Initiate hemodialysis 3. Start vitamin K subcutaneously 4. Start broad-spectrum antibiotic therapy

Answer: 4 - Start broad-spectrum antibiotic therapy Explanations: This patient has acute on chronic liver failure (ACLF) type C, given that he has decompensated cirrhosis over a period of 2 months before the current clinical presentation. He had ascites for two months, and at the time of this presentation to the ED, he was febrile and confused. The peritoneal fluid analysis confirmed spontaneous bacterial peritonitis (SBP), which likely exacerbated the hepatic encephalopathy. The best next step in the management is to start a broad-spectrum antibiotic therapy. ACLF type-C is an acute worsening of liver functions in decompensated cirrhosis. Specific treatment is needed when the etiology is known, in this case, SBP. The broad-spectrum antibiotic regimen can be adjusted pending culture and sensitivity results, and the treatment should be continued for five days unless unusual organisms are found. ACLF requires supportive care and management of complications, including renal dysfunction, coagulopathy, and sepsis, in this case. Hemodialysis is not indicated yet, and the patient has alcohol use disorder despite having decompensated cirrhosis. Therefore initiation of a liver transplant evaluation is inaccurate at this time. It is alright to give vitamin K, but the initiation of broad-spectrum antibiotic therapy is the best next step. Go to the next page if you knew the correct answer, or click the link image(s) below to further research the concepts in this question (if desired).

Research Concepts: Acute On Chronic Liver Failure

We update eBooks quarterly and Apps daily based on user feedback. Please tap flag to report any questions that need improvement.

Question 924: A 17-year-old female presents to the clinic for headache, nausea, dizziness, and photophobia that has persisted for 5 weeks after a fall during a basketball game at school. She reports that she fell and hit her head on the floor but did not experience loss of consciousness. She reports that she felt “dazed” and took a few minutes before returning to play. Shortly after returning to the game, she began to experience a headache and dizziness and returned to the sideline. She has not tried to play basketball or run since that time. She did not seek medical attention after the incident. Her sports trainer told her to see a healthcare provider, and she was diagnosed with having sustained a concussion. She endorses a history of a previous concussion that occurred 6 months before this injury, but states she “was back to normal by the end of the week.” Her initial evaluation in the clinic today reveals 12 positive symptoms on SCAT5 (Sport Concussion Assessment Tool fifth Edition) and is VOMS (Vestibular/OcularMotor Screening) positive for concussion. What would be the best recommendation at this time?

Choices: 1. Return to play 2. Obtain a CT head 3. Obtain brain MRI, continue to rest, and take NSAIDs for headache 4. Obtain brain MRI, exercise or activity as tolerated, and re-evaluate in 1 month

Answer: 4 - Obtain brain MRI, exercise or activity as tolerated, and re-evaluate in 1 month Explanations: A patient who is symptomatic at 1-month post-injury should obtain a further evaluation. Head CT is recommended in the emergent setting, but for patients that remain symptomatic for great than 1 month, an MRI is recommended. At 1 month post-injury, patients should not continue the use of NSAIDs for pain or headache but may use NSAIDs within the first week of the injury. Delayed cognitive and physical rest has not shown benefit, and improved outcomes have been noted in patients who return to normal activity as soon as tolerated. Patients will require close follow-up to assess for the further need for cervical physical therapy, vestibular therapy, or vision therapy. This patient is at high-risk for delayed recovery and postconcussive syndrome due to female gender, age, and prior history of concussion just 6 months before the most recent injury. Go to the next page if you knew the correct answer, or click the link image(s) below to further research the concepts in this question (if desired).

Research Concepts: Postconcussive Syndrome

We update eBooks quarterly and Apps daily based on user feedback. Please tap flag to report any questions that need improvement.

Question 925: A 34-year-old female admitted to an inpatient unit. She was diagnosed with primary syphilis a few days ago, and she is started on IV medications. Two hours later, she complains of feeling feverish and headache. Examination reveals a temperature of 100.2 F, blood pressure of 135/80 mmHg, and a pulse rate of 68/minute. A careful examination of the patient's body reveals a rash, previously absent. There is no history of drug allergies in the past. Her past medical history is unremarkable. What is the most likely cause of the patient's current condition?

Choices: 1. Drug side effect 2. Allergic reaction 3. Tissue invasion by spirochetes 4. Toxin and cytokine release secondary to lysis of spirochetes

Answer: 4 - Toxin and cytokine release secondary to lysis of spirochetes Explanations: Herxheimer reaction occurs because of toxins and cytokines released during the lysis of spirochetes following antibiotic treatment. Herxheimer reactions show an increase in inflammatory cytokines during the period of exacerbation, including interleukin 6, interleukin 8, and tumor necrosis factor-alpha. The release of toxins and cytokines results in body aches, fevers, rashes, nausea, and vomiting, along with other symptoms. A careful history taking is essential, covering any known drug allergies. Go to the next page if you knew the correct answer, or click the link image(s) below to further research the concepts in this question (if desired).

Research Concepts: Jarisch Herxheimer Reaction

We update eBooks quarterly and Apps daily based on user feedback. Please tap flag to report any questions that need improvement.

Question 926: A 71-year-old man is being evaluated in the ICU for profound weakness, after being admitted for three weeks. Electrodiagnostic studies are performed at the bedside. Which of the following muscles is most likely to show early recruitment of small-amplitude motor unit action potentials on needle EMG in this patient?

Choices: 1. Abductor pollicis brevis 2. Tibialis anterior 3. Diaphragm 4. First dorsal interossei

Answer: 3 - Diaphragm Explanations: In critical illness myopathy, proximal muscles more than distal muscles, including the diaphragm muscle, will show early recruitment of small-amplitude motor unit action potentials on needle EMG. Patients with profound weakness following a prolonged ICU admission or intubation should be evaluated for critical illness myopathy. Although it is technically challenging in the ICU, a needle EMG of the diaphragm muscle may reveal abnormal spontaneous potentials with MUAPs of small amplitude, shorter duration, and early recruitment. To accurately diagnose CIM, EMG studies should be performed in at least three extremities, comparing both distal and proximal muscles, and ideally, evaluation of the diaphragm should be performed as well. Go to the next page if you knew the correct answer, or click the link image(s) below to further research the concepts in this question (if desired).

Research Concepts: Electrodiagnostic Evaluation Of Critical Illness Myopathy

We update eBooks quarterly and Apps daily based on user feedback. Please tap flag to report any questions that need improvement.

Question 927: A 65-year-old patient presents to the emergency department with complaints of loose, watery stools for the past 2 days. He has had 6-7 episodes per day associated with crampy abdominal pain and fever. His past medical history includes hospitalization 6 months back for a bout of pneumonia during which he had developed a prolonged bout of diarrhea. On examination, he is pyrexial with a temperature of 100 F, blood pressure of 90/50 mmHg, a pulse of 110 beats per minute, and a respiratory rate of 20 breaths per minute. His abdominal examination shows tenderness in his left lower quadrant. His investigations revealed a white blood cell count of 28,000 per microL, CRP of 50 mg/l, serum creatine of 1.3mg/dl, and albumin of 3.0 g/l. His stool assay is positive for glutamate hydrogenase and toxin B. He is started on appropriate therapy. Addition of which of the following agents is associated with a reduced incidence of relapse in this patient?

Choices: 1. Infliximab 2. Prednisolone 3. Bezlotoxumab 4. Ciclosporin

Answer: 3 - Bezlotoxumab Explanations: This is an elderly patient with a history of hospitalization and likely a previous episode of Clostridium difficile associated diarrhea, who has presented with severe diarrhea. The diagnosis is a relapse of Clostridium difficile colitis. Bezlotoxumab has been approved by the FDA to be used as an adjunct to reduced the likely hood of relapse in high-risk patients. High-risk patients are those with advanced age, greater than 65 years, previous episodes of Clostridium difficile colitis, or with severe disease. MODIFY 1 and MODIFY 2 trials demonstrated the superiority of bezlotoxumab in preventing recurrence while compared to placebo. It is a human monoclonal antibody that is directed against clostridial exotoxin B. It is no used as an isolated treatment but rather in combination with the standard of care. Pulsed vancomycin, followed by tapering, can also be used to prevent a recurrence. Infliximab and ciclosporin are immunosuppressive agents used in the treatment of inflammatory bowel disease. Go to the next page if you knew the correct answer, or click the link image(s) below to further research the concepts in this question (if desired).

Research Concepts: Clostridium Difficile

We update eBooks quarterly and Apps daily based on user feedback. Please tap flag to report any questions that need improvement.

Question 928: A 75-year-old woman presents to the emergency department with significant shortness of breath and sputum production. Her past medical history is significant for depression, for which she takes fluoxetine. Initial vital signs are significant for a heart rate of 101/min and temperature of 38.2 C. She is producing copious amounts of sputum during the encounter. Culture results reveal mecA positive gram-positive cocci. What is the best initial therapy for this patient?

Choices: 1. Linezolid 2. Vancomycin 3. Ceftriaxone 4. Nafcillin

Answer: 2 - Vancomycin Explanations: This patient most likely has Staphylococcal pneumonia. mecA positivity indicates MRSA (methicillin-resistance staphylococcus aureus). Vancomycin is, therefore, the correct answer as it covers MRSA and does not have a contraindication with SSRIs. Linezolid covers MRSA but should not be used if the patient is on an SSRI. Ceftriaxone can be used with SSRIs, but it does not cover MRSA, for which this patient is infected as she is mecA positive. Nafcillin covers MSSA but not MRSA. Go to the next page if you knew the correct answer, or click the link image(s) below to further research the concepts in this question (if desired).

Research Concepts: Staphylococcal Pneumonia

We update eBooks quarterly and Apps daily based on user feedback. Please tap flag to report any questions that need improvement.

Question 929: A 38-year-old man with sickle cell anemia presents to the hospital with a fever, borderline hypotension, and left flank pain. Urosepsis is suspected as the source of this patient's early septic shock. She has no peripheral venous access, and there is no intraosseous drill available in the emergency department. The decision is made to proceed with an internal jugular central venous catheter. A superficial cervical plexus block is planned. Which of the following complications is most likely to occur in this patient?

Choices: 1. Malignant hyperthermia 2. Facial nerve paralysis 3. Torticollis 4. Puncture of the external jugular vein

Answer: 4 - Puncture of the external jugular vein Explanations: Puncture of the external jugular vein and carotid artery is possible if the needle tip is not kept under direct visualization at all times. To prevent this, it is also highly recommended to not advance the needle more than 3 cm in depth. Inadvertent deep injection or excessive local anesthetic volume can also cause dysfunction of the phrenic nerve, recurrent laryngeal nerve, deep cervical plexus, and brachial plexus. Malignant hyperthermia is a complication of general anesthesia. Accidentally blocking the accessory nerve can cause sternocleidomastoid and trapezius muscle weakness. Go to the next page if you knew the correct answer, or click the link image(s) below to further research the concepts in this question (if desired).

Research Concepts: Cervical Plexus Block

We update eBooks quarterly and Apps daily based on user feedback. Please tap flag to report any questions that need improvement.

Question 930: A 65-year-old male presents with his daughter for fever and increased drowsiness. His daughter states that he has been having high-grade fever and chills for the past week. For the last two days, her father has become progressively more confused and difficult to arouse. He has a history of diabetes mellitus and benign prostatic hyperplasia. His medications include metformin, glibenclamide, dapagliflozin, multivitamin, and tamsulosin. His examination reveals a blood pressure of 90/40 mmHg, a pulse of 125 beats per minute, respiratory rate of 12/min, a temperature of 102 F (38.9 C), and spO2 of 88%. His physical examination reveals suprapubic and right flank discomfort. His investigations reveal a WBC count of 17,000 per microL, hemoglobin of 11 gm/dl, platelet count of 120,000 per microL, and CRP of 75 mg/l. Ultrasound demonstrates mild right renal pelvis dilatation and the presence of a well-defined 7 cm hypoechoic collection in the right perinephric space around the lower pole of the kidney. The lesion has increased peripheral vascularity. His post-void residual urine volume is 175 mL. He is started on broad-spectrum antibiotics, intravenous fluids, and vasopressors. Which additional treatment modality should be used next in this patient?

Choices: 1. Bladder irrigation 2. Cystoscopy with double J stenting 3. Percutaneous drainage of the perinephric collection 4. Nephrectomy

Answer: 3 - Percutaneous drainage of the perinephric collection Explanations: This patient has presented with signs of fever and altered sensorium. His examination reveals the presence of suprapubic and costovertebral angle tenderness, indicating the presence of an underlying urinary tract infection. His investigations reveal leukocytosis and an elevated CRP. His ultrasound demonstrates a perinephric collection of fluid, indicating the development of a perinephric abscess. Urosepsis requires intensive care and early goal-directed therapy. Institution of antibiotics, fluids, and pressors should be initiated as appropriate. The primary and critical treatment is drainage of the abscess. Image-guided percutaneous drainage should be performed for this perinephric collection. Drainage will ensure a response to antibiotics and lead to a reduction of the microbial load and bacteremia. A pigtailed catheter can be left in situ to ensure complete resolution. This patient is currently hemodynamically unstable and cannot safely tolerate a major surgical procedure. Double J stents are useful in the case of ureteric obstruction, but that is not the case here as there is no hydronephrosis. A Foley catheter should be placed in this individual for his urinary retention, but bladder irrigation would not be helpful. Go to the next page if you knew the correct answer, or click the link image(s) below to further research the concepts in this question (if desired).

Research Concepts: Urosepsis

We update eBooks quarterly and Apps daily based on user feedback. Please tap flag to report any questions that need improvement.

Question 931: Targeted Temperature Management (TTM) is a recommendation following cardiac arrest if a return of spontaneous circulation occurs and the patient remains comatose. What is the target temperature range?

Choices: 1. 35 to 36 degrees Celsius 2. 32 to 36 degrees Celsius 3. 28 to 32 degrees Celsius 4. 24 to 28 degrees Celsius

Answer: 2 - 32 to 36 degrees Celsius Explanations: The target temperature, according to AHA, is 32-36 degrees Celsius. Targeted temperature management (TTM) is useful when the return of spontaneous circulation (ROSC) has occurred, and the patient remains in an unresponsive/comatose state according to the American Heart Association 2015 update. In adults, TTM is maintained for twenty-four hours. The routine cooling of patients in a prehospital setting after the return of spontaneous circulation (ROSC) is no longer a recommendation as of the 2015 AHA update. Go to the next page if you knew the correct answer, or click the link image(s) below to further research the concepts in this question (if desired).

Research Concepts: Anoxic Encephalopathy

We update eBooks quarterly and Apps daily based on user feedback. Please tap flag to report any questions that need improvement.

Question 932: A 65-year-old woman presents to the emergency department with a headache and dyspnea that have been worsening in the last several hours. Further questioning reveals that she has new-onset pain and swelling of her hands, cold, painful fingers with cold exposure. On examination she appears anxious, beak-like nose, thickening of fingers skin and bilateral crackles. Her vital signs are pertinent for blood pressure 240/110 mmHg, heart rate 90/minute, afebrile. Laboratory investigations show white blood cell 3.7 x10^9/L, hemoglobin 10 g/dL, platelets 460 x10^9/L, blood urea nitrogen 52 mg/dL, creatinine 3.4 mg/dL, positive anti-nuclear antibody (ANA) at titre 1:320 in speckled pattern, anti-ribonucleic acid (RNA) polymerase antibody 30 units/mL (20 units/mL), presence of anti scleroderma-70 antibody 2 U (1 U), normal complements. Which of the following is the most appropriate treatment initial treatment for her blood pressure?

Choices: 1. Oral hydralazine 2. Labetalol infusion 3. Intravenous enalaprilat 4. Oral hydrochlorthiazide

Answer: 3 - Intravenous enalaprilat Explanations: Systemic sclerosis (SSc) is a multisystem autoimmune disorder that can manifest as diffuse cutaneous or limited cutaneous variant. Scleroderma renal crisis (SRC) complicates the course of 10% of patients of systemic sclerosis. It is a life-threatening complication characterized by endothelial dysfunction, renin-angiotensin-aldosterone system (RAAS) activation, that presents with the abrupt onset of severe hypertension, rapidly progressive renal failure, and microangiopathic hemolytic anemia. Most cases of scleroderma renal crisis occur in patients with diffuse scleroderma (10% to 25%) as compared to limited cutaneous variant (1% to 2%). The disease usually occurs early in the course of scleroderma, up to 75% of cases of scleroderma renal crisis developing within the first four years from the diagnosis. This patient is presenting with hypertensive emergency, renal failure, anemia, serology pointing towards scleroderma. As RAAS activation plays central role in this condition, angiotensin inhibitors have been used and found to reduce mortality rate from 85% to 24%. Go to the next page if you knew the correct answer, or click the link image(s) below to further research the concepts in this question (if desired).

Research Concepts: Scleroderma And Renal Crisis

We update eBooks quarterly and Apps daily based on user feedback. Please tap flag to report any questions that need improvement.

Question 933: A 91-year-old woman is admitted to the hospital with a cough and fever. She was recently diagnosed with stage 3 small cell lung cancer. A chest x-ray is suggestive of right lower lobe pneumonia. The patient is intubated, transferred to the intensive care unit, and treated with appropriate antibiotics and fluids. After two days, the patient's condition does not improve, and the family expresses their decision to extubate the patient terminally. After a detailed discussion and appropriate documentation, terminal extubation is planned. The patient is extubated, but she does not regain consciousness. She continues to breathe at 10 breaths per minute, saturating at 70%. Her family is at the bedside. Which of the following is the next best step in the management of this patient?

Choices: 1. Repeat chest x-ray 2. High flow oxygen 3. Turn off monitor alarms 4. IV morphine

Answer: 3 - Turn off monitor alarms Explanations: The clinical scenario describes a patient needing end of life care after terminal extubation. The patient is breathing at 10 breaths per minute and saturating at 70% after being extubated. Her condition is highly suggestive of the patient passing away in the next hour or so. As the end of life approaches for a patient, the alarms monitoring the patient's vital signs are likely to start blaring. To decrease the family's anxiety of watching their loved one die, it is appropriate to turn off the alarms. Repeating the chest x-ray serves no purpose as the patient is likely to pass away soon. High flow oxygen is considered too aggressive a therapy for end-of-life care. The patient is not conscious and likely not in any distress. There is no indication for IV morphine for palliation. Go to the next page if you knew the correct answer, or click the link image(s) below to further research the concepts in this question (if desired).

Research Concepts: End of Life Care

We update eBooks quarterly and Apps daily based on user feedback. Please tap flag to report any questions that need improvement.

Question 934: A 65-year-old male with a past medical history of paroxysmal atrial fibrillation, ischemic cardiomyopathy, severe left ventricular dysfunction, and COPD presents with symptoms of the increasing burden of heart palpitations and fatigue for one week. On physical exam, he is well-nourished and in no acute distress. His breath sounds are diminished but clear, and he is noted to have an irregular rhythm at a rapid rate. Implantable loop recorder interrogation reveals atrial fibrillation with a rapid ventricular rate for the past one week. He has a normal renal function, and EKG reveals a QT interval of 414 msec. He was treated with amiodarone for the maintenance of sinus rhythm in the past but was discontinued due to abnormal pulmonary function testing. What would be the best course of action?

Choices: 1. Start flecainide, and if persistent atrial fibrillation at the next outpatient visit, proceed to cardioversion 2. Admit to hospital, and start propafenone awaiting pharmacologic conversion 3. Rechallenge amiodarone short term with plans for discontinuation after pharmacologic conversion 4. Admit to hospital, and start dofetilide, and await pharmacologic conversion

Answer: 4 - Admit to hospital, and start dofetilide, and await pharmacologic conversion Explanations: Admitting to the hospital is important in the setting dosing dofetilide as it is proarrhythmic upon initiation and can cause dangerous ventricular arrhythmias including polymorphic ventricular tachycardia. Initiating dofetilide is important in this situation as he has symptomatic atrial fibrillation with significant previous history with medication failure. Dofetilide is a class III antiarrhythmic used for the maintenance of sinus rhythm in persistent atrial fibrillation and is useful for application in this patient. Dofetilide is safe in patients with underlying lung disease while amiodarone is contraindicated given its potential side-effect of pulmonary toxicity. Dofetilide is safe in patients with underlying structural heart disease. On the contrary, Flecainide and propafenone are contraindicated with underlying heart disease. Go to the next page if you knew the correct answer, or click the link image(s) below to further research the concepts in this question (if desired).

Research Concepts: Dofetilide

We update eBooks quarterly and Apps daily based on user feedback. Please tap flag to report any questions that need improvement.

Question 935: A 65-year-old man presents with a one-week history of central chest pain. He also complains of shortness of breath that has been worsening over the same period. His past medical history includes hypertension, hyperlipidemia, lymphoma in remission, and diabetes type 2 with a recent HbA1c of 9.2%. He currently takes aspirin, atorvastatin, amlodipine, hydrochlorothiazide, and insulin. He completed a course of anthracycline-based chemotherapy for his lymphoma six months ago. On examination, his vital signs are all within normal limits. There are bilateral decreased breath sounds on auscultation of the lungs, and the cardiovascular exam is unremarkable. On a chest x-ray, there is bilateral vascular congestion and reduced aeration throughout the lung fields. Which of the following findings most supports a diagnosis of cardiotoxicity secondary to his chemotherapy?

Choices: 1. Elevated cardiac troponin levels 2. T-wave inversions in V1, V2, and V3 leads 3. Decreased left ventricular ejection fraction 4. Prolonged QRS complex in leads V1 and V2

Answer: 3 - Decreased left ventricular ejection fraction Explanations: Due to the cardiotoxic profile of adriamycin, baseline left ventricular ejection fraction (LVEF) should be done before starting adriamycin and should be monitored routinely using an echocardiogram. An LVEF drop of more than 10% from baseline is considered to indicate cardiotoxicity from adriamycin, and immediate intervention is required. An alternative to echocardiogram monitoring is radionuclide ventriculography, such as multi-gated acquisition angiogram (MUGA) or equilibrium radionuclide angiogram (ERNA) scans, although these are less frequently used. Regular echocardiogram monitoring in patients with current or recent use of adriamycin is essential. Troponin levels and ECG abnormalities are not specific for adriamycin toxicity. Go to the next page if you knew the correct answer, or click the link image(s) below to further research the concepts in this question (if desired).

Research Concepts: Anthracycline Medications (Doxorubicin)

We update eBooks quarterly and Apps daily based on user feedback. Please tap flag to report any questions that need improvement.

Question 936: A retrospective study is studying the sensitivity of organisms to ampicillin and its extended coverage combination ampicillin-sulbactam. Patients with positive blood cultures who were given the antibiotics during hospitalization were followed up with blood cultures to document the eradication of the bacteria. Patients infected with which bacteria are most likely to have positive blood cultures after treatment with ampicillin but negative blood culture with ampicillin-sulbactam?

Choices: 1. Listeria monocytogenes 2. Escherichia coli 3. Acinetobacter baumannii 4. Streptococcus pneumoniae

Answer: 3 - Acinetobacter baumannii Explanations: The combination of ampicillin-sulbactam acts synergistically to cover strains of bacteria resistant to ampicillin, thus providing broader coverage than ampicillin alone. Ampicillin-sulbactam is used to treat skin and subcutaneous infections caused by betalactamase producing strains of Staphylococcus aureus, Klebsiella species, including K. pneumoniae, Escherichia coli, Proteus mirabilis, Bacteroides fragilis, Acinetobacter calcoaceticus, Acinetobacter baumannii, and Enterobacter species. Ampicillin-sulbactam is used to treat intra-abdominal infections caused by beta-lactamase producing strains of Escherichia coli, Klebsiella species, including K. pneumoniae, Enterobacter species, and Bacteroides species, including B. fragilis. Ampicillin-sulbactam is used to treat gynecological infections caused by beta-lactamase producing strains of Bacteroides species, including B. fragilis and Escherichia coli. Go to the next page if you knew the correct answer, or click the link image(s) below to further research the concepts in this question (if desired).

Research Concepts: Ampicillin/Sulbactam

We update eBooks quarterly and Apps daily based on user feedback. Please tap flag to report any questions that need improvement.

Question 937: An 82-year-old woman with a history of congestive heart failure and chronic obstructive pulmonary disease is brought to the hospital with acute onset left dense hemiplegia. She remains lucid and communicative and expresses her wishes to ‘die at home’ and agrees with her advance directives for ‘do not resuscitate’ and ‘do not intubate.’ Which of the following is the next most appropriate course of action?

Choices: 1. Discharge the patient home 2. Continue active intensive care management 3. Recommend funeral arrangements to the family 4. End-of-life care assessment

Answer: 4 - End-of-life care assessment Explanations: The hospice team can help end-of-life patients, who are expected to live for several days or more to ‘die at home.’ The hospice care team and multidisciplinary support (including intensivist, nursing staff, primary care physician, palliative care team, social worker, physical therapist, etc.) for providing home hospice care. The hospice team can provide the resources and help that a family needs to take care of a terminal patient at home. It is unwise to send the patient home without making arrangements first. Go to the next page if you knew the correct answer, or click the link image(s) below to further research the concepts in this question (if desired).

Research Concepts: Termination of Life Support

We update eBooks quarterly and Apps daily based on user feedback. Please tap flag to report any questions that need improvement.

Question 938: A 29-year-old woman with a history of generalized anxiety disorder presents to the hospital with acute onset of shortness of breath. Labs are unremarkable with a creatinine of 0.8 mg/dL. She is started on enoxaparin for VTE prophylaxis while she is evaluated for the cause of her symptoms. If she begins to bleed excessively, what is the mechanism of action of the most appropriate reversal agent used for enoxaparin?

Choices: 1. Humanized monoclonal antibody 2. Competitive binding to factor Xa 3. A cationic peptide that binds to low molecular weight heparin forming an ionic complex to neutralize the effect 4. Administration of an essential cofactor for the gamma-carboxylase enzymes

Answer: 3 - A cationic peptide that binds to low molecular weight heparin forming an ionic complex to neutralize the effect

Explanations: Protamine sulfate is a cationic peptide that binds to low molecular weight heparin, thereby forming an ionic complex with no anticoagulant activity. Protamine neutralizes about 60% of low molecular weight heparin anticoagulant activity. The recommended dose is 1 mg protamine sulfate per 1 mg of enoxaparin or 100 antiFactor-Xa units of dalteparin. The use of protamine sulfate correlates with fewer bleeding complications post percutaneous coronary intervention (PCI). Go to the next page if you knew the correct answer, or click the link image(s) below to further research the concepts in this question (if desired).

Research Concepts: Enoxaparin

We update eBooks quarterly and Apps daily based on user feedback. Please tap flag to report any questions that need improvement.

Question 939: A 68-year-old man with severe lion pain is found to have calculi in his right ureter. On further evaluation, he is found to have hydronephrosis and acute kidney injury. He undergoes percutaneous nephrostomy tube placement by to relieve his obstruction. Immediately afterward, he suffers septic shock. He is started on broad-spectrum antibiotics covering gramnegative bacteria after a culture is sent. He is started on norepinephrine and vasopressin at maximal doses. However, he still remains in shock and consideration for angiotensin II is made. Which of the following is the most appropriate intervention before starting angiotensin II therapy in this patient?

Choices: 1. Serum lactate level 2. Serum prolactin level 3. Serum troponin level 4. IV 30 mL/kg normal saline

Answer: 4 - IV 30 mL/kg normal saline Explanations: In the ATHOS-3 trial, all patients who were enrolled and received angiotensin II were adequately resuscitated, with CVP of 8 to 12 cmH2O, and had a central venous catheter and arterial line for monitoring. Starting angiotensin II without adequately resuscitating a patient does not make sense physiologically, as well as you don't want to vasoconstrict without intravascular volume repletion. The only guidance for this medication that we have in our clinical practice is from the clinical trial ATHOS-3. Thus, to ensure the best result, if there is no contraindication, the conditions of the trial should be replicated. Obtaining lactic acid, procalcitonin or troponin is not a prerequisite for starting any vasopressor, let alone angiotensin II. Go to the next page if you knew the correct answer, or click the link image(s) below to further research the concepts in this question (if desired).

Research Concepts: Angiotensin II

We update eBooks quarterly and Apps daily based on user feedback. Please tap flag to report any questions that need improvement.

Question 940: A 67-year-old man with a past medical history of hypertension, alcohol use, depression, gout, and benign prostatic hyperplasia is admitted to the intensive care unit with a grade 3 subarachnoid hemorrhage (Modified Fischer scale). A cerebral aneurysm is identified on angiography, and endovascular coiling is done to reduce the risk of further bleeding. 7 days posttreatment of the aneurysm, the patient starts complaining of nausea and headache. CT imaging of the brain does not show new findings. There is no dizziness on standing. Capillary refill time is normal. The patient's vitals show no new increase in the heart rate or fall of blood pressure. A current set of labs is shown below. Patient value Reference range Sodium 130 mEq/L 134-144 mEq/L Potassium 4.2 mEq/L 3.6-5.0 mEq/L Chloride 100 mEq/L 98-107 mEq/L Bicarbonate 24 mEq/L 21-28 mEq/L Which of the following is the best next step in the management of this patient?

Choices: 1. 3% sodium chloride solution 2. 0.9% sodium chloride solution 3. 0.9% sodium chloride with 5% dextrose solution 4. Total fluid restriction

Answer: 1 - 3% sodium chloride solution Explanations: This scenario describes a case of hyponatremia following subarachnoid hemorrhage (SAH) due to increased secretion of anti-diuretic hormone (ADH). Hyponatremia following SAH is either due to increased secretion of ADH (majority) or due to cerebral salt wasting. If the hyponatremia is due to increased ADH, the patient is euvolemic. If the hyponatremia is due to cerebral salt wasting, there is clear evidence of hypovolemia. Hyponatremia, here, is best treated with hypertonic saline (3% sodium chloride). Some clinicians prefer isotonic normal saline (0.9% sodium chloride) if they feel hyponatremia is due to CSW alone. Fluid restriction leads to hypovolemia, which aggravates the risk of cerebral vasospasm. Fluid restriction should be avoided. Go to the next page if you knew the correct answer, or click the link image(s) below to further research the concepts in this question (if desired).

Research Concepts: Subarachnoid Hemorrhage

We update eBooks quarterly and Apps daily based on user feedback. Please tap flag to report any questions that need improvement.

Question 941: A patient presents with a blood pressure of 70/40 mmHg, a pulse of 135 bpm, a pulmonary capillary wedge pressure of 27 mmHg, a CVP of 11 mmHg, and a systemic vascular resistance of 2000 dynes sec cm. The hemodynamic profile of the patient suggests which of the following shock states?

Choices: 1. Septic shock 2. Hypovolemic shock 3. Neurogenic shock 4. Cardiogenic shock

Answer: 4 - Cardiogenic shock Explanations: Invasive monitoring is useful in helping to rule out other causes of shock. Cardiogenic shock usually has an elevated wedge pressure and a cardiac index of less than 2.2 liters/min. The presence of large V waves on the wedge pressure tracing indicates the presence of severe mitral regurgitation. A step up of oxygen saturation between the right atrium and right ventricle is diagnostic of a ventricular septal rupture. Go to the next page if you knew the correct answer, or click the link image(s) below to further research the concepts in this question (if desired).

Research Concepts: Cardiogenic Shock

We update eBooks quarterly and Apps daily based on user feedback. Please tap flag to report any questions that need improvement.

Question 942: A 26-year-old previously well male with no comorbidities is admitted to the intensive care unit following a traumatic head injury. He is currently on day 4 post endotracheal intubation and initiation of mechanical ventilation. He has been receiving enteral feeds directly into the small intestine. He is now noted to have a temperature of 39C, a spontaneous respiratory rate of 30/minute, a blood pressure of 96/64 mmHg, a heart rate of 122 beats/minute, oxygen saturation of 92% in room air and finger-prick blood glucose of 6 mmol/L. A complete blood count reveals a hemoglobin of 15.1g/dL and a white blood cell count of 15 000 cells/mm3. Purulent secretions are noted to be present in the endotracheal tube. He has been receiving enteral feeds directly into the small intestine. The culture result of an endotracheal aspirate specimen is pending. He is commenced on empiric antimicrobial therapy and intravascular fluid volume expansion is escalated. Which of the following represents the most likely mechanism of introduction of the micro-organisms typically implicated in the etiology of the patient’s condition?

Choices: 1. Ileus 2. Urinary catheterization 3. Prone positioning 4. Sedation break

Answer: 1 - Ileus Explanations: The patient in this scenario has a likely diagnosis of ventilator-associated pneumonia (VAP). VAP is defined as the development of pneumonia more than 48 hours after the initiation of mechanical ventilation. The diagnosis of ventilator-associated pneumonia (VAP) is based on a positive semiquantitative culture result from a specimen obtained via a non-invasive sampling technique (i.e. endotracheal aspiration) and the presence of clinical criteria (such as pyrexia and leucocytosis [white blood cell count > 11 000 cells/mm3]), as per the American Thoracic Society and Infectious Diseases Society of America guideline for the management of VAP, jointly published in 2016. Aerobic gram-negative bacteria (AGNB) are typically implicated in the etiology of VAP. According to the gastropulmonary hypothesis, gastric colonization with AGNB occurs, followed by colonization and infection of the respiratory tract via gastric reflux and aspiration, which may manifest as ventilator-associated pneumonia. As per the hypothesis, AGNB are introduced into the stomach exogenously (for example, through contaminated nasogastric tube feeds) or endogenously (via reflux of intestinal content). Ileus enables the reflux of intestinal content into the stomach. The presence of bile acids in the refluxed intestinal content causes an increase in gastric pH, which promotes aerobic Gram-negative bacterial colonization and growth in the stomach. The pH of enteric feeds is between 6 and 7. Although not a part of routine, standard practice, enteral feeds may be administered directly into the small intestine to prevent the effect of an increased gastric pH from enteral feeds. An increased gastric pH promotes aerobic Gram-negative bacterial colonization and growth in the stomach in the pathogenesis of ventilator-associated pneumonia. Go to the next page if you knew the correct answer, or click the link image(s) below to further research the concepts in this question (if desired).

Research Concepts: Ventilator Complications

We update eBooks quarterly and Apps daily based on user feedback. Please tap flag to report any questions that need improvement.

Question 943: Which of the following adjustment should be made to the regimen of an elderly white female with history of congestive heart failure with an ejection fraction of 30% on an ACE inhibitor, a beta-blocker, and a loop diuretic with a well controlled blood pressure but who is experiencing orthopnea and is found to have pulmonary rales and pitting edema peripherally?

Choices: 1. Start warfarin 2. Start nitrates 3. Stop beta-blockers 4. Start spironolactone

Answer: 4 - Start spironolactone Explanations: Spironolactone is beneficial for patients with Class III (limitation in normal activities) or class IV CHF (dyspnea at rest) and an ejection fraction 35%. Beta-blockers have been shown to decrease mortality and are indicated for all stable CHF patients but are not indicated in acute congestive heart failure. Warfarin is indicated in patients with atrial fibrillation that are at a high risk of developing a stroke. Nitrates can help in an acute congestive heart failure exacerbation by decreasing afterload and allowing the heart to pump against less resistance. Go to the next page if you knew the correct answer, or click the link image(s) below to further research the concepts in this question (if desired).

Research Concepts: Congestive Heart Failure

We update eBooks quarterly and Apps daily based on user feedback. Please tap flag to report any questions that need improvement.

Question 944: A 60-year-old woman with a history of leukemia is brought to the emergency department with altered mentation. On physical examination, her blood pressure is 80/40 mmHg, her pulse rate is 110/min, and she has a low-grade temperature. She is disoriented and lethargic. She has recently been prescribed sunitinib for leukemia treatment. An initial set of investigations is shown below. Reference Patient results range Hemoglobin 9 g/dL 11.5-16 g/dL 4100WBC count 9840/microL 10900/microL 150000Platelets 297000/microL 400000/microL Creatinine 1.09 mg/dL 0.8-1.4 mg/dL 134-144 Sodium 126 mEq/L mEq/L Potassium 5.2 mEq/L 3.6-5.0 mEq/L Chloride 93 mEq/L 98-107 mEq/L Bicarbonate 21 mEq/L 21-28 mEq/L Urinalysis showed cloudy urine, with a trace of blood, increased WBC, and leukocyte esterase consistent with a urinary tract infection. A chest radiography, CT of the abdomen and pelvis, and CT brain do not reveal any abnormalities. Blood and urine cultures are sent. The patient is given 3 liters of intravenous fluid resuscitation along with broad-spectrum antibiotics. However, her hypotension persists, so vasopressors are then initiated. Which of the following cause is contributing to the persistent hypotension?

Choices: 1. Severe sepsis from urinary tract infection 2. Sunitinib induced pituitary failure 3. Sunitinib induced adrenal insufficiency 4. Anemia

Answer: 3 - Sunitinib induced adrenal insufficiency Explanations: The patient has an adrenal crisis precipitated by sunitinib use. Patients on tyrosine kinase inhibitors should be educated that they can precipitate adrenal insufficiency. The patient has hypotension with tachycardia, hyponatremia, anemia, and hyperkalemia with confusion, confirming the diagnosis of adrenal crisis. Emergency treatment of suspected adrenal crisis is immediate administration of iv hydrocortisone, aggressive IV fluid resuscitation, and appropriate antibiotics. The dose of intravenous hydrocortisone should be 100 mg (stress dose ) intravenously or intramuscularly (IV/IM) as an initial bolus followed by 100 to 300 mg daily after that for another 2-3 days of boluses every 6 hours or as continuous infusion until complete recovery. These individuals also require aggressive fluid and vasopressors. A thorough search should be done for the cause, and empiric antibiotics are recommended. Close monitoring in the intensive care unit is required. At discharge, the patient should be given a medical bracelet that alerts medical personnel about adrenal insufficiency. Adrenal crises can lead to severe refractory shock and death if not promptly recognized. Even though the patient has severe sepsis from urinary tract infection, treating underlying sunitinib-induced adrenal insufficiency with intravenous steroids, along with IV fluid resuscitation, and antibiotics will help improve hypotension. Sunitinib does not cause pituitary failure. Go to the next page if you knew the correct answer, or click the link image(s) below to further research the concepts in this question (if desired).

Research Concepts: Adrenal Crisis

We update eBooks quarterly and Apps daily based on user feedback. Please tap flag to report any questions that need improvement.

Question 945: A 65-year-old patient with a left-sided malignant middle cerebral artery infarction was being managed in the neurology intensive care unit (ICU) by placing an intraventricular ICP monitoring catheter. The neurosurgeon was asked for consultation following the appearance of ICP waves rising 50 mm (normal value of 5-15 mm of Hg) above baseline and lasting for 5 to 20 minutes. Which of the following is the most appropriate next management plan for the patient?

Choices: 1. keeping the patient in the ventilator and sedated 2. Asking for mannitol infusion 3. Hypothermia 4. Decompressive hemicraniectomy

Answer: 4 - Decompressive hemicraniectomy Explanations: The given clinical scenario is typical of a raised ICP following malignant middle cerebral artery infarction owing to the presence of Lundberg A waves in the ICP waveform. Decompressive hemicraniectomy (DHC) is advisable for the life-saving procedures for the same. Timely performed DHC with durotomy allows for the immediate release of the raised ICP and is effective in reducing mortality as well as high dependency among these patients compared to medical management. Other tiers of medical management such as barbiturate coma, mannitol infusion, hypothermia have not been validated for reducing mortality compared to surgical management. Go to the next page if you knew the correct answer, or click the link image(s) below to further research the concepts in this question (if desired).

Research Concepts: Intracranial Pressure Monitoring

We update eBooks quarterly and Apps daily based on user feedback. Please tap flag to report any questions that need improvement.

Question 946: A 46-year-old man presents for an elective liver transplant. He has a past medical history of end-stage chronic kidney disease. Blood tests show creatinine 3.4 mg/dL (0.81.3), bilirubin 7.3 mg/dL (0.1-1.2), international nationalized ratio (INR) 2.3 (1.1 or below), and sodium 125 mEq/L (135-145). A non-depolarizing neuromuscular blocking drug is administered to facilitate endotracheal intubation and provide skeletal muscle relaxation during surgery. How is this drug metabolized?

Choices: 1. Non-enzymatic Hoffman elimination 2. Diffusion from the neuromuscular junction 3. Redistribution into adipose tissue 4. Via cholinesterases located in the pulmonary endothelium

Answer: 1 - Non-enzymatic Hoffman elimination Explanations: Atracurium is a neuromuscular blocking agent of the benzylisoquinolinium class. It undergoes Hoffman's elimination. This is a nonenzymatic process that is independent of renal or hepatic function. Hoffman elimination is a temperature and pH-dependent process that can be slowed by hypothermia and acidosis. Neuromuscular blocking agents of the Benzylisoquinolinium class are preferred in critically ill patients as their metabolism is not affected by renal or hepatic dysfunction. Go to the next page if you knew the correct answer, or click the link image(s) below to further research the concepts in this question (if desired).

Research Concepts: Atracurium

We update eBooks quarterly and Apps daily based on user feedback. Please tap flag to report any questions that need improvement.

Question 947: A 34-year-old woman with a history of epilepsy is brought to the emergency department after several generalized seizures. There is no history of trauma or any sign of injury. Convulsive movements stopped after benzodiazepine administration, but she remains unresponsive 30 minutes after apparent seizure cessation. Which of the following is the next best step in the management of this patient?

Choices: 1. CT head 2. MRI head 3. Cranial angiography 4. EEG

Answer: 4 - EEG Explanations: Persistent unresponsiveness after generalized convulsive status epilepticus may indicate the transformation of the seizures to a type of nonconvulsive generalized status epilepticus. The highly abnormal electrical activity of the brain continues but may no longer be reflected with motor movements. An EEG is necessary to detect continuing seizures. Generalized convulsive status epilepticus may evolve or change into a state with persistent coma or unresponsiveness without convulsions. Terms used to describe this include subtle status epilepticus or transformed status epilepticus. This is a continuing emergency and should be detected by EEG and treated. Continuous EEG monitoring is becoming used more frequently in ICU settings. Unexpected seizures on EEG are detected surprisingly often. CT or MRI are commonly performed after stabilization of patients with status epilepticus, but for patients with a history of epilepsy, the yield of new findings with neuroimaging is low. Go to the next page if you knew the correct answer, or click the link image(s) below to further research the concepts in this question (if desired).

Research Concepts: Coma

We update eBooks quarterly and Apps daily based on user feedback. Please tap flag to report any questions that need improvement.

Question 948: A 20-year-old male missionary worker returned from a mission in Indonesia one week ago. Upon return, he started to have intermittent fever, headache, and malaise. He took over-the-counter acetaminophen 300 mg every 6 hours daily. His symptoms worsened over five days. In the emergency department, he has a high-grade fever, hypotension, and somnolence with brisk deep tendon reflexes and hypertonia. CBC reveals severe anemia and thrombocytopenia. A malaria Rapid Diagnostic Test (RTD) is negative. Lumbar puncture is unremarkable. He then develops a sudden onset of limb jerking for approximately 60 seconds and goes unresponsive afterward. MRI brain of this patient would most likely reveal which of the following?

Choices: 1. Ring enhancing lesion in the temporal lobe 2. Significantly increased brain volume 3. Elliptical collection of subdural fluid 4. Bilateral ventriculomegaly

Answer: 2 - Significantly increased brain volume Explanations: In a patient returning from a malaria-endemic region with intermittent fever and neurologic symptoms, cerebral malaria should be high on the differential. In patients with suspected cerebral malaria, meningitis must be ruled out with a lumbar puncture. Malaria RTDs currently can only detect P. falciparum. In a patient returning from Southeast Asia, P. vivax infection is likely and will not be detected with RTD. MRI brain in patients with cerebral malaria will likely reveal significantly increased brain volume due to vascular congestion, impaired perfusion, endothelial activation, blood-brain barrier changes, and cerebral edema. Go to the next page if you knew the correct answer, or click the link image(s) below to further research the concepts in this question (if desired).

Research Concepts: Malaria

We update eBooks quarterly and Apps daily based on user feedback. Please tap flag to report any questions that need improvement.

Question 949: A 65-year-old man is brought to the emergency department with complaints of new-onset confusion, hyperventilation, and vomiting that started five hours ago in a nursing home. The patient has a past medical history of mild dementia, high blood pressure, and chronic back pain. His medications include lisinopril, aspirin, amlodipine, and ginseng. The nurse accompanying the patient is unsure if he has been compliant with his medication. Currently, the patient is afebrile and has a blood pressure of 145/90 mm Hg. Physical examination reveals a patient who is not oriented to time, place, or person. There is no sign of dehydration on the exam. Arterial blood gas results include a pH of 7.41, Pa CO2 of 24 mm Hg, bicarbonate level of 15 mEq/L, sodium level of 143 mEq/L, chloride levels of 100 mEq/L and PaO2 of 98 mm Hg. What is the best next step in the management of this patient's condition?

Choices: 1. Measurement of serum thiamine 2. Measurement of serum ethylene glycol 3. Measurement of serum salicylate levels 4. Measurement of serum ketones

Answer: 3 - Measurement of serum salicylate levels Explanations: The drug intoxication most likely cause of mixed acid-base disturbance of respiratory alkalosis, and anion gap metabolic acidosis is a salicylate (aspirin). Aspirin (salicylate) directly stimulates the medullary respiratory center to cause tachypnea and respiratory alkalosis. In addition, aspirin causes an anion gap metabolic acidosis due to increased production and decreased renal elimination of organic acids (eg, lactic acid, ketoacids). Although thiamine deficiency can cause elevation of lactate levels in the setting of beriberi, it does not cause respiratory alkalosis. Serum osmolality should be measured in suspected cases with anion gap acidosis, given ingestion of certain substances such as ethylene glycol or methanol can cause metabolic acidosis and an osmolal gap. However, these substances do not cause respiratory alkalosis and would not explain the mixed acid-base disturbance, as in this case. Go to the next page if you knew the correct answer, or click the link image(s) below to further research the concepts in this question (if desired).

Research Concepts: High Anion Gap Metabolic Acidosis

We update eBooks quarterly and Apps daily based on user feedback. Please tap flag to report any questions that need improvement.

Question 950: A patient undergoes abdominal surgery for a splenic rupture and receives many units of transfused blood. Post-surgery he complains of numbness around his mouth and a carpopedal spasm. An electrocardiogram reveals a prolonged QT interval. Which of the following would be the best intravenous treatment for this patient?

Choices: 1. Bicarbonate 2. Potassium 3. Calcium 4. D50

Answer: 3 - Calcium Explanations: Chvostek sign occurs when tapping the skin over the facial nerve causes ipsilateral contraction of facial muscles. This is a suggestive sign of hypocalcemia. Trousseau's sign occurs when the blood pressure cuff is inflated on the arm and causes flexion of the wrist and extension of interphalangeal joints. The thumb usually adducts. Trousseau's sign is more specific for hypocalcemia. Other abnormalities that may occur in hypocalcemia include athetosis, dystonic spasm, Parkinson disease, and hemiballismus. Coarse hair, brittle nails, and dry skin are other features of long-standing hypocalcemia. Go to the next page if you knew the correct answer, or click the link image(s) below to further research the concepts in this question (if desired).

Research Concepts: Hypocalcemia

We update eBooks quarterly and Apps daily based on user feedback. Please tap flag to report any questions that need improvement.

Question 951: A 72-year-old man is brought to the hospital with confusion. He has normal vital signs except for a respiratory rate of 32/min. Initial labs show 136 mEq/L, potassium 2.6 mEq/L, bicarbonate 14 mEq/L, chloride 112 mEq/L, BUN 17 mg/dL, creatinine 0.9 mg/dL, glucose 89 mg/dL, calcium 9.8 mg/dL, and urine pH 7.6. Arterial blood gas shows pO2 91 mmHg, pCO2 30 mmHg, and pH 7.26. Which of the following tests is most likely to reveal the underlying diagnosis in this patient?

Choices: 1. Acid load test 2. Bicarbonate infusion test 3. Serum aldosterone level 4. Urine culture

Answer: 1 - Acid load test Explanations: The patient has a hyperchloremic non-anion gap metabolic acidosis with appropriate respiratory compensation. It can be caused by gastrointestinal bicarbonate losses from diarrhea, proximal renal tubular acidosis (RTA), hypokalemic distal RTA, type I, and hyperkalemic distal RTA, type IV. The fact that the patient has an elevated urine pH in the setting of acidosis points to a distal RTA because the normal response would be a lowering of urine pH in the setting of metabolic acidosis. The acid load test is the specific test for infusing acid into the blood with 100 mg/kg of ammonium chloride and checks urine pH hourly and plasma bicarbonate at a 3-hour interval. A healthy person will be able to excrete acid and will decrease urine pH. Those with distal RTA cannot excrete acid, and urine pH will remain basic despite increasingly acidic serum. Plasma bicarbonate should drop below 21 mEq/L unless the patient vomits (in which case the test should be repeated with antiemetic). If urine pH remains greater than 5.5 despite plasma bicarbonate of 21 mEq/L, the diagnosis of type 1 RTA is confirmed. Treatment for distal renal tubular acidosis is with oral sodium bicarbonate, sodium citrate, or potassium citrate. Go to the next page if you knew the correct answer, or click the link image(s) below to further research the concepts in this question (if desired).

Research Concepts: Renal Tubular Acidosis

We update eBooks quarterly and Apps daily based on user feedback. Please tap flag to report any questions that need improvement.

Question 952: A 65-year-old female patient presents with chest pain. The pain is described as tearing and radiates to her back. Her blood pressure is 160/110 mmHg, her pulse rate is 92 beats/min, and her temperature is 98.6 F (37 C). She is suffering from hypertension, dyslipidemia, obesity, and she has a 40 pack-years smoking history. On examination, the right lower limb has all pulses palpable; however, on the left lower limb, there is a weak femoral pulse, and all other pulses are absent. CT scan shows the displacement of intimal calcification. What is the most common site of tears in this condition in general?

Choices: 1. Just distal to the left subclavian artery 2. Within a few centimeters of the proximal ascending aorta 3. At the proximal aortic arch 4. Distal to the descending aorta

Answer: 2 - Within a few centimeters of the proximal ascending aorta Explanations: Aortic dissection is due to the separation of the layers of the aortic wall. A tear in the intimal layer results in the progression of the dissection (either proximal or retrograde) chiefly due to the entry of blood between the intima and media. Cystic medial degeneration plays a role in the pathology of aneurysmal formation and dissection in the ascending aorta. Type 1 dissection involves the entire aorta, type 2 involves only the ascending aorta, and type 3 involves the descending aorta. Other factors related to dissection in the ascending aorta are hypertension, cannulation during cardiopulmonary bypass, Marfan syndrome, pregnancy, retrograde dissection, and a bicuspid aortic valve. The majority of dissections (95%) occur in two places. The most common site is within several centimeters of the atrioventricular valve, and the second most common is just beyond the ligamentum arteriosum in the descending aorta. The tear is usually transverse and can progress in either a retrograde or an antegrade fashion. In the patient in the scenario, the dissection is distal to these two sites, most likely in the distal aorta. Go to the next page if you knew the correct answer, or click the link image(s) below to further research the concepts in this question (if desired).

Research Concepts: Aortic Dissection

We update eBooks quarterly and Apps daily based on user feedback. Please tap flag to report any questions that need improvement.

Question 953: A 37-year-old woman who is 25 weeks pregnant (gravida 4, para 3) presents to the emergency department with a two-hour history of vomiting. Her husband states that she has never experienced such symptoms before. A clinical evaluation is significant for a blood pressure of 160/110 mmHg and bilateral lower extremity edema. Laboratory investigations are normal except for urinalysis (proteinuria 3+). The patient is now being treated and is administered digoxin immune Fab. A few hours later, the patient started to clinically improve with a reduction in blood pressure and severity of abdominal pain. Which of the following possible mechanisms best explains the clinical utility of digoxin immune Fab in this patient?

Choices: 1. Upregulates tumor necrosis factor-alpha release 2. Binds to placental sodium-potassium ATPase pumps 3. Binds to vascular endothelial surface adhesion molecules 4. Binds endogenous compounds such as bufadienolide

Answer: 4 - Binds endogenous compounds such as bufadienolide Explanations: Lower extremity edema, elevated blood pressure, vomiting, and proteinuria in a pregnant female are definitive clinical signs of pre-eclampsia. Pre-eclampsia is known to occur due to incomplete trophoblastic invasion by the placenta, and it is considered an exaggerated inflammatory condition compounded by endothelial dysfunction. Several other biologically active factors are also implicated in its pathogenesis. One such factor found in the maternal serum of pre-eclamptic patients is the endogenous digoxin-like factors (EDLFs). The endogenous digoxin-like factor binds to and inhibits mesenteric artery Na/K ATPase pumps, resulting in the accumulation of intracellular calcium leading to vasoconstriction and maternal hypertension. These factors are also known to preserve maternal renal function, lower maternal pulmonary edema, and the risk of intraventricular hemorrhage in infants. Examples of EDLFs include cardenolide, ouabain, bufadienolide, and marinobufagenin. Digoxin immune Fab are anti-digoxin antibodies that can bind and neutralize endogenous digoxin-like factors. More specifically, it reverses the Na/K ATPase pump inhibition caused by EDLFs, resulting in an improvement in maternal symptoms and hypertension. Digoxin immune Fab also reduces TNF-alpha-induced endothelial surface adhesion molecules such as ICAM, VCAM, and E-selectin. It also inhibits TNF-alpha mediated downregulation of Na/K ATPase pumps. Go to the next page if you knew the correct answer, or click the link image(s) below to further research the concepts in this question (if desired).

Research Concepts: Digoxin Immune Fab

We update eBooks quarterly and Apps daily based on user feedback. Please tap flag to report any questions that need improvement.

Question 954: A 42-year-old female presents to the emergency department with breathlessness and an altered state. Her blood pressure is 90/58 mmHg, heart rate 121/min, respiratory rate 33/min, temperature 99 F, O2 saturation of 67% on room air. She is started on norepinephrine, intubated, and taken to the intensive care unit. She has a past medical history of diabetes mellitus type 2, which is well controlled on metformin. Complete blood count and basic metabolic panel are unremarkable. Chest X-ray shows cardiac silhouette within normal limits, bilateral reticular, and alveolar infiltrates. Arterial blood gases are ordered, which demonstrates pH 7.3, paCo2 55 mmHg, PaO2 90 mmHg while on FiO2 of 60%. What is the most appropriate treatment?

Choices: 1. Tidal volume 10 ml/kg of predicted body weight, plateau pressure >40 cm H20 2. Tidal volume 12 ml/kg of predicted body weight, plateau pressure >30 cm H20 3. Tidal volume 6 ml/kg of predicted body weight, plateau pressure 30 cm H20 4. Tidal volume 2 ml/kg of predicted body weight, plateau pressure 30 cm H20

Answer: 3 - Tidal volume 6 ml/kg of predicted body weight, plateau pressure 30 cm H20 Explanations: The patient has acute respiratory distress syndrome based on chest X-ray with bilateral infiltrates present and PaO2/FiO2 ratio less than 300 based on the Berlin criteria. Acute respiratory distress syndrome treatment is based on low tidal volume, approximately 6 ml/kg of predicted body weight, and a goal of maintaining plateau pressure 30 cm H20. Acute respiratory distress syndrome can be diagnosed as mild, moderate, severe based on the PaO2/FiO2 ratio. A ratio of less than 100 is severe, 100-200 is moderate, 200-300 is mild. In addition to identifying acute respiratory distress syndrome quickly and adjusting the vent appropriately to improve mortality, these patients also benefit from early proning. Go to the next page if you knew the correct answer, or click the link image(s) below to further research the concepts in this question (if desired).

Research Concepts: Fraction of Inspired Oxygen

We update eBooks quarterly and Apps daily based on user feedback. Please tap flag to report any questions that need improvement.

Question 955: A 35-year-old man with a history of obesity is undergoing an elective herniorrhaphy. Anesthesia is successfully induced. Right after giving a muscle relaxant, positive pressure ventilation by face mask fails to provide an adequate seal. His oxygen saturation of the patient begins to fall. What is the next best step in the management of this patient?

Choices: 1. Ask for a large plastic adhesive, cut a hole into it and place over the patient's mouth in an attempt to make a seal 2. Rush to intubation as mask ventilation is failing 3. Attempt to find a larger mask in order to make a better seal 4. Place a second-generation laryngeal mask airway (LMA)

Answer: 4 - Place a second-generation laryngeal mask airway (LMA) Explanations: After induction of anesthesia, including a muscle relaxant, maintaining pre-oxygenation and some ventilation is of paramount importance to avoid hypoxia. Especially in larger individuals, redundant tissue may make positive pressure ventilation difficult. Once the face mask fails to provide an adequate seal, an LMA is a good choice to provide a seal further down the airway. The LMA is designed to create a seal around the larynx in order to allow positive pressure ventilation up to a certain pressure, which can vary between patients and the design of the LMA. Once the patient's saturation begins to fall, swift and decisive action such as inserting an LMA will help to prevent further hypoxia. In addition to bypassing facial features, missing teeth, and full beards, LMAs can also be useful to slide past redundant pharyngeal tissues which might obstruct the upper airway. LMA's have different designs and features, such as silicone cuffs, gastric access ports, noninflatable gel cuffs. 2nd generation LMAs often provide higher seal pressure to allow for higher positive pressure ventilation pressures and are preferred in a rescue situation. LMA's, preferably 2nd generation, should be immediately available during induction of anesthesia if airway rescue becomes necessary. Go to the next page if you knew the correct answer, or click the link image(s) below to further research the concepts in this question (if desired).

Research Concepts: Laryngeal Mask Airway

We update eBooks quarterly and Apps daily based on user feedback. Please tap flag to report any questions that need improvement.

Question 956: A 50-year-old female presents with daily generalized headaches for a month. She reports the headaches are worst at night and in the morning before work. She recently separated from her husband of 25 years and moved into an older house. She admits she has been depressed and was started on an SSRI after her pet cat died in the basement. Her neurologist gave her sumatriptan for the headaches. The patient is brought to the emergency department by paramedics, unresponsive and flushed. What is the diagnosis?

Choices: 1. Serotonin syndrome 2. Lead poisoning 3. Carbon monoxide poisoning 4. Drug overdose

Answer: 3 - Carbon monoxide poisoning Explanations: Carbon monoxide from an old furnace could cause this scenario. The cat died of chronic exposure, while the patient had headaches while at home. Carbon monoxide binds to hemoglobin causing progressive cerebral anoxia. Other symptoms associated with carbon monoxide are dizziness and flu-like symptoms while patients with poisoning have flushing and pink skin. Go to the next page if you knew the correct answer, or click the link image(s) below to further research the concepts in this question (if desired).

Research Concepts: Carbon Monoxide Toxicity

We update eBooks quarterly and Apps daily based on user feedback. Please tap flag to report any questions that need improvement.

Question 957: A 56-year-old male farmer presents to the emergency department with a chief complaint of chest pain, shortness of breath, and generalized fatigue. He reports that he was feeding the animals when he had a sudden onset of symptoms. On exam, the patient is awake, alert with mild respiratory distress. His vital signs are heart rate 110/min, blood pressure 127/63 mmHg, SpO2 84% on 4L nasal cannula, and respiratory rate 12/min. The nurse reports his blood appears darker than normal. Which testing modality is most likely to reveal the diagnosis?

Choices: 1. Co-oximetry 2. Arterial blood gas 3. Venous blood gas 4. Chest x-ray

Answer: 1 - Co-oximetry Explanations: This patient has induced methemoglobinemia as a result of exposure to nitrogen dioxide. The patient will have apparent hypoxemia, but ABG will show a normal partial pressure of oxygen. Co-oximetry will show a methemoglobin level greater than 15%. Chocolate brown blood is a hallmark of methemoglobinemia. Go to the next page if you knew the correct answer, or click the link image(s) below to further research the concepts in this question (if desired).

Research Concepts: Nitrogen Dioxide Toxicity

We update eBooks quarterly and Apps daily based on user feedback. Please tap flag to report any questions that need improvement.

Question 958: A 61-year-old woman with a history of hypertension, type 2 diabetes, paroxysmal atrial fibrillation, and obesity is brought to the hospital with severe 9/10 diffuse abdominal pain that started suddenly 2 hours ago. She denies diarrhea/constipation. Her last bowel movement was normal yesterday. Her pulse is irregular 108-115/min, blood pressure 135/70 mmHg, and respiratory rate 22/min. On physical exam, the abdomen is soft with mild diffuse tenderness with no signs of peritonitis. WBC count is 12000/microL, hemoglobin 9.5 g/dL, lactate 2.2 mg/dL, and creatinine 2.4 mg/dL. Non-contrast abdominal CT scan shows mild small bowel loops distention and thickening. She has never had abdominal surgery. The last colonoscopy 5 years ago was normal. Which of the following is the next best step in the management of this patient?

Choices: 1. Exploratory laparotomy 2. Diagnostic laparoscopy 3. Colonoscopy 4. CT angiography

Answer: 4 - CT angiography Explanations: This patient most likely has mesenteric ischemia. The most common presenting symptom is abdominal pain. The patient’s symptoms and presentation may help determine the etiology of the ischemia. An arterial embolism usually causes sudden, severe, periumbilical pain and is associated with nausea and vomiting. Thrombotic mesenteric arterial occlusion is usually associated with pain that is worse after eating. Patients with mesenteric venous thrombosis usually have slower-onset, “waxing and waning” abdominal pain. The physical exam may be normal. In fact, patients present with pain that is out of proportion to the initial physical exam. There may be mild distension present, but peritoneal signs only begin to show when transmural bowel infarction and necrosis develops. CT angiography is the best non-invasive test for suspected mesenteric ischemia. Exploratory laparotomy is indicated for peritonitis or bowel gangrene. Diagnostic laparoscopy is reserved for patients with unclear diagnoses once all noninvasive tests are done and still inconclusive. Go to the next page if you knew the correct answer, or click the link image(s) below to further research the concepts in this question (if desired).

Research Concepts: Mesenteric Artery Ischemia

We update eBooks quarterly and Apps daily based on user feedback. Please tap flag to report any questions that need improvement.

Question 959: A 65-year-old female with a lung malignancy presents to the emergency department from her oncologist for a thoracentesis. She has shortness of breath and cough. Her temperature is 101.4 F, blood pressure is 92/60 mmHg, respiratory rate is 28 breaths/min, and her heart rate is 122/min. On physical examination, there are decreased breath sounds at the right lung base. An ultrasound is performed, confirming a right-sided pleural effusion. While performing thoracentesis, the patient's respiratory rate increases to 40/min. Which of the following clinical course is expected during the procedure?

Choices: 1. The patient collapses and immediate resuscitation is needed. 2. The patient begins to improve clinically, oxygen saturation is unchanged, and he complains of an abnormal sensation in the chest. 3. The patient begins to cough, oxygen saturation decreases, and he complains of a strange sensation in the thoracic cavity as lung re-expands. 4. The patient remains asymptomatic and oxygen saturation remains unchanged.

Answer: 3 - The patient begins to cough, oxygen saturation decreases, and he complains of a strange sensation in the thoracic cavity as lung re-expands.

Explanations: As lungs reexpand, patients often will begin coughing and complain of a strange sensation in the chest. Oxygen saturation often decreases acutely secondary to ventilation-perfusion mismatch. Blood is shunted away from the collapsed lung, following the procedure, the lung will have increase ventilation but takes time for perfusion to occur. Lung re-expansion is often uncomfortable for patients. Imaging eliminates the concern for damage to nearby organs or other causes of chest pain. Go to the next page if you knew the correct answer, or click the link image(s) below to further research the concepts in this question (if desired).

Research Concepts: Thoracentesis

We update eBooks quarterly and Apps daily based on user feedback. Please tap flag to report any questions that need improvement.

Question 960: A 67-year-old man is with stage 3 prostate cancer is referred for cerebral spinal fluid (CSF) testing. Previous investigations identified a lesion on the lower cervical cord at C6 to C7 and multiple lumbar lesions from L2 to L5. The lesions seem intradural, and because the primary tumor was prostate, diagnostic CSF has been requested. A lumbar puncture is contraindicated due to the tumor invasion. Where is the best place to obtain a CSF sample in this patient?

Choices: 1. Above the arch of C1 2. Between C2 and C3 3. Upper thoracic area, between T1 and T2 4. Lower thoracic area, between T11 and T12

Answer: 1 - Above the arch of C1 Explanations: Suboccipital puncture is performed between occiput and C1 with a midline or lateral approach. Suboccipital puncture is indicated when there is a complete obstruction documented or for the definition of the superior margin in a complete block. Suboccipital puncture is contraindicated in patients with craniocervical deformities. Below C2, this procedure is too risky because of the anatomy of the cervical and thoracic spinal cord, which occupies most of the intradural compartment. Go to the next page if you knew the correct answer, or click the link image(s) below to further research the concepts in this question (if desired).

Research Concepts: Suboccipital Puncture

We update eBooks quarterly and Apps daily based on user feedback. Please tap flag to report any questions that need improvement.

Question 961: A 43-year-old male patient came into the emergency room with a decreased level of consciousness and an acute episode of falling down. His vital signs were blood pressure: 65/40 mmHg, pulse rate: 130 beats per minute, respiratory rate: 24 /min, and temperature: 39 C. He obeyed the commands and hardly replied to the questions. Following intravenous hydration with 1000 cc of Ringer's lactate serum, his vital signs were blood pressure: 60/40 mmHg, pulse rate: 120 bpm, respiratory rate: 23/min, and temperature: 38.9 C. His focused abdominal ultrasonography for trauma (FAST) was positive for severe free fluid in Morrison's pouch and pelvic area. His only past medical history is for depression and chronic pain syndrome. He cannot remember the names of her medications. He is induced with general anesthesia and started prophylactically on a vasopressor infusion. Shortly after beginning the infusion, he has a much greater-than-expected elevation in blood pressure that remains much longer than expected after turning off the infusion. What is most likely the mechanism of this event?

Choices: 1. Decreased clearance of norepinephrine from the synaptic space 2. Decreased pain receptors leading to hyperalgesia 3. Increased V2 activity at precapillary arterioles 4. The infusion was incorrectly mixed with a dextrose-containing solution

Answer: 1 - Decreased clearance of norepinephrine from the synaptic space Explanations: The effects seen here are most likely to reduced norepinephrine uptake from the synaptic cleft. Remember that the termination of norepinephrine’s effects is primarily via reuptake into the presynaptic neuron. Certain drugs, such as many antidepressants, will block the reuptake of norepinephrine, both endogenous and exogenous administration. Care is necessary when using norepinephrine concomitantly with monoamine oxidase inhibitors or amitriptyline and imipramine-type antidepressants. The combination of any of these drugs can lead to severe, prolonged hypertension. Go to the next page if you knew the correct answer, or click the link image(s) below to further research the concepts in this question (if desired).

Research Concepts: Norepinephrine

We update eBooks quarterly and Apps daily based on user feedback. Please tap flag to report any questions that need improvement.

Question 962: A 66-year-old man with a past medical history of alcohol use disorder is brought to the hospital after a witnessed seizure. Ten days ago, he underwent a procedure for a right chronic subdural hematoma using a burr hole and was discharged home two days later. The patient is somnolent. The patient also reports headaches, fever, and chills for the past two days. His temperature is 39 C. Physical examination reveals left upper extremity weakness. Which of the following risk factors is most likely responsible for the patient’s condition?

Choices: 1. Epilepsy 2. Recent surgical procedure 3. Skull fracture 4. Alcohol use disorder

Answer: 2 - Recent surgical procedure Explanations: Predisposing factors associated with the development of subdural empyema include prior cranial surgery, head trauma with open skull fractures or penetrating injury, infected hematoma or subdural effusion, and not properly treated ear and sinus infections. Infected subdural hematomas after surgical drainage may develop into empyema. Up to 20% of the subdural empyemas occur following head trauma or cranial surgical procedures. Seizures can occur in subdural empyemas but are not predisposing factors. Go to the next page if you knew the correct answer, or click the link image(s) below to further research the concepts in this question (if desired).

Research Concepts: Subdural Empyema

We update eBooks quarterly and Apps daily based on user feedback. Please tap flag to report any questions that need improvement.

Question 963: A 16-year-old male presented to the emergency department following a high-speed motor vehicle collision. The patient fractured both femurs and developed a tension pneumothorax in his right lung. Chest intubation was done to relieve the pneumothorax. He was transferred to the intensive care unit (ICU) and was kept on noninvasive ventilation (NIV). Intravenous fluids were administered through a central percutaneous catheter, passed into the medial cubital vein. At the time of discharge, while removing the catheter, the tip breaks off, and the removed catheter is 1 cm shorter than the length inserted. What is the next best step?

Choices: 1. Ambulate the patient 2. Perform vagal maneuvers 3. Place a tourniquet above the insertion site 4. Place the patient in Fowler's position

Answer: 3 - Place a tourniquet above the insertion site Explanations: A tourniquet should be applied above the insertion site. The patient should be placed on bedrest, monitored, and kept calm. The patient may be asymptomatic or may have cyanosis, dyspnea, chest pain, hypotension, tachycardia, increased central venous pressure (CVP), loss of consciousness, arrhythmias, or a pulmonary thrombus. X-ray confirmation of the catheter fragment is performed, and surgical removal may be necessary. The same treatment is indicated for a peripheral catheter. Go to the next page if you knew the correct answer, or click the link image(s) below to further research the concepts in this question (if desired).

Research Concepts: Percutaneous Central Catheter

We update eBooks quarterly and Apps daily based on user feedback. Please tap flag to report any questions that need improvement.

Question 964: A 65-year-old male comes to the clinician with fevers, malaise, weakness, and shortness of breath for the past few days. Upon physical examination, there is a systolic murmur along the left lower sternal border. There are tender nodules on the fingers. The fundoscopy shows retinal hemorrhages. The diagnosis is made, and the patient is moved to the critical care unit. Patients with which of the following conditions could have benefitted from the antibiotic prophylaxis for the pathology seen in this patient?

Choices: 1. Previous pacemaker placement 2. Previous coronary artery bypass graft 3. Previous episode of bacterial endocarditis 4. Mitral valve prolapse without regurgitation

Answer: 3 - Previous episode of bacterial endocarditis Explanations: Indications for subacute bacterial endocarditis (SBE) prophylaxis include previous endocarditis, prosthetic cardiac valve, congenital cardiac defects (except for those specified), hypertrophic cardiomyopathy, previous valvular surgery, and rheumatic valvular dysfunction. Indications for SBE prophylaxis include surgically constructed systemic or pulmonic shunts or conduits. Dental or surgical indications for subacute bacterial endocarditis prophylaxis include procedures likely to cause mucosal bleeding, tonsillectomy/adenoidectomy, surgery involving the respiratory or gastrointestinal mucosa, and rigid bronchoscopy. Prophylaxis against endocarditis is not recommended in patients who are at risk of infective endocarditis for other non-dental procedures, for example, TEE, esophagogastroduodenoscopy, colonoscopy, or cystoscopy, in the absence of active infection. Currently, there is no indication for dental, gastrointestinal, or genitourinary procedural prophylaxis for patients with implantable cardiovascular devices. However, prophylaxis with an anti-staphylococcal antibiotic is indicated at the time of cardiovascular device implantation and any subsequent manipulation of the surgically created device pocket. Go to the next page if you knew the correct answer, or click the link image(s) below to further research the concepts in this question (if desired).

Research Concepts: Subacute Bacterial Endocarditis Prophylaxis

We update eBooks quarterly and Apps daily based on user feedback. Please tap flag to report any questions that need improvement.

Question 965: A 17-year-old obese male patient involved in a motor vehicle accident had to be emergently intubated in the field by the EMS personnel. Intubation was reported as difficult because of his body habitus, and it required four attempts to successfully place the endotracheal tube. The patient suffered a small subdural hemorrhage, which was conservatively managed. On day 3, spontaneous awakening trial and spontaneous breathing trial (SBT) were done. The patient was awake, following commands, and passed his SBT. What is the next important step prior to extubating the patient?

Choices: 1. Obtain CT head prior to extubation 2. Perform cuff leak test 3. Administer intravenous lorazepam 4. Perform SBT for 6 hours

Answer: 2 - Perform cuff leak test Explanations: Cuff leak test should be routinely done prior to extubation, especially in someone who has risk factors for laryngeal edema including difficult intubation, multiple intubation attempts, airway injury, prolonged mechanical ventilation, etc. Cuff leak test is performed by deflating the cuff of the endotracheal tube and checking for an air leak around the tube. A patient who has an absent cuff leak should be evaluated for airway or laryngeal edema prior to extubation. These patients with absent cuff leak should be treated with intravenous steroids for a couple of days to treat the laryngeal edema prior to extubation to reduce the risk of stridor and post-extubation respiratory failure. Go to the next page if you knew the correct answer, or click the link image(s) below to further research the concepts in this question (if desired).

Research Concepts: Ventilator Weaning

We update eBooks quarterly and Apps daily based on user feedback. Please tap flag to report any questions that need improvement.

Question 966: A 65-year-old woman is brought to the emergency department with an intermittent loss of consciousness. ECG reveals prolongation of the QT interval. The patient recently started antifungal therapy for a complicated skin infection. Her current medications are omeprazole, pimozide, acetaminophen, and ketoconazole. A drug interaction between which of the following medications most likely caused this patient presentation?

Choices: 1. Omeprazole and acetaminophen 2. Pimozide and acetaminophen 3. Ketoconazole and omeprazole 4. Ketoconazole and pimozide

Answer: 4 - Ketoconazole and pimozide Explanations: Ketoconazole can inhibit the metabolism of pimozide. High plasma concentrations of pimozide can cause cardiac arrhythmias. Pimozide is used to treat Tourette syndrome. Ketoconazole is an antifungal. Go to the next page if you knew the correct answer, or click the link image(s) below to further research the concepts in this question (if desired).

Research Concepts: Drug Metabolism

We update eBooks quarterly and Apps daily based on user feedback. Please tap flag to report any questions that need improvement.

Question 967: A 30-year-old previously healthy patient is transferred to your intensive care unit (ICU) for further management. He was diagnosed with influenza three days prior and presented to another hospital with shortness of breath and developed acute respiratory failure requiring intubation and mechanical ventilation. His oxygenation has continued to deteriorate, and he has developed a requirement for significant ventilator settings. His pulse oximetry is only reading 82% on FiO2 of 1 and a mean airway pressure of 30. Due to the patient's acuity and potential for needing further support, he is transferred to a tertiary care center. On arrival, his oxygenation index is 40, and his P: F ratio is 60. He is not requiring vasopressor support. He is at a center that provides extracorporeal membrane oxygenation (ECMO) support, and it is anticipated that this patient will need it. Of the following options, what is the best-standardized approach?

Choices: 1. Alert your ICU team, surgical team, and ECMO team that this patient will likely need venoarterial ECMO. 2. Alert your ICU team, surgical team, and ECMO team that this patient will likely need venovenous ECMO. 3. Alert your surgical team and ICU team that this patient will need veno-arterial ECMO. 4. Continue to manage the patient with the current ventilator support and alert the ECMO team if he were to deteriorate clinically.

Answer: 2 - Alert your ICU team, surgical team, and ECMO team that this patient will likely need veno-venous ECMO.

Explanations: Veno-venous ECMO support is an appropriate mode of ECMO support in this patient as his cardiac function appears to be intact, as shown by his hemodynamic profile. Veno-venous ECMO requires cannulation via the vena cavae with a double lumen cannula. This cannula provides the ability to remove venous return and send it to the ECMO circuit for oxygenation and carbon dioxide removal and sent back to the patient via the same cannula in the second lumen directed towards the right side of the patient's heart. This mode requires that the patient's cardiac function is intact. Veno-venous ECMO is a mode that is appropriate for patients who have isolated respiratory pathology. Through simulation-based training, checklists are often formed to assess patient eligibility. The patient in the scenario has acute respiratory distress syndrome (ARDS) that is being managed with ventilatory settings that may be causing worsening baro- and volu-trauma to his lungs. Continuing to manage the patient as he is maybe detrimental and cause further damage. When a patient may need ECMO support, it should be an approach that involves all teams that would be involved in placing the patient on ECMO, management of the patient afterward, and preparing and managing the ECMO circuit through the patient's course. Go to the next page if you knew the correct answer, or click the link image(s) below to further research the concepts in this question (if desired).

Research Concepts: Extracorporeal Membrane Oxygenation Simulation

We update eBooks quarterly and Apps daily based on user feedback. Please tap flag to report any questions that need improvement.

Question 968: At what interval is cardioversion most effective in patients with atrial fibrillation (AF)?

Choices: 1. >14 days after onset of AF 2. >21 days after onset of AF 3. 7 days after onset of AF 4. >7 days after onset of AF

Answer: 3 - 7 days after onset of AF Explanations: During AF, the atria contract chaotically and irregularly when compared to the ventricles. Cardioversion is most successful if attempted 7 days after onset of AF. It should be done emergently for those with symptomatic (hypotension, heart failure or angina) AF. Because of the risk of thromboembolism, obtaining an echo is highly recommended before cardioverting a patient. If a blood clot is seen in the left atrium, the patient should be anticoagulated before the procedure. Go to the next page if you knew the correct answer, or click the link image(s) below to further research the concepts in this question (if desired).

Research Concepts: Atrial Fibrillation

We update eBooks quarterly and Apps daily based on user feedback. Please tap flag to report any questions that need improvement.

Question 969: A 68-year-old man with a history of major depressive disorder, osteoarthritis, and type 2 diabetes mellitus is brought to the emergency department by his wife due to attempted suicide. She says she is not sure what medication he ingested. His current medications include lisinopril, fluoxetine, and over the counter naproxen as needed. He appears drowsy and has sinus tachycardia; however, all other vitals are normal. On further investigation, he is noted to have severe metabolic acidosis, and his BUN/Creatinine has increased significantly compared to previous records. What is the next best step in the management of this patient?

Choices: 1. Supportive treatment only 2. Hemodialysis 3. Activated charcoal 4. Continuous monitoring only until the patient stabilizes

Answer: 2 - Hemodialysis Explanations: This patient presents to the emergency department with a possible overdose of naproxen. Naproxen overdose is common due to its over the counter availability, but the overdose is usually mild in severity, and serious adverse effects from overdose are rare. There is no available antidote for naproxen overdose, however, monitoring of vital signs and supportive care is recommended. However, ingestion of large amounts of naproxen can lead to severe toxicity causing seizures and metabolic acidosis, which can potentially cause renal failure. Therefore, although hemodialysis is not generally recommended, in specific situations, it can correct acid-base disturbances and provide additional support to those with renal impairment. The role of activated charcoal is uncertain due to time constraints and unclear benefits. Go to the next page if you knew the correct answer, or click the link image(s) below to further research the concepts in this question (if desired).

Research Concepts: Naproxen

We update eBooks quarterly and Apps daily based on user feedback. Please tap flag to report any questions that need improvement.

Question 970: After coronary angioplasty, a patient develops decreased pedal pulses, livido reticularis, abdominal pain, and confusion. Laboratories show BUN 120 mg/dL, creatinine 5.0 mg/dL, and phosphate 9.2 mg/dL. Urinalysis has 8 to 10 WBC/HPF, 5 10 to 15 RBC/HPF, and few hyaline casts. Select the most likely diagnosis.

Choices: 1. Acute tubular necrosis 2. Cholesterol embolism 3. Renal artery thrombosis 4. Acute tubular necrosis secondary to hypotension

Answer: 2 - Cholesterol embolism Explanations: Disruptions of plaques in the aorta or renal arteries during left heart catheterization can dislodge cholesterol. This can affect peripheral vasculature, bowel, pancreas, and kidneys. The urinalysis can show amorphous sediment or hyaline casts but, with ATN, granular casts would be more likely. Eosinophiluria and decreased complement levels are possible. Go to the next page if you knew the correct answer, or click the link image(s) below to further research the concepts in this question (if desired).

Research Concepts: Cholesterol Emboli

We update eBooks quarterly and Apps daily based on user feedback. Please tap flag to report any questions that need improvement.

Question 971: A 30-year-old female underwent a popliteal fossa nerve block in preparation for a complex ankle repair with ultrasound guidance to facilitate placement of 35 mL of 0.25% bupivacaine. Five minutes after administration of the nerve block, the patient experiences confusion followed by rapid cardiovascular collapse and asystole on the electrocardiogram. Lipid emulsion is not available from the pharmacy due to a nationwide shortage. Standard advanced cardiac life support measures are ineffective after 20 minutes. Which of the following measures should be considered for this patient?

Choices: 1. Emergent plasmapheresis 2. Emergent dialysis 3. Emergency placement of a left ventricular assist device 4. Emergent cardiopulmonary bypass

Answer: 4 - Emergent cardiopulmonary bypass Explanations: Prior to lipid emulsion being a treatment of choice for local anesthetic systemic toxicity, a cardiopulmonary bypass in order to support the patient's cardiovascular status was used as a life-saving maneuver. Bupivacaine is a long-acting local anesthetic, so prolonged support may be needed in cases of profound toxicity. Because access to cardiopulmonary bypass takes time and the expertise of surgical colleagues and support staff, this therapy needs to be considered early so that preparations can be in place. Smaller hospitals may not have access to the equipment or staff needed. A left ventricular assist device would take too long to place and is not appropriate therapy for this situation. Due to particle size, dialysis or plasmapheresis would not be effective at removing bupivacaine from the circulation and are not appropriate treatments. Go to the next page if you knew the correct answer, or click the link image(s) below to further research the concepts in this question (if desired).

Research Concepts: Bupivacaine

We update eBooks quarterly and Apps daily based on user feedback. Please tap flag to report any questions that need improvement.

Question 972: A 45-year-old male has a blood transfusion after being involved in a motor vehicle collision. Forty minutes after, he reports chills, discomfort at the infusion site, and severe flank pain. His vital signs show oxygen saturation of 98% on room air, respiratory rate of 21 breaths per minute, heart rate 125 beats per minute, blood pressure 135/83 mmHg, and temperature 102.7 F (39.2 C). The transfusion is stopped, but he has no relief of the symptoms. His catheter bag becomes red, and he begins to bleed around his intravenous catheter site. Which of the following is the most likely cause of this reaction?

Choices: 1. ABO incompatibility 2. Febrile non-hemolytic transfusion reaction 3. Reaction to cytokines stored in the transfused blood 4. IgA deficiency

Answer: 1 - ABO incompatibility Explanations: Acute hemolytic transfusion reaction (AHTR) typically develops within an hour after the transfusion is started. It is a medical emergency that is usually caused by ABO incompatibility; symptoms include fever, chills, hemoglobinuria, flank pain, and discomfort at the infusion site. This may advance to renal failure due to immune complex deposition or disseminated intravascular coagulation (DIC) (e.g., bleeding from the intravenous site in this patient). The management includes the immediate cessation of transfusion while maintaining intravenous access for fluids (normal saline) and supportive care (diuretics such as mannitol). Massive blood transfusions can often cause hypothermia, hypocalcemia, and hyperkalemia. Go to the next page if you knew the correct answer, or click the link image(s) below to further research the concepts in this question (if desired).

Research Concepts: Hemolytic Transfusion Reaction

We update eBooks quarterly and Apps daily based on user feedback. Please tap flag to report any questions that need improvement.

Question 973: A 31-year-old marathon runner presented with a complaint of fatigue for the past day. Her blood pressure was measured which turned out to be 200/130 mmHg. She had no previous memory of ever measuring her blood pressure. Her medical and family histories were unremarkable. Her other vital signs were normal and physical examination was unremarkable. The targeted blood pressure in the next 24hours is 130/80 mmHg. What would be the most appropriate way of normalizing her blood pressure?

Choices: 1. 5 to 15% in the 1st hour and 10 to 20% over the next 24 hours. 2. 10 to 20% in the 1st hour and 10 to 20% over the next 24 hours. 3. 10 to 20% in the 1st hour and 5 to 15% over the next 24 hours. 4. 5 to 15% in the 1st hour and 5 to 15% over the next 24 hours.

Answer: 3 - 10 to 20% in the 1st hour and 5 to 15% over the next 24 hours. Explanations: Malignant hypertension is extremely high blood pressure that develops rapidly and causes some type of organ damage. About 1% of people who have a history of high blood pressure develop this life-threatening condition. It is not recommended to decrease blood pressure too fast or too much, as ischemic damage can occur in vascular territories that have become habituated with the elevated level of blood pressure. For the majority of hypertensive emergencies, mean arterial pressure (MAP) should be reduced by approximately 10 to 20% within the first hour and by another 5% to 15% over the next 24 hours. This often results in a target BP of less than 180/120 mm Hg for the first hour and less than 160/110 mm Hg for the next 24 hours, but rarely less than 130/80 mm Hg during that time frame. Go to the next page if you knew the correct answer, or click the link image(s) below to further research the concepts in this question (if desired).

Research Concepts: Malignant Hypertension

We update eBooks quarterly and Apps daily based on user feedback. Please tap flag to report any questions that need improvement.

Question 974: A 55-year-old man was recently diagnosed with hypertension. He was started on hydrochlorothiazide and amiloride. The coadministration of thiazide with amiloride is done because amiloride does which of the following?

Choices: 1. Potentiates the effect of antidiuretic hormone 2. Has anticholinergic action 3. Blocks the epithelial sodium channel in the kidney 4. Inhibits carbonic anhydrase in the proximal convoluted tubule

Answer: 3 - Blocks the epithelial sodium channel in the kidney Explanations: Both amiloride and triamterene work by inhibiting the epithelial sodium channels (ENaC) in the distal nephron. Amiloride is co-administered with thiazide to prevent thiazide-induced hypokalemia. Normally the sodium moves down its electrochemical gradient to enter the tubular cells through the ENaCs. This gradient is created by the basolateral membrane Na/K ATPase. Reabsorption of sodium is associated with depolarization of the apical membrane which creates a lumen-negative transepithelial potential difference. This potential difference enhances potassium secretion through the apical potassium channels and subsequently potassium excretion Amiloride selectively inhibits ENaCs resulting in a decrease in hyperpolarization of the apical membrane and subsequently decrease in potassium, hydrogen, calcium, and magnesium secretion It should be noted that other potassium-sparing diuretics, spironolactone, and eplerenone, act via a different mechanism. These drugs are aldosterone antagonists and they are helpful in cases of secondary hyperaldosteronism in edematous states like congestive heart failure Go to the next page if you knew the correct answer, or click the link image(s) below to further research the concepts in this question (if desired).

Research Concepts: Amiloride

We update eBooks quarterly and Apps daily based on user feedback. Please tap flag to report any questions that need improvement.

Question 975: A 48-year-old man with severe obesity and a history of IV drug use endstage renal disease on dialysis is brought to the hospital with shortness of breath. Shortly after arrival, the patient suffers a cardiac arrest. IV access cannot be obtained. What is the most appropriate access to obtain at this time?

Choices: 1. IV access using ultrasound 2. A central line via the patient's femoral vein 3. An intraosseous line 4. An external jugular IV access

Answer: 3 - An intraosseous line Explanations: In a critically ill patient, IO access can provide quick access when IV access cannot be obtained promptly. Patients with a history of IV drug use, obesity, hemodialysis, or hypotensive will be more difficult to obtain IV access and will likely take longer to obtain a peripheral IV even with the guidance of ultrasound. When these patients are critically ill, do not delay care by attempting ultrasound-guided IV multiple times. It might be more prudent to switch to another form of access. While there are no absolute contraindications to the use of ultrasound-guided IV access, there are relative contra-indications such as multiple failed attempts, infections, or injuries to the site, or when time is critical. Go to the next page if you knew the correct answer, or click the link image(s) below to further research the concepts in this question (if desired).

Research Concepts: Ultrasound Intravascular Access

We update eBooks quarterly and Apps daily based on user feedback. Please tap flag to report any questions that need improvement.

Question 976: An 84-year-old man with a past medical history of hypertension, hyperlipidemia, benign prostatic hyperplasia, and bladder cancer has been admitted to the intensive care unit with acute hypoxemic respiratory failure in the setting of sepsis due to a urinary tract infection. The patient has been intubated for four days and is unable to be weaned off the vent. The ICU team orders a nerve conduction study and electromyography to rule out critical illness neuropathy or myopathy. Which of the following is the most appropriate reason to avoid performing this study on this patient?

Choices: 1. Subcutaneous heparin use 2. Cellulitis of the patient's leg 3. Too early in the disease process to find electrodiagnostic evidence 4. Too much electrical interference in the ICU setting

Answer: 3 - Too early in the disease process to find electrodiagnostic evidence Explanations: Wallerian degeneration (axonal denervation) may take at least seven days or longer before becoming evident in an electrodiagnostic study. This patient has only been septic and intubated for four days and therefore it is unlikely that the study will provide any clinical use at this point in the admission. If the patient remains ventilated for longer, NCS/EMG should be performed to assess critical illness neuropathy/polyneuropathy. While caution should be taken in performing a needle study on patients who are on blood thinners, one of the only true contraindications to an EMG study would be in a patient with a severe bleeding disorder or who has an uncontrolled coagulation profile such a supratherapeutic INR while on warfarin. Subcutaneous heparin is not the reason to postpone this exam at this time. EMGs should not be performed directly over an area of soft tissue infection such as cellulitis. However, a complete study could still be performed using the other 3 limbs and would still provide enough electrodiagnostic information. In this case, the cellulitis does not prevent the healthcare provider from performing the exam at this time, but the patient has only been in the ICU for four days, and therefore an EMG at this time will likely not show significant results. Although it is true that the electrical interference in the ICU setting may create a suboptimal environment to perform electrodiagnostic studies, the test can still be performed. Some strategies to improve the quality of the study include ensuring electrodes properly adhere, using the notch filter on the machine, turning off unnecessary machines or monitors, and unplugging the bed. Go to the next page if you knew the correct answer, or click the link image(s) below to further research the concepts in this question (if desired).

Research Concepts: Electrodiagnostic Evaluation Of Critical Illness Neuropathy

We update eBooks quarterly and Apps daily based on user feedback. Please tap flag to report any questions that need improvement.

Question 977: A 42-year-old male, who has been in the medical intensive care unit for 2 weeks due to acute respiratory distress syndrome (ARDS) has been failing his spontaneous breathing trials (SBTs) for 4 straight days. Before the start of SBT, he is awake and alert and hemodynamically stable. Fifteen minutes into his SBT, his tidal volumes are in the range of 100150 mL with a respiratory rate of 35/minute. He becomes lethargic and is minimally responsive now. He is switched back to full ventilator support. Arterial blood gas obtained at this time shows a pH of 7.2, pCO2 60 mmHg, and pO2 100 mmHg. Chest x-ray shows low lung volumes with otherwise normal parenchyma. Ultrasound of the diaphragm shows bilateral diaphragm thickness of 10 mm with thickening to 11 mm on deep inspiration. What is the next best step in management?

Choices: 1. Proceed with a diaphragmatic pacer 2. Surgical diaphragmatic plication 3. Discuss tracheostomy and transfer to long term acute care hospital (LTACH)/rehabilitation facility 4. Continue with daily SBT trials

Answer: 3 - Discuss tracheostomy and transfer to long term acute care hospital (LTACH)/rehabilitation facility

Explanations: The patient has bilateral diaphragmatic atrophy and weakness from prolonged mechanical ventilation, as evidenced by a thinned diaphragm and reduced fractional shortening on maximal inspiration. Critical illness polyneuropathy (CIP) is a common complication of critical illness affecting the motor and sensory neurons. Muscle involvement causing loss of muscle mass and eventual weakness has been referred to as critical illness myopathy. These patients have worse outcomes with prolonged weaning, higher hospital length of stays, and dependency on mechanical ventilation. There is no current evidence of using diaphragmatic pacing for these patients. Trials examining their role are currently undergoing. The diaphragm strength and function would usually recover over time with intensive physical therapy and nutrition. Therefore, tracheostomy and transfer to LTACH/rehabilitation center is a reasonable approach. Diaphragm ultrasound, if used early in the weaning phase, can help identify patients who are at risk of weaning or extubation failure. Go to the next page if you knew the correct answer, or click the link image(s) below to further research the concepts in this question (if desired).

Research Concepts: Diaphragm Disorders

We update eBooks quarterly and Apps daily based on user feedback. Please tap flag to report any questions that need improvement.

Question 978: A 65-year-old man comes to the emergency department with a complaint of tongue deviation. He has a history of diabetes mellitus for ten years, but he is not compliant with the medications. He has smoked 30 cigarettes a day for the past 25 years. On examination, his blood pressure is 160/95 mm Hg, pulse is 85 bpm, and respiratory rate is 20/min. On neurological examination, there is a decrease in the power of the muscle on the left side of the body. Decreased position and vibratory sense in the left side of the body is seen, but other sensations are normal. The tongue deviates to the right, and while the rest of the cranial nerve examination is unremarkable. Which imaging modality should be used to visualize the lesion?

Choices: 1. Computerized tomography (CT) scan 2. Magnetic resonance imaging (MRI) of the brain 3. Ultrasonogram (USG) of the brain 4. Computerized tomography (CT) angiogram of the brain

Answer: 2 - Magnetic resonance imaging (MRI) of the brain Explanations: Contralateral hemiplegia, ipsilateral hypoglossal dysfunction, and contralateral decrease proprioception is the diagnostic triad of medial medullary syndrome or Dejerine syndrome. For the diagnosis of medial medullary syndrome, one can do both CT scan and MRI. MRI can visualize the brain lesion better than a CT scan because it is difficult to visualize posterior cranial fossa structures by CT scan, which may be obscure by the bony structures. The patient tongue deviates to the left, and there is a weakness in the right side of the body. So the diagnosis is medial medullary syndrome or Dejerine syndrome. The patient has two risk factors (diabetes mellitus and smoking) of atherosclerosis. These risk factors are among the most common risk factors for Dejerine syndrome. The Dejerine syndrome is most commonly caused by atherothrombotic occlusion of paramedian branches of the anterior spinal artery or vertebral artery or basilar artery. Common risk factors are dyslipidemia, hypertension, diabetes, and smoking. Other risk factors are atrial fibrillation, atrial septal defect, migraine, and Takayasu arteritis. Dissection of the vertebral artery can also cause medial medullary syndrome, especially among young patients. Go to the next page if you knew the correct answer, or click the link image(s) below to further research the concepts in this question (if desired).

Research Concepts: Medial Medullary Syndrome

We update eBooks quarterly and Apps daily based on user feedback. Please tap flag to report any questions that need improvement.

Question 979: A 60-year old male presents to the emergency department (ED) with fatigue and weakness. His blood pressure is 76/62 mmHg, and his heart rate is 49 beats per minute. EKG indicates PR prolongation. He states that his only medication is verapamil. A urine toxicology screen is negative. He continues to be hypotensive and bradycardic despite IV bolus of normal saline and 3 atropine doses. Which of the following is an appropriate next step?

Choices: 1. Intraarterial calcium gluconate infusion delivered over 3 hours 2. Intravenous calcium gluconate infusion and recheck calcium level 3. Intravenous pralidoxime bolus administered twice 4. Increase verapamil dosage and monitor for symptomatic improvement

Answer: 2 - Intravenous calcium gluconate infusion and recheck calcium level Explanations: Hypotension, bradycardia, PR prolongation on EKG are characteristics of calcium channel blocker (CCB) toxicity, likely from verapamil. Calcium gluconate is part of the treatment for patients with persistent shock despite fluid resuscitation and atropine in patients with CCB toxicity. CCB toxicity should be treated with intravenous (not intraarterial) calcium gluconate. Dosing is a bolus of 0.6 mL/kg of 10% calcium gluconate solution, followed by a continuous infusion of 0.6-1.5 mL/kg/hr. Throughout the CCB toxicity treatment, ionized calcium levels should be monitored to achieve a goal of calcium level two times normal. Pralidoxime is administered for organophosphate poisoning. It has no role in the treatment of CCB toxicity. Verapamil dosage should not be increased in this patient who in shock likely due to toxicity from this medication. Intraarterial calcium might be utilized during moderate to severe burn. Go to the next page if you knew the correct answer, or click the link image(s) below to further research the concepts in this question (if desired).

Research Concepts: Calcium Gluconate

We update eBooks quarterly and Apps daily based on user feedback. Please tap flag to report any questions that need improvement.

Question 980: A 53-year-old man is brought to the emergency department with collapse and seizure. The patient was on a long walk with his wife and had missed lunch. The patient was well until 6 weeks ago. For the last 6 weeks, the patient has been complaining of occasional sudden onset sweating, tremors, blurred vision, and mental fog. These episodes usually occurred after exertion or delaying his meal. His past medical history includes hypertension, for which he takes amlodipine. His family history includes ischemic heart disease and type 2 diabetes. Physical examination is unremarkable. Point-of-care blood glucose is 40 mg/dL. Insulin, Cpeptide, and beta-hydroxybutyrate are ordered. Which of the following is the next best step in the evaluation of this patient?

Choices: 1. Thyroid function test 2. 72-hour fast 3. Liver function tests 4. Serum glucose by laboratory

Answer: 4 - Serum glucose by laboratory Explanations: The patient has a low blood glucose level on fingerprick testing. However, it is important to remember that a glucometer's blood glucose reading can occasionally give false and inaccurate readings. Therefore, any abnormal blood glucose readings of a glucometer should be confirmed with a serum glucose measurement. Furthermore, C-peptide and insulin levels can only be accurately interpreted with the serum glucose. Thyroid function would be part of the panel. There have been case reports of hypoglycemia in patients with severe hypothyroidism or hyperthyroidism. 72-hour fast is an important test to undertake for diagnosing hypoglycemia and its cause. This test may need to be arranged at a later date. A liver function test would also form part of the panel. Go to the next page if you knew the correct answer, or click the link image(s) below to further research the concepts in this question (if desired).

Research Concepts: Non-diabetic Hypoglycemia

We update eBooks quarterly and Apps daily based on user feedback. Please tap flag to report any questions that need improvement.

Question 981: A 40-year-old woman with severe persistent asthma presents to the emergency department (ED). She is well known to the intensive care unit, having required intubation with mechanical ventilation in the past. On initial evaluation, the patient is maintaining adequate oxygen saturation; however, she is tachypneic while on non-invasive positive pressure ventilation, and her chest examination reveals diffuse wheezing. In the ED, the patient is intubated, and her transport to the ICU is to be expedited, following portable chest radiography. After arriving at the ICU, the respiratory therapist notes elevated peak inspiratory pressures, unequal chest wall expansion, and unilateral wheezing. Attempts to view the postintubation radiograph are unsuccessful as the file is corrupted. Bedside thoracic sonography reveals the presence of lung sliding in the right lung field. While lung sliding is absent in the left lung field, rhythmic pleural movements in concert with the cardiac cycle are noted. Suddenly, the patient develops hypoxia. A normal waveform is seen on the pulse oximeter. Which of the following is the next best step in the management of this patient?

Choices: 1. Reposition the pulse oximeter 2. Increase the PEEP 3. Perform an emergent left-sided needle thoracostomy 4. Retract the ETT

Answer: 4 - Retract the ETT Explanations: Right main bronchus intubation requires prompt identification and treatment with retraction of the ETT to a suitable position (=2 cm) above the tracheal carina. In this patient, elevated peak inspiratory pressures, unequal chest wall expansion, and unilateral wheezing are highly suggestive of right main bronchus intubation. As mentioned in the article, airflow is required to produce wheezing. The absence of wheezing in the left lung field is consistent with the absence of airflow. It is imperative that proper placement of the endotracheal tube be established as soon as possible after its insertion. Clinical examination should always be performed; however, it is insufficient for the determination of ETT placement. Quantitative capnography is a simple, but accurate method, for confirming the placement of the ETT in the airway (rather than the esophagus) and should always be performed. This method, however, cannot determine the depth of the ETT in the trachea, which is best established with a chest radiograph. The findings of bedside thoracic sonography effectively rule out pneumothorax, making emergent needle thoracostomy an incorrect answer choice. Additionally, while increasing PEEP is a solution to hypoxia, it is inappropriate in this clinical scenario, as doing so could potentiate barotrauma, given the already elevated peak inspiratory pressures. Lastly, the repositioning of the pulse oximeter would be futile, given the normal waveform on the pulse oximeter, making this an incorrect answer choice. Go to the next page if you knew the correct answer, or click the link image(s) below to further research the concepts in this question (if desired).

Research Concepts: Wheezing

We update eBooks quarterly and Apps daily based on user feedback. Please tap flag to report any questions that need improvement.

Question 982: An 85-year-old male is brought to the emergency department after being assaulted by a group of teenagers in a pub. He was stabbed in the left chest and the abdomen. His blood pressure is 80/50 mm Hg, and his pulse rate is 114/min. A chest x-ray reveals a left-sided tension pneumothorax, and an emergent ultrasound shows a puncture in the spleen. A left-sided chest tube insertion is performed to relieve the tension pneumothorax. A laparotomy is planned to repair the damaged spleen. A central venous line is passed in the internal jugular vein to administer fluids. During central venous access of this vein, the needle is directed towards what at a 45-degree angle?

Choices: 1. Hip 2. Posterior spine 3. Ipsilateral nipple 4. Contralateral nipple

Answer: 3 - Ipsilateral nipple Explanations: During central venous access of the internal jugular vein, the finder needle is angled at a 45degree angle towards the patient's ipsilateral nipple. The catheter is inserted into the internal jugular vein between the clavicular heads of the sternocleidomastoid, at the apex of the triangle formed by the clavicle and the heads of the muscles. Ultrasound guidance reduces failure rate, number of attempts, misplacement, and subsequent line sepsis. The bottom line is that one must know the anatomy of the jugular vein. It is lateral to the carotid artery. Complications occur when the line is inserted by physicians who have little experience. Go to the next page if you knew the correct answer, or click the link image(s) below to further research the concepts in this question (if desired).

Research Concepts: Internal Jugular Vein Central Venous Access

We update eBooks quarterly and Apps daily based on user feedback. Please tap flag to report any questions that need improvement.

Question 983: A 66-year-old female presented to the hospital with a complaint of an episode of blood in the urine that appeared an hour ago. She also complains of pain in her left flank. On further questioning, she reveals that she has had a burning sensation with urination for the past two days. A detailed medical history reveals that she was diagnosed as a case of diabetes mellitus twelve years ago, and her blood sugar levels are well controlled. Her vital signs show blood pressure 120/80 mm Hg, heart rate 110/min, respiratory rate 16/min, and temperature 102.2 F (39 C). A physical examination performed reveals left flank tenderness on palpation. A urinalysis performed reveals 15 white blood cells per cubic millimeter of centrifuged urine. What is the complication associated with her condition that causes necrotizing infection?

Choices: 1. Xanthogranulomatous pyelonephritis 2. Emphysematous pyelonephritis 3. Corticomedullary abscess 4. Acute lobar nephroma

Answer: 2 - Emphysematous pyelonephritis Explanations: The clinical scenario is most consistent with complicated acute pyelonephritis as the patient is above 65. Acute pyelonephritis is an ascending infection of the urinary tract. Emphysematous pyelonephritis is a severe necrotizing infection that occurs as a complication of acute pyelonephritis of the kidney. It is usually caused by Escherichia coli or Klebsiella pneumonia and occurs mostly in females. It is associated with diabetes mellitus. Corticomedullary abscess, acute lobar nephroma, xanthogranulomatous pyelonephritis are complications of acute pyelonephritis as well but are non-necrotizing complications. Go to the next page if you knew the correct answer, or click the link image(s) below to further research the concepts in this question (if desired).

Research Concepts: Acute Pyelonephritis

We update eBooks quarterly and Apps daily based on user feedback. Please tap flag to report any questions that need improvement.

Question 984: A 30-year-old woman presents to the emergency department with severe breathing difficulty. She has a history of asthma. Chest auscultation reveals bilateral wheeze, and she is unable to complete a sentence. Treatment is commenced with back-to-back nebulizations with salbutamol and ipratropium bromide; IV hydrocortisone is also given. She is confused and exhausted. Rapid sequence induction and intubation with a size 8.0 endotracheal tube is performed. However, it is impossible to ventilate the patient, and the saturations drop to 70% on 100% oxygen. Which of the following is the next best step in the management of this patient?

Choices: 1. Check the position and patency of the endotracheal tube 2. Needle decompression 3. Administer further salbutamol and ipratropium nebulizers 4. Magnesium sulfate

Answer: 1 - Check the position and patency of the endotracheal tube Explanations: If it is impossible to bag a patient after intubation, you must ensure your endotracheal tube's patency and position. This question is really more about testing your ability to apply common sense in a stressful situation rather than acute severe asthma management. Severe bronchospasm may make ventilation very difficult, and only very low tidal volumes may be achievable, but a complete obstruction to any gas flow is unlikely. Gas trapping makes over-distension a hazard, and there is a high risk of barotraumas and pneumothoraces. If the patient is truly impossible to ventilate, you should disconnect the circuit and manually decompress the chest before further ventilation attempts. Generally, ventilation strategies should include low tidal volumes and a low respiratory rate with prolonged expiratory times. The use of PEEP is controversial. Go to the next page if you knew the correct answer, or click the link image(s) below to further research the concepts in this question (if desired).

Research Concepts: Status Asthmaticus

We update eBooks quarterly and Apps daily based on user feedback. Please tap flag to report any questions that need improvement.

Question 985: A 58-year-old male with a history of hypertrophic cardiomyopathy presents with orthopnea, dyspnea at rest, and 15 lb (7 kg) weight gain. He has been managed previously on metoprolol and verapamil. An echocardiogram reveals reduced ejection fraction and left ventricular chamber enlargement. Which management strategy is best for this patient?

Choices: 1. Cardioversion 2. Septal ablation 3. Add disopyramide 4. Discontinue verapamil and add diuretic and ace inhibitor

Answer: 4 - Discontinue verapamil and add diuretic and ace inhibitor Explanations: A small percentage of patients with hypertrophic cardiomyopathy (HCM) develop systolic dysfunction and a clinical picture consistent with dilated cardiomyopathy. Verapamil should be discontinued as it can reduce output and more conventional heart failure medical optimization should be implemented. Cardioversion is appropriate for managing atrial fibrillation as a complication of HCM. Septal ablation is considered in cases of severe symptomatic obstructive HCM. Go to the next page if you knew the correct answer, or click the link image(s) below to further research the concepts in this question (if desired).

Research Concepts: Hypertrophic Cardiomyopathy

We update eBooks quarterly and Apps daily based on user feedback. Please tap flag to report any questions that need improvement.

Question 986: A 61-year-old man is brought to the emergency department for abrupt loss of consciousness after a fall at his home that happened 2 hours ago. His past medical history is notable for atrial fibrillation. He has otherwise been well without additional medical problems. His medications include warfarin and metoprolol. Currently, his temperature is 98.6, his blood pressure is 125/75 mm Hg, his pulse rate is 77/min and irregular, and his respiration rate is 17/min. The patient is obtunded without localizing neurologic findings. The cardiac examination reveals an irregularly irregular rhythm. The remainder of the examination is unremarkable. A head CT scan shows a large subdural hematoma. Lab values indicate a hemoglobin level of 12.8 g/dL, platelet count of 283,000/µL, and INR of 3.0. Intravenous vitamin K is administered, and plans are made for emergent neurosurgery. Which of the following is the next most appropriate step in managing this patient?

Choices: 1. Fresh frozen plasma 2. Factor IX complex 3. Cryoprecipitate 4. Idarucizumab

Answer: 2 - Factor IX complex Explanations: Factor IX complex, known as prothrombin complex concentrate (PCC), should be used to reverse the effects of warfarin anticoagulation in patients experiencing severe bleeding and those requiring urgent surgery. Although vitamin K alone can be effective in reversing the effect of warfarin, its hemostatic effect can take several hours. In urgent situations, simultaneous replacement of the vitamin K–dependent coagulation factors is necessary. Cryoprecipitate would be indicated to treat severe hypofibrinogenemia, usually arising as a consequence of disseminated intravascular coagulation (DIC) or severe liver disease. Idarucizumab is a monoclonal antibody that binds the non–vitamin K antagonist oral anticoagulant dabigatran and causes a rapid reduction in available dabigatran in the body for up to 24 hours. Idarucizumab will not reverse warfarin anticoagulation. Go to the next page if you knew the correct answer, or click the link image(s) below to further research the concepts in this question (if desired).

Research Concepts: Prothrombin Complex Concentrate

We update eBooks quarterly and Apps daily based on user feedback. Please tap flag to report any questions that need improvement.

Question 987: A 68-year-old male with a past medical history of hyperlipidemia, hypertension, and benign prostatic hypertrophy is admitted to the hospital with septic shock. He is treated with intravenous fluids, piperacillin-tazobactam, and norepinephrine. On day 2 of admission, he complains of abdominal pain in the left lower quadrant. The pain started approximately 2 hours ago and 50 minutes before the evaluation, he had an episode of hematochezia. His home medications include atorvastatin, low-dose aspirin, and chlorthalidone. On physical examination today, the patient is alert and oriented. Vital signs reveal temperature 38.7 C (101.6 F), blood pressure 100/60 mmHg, pulse rate 85/min, respiratory rate 19/min, and oxygen saturation 94% on ambient air. Abdominal examination reveals tenderness over the left side of the abdomen without guarding. Which of the following is the most appropriate next step in the management of this patient?

Choices: 1. CT angiography 2. MR angiography 3. Plain radiography 4. Sigmoidoscopy

Answer: 1 - CT angiography Explanations: Patients with acute colonic ischemia commonly present with sudden onset cramping abdominal pain, which usually involves the left side. The pain can be accompanied by an urgent desire for a bowel movement. Instead of the periumbilical pain characteristic of small intestine ischemia, patients with colonic ischemia usually feel the pain laterally. In hemodynamically stable patients with clinical features of acute colonic ischemia, a definitive diagnosis requires advanced abdominal imaging. The initial test of choice is computed tomographic (CT) angiography of the abdomen. The CT scan should be obtained without oral contrast, as the presence of contrast within the bowel can obscure the mesenteric vessels and bowel wall enhancement. Colonoscopy or sigmoidoscopy with biopsy is then performed to confirm the diagnosis. This should be performed within 48 hours of symptom onset, as delayed evaluation will decrease the biopsy yield. Go to the next page if you knew the correct answer, or click the link image(s) below to further research the concepts in this question (if desired).

Research Concepts: Bowel Ischemia

We update eBooks quarterly and Apps daily based on user feedback. Please tap flag to report any questions that need improvement.

Question 988: A 48-year-old woman is referred to the hospital from her primary care provider’s office for the management of high blood pressure, which is unresponsive to therapy. She has a history of neurofibromatosis type 1, though without any neurological deficits. She has multiple café-au-lait spots on her body. She is found to have a blood pressure of 168/128 mmHg. Her 24-hour urine metanephrines and vanillyl mandelic acid (VMA) come back elevated. Her abdominal CT shows a 2cm adrenal mass. It is decided that the patient needs surgery; however, she needs to have her blood pressure under control. What is the best medication for use prior to surgery?

Choices: 1. Clonidine 2. Propranolol 3. Captopril 4. Phenoxybenzamine

Answer: 4 - Phenoxybenzamine Explanations: This patient likely has a pheochromocytoma. The drug of choice in this patient is phenoxybenzamine, which is a nonspecific, irreversible alpha antagonist. It is used before and during surgery to lower blood pressure in patients with pheochromocytoma. It is important to treat with an alpha-blocker prior to a beta-blocker as beta-blocker therapy alone will lead to unopposed alpha stimulation, resulting in a hypertensive crisis similar to that seen with cocaine intoxication. It is also used off-label to treat thyrotoxicosis. Go to the next page if you knew the correct answer, or click the link image(s) below to further research the concepts in this question (if desired).

Research Concepts: Chromaffin Cell Cancer

We update eBooks quarterly and Apps daily based on user feedback. Please tap flag to report any questions that need improvement.

Question 989: A 30-year-old female is brought to the emergency in an altered mental state. She had a seizure en route to the hospital. Her sister states she found her unconscious with an empty bottle of unknown medicine. Vital signs are pulse rate 108/min, blood pressure 96/54 mm Hg, respiratory rate 23/min, and temperature 101 F (38.3°C). Physical examination reveals hot and dry skin. ECG shows sinus tachycardia with QT prolongation. The patient starts seizing again. What is the next step in the management of this patient?

Choices: 1. Phenytoin 2. Haloperidol 3. Etomidate 4. Benzodiazepines

Answer: 4 - Benzodiazepines Explanations: Tricyclic antidepressants impose their therapeutic effects by inhibiting presynaptic reuptake of norepinephrine and serotonin in the central nervous system (CNS). This effect in the CNS can cause seizures. Seizures secondary to tricyclic antidepressant toxicity are usually self-limited. Seizures usually respond to benzodiazepines, but in cases of refractory seizures, prompt administration of anticonvulsants, such as phenobarbital or propofol, or even general anesthesia, should be considered. The seizure activity is secondary to blockage of sodium channels. As with all cases of poisoning, proper management of airway, breathing, and circulation is critical in cases of TCA poisoning. Go to the next page if you knew the correct answer, or click the link image(s) below to further research the concepts in this question (if desired).

Research Concepts: Tricyclic Antidepressant Toxicity

We update eBooks quarterly and Apps daily based on user feedback. Please tap flag to report any questions that need improvement.

Question 990: A 70-year-old woman with a past medical history of diabetes mellitus, poorly controlled hypertension, and alcohol use disorder presents to the emergency department with 2 days of worsening abdominal pain, intractable nausea, and vomiting for 24 hours progressing to coffee ground vomitus. The abdominal pain is described as sharp, epigastric, and non-radiating. It is worsened by oral intake and does not appear to have any alleviating factors. On examination, the patient has epigastric tenderness and a normal digital rectal exam. Vital signs show a heart rate of 110/min and blood pressure of 90/65 mmHg. An initial set of investigations is shown below. Patient value Reference range Hemoglobin 8 g/dL 13.2-17.5 g/dL 4100WBC count 16000/microL 10900/microL Lactic acid 25.2 mg/dL 4.5-19.8 mg/dL Serum creatinine 2.21 mg/dL 0.6-1.2 mg/dL ALT 121 IU/L 10-35 IU/L AST 175 IU/L 35 IU/L ALP 64 IU/L 44-147 IU/L Amylase 30 IU/L 30-125 IU/L Total bilirubin 1.2 mg/dL 0.3-1.0 mg/dL A CT of the abdomen and pelvis without contrast is unremarkable. Which of the following is the next best step in the management of this patient?

Choices: 1. Ultrasound of upper abdomen 2. Magnetic resonance cholangiopancreatography 3. Upper gastroesophageal endoscopy 4. CT angiogram

Answer: 3 - Upper gastroesophageal endoscopy Explanations: Upper gastroesophageal endoscopy provides direct visualization and is the only test that can diagnose acute esophageal necrosis. Commonly reported co-morbidities associated with acute esophageal necrosis include diabetes (38%), hypertension (37%), and alcohol use disorder (25%). Upper gastrointestinal bleeding, vomiting, epigastric pain, and tenderness with hemodynamic compromise are common presenting features for acute esophageal necrosis. The main clinical findings, in this case, are signs of septic shock in the form of hypotension and leukocytosis. Liver function tests do not show an obstructive picture, and amylase is normal; hence ultrasound and MRCP are unlikely to provide the diagnosis. CT angiogram is reserved for lower gastrointestinal bleeding and is therefore not indicated. Go to the next page if you knew the correct answer, or click the link image(s) below to further research the concepts in this question (if desired).

Research Concepts: Esophageal Necrosis

We update eBooks quarterly and Apps daily based on user feedback. Please tap flag to report any questions that need improvement.

Question 991: A 66-year-old man with a history of type 2 diabetes mellitus, hypertension, and diabetic neuropathy is brought to the hospital with fever, chills, cough, shortness of breath nausea and vomiting. He currently takes lisinopril, insulin glargine 15 units twice a day, and gabapentin 600 mg three times a day. Vital signs show temperature 101 F, heart rate 110/min, blood pressure 80/60 mmHg, and respiratory rate 24/min. Physical exam shows the patient being extremely lethargic, tachycardic, and has left lower lobe crackles on chest auscultation. Chest xray shows left lower lobe consolidation. CT head shows no acute findings. The patient is diagnosed with pneumonia, with septic shock. Labs show WBC count 22,000/microL, creatinine 4.8 mg/dL, creatinine clearance less than 15 mL/min, potassium 3.8 mEq/L. An arterial blood gas is within normal limits. He is started on IV fluids, antibiotics and epinephrine, and blood cultures are drawn. Which of the following is the next best step in the management of this patient?

Choices: 1. Discontinue gabapentin 2. Hemodialysis 3. IV sodium bicarbonate 4. IV potassium chloride

Answer: 1 - Discontinue gabapentin Explanations: Gabapentin is primarily excreted through the renal path, with no active metabolites. It needs to be stopped when the patient is extremely lethargic with severe worsening of creatinine clearance. Dosage adjustment is necessary for patients with renal impairment. Hemodialysis needs to be considered in usual indications for renal failure, not if the patient is able to make urine as the patient can clear gabapentin slowly even with creatine clearance around 15 mL/min. Go to the next page if you knew the correct answer, or click the link image(s) below to further research the concepts in this question (if desired).

Research Concepts: Gabapentin

We update eBooks quarterly and Apps daily based on user feedback. Please tap flag to report any questions that need improvement.

Question 992: A 24-year-old man presents to the hospital with vomiting, dyspnea, palpitations, and hematochezia. His friend said that he ate some seeds while they were out hiking. Physical examination is significant for inspiratory crackles bilaterally. His blood pressure is measured to be 84/49 mmHg and his heart rate is 110/min. Despite aggressive management, the patient passes away. Which of the following cellular processes is most likely affected by the toxin responsible for the patient's demise?

Choices: 1. Transcription 2. Translation 3. Oxidative phosphorylation 4. Glycolysis

Answer: 2 - Translation Explanations: Pulmonary edema, gastrointestinal bleeding, vomiting, and diarrhea are signs of ricin toxicity. Ricin inhibits ribosomes which would prevent the translation of proteins. Ricin has no antidote, supportive care must be given. Ricin is found in castor beans. Chewing these beans will release ricin. Go to the next page if you knew the correct answer, or click the link image(s) below to further research the concepts in this question (if desired).

Research Concepts: Castor Oil

We update eBooks quarterly and Apps daily based on user feedback. Please tap flag to report any questions that need improvement.

Question 993: A 17-year-old male presents to the emergency department following a syncopal episode in which he lost consciousness during football practice. He denies any injury. He does not smoke or take alcohol or any illicit drugs. Family history reveals the sudden death of his father at the age of 40 due to cardiac failure. Temperature is 37 C (98.6 F), the pulse is 84/min, blood pressure is 130/80 mm Hg, and respirations are 16/min. Physical examination reveals a bifid carotid pulse with a brisk upstroke. Chest auscultation reveals a fourth heart sound and systolic ejection murmur without carotid radiation that decreases in intensity while squatting and increases with Valsalva. Which of the following is the best management of this patient?

Choices: 1. ECG should be done and if normal, sports activities can be resumed 2. Ambulatory Holter monitoring should be done 3. An ECG and an echocardiogram should be done and the patient should refrain from sports pending the results 4. A head CT scan should be done

Answer: 3 - An ECG and an echocardiogram should be done and the patient should refrain from sports pending the results

Explanations: This patient likely has hypertrophic cardiomyopathy (HCM). HCM occurs due to the mutations of genes encoding cardiac contractile proteins. It leads to left ventricular hypertrophy and subsequently left ventricle outflow tract (LVOT) obstruction and diastolic dysfunction. In HCM, syncope is caused by inadequate cardiac output with exertion. This is due to the hypertrophy of the left ventricle that causes obstruction of outflow (LVOT). The murmur of hypertrophic obstructive cardiomyopathy increases with Valsalva and standing and decreases with squatting and handgrip. HCM can lead to acute LVOT obstruction, arrhythmia, and myocardial ischemia, especially during exertion. All these can cause sudden cardiac death as well as heart failure. Therefore, HCM needs to be ruled out with ECG and echocardiogram before the patient may resume physical exercise due to the risk of sudden cardiac death. ECG alone cannot rule out HCM. The patient should refrain from participating in sports until the test results are available. Go to the next page if you knew the correct answer, or click the link image(s) below to further research the concepts in this question (if desired).

Research Concepts: Hypertrophic Obstructive Cardiomyopathy

We update eBooks quarterly and Apps daily based on user feedback. Please tap flag to report any questions that need improvement.

Question 994: A 64-year-old patient has been admitted to the intensive care unit after major abdominal surgery and develops signs of shock and chest discomfort. An electrocardiogram shows that the QRS duration is 140 ms with a heart rate of 130 beats per minute. Which of the following is the most appropriate initial pharmacological treatment?

Choices: 1. Digoxin 2. Adenosine 3. Diltiazem 4. Lidocaine

Answer: 4 - Lidocaine Explanations: Lidocaine is a class IB antiarrhythmic and can be used to treat ventricular tachydysrhythmias. In normal cardiac myocytes, lidocaine shortens the action potential without affecting conduction velocity, but in ischemia, it prolongs the action potential, and the conduction velocity is reduced. Lidocaine is not indicated for supraventricular arrhythmias. Lidocaine may have a particular role in ventricular tachycardias that are refractory to other treatments. Go to the next page if you knew the correct answer, or click the link image(s) below to further research the concepts in this question (if desired).

Research Concepts: Lidocaine

We update eBooks quarterly and Apps daily based on user feedback. Please tap flag to report any questions that need improvement.

Question 995: A 68-year-old man with a mechanical mitral valve and a history of atrial fibrillation and hypertension presents to the emergency department for sudden onset slurred speech. His current medications include warfarin and amlodipine. According to the patient, he had a mild fever with chills over the last couple of days. On initial evaluation, his National Institutes of Health Stroke Scale (NIHSS) score is 1. His blood pressure is 118/89 mmHg, temperature 38.6 C, pulse rate 85 bpm, and respiratory rate 12 bpm. Physical examination shows mild muscle weakness in the left arm but is otherwise unremarkable. Transoesophageal cardiac echocardiography reveals a 4x4 mm mass on the atrial side of the mitral valve. Staphylococcus aureus is found in three sets of blood cultures. What is the next step for a definitive diagnosis?

Choices: 1. CT brain with contrast 2. MRA 3. Diffusion-weighted MRI 4. Carotid doppler sonography

Answer: 3 - Diffusion-weighted MRI Explanations: The patient is suffering from acute ischemic stroke with infective endocarditis. Staphylococcus aureus is the most common causative pathogen for embolism in IE. Urgent brain imaging is mandatory in all patients with sudden neurologic deficits or acute stroke. Noncontrast brain CT, rather than a contrast CT, is the image of choice to exclude or confirm hemorrhage. Recent studies show MRI-DWI (MRI with diffusion-weighted imaging) can be used as the imaging modality of choice for suspected acute ischemic or hemorrhagic stroke in those patients without MRI contraindications. However, a CT scan is more widely available than MRI in institutes that are not stroke centers. Noncontrast brain CT, rather than a contrast CT, is the image of choice to exclude or confirm hemorrhage. MR angiography (MRA) identify vascular stenosis or occlusion. Carotid duplex ultrasound is a noninvasive examination to evaluate extracranial atherosclerotic disease. Go to the next page if you knew the correct answer, or click the link image(s) below to further research the concepts in this question (if desired).

Research Concepts: Neurological Sequelae Of Endocarditis

We update eBooks quarterly and Apps daily based on user feedback. Please tap flag to report any questions that need improvement.

Question 996: A 76-year-old male with a past medical history of coronary artery disease, chronic obstructive pulmonary disease (COPD), hypertension, and type 2 diabetes mellitus was admitted to the ICU for ventilatory support due to COPD exacerbation. He was intubated and mechanically ventilated for hypoxemic hypercapnic respiratory failure the day prior. The patient is sufficiently sedated with propofol and fentanyl drips, but it is noticed that he developed an acute rise in his peak pressures on the ventilator to 30 mm Hg from 19 mm Hg. His plateau pressure has remained unchanged at 22 mm Hg. Which one of the following is the potential cause for the increase in his peak pressure?

Choices: 1. Pneumothorax 2. Pleural effusion 3. Mucus plug 4. Acute respiratory distress syndrome

Answer: 3 - Mucus plug Explanations: The peak pressure represents the resistance of the respiratory system from the ventilator tubing down to the segmental bronchi. Any factor that affects the resistance of the tubes will cause a change to the peak pressure. Examples including mucus plugging, bronchospasm, patient biting down on the endotracheal tube, and blood clots. A pleural effusion will cause plateau pressures to rise. Plateau pressures are representative of the compliance of the respiratory system (chest wall, lung, abdomen). A decrease in the compliance of the system will cause plateau pressures to rise. A pneumothorax will cause plateau pressures to rise. Plateau pressures are representative of the compliance of the respiratory system (chest wall, lung, abdomen). A decrease in the compliance of the system will cause plateau pressures to rise. Acute respiratory distress syndrome will cause plateau pressures to rise. Plateau pressures are representative of the compliance of the respiratory system (chest wall, lung, abdomen). A decrease in the compliance of the system will cause plateau pressures to rise. Go to the next page if you knew the correct answer, or click the link image(s) below to further research the concepts in this question (if desired).

Research Concepts: Positive Pressure Ventilation

We update eBooks quarterly and Apps daily based on user feedback. Please tap flag to report any questions that need improvement.

Question 997: A patient initially notices double vision after eating supper. Shortly thereafter, she starts to experience weakness in arms and legs, difficulty with swallowing, extreme trouble focusing her vision, and some minor breathing difficulties. 911 is called and she is brought to the ER. The clinician notices bilateral drooping of the eyelids and dilated fixed pupils. Her respiratory status has become more fragile. Which of the following is the antibiotic of choice?

Choices: 1. Chloramphenicol 2. Clindamycin 3. Penicillin 4. Gentamicin

Answer: 3 - Penicillin Explanations: High-dose intravenous penicillin therapy is the antibiotic of choice for wound botulism. Chloramphenicol and clindamycin are alternate choices. Botulinum inhibits acetylcholine release, thus blocks motor neuron to muscle cell communication. Typically, muscle weakness and paralysis begins in the face and spreads peripherally to the upper and lower extremities. Diaphragmatic paralysis is life threatening. Diagnosis is made by establishing the presence of botulinum toxin or bacteria in the stomach, intestinal contents, vomit, feces or blood. Tests used include enzyme-linked immunosorbent assays (ELISAs), electrochemiluminescent (ECL) tests, and mouse bioassay. Go to the next page if you knew the correct answer, or click the link image(s) below to further research the concepts in this question (if desired).

Research Concepts: Botulism

We update eBooks quarterly and Apps daily based on user feedback. Please tap flag to report any questions that need improvement.

Question 998: After an injury, when do the symptoms of acute respiratory distress syndrome usually begin?

Choices: 1. 1 hour 2. Within 24 hours 3. Within 5 days 4. 1 week

Answer: 2 - Within 24 hours Explanations: The symptoms of acute respiratory distress syndrome are very rapid. Most people have some clinical and x-ray features within 24 hours. In fact, the majority of patients will have hypoxia within 6 hours and x-ray features within 24 to 48 hours. Initial symptoms are non-specific and include tachypnea, tachycardia, hypotension, and altered mentation. Additional symptoms will be present depending on the precipitating cause of the lung injury. Go to the next page if you knew the correct answer, or click the link image(s) below to further research the concepts in this question (if desired).

Research Concepts: Acute Respiratory Distress Syndrome

We update eBooks quarterly and Apps daily based on user feedback. Please tap flag to report any questions that need improvement.

Question 999: A 42-year-old man was diagnosed with acromegaly and recently underwent transsphenoidal resection of a pituitary adenoma. What is the next most accurate step to assess if the surgery was successful?

Choices: 1. MRI pituitary immediately post op 2. IGF-1 level immediately post-surgery 3. GH level immediately post-surgery 4. GH level after OGTT 4 months after surgery

Answer: 4 - GH level after OGTT 4 months after surgery Explanations: To assess for remission for acromegaly after surgery, IGF-1 level should be done. The timing is important. Immediate post-surgery IGF-1 levels are elevated due to their long halflife and can take at least 3 months to normalize. In addition to checking IGF-1 levels, one can check a GH level after 75 g of glucose being administered. Some argue that checking GH levels after a normal IGF-1 level may not add additional information; however, up to 30% of patients can have discordant results. A normal IGF-1 level and GH suppressed to less than 1 ng/mL after OGTT is indicative of remission. These levels should be checked at least annually as relapse can occur many years after remission. Even though repeat imaging should be done after surgery, it should be done a minimum of 3 months after the surgery as it can take that long for the sterile compressed sponge and fat packing to be resorbed and not affect image interpretation assessing for disease recurrence. Go to the next page if you knew the correct answer, or click the link image(s) below to further research the concepts in this question (if desired).

Research Concepts: Acromegaly

We update eBooks quarterly and Apps daily based on user feedback. Please tap flag to report any questions that need improvement.

Question 1000: A 65-year-old homeless woman is brought to the emergency department with complaints of abdominal pain and decreased urination. She is afebrile, has blood pressure 180/120vmmHg, pulse rate 100/min, and respiratory rate 20/min. Physical examination is significant for IV drug use tracings in her upper extremities and nonhealing ulcers in lower extremities just above the medial malleolus. Her initial labs showed potassium 6.2 mEq/L, bicarbonate 14 mEq/L, BUN/Cr 40/5.6 mg/dL, and urinalysis is significant for WBC, RBC, and RBC casts. Further labs show a high HCV viral load, AST/ALT 80/100 IU/L, negative ANA, normal C3, low C4, and a low titer of cryoglobulins. Which of the following is the best initial therapy for this patient?

Choices: 1. Ribavirin and PEG- interferon-alpha 2. IV immunoglobulins 3. Corticosteroids and sofosbuvir 4. Plasmapheresis

Answer: 4 - Plasmapheresis Explanations: Plasmapheresis is used in life-threatening situations of mixed cryoglobulinemia. Plasmapheresis aids in the fast removal of circulating immune complexes thereby removing the major insult to the organs. Renal failure is involved with increased mortality in patients with mixed cryoglobulinemia and therefore a fast-acting treatment is required. Ribavirin and interferon-alpha used to be the treatment strategy for hepatitis C prior to the era of direct-acting antivirals (like sofosbuvir). They were helpful in mild cases of mixed cryoglobulinemia previously as they helped in antigen clearance (viral clearance). Corticosteroids are used in mild cases of mixed cryoglobulinemia but are not of much use in serious complications such as renal failure. IV immunoglobulins are contraindicated in mixed cryoglobulinemia as they cause precipitation of immune complexes leading to multiorgan failure (PMID- 27870771). Antiviral agents take several doses to show effect and hence not useful in our situation. Antiviral therapy needs to be started in this patient after plasmapheresis when she is in a more stable situation. Go to the next page if you knew the correct answer, or click the link image(s) below to further research the concepts in this question (if desired).

Research Concepts: Cryoglobulinemic Vasculitis

We update eBooks quarterly and Apps daily based on user feedback. Please tap flag to report any questions that need improvement.